Está en la página 1de 753

1.

Una mujer de 71 años se presenta a su médico de atención primaria quejándose de


sangrado rectal. Tenía algunos calambres abdominales leves en el lado izquierdo que
disminuyeron en pocos minutos. Ella nunca ha tenido un episodio previo de sangrado
rectal. El examen físico revela dolor abdominal leve en el cuadrante inferior izquierdo sin
evidencia de defensa o rebote. El examen rectal revela que no hay sangre fresca en la
ampolla rectal. La colonoscopia revela varias colecciones fuera de la pared del colon
sigmoideo sin evidencia de sangrado o perforación. El resto de la colonoscopia está
dentro de los límites normales. El recuento de glóbulos blancos es normal. ¿Cuál es el
tratamiento más apropiado

resp E

2. Un hombre de 19 años fue pateado en el abdomen durante una pelea en un bar. Acudió
a su médico de atención primaria, quien ordenó una tomografía computarizada, que
reveló un hematoma esplénico subcapsular. Se le dijo al hombre que restringiera la
actividad física. Dos semanas después, se presenta en el departamento de emergencias
debido a un fuerte dolor abdominal. Se somete a una esplenectomía. Después de la
operación, se ordena un frotis periférico. ¿Qué tipo de célula se puede encontrar en este
paciente?
resp C
3. Una mujer afroamericana de 42 años se somete a una colecistectomía laparoscópica por
dolor crónico en el cuadrante superior derecho. La tomografía computarizada demostró
cálculos biliares y líquido pericolecístico. El procedimiento quirúrgico no fue complicado.
El análisis patológico de los cálculos biliares reveló cálculos de bilirrubinato de calcio.
¿Cuál es la explicación más probable de estos hallazgos?
respD

4. Un hombre de 22 años sufre una herida por arma de fuego en el flanco izquierdo, el
paciente está hipotenso pero tiene pulso. Se inician bolus con 2L de líquido. El paso a
seguir es:
resp B. Tomografia abdominal
5. Un niño de 3 meses presenta con vómitos persistentes asociados a estenosis pilórica .
El cambio metabólico más probable es :
resp c

6. Mujer de 62 años con estreñimiento de 6 meses de evolución que ha empeorado en el


último mes. La paciente comenta “que sus evacuaciones son tan delgadas como un
lápiz”. En el colon por enema hay una imagen de mordida de manzana en la unión de
recto sigmoides. ¿Cuál es el diagnóstico más probable?
A. enfermedad de crohn
B. o carcinoma’ no se
7. Una mujer de 53 años se presenta a su médico de atención primaria con un historial de
dolor de cuello de 12 meses. Ella se queja de un aumento de peso de 15 libras y
malestar generalizado. Tiene antecedentes médicos de hipertensión y diabetes mellitus.
Sus medicamentos actuales incluyen un hipoglucemiante oral. El examen físico revela
sensibilidad a lo largo del curso de la glándula tiroides sin evidencia de una masa
discreta. ¿Cuál es el diagnóstico más probable?
B. Tiroiditis de Hashimoto

8. Un hombre sufre una discolación posterior de su rodilla con pérdida de pulso. Se logra
reducir la dislocación y ahora el pulso es palpable. Cuál es el próximo paso más
apropiado a seguir en este paciente :
resp A
9. Una mujer de 19 años comenzó a amamantar por primera vez. Al principio, era difícil
para su bebé alimentarse. Ahora, sus senos están rojos, cálidos y doloridos. Ella ha
seguido amamantando, a pesar del dolor; sin embargo, recientemente comenzó a usar
un extractor de leche en lugar de amamantar. Ella inicia un curso de antibióticos orales.
¿Qué afección tiene esta paciente riesgo de desarrollar? A.

10. Una mujer de 56 años asiste al cuarto de urgencias con dolor abdominal severo. No
tiene otros antecedentes médicos. Toma licor ocasionalmente. Los laboratorios muestran
amilasa en 24.000 y lipasa en 7.000. Radiografía de abdomen no muestra patología. Su
WBC: 17.000. Su PA es 80/40 y su FC es 120. Afebril. Al examen tiene dolor en el
epigastrio. El paso más apropiado a seguir es : A
11. El factor más importante en la cicatrización de las heridas por intención primaria es : B
creo

12. Un hombre obeso de 49 años se presenta a su médico de atención primaria para un


examen de seguimiento. Tiene antecedentes de diabetes mellitus no controlada y
trastorno bipolar. Sus medicamentos actuales incluyen litio y leche de magnesio. El
examen físico del corazón, los pulmones y el abdomen están dentro de los límites
normales. Los estudios de laboratorio revelan calcio sérico de 14 mg / dL. ¿Cuál es la
explicación más probable para estos hallazgos?E
13. Permite establecer el pronóstico en la Pancreatitis Aguda, Excepto: C

14. Una mujer de 47 años con antecedentes de masa tiroidea izquierda se somete a una
lobectomía tiroidea izquierda. La patología revela un carcinoma papilar de 1.3 cm sin
evidencia de extensión extracapsular. ¿Cuál es el siguiente paso más apropiado en el
tratamiento de este paciente? A
15. Una mujer de 62 años se presenta con tos a su médico de atención primaria. Ella
también se queja de hemoptisis. La historia social revela que es fumadora de 55
paquetes al año. Ella es una alcohólica en recuperación. El examen físico revela
sibilancias bilaterales. Los exámenes cardíacos, pulmonares y abdominales no son
notables. Los valores de laboratorio revelan calcio sérico de 13 mg / dL. La electroforesis
de proteínas séricas no muestra picos anormales. ¿Cuál es el diagnóstico más
probable? A
limites

16. Una mujer de 39 años se queja de dolor epigástrico al comer durante los últimos 3 o 4
meses. Ella admite una historia de problemas crónicos de espalda. Ella nota un aumento
de peso de 20 lb en los últimos 4 meses. Ella niega el uso de agentes antiinflamatorios
no esteroideos. Ella niega las náuseas y los vómitos. El examen físico del corazón, los
pulmones y el abdomen están dentro de los límites normales. ¿Cuál es el patógeno más
probable asociado con esta afección? Helicobacter pylori

17. Varón de 55 años acude para revisión anual de rutina y en la colonoscopia se encuentra
un pólipo pediculado de 3 cm, lo que se realiza polipectomia con asa. La patología
reporta pólipo adenomatoso con carcinoma invasor en la punta, con margen de
resección 4 mm y sin células tumorales en la base del pólipo. ¿Cuál es el paso más
apropiado en el tratamiento
Respuesta C

18. Paciente de 60 años asintomático con historia familiar de cáncer de colon. El


procedimiento de tamizaje recomendado es:
Respuesta C
19. Si un paciente tiene una presión de 50 mmHg en la arteria tibial posterior derecha y 140
en la arteria braquial, dolor de reposo y una úlcera de 4 meses que no cicatriza, el
diagnóstico más probable es: C

20. El reflujo gastroesofágico que se produce en un lactante con una EPCI es


fundamentalmente a causa de
D
21. ¿Cuál de las siguientes técnicas se utiliza mejor para definir una glándula paratiroidea
agrandada?
E

22. Cuáles son las células más importantes que participan en la cicatrización de las heridas :
Cuales son las células más importantes que participan en la cicatrización de las heridas:

a. Polimorfonucleares

b. Macrofagos

c. Plaquetas

d. Linfocitos
23. Un hombre de 55 años se presenta a su médico quejándose de poliuria, polidipsia,
polifagia y un sarpullido rojo y escamoso en la cara en las últimas 2 semanas. Orina 18
veces / día con una buena fuerza de flujo. La glucemia en ayunas fue de 325 mg / dL.
Ha perdido 20 libras en los últimos 2 meses y nunca ha tenido niveles elevados de
glucosa en sangre en el pasado. El examen físico del corazón, los pulmones y el
abdomen son normales. ¿Cuál es la explicación más probable para estos hallazgos? C

24. Un hombre de 78 años es llevado al departamento de emergencias con un historial de


12 horas de dolor abdominal, diarrea y vómitos. Tiene antecedentes de fibrilación
auricular y fue tratado previamente por insuficiencia cardíaca congestiva con digoxina. El
examen físico revela un abdomen distendido con defensa significativa. El examen rectal
revela heces guayaco positivas. El recuento de glóbulos blancos es de 24,000 / mL. La
radiografía abdominal revela edema de la pared intestinal. ¿Cuál es el tratamiento más
apropiado para este paciente?
E
25. Un hombre de 57 años es llevado al departamento de emergencias quejándose de
disnea y dolor en el pecho. También admite una pérdida de peso de 20 libras. Se queja
de fiebres, escalofríos y sudores nocturnos. El examen físico revela adenopatía
supraclavicular. El examen de tórax revela ruidos cardíacos distantes. Los estudios de
laboratorio revelan un recuento de glóbulos blancos de 170,000 / mL. La radiografía de
tórax y la ecocardiografía revelan un derrame pericárdico. ¿Cuál es la explicación más
probable de estos hallazgos?
C

26. Para pacientes pediátricos con apendicitis cuál de los siguientes enunciados no es
cierto:
R: D. La tomografia
27. En un hemotórax masivo, cuál de las siguientes es una indicación para toracotomía de
emergencia?
R: D. Produccion 1500 cc

28. La capa más fuerte de la pared intestinal es :


R: B. Submucosa
29. Un estudiante universitario masculino de 21 años se presenta en la clínica ambulatoria
para un examen de rutina al comienzo del semestre de otoño. Tiene antecedentes de
síndrome del intestino irritable. El examen físico del corazón, los pulmones y el abdomen
no tienen alteraciones. El examen genitourinario revela que los testículos descienden
bilateralmente. Un varicocele izquierdo de grado 1 está presente. No hay masas
testiculares. El pene no está circuncidado y el prepucio no puede retraerse detrás del
glande. ¿Cuál es el diagnóstico más probable?
R: E. Fimosis

30. Un motociclista de 29 años de edad, que no utilizaba casco sufre un accidente vehicular
que le ocasiono un traumatismo craneoencefálico cerrado grave. Fue intubado en el sitio
del accidente y se le transportó a un centro de trauma. Usted es el médico de atención
en el cuarto de urgencias lo primero que tiene que verificar es:
R: A. Via aerea
31. El estudio indicado en paciente con papiloma intraductal debe ser:
R: C. Ductografia

32. Una mujer de 23 años que se queja de heces grasas y fétidas(esteatorrea), debilidad
generalizada y pérdida de cabello, se presenta a su médico de atención primaria para su
evaluación. El examen físico del corazón, los pulmones y el abdomen son irrelevantes.
Ella no tiene defensa, terneza, dolorimiento o rebote. Los sonidos intestinales están
presentes en todos los cuadrantes. El examen pélvico femenino fue diferido a pedido de
la paciente. ¿Cuál es la explicación más probable de estos hallazgos?
R: D. Insuficiencia
33. En el choque hemorrágico inicialmente el compartimiento que trata de mantener el
volumen es:
R: C. Intersticial

34. Ante un caso de pancreatitis aguda (PA), ¿En qué situación considera que puede estar
indicada la CPRE?:
D. todas
automo

35. Un hombre de 59 años resultó herido en un accidente automovilístico. Una tomografía


computarizada abdominal revela una ruptura del bazo. Su presión arterial es de 90/40
mm Hg, y su pulso es de 140 latidos por minuto. Se toma al paciente para laparotomía.
Se realiza esplenectomía. ¿Cuál de las siguientes anormalidades de laboratorio es
probable después de este procedimiento?
a. anemia

35. Una estudiante de la facultad de medicina de 21 años conduce bajo la influencia del
alcohol, a pesar de las recomendaciones de amigos de no conducir. Ella es golpeada por otro
conductor. La fuerza del impacto hace que golpee el área temporal de su cráneo contra la
ventana. Ella desarrolla un dolor de cabeza leve pero no pierde el conocimiento. Varias horas
después, desarrolla un fuerte dolor de cabeza con náuseas y vómitos. ¿Cuál es el diagnóstico
más probable?
c. hematoma hepidural

36. Todo lo anterior es cierto con respecto a las lesiones por mordeduras humanas excepto
D

37. Un niño de 5 años es llevado al departamento de emergencias después de ingerir un


limpiador de drenaje líquido. El niño se quedó sin vigilancia mientras su niñera estaba hablando
por teléfono. El niño está ronco y tiene un evidente estridor. ¿Cuál es el tratamiento inicial más
apropiado para este paciente?
E. Traqueostomía

38. La mucosa esofágica está compuesta por una capa de epitelio:

c. Escamoso estratificado no
queratinizado.

39. Las mordeduras humanas son:


d. Se consideran heridas contuso-punzantes

40. La principal causa etiopatológica de apendicitis en niños es


a. fecalito

41. Un enfermo de 45 años, bebedor de 60 grs. de alcohol diarios, desde hace 6 meses aqueja
disfagia para sólidos, odinofagia, sialorrea y anemia ferropénica. En la endoscopia se ha
encontrado una estenosis esofágica 9 cm. por encima del cardias y en la biopsia epitelio
columnar displásico ¿Cuál es el diagnóstico de presunción?:R/= E
42. Un granjero de 37 años de edad sufre accidente por volcamiento de un tractor quedando
ambas piernas atrapadas por 4 horas asociadas a fracturas de fémur bilaterales. Los
laboratorios luego de 24 horas demuestran Na 140, K 5.7, Cl 100, HCO3 10 meq/L. BUN 6
mg/dl, creatinina 2.2 mg/dl y CPK 15 000. El primer paso para prevenir la falla renal en este
paciente debe ser: R/= C

43. Todo lo siguiente influye favorablemente en disminuir las tasas de infecciones quirúrgicas
excepto: R/= C
44. Con relación a la fisiopatología de la isquemia arterial obstructiva crónica:
C.

45. Un hombre de 26 años se encuentra en UCI 6 horas luego de una esplenectomía luego de
un accidente automovilístico. El paciente estuvo en un traslado prolongado, recibe 20 unidades
de sangre antes de su llegada al cuarto de urgencias. Su presion en el ventilador (peak airway
pressure ) es 65, su abdomen está distendido, no está orinando, su presión de vejiga es 40. Su
PVC es 18. La maniobra más importante a realizar en este paciente es : R/= D
46. El tratamiento de pacientes con quemaduras severas y mioglobinuria consiste en:
R/= C

47. Un niño de 8 años se somete a una ecografía del cuadrante superior derecho por molestias
persistentes en el cuadrante superior derecho. No tiene antecedentes médicos o quirúrgicos
previos. No tiene alergias conocidas y no toma medicamentos. Su madre tiene antecedentes de
cálculos biliares. Los hallazgos de la ecografía incluyen una dilatación fusiforme del conducto
biliar común. ¿Cuál es la explicación más probable para estos hallazgos?
R/= A
48. Paciente A de 62 años asintomático, se encuentra en el examen físico una masa pulsátil en
el abdomen, entre el xifoides y el ombligo.
Paciente B de 62 años tiene malestar epigástrico vago y mal descrito en la parte superior de la
espalda. Se encuentra en el examen físico que tiene una masa pulsátil de 6 cm en el abdomen,
entre el xifoides y el ombligo. La masa es sensible a la palpación.
Paciente C de 68 años es llevado al servicio de urgencias con dolor de espalda insoportable que
comenzó repentinamente hace 45 minutos. Está diaforético y tiene una PA sistólica 90. Hay una
masa pulsátil de 8 cm palpable en el abdomen, arriba del ombligo. R/C xd
OA a- PacienteÁ no necesita una reparación quirúrgica electiva, debe citarse
dentro de un año para un ultrasonido y si tiene un aneurisma mayor de 6 cm la
angio tomografía computarizada está indicada. Paciente B tiene un aneurisma
aórtico abdominal que está empezando a filtrarse, debe consultarse
inmediatamente a cirugía vascular ya que es necesaria una reparación
quirúrgica. Paciente C el aneurisma se está rompiendo en este momento por lo
tanto necesita una cirugía inmediata sin mayor demora.

O 8. b- PacienteA necesita una reparación quirúrgica electiva, pero debido a que


nuestras decisiones se basan en el tamaño del aneurisma, necesitamos una
medición más precisa, por lo tanto esta indicada una angio tomografía
computarizada. Paciente B tiene un aneurisma aórtico abdominal que está
empezando a filtrarse, debe consultarse inmediatamente a cirugía vascular ya
que es necesaria una reparación quirúrgica. Paciente C el aneurisma se está
rompiendo en este momento por lo tanto necesita una cirugía inmediata sin
mayor demora.

Oc. c- PacienteA necesita una reparación quirúrgica electiva, pero debido a que
nuestras decisiones se basan en el tamaño del aneurisma, necesitamos una
medición más precisa, por lo tanto esta indicada una angio tomografía
computarizada. Paciente B tiene un aneurisma aórtico abdominal que está
empezando a filtrarse. Se debe realizar un aortograma abdominal por cirugía
vascular para determinar si es necesaria una reparación quirúrgica abierta o por
vía endovascular. Paciente C el aneurisma se está rompiendo en este momento
por lo tanto necesita una cirugía inmediata sin mayor demora.

OD. d- PacienteA necesita una reparación quirúrgica electiva, pero debido a que
nuestras decisiones se basan en el tamaño del aneurisma, necesitamos una
medición más precisa, por lo tanto esta indicada una angio tomografía
computarizada. Paciente B tiene un aneurisma aórtico abdominal que está
empezando a filtrarse, debe consultarse inmediatamente a cirugía vascular ya
que es necesaria una reparación quirúrgica. Paciente C El aneurisma se está
rompiendo en este momento. Él necesita un aortograma abdominal para decidir
. . . . . .
cienatla sa a rnalizarima crminia da amaraancia ahinrta a andaimoemilar
49. De los signos pronósticos de Ranson para valoración de la gravedad de la pancreatitis
aguda, ¿cuál se documenta en el momento del ingreso?:
A.

50. Varón de 30 años de edad acude a la sala de urgencias con dolor epigástrico intenso, súbito,
asociado a nauseas. A la exploración física se encuentra diaforético y evita moverse. PA 100/60,
FC 110 lpm FR 28. Hay rigidez abdominal, disminución de os ruidos intestinales. La radiografía
de tórax muestra neumoperitoneo. Todas las siguientes pueden producir el siguiente cuadro
excepto:
A.
51. Masculino de 48 años quien toma cerveza una caja al día, desarrolla nauseas y vómitos.
Luego de esto, el paciente presenta dolor torácico severo y es traido al cuarto de urgencias. El
tiempo transcurrido desde su episodio de vómito hasta que llega al cuarto de urgencia son 5
horas. Su PA es 100/50 y su FC es 110, se le dan dos bolos de líquido intravenoso. Se obtiene
una radiografía de tórax y muestra una efusión izquierda y un neumotórax. Cuál es el estudio
más sensitivo para diagnosticar la condición de este paciente:
D.

52. Los problemas pulmonares de la respuesta inflamatoria es mediada por:


B,
53. Paciente A de 25 años recibe un disparo con un revólver calibre .22. La herida de entrada se
encuentra en la cara anterolateral de su muslo, y la bala se ve por rayos X incrustándose en los
músculos, posterolateral al fémur.
Paciente B de 25 años recibe un disparo con un revólver calibre .22. La herida de entrada está
en el aspecto anteromedial de la parte superior del muslo, y la herida de salida está en la cara
posterolateral del muslo. Tiene pulsos normales en la pierna y ningún hematoma en el sitio de
entrada. Los rayos X muestran que el fémur está intacto.
Paciente C de 25 años recibe un disparo con un revólver calibre .22. La herida de entrada está
en el aspecto anteromedial de la parte superior del muslo, y la herida de salida está en la cara
posterolateral del muslo. Él tiene un gran hematoma en expansión en la parte superior e interna
del muslo. El hueso está intacto.
Acerca de los 3 pacientes anteriores, cuál es la mejor respuesta?
D.
54. Un hombre de 78 años se presenta en el departamento de emergencia para evaluar un dolor
progresivo del cuadrante superior derecho, náuseas, vómitos y una pérdida de peso de 30 libras
en los últimos 3 meses. Tiene antecedentes médicos previos de colelitiasis, diabetes mellitus,
hipertensión y demencia. El examen físico revela ictericia escleral bilateralmente. El examen
abdominal revela sensibilidad en el cuadrante superior derecho y una masa palpable. Los signos
peritoneales están ausentes. La tomografía computarizada revela ganglios linfáticos
pancreáticos, duodenales y coledocales. Hay un engrosamiento asimétrico de la vesícula biliar.
¿Cuál es el hallazgo patológico más probable en la laparotomía exploratoria y la biopsia?
A.
55. Una mujer de 25 años encontró un bulto en el seno derecho en el autoexamen. Ella no tiene
antecedentes familiares de cáncer de mama. El bulto es libremente móvil y está bien
circunscrito. ¿Cuál es la mejor opción para evaluar una masa mamaria en una mujer joven?
D. Ultrasonido
hipoglucemi

56. Un hombre de 69 años se presenta a su dermatólogo con una lesión en la nariz. Es un


jardinero que pasa gran parte de su tiempo al aire libre. Tiene antecedentes médicos previos de
rinitis alérgica, hipertensión y diabetes mellitus. Sus medicamentos actuales incluyen un
betabloqueante y un hipoglucemiante oral. El examen físico de su nariz revela una lesión
papular brillante y elevada con pequeños vasos sanguíneos. ¿Cuál es el diagnóstico más
probable?
A.

57. Masculino de 18 años viene a su oficina 3 meses luego de un accidente automovilístico con
dolor en tórax izquierdo. Se obtiene una Rx de tórax y se ven niveles hidroaéreos en el pecho. El
paso a seguir es : A

58. ¿Qué fármaco se ha mostrado eficaz en pacientes con pancreatitis aguda?:


59. El gen supresor de la tumorgénesis en el cáncer de colon es:
disparo

60. Un joven recibe un disparo en el brazo con un revólver calibre .38. La trayectoria de la bala
atraviesa la extremidad, desde el lado medial al lateral. Tiene un gran hematoma en el aspecto
interno del brazo, sin pulsos distales, parálisis del nervio radial y húmero destrozado. La
secuencia quirúrgica para la reparación de esta grave lesión sería:
D. la que termina en fasciotomia
61. Una mujer de 31 años se queja de una historia de 6 meses de diarrea con sangre, dolor
abdominal y fiebre intermitente. Tiene antecedentes de síndrome del intestino irritable, pero ha
empeorado sus síntomas durante el período de tiempo anterior. Su historial médico pasado es
irrelevante. El examen físico revela distensión abdominal. Los sonidos intestinales están
presentes en todos los cuadrantes. El examen rectal revela múltiples fisuras anales. ¿Cuál es la
prueba de diagnóstico más adecuada para este paciente?
62. Un hombre de 47 años con múltiples problemas médicos y enfermedad del parénquima
pulmonar en etapa terminal se somete a un trasplante de pulmón. Tiene antecedentes médicos
previos de enfermedad pulmonar obstructiva. Tiene un tío con fibrosis quística. Su padre tiene
enfermedad pulmonar restrictiva y su hermano tiene hipertensión pulmonar. ¿Cuál de las
siguientes enfermedades augura la mejor supervivencia después de un trasplante de pulmón
para este paciente?

63. Una mujer de 51 años de edad acude a consulta por padecimiento actual de dos meses de
evolución con secreción sanguinolenta a través del pezón derecho. No tiene antecedentes de
importancia. Abuela materna con cáncer de mama postmenopáusico. Examen físico de la mama
y axilar normal, solo la secreción sanguinolenta por el pezón. La mamografía bilateral normal.
¿Cuál es el diagnóstico más probable?
64. Un hombre diestro de 62 años de edad tiene episodios transitorios de debilidad en la mano
derecha, visión borrosa y dificultad para expresarse. No hay dolor de cabeza asociado, los
episodios tienen un inicio repentino, duran alrededor de 5 o 10 minutos como máximo, y se
resuelven espontáneamente, sin dejar secuelas neurológicas. La secuencia a seguir con éste
paciente será:

65. Una mujer de 25 años sufre un accidente automovilístico resultando en la laceración


hepática posterior severa que usted está tratando con control de daños. Se empaca el abdomen
y se lleva a la paciente a UCI. Recibe 20 unidades de glóbulos rojos, 10 plasma fresco
congelado, 10 unidades de plaquetas pero permanece hipotensa (PA 80/60). Su hematocrito
está estable en 30. La causa más probable de su hipotensión es :
A

66. Masculino de 25 años sufre una herida por arma de fuego en abdomen bajo. A la
exploración , el uréter está transectado arriba del reborde pélvico y hay ausencia de un
segmento de 2.5 cm. El manejo de esta lesión es :
A

67. Cuál es la principal indicación quirúrgica de la pancreatitis crónica?:


E
68. Un hombre de 41 años tiene reflujo gastroesofágico crónico. Actualmente se maneja con un
bloqueador H2. El examen físico del corazón, los pulmones y el abdomen están dentro de los
límites normales. ¿Cuál de los siguientes factores sería el menos protector del esófago en
términos de la exposición continua inducida por esta afección?
R//A

69. Acerca de la respuesta metinflamatoabólica del trauma la citoquina modula la respuesta


inflamatoria es:
R// D
70. Un hombre de 71 años con aparición repentina de dolor abdominal y de espalda intenso es
llevado al departamento de emergencias para su evaluación. Tiene antecedentes de
hipertensión. Pesa 300 libras. Tiene historia de fumar un paquete diario desde hace 45 años. El
examen físico revela una masa abdominal pulsátil. Ambas extremidades inferiores revelan
palidez con pulsos pedios disminuidos. ¿Cuál es la causa más probable de la condición de este
paciente?
R// A

71. A la vista de los conocimientos actuales, el esófago de Barrett es un fenómeno


probablemente debido a: 72????
A. Daño a la mucosa por RGE
72. Un jugador de tenis masculino de 20 años se estrella contra una cerca tratando de perseguir
una pelota que pensó que podría alcanzar durante un partido importante. Su rodilla derecha
sufre la peor parte de la lesión. El examen físico revela edema y disminución del rango de
movimiento de la rodilla en flexión y extensión. La resonancia magnética (MRI) se realiza y
revela la dislocación de la articulación. No se palpa el pulso detrás de la articulación de la rodilla.
¿Cuál es la explicación más probable para este hallazgo?
C. espasmo de la arteria poplitea

73. Con relación a la fisiopatología de la isquemia arterial obstructiva aguda :


A. Una disminucion del flujo sanguineao solamente produce el daño a las celulas
74. Sir David Cuthbertson en 1930 describe varias fases de respuesta a la agresión, cual
enunciado es el correcto ?
D. en la fase flow ocurre aumento de citoquinas, catecolaminas excrecion de N2 y DO2

75. Una mujer de 75 años de edad es llevada a la sala de urgencias por la enfermera de un asilo
por ictericia y confusión mental. La enfermera notó que disminuyó la respuesta en forma gradual
y más tarde desarrollo ictericia. Es hipertensa, diabética y antecedente de cáncer de colon
operado. Tiene temperatura de 38,5 grados, FC 110 lpm, PA 70/40. Dolor en hipocondrio
derecho. El diagnóstico más probable debe ser:
C. Colangitis
76. Masculino de 20 años sufre una fractura severa de pelvis y tiene hematuria . Se obtiene un
cistoureterograma retrógrado y se aprecia extravasación extraperitoneal de la vejiga. El
tratamiento más apropiado es :
A. Drenaje
78.Una mujer de 41 años se queja de dolores de cabeza constantes durante los últimos 6
meses. También se ha quejado de infertilidad femenina y no ha podido tener hijos, a pesar de
haber tenido relaciones sexuales sin protección con su esposo durante los últimos 15 años. El
examen físico revela déficits en los movimientos extraoculares bilateralmente. El examen de los
senos revela ginecomastia femenina bilateral. ¿Cuál de las siguientes pruebas de laboratorio
sería más útil para diagnosticar a este paciente?
E, prolactina

79 Un niño de 4 años está en la lista de espera para un trasplante de hígado. Tiene enfermedad
hepática en etapa terminal y actualmente está hospitalizado por hemorragia por varices
esofágicas. ¿Cuál es la causa más probable de insuficiencia hepática en este paciente?

A, atresia biliar
edema

80Paciente de sexo masculino de 60 años de edad que se presenta con edema de miembro
inferior izquierdo de tres semanas de evolución, sin otros síntomas asociados:.
c. Considera que uno de sus diagnósticos diferenciales

81 La pared de los divertículos falsos del tubo gastrointestinal está formada por:
mucosa y submucosa
82 Una mujer de 41 años con enfermedad de Crohn se sometió a múltiples procedimientos
quirúrgicos. Recientemente se sometió a una ileostomía, pero aún tiene evidencia de alguna
enfermedad yeyunal distal. Sus medicamentos actuales incluyen prednisona y ácido
aminosalicílico. ¿Cuáles de los siguientes efectos de la terapia prolongada con glucocorticoides
son posibles para este paciente?
C, disfunción de fibroblastos

83 Son manifestaciones de perforación de esófago torácico todas, EXCEPTO


C
84 Un agresor apuñaló a un hombre de 18 años en su abdomen varias veces durante un
altercado que implicaba la venta de drogas ilícitas. Lo llevan al departamento de emergencias
para su evaluación. Tiene cuatro heridas de arma blanca en el abdomen, tres en el cuadrante
superior derecho y una en el cuadrante inferior izquierdo. El examen físico del abdomen revela
defensa y sensibilidad de rebote. El paciente es llevado a cirugía para una laparotomía
exploratoria. Se encuentra una lesión penetrante en la vesícula biliar. ¿Cuál de las siguientes
vísceras asociadas es probable que se lesione?
D

85. ¿Cuál es el punto de referencia más útil para indicar a qué distancia se localiza una lesión
intraesofágica en la exploración endoscópica?
A
86. Una niña de 2 años es llevada al departamento de emergencias debido a varios episodios de
sangrado rectal. Una exploración de perfusión de tecnecio-99m revela una bolsa ileal de 3 cm
ubicada a 50 cm de la válvula ileocecal. ¿Cuál de los siguientes tipos de tejido ectópico contiene
esta estructura?
C

87. Una mujer de 46 años se presenta en el departamento de emergencias quejándose de dolor


en el cuadrante superior derecho y fiebre a 38.9 ° C. El examen físico revela ictericia escleral y
dolor significativo en el cuadrante superior derecho a la palpación. Los signos de irritación
peritoneal están ausentes. Los ruidos intestinales están presentes. ¿Cuál de los siguientes debe
incluirse en el tratamiento inicial de este paciente?
A
88. Una mujer premenopáusica de 31 años con una masa mamaria izquierda se somete a una
mastectomía radical izquierda modificada. La patología revela carcinoma ductal infiltrante que
mide 3 cm de tamaño con ganglios linfáticos negativos. El estudio del receptor de estrógeno es
negativo. ¿Cuál es la terapia adyuvante más adecuada para este paciente?
A

89.. Una mujer de 26 años sufre un grave accidente automovilístico. Está hipotensa (PA 80/40,
FC 120), intubada, abdomen distendido. Tiene una fractura pélvica. Recibe 2 Litro de lactato
ringers y se le inicia transfusión de sangre pero aún permanece hipotensa. El paso más
apropiado a seguir es :
R: C
90. Una niña de 6 años se aplastó la mano, el antebrazo y la parte inferior del brazo en un
accidente automovilístico. Toda la extremidad superior se ve magullada y maltratada, aunque los
pulsos son normales y los huesos no están rotos.
Usted debe tomar en consideración en el manejo de este paciente:
C. Cursa con HIPERPOTASEMIA MIOGLOBINEMIA NOSEQUE FASCIOTOMIA DE ULTIMO
92. Un lactante de 5 semanas de edad es llevado a consulta por padecimiento actual de una
semana de evolución con vomito progresivo, sin bilis y que se relaciona con antecedente de
disminución del volumen urinario de 24 horas. El niño se encuentra activo y ávido de alimento. A
la exploración se encuentra depresión de la fontanela y disminución de la turgencia cutánea.
Hay abdomen en batea y al realizar la prueba de alimentación se observa onda peristáltica en el
epigastrio. ¿Cuál es el diagnóstico más probable?
R: C
93. Mujer de 55 años de edad acude con historia de seis meses de evolución de pérdida de
peso, dolor abdominal y diarrea intermitente no sanguinolenta. A la exploración hay distensión
leve del abdomen y se palpa masa en cuadrante inferior derecho. Los cultivos de heces reportan
flora fecal normal. El CT con contraste hay masa inflamatoria en el cuadrante inferior derecho
con engrosamiento del íleon terminal y válvula ileocecal.
¿Cuál es el diagnóstico más probable?
R: C

94. Una mujer de 39 años se presenta al servicio de urgencias quejándose de dolor abdominal
intenso. Tiene antecedentes de enfermedad ulcerosa péptica. El examen físico revela defensa y
rebote. La llevan a la sala de operaciones para laparotomía exploratoria. Durante el
procedimiento, el cirujano que abre el ligamento gastrosplénico para alcanzar el saco menor
corta accidentalmente una arteria. ¿Cuál de los siguientes vasos es el más probable?
R: C
95.En la manometria esofágica el hallazgo de una presión alta en reposo o bien una relajación
nula o incómpleta durante la deglución a nivel del esfinter esofágico inferior, es patognomónico
de: R: A

96.Paciente con choque hemorrágico el gasto cardiaco se mantiene inicialmente debido a


97.¿Qué estructura pasa por el hiato esofágico, además del propio esófago?:
E

98.Un hombre de 25 años sufre un accidente automovilístico al examen , sólo abre sus ojos al
estímulo doloroso, no pronuncia palabras responde con lenguaje incomprensible y evita estímulo
doloroso retirando el segmento corporal explorado. El puntaje de Glascow de este paciente es :
99.El tratamiento más apropiado en el papiloma intraductal sería:
D.

100.Se realiza una laparoscopía a un hombre de 25 años con diagnóstico presunto de


apendicitis, se encuentra una ileitis terminal que no está envolviendo el área cecal. La ileitis no
es obstructiva , el paso más apropiado a seguir es :
A.apendi

Un hombre de 46 años se presenta a su médico de atención primaria para la evaluación de una


lesión cutánea. Se queja de hipopigmentación de la piel de la espalda baja. Tiene antecedentes
médicos previos de eccema y carcinoma basocelular. Es un granjero que pasa mucho tiempo al
aire libre. ¿Qué células son responsables de esta condición?
R/= D
Semestral cirugía
Med12 TOPITOS

Mujer de 65 años sin comorbilidades, operada de cirugía conservadora de mama


izquierda con biopsia selectiva del ganglio centinela. La anatomía patológica de la pieza
muestra un carcinoma ductal infiltrante grado III de 11 mm, dos ganglios centinelas
negativos, receptores de estrógenos: 0%, receptores de progesterona: 0%, Ki 67: 70%,
HER2: negativo. El estudio de extensión es negativo. El tratamiento adyuvante sería:
Qmterapia basada en antra, taxanos, radio

La radioterapia en el cáncer de mama NO está indicada en:


Estadio I post-mastectomía.

Un hombre de 67 años que se recupera de una exenteración pélvica secundaria a un


cáncer de recto localmente avanzado se inicia con nutrición parenteral total para el íleo
prolongado a través de un catéter central insertado periféricamente en el lado derecho
(PICC). Varios días después, la enfermera informa que no puede extraer sangre de su
línea PICC. El brazo está hinchado. La ecografía confirma un coágulo en las venas
basílica y axilar. ¿Cuál es el manejo adecuado de su condición?la cicatrizacion tardia de
las heridas se debe a que no se hidroxila la lisina

Ante un paciente con pericarditis aguda y derrame pericárdico grave ¿cuál de las
siguientes medidas es INCORRECTA?: Administracion de diureticos que faciliten la
desaparicion del derrame

Un niño de 12 años severamente desnutrido se presenta con múltiples manchas


pigmentadas en sus muslos bilaterales, encías sangrantes, dientes flojos y faltantes, y
varias heridas supurantes. Recientemente llegó como refugiado de un país
subdesarrollado. Su historial médico es escaso. Su dieta consistía principalmente en
cereales cocidos. ¿Cuál de las siguientes afirmaciones es verdadera con respecto a la
deficiencia vitamínica más probable en este paciente?

¿Cuál de las siguientes afirmaciones es verdadera con respecto a las quemaduras?


La quemadura de tercer grado de espesor total...fascia subyacente

La hipoglucemia postabsortiva por tumores no insulinomas se caracteriza:


niveles de proinsulina, insulina y peptido c disminuidos

Cuatro días después de una pancreatoduodenectomía por adenocarcinoma de páncreas, un


hombre de 65 años presenta fiebre y taquicardia. El examen revela dolor a la palpación, edema
y eritema sobre el ángulo de la mandíbula. ¿Cuál de las siguientes afirmaciones es verdadera
con respecto a esta afección? Esta asociado con una disminucion de formacion de la saliva

Un hombre de 45 años se presenta con náuseas, vómitos y disnea. La historia pasada es


significativa por una colisión de vehículo de motor 5 años antes por la cual estuvo hospitalizado
durante una semana, sin requerir cirugía. Hay disminución de los ruidos respiratorios en el
hemitorax izquierdo. La radiografía de tórax (CXR) muestra un hemidiafragma izquierdo elevado,
con borramiento del ángulo costofrénico, así como múltiples burbujas de gas en la parte inferior
izquierda del tórax. El recuento de leucocitos es de 18.000 células / μL. ¿Cuál de las siguientes
es la mejor recomendación de manejo? Laparotomía exploratoria
(

Hombre de 57 años, exfumador, con antecedentes de pancreatitis aguda alitiásica, gastritis


crónica por AINES y disfunción eréctil en tratamiento con pantoprazol y sildenafilo. Consulta por
dolor torácico opresivo a las 7:30 h a.m. mientras se encontraba en reposo. PA 148/82 mmHg,
FC 85 lpm, Sat 98% basal. AC: rítmico, sin soplos. AP: murmullo vesicular conservado. No
edemas, pulsos presentes y simétricos. Se le realiza un ECG de superficie (imagen). ¿Cuál de
los siguientes fármacos está contraindicado en este paciente? Atenolol

De los cinco momentos recomendados en la higiene de manos, señale la respuesta


INCORRECTA: antes del contacto con el entorno CREO

Un hombre de 67 años que se recupera de una exenteración pélvica secundaria a un cáncer de


recto localmente avanzado se inicia con nutrición parenteral total para el íleo prolongado a
través de un catéter central insertado periféricamente en el lado derecho (PICC). Varios días
después, la enfermera informa que no puede extraer sangre de su línea PICC. El brazo está
hinchado. La ecografía confirma un coágulo en las venas basílica y axilar. ¿Cuál es el manejo
adecuado de su condición? iniciar heparina, mantener la guía y anticoagulacion de 3 a 6 meses

Una mujer de 28 años se somete a adhesiolisis (lisis de adherencias)por una obstrucción aguda
del intestino delgado. Durante el curso de la cirugía, requiere una resección ileal segmentaria
con anastomosis primaria. En el sexto día posoperatorio, se observa que tiene un líquido espeso
de color biliar que emana de la línea media de la herida. Después de la hidratación intravenosa,
el siguiente paso en el manejo debe ser: reexploracion operativa creo

Hombre de 35 años, ingresado por traumatismo torácico grave con múltiples fracturas costales.
Tras responder favorablemente al tratamiento con analgésicos y oxígeno, comienza a presentar
hipoxemia grave. Señale cuál es la causa más probable de este deterioro: Alteración del
intercambio gaseoso por contusión pulmonar CREO

Para conocer si el uso habitual de ácido acetilsalicílico se asocia a un mayor riesgo de


hipertensión se selecciona un grupo de sujetos, se averigua cuántos están tomando ácido
acetilsalicílico y se les sigue durante 5 años para identificar los casos nuevos de hipertensión.
Señale la respuesta correcta sobre el diseño de este estudio:
COHORTE

Pregunta vinculada a la imagen Hombre de 56 años, fumador activo, no hipertenso ni


dislipémico, que consulta porque hace 7 días presentó malestar general con dolor centrotorácico
y disnea de esfuerzo, encontrándose asintomático en el momento de la consulta. ¿Cuál es el
diagnóstico más probable, según el trazado electrocardiográfico? Necrosis e isquemia
subepicardica inferior CREO

¿Cuál de las siguientes enfermedades se relaciona correctamente con el tipo de colágeno


afectado? EPIDERMOLISIS AMPOLLOSA COLAGENO TIPO 7
Hombre de 20 años con antecedentes de asma, que consulta por disfagia e impactación
alimentaria. Se le realiza una endoscopia digestiva alta con toma de biopsias esofágicas. ¿Con
qué diagnóstico es compatible esta biopsia de esófago proximal?: eosinofílica

Un bebé recién nacido nace con un defecto de la pared abdominal. El defecto afecta al ombligo
y tiene una membrana asociada. ¿Cuál de las siguientes afirmaciones es verdadera con
respecto a este tipo de defecto? tienen anomalías cardiacas y genéticas asociadas

Paciente de 47 años, sin antecedentes patológicos de interés. Presenta un cuadro de fiebre y


dolor en punta de costado derecho y ha sido diagnosticado de neumonía adquirida en la
comunidad en su Centro de Salud. En una radiografía simple de control, realizada 3 días
después del inicio del tratamiento antibiótico, se objetiva un derrame pleural ipsilateral que
ocupa un tercio del hemitórax derecho. Señale la respuesta correcta: Debe sospecharse el
diagnostico de empiema si persiste la fiebre y leucocitosis

En un paciente con cáncer de pulmón sin comorbilidad, la combinación de quimioterapia y


radioterapia es el tratamiento de primera elección en: Cáncer de célula pequeña

Mujer de 67 años con antecedentes de dislipidemia, que acude a urgencias por un cuadro de
disuria y poliaquiuria seguido de fiebre, escalofríos y deterioro del estado general. A su llegada
impresiona de gravedad y está taquicárdica, taquipneica, con tensión arterial 60/40 mmHg y
temperatura de 39°C. ¿Qué medida de entre las siguientes NO estaría incluida en el manejo
INICIAL?: Perfusión intravenosa de dobutamina

Mujer de 67 años, obesa, que acude a urgencias por dolor abdominal en cuadrante superior
derecho del abdomen, fiebre de 39,5°C y orinas muy oscuras. Ante este cuadro se debería
sospechar en primer lugar: COLEDOCOLITIASIS
Pregunta vinculada a la imagen Joven de 16 años sin antecedentes previos, que es traído a
urgencias por sus familiares por dificultad respiratoria aguda y dolor torácico. A su llegada
muestra intenso trabajo respiratorio, frecuencia cardiaca 120 lpm, tensión arterial 75/40 mmHg y
SatO2 86 % con aire ambiente. Considerando la radiografía de tórax, ¿cuál es la medida a
tomar más apropiada?: Colocar un drenaje en la cavidad pleural derecha

Hombre de 73 años con antecedente de infarto agudo de miocardio tratado con angioplastia
coronaria transluminal percutánea hace 6 semanas. Acude a urgencias por dolor brusco, frialdad
y pérdida de motilidad de la extremidad inferior derecha. En la exploración está presente el pulso
femoral derecho, con ausencia del resto de los pulsos de esa extremidad. Conserva pulsos a
todos los niveles en la extremidad contralateral. Respecto al diagnóstico de sospecha, señale la
afirmación FALSA: En caso de parálisis rígida con anestesia profunda y ausencia de señal
Doppler esta indicada una arteriografia urgente (creo, no sé)

¿En qué individuos se observa una elevación plasmática de la isoenzima BB de la creatina


quinasa (CK)?En pacientes con un accidente cerebrovascular

Un hombre obeso de 55 años acude al hospital para su procedimiento de gastrectomía en


manga bariátrica. Sus comorbilidades incluyen diabetes e hipertensión, y afirma que hace dos
semanas le diagnosticaron “neumonía ambulante” y le recetaron antibióticos, que ya terminó.
¿Cuál de las siguientes opciones no sería beneficiosa si se siguieran las medidas del SCIP para
el cuidado preoperatorio y posoperatorio? suspender los antibioticos el primer dia post op

¿Cuál de las siguientes funciones NO corresponde a los Comités de Ética Asistencial?: Explorar
las necesidades formativas en el campo…...

Pregunta vinculada a la imagen Mujer de 72 años con antecedentes de fibrilación auricular en


tratamiento con anticoagulantes orales. Presenta dolor abdominal focalizado en hipogastrio y
fosa ilíaca izquierda. INR 5. Hemoglobina 8,7 g/dl, hematocrito 25 %. ¿Cuál es el diagnóstico
que le sugiere la imagen?: Hematoma en la vaina del recto anterior izquierdo del abdomen
Hombre de 65 años que consulta por disfagia a sólidos desde hace dos meses. La
esofagoscopia evidencia tumoración a 30 cm de arcada dental, parcialmente estenosante, con
anatomía patológica de carcinoma epidermoide. Se solicita ecografía endoscópica y PET-TC
donde no se observan adenopatías patológicas. ¿Cuál de las siguientes opciones sería la más
correcta?: quimioterapia más radioterapia

Un hombre de 35 años es trasladado al servicio de urgencias tras ser apuñalado en el abdomen


derecho. Se queja de dolor abdominal mínimo sin rebote ni defensa. Durante la exploración local
de la herida (LWE), parece que no se viola la fascia anterior. Sus signos vitales son normales.
¿Cuál de los siguientes es el manejo más apropiado? admision para observacion de 24 horas

Mujer de 27 años con tos no productiva, fiebre y dolor pleurítico de un mes de evolución. En la
radiografía de tórax se objetiva un derrame pleural izquierdo. El análisis del líquido pleural
muestra un exudado linfocítico con pH 7,32, glucosa 66 mg/dL y adenosin-deaminasa 59 U/L.
¿Cuál es la etiología más probable?: tuberculosis pleural CREO

Una mujer de 28 años se somete a adhesiolisis (lisis de adherencias)por una obstrucción aguda
del intestino delgado. Durante el curso de la cirugía, requiere una resección ileal segmentaria
con anastomosis primaria. En el sexto día posoperatorio, se observa que tiene un líquido espeso
de color biliar que emana de la línea media de la herida. Después de la hidratación intravenosa,
el siguiente paso en el manejo debe ser: reexploración operativa

Un paciente es traído a urgencias desde un incendio producido dentro de una nave cerrada
donde había espumas de poliuretano. Está consciente, pero presenta creciente torpeza mental,
cefalea e intensa disnea. La saturación de oxígeno mediante pulsoximetría es del 92% y el ácido
láctico capilar es de 8 mEq/l. ¿Qué tratamiento específico considera más adecuado?:
administración de oxígeno al 50%

Hombre de 42 años con antecedente de cirrosis hepática compensada, traído a urgencias por
ictericia, fiebre, aumento del perímetro abdominal y deterioro significativo del estado general.
¿Cuál de los siguientes parámetros analíticos NO le aportaría información acerca del
pronóstico?:Bilirrubina total serica o transaminasas?
Un paciente de 69 años con un tumor en la unión rectosigmoidea se somete a colectomía
sigmoidea laparoscópica. El dolor posoperatorio se controla bien con anestesia epidural torácica
controlada por el paciente. El día 1 del postoperatorio se inicia la anticoagulación profiláctica con
heparina de bajo peso molecular (HBPM). Se está drenando la vejiga con un catéter de Foley
permanente. ¿Cuál de las siguientes afirmaciones es verdadera con respecto a la anestesia
epidural? Ni idea, El cateterismo vesical debe continuar mientras se coloca la epidural torácica

Pregunta vinculada a la imagen Hombre de 56 años con antecedentes de hipertensión arterial y


cáncer de colon con metástasis hepáticas, actualmente en tratamiento quimioterápico. Acude a
urgencias por disnea. Constantes: frecuencia cardiaca 110 lpm, tensión arterial 115/55 mmHg,
frecuencia respiratoria 29 rpm, SatO2 89 %. Exploración: uso de musculatura respiratoria
accesoria, murmullo vesicular conservado en todos los campos. La primera prueba diagnóstica
de la que se dispone es el electrocardiograma. ¿Cuál es la sospecha clínica más probable?:
TEP

Paciente con insuficiencia cardiaca crónica en quien detectamos unas ondas "v" prominentes en
el pulso venoso yugular y en la auscultación cardiaca un soplo holosistólico en el área del
apéndice xifoides que se acentúa con la inspiración profunda. ¿Cuál es la valvulopatía que
sugiere esta exploración física?: estenosis aórtica? CREO que si es

Un hombre de 29 años llega al servicio de urgencias después de una colision de vehiculo de alta
velocidad con una escala de coma de Glasgow (GCS) de 4. Tiene un collarín cervical que le
colocaron los servicios médicos de emergencia (EMS). Se lo intuba y se le realiza una
tomografía computarizada, que muestra una hemorragia subdural grande y una hemorragia
punteada difusa sin evidencia de lesión de la columna cervical. Está ingresado en la UCI. Con
respecto al manejo del collarín cervical, ¿cuál de los siguientes se recomienda? Cambie el
coolarin por uno blando y realizarle una mri cspine

La imagen de TC sin administración de contraste es característica de:

HEMATOMA INTRAUMRUAL AORTICO


¿Cuál de las siguientes afirmaciones respecto a la tiroiditis autoinmune o de Hashimoto es
INCORRECTA?: Los pacientes con Klinefelter y Turner tienen menor incidencia

¿Cuál de los siguientes pacientes tiene la mejor indicación para una colecistectomía
laparoscópica? Mujer de 22 años embarazada con colelitiasis sintomática

Mujer de 30 años que acude a su consulta porque se nota un bulto en la mama derecha de
reciente aparición. Su abuela tuvo cáncer de mama. A la exploración se palpa en cuadrante
súpero-externo un nódulo de 2,5 cm de bordes regulares. No tiene ninguna prueba de imagen
previa. Señale la respuesta correcta: SOLICITA ECOGRAFIA MAMARIA

Una mujer de 34 años se somete a una tiroidectomía subtotal por enfermedad de Graves. En la
sala de recuperación, desarrolla ansiedad y dificultad respiratoria progresiva con estridor. Su
incisión está abultada y tensa en el examen. El paso inicial más importante sería: intubación de
secuencia rapida

De los siguientes métodos, ¿cuál es el más fiable para la estimación del intervalo post-mortal en
el período precoz de la muerte?:valoracion de la reactividad ocular
SS-Med10

SEMESTRAL DE CX
Gran parte del semestral fue sacado del Blueprints, el doctor cambio varias respuestas.
Tengan en cuenta eso, al momento de comparar y que la fuerza los acompañe.

1. Un hombre de 18 años es llevado al departamento de emergencias


con un dolor abdominal repentino insoportable localizado en el
cuadrante inferior derecho, náuseas y vómitos, fiebre leve y
taquicardia leve. Tiene antecedentes médicos previos de otitis media
recurrente. El examen físico revela marcado rebote inferior derecho
y defensa. El recuento de glóbulos blancos en suero es de 18,000 /
mL. La radiografía simple de abdomen revela gas en el intestino
delgado y grueso. ¿Cuál es el diagnóstico más probable?
a. Apendicitis
2. Una mujer de 36 años se queja de ictericia e hinchazón periférica. Se
ordena un ecocardiograma y se determina que el paciente tiene
insuficiencia cardíaca derecha con congestión hepática y edema
periférico. No se detecta murmullo. ¿Cuál es la explicación más
adecuada para estos hallazgos?
a. Comunicación interauricular
3. Un hombre de 30 años se somete a una tomografía computarizada de
abdomen después de un accidente automovilístico. Conducía sin
cinturón y fue arrojado del vehículo. No se encuentran lesiones
abdominales agudas. La tomografía computarizada revela riñones
bilateralmente agrandados con quistes múltiples presentes de
diferentes tamaños. El riñón derecho mide 15 cm y el riñón izquierdo
tiene 16 cm de longitud. El examen físico del corazón, los pulmones y
el abdomen están dentro de los límites normales, además de cierta
sensibilidad leve a la palpación en los cuadrantes superiores derecho
e izquierdo. ¿Cuál de las siguientes patologías del sistema nervioso
central está más fuertemente asociada con este hallazgo?
a. Aneurisma del poligono de Willis
4. Un niño de 4 años está en la lista de espera para un trasplante de
hígado. Tiene enfermedad hepática en etapa terminal y actualmente
está hospitalizado por hemorragia por varices esofágicas. ¿Cuál es la
causa más probable de insuficiencia hepática en este paciente?
a. Atresia biliar
5. Un joven recibe un disparo en el brazo con un revólver calibre .38. La
trayectoria de la bala atraviesa la extremidad, desde el lado medial al
lateral. Tiene un gran hematoma en el aspecto interno del brazo, sin
pulsos distales, parálisis del nervio radial y húmero destrozado. La
secuencia quirúrgica para la reparación de esta grave lesión sería:
a. Estabilización de la fractura por ortopedia, luego la repación
vascular tanto de la arteria como de la vena, seguido de la
reparación del nervio y posteriormente se realizara una
fasciotomia.
SS-Med10

6. Un hombre de 55 años se presenta a su médico quejándose de


poliuria, polidipsia, polifagia y un sarpullido rojo y escamoso en la
cara en las últimas 2 semanas. Orina 18 veces / día con una buena
fuerza de flujo. La glucemia en ayunas fue de 325 mg / dL. Ha perdido
20 libras en los últimos 2 meses y nunca ha tenido niveles elevados
de glucosa en sangre en el pasado. El examen físico del corazón, los
pulmones y el abdomen son normales. ¿Cuál es la explicación más
probable para estos hallazgos?
a. Glucagonoma
7. Paciente A de 25 años recibe un disparo con un revólver calibre .22.
La herida de entrada se encuentra en la cara anterolateral de su
muslo, y la bala se ve por rayos X incrustándose en los músculos,
posterolateral al fémur. Paciente B de 25 años recibe un disparo con
un revólver calibre .22. La herida de entrada está en el aspecto
anteromedial de la parte superior del muslo, y la herida de salida está
en la cara posterolateral del muslo. Tiene pulsos normales en la
pierna y ningún hematoma en el sitio de entrada. Los rayos X
muestran que el fémur está intacto. Paciente C de 25 años recibe un
disparo con un revólver calibre .22. La herida de entrada está en el
aspecto anteromedial de la parte superior del muslo, y la herida de
salida está en la cara posterolateral del muslo. Él tiene un gran
hematoma en expansión en la parte superior e interna del muslo. El
hueso está intacto. Acerca de los 3 pacientes anteriores, cuál es la
mejor respuesta?
a. Es muy probable que el px C por la trayectoria de la bala y el
hematoma en expasion, tenga una lesión vascular por lo que
debe ser intervenido quirúrgicamente sin ninguna otra demora
8. Una mujer premenopáusica de 31 años con una masa mamaria
izquierda se somete a una mastectomía radical izquierda modificada.
La patología revela carcinoma ductal infiltrante que mide 3 cm de
tamaño con ganglios linfáticos negativos. El estudio del receptor de
estrógeno es negativo. ¿Cuál es la terapia adyuvante más adecuada
para este paciente?
a. Quimioterapia
9. Una mujer de 21 años es apuñalada en el pecho por su novio. La
llevan al departamento de emergencias para su evaluación. Su
presión arterial es de 130/80 mmHg, y su pulso es de 90 latidos por
minuto. El examen físico revela una herida de arma blanca en el
quinto espacio intercostal izquierdo en la línea medioclavicular. El
examen del cuello es normal. La tráquea está en la línea media y las
venas yugulares no están distendidas. Ella tiene una disminución de
los sonidos respiratorios en los campos pulmonares izquierdos.
¿Cuál de los siguientes diagnósticos hay que descartar sobre la base
de la información anterior?
a. Neumotórax abierto
SS-Med10

10. Un hombre de 72 años con un historial de tabaquismo de 80


años, se queja de debilidad y malestar general. Recientemente ha
desarrollado disfagia a alimentos sólidos. Ha perdido 15 libras en
los últimos 3 meses. El examen físico revela linfadenopatía
supraclavicular derecha. Los exámenes cardíacos y pulmonares no
son relevantes. No tiene defensa o rebote. ¿Cuál de los siguientes
estudios proporcionará el diagnóstico definitivo?
a. Esofagogastrocospia con biopsia
11. Un hombre diestro de 62 años de edad tiene episodios
transitorios de debilidad en la mano derecha, visión borrosa y
dificultad para expresarse. No hay dolor de cabeza asociado, los
episodios tienen un inicio repentino, duran alrededor de 5 o 10
minutos como máximo, y se resuelven espontáneamente, sin dejar
secuelas neurológicas. La secuencia a seguir con éste paciente será:
a. La sintomatología es típica de una isquemia cerebral
transitoria y debe realizarse un doppler dúplex carotideo para
determinar si la estenosis es mayor de 70% y entonces
proceder con una endarterectomia carotidea izquierda
12. Una mujer de 28 años se presenta a su médico para evaluar un
bulto en su seno derecho que se encuentra en el autoexamen. Tiene
antecedentes familiares de cáncer de seno en el sentido de que su
madre murió a los 40 años por esta afección. La madre tuvo una
mastectomía radical modificada seguida de quimioterapia. El examen
físico revela un bulto en el seno que es libremente móvil y está bien
circunscrito. No hay hoyuelos, asimetría o retracciones. La lesión
mide 2 cm. ¿Cuál es el siguiente paso en el manejo de este paciente?
a. Ultrasonido del seno y concideracion para biopsia de seno
13. Un hombre de 68 años es llevado al departamento de
emergencias quejándose de dolor abdominal y en las piernas durante
2 semanas. Tiene antecedentes de hipertensión e
hipercolesterolemia; pesa 290 lb. El examen físico revela una masa
abdominal pulsátil en la línea media. Las extremidades inferiores
tienen pulsos desiguales. ¿Cuál de los siguientes es el mejor próximo
paso en la evaluación de este paciente?
a. Tomografía computarizada del abdomen y pelvis
14. Un agresor apuñaló a un hombre de 18 años en su abdomen
varias veces durante un altercado que implicaba la venta de drogas
ilícitas. Lo llevan al departamento de emergencias para su
evaluación. Tiene cuatro heridas de arma blanca en el abdomen, tres
en el cuadrante superior derecho y una en el cuadrante inferior
izquierdo. El examen físico del abdomen revela defensa y sensibilidad
de rebote. El paciente es llevado a cirugía para una laparotomía
exploratoria. Se encuentra una lesión penetrante en la vesícula biliar.
¿Cuál de las siguientes vísceras asociadas es probable que se
lesione?
a. Hígado
SS-Med10

15. Una mujer de 23 años que se queja de heces grasas y


fétidas(esteatorrea), debilidad generalizada y pérdida de cabello,
se presenta a su médico de atención primaria para su evaluación.
El examen físico del corazón, los pulmones y el abdomen son
irrelevantes. Ella no tiene defensa, terneza, dolorimiento o rebote.
Los sonidos intestinales están presentes en todos los cuadrantes. El
examen pélvico femenino fue diferido a pedido de la paciente. ¿Cuál
es la explicación más probable de estos hallazgos?
a. Insuficiencia pancreática
16. Un hombre de 69 años se presenta a su dermatólogo con una
lesión en la nariz. Es un jardinero que pasa gran parte de su tiempo al
aire libre. Tiene antecedentes médicos previos de rinitis alérgica,
hipertensión y diabetes mellitus. Sus medicamentos actuales
incluyen un betabloqueante y un hipoglucemiante oral. El examen
físico de su nariz revela una lesión papular brillante y elevada con
pequeños vasos sanguíneos. ¿Cuál es el diagnóstico más probable?
a. Carcinoma de células basales
17. Un hombre de 35 años con antecedentes de enfermedad de
Crohn se presenta a su médico para un examen de seguimiento.
Tiene enfermedad ileocolónica difusa en una tomografía
computarizada reciente. Sus medicamentos actuales incluyen
sulfasalazina. El examen físico revela dolor en el cuadrante inferior
derecho a palpación profunda. ¿Debería considerarse la terapia con
antibióticos en este paciente, a cuál de los siguientes organismos
debería dirigirse?
a. Especia de Mycobacterium
18. Una mujer de raza negra de 42 años se somete a una
colecistectomía laparoscópica por dolor crónico en el cuadrante
superior derecho. La tomografía computarizada demostró cálculos
biliares y líquido pericolecístico. El procedimiento quirúrgico no fue
complicado. El análisis patológico de los cálculos biliares reveló
cálculos de bilirrubinato de calcio. ¿Cuál es la explicación más
probable de estos hallazgos?
a. Anemia falciforme
19. Un hombre de 78 años es llevado al departamento de
emergencias con un historial de 12 horas de dolor abdominal, diarrea
y vómitos. Tiene antecedentes de fibrilación auricular y fue tratado
previamente por insuficiencia cardíaca congestiva con digoxina. El
examen físico revela un abdomen distendido con defensa
significativa. El examen rectal revela heces guayaco positivas. El
recuento de glóbulos blancos es de 24,000 / mL. La radiografía
abdominal revela edema de la pared intestinal. ¿Cuál es el
tratamiento más apropiado para este paciente?
a. Exploración quirúrgica
20. Una mujer de 19 años comenzó a amamantar por primera vez.
Al principio, era difícil para su bebé alimentarse. Ahora, sus senos
SS-Med10

están rojos, cálidos y doloridos. Ella ha seguido amamantando, a


pesar del dolor; sin embargo, recientemente comenzó a usar un
extractor de leche en lugar de amamantar. Ella inicia un curso de
antibióticos orales. ¿Qué afección tiene esta paciente riesgo de
desarrollar?
a. Absceso mamario
21. Un hombre de 44 años con enfermedad renal en etapa terminal
se somete con éxito a un trasplante renal. Tiene antecedentes
médicos previos de hiperparatiroidismo. Seis meses después del
trasplante renal, su calcio sérico sigue siendo 13 mg / dL. ¿Cuáles de
los siguientes hallazgos de laboratorio son posibles en este paciente?
a. Calcio de orina elevado
22. Una mujer de 71 años se presenta a su médico de atención
primaria quejándose de sangrado rectal. Tenía algunos calambres
abdominales leves en el lado izquierdo que disminuyeron en pocos
minutos. Ella nunca ha tenido un episodio previo de sangrado rectal.
El examen físico revela dolor abdominal leve en el cuadrante inferior
izquierdo sin evidencia de defensa o rebote. El examen rectal revela
que no hay sangre fresca en la ampolla rectal. La colonoscopia revela
varias colecciones fuera de la pared del colon sigmoideo sin
evidencia de sangrado o perforación. El resto de la colonoscopia está
dentro de los límites normales. El recuento de glóbulos blancos es
normal. ¿Cuál es el tratamiento más apropiado para este paciente?
a. Espera vigilante
23. Una estudiante de la facultad de medicina de 21 años conduce
bajo la influencia del alcohol, a pesar de las recomendaciones de
amigos de no conducir. Ella es golpeada por otro conductor. La
fuerza del impacto hace que golpee el área temporal de su cráneo
contra la ventana. Ella desarrolla un dolor de cabeza leve pero no
pierde el conocimiento. Varias horas después, desarrolla un fuerte
dolor de cabeza con náuseas y vómitos. ¿Cuál es el diagnóstico más
probable?
a. Hematoma epidural
24. Un joven de raza negra de 15 años se sometió a una
esplenectomía después de sufrir una herida de cuchillo durante una
pelea. Se presenta a su médico de atención primaria para un examen
físico deportivo. Su madre leyó en la Web que tiene un mayor riesgo
de infección. ¿Debería recibir una de las siguientes vacunas para
prevenir infecciones graves?
a. Vacuna contra organismos encapsulados comunes
25. Sir David Cuthbertson en 1930 describe varias fases de
respuesta a la agresión, cual enunciado es el correcto ?
a. En la fase flow ocurre aumento de citoquinas, catecolaminas,
excreción de N2, DO2
26. Un médico evalúa a un hombre de 39 años para detectar
cálculos renales recurrentes. Ha sido tratado en el pasado con
SS-Med10

litotricia extracorpórea por ondas de choque, ureteroscopia y


una nefrolitotripsia percutánea. ¿Cuál de las siguientes
características sugeriría el diagnóstico de hiperparatiroidismo
primario?
a. Piedras del lado derecho y izquierdo de 1 mm, 2mm, y 3 mm en
la radiografía, de abdomen simple.
27. Paciente de 7 meses de edad que acude por dolor abdominal
súbito, con llanto inusual que inicio hace más o menos 12 horas y que
ha presentado 1 evacuación con moco y sangre. El único
antecedente haber estado con resfriado. Su primer diagnostico es:
a. Invaginación intestinal
28. Un varón recién nacido tiene una abertura de la pared
abdominal en el ombligo. No tiene otro antecedente médico o
quirúrgico previo. El historial del nacimiento no es contribuyente.
Durante el resto del examen físico y las pruebas de diagnóstico,
¿Cuál de los siguientes hallazgos es más probable?
a. Paladar hendido
29. Un hombre de 76 años de edad con antecedentes de dolor vago
en el cuadrante superior derecho, una pérdida de peso de 25 libras y
anorexia se presenta a su médico de atención primaria para su
evaluación. El examen físico revela ictericia escleral. El examen
abdominal revela una masa en el cuadrante superior derecho. La Rx
de Abdomen Riñón, uréter y vejiga (KUB) revelan una calcificación
circular en el cuadrante superior derecho. La laparotomía
exploratoria revela un proceso neoplásico que involucra la vesícula
biliar y el hígado. ¿Cuál es la patología más probable que causa esta
afección?
a. Adenocarcinoma
30. Un varón recién nacido nacido a término de una mujer
drogadicta de 27 años, tiene una hernia umbilical pequeña. Sus
signos vitales son estables. Sus exámenes cardíacos y pulmonares
no son contributivos. ¿Cuál de las siguientes es la explicación más
probable para este hallazgo?
a. Anillo umbilical permeable
31. Una mujer de 31 años se queja de una historia de 6 meses de
diarrea con sangre, dolor abdominal y fiebre intermitente. Tiene
antecedentes de síndrome del intestino irritable, pero ha empeorado
sus síntomas durante el período de tiempo anterior. Su historial
médico pasado es irrelevante. El examen físico revela distensión
abdominal. Los sonidos intestinales están presentes en todos los
cuadrantes. El examen rectal revela múltiples fisuras anales. ¿Cuál
es la prueba de diagnóstico más adecuada para este paciente?
a. Colonoscopia
32. Un hombre de 85 años es llevado al servicio de urgencias
debido a dolor abdominal agudo y distensión abdominal progresiva.
Es residente de un hogar de ancianos local. No ha estado comiendo
SS-Med10

debido a las náuseas progresivas. Las radiografías abdominales


revelan un colon masivamente sigmoideo. ¿Cuál es el tratamiento
inicial para este paciente?
a. Descompresión del tubo rectal
33. Un hombre de 72 años se derrumba mientras camina en un
centro comercial. Está sin pulso y apneico. No hay antecedentes de
trauma. La reanimación cardiopulmonar se inicia hasta que llega un
escuadrón de rescate. Se inicia el protocolo avanzado de soporte
vital cardíaco. Es declarado muerto 40 minutos después. La autopsia
revela necrosis miocárdica con ruptura del ventrículo izquierdo.
¿Cuál de los siguientes es el factor de riesgo más probable que
contribuyó a su muerte?
a. Antecedentes familiares de DM2
34. Una mujer de 41 años con enfermedad de Crohn se sometió a
múltiples procedimientos quirúrgicos. Recientemente se sometió a
una ileostomía, pero aún tiene evidencia de alguna enfermedad
yeyunal distal. Sus medicamentos actuales incluyen prednisona y
ácido aminosalicílico. ¿Cuáles de los siguientes efectos de la terapia
prolongada con glucocorticoides son posibles para este paciente?
a. Disfunción de fibroblastos
35. Una mujer de 46 años se presenta en el departamento de
emergencias quejándose de dolor en el cuadrante superior derecho y
fiebre a 38.9 ° C. El examen físico revela ictericia escleral y dolor
significativo en el cuadrante superior derecho a la palpación. Los
signos de irritación peritoneal están ausentes. Los ruidos intestinales
están presentes. ¿Cuál de los siguientes debe incluirse en el
tratamiento inicial de este paciente?
a. Antibiótico
36. Una mujer de 40 años se queja de dolor en el pecho y disfagia a
los sólidos. Ella se presenta a un especialista para su evaluación. Se
realizan estudios manométricos esofágicos que revelan
contracciones de gran amplitud y eventual relajación normal del
esfínter esofágico inferior. La ingestión de bario es normal. ¿Cuál es
el diagnóstico más probable?
a. Espasmo esofágico difuso
37. Un hombre de 44 años con pancreatitis recurrente es llevado al
departamento de emergencias con otro episodio de pancreatitis.
¿Cuál de los siguientes es el factor más tranquilizador con respecto a
la gravedad de su condición?
a. Edad
38. Un hombre de 46 años se presenta a su médico de atención
primaria para la evaluación de una lesión cutánea. Se queja de
hipopigmentación de la piel de la espalda baja. Tiene antecedentes
médicos previos de eccema y carcinoma basocelular. Es un granjero
que pasa mucho tiempo al aire libre. ¿Qué células son responsables
de esta condición?
SS-Med10

a. Melanocito
39. Una mujer de 47 años con antecedentes de masa tiroidea
izquierda se somete a una lobectomía tiroidea izquierda. La
patología revela un carcinoma papilar de 1.3 cm sin evidencia de
extensión extracapsular. ¿Cuál es el siguiente paso más apropiado en
el tratamiento de este paciente?
a. Radioterapia de haz externo
40. Un profesor de química de 37 años sufre una salpicadura
química de ácido en su ojo derecho mientras intenta realizar una
demostración en su clase de ciencias de la escuela secundaria. Él
tiene un dolor significativo. Mientras está en el aula y espera que una
ambulancia lo transporte al hospital, ¿cuál de las siguientes
intervenciones debe realizarse?
a. Lavar los ojos con 1 a 2 L de solución salina normal
41. Un niño de 17 años es llevado al departamento de emergencias
después de sufrir dolor en el pecho y disnea durante un juego de
baloncesto. El examen físico revela un soplo crescendo-decrescendo
sistólico, que se escucha mejor en el segundo espacio intercostal
derecho. El soplo se irradia a la arteria carótida derecha. La
radiografía de tórax revela un tamaño cardíaco normal. ¿Cuál de los
siguientes hallazgos se esperaría ver en un electrocardiograma en
este paciente?
a. Hipertrofia del ventrículo izquierdo
42. Durante su rotación de medicina de emergencia, lo llaman al
departamento de emergencias para evaluar a un paciente con una
lesión oftálmica. ¿Cuál de las siguientes lesiones traumáticas
oftálmicas requiere un manejo inmediato en el sitio y una derivación
al oftalmólogo de guardia?
a. Quemadura acida
43. Un estudiante universitario masculino de 21 años se presenta
en la clínica ambulatoria para un examen de rutina al comienzo del
semestre de otoño. Tiene antecedentes de síndrome del intestino
irritable. El examen físico del corazón, los pulmones y el abdomen no
tienen alteraciones. El examen genitourinario revela que los
testículos descienden bilateralmente. Un varicocele izquierdo de
grado 1 está presente. No hay masas testiculares. El pene no está
circuncidado y el prepucio no puede retraerse detrás del glande.
¿Cuál es el diagnóstico más probable?
a. Fimosis
44. Un hombre de 59 años resultó herido en un accidente
automovilístico. Una tomografía computarizada abdominal revela una
ruptura del bazo. Su presión arterial es de 90/40 mm Hg, y su pulso es
de 140 latidos por minuto. Se toma al paciente para laparotomía. Se
realiza esplenectomía. ¿Cuál de las siguientes anormalidades de
laboratorio es probable después de este procedimiento?
a. Anemia
SS-Med10

45. Con relación a la fisiopatología de la isquemia arterial


obstructiva crónica:
a. Si la placa ateromatosa se ulcera se puede producir
émbolos, y causar una obstrucción arterial
46. Un hombre de 47 años con antecedentes de enfermedad
pulmonar terminal del pulmón derecho está programado para un
trasplante de pulmón. La función cardíaca preoperatoria es buena.
No tiene antecedentes de defectos congénitos. ¿Cuál de las
siguientes es la incisión quirúrgica más apropiada para este
paciente?
a. Toracotomía lateral
47. Un hombre de 71 años con aparición repentina de dolor
abdominal y de espalda intenso es llevado al departamento de
emergencias para su evaluación. Tiene antecedentes de
hipertensión. Pesa 300 libras. Tiene historia de fumar un paquete
diario desde hace 45 años. El examen físico revela una masa
abdominal pulsátil. Ambas extremidades inferiores revelan palidez
con pulsos pedios disminuidos. ¿Cuál es la causa más probable de la
condición de este paciente?
a. Ateroesclerosis
48. Un hombre de 45 años aparece en el servicio de urgencias con
una extremidad inferior pálida, fría, sin pulso, parestésica, dolorosa y
paralítica. El proceso comenzó repentinamente hace 2 horas. El
examen físico no muestra pulsos en ninguna parte de esa extremidad
inferior. El pulso en la muñeca es 95 / min, extremadamente irregular.
¿Qué es?
a. Lo más probable es que sea una embolizacion de un coágulo
de la aurícula izquierda. El estudio de elección debe ser
doppler dúplex arterial de la extremidad inferior afectada, si
tiene una oclusión completa, se de realizar una embolectomía
con catéteres de Fogarty y si estuvo isquémico durante varias
horas, una fasciotomía para prevenir el síndrome
compartamental. Se debe dejar heparinizado y más adelante
realizar un eco cardiograma cardiaco para determinar la
naturaleza de la arritmia. Si la oclusión es incompleta se puede
tratar con medicamentos fibrinoliticos.
49. Un hombre de 78 años se presenta en el departamento de
emergencia para evaluar un dolor progresivo del cuadrante superior
derecho, náuseas, vómitos y una pérdida de peso de 30 libras en los
últimos 3 meses. Tiene antecedentes médicos previos de colelitiasis,
diabetes mellitus, hipertensión y demencia. El examen físico revela
ictericia escleral bilateralmente. El examen abdominal revela
sensibilidad en el cuadrante superior derecho y una masa palpable.
Los signos peritoneales están ausentes. La tomografía
computarizada revela ganglios linfáticos pancreáticos, duodenales y
coledocales. Hay un engrosamiento asimétrico de la vesícula biliar.
SS-Med10

¿Cuál es el hallazgo patológico más probable en la laparotomía


exploratoria y la biopsia?
a. Adenocarcinoma
50. Una mujer de 39 años se presenta al servicio de urgencias
quejándose de dolor abdominal intenso. Tiene antecedentes de
enfermedad ulcerosa péptica. El examen físico revela defensa y
rebote. La llevan a la sala de operaciones para laparotomía
exploratoria. Durante el procedimiento, el cirujano que abre el
ligamento gastrosplénico para alcanzar el saco menor corta
accidentalmente una arteria. ¿Cuál de los siguientes vasos es el más
probable?
a. Arteria gastroepiploica izquierda
51. Un hombre obeso de 49 años se presenta a su médico de
atención primaria para un examen de seguimiento. Tiene
antecedentes de diabetes mellitus no controlada y trastorno bipolar.
Sus medicamentos actuales incluyen litio y leche de magnesio. El
examen físico del corazón, los pulmones y el abdomen están dentro
de los límites normales. Los estudios de laboratorio revelan calcio
sérico de 14 mg / dL. ¿Cuál es la explicación más probable para estos
hallazgos?
a. Hiperplasia de paratiroides
52. Un hombre de 65 años con antecedentes de enfermedad
arterial coronaria puede ser un bypass aortobifemoral. ¿Cuál de las
siguientes maniobras de manejo intraoperatorio disminuirá su riesgo
de infarto de miocardio intraoperatorio?
a. Bloqueadores beta
53. Un hombre de 27 años es llevado al departamento de
emergencias después de cortarse la mano con un cuchillo mientras
intenta cortar un bagel. Tiene antecedentes médicos previos de
infecciones sinusales recurrentes. Su historial quirúrgico previo
señaló la reparación de una fractura nasal. El examen físico revela
una laceración limpia de 4 cm a lo largo del aspecto palmar de su
mano. Los principios más relevantes para este caso específico:
a. Manejo suave del tejido y cierre sin tensión
54. Un hombre de 19 años salta del tercer piso de su dormitorio en
un aparente intento de suicidio. Lo lleva al departamento de
emergencias inconsciente. Tiene lesiones visibles en la cabeza y las
extremidades inferiores. Tiene un pulso de 110 latidos por minuto,
pero está apneico. ¿Cuál es el mejor manejo de la vía aérea para este
paciente?
a. Traqueotomía
55. Un hombre de 47 años con múltiples problemas médicos y
enfermedad del parénquima pulmonar en etapa terminal puede ser
un trasplante de pulmón. Tiene antecedentes médicos previos de
enfermedad pulmonar obstructiva. Tiene un tío con fibrosis quística.
Su padre tiene enfermedad pulmonar restrictiva y su hermano tiene
SS-Med10

hipertensión pulmonar. ¿Cuál de las siguientes enfermedades


augura la mejor supervivencia después de un trasplante de
pulmón para este paciente?
a. EPOC
56. Una mujer de 41 años se queja de dolores de cabeza
constantes durante los últimos 6 meses. También hemos tenido
quejado de infertilidad femenina y no he tenido tener hijos, a pesar de
haber tenido relaciones sexuales sin protección con su esposo
durante los últimos 15 años. El examen físico revela déficits en los
movimientos extraoculares bilateralmente. El examen de los senos
revela ginecomastia femenina bilateral. ¿Cuál de las siguientes
pruebas de laboratorio serían más útiles para diagnosticar a este
paciente?
a. Prolactina
57. Un niño de 8 años se somete a una ecografía del cuadrante
superior derecho por molestias persistentes en el cuadrante superior
derecho. No tiene antecedentes médicos o quirúrgicos previos. No
tiene alergias conocidas y no toma medicamentos. Su madre tiene
antecedentes de cálculos biliares. Los hallazgos de la ecografía
incluyen una dilatación fusiforme del conducto biliar común. ¿Cuál es
la explicación más probable para estos hallazgos?
a. Quiste de colédeco tipo I
58. Un hombre de 19 años fue pateado en el abdomen durante una
pelea en un bar. Acudió a su médico de atención primaria, quien
ordenó una tomografía computarizada, que reveló un hematoma
esplénico subcapsular. Se le dijo al hombre que restringió la
actividad física. Dos semanas después, se presenta en el
departamento de emergencias debido a un fuerte dolor abdominal.
Se somete a una esplenectomía. Después de la operación, se ordena
un frotis periférico. ¿Qué tipo de célula se puede encontrar en este
paciente?
a. Cuerpos de Howell-Jolly
59. Una mujer de 39 años se queja de dolor epigástrico al comer
durante los últimos 3 o 4 meses. Ella admite una historia de
problemas crónicos de espalda. Ella nota un aumento de peso de 20
lb en los últimos 4 meses. Ella niega el uso de agentes
antiinflamatorios no esteroideos. Ella niega las náuseas y los vómitos.
El examen físico del corazón, los pulmones y el abdomen están
dentro de los límites normales. ¿Cuál es el patógeno más probable
asociado con esta afección?
a. Helicobarte pylori
60. Una mujer de 40 años se le repara una hernia femoral derecha.
Durante el procedimiento, el canal femoral se diseca. Los límites
anatómicos del canal femoral incluyen ¿Cuál de los siguientes?
a. Ligamento de Cooper
SS-Med10

61. Un niño de 12 años es llevado a su médico para la


evaluación de una masa en el cuello. Tiene antecedentes de
sinusitis recurrente e infecciones de amígdalas. El examen físico
revela una masa en el cuello en la línea media que mide 1,5 cm que se
mueve al tragar. No hay evidencia de linfadenopatía. ¿Cuál es el
diagnóstico más probable?
a. Quiste de conductor tirogloso
62. Paciente A de 62 años asintomático, se encuentra en el examen
físico una masa pulsátil en el abdomen, entre los xifoides y el
ombligo. Paciente B de 62 años tiene macho epigástrico vago y mal
descrito en la parte superior de la espalda. Se encuentra en el
examen físico que tiene una masa pulsátil de 6 cm en el abdomen,
entre los xifoides y el ombligo. La masa es sensible a la palpación.
Paciente C de 68 años es llevado al servicio de urgencias con dolor
de espalda insoportable que comenzó repentinamente hace 45
minutos. Está diaforético y tiene una PA sistólica 90. Hay una masa
pulsátil de 8 cm palpable en el abdomen, arriba del ombligo.
a. Paciente A necesita una reparación quirúrgica electiva, pero
debido a nuestras decisiones se basan en el tamaño de la
aneurisma, necesita una medición más precisa, por lo tanto
está indicada una angio tomografía computarizada. Paciente B,
tiene una aneurisma aórtica abdominal que está empezando a
filtrarse, debe consultar inmediatamente una cirugía vascular
ya que es necesaria una reparación quirúrgica. Paciente C, el
aneurisma se está rompiendo en este momento por lo tanto
necesita una cirugía inmediata sin mayor demora.
63. Una mujer de 45 años se queja de diarrea crónica y sudoración.
Se realiza una colonoscopia y se realiza una biopsia de una lesión en
su íleon. El informe de patología muestra que el tumor está
compuesto de células neuroendocrinas. ¿Cuál es el tratamiento
médico para esta afección?
a. Octeotrida
64. Un paciente se presenta en el departamento de emergencias
luego de ser cortado con un cuchillo en la pierna izquierda. La herida
parece limpia y los bordes están bien opuestos. ¿Cuál de los
siguientes es el método más simple de cierre de heridas para esta
lesión?
a. Cierre primario
65. Una mujer de 62 años se presenta con su médico de atención
primaria. Ella también se queja de hemoptisis. La historia social
revela que es fumadora de 55 paquetes al año. Ella es una alcohólica
en recuperación. El examen físico revela sibilancias bilaterales. Los
exámenes cardíacos, pulmonares y abdominales no son notables. Los
valores de laboratorio revelan calcio sérico de 13 mg / dL. La
electroforesis de proteínas séricas no muestra picos anormales.
¿Cuál es el diagnóstico más probable?
SS-Med10

a. Síndrome de GoodPasteur
66. Un hombre de 47 años está involucrado en un accidente
automovilístico. Sufre fracturas de costillas 9, 10 y 11 en el lado
izquierdo. Él es hemodinámicamente inestable y tiene una presión
arterial de 90/50 mm Hg y un pulso de 120 latidos / minuto a pesar de
la transfusión de 3 U de glóbulos rojos empacados. Lo lleva al
quirófano para una laparotomía exploratoria. Se identifica y se extrae
un bazo roto. Al buscar un bazo accesorio potencial, ¿cuál es la
ubicación más probable para encontrarlo?
a. Hilio esplénico
67. Un jugador de tenis masculino de 20 años se estrella contra
una cerca de tratar de perseguir una pelota que podría tratar de
alcanzar durante un partido importante. Su rodilla derecha sufre la
peor parte de la lesión. El examen físico revela edema y disminución
del rango de movimiento de la rodilla en flexión y extensión. La
resonancia magnética (MRI) se realiza y revela la dislocación de la
articulación. No se palpa el pulso detrás de la articulación de la
rodilla. ¿Cuál es la explicación más probable para este hallazgo?
a. Espasmo de la arteria poplítea
68. Un hombre de 28 años con antecedentes de dolor abdominal
recurrente y diarrea sanguinolenta se presenta a su médico
quejándose de un dolor rectal significativo con las deposiciones. Ha
perdido 15 libras en los últimos 3 meses. El examen físico revela
dolor en el cuadrante inferior derecho e izquierdo hasta la palpación.
Los valores de laboratorio revelan un hematocrito del 28% y una
velocidad de sedimentación globular elevada. La colonoscopia
realizada en este paciente probablemente revelaría:
a. Mucosa colónica y recto engrosada fiable
69. Una mujer de 25 años encontró un bulto en el seno derecho en
el autoexamen. Ella no tiene antecedentes familiares de cáncer de
mamá. El bulto es libremente móvil y está bien circunscrito. ¿Cuál es
la mejor opción para evaluar una masa mamaria en una mujer joven?
a. USG
70. Un hombre de 50 años tiene diarrea después de una resección
intestinal sin complicaciones. La elección de fluidos que más se
asemeja a sus requerimientos diarios es
a. Solución de L/R
71. Un hombre de 41 años con una larga historia de cálculos
renales e hipercalcemia tiene un adenoma de la glándula paratiroidea
superior derecha. Él va a someterse a una escisión quirúrgica de
esta lesión. ¿Cuál es el mejor punto de referencia quirúrgico para
esta lesión?
a. Unión del tercio superior y medio de la glandula tiroides
72. Un hombre de 52 años se queja de dolor abdominal crónico. Ha
sido hospitalizado siete veces en los últimos 2 años por ataques
recurrentes de dolor por pancreatitis crónica. Ha sido tratado con
SS-Med10

analgésicos y una pancreatectomía distal parcial. Su dolor aún


persiste. ¿Cuál es el siguiente paso en el tratamiento de este
paciente?
a. Esplacniceptomía
73. Una niña de 6 años se aplastó la mano, el antebrazo y la parte
inferior del brazo en un accidente automovilístico. Toda la extremidad
superior se ve magullada y maltratada, aunque los pulsos son
normales y los huesos no están rotos. Usted debe tomar en el manejo
de este paciente:
a. Esta es una lesión típica por aplastamiento que puede cursar
con hiperpotasemia, mioglbinemia y mioglobinuria que lleva
una insuficiencia renal aguda, gran inflamación y edema que
puede conducir a un síndrome compartamental. El manejo
debe incluir liquidos IV abundantes, diuréticos osmóticos
(manitol) y la alcalinización de la orina para proteger al riñon y
una fasciotomía.
74. Una mujer de 38 años se presenta a su médico de atención
primaria para evaluar un dolor vago intermitente en el cuadrante
superior derecho. Tiene antecedentes de hipotiroidismo e
hipertensión. Sus medicamentos actuales incluyen el reemplazo
sintético de la hormona tiroidea y un bloqueador de los canales de
calcio. El examen físico revela dolor leve en el cuadrante superior
derecho hasta la palpación profunda. La ecografía revela un cálculo
biliar de 3 cm. ¿Cuál es el tipo de piedra más probable que esté
presente en este paciente?
a. Calculo de colesterol tipo II
75. Un hombre de 74 años de edad tiene una aparición repentina
de dolor en el pecho extremadamente intenso y lacerante que se
irradia hacia la parte posterior y migra hacia abajo poco después de
su aparición. Su presión arterial es de 220/110 mm Hg, y tiene pulsos
diseñados en las extremidades superiores y un mediastino ancho en
la radiografía de tórax. El ECG y las enzimas cardíacas son negativas
para el MI.
a. Esto es una disección de la aorta torácica. La angiotomografía
computarizada es el mejor estudio para confirmar el
diagnostico de una manera no invasiva. Si la disección se
encuentra en la aorta ascendente (Stanford A), se debe
realizar una cirugía de emergencia. Si está en la aorta
descendente (Stanford B), la opción preferible es la terapia
intensiva en la UCI para el manejo de la hipertensión.
76. Una mujer de 46 años presenta a su médico antecedentes de
disfagia progresiva. Tiene antecedentes de una pérdida de peso de
15 libras en los últimos 6 meses. El examen físico del cuello, el
corazón, los pulmones y el abdomen no son contributivos. Las
pruebas de laboratorio revelan un hematocrito del 33% y un volumen
SS-Med10

corpuscular medio de 70. La endoscopia gastrointestinal


superior revela una red esofágica. ¿Cuál es el diagnóstico más
probable?
a. Síndrome de Plummer-Vinsson
77. Una mujer de 46 años con enfermedad renal poliquística está
recibiendo un trasplante de riñón. A los pocos minutos de las
anastomosis de la arteria y la vena renales a la respectiva arteria y la
vena ilíaca externa, el riñón se recupera rápidamente una coloración
rosa y una turgencia normal del tejido y comienza una excretar orina.
El paciente es dado de alta del hospital y es visto a 1 mes de
seguimiento. La creatinina sérica es de 4.2 mg / dL. La producción de
orina es de 20 ml / hora. El examen físico del corazón, los pulmones y
el abdomen están dentro de los límites normales. El riñón
trasplantado es palpable en la fosa ilíaca derecha. La biopsia
posterior del trasplante muestra inflamación y edema extenso. ¿Cuál
es la explicación más probable para estos hallazgos?
a. Rechazo agudo de trasplante
78. Paciente es un hombre rico y jubilado que tiene claudicación
cuando camina más de 15 cuadras. Paciente B es un cartero de 56
años de edad que describe un dolor severo en la pantorrilla derecha
cuando camina 2 o 3 cuadras. El dolor se alivia descansando 10 o 15
minutos, pero se repite si vuelve a caminar la misma distancia. No
puede hacer su trabajo de esta manera, y aún no califica para la
jubilación, por lo que está ansioso por resolver este problema. No
fuma Paciente C te consulta porque "no puede dormir". Al
preguntarle, resulta que tiene dolor en la pantorrilla derecha, lo que
le impide conciliar el sueño. Relata que el dolor desaparece si se
sienta al lado de la cama y cuelga la pierna. Su esposa agrega que lo
ve hacer eso todas las noches, y se ha dado cuenta de que la pierna,
que estaba muy pálida cuando estaba acostado, se vuelve de un
color púrpura oscuro varios minutos después de sentarse. En el
examen físico, la piel de esa pierna es brillante, no tiene vello y no
tiene pulsos periféricos palpables. ¿Cuál sería su recomendación
para estos 3 pacientes?
a. Paciente A si fuma, debe dejarlo, y se beneficia con un
programa de ejercicios y el uso de cilostazol. Paciente B,
necesita ayuda, la secuencia de estudios seria: ABI (índice
tobillo-braquial), doppler dúplex arterial, si tiene un gradiente
significativo entonces angiografía por TC o Angiografia por
resonancia magnética, seguido de cirugía de derivación vs
angioplastia y colocación de stens. Paciente C, tiene dolor en
reposo y necesita urgente los estudios para ver si la cirugía
vascular podría ayudarlo
79. Un niño de 5 años es llevado al departamento de emergencias
después de ingerir un limpiador de drenaje líquido. El niño se quedó
sin vigilancia mientras su niñera estaba hablando por teléfono. El
SS-Med10

niño está ronco y tiene un evidente estridor. ¿Cuál es el


tratamiento inicial más adecuado para este paciente?
a. Traqueotomía
80. Una mujer de raza asiática de 52 años tiene pigmentación
melanótica de la mucosa bucal, los labios y los dedos. La
colonoscopia revela hamartomas en todo el tracto gastrointestinal.
Los pólipos fueron extirpados debido a su mayor riesgo de cáncer.
¿Qué otro cáncer está asociado con esta afección?
a. Cáncer de Ovario
81. Un trabajador de la construcción de 37 años sufrió una lesión
por aplastamiento en el muslo derecho después de que una grúa
cayó sobre su pierna en el lugar de trabajo. Lo lleva al departamento
de emergencias para su evaluación. Tiene dolor significativo en la
pierna derecha y dolor con estiramiento pasivo. La pierna está tensa
a la palpación. ¿Cuál es la medición de presión intracompartamental
más probable de la pierna derecha de este paciente?
a. 35 mmHG
82. Un hombre de 57 años es llevado al departamento de
emergencias quejándose de disnea y dolor en el pecho. También
admite una pérdida de peso de 20 libras. Se queja de fiebres,
escalofríos y sudores nocturnos. El examen físico revela adenopatía
supraclavicular. El examen de tórax revela ruidos cardíacos
distantes. Los estudios de laboratorio revelan un recuento de
glóbulos blancos de 170,000 / mL. La radiografía de tórax y la
ecocardiografía revelan un derrame pericárdico. ¿Cuál es la
explicación más probable de estos hallazgos?
a. Linfoma
83. Un trabajador de la construcción masculina de 44 años se
somete a una reparación de hernia inguinal derecha. El
procedimiento quirúrgico sin incidentes. No tiene antecedentes
médicos o quirúrgicos previos. Regresa para el seguimiento en el día
3 postoperatorio para un chequeo de heridas. La herida está limpia,
seca e intacta. ¿Cuál es el período de convalecencia óptimo
requerido antes de regresar a trabajar para este paciente?
a. Desconocido
84. Un hombre de 41 años sufre una amputación traumática de tres
dedos en una cortadora de carne. No tiene antecedentes médicos o
quirúrgicos previos. ¿Cuál de las siguientes modalidades debe tener
para transportar los dedos amputados con el paciente?
a. Colocarlos en una bolsa de plástico limpia en un refrigerador
lleno de hielo triturado y agua
85. Una mujer de 47 años con pancreatitis biliar es hospitalizada.
Tiene antecedentes de hipertensión e hipercolesterolemia. La
colecistectomía temprana está indicada para prevenir las
complicaciones de las siguientes complicaciones?
a. Pancreatitis recurrentes
SS-Med10

86. Con relación a la fisiopatología de la isquemia arterial


obstructiva aguda:
a. Los radicales libres derivados del oxigeno son nocivos para
las células al empeorar el edema celular
87. Una mujer de 27 años con 12 horas después de un trasplante
de páncreas cadavérico y actualmente en la unidad de cuidados
intensivos quirúrgicos. Tiene antecedentes médicos de diabetes
insulino dependiente desde los 5 años. Sus signos vitales son
normales. El pecho está claro para la
auscultación, y el examen cardíaco revela una
frecuencia y un ritmo regular. El apósito de la
herida está limpio, seco e intacto. ¿Cuál de los
siguientes es el mejor método para monitorear
el páncreas trasplantado?
a. Nivel de amilasa urinaria
88. Un paciente ha sido intervenido por una
peritonitis difusa secundaria a una perforación de un asa yeyunal por
una espina de pescado. Se retiraron los drenajes a los siete días. A
los quince días de la intervención el paciente tiene el abdomen
blando y tolera bien la alimentación oral, pero aparece un cuadro
febril en picos y se realiza un TAC que muestra la imagen siguiente:
PDF 1
a. Se trata de un absceso subfrénico que debe ser drenado
percutáneamente con control radiológico o ecográfico
89. ¿Cuál de las siguientes técnicas se utiliza mejor para definir
una glándula paratiroidea agrandada?
a. USG
90. Una niña de 2 años es llevada al departamento de emergencias
debido a varios episodios de sangrado rectal. Una exploración de
perfusión de tecnecio-99m revela una bolsa ileal de 3 cm ubicada a
50 cm de la válvula ileocecal. ¿Cuál de los siguientes tipos de tejido
ectópico contiene esta estructura?
a. Gástrico
91. Una mujer negra de 29 años se presenta a su médico de
atención primaria debido a un crecimiento en su oreja izquierda, que
ocurrió después de que le perforaron la oreja por primera vez hace
una semana. Ella notó que su oído desarrolló rápidamente un
crecimiento en él con bastante rapidez. Nunca se había perforado la
oreja antes. ¿Cuál es la explicación más probable para estos
hallazgos?
a. Queloide
92. Un hombre de 56 años se queja de tos recurrente y hemoptisis.
Tiene antecedentes de neumonías recurrentes. No fuma y no tiene
riesgo laboral de enfermedad pulmonar. El examen físico revela una
disminución de los sonidos respiratorios en el lóbulo superior
derecho. La radiografía de tórax revela una pequeña masa en el
SS-Med10

cuadrante superior derecho. La broncoscopia revela un angioma.


¿Cuál es el tratamiento más adecuado para este paciente?
a. Espera vigilante
93. Un hombre de 45 años sufre una quemadura del 60% de su
cuerpo, requiere intubación y ventilación mecánica, sedación y
analgesia. El intensivista le explica a la familia que la probabilidad de
sobrevida dependerá de la respuesta metabólica a la agresión, la
cual presenta:
a. Mayor excreción de N2 en orina
94. Una mujer de 53 años se presenta a su médico de atención
primaria con un historial de dolor de cuello de 12 meses. Ella se queja
de un aumento de peso de 15 libras y malestar generalizado. Tiene
antecedentes médicos de hipertensión y diabetes mellitus. Sus
medicamentos actuales incluyen un hipoglucemiante oral. El examen
físico revela sensibilidad al largo del curso de la glándula tiroides sin
evidencia de una masa discreta. ¿Cuál es el diagnóstico más
probable?
a. Tiroiditis de Hashimoto
95. Un médico desea administrar un anestésico local por vía
subcutánea a un paciente con una lesión sospechosa en el frente. La
lesión mide 1 cm y el cirujano planea una incisión elíptica para
extirparla. ¿Cuál de las siguientes capas epidérmicas de la piel
penetrará primero el médico con la aguja anestésica local?
a. Estrato córneo
96. Paciente de sexo masculino de 60 años de edad que se
presenta con edema de miembro inferior izquierdo de tres semanas
de evolución, sin otros síntomas asociados :.
a. Considere que uno de sus diagnosticos diferenciales dese ser
una trombosis venosa y la realización de un examen físico
exhaustivo incluyendo tacto rectal
97. Un hombre sano de 35 años se le diagnostica una hernia
inguinal. No tiene antecedentes de sangrado anormal. ¿Cuál de las
siguientes pruebas es absolutamente necesario antes de llevarlo al
quirófano?
a. Ninguna de las anteriores
98. Un hombre de 34 años con un nódulo tiroideo se somete a una
exploración del cuello. Durante el procedimiento, es posible que se
someta a una tiroidectomía. ¿Cuál de las siguientes afirmaciones
sobre el nervio laríngeo superior y la inervación de la glándula
tiroides es correcta?
a. La lesión del nervio provoca inclinación de las cuerdas vocales
durante la fonación
99. Si un paciente tiene una presión de 50 mmHg en la arteria tibial
posterior derecha y 140 en la arteria braquial, dolor de reposo y una
úlcera de 4 meses que no cicatriza, el diagnóstico más probable es:
a. Isquemia que amenaza la viabilidad de la extremidad
SS-Med10

100. Un hombre de 41 años tiene reflujo gastroesofágico


crónico. Actualmente se maneja con un bloqueador H2. El
examen físico del corazón, los pulmones y el abdomen están
dentro de los límites normales. ¿Cuál de los siguientes factores sería
el menos protector del esófago en términos de la exposición continua
inducida por esta afección?
a. Ligamento arqueado
Bien Dañao con mi Mono

EXAMEN SEMESTRAL DE CIRUGÍA MED-9

1. Un hombre de 71 años con aparición repentina de dolor abdominal y de espalda intenso es


llevado al departamento de emergencias para su evaluación. Tiene antecedentes de
hipertensión. Pesa 300 lb. Tiene un historial de fumar de 45 paquetes al año. El examen físico
revela una masa abdominal pulsátil. Ambas extremidades inferiores revelan palidez con pulsos
de pedal disminuidos. ¿Cuál es la causa más probable de la condición de este paciente?
A. Aterosclerosis
B. Síndrome de Marfan
C. Infección meningocócica
D. Sífilis
E. Trauma

2. Un hombre de 78 años es llevado al departamento de emergencias con un historial de dolor


abdominal, diarrea y vómitos de 12 horas. Tiene antecedentes de fibrilación auricular y fue
tratado previamente por insuficiencia cardíaca congestiva con digoxina. El examen físico revela
un abdomen distendido con protección significativa. El examen rectal revela heces guayaco
positivas en la bóveda. El recuento de glóbulos blancos es de 24,000 / mL. La radiografía
abdominal revela edema de la pared intestinal. ¿Cuál es el tratamiento más apropiado para este
paciente?
A. Embolización angiográfica.
B. Terapia antibiótica con ampicilina y gentamicina.
C. Terapia antibiótica con gentamicina
D. Heparinización seguida de warfarina oral.
E. Exploración quirúrgica.

3. Un jugador de tenis masculino de 20 años se estrella contra una cerca tratando de perseguir
una pelota que pensó que podría alcanzar durante un partido importante. Su rodilla derecha sufre
la peor parte de la lesión. El examen físico revela edema y disminución del rango de movimiento
de la rodilla en flexión y extensión. La resonancia magnética (MRI) se realiza y revela la
dislocación de la articulación. No se palpa el pulso detrás de la articulación de la rodilla. ¿Cuál
es la explicación más probable para este hallazgo?
A. Ruptura de la arteria tibial anterior
B. Hematoma de la arteria peronea.
C. Espasmo de la arteria poplítea
D. Hematoma de la arteria tibial posterior.
E. Espasmo superficial de la arteria femoral

4. Una mujer de 25 años encontró un bulto en su seno derecho en el autoexamen. Ella no tiene
antecedentes familiares de cáncer de mama. El bulto es libremente móvil y está bien circunscrito.
¿Cuál es la mejor opción para evaluar una masa mamaria en una mujer joven?
A. Biopsia
B. Mamografía
C. Pruebas para el gen del cáncer de seno (BRCA)
D. Ultrasonido
E. Espera vigilante
5. Una mujer de 19 años comenzó a amamantar por primera vez. Al principio, era difícil para su
bebé alimentarse. Ahora, sus senos están rojos, cálidos y doloridos. Ella ha seguido
amamantando, a pesar del dolor; sin embargo, recientemente comenzó a usar un extractor de
leche en lugar de amamantar. Ella se inicia en un curso de antibióticos orales. ¿Qué afección
tiene este paciente en riesgo de desarrollar?
A. Absceso mamario
B. Enfermedad fibroquística
C. Cáncer de mama inflamatorio
D. Prolactinoma
E. Tuberculosis

6. Una mujer premenopáusica de 31 años con una masa mamaria izquierda se somete a una
mastectomía radical izquierda modificada. La patología revela carcinoma ductal infiltrante de 3
cm de tamaño con ganglios linfáticos negativos. El estado del receptor de estrógeno es negativo.
¿Cuál es la terapia adyuvante más adecuada para este paciente?
A. Quimioterapia (multiagente)
B. Radioterapia de haz externo
C. Terapia de ultrasonido enfocada en alta energía
D. Tamoxifeno
E. Espera vigilante

7. Una mujer de 31 años se queja de una historia de 6 meses de diarrea con sangre, dolor
abdominal y fiebres intermitentes. Tiene antecedentes de síndrome del intestino irritable, pero ha
empeorado sus síntomas durante el período de tiempo anterior. Su historial médico pasado es
irrelevante. El examen físico revela distensión abdominal.
Los sonidos intestinales están presentes en todos los cuadrantes. El examen rectal revela
múltiples fisuras anales. ¿Cuál es la prueba de diagnóstico más adecuada para este paciente?
A. Anoscopia
B. Colonoscopia
C. Sigmoidoscopia flexible
D. Sigmoidoscopia rígida
E. No se requieren más pruebas de diagnóstico para este paciente.

8. Una mujer de 71 años se presenta a su médico de atención primaria quejándose de sangrado


rectal. Tenía algunos calambres abdominales leves en el lado izquierdo que disminuyeron en
pocos minutos. Ella nunca ha tenido un episodio previo de sangrado rectal. El examen físico
revela dolor abdominal leve en el cuadrante inferior izquierdo sin evidencia de sensibilidad de
protección o rebote. El examen rectal revela que no hay sangre fresca en la bóveda rectal. La
colonoscopia revela varias bolsas de la pared del colon sigmoidea sin evidencia de sangrado o
perforación. El resto de la colonoscopia está dentro de los límites normales. El recuento de
glóbulos blancos es normal. ¿Cuál es el tratamiento más apropiado para este paciente?
A. Terapia antibiótica con ampicilina y gentamicina.
B. Hemicolectomía izquierda
C. Hemicolectomía derecha
D. Colectomía subtotal
E. Espera vigilante
9. Un hombre de 85 años es llevado al departamento de emergencias por dolor abdominal agudo
y distensión abdominal progresiva. Es residente de un hogar de ancianos local. No ha estado
comiendo debido a las náuseas progresivas. Las radiografías abdominales revelan un colon
masivamente sigmoide. ¿Cuál es el tratamiento inicial para este paciente?
A. Enema de gastrografina
B. Dieta rica en fibra
C. Lactulosa
D. Descompresión del tubo rectal
E. Resección quirúrgica

10. Una mujer de 41 años se queja de dolores de cabeza constantes durante los últimos 6 meses.
También se ha quejado de infertilidad femenina y no ha podido tener hijos, a pesar de haber
tenido relaciones sexuales sin protección con su esposo durante los últimos 15 años. El examen
físico revela déficits en los movimientos extraoculares bilateralmente. El examen de los senos
revela ginecomastia femenina bilateral. ¿Cuál de las siguientes pruebas de laboratorio sería más
útil para diagnosticar a este paciente?
A. Ferritina
B. Hemoglobina
C. Hematocrito
D. Hierro
E. Prolactina

11. Una mujer de 41 años con enfermedad de Crohn se sometió a múltiples procedimientos
quirúrgicos. Recientemente se sometió a una ileostomía, pero aún tiene evidencia de alguna
enfermedad yeyunal distal. Sus medicamentos actuales incluyen prednisona y ácido
aminosalicílico. ¿Cuáles de los siguientes efectos de la terapia prolongada con glucocorticoides
son posibles para este paciente?
A. Producción de anticuerpos
B. Formación de colágeno
C. Disfunción de fibroblastos
D. Migración celular inflamatoria
E. Deterioro de heridas

12. Un hombre obeso de 49 años se presenta a su médico de atención primaria para un examen
de seguimiento. Tiene antecedentes de diabetes mellitus no controlada y trastorno bipolar. Sus
medicamentos actuales incluyen litio y leche de magnesio. El examen físico del corazón, los
pulmones y el abdomen están dentro de los límites normales. Los estudios de laboratorio revelan
calcio sérico de 14 mg / dL. ¿Cuál es la explicación más probable para estos hallazgos?
A. Indiscreción dietética
B. Sobredosis de medicación
C. Síndrome de leche y álcali
D. Adenoma paratiroideo
E. Hiperplasia paratiroidea

13. Un hombre de 41 años tiene reflujo gastroesofágico crónico. Actualmente se maneja con un
bloqueador H2. El examen físico del corazón, los pulmones y el abdomen están dentro de los
límites normales. ¿Cuál de los siguientes factores sería el menos protector del esófago en
términos de la exposición continúa inducida por esta afección?
A. Ligamento arqueado
B. Capacidad de vaciado gástrico
C. Efecto gravitacional
D. Productos secretores de glándulas salivales
E. Ondas peristálticas secundarias

14. Una mujer de 40 años se queja de dolor en el pecho y disfagia a los sólidos. Ella se presenta
a un especialista para su evaluación. Se realizan estudios manométricos esofágicos que revelan
contracciones de gran amplitud y eventual relajación normal del esfínter esofágico inferior. La
ingestión de bario es normal. ¿Cuál es el diagnóstico más probable?
A. Espasmo muscular cricofaríngeo
B. Espasmo esofágico difuso
C. Esclerodermia
D. Tuberculosis
E. Trastorno de deglución psicógena

15. Un niño de 5 años es llevado al departamento de emergencias después de ingerir un


limpiador de drenaje líquido. El niño quedó desatendido mientras su niñera estaba hablando por
teléfono. El niño es ronco y tiene un evidente estridor. ¿Cuál es el tratamiento inicial más
apropiado para este paciente?
A. Antibióticos
B. Corticosteroides
C. Inducción de vómitos con ipecacuana
D. Colocación de sonda nasogástrica y lavado
E. Traqueostomía

16. Un hombre de 76 años con antecedentes de dolor vago en el cuadrante superior derecho,
pérdida de peso de 25 lb y anorexia se presenta a su médico de atención primaria para su
evaluación. El examen físico revela ictericia escleral. El examen abdominal revela una masa en
el cuadrante superior derecho. Riñón, uréter y vejiga (KUB) revela una calcificación circular en el
cuadrante superior derecho. La laparotomía exploratoria revela un proceso neoplásico que
involucra la vesícula biliar y el hígado. ¿Cuál es la patología más probable que causa esta
afección?
A. Adenocarcinoma
B. Sarcoma
C. Carcinoma de células escamosas
D. Carcinoma de células de transición
E. Granuloma tuberculoso

17. Una mujer de 38 años se presenta a su médico de atención primaria para evaluar el dolor
vago intermitente en el cuadrante superior derecho. Tiene antecedentes de hipotiroidismo e
hipertensión. Sus medicamentos actuales incluyen el reemplazo sintético de la hormona tiroidea
y un bloqueador de los canales de calcio. El examen físico revela dolor leve en el cuadrante
superior derecho hasta la palpación profunda. La ecografía revela un cálculo biliar de 3 cm. ¿Cuál
es el tipo de piedra más probable que esté presente en este paciente?
A. Cálculo biliar negro
B. Cálculos biliares marrones
C. Cálculos biliares de oxalato de calcio
D. Cálculo de colesterol tipo I
E. Cálculo de colesterol tipo II

18. Una mujer de 46 años se presenta al servicio de urgencias quejándose de dolor en el


cuadrante superior derecho y fiebre a 39 ° C. El examen físico revela ictericia escleral y dolor
significativo en el cuadrante superior derecho a la palpación. Los signos peritoneales están
ausentes. Los ruidos intestinales están presentes. ¿Cuál de los siguientes debe incluirse en el
tratamiento inicial de este paciente?
A. Antibióticos
B. Coledocoyeyunostomía
C. Descompresión con tubo en T
D. Esfinterotomía endoscópica
E. Drenaje transhepático percutáneo

19. Un niño de 17 años es llevado al departamento de emergencias después de sufrir dolor en


el pecho y disnea durante un juego de baloncesto. El examen físico revela un soplo crescendo-
decrescendo sistólico, que se escucha mejor en el segundo espacio intercostal derecho. El soplo
se irradia a la arteria carótida derecha. La radiografía de tórax revela un tamaño cardíaco normal.
¿Cuál de los siguientes hallazgos se esperaría ver en un electrocardiograma en este paciente?
A. Inversión de ondas T en derivaciones V1 – V4
B. Hipertrofia del ventrículo izquierdo
C. Bloque de rama derecha
D. Hipertrofia ventricular derecha
E. Hipertrofia auricular derecha

20. Un hombre de 72 años se derrumba mientras camina en un centro comercial. No tiene pulso
y está apneico. No hay antecedentes de trauma. La reanimación cardiopulmonar se inicia hasta
que llega un escuadrón de rescate. Se inicia el protocolo avanzado de soporte vital cardíaco. Es
declarado muerto 40 minutos después. La autopsia revela necrosis miocárdica con ruptura del
ventrículo izquierdo. ¿Cuál de los siguientes es el factor de riesgo más probable que contribuyó
a su muerte?
A. Antecedentes familiares de diabetes mellitus.
B. Hipotensión
C. Obesidad
D. Estilo de vida sedentario
E. Trauma

21. Un hombre de 57 años es llevado al departamento de emergencias quejándose de disnea y


dolor en el pecho. También admite una pérdida de peso de 20 libras. Se queja de fiebres,
escalofríos y sudores nocturnos. El examen físico revela adenopatía supraclavicular. El examen
de tórax revela ruidos cardíacos distantes. Los estudios de laboratorio revelan un recuento de
glóbulos blancos de 170,000 / mL. La radiografía de tórax y la ecocardiografía revelan un derrame
pericárdico. ¿Cuál es la explicación más probable de estos hallazgos?
A. Mixoma auricular
B. Fibrilación auricular
C. Linfoma
D. Carcinoma colorrectal metastásico
E. Pericarditis

22. Un varón recién nacido tiene una abertura de la pared abdominal en el ombligo. No tiene otro
historial médico o quirúrgico previo. El historial de nacimientos no fue notable. Durante el resto
del examen físico y las pruebas de diagnóstico, ¿cuál de los siguientes hallazgos es más
probable?
A. Labio leporino
B. Paladar hendido
C. Hernia de diafragma
D. Pericardio
E. Vejiga urinaria en retroperitoneo

23. Un trabajador de la construcción masculino de 44 años se somete a una reparación de hernia


inguinal derecha. El procedimiento quirúrgico es sin incidentes. No tiene antecedentes médicos
o quirúrgicos previos. Regresa para el seguimiento en el día 3 postoperatorio para un chequeo
de heridas. La herida está limpia, seca e intacta. ¿Cuál es el período de convalecencia óptimo
requerido antes de regresar a trabajar para este paciente?
A. 1 semana
B. 4 semanas
C. 6 a 8 semanas
D. 12 semanas
E. Desconocido

24. Una mujer de 40 años se repara una hernia femoral derecha. Durante el procedimiento, se
diseca el canal femoral. Los límites anatómicos del canal femoral incluyen ¿cuál de los
siguientes?
A. Ligamento de Cooper
B. Ligamento inguinal
C. Espina ciática
D. Ligamento lacunar
E. Nervio (femoral)

25.

26.

27.

28. Un hombre de 78 años se presenta en el departamento de emergencias para evaluar el dolor


progresivo en el cuadrante superior derecho, náuseas, vómitos y una pérdida de peso de 30
libras en los últimos 3 meses. Tiene antecedentes médicos previos de colelitiasis, diabetes
mellitus, hipertensión y demencia. El examen físico revela ictericia escleral bilateralmente. El
examen abdominal revela sensibilidad en el cuadrante superior derecho y una masa palpable.
Los signos peritoneales están ausentes. La tomografía computarizada revela ganglios linfáticos
pancreáticos, duodenales y coledocales. Hay un engrosamiento asimétrico de la vesícula biliar.
¿Qué es lo más probable
hallazgo patológico en laparotomía exploratoria y biopsia?
A. Adenocarcinoma
B. Fibroma
C. Lipoma
D. Mixoma
E. Mioma

29. Un niño de 8 años se somete a una ecografía del cuadrante superior derecho por molestias
persistentes en el cuadrante superior derecho. No tiene antecedentes médicos o quirúrgicos
previos. No tiene alergias conocidas y no toma medicamentos. Su madre tiene antecedentes de
cálculos biliares. Los hallazgos de la ecografía incluyen una dilatación fusiforme del conducto
biliar común. ¿Cuál es la explicación más probable para estos hallazgos?
A. Quiste de colédoco tipo I
B. Quiste de colédoco tipo II
C. Quiste de colédoco tipo III
D. Quiste de colédoco tipo IV
E. Quiste de colédoco tipo V

30. Un agresor apuñaló a un hombre de 18 años en su abdomen varias veces durante un


altercado que implicaba la venta de drogas ilícitas. Lo llevan al departamento de emergencias
para su evaluación. Tiene cuatro heridas de arma blanca en el abdomen: tres están en el
cuadrante superior derecho y una está en el cuadrante inferior izquierdo. El examen físico del
abdomen revela protección y sensibilidad de rebote. El paciente es llevado a cirugía para una
laparotomía exploratoria. Se encuentra una lesión penetrante en la vesícula biliar. ¿Cuál de las
siguientes vísceras asociadas es probable que se lesione?
A. Aorta
B. Colon
C. Riñón
D. Hígado
E. Vejiga urinaria

31. Una mujer de 62 años presenta una tos a su médico de atención primaria. Ella también se
queja de hemoptisis. La historia social revela una historia de fumar de 55 paquetes al año. Ella
es una alcohólica en recuperación. El examen físico revela sibilancias bilaterales. Los exámenes
cardíacos, pulmonares y abdominales no son notables. Los valores de laboratorio revelan calcio
sérico de 13 mg / dL. La electroforesis de proteínas séricas no muestra picos anormales. ¿Cuál
es el diagnóstico más probable?
A. Síndrome de Goodpasture
B. Mieloma
C. Adenoma renal
D. Carcinoma de células pequeñas de pulmón
E. Carcinoma de células escamosas de pulmón

32.
33.

34. Un estudiante universitario masculino de 21 años se presenta en la clínica ambulatoria para


un examen de rutina al comienzo del semestre de otoño. Tiene antecedentes de síndrome del
intestino irritable. El examen físico del corazón, los pulmones y el abdomen no son notables. El
examen genitourinario revela que los testículos descienden bilateralmente. Un varicocele
izquierdo de grado 1 está presente. No hay masas testiculares. El pene no está circuncidado y
el prepucio no puede retraerse detrás del glande. ¿Cuál es el diagnóstico más probable?
A. Balanitis
B. Hipospadias
C. Epispadias
D. Parafimosis
E. Fimosis

35.

36.

37.

38. Un estudiante universitario de 19 años conduce bajo la influencia del alcohol, a pesar de las
recomendaciones de amigos de no conducir. Otro conductor la golpea. La fuerza del impacto
hace que golpee el área temporal de su cráneo contra la ventana. Desarrolla un leve dolor de
cabeza pero no pierde el conocimiento. Varias horas después, desarrolla un fuerte dolor de
cabeza con náuseas y vómitos. ¿Cuál es el diagnóstico más probable?
A. Infección bacteriana
B. Aneurisma de bayes
C. Hematoma epidural
D. Hematoma subaracnoideo
E. Hemorragia subdural

39. Un paciente ha sido intervenido por una peritonitis difusa


secundaria a una perforación de un asa yeyunal por una
espina de pescado. Se retiraron los drenajes a los siete días.
A los quince días de la intervención el paciente tiene el
abdomen blando y tolera bien la alimentación oral, pero
aparece un cuadro febril en picos y se realiza un TAC que
muestra la imagen siguiente:

A.- Se trata de un absceso subfrénico que debe ser drenado percutáneamente con control
radiológico o ecográfico.
B.- Se debe tratar con amoxicilina y clavulánico siete días y después repetir el TAC.
C.- Se debe hacer una punción para tomar un cultivo y tratar con el antibiótico adecuado durante
siete días.
D.- Lo más probable es que la colección se reabsorba espontáneamente con tratamiento
exclusivamente antiinflamatorio.
E.- Por tener aire en la parte superior de la imagen (ver flecha superior) se debe descartar el
absceso y valorar otro diagnóstico (angioma, metástasis etc.).

40. Un hombre de 47 años con antecedentes de enfermedad pulmonar terminal del pulmón
derecho está programado para un trasplante de pulmón. La función cardíaca preoperatoria es
buena. No tiene antecedentes de defectos congénitos. ¿Cuál de las siguientes es la incisión
quirúrgica más adecuada para este paciente?
A. Chevron abdominal
B. Toracotomía lateral
C. Línea media abdominal
D. Toracotomía anterior transversal
E. Pfannenstiel

41. Un hombre de 47 años con múltiples problemas médicos y enfermedad del parénquima
pulmonar en etapa terminal se somete a un trasplante de pulmón. Tiene antecedentes médicos
previos de enfermedad pulmonar obstructiva. Tiene un tío con fibrosis quística. Su padre tiene
enfermedad pulmonar restrictiva y su hermano tiene hipertensión pulmonar. ¿Cuál de los
siguientes augura la mejor supervivencia después de un trasplante de pulmón para este
paciente?
A. Carcinoma broncogénico
B. Fibrosis quística
C. Enfermedad pulmonar obstructiva
D. Hipertensión pulmonar
E. Enfermedad pulmonar restrictiva

42. Un niño de 4 años está en la lista de espera para un trasplante de hígado. Tiene una
enfermedad hepática en etapa terminal y actualmente está hospitalizado por hemorragia varicosa
esofágica. ¿Cuál es la causa más probable de insuficiencia hepática en este paciente?
A. Atresia biliar
B. Hepatitis A
C. Cirrosis biliar primaria
D. Colangitis esclerosante primaria
E. Tuberculosis

43. Una mujer de 23 años que se queja de heces grasas y olorosas, debilidad generalizada y
pérdida de cabello se presenta a su médico de atención primaria para su evaluación. El examen
físico del corazón, los pulmones y el abdomen no son notables. Ella no tiene protección ni ternura
de rebote. Los sonidos intestinales están presentes en todos los cuadrantes. El examen pélvico
femenino fue diferido a pedido del paciente. ¿Cuál es la explicación más probable de estos
hallazgos?
A. Úlcera gástrica con sangrado.
B. Malabsorción de glucosa
C. Menstruación
D. Insuficiencia pancreática
E. Tumor pituitario
44. Una mujer de 27 años está en estado de 12 horas después del trasplante de páncreas
cadavérico y actualmente en la unidad de cuidados intensivos quirúrgicos. Tiene antecedentes
médicos de diabetes insulinodependiente desde los 5 años. Sus signos vitales son normales. El
pecho es claro para la auscultación, y el examen cardíaco revela una frecuencia regular con un
ritmo regular. El vendaje de limpieza está limpio, seco e intacto. ¿Cuál de los siguientes es el
mejor método para controlar el páncreas trasplantado?
A. Nivel sérico de amilasa
B. Nivel de glucosa en suero
C. Nivel de insulina sérica
D. Ecografía de los vasos pancreáticos.
E. Nivel de amilasa urinaria

45. Un hombre de 44 años con pancreatitis recurrente es llevado al departamento de


emergencias con otro episodio de pancreatitis. ¿Cuál de los siguientes es el factor más
tranquilizador con respecto a la gravedad de su condición?
A. Edad
B. Nivel de glucosa en sangre de 300 mg / dL
C. Nivel de lactato deshidrogenasa de 400 UI / L
D. Calcio sérico de 6 mg / dL
E. Nivel de hematocrito en suero del 29%

46. Un hombre de 41 años con una larga historia de cálculos renales e hipercalcemia tiene un
adenoma de la glándula paratiroidea superior derecha. Va a someterse a una extirpación
quirúrgica de esta lesión. ¿Cuál es el mejor hito quirúrgico para esta lesión?
A. Bifurcación de las arterias carótidas.
B. Seno carotídeo
C. Unión de la arteria tiroidea inferior y nervio laríngeo recurrente
D. Unión del tercio superior y medio de la glándula tiroides.
E. Nervio laríngeo recurrente

47. ¿Cuál de las siguientes técnicas se utiliza mejor para definir una glándula paratiroidea
agrandada?
A. Tomografía computarizada del cuello
B. Doble trazado de imágenes
C. Resonancia magnética del cuello
D. Angiografía tirocervical
E. Ultrasonido

48. Un hombre de 44 años con enfermedad renal en etapa terminal se somete con éxito a un
trasplante renal. Tiene antecedentes médicos previos de hiperparatiroidismo. Seis meses
después del trasplante renal, su calcio sérico sigue siendo 13 mg / dL. ¿Cuáles de los siguientes
hallazgos de laboratorio son posibles en este paciente?
A. Fosfato sérico elevado
B. Elevación sérica de ácido láctico deshidrogenasa
C. Orina elevada en calcio
D. Creatinina en orina elevada
E. Proteína urinaria elevada
49. Un hombre de 46 años se presenta a su médico de atención primaria para la evaluación de
una lesión cutánea. Se queja de hipopigmentación de la piel de la espalda baja. Tiene
antecedentes médicos previos de eccema y carcinoma basocelular. Es un granjero que pasa
mucho tiempo al aire libre. ¿Qué células son responsables de esta afección?
A. Adipocitos
B. Células productoras de queratina
C. Células de Langerhans
D. Melanocitos
E. Células de Merkel

50. Un hombre de 69 años se presenta a su dermatólogo con una lesión presente en la nariz. Es
un jardinero que pasa gran parte de su tiempo al aire libre. Tiene antecedentes médicos previos
de rinitis alérgica, hipertensión y diabetes mellitus. . Sus medicamentos actuales incluyen un
betabloqueante y un hipoglucemiante oral. El examen físico de su nariz revela una lesión papular
brillante y elevada con pequeños vasos sanguíneos. ¿Cuál es el diagnóstico más probable?
A. Carcinoma de células basales
B. Histiocitosis X
C. Melanoma
D. Queratosis seborreica
E. Carcinoma de células escamosas

51. Una mujer negra de 29 años se presenta a su médico de atención primaria debido a un
crecimiento en su oreja izquierda, que ocurrió después de que le perforaron la oreja por primera
vez hace una semana. Ella notó que su oído parecía desarrollar un crecimiento en él con bastante
rapidez. Nunca se había perforado la oreja antes. ¿Cuál es la explicación más probable para
estos hallazgos?
A. Carcinoma de células basales
B. Nevo azul
C. Melanoma juvenil
D. Queloide
E. Molusco contagioso

52. Una mujer asiática estadounidense de 52 años tiene pigmentación melanótica de la mucosa
bucal, los labios y los dedos. La colonoscopia revela hamartomas en todo el tracto
gastrointestinal. Los pólipos se extirparon debido a su mayor riesgo de cáncer. ¿Qué otro cáncer
se asocia con esta afección?
A. Cáncer de cuello uterino
B. Cancer de riñon
C. Cáncer de hígado
D. Cáncer de ovarios
E. Cáncer de páncreas

53. Una mujer de 45 años se queja de diarrea crónica y sudoración. Se realiza una colonoscopia
y se realiza una biopsia de una lesión en su íleon. El informe de patología muestra que el tumor
está compuesto de células neuroendocrinas. ¿Cuál es el tratamiento médico para esta afección?
A. Corticosteroides intravenosos
B. Corticosteroides tópicos
C. Furosemida
D. Octreotida
E. Tetraciclina

54. Un hombre de 18 años es llevado al departamento de emergencias con un dolor abdominal


repentino insoportable localizado en el cuadrante inferior derecho, náuseas y vómitos, fiebre leve
y taquicardia leve. Tiene antecedentes médicos previos de otitis media recurrente. El examen
físico revela marcada sensibilidad de rebote y rebote en el cuadrante inferior derecho. El recuento
de glóbulos blancos en suero es de 18,000 / mL. La radiografía KUB revela gas intestinal en el
intestino delgado y grueso. ¿Cuál es el diagnóstico más probable?
A. Apendicitis
B. Enfermedad de Crohn
C. Diverticulitis
D. Pancreatitis
E. Colitis ulcerosa

55. Una mujer de 39 años se presenta al servicio de urgencias quejándose de dolor abdominal
intenso. Tiene antecedentes de enfermedad ulcerosa péptica. El examen físico revela protección
y ternura de rebote. La llevan a la sala de operaciones para realizar una laparotomía exploratoria.
Durante el procedimiento, el cirujano que abre el ligamento gastrosplénico para alcanzar el saco
menor corta accidentalmente una arteria. ¿Cuál de los siguientes vasos es el más probable?
A. Arteria gastroduodenal
B. Arteria gástrica izquierda
C. Arteria gastroepiploica izquierda
D. Arteria gástrica derecha
E. Arteria esplénica

56. Una mujer de 2 años es llevada al departamento de emergencias debido a varios episodios
de sangrado rectal. Una exploración de perfusión de tecnecio-99m revela una bolsa ileal de 3 cm
ubicada a 50 cm de la válvula ileocecal. ¿Cuál de los siguientes tipos de tejido ectópico contiene
esta estructura?
A. Duodenal
B. Esofágico
C. Gástrico
D. Hepático
E. Yeyunal

57. Una mujer de 39 años se queja de dolor epigástrico al comer en los últimos 3 o 4 meses. Ella
admite una historia de problemas crónicos de espalda. Ella nota un aumento de peso de 20 lb en
los últimos 4 meses. Ella niega el uso de agentes antiinflamatorios no esteroideos. Ella niega las
náuseas y los vómitos. Examen físico del corazón, pulmones,
y el abdomen están dentro de los límites normales. ¿Cuál es el patógeno más probable asociado
con esta afección?
A. Escherichia coli enterohemorrágica
B. Escherichia coli
C. Helicobacter pylori
D. Shigella sonnei
E. Streptococcus pyogenes

58. Un hombre de 59 años resultó herido en un accidente automovilístico. Una tomografía


computarizada abdominal revela una ruptura del bazo. Su presión arterial es de 90/40 mm Hg, y
su pulso es de 140 latidos por minuto. El paciente es llevado a una laparotomía. Se realiza
esplenectomía. ¿Cuál de las siguientes anormalidades de laboratorio es probable después de
este procedimiento?
A. Anemia
B. Basofilia
C. Eosinofilia
D. Trombocitopenia
E. Trombocitosis

59. Un hombre de 19 años fue pateado en el abdomen durante una pelea en un bar. Acudió a su
médico de atención primaria, quien ordenó una tomografía computarizada, que reveló un
hematoma esplénico subcapsular. Se le dijo al hombre que restringiera la actividad física. Dos
semanas después, se presenta en el departamento de emergencias debido a un fuerte dolor
abdominal. Se somete a una esplenectomía. Después de la operación, se ordena un frotis
periférico. ¿Qué tipo de célula se puede encontrar en este paciente?
A. Punteado basófilo
B. Células de la ampolla
C. Cuerpos de Howell-Jolly
D. Glóbulos rojos nucleados.
E. Esferocitos

60. Un hombre afroamericano de 15 años se sometió a una esplenectomía después de sufrir una
herida de cuchillo durante una pelea. Se presenta a su médico de atención primaria para un
examen físico deportivo. Su madre leyó en la Web que tiene un mayor riesgo de infección.
¿Debería recibir una de las siguientes vacunas para prevenir infecciones graves?
A. Rubéola
B. Sarampión
C. Tétanos
D. Vacunas contra organismos encapsulados comunes.
E. Varicela

61. Una mujer de 53 años se presenta a su médico de atención primaria con un historial de dolor
de cuello de 12 meses. Ella se queja de un aumento de peso de 15 libras y malestar generalizado.
Tiene antecedentes médicos de hipertensión y diabetes mellitus. Sus medicamentos actuales
incluyen un hipoglucemiante oral. El examen físico revela sensibilidad a lo largo del curso de la
glándula tiroides sin evidencia de una masa discreta. ¿Cuál es el diagnóstico más probable?
A. Tiroiditis aguda
B. Tiroiditis de Hashimoto
C. Carcinoma papilar de tiroides
D. Tiroiditis de Riedel
E. Tiroiditis subaguda
62. Una mujer de 47 años con antecedentes de masa tiroidea izquierda se somete a una
lobectomía tiroidea izquierda. La patología revela un carcinoma papilar de 1.3 cm sin evidencia
de extensión extracapsular. ¿Cuál es el siguiente paso más apropiado en el tratamiento de este
paciente?
A. Radioterapia de haz externo
B. Quimioterapia multiagente
C. Tiroidectomía subtotal
D. Tiroidectomía total
E. Espera vigilante con seguimiento periódico

63. Un hombre de 34 años con un nódulo tiroideo se somete a una exploración del cuello. Durante
el procedimiento, es posible que se someta a tiroidectomía. ¿Cuál de las siguientes afirmaciones
sobre el nervio laríngeo superior y la inervación de la glándula tiroides es correcta?
A. La lesión del nervio provoca la inclinación de las cuerdas vocales durante la fonación
B. La lesión nerviosa puede pasar desapercibida en los cantantes.
C. El nervio rara vez está en riesgo durante procedimientos de la cirugía de tiroides
D. El nervio laríngeo superior es principalmente un nervio motor.
E. El nervio laríngeo superior es principalmente un nervio sensorial.

64. Un hombre de 19 años salta del tercer piso de su dormitorio en un aparente intento de
suicidio. Lo llevan al departamento de emergencias inconsciente. Tiene lesiones visibles en la
cabeza y las extremidades inferiores. Tiene un pulso de 110 latidos por minuto, pero es apneico.
¿Cuál es el mejor manejo de la vía aérea para este paciente?
A. Intubación nasotraqueal
B. Intubación oral
C. Intubación oral con elevación del mentón
D. Traqueostomía
E. La intubación no es necesaria para este paciente.

65. Una mujer de 21 años es apuñalada en el pecho por su novio. La llevan al departamento de
emergencias para su evaluación. Su presión arterial es de 130/80 mmHg y su pulso es de 90
latidos por minuto. El examen físico revela una herida de arma blanca en el quinto espacio
intercostal izquierdo en la línea medioclavicular. El examen del cuello es normal. La tráquea es
la línea media y las venas yugulares no están distendidas. Ella tiene una disminución de los
sonidos respiratorios en los campos pulmonares izquierdos. ¿Cuál de los siguientes diagnósticos
se puede descartar con base en la información anterior?
A. Hemotórax izquierdo grande
B. Neumotórax abierto
C. Taponamiento pericárdico
D. Ruptura del bronquio principal izquierdo
E. Neumotórax a tensión

66. Un hombre de 41 años sufre una amputación traumática de tres de sus dedos en una
cortadora de carne. No tiene antecedentes médicos ni quirúrgicos previos. ¿Cuál de las
siguientes modalidades debe usarse para transportar los dedos amputados con el paciente?
A. Colocar en una bolsa de plástico limpia y empacar con hielo seco.
B. Colocar en una bolsa de plástico limpia llena de agua a temperatura ambiente.
C. Colocar en una bolsa de plástico limpia en un cofre lleno de hielo picado y agua.
D. Colocar en una bolsa de plástico limpia llena de agua caliente.
E. Envuelva los dedos amputados con una gasa seca estéril.

67.

68.

69. Un profesor de química de 37 años sufre una salpicadura química de ácido en su ojo derecho
mientras intenta realizar una demostración en su clase de ciencias de la escuela secundaria. Le
duele mucho. Mientras está en el aula y espera que una ambulancia lo transporte al hospital,
¿cuál de las siguientes intervenciones debe realizarse?
a. Instilación de gotas para los ojos con solución salina normal
B. Colocación del parche ocular
C. Lavar los ojos con 1 a 2 L de solución salina normal.
D. Colocación del ojo bajo la luz solar directa.
E. Vigilante espera hasta que llegue la ambulancia

70. Un trabajador de la construcción de 37 años sufrió una lesión por aplastamiento en el muslo
derecho después de que una grúa cayó sobre su pierna en el lugar de trabajo. Lo llevan al
departamento de emergencias para su evaluación. Tiene un dolor significativo en la pierna
derecha y dolor con estiramiento pasivo. La pierna está tensa a la palpación. ¿Cuál es la medición
de presión intracompartimental más probable de la pierna derecha de este paciente?
A. 5 mm Hg
B. 10 mm Hg
C. 15 mm Hg
D. 25 mm Hg
E. 35 mm Hg

71.

72.

73. Un hombre de 65 años con antecedentes de enfermedad coronaria se somete a un bypass


aortobifemoral. ¿Cuál de las siguientes maniobras de manejo intraoperatorio disminuirá su riesgo
de infarto de miocardio intraoperatorio?
A. Bloqueo beta
B. Bloqueo de los canales de calcio
C. Administración de solución salina normal en lugar de solución de Ringer lactato.
D. Uso de propofol
E. Uso de morfina

74. A un hombre sano de 35 años se le diagnostica una hernia inguinal. No tiene antecedentes
de sangrado anormal. ¿Cuál de las siguientes pruebas es absolutamente necesaria antes de
llevarlo al quirófano?
A. Hematocrito
B. Recuento de plaquetas
C. Potasio
D. Recuento de glóbulos blancos
E. Ninguna de las anteriores

75. Un hombre de 50 años tiene diarrea después de una resección intestinal sin complicaciones.
La elección fluida que más se asemeja a su producción es
A. Solución salina normal
B. Solución salina media normal con 20 mEq de potasio
C. Dextrosa al 5% en agua con 3 ampollas de bicarbonato
D. La solución de Ringer lactato
E. Dextrosa al 5 % en SSN

76. Un hombre de 27 años es llevado al departamento de emergencias luego de cortarse la mano


con un cuchillo mientras intenta cortar un panecillo. Tiene antecedentes médicos previos de
infecciones sinusales recurrentes. Su historial quirúrgico previo es notable por la reparación de
una fractura nasal. El examen físico revela una laceración limpia de 4 cm a lo largo del aspecto
palmar de su mano. Los principios más relevantes para este caso serían:
A. Manejo suave del tejido y cierre sin tensión.
B. Injerto de piel cuando sea necesario para cubrir el área
C. Desbridamiento de tejido desvitalizado.
D. Uso de una esponja de vacío para promover la curación.
E. Reconstrucción de colgajo con tejido vascularizado.

77. Un hombre de 68 años es llevado al departamento de emergencias quejándose de dolor


abdominal y en las piernas durante 2 semanas. Tiene antecedentes de hipertensión e
hipercolesterolemia; pesa 290 lb. El examen físico revela una masa abdominal pulsátil en la línea
media. Las extremidades inferiores tienen pulsos desiguales. ¿Cuál de los siguientes es el mejor
próximo paso en la evaluación de este paciente?
A. Aortograma
B. Tomografía computarizada del abdomen y la pelvis
C. Ultrasonido, riñones y vejiga.
D. Ultrasonido, hígado y bazo.
E. Venacavograma

78. Una mujer de 28 años se presenta a su médico para evaluar un bulto en su seno derecho
que se encuentra en la autoexaminación. Tiene antecedentes familiares de cáncer de seno en el
sentido de que su madre murió a los 40 años por esta afección. La madre tuvo una mastectomía
radical modificada seguida de quimioterapia. El examen físico revela un bulto en el seno que es
libremente móvil y está bien circunscrito. No hay hoyuelos, asimetría o retracciones. La lesión
mide 2 cm. ¿Cuál es el siguiente paso en el manejo de este paciente?
A. Biopsia de la lesión con guía ecográfica.
B. Mamografía seguida de tomografía computarizada estereotáctica
C. Prueba para el gen BRCA
D. Ultrasonido de la mama y considerar biopsia de mama
E. Vigilar y seguimiento por médico de atención primaria en 1 año
79. Un hombre de 28 años con antecedentes de dolor abdominal recurrente y diarrea
sanguinolenta se presenta a su médico quejándose de un dolor rectal significativo con las
deposiciones. Ha perdido 15 libras en los últimos 3 meses. El examen físico revela dolor en el
cuadrante inferior derecho e izquierdo hasta la palpación. Los valores de laboratorio revelan un
hematocrito del 28% y una tasa elevada de sedimentación globular. La colonoscopia realizada
en este paciente probablemente revelaría:
A. Lesión de masa colónica
B. Diverticulosis
C. Hemorroides internas
D. Intestino normal
E. Mucosa colónica y rectal engrosada friable

80. Un hombre de 55 años se presenta a su médico quejándose de poliuria, polidipsia, polifagia


y un sarpullido rojo y escamoso en la cara en las últimas 2 semanas. Se vacía 18 veces al día
con una buena corriente. La glucemia en ayunas fue de 325 mg / dL. Ha perdido 20 libras en los
últimos 2 meses y nunca ha tenido niveles elevados de glucosa en sangre en el pasado. El
examen físico del corazón, los pulmones y el abdomen son normales. ¿Cuál es la explicación
más probable para estos hallazgos?
A. Diabetes mellitus tipo I
B. Diabetes mellitus tipo II
C. Glucagonoma
D. Insulinoma
E. Enfermedad de Verner-Morrison

81. Un hombre de 72 años con un historial de tabaquismo de 80 años presenta a su médico que
se queja de debilidad y malestar general. Recientemente ha desarrollado disfagia a alimentos
sólidos. Ha perdido 15 libras en los últimos 3 meses. El examen físico revela linfadenopatía
supraclavicular derecha. Los exámenes cardíacos y pulmonares no son notables. No tiene
ternura de protección o rebote. ¿Cuál de los siguientes estudios proporcionará el diagnóstico
más definitivo?
A. Esofagograma de bario
B. Tomografía computarizada del abdomen y la pelvis
C. Esofagogastroscopia con biopsia.
D. Resonancia magnética del abdomen
E. Ultrasonido del cuadrante superior derecho

82. Una mujer afroamericana de 42 años se somete a una colecistectomía laparoscópica por
dolor crónico en el cuadrante superior derecho. La tomografía computarizada demostró cálculos
biliares y líquido pericolequístico. El procedimiento quirúrgico fue sencillo. El análisis patológico
de los cálculos biliares reveló cálculos de bilirrubinato de calcio. ¿Cuál es la explicación más
probable de estos hallazgos?
A. Altos niveles de colesterol en suero
B. Niveles elevados de lípidos en suero
C. Dieta alta en grasas
D. Anemia falciforme
E. Tumor
83. Una mujer de 36 años se queja de ictericia e hinchazón periférica. Se ordena un
ecocardiograma y se determina que el paciente tiene insuficiencia cardíaca derecha con
congestión hepática y edema periférico. No se detecta ningún soplo. ¿Cuál es la explicación más
adecuada para estos hallazgos?
A. Estenosis aórtica
B. Comunicación interauricular
C. Ductus arterioso permeable
D. Tetralogía de Fallot
E. Defecto septal ventricular

84. Un hombre recién nacido nacido a término de una mujer de abuso de drogas por vía
intravenosa de 27 años tiene una pequeña hernia umbilical. Sus signos vitales son estables. Sus
exámenes cardíacos y pulmonares no son contributivos. ¿Cuál de las siguientes es la explicación
más probable para este hallazgo?
A. Foramen oval permeable
B. Conducto onfalomesentérico permeable
C. Anillo umbilical permeable
D. Uraco patente
E. Conducto vitelino permeable

85.

86. Una mujer de 47 años con pancreatitis biliar es hospitalizada. Tiene antecedentes de
hipertensión e hipercolesterolemia. La colecistectomía temprana está indicada para prevenir cuál
de las siguientes complicaciones?
A. Colangitis
B. Pancreatitis recurrente
C. Úlcera gástrica
D. Íleo biliar
E. Perforación de la vesícula biliar

87. Un hombre de 56 años se queja de tos recurrente y hemoptisis. Tiene antecedentes de


neumonías recurrentes. No fuma y no tiene riesgo laboral de enfermedad pulmonar. El examen
físico revela una disminución de los sonidos respiratorios en el lóbulo superior derecho. La
radiografía de tórax revela una pequeña masa en el cuadrante superior derecho. La broncoscopia
revela un angioma. ¿Cuál es el tratamiento más apropiado para este paciente?
A. Antibióticos
B. Corticosteroides
C. Lobectomía izquierda
D. Lobectomía del lóbulo superior derecho
E. Espera vigilante

88.

89. Un hombre de 30 años se somete a una tomografía computarizada de su abdomen después


de un accidente automovilístico. Era un conductor sin restricciones y fue arrojado del vehículo.
No se encuentran lesiones abdominales agudas. La tomografía computarizada revela riñones
bilaterales agrandados con quistes múltiples presentes en diferentes tamaños. El riñón derecho
mide 15 cm y el riñón izquierdo tiene 16 cm de longitud. El examen físico del corazón, los
pulmones y el abdomen están dentro de los límites normales, aparte de cierta sensibilidad leve
a la palpación en los cuadrantes superiores derecho e izquierdo. ¿Cuál de las siguientes
patologías del sistema nervioso central está más fuertemente asociada con este hallazgo?
A. Círculo de aneurisma de Willis
B. Cisticercosis
C. Infarto
D. Glioma
E. Hematoma subdural

90. Una mujer de 46 años con enfermedad renal poliquística está recibiendo un trasplante de
riñón. A los pocos minutos de las anastomosis de la arteria y la vena renales a la respectiva
arteria y vena ilíaca externa, el riñón recupera rápidamente una coloración rosa y una turgencia
normal del tejido y comienza a excretar orina. El paciente es dado de alta del hospital y es visto
a 1 mes de seguimiento. La creatinina sérica es de 4.2 mg / dL. La producción de orina es de 20
ml / hora. El examen físico del corazón, los pulmones y el abdomen están dentro de los límites
normales. El riñón trasplantado es palpable en la fosa ilíaca derecha. La biopsia posterior del
trasplante muestra inflamación y edema extensos. ¿Cuál es la explicación más probable para
estos hallazgos?
A. Rechazo agudo de trasplante
B. Rechazo crónico de trasplante
C. Enfermedad de injerto contra huésped
D. Rechazo de trasplante hiperagudo
E. Proceso normal de trasplante

91. Un hombre de 52 años se queja de dolor abdominal crónico. Ha sido hospitalizado siete veces
en los últimos 2 años por ataques recurrentes de dolor por pancreatitis crónica. Ha sido tratado
con analgésicos y una pancreatectomía distal parcial. Su dolor aún persiste. ¿Cuál es el siguiente
paso en el tratamiento de este paciente?
A. Uso continuo de analgésicos orales.
B. Corticosteroides
C. Esplacnicectomía
D. Pancreatectomía total
E. Espera vigilante

92. Un médico evalúa a un hombre de 39 años para detectar cálculos renales recurrentes. Ha
sido tratado en el pasado con litotricia extracorpórea por ondas de choque, ureteroscopia y
nefrolitotripsia percutánea. ¿Cuál de las siguientes características sugeriría el diagnóstico de
hiperparatiroidismo primario?
A. Cálculo de 1 mm en rayos X KUB
B. Cálculo de 2 mm en rayos X KUB
C. Cálculo del lado derecho de 1 mm y 2 mm en rayos X KUB
D. Cálculos de los lados derecho e izquierdo de 1 mm, 2 mm y 3 mm en rayos X KUB
E. Cálculo de polo inferior derecho de 4 mm en tomografía computarizada
93. Un médico desea administrar un anestésico local por vía subcutánea a un paciente con una
lesión sospechosa en la frente. La lesión mide 1 cm y el cirujano planea una incisión elíptica para
extirparla. ¿Cuál de las siguientes capas epidérmicas de la piel penetrará primero el médico con
la aguja anestésica local?
A. Estrato basal
B. Estrato córneo
C. Estrato granuloso
D. Stratum lucidum
E. Estrato espinoso

94. Un hombre de 35 años con antecedentes de enfermedad de Crohn se presenta a su médico


para un examen de seguimiento. Tiene una enfermedad ileocolónica difusa en una tomografía
computarizada reciente. Sus medicamentos actuales incluyen sulfasalazina. El examen físico
revela dolor en el cuadrante inferior derecho a palpación profunda. ¿Debería considerarse la
terapia con antibióticos en este paciente, a cuál de los siguientes organismos debería dirigirse?
A. Especies de Mycobacterium
B. Pseudomonas aeruginosa
C. Staphylococcus aureus
D. Streptococcus pneumoniae
E. Streptococcus pyogenes

95. Una mujer de 46 años presenta a su médico antecedentes de disfagia progresiva. Tiene
antecedentes de una pérdida de peso de 15 libras en los últimos 6 meses. El examen físico del
cuello, corazón, pulmones y abdomen no es contributivo. Las pruebas de laboratorio revelan un
hematocrito del 33% y un volumen corpuscular medio de 70. La endoscopia gastrointestinal
superior revela una red esofágica. ¿Cuál es el diagnóstico más probable?
A. Tumor carcinoide
B. Leucemia
C. Linfoma
D. Síndrome de Plummer-Vinson
E. Artritis Reumatoide

96. Un hombre de 47 años está involucrado en un accidente automovilístico. Sufre fracturas en


las costillas 9, 10 y 11 en el lado izquierdo. Es hemodinámicamente inestable y tiene una presión
arterial de 90/50 mm Hg y un pulso de 120 latidos / minuto a pesar de la transfusión de 3 U de
glóbulos rojos. es llevado a la sala de operaciones para laparotomía exploratoria. Se identifica y
se extrae un bazo roto. Al buscar un bazo accesorio potencial, ¿cuál es el lugar más probable
para encontrarlo?
A. Epiplón mayor
B. Cuadrante inferior derecho
C. Cuadrante superior derecho
D. Hilio esplénico
E. Ligamentos esplénicos

97. Un niño de 12 años es llevado a su médico para la evaluación de una masa en el cuello.
Tiene antecedentes de sinusitis recurrente e infecciones de amígdalas. El examen físico revela
una masa en el cuello en la línea media que mide 1,5 cm que se mueve al tragar. No hay
evidencia de linfadenopatía. ¿Cuál es el diagnóstico más probable?
A. Leucemia
B. Linfoma
C. Quiste del conducto tirogloso
D. Fístula tiroglosa
E. Carcinoma de tiroides

98. Durante su rotación de Medicina de Emergencia, lo llaman al departamento de emergencias


para evaluar a un paciente con una lesión oftálmica. ¿Cuál de las siguientes lesiones por
traumatismo oftálmico requiere tratamiento inmediato en el lugar y derivación al oftalmólogo de
guardia?
A. Quemadura ácida
B. Nubosidad corneal
C. Laceración corneal
D. Hifema
E. Quemosis conjuntival severa

99. Un paciente se presenta en el departamento de emergencias luego de ser cortado con un


cuchillo en la pierna izquierda. La herida parece limpia y los bordes están bien opuestos. ¿Cuál
de los siguientes es el método más simple para cerrar esta herida?
A. Cierre primario retrasado
B. Injerto
C. Aletas locales
D. Cierre primario
E. Intención secundaria

100.
1. Tras realizar una endoscopia digestiva a un paciente de 51 años, se nos informa que el
diagnóstico de sospecha es esófago de Barrett. ¿Cuál es, entre las siguientes, la conducta
más adecuada?:
R. C.

2. Un paciente que se sometió a una cirugía bariátrica hace más de un año con pérdida de
más o menos 100 lbs, se presenta para seguimiento en su oficina con un dolor en
hipocondrio derecho intermitente. Sospechamos como causa más frecuente
R. E
3. Ante una lesión cutánea pigmentada conocida, que se hace pruriginosa, ¿cuál sería la
actitud correcta?:
D. Biopsia escisional

4 Femenina de 52 años se presenta al cuarto de urgencia con dolor abdominal severo de 6 horas
de evolución. Resultados de laboratorios con amilasa y lipasa > 300. Antecedentes personales de
VIH, glaucoma y trastorno de ansiedad todos medicados. ¿Cúal es la condición cuyo tratamiento
medicamentoso sería el más probable responsable del cuadro clínico?
B. VIH
5. .Paciente masculino de 72 años con historia de 3 meses de pérdida de peso, dolor epigástrico
intermitente y heces blancas. Al examen físico ictericia escleral y masa palpable en hipcondrio
derecho, sin dolor abdominal. Se palpa ganglio supraclavicular izquierdo. ¿Cuál es el diagnóstico
más probable en este caso?
A. Cáncer de Pancreas

6 Respecto a las hernias inguinales lo siguiente es cierto EXCEPTO:


B. Una hernia es deslizante cuando el contenido del saco entra fácilmente en la cavidad abdominal
7
7 En cuál de los siguientes pacientes se debería recomendar Cirugía Bariátrica.
R. A

8 Paciente se le realizó una Billroth II luego de un tumor gástrico distal, presenta dos semanas
después de la cirugía al cuarto de urgencias con vómitos biliosos y dolor en el cuadrante superior
derecho. Luego de vomitar , siente gran alivio. Cuál es el diagnóstico más probable?
B. Síndrome de Asa aferente

R: Síndrome de Asa Aferente

9 Señale en cuál de las siguientes enfermedades la manometría esofágica estacionaria constituye


la prueba diagnóstica definitiva B

10 Paciente femenina de 40 años se presenta al cuarto de urgencias con 3 horas de dolor


abdominal cólico y múltiples episodios de vómito, siente leve mejoría luego de vomitar. Dice que el
vómito era verdoso. Su última evacuación fue hace 2 días, pero ha continuado pasando flatus . Sus
signos vitales son normales, excepto por el pulso en 115/lpm. Al examen físico está en leve distres,
su abdomen está distendido y dolor difuso a la palpación con ruidos hidroaéreos aumentados, tiene
múltiples cicatrices de cirugías previas en el abdomen. Hemograma, urinálisis y niveles de
electrolitos están normales. Rx de abdomen de pie y acostado revela múltiples niveles hidroaéreos,
hay aire en el colon. Cuál de las siguientes es el manejo más apropriado? b

11 Masculino de 48 años quien toma cerveza una caja al día, desarrolla nauseas y vómitos. Luego
de esto, el paciente presenta dolor torácico severo y es traido al cuarto de urgencias. El tiempo
transcurrido desde su episodio de vómito hasta que llega al cuarto de urgencia son 5 horas. Su PA
es 100/50 y su FC es 110, se le dan dos bolos de líquido intravenoso. Se obtiene una radiografía de
tórax y muestra una efusión izquierda y un neumotórax. Cuál es el estudio más sensitivo para
diagnosticar la condición de este paciente :
12 Los estudios en un paciente revelan sangrado por várices esofágicas. El siguiente tratamiento
sería: A

13 Acude a consulta un paciente de 72 años por presentar disfagia para sólidos y líquidos, desde
hace 4 meses. Además, aqueja adelgazamiento de 8 Kg. En el estudio radiológico con contraste se
observa un esófago ligeramente dilatado que se estrecha en su parte distal. ¿Cuál de los siguientes
debe ser el próximo paso?:

14 Una de las siguientes es un efecto metabólica de las cirugías bariátricas :


C.

15 Cuál de las siguientes estructuras anatómicas es más profunda en la región inguinal: b

R: fascia transversalis
16 Un hombre de 62 años se queja de incomodidad perianal e informa que hay vetas fecales que
ensucian su ropa interior. Hace cuatro meses tuvo un absceso perirrectal drenado quirúrgicamente.
El examen físico muestra una abertura perianal en la piel, y se puede palpar un tramo en forma de
cordón desde la abertura hacia el interior del canal anal. La secreción purulenta de color marrón
puede expresarse desde el tracto. D

R: Esta entidad sigue la ley de Goodsall que dice que las fístulas anteriores son rectas y que
posteriores son curvas y tienen doble orificio

17 Mujer de 41 años con enfermedad de Crohn´s ha sido sometida a múltiples intervenciones


quirúrgicas. Recientemente se le realizó una ileostomía pero aún tiene evidencia de enfermedad en
el jejuno distal. Sus medicamentos incluyen prednisona, ácido amino salicílico. Cuál de los
siguientes efectos de la terapia prolongada con glucocorticoides es posible en este paciente? C
18 La mucosa esofágica está compuesta por una capa de epitelio:C.

19 Cuál de las siguientes situaciones no determina la aparición de reflujo gastroesofágico?:E. Estoy


en duda
ESCLERODERMIA ES DISFAGIA MOTORA
20 Este tipo de Tumor se origina en

21 El concepto de “cirugía profiláctica” puede aplicarse a las siguientes patologías EXCEPTO:B.


22 Una mujer de 56 años asiste al cuarto de urgencias con dolor abdominal severo. No tiene otros
antecedentes médicos . Toma licor ocasionalmente . Los laboratorios muestran amilasa en 24.000 y
lipasa en 7.000. Radiografía de abdomen no muestra patología. Su wbc : 17.000. Su PA es 80/40 y
su FC es 120. Afebril. Al examen tiene dolor en el epigastrio. El paso más apropiado a seguir es : B
en duda

23 Cuál de los siguientes tumores se considera más radiosensible?: A.


24 El esófago de Barret consiste en la sustitución del epitelio escamoso normal de la parte distal del
esófago por epitelio de tipo:

D.
25 Los siguientes son síntomas comunes del dumping , excepto:
C
26 Es causa de esofagitis por ERGE…
D

27 Paciente femenina de 43 años se presenta con pérdida de peso, diarrea y dolor abdominal difuso
de 2 meses de evolución. El CAT muestra masa en cabeza de páncreas. ¿Cuál es el hallazgo de
laboratorio más probable en este caso?
C
28 Femenina de 47 años se presenta a Urgencias con 4 horas de dolor abdominal irradiado a
espalda y nauseas. Es casada, con 3 hijos y trabaja de secretaria. Niega consumo de alcohol,
tabaco o drogas. Al examen físico: febril y taquicárdica. Estudios de imagen muestran líquido
peripancreático sin necrosis y colelitiasis. Luego de su trtamiento inicial con líquidos intravenosos,
analgesia y reposo intestinal presenta mejoría completa y se programa el egreso. ¿Cuál es el
siguiente paso más apropiado en su manejo?
D

29 Respecto a las hernias crurales lo siguiente es cierto:


mas frecuentes en mujeres

30 Paciente Femenina de 32 años acude a su oficina con hx de 6 meses de evacuaciones líquidas,


aproximadamente 8 por día. Hay presencia de sangre en ellas. Ha perdido 30 libras y en las últimas
6 semanas ha tenido fiebre intermitentemente. No tiene historia de problemas gastrointestinales, y
no hay historia familiar de problemas gastrointestinales. Al examen, paciente en distrés. BP 130/70.
Pulso 108 regular. Hay dolor abdominal difuso, sin rebote. Una sigmoidoscopía revela una mucosa
friable con varios puntos de sangrado . Cuál es el diagnóstico más probable de este paciente?
D

31 Estudio de elección para determinar enfermedad ganglionar y a distancia

32 Una mujer de 23 años describe un dolor exquisito con defecación y vetas de sangre en el
exterior de las heces. Debido al dolor, evita evacuar y cuando finalmente lo hace, las heces son
duras e incluso más dolorosas. No se puede realizar el examen físico, ya que ella se niega a
permitir que alguien separe las nalgas para mirar el ano por miedo a provocar el dolor.
E
inguinal

33 Cuando existe una hernia inguinal la siguiente estructura pueden incluirse en el cordón
espermático: C

34 En relación con la patología motora del esófago, una de las siguientes afirmaciones
es FALSA
35. Un hombre VIH positivo de 55 años tiene una masa fungosa que crece fuera del ano y ganglios
linfáticos agrandados y duros como la roca en ambas ingles. Ha perdido mucho peso, y se ve
demacrado y enfermo. ¿Qué es?
E
36 Acerca del cáncer mamario en el varón, señale la respuesta incorrecta:
D

37. ¿Qué estructura pasa por el hiato esofágico, además del propio esófago?:
C

38. Todos son signos de hipertensión portal excepto:


B
39. Paciente masculino de 28 años acude a cuarto de urgencias con dolor abdominal difuso, fiebre,
malestar general. Al examen físico con dificultad respiratoria franca, leucocitosis de 30,000. el
paciente es admitido en UCI intubado e inicia antibióticos. Dos días después persiste febril con
leucocitosis igual. Se le realiza una tomografía y se evidencia una apendicitis perforada. Es llevado
a salón de operaciones donde se le realiza LPE y apendicectomía + lavado de absceso
intrabdominal. A los tres días el paciente con ictericia y fiebre. Cuál sería el abordaje diagnóstico a
realizarle a este paciente ?
B

40 La pared del esófago carece de:


E
41 A la vista de los conocimientos actuales, el esófago de Barrett es un fenómeno probablemente
debido a:
A
42 Los fibroadenomas de mama durante el embarazo, es más probable que sufran:
A

43 Cuál es el tratamiento de elección de un melanoma > de 1 mm, sin evidencia clínica de


afectación ganglionar?:
R: Extirpación amplia y biopsia selectiva del ganglio centinela
44 De los referidos a continuación, ¿ cuál el factor pronóstico mas importante del melanoma ?
R: Presencia de metástasis ganglionares

45 Un paciente de 50 años refiere disfagia lentamente progresiva y fenómeno de Raynaud. Cuál es


el diagnóstico más probable
R: Esclerodermia**
46 Paciente femenina de 40 años se presenta al cuarto de urgencias con 2 días de dolor abdominal
moderado acompañado de náuseas y vómitos. No ha presentado evacuación o pasado flatus desde
hace 4 días. Su historia médica es relevante por síndrome de dolor crónico para lo que toma
opioides regularmente. Sus signos vitales son normales. Al examen, está en leve distres, su
abdomen está distendido y está doloroso difusamente a la palpación, ruidos hidroaèreos están
ausentes. No tiene ni defensa ni rebote. Hemograma, urinàlisis y panel metabólico y otros
laboratorios no presentan anormalidades. Radiografía de abdomen supino y de pie revelan
distensión general del intestino delgado y colon, hay aire a través de las asas. Cuál de las
siguientes es el diagnóstico más probable?
R: íleo paralítico

47 Masculino de 48 años, alcoholismo severo, se presenta con historia de dolor abdominal intenso
desde hace 2 semanas. Se le realiza estudios de imágenes que muestran una lesión de paredes
delgadas en íntima relación con el páncreas que mide 4 cm y que se comunica con el ducto
pancreático principal.¿Cuál es el manejo de elección en este caso?
R: observación
48 Usted le está dando seguimiento a un paciente con diagnóstico de Enfermedad de Crohn quien
se está recuperando luego de una resección intestinal. Cuál de las siguientes afirmaciones le indica
que el paciente entiende a fondo su enfermedad?
R: la enfermedad puede reaparecer en otra parte del intestino

49 El músculo cremaster es una derivación de una de las siguientes estructuras anatómicas:


B. OBLICUO INTERNO
acalasia

50 ¿Cuál de las siguientes exploraciones tiene más rentabilidad díagnóstica en la acalasia?:


D. MANOMETRIA ESOFAGICA

51 Respecto a las hernias deslizantes es CIERTO que:


A. EL SACO INCLUYE*******
52 Masculino de 48 años, alcoholismo severo, se presenta con historia de dolor abdominal intenso
desde hace 2 semanas. Se le realiza estudios de imágenes que muestran una lesión de paredes
delgadas en íntima relación con el páncreas que mide 4 cm y que se comunica con el ducto
pancreático principal. ¿Cuál es el diagnóstico más probable?
A. PSEUDOQUISTE DE PÁNCREAS

53 Los siguientes enunciados son ciertos para el Cáncer gástrico tipo Difuso (Clasificación de
Lauren)
A. SE PRESENTA EN PACIENTES JÓVENES
54 Cuál de las siguientes No es una manifestación extraintestinal de la Enfermedad de Crohn?
d. Dermatitis herpertiforme

55 Usted es el médico de turno y está valorando un paciente con diagnóstico de colitis ulcerativa. El
paciente presenta alteración del estado mental, fiebre, distensión abdominal. Cuál de los siguientes
hallazgos confirma el diagnóstico de megacolon Tóxico?
c. Leucocitosis O Constipacion no estoy segura
56 Son manifestaciones de perforación de esófago torácico todas, EXCEPTO:
c. Neumoperitoneo

57 Se sabe que los siguientes estados presentan un patrón familiar relacionado a una
predisposición a padecer un cáncer EXCEPTO:
d. Peutz Jeghers
58 Masculino de 56 años con alcoholismo crónico que acude con dolor abdominal epigástrico 10/10
de 5 dias de evolución. Se ordena CAT que revele liquido peripancreático inflamatorio. ¿Cuál
laboratorio es mas probable que este anormal en este paciente?
c. Lipasa elevada

59 ¿Cuál de las siguientes exploraciones tiene más rentabilidad díagnóstica en la acalasia?:


A. El estudio radiografico con bario
60 Paciente femenina de 40 años afroamericana se presenta al departamento de urgencias
quejándose de confusión, dolor abdominal cólico, vómitos, fiebre, y escalofríos. Ha notado que en
los dos últimos días sus heces han estado más pálidas de lo normal. Su temperatura es 39.6C
(103.3F), pulso 120/min, presión arterial es 85/55, fr 25/min. Al examen se encuentra en distrés,
escleras ictéricas, ruidos intestinales normales, abdomen está doloroso a la palpación en el
cuadrante superior derecho sin rebote. Laboratorios revelan: leu 18,000, bilirrubina total 4.5, AST 90
I/U, ALT 100 I/U, fosfatasa alcalina es 250 I/U. CAT revela vías biliares intrahepàticas dilatadas, hay
neumobilia y piedras en la vesícula. Cuál de las siguientes es una prioridad en el manejo de este
paciente?
c. descomprension de las vias biliares
61 Durante el examen físico en un paciente con epigastralgia y pérdida de peso usted palpa un
ganglio supraclavicular izquierdo (ganglio de Virchow) y un nódulo umbilical que es conocido como:

rB

62 El reflujo gastroesofágico se produce fundamentalmente a causa de


D. Incompetencia del esfinter esofagico inferior
63 Un hombre de 60 años que se sabe que tiene hemorroides informa sangre roja brillante en el
papel higiénico después de la evacuación. ¿Qué es?
A. Hemorroides internas que sangran

64 Estudio de elección para determinar profundidad de invasión en el cáncer gástrico


C. usg endoscópico
65 Paciente femenina de 55 años, enfermera. Cuadro clínico de 6 meses de evolución
caracterizado por epigastralgia, dispepsia y pérdida de peso no cuantificada.
Estudio Diagnóstico de elección
D. ENDOSCOPIA profetica

66. Paciente de 320 lbs fue sometido a un bypass gástrico laparoscópico, fue difícil intubar al
paciente antes de la cirugía, tiene dolor en las heridas, el pulso está en 120, piden evaluación al
médico ya que presenta fiebre de 38 grados , se le administra algo para el manejo del dolor y se le
hidrata adecuadamente . Aún permanece taquicardico y presenta saturación en 82% con cánula
nasal. Qué debemos evaluar en este paciente :
D. evaluar por fuga o leak de anastomosis, mediante estudio radiológico profetica
67. Paciente de 39 años, masculino, en cuarto de urgencias con dolor abdominal 2 dias de
evolución. Antecedente de alcoholismo severo. Al examen físico con dolor abdominal y equimosis
periumbilical y flancos bilaterales. ¿Cuál es la mejor estrategia de tratamiento en este caso?
B. Unidad Cuidados Intensivos, reposo intestinal, hidratación endovenosa profetica

68. Cual de las siguientes entidades puede cursar con alteración de la motilidad
esofágica.
E. TODAS
69. Paciente femenina de 79 años acude por sus propios medios al cuarto de urgencias por dolor
abdominal difuso, llenura post prandial temprana, náuseas y pérdida de peso. Su médico de
cabecera le realiza estudios de laboratorios que están normales. Se le realiza USG abdominal que
evidencia masa quística uniloculada de 8 cm en lóbulo hepático izquierdo adyacente al antro
gástrico compatible con quiste simple. Cuál sería el tratamiento de elección para esta paciente?:
D. Drenaje + agente

70. Una endoscopía reporta lesión exofítica y friable en antro gástrico. Se toma biopsia.
¿Cuál es la histología más frecuente en Cáncer gástrico?
c. adenocarcinoma
71. La pared de los divertículos falsos del tubo gastrointestinal está formada por:
C. MUCOSA Y SUB

72. Un enfermo de 45 años, bebedor de 60 grs. de alcohol diarios, desde hace 6 meses aqueja
disfagia para sólidos, odinofagia, sialorrea y anemia ferropénica. En la endoscopia se ha encontrado
una estenosis esofágica 9 cm. por encima del cardias y en la biopsia epitelio columnar displásico
¿Cuál es el diagnóstico de presunción?:
E.
73. Los divertículos por tracción son más comunes en:
R// D

74¿Cuál de los siguientes constituye el tratamiento más efectivo y seguro a largo plazo en los
pacientes con acalasia?
R//D
75La hipoglucemia postabsortiva por tumores no insulinomas se caracteriza:
R//A

76. Qué medicamentos son considerados efectivos en el manejo de la enfermedad inflamatoria


intestinal leve a moderada?
R// B
45 año

77. Masculino de 45 años con historia de cólico biliar que se presenta con vomitos y dolor
abdominal irradiado a espalda. Febril, PA 102/78 mmHg y Fc 112/min. Dolor a la palpación de
epigastrio e hipocondrio derecho. Laboratorios: glóbulos blancos 18,200/uL; Fosfatasa alcalina 650
U/L; Bilirrubinas totales 2.5 mg/dL, amilasa 500 U/L y lipasa 1160 U/L. ¿Cuál de los valores de
laboratorio está asociado a incremento en la mortalidad para este paciente?
e. lipasa

78. En la clasificación TNM la designación T0 (cero) significa:


R//B
79. Cómo se forma un absceso en el hígado?
R//C

80. Un hombre de 60 años que se sabe que tiene hemorroides se queja de comezón y malestar
anal, particularmente hacia el final del día. Él tiene dolor perianal leve al sentarse y se encuentra
sentado de lado para evitar la incomodidad. Cuál es la incorrecta?
R. A
81 Una de las siguientes es falso con respecto a la cirugía bariátrica :

R. E
82 El manejo del absceso hepático es:
R. A

83Masculino de 52 años con antecedente de etilismo se presenta al cuarto de urgencias con


hematemesis, taquicárdico, pálido. Cuáles serían los pasos a seguir en su manejo:
R. D
84 Luego de una terapia apropiada se encuentra que el paciente es cirrótico con un año de
abstinencia al alcohol, múltiples várices gástricas y esofágicas, ascitis, trombocitopenia, leucopenia
asociado a esplenomegalia de 15 cm.
R. A

85 Paciente Masculino de 25 años con historia de dolor abdominal crónico por 18 meses. Ha visto a
varios médicos, le han dicho que tiene problemas estomacales, síndrome de intestino irritable y
depresión. Asociado al dolor abdominal presenta diarrea, anorexia, pérdida de 20 lb. Al examen
tiene dolor alrededor del ano, dolor abdominal difuso y eritema en el área peri rectal. Cuál es el
diagnóstico más probable?
R. C
86 Un paciente de 32 años exhibe una historia de disfagia, de varios años de evolución, que ha
progresado en las últimas semanas hasta tener dificultades, no diarias, para ingerir líquidos.
Describe también episodios aislados de regurgitación no ácida de alimentos y ha tenido, el año
pasado, dos episodios de neumonía. La exploración que más probablemente hará el díagnóstico
correcto será
R. a

87 Un hombre de 44 años aparece en el servicio de urgencias a las 11 pm con un exquisito dolor


perianal. Él no puede sentarse, informa que las evacuaciones intestinales son muy dolorosas, y ha
tenido escalofríos y fiebre. El examen físico muestra una masa caliente, sensible, roja y fluctuante
entre el ano y la tuberosidad isquiática.
R: C Abceso isquiorrectal

88 ¿Con qué frecuencia deben de realizarse mamografías en el seguimiento de la paciente que ha


padecido cáncer de mama?:
R: C. Anual

89 El tratamiento más eficaz disponible no quirúrgico de la Acalasia es


R: C. Dilatacion con globo neumatico
90 Paciente presenta con diarrea crónica luego de gastrectomía total que le causa deficiencias
vitamínicas y mucho picor en el ano, cuál es las siguientes se considera una alternativa de tra
tamiento:
R: B. Colestiramina

91 Paciente masculino de 50 años que se presenta con cuadro de 1 año de evolución de llenura
postprandial, saciedad temprana y pérdida de peso no cuantificada. Se realiza endoscopía que
reporta mucosa sin lesiones, pero con imagen de compresión extrínseca.
Su principal sospecha diagnóstica es
R: D. Tumor del estroma
92 Tratamiento de elección de MALT
R: E. Triple terapia

93 Masculino de 56 años acude a su medico familiar ya que su esposa lo nota “amarillo”. Paciente
niega cualquier síntoma, excepto haber perdido peso en los últimos 4 meses Antecedente: diabetes
mellitus tipo 2. Laboratorios: BR totales 15 mg/dL, BR directa 10 mg/dL y fosfatasa alcalina 560
IU/L. USG abdominal no muestra litos en vesícula o vía biliar. ¿Cuál es el siguiente paso
recomendado?
R: D. Cat Abdominal
94Un hombre de 28 años es llevado a la oficina por su madre. En los últimos 4 meses ha tenido 3
operaciones -realizadas en otro lado- para una fístula perianal, aunque después de cada una el
área no se ha curado, y de hecho las heridas quirúrgicas se han agrandado. El paciente ahora tiene
múltiples úlceras, fisuras y fístulas que no cicatrizan alrededor del ano, con secreción purulenta. No
hay masas palpables. Todo lo anterior es cierto excepto:
95Paciente femenina de 28 años acude a su ginecólogo para iniciar consejería en planificación
familiar. La paciente toma anticonceptivos orales desde hace 8 años. En los estudios previos se
realiza USG abdominal que evidencia una masa en lóbulo hepático derecho de 5 cm con refuerzo
periférico sólida. Cuáles serían los siguientes pasos en el abordaje diagnóstico para esta paciente?
C. onservacion y biopsia
96El cordón espermático contiene todo lo siguiente EXCEPTO:
b.- nervio genitofemoral

97Se le realiza ultrasonido y se evidencia una masa en lóbulo hepático derecho de 7x 9 cm con
nivel hidroaereo y componente quístico en continuidad con la rama derecha de la vía biliar. Cuál
sería el posible diagnóstico del paciente:
b no seguro

98La capa más fuerte de la pared intestinal es :


mucosa
99 Tratamiento de elección para Adenocarcinoma Gástrico de cardias
A

100Hombre de 66 años de edad con hipertensión, enfermedad úlcero péptica y melanoma, se


presenta al cuarto de urgencias con aparición súbita de dolor abdominal severo 9/10, en el
cuadrante superior izquierdo. Niega náuseas y vómitos, dice no tener apetito a pesar que su última
comida fue hace 6 horas. El dolor se irradia a su hombro izquierdo y espalda y ha ido empeorando
con el tiempo. Está acostado en la camilla con sus rodillas en su pecho. Tu temperatura es 37C
(100.2F), presión arterial 152/88mm Hg, pulso es 110/min, fr 18/min. Al examen físico el abdomen
está distendido, doloroso y rígido, timpanismo aumentado. Ruidos intestinales no se encontraron. El
resto del examen físico no es contributorio. Radiografía abdominal muestra aire libre intraperitoneal.
Laboratorios relevantes muestran: Leu: 14,000/mm3, Hemoglobina: 13.2 g/dl, Plaquetas:
303,000/mm3, Bilirrubina directa: 0.2 mg/dl, Bilirrubina total: 0.9mg/dl, Amilasa 210 U/I, Lipasa 20
U/L, AST: 33 U/I, ALT 30 U/L, Fosfatasa alcalina: 45 U/I. Cuál de las siguientes es el diagnóstico?
D. ÚLCERA PÉPTICA PERFORADA
TOPITOS MED12

CIRUGÍA 2

Una mujer de 56 años asiste al cuarto de urgencias con dolor abdominal severo. No tiene
otros antecedentes médicos . Toma licor ocasionalmente . Los laboratorios muestran
amilasa en 24.000 y lipasa en 7.000. Radiografía de abdomen no muestra patología. Su
wbc : 17.000. Su PA es 80/40 y su FC es 120. Afebril. Al examen tiene dolor en el
epigastrio. El paso más apropiado a seguir es :
A- INICIAR LÍQUIDOS IV Y CONSEGUIR USG

Una mujer de 56 años asiste al cuarto de urgencias con dolor abdominal


severo. No tiene otros antecedentes médicos . Toma licor
ocasionalmente . Los laboratorios muestran amilasa en 24.000 y lipasa
en 7.000. Radiografía de abdomen no muestra patología. Su wbc :
17.000. Su PA es 80/40 y su FC es 120. Afebril. Al examen tiene dolor en
el epigastrio. El paso más apropiado a seguir es :
Admisnistrar luquidos y conseguir un ultrasonido

Se realiza una laparoscopía a un hombre de 25 años con diagnóstico presunto de


apendicitis, se encuentra una ileitis terminal que no está envolviendo el área cecal. La
ileitis no es obstructiva , el paso más apropiado a seguir es :
apendicectomia

Masculino de 48 años quien toma cerveza una caja al día, desarrolla nauseas y vómitos.
Luego de esto, el paciente presenta dolor torácico severo y es traido al cuarto de
urgencias. El tiempo transcurrido desde su episodio de vómito hasta que llega al cuarto
de urgencia son 5 horas. Su PA es 100/50 y su FC es 110, se le dan dos bolos de líquido
intravenoso. Se obtiene una radiografía de tórax y muestra una efusión izquierda y un
neumotórax. Cuál es el estudio más sensitivo para diagnosticar la condición de este
paciente :
D- TRAGO CON GASTROGRAFINA, LUEGO DE HACER TRAGO DE BARIO

La capa más fuerte de la pared intestinal es: mucosa

Paciente de 320 lbs fue sometido a un bypass gástrico laparoscópico, fue difícil intubar al
paciente antes de la cirugía, tiene dolor en las heridas, el pulso está en 120, piden
evaluación al médico ya que presenta fiebre de 38 grados , se le administra algo para el
manejo del dolor y se le hidrata adecuadamente . Aún permanece taquicardico y presenta
saturación en 82% con cánula nasal. Qué debemos evaluar en este paciente :
A- DIURESIS ADECUADA, PUEDE ESTAR DESHIDRATADO *SEGÚN EL DR
Paciente se le realizó una Billroth II luego de un tumor gástrico distal, presenta dos
semanas después de la cirugía al cuarto de urgencias con vómitos biliosos y dolor en el
cuadrante superior derecho. Luego de vomitar , siente gran alivio. Cuál es el diagnóstico
más probable? SINDROME DE ASA AFERENTE

Los siguientes son síntomas comunes del dumping , excepto: debilidad mareos

Paciente presenta con diarrea crónica luego de gastrectomía total que le causa
deficiencias vitamínicas y mucho picor en el ano, cuál es las siguientes se considera una
alternativa de tratamiento: COLESTIRAMINA PARA MEJORAR LA ABSORCIÓN DE SALES
BILIARES

16. Los siguientes son síntomas comunes del dumping , excepto:


C- DEBILIDAD Y MAREOS

Paciente femenina de 40 años se presenta al cuarto de urgencias con 2 días de dolor


abdominal moderado acompañado de náuseas y vómitos. No ha presentado evacuación o
pasado flatus desde hace 4 días. Su historia médica es relevante por síndrome de dolor
crónico para lo que toma opioides regularmente. Sus signos vitales son normales. Al
examen, está en leve distres, su abdomen está distendido y está doloroso difusamente a
la palpación, ruidos hidroaèreos están ausentes. No tiene ni defensa ni rebote.
Hemograma, urinàlisis y panel metabólico y otros laboratorios no presentan
anormalidades. Radiografía de abdomen supino y de pie revelan distensión general del
intestino delgado y colon, hay aire a través de las asas. Cuál de las siguientes es el
diagnóstico más probable? ileo paralitico

Paciente femenina de 40 años afroamericana se presenta al departamento de urgencias


quejándose de confusión, dolor abdominal cólico, vómitos, fiebre, y escalofríos. Ha
notado que en los dos últimos días sus heces han estado más pálidas de lo normal. Su
temperatura es 39.6C (103.3F), pulso 120/min, presión arterial es 85/55, fr 25/min. Al
examen se encuentra en distrés, escleras ictéricas, ruidos intestinales normales,
abdomen está doloroso a la palpación en el cuadrante superior derecho sin rebote.
Laboratorios revelan: leu 18,000, bilirrubina total 4.5, AST 90 I/U, ALT 100 I/U, fosfatasa
alcalina es 250 I/U. CAT revela vías biliares intrahepàticas dilatadas, hay neumobilia y
piedras en la vesícula. Cuál de las siguientes es una prioridad en el manejo de este
paciente? ACCESO VASCULAR Y LÍQUIDOS IV

Paciente femenina de 40 años se presenta al cuarto de urgencias con 3 horas de dolor


abdominal cólico y múltiples episodios de vómito, siente leve mejoría luego de vomitar.
Dice que el vómito era verdoso. Su última evacuación fue hace 2 días, pero ha continuado
pasando flatus . Sus signos vitales son normales, excepto por el pulso en 115/lpm. Al
examen físico está en leve distres, su abdomen está distendido y dolor difuso a la
palpación con ruidos hidroaéreos aumentados, tiene múltiples cicatrices de cirugías
previas en el abdomen. Hemograma, urinálisis y niveles de electrolitos están normales.
Rx de abdomen de pie y acostado revela múltiples niveles hidroaéreos, hay aire en el
colon. Cuál de las siguientes es el manejo más apropriado? Admitir al hospital, nxb,
sonda ng y liquidos iv
Paciente femenina de 40 años se presenta al cuarto de urgencias con 3 horas de dolor
abdominal cólico y múltiples episodios de vómito, siente leve mejoría luego de vomitar.
Dice que el vómito era verdoso. Su última evacuación fue hace 2 días, pero ha continuado
pasando flatus . Sus signos vitales son normales, excepto por el pulso en 115/lpm. Al
examen físico está en leve distrés, su abdomen está distendido y dolor difuso a la
palpación con ruidos hidroaéreos aumentados, tiene múltiples cicatrices de cirugías
previas en el abdomen. Hemograma, urinálisis y niveles de electrolitos están normales.
Rx de abdomen de pie y acostado revela múltiples niveles hidroaéreos, hay aire en el
colon. Cuál de las siguientes es el manejo más apropiado? B- ADMITIR A HOSPITAL,
NADA POR BOCA, COLOCAR SNG, DAR LÍQUIDOS INTRAVENOSOS

Hombre de 66 años de edad con hipertensión, enfermedad úlcero péptica y melanoma, se


presenta al cuarto de urgencias con aparición súbita de dolor abdominal severo 9/10, en
el cuadrante superior izquierdo. Niega náuseas y vómitos, dice no tener apetito a pesar
que su última comida fue hace 6 horas. El dolor se irradia a su hombro izquierdo y
espalda y ha ido empeorando con el tiempo. Está acostado en la camilla con sus rodillas
en su pecho. Tu temperatura es 37C (100.2F), presión arterial 152/88mm Hg, pulso es
110/min, fr 18/min. Al examen físico el abdomen está distendido, doloroso y rígido,
timpanismo aumentado. Ruidos intestinales no se encontraron. El resto del examen físico
no es contributorio. Radiografía abdominal muestra aire libre intraperitoneal. Laboratorios
relevantes muestran: Leu: 14,000/mm3, Hemoglobina: 13.2 g/dl, Plaquetas: 303,000/mm3,
Bilirrubina directa: 0.2 mg/dl, Bilirrubina total: 0.9mg/dl, Amilasa 210 U/I, Lipasa 20 U/L,
AST: 33 U/I, ALT 30 U/L, Fosfatasa alcalina: 45 U/I. Cuál de las siguientes es el
diagnóstico?
D- ÚLCERA PÉPTICA PERFORADA

Masculino de 48 años, alcoholismo severo, se presenta con historia de dolor abdominal


intenso desde hace 2 semanas. Se le realiza estudios de imágenes que muestran una
lesión de paredes delgadas en íntima relación con el páncreas que mide 4 cm y que se
comunica con el ducto pancreático principal. ¿Cuál es el diagnóstico más probable?
A- PSEUDOQUISTE DE PÁNCREAS

Masculino de 48 años, alcoholismo severo, se presenta con historia de dolor abdominal


intenso desde hace 2 semanas. Se le realiza estudios de imágenes que muestran una
lesión de paredes delgadas en íntima relación con el páncreas que mide 4 cm y que se
comunica con el ducto pancreático principal.¿Cuál es el manejo de elección en este
caso? OBSERVACION

Femenina de 52 años se presenta al cuarto de urgencia con dolor abdominal severo de 6


horas de evolución. Resultados de laboratorios con amilasa y lipasa > 300. Antecedentes
personales de VIH, glaucoma y trastorno de ansiedad todos medicados. ¿Cúal es la
condición cuyo tratamiento medicamentoso sería el más probable responsable del cuadro
clínico? VIH

Paciente de 39 años, masculino, en cuarto de urgencias con dolor abdominal 2 dias de


evolución. Antecedente de alcoholismo severo. Al examen físico con dolor abdominal y
equimosis periumbilical y flancos bilaterales. ¿Cuál es la mejor estrategia de tratamiento
en este caso? UNIDAD DE CUIDADOS INTENSIVOS, REPOSO INTESTINAL, HIDRATACIÓN
IV
Masculino de 56 años acude a su medico familiar ya que su esposa lo nota “amarillo”.
Paciente niega cualquier síntoma, excepto haber perdido peso en los últimos 4 meses
Antecedente: diabetes mellitus tipo 2. Laboratorios: BR totales 15 mg/dL, BR directa 10
mg/dL y fosfatasa alcalina 560 IU/L. USG abdominal no muestra litos en vesícula o vía
biliar. ¿Cuál es el siguiente paso recomendado? CAT abdominal

Masculino de 56 años con alcoholismo crónico que acude con dolor abdominal
epigástrico 10/10 de 5 dias de evolución. Se ordena CAT que revele liquido
peripancreático inflamatorio. ¿Cuál laboratorio es mas probable que este anormal en este
paciente? lipasa elevada

Femenina de 47 años se presenta a Urgencias con 4 horas de dolor abdominal irradiado a


espalda y nauseas. Es casada, con 3 hijos y trabaja de secretaria. Niega consumo de
alcohol, tabaco o drogas. Al examen físico: febril y taquicárdica. Estudios de imagen
muestran líquido peripancreático sin necrosis y colelitiasis. Luego de su trtamiento inicial
con líquidos intravenosos, analgesia y reposo intestinal presenta mejoría completa y se
programa el egreso. ¿Cuál es el siguiente paso más apropiado en su manejo?
Colecistectomia laparóscopica

Paciente masculino de 72 años con historia de 3 meses de pérdida de peso, dolor


epigástrico intermitente y heces blancas. Al examen físico ictericia escleral y masa
palpable en hipcondrio derecho, sin dolor abdominal. Se palpa ganglio supraclavicular
izquierdo. ¿Cuál es el diagnóstico más probable en este caso? CÁNCER DE PÁNCREAS

Paciente femenina de 55 años, enfermera. Cuadro clínico de 6 meses de evolución


caracterizado por epigastralgia, dispepsia y pérdida de peso no cuantificada.
Estudio Diagnóstico de elección D- ENDOSCOPIA

Durante el examen físico en un paciente con epigastralgia y pérdida de peso usted palpa
un ganglio supraclavicular izquierdo (ganglio de Virchow) y un nódulo umbilical que es
conocido como: mary joseph
creo que no todos tan copiando
fold el doc
esta bien profetico, eso es lo que pasa
Ey, dejen algunas malas para disimular
cuanto son algunas? 5 ? 6?
Como 9? Al azar asi que cada uno tenga una mala diferente
digo que como 5 ta cool
pienso que mas de 5 esta cool
a weno, y si, al azar

De los referidos a continuación, ¿ cuál el factor pronóstico mas importante del melanoma
?
Espesor de Breslow
Parcial 2 de cirugia - MED 10
1. Mujer de 41 años con enfermedad de Crohn's ha sido sometida a múltiples intervenciones
quirúrgicas. Recientemente se le realizó una ileostomía pero aún tiene evidencia de
enfermedad en el jejuno distal. Sus medicamentos incluyen prednisona, ácido amino salicílico.
Cuál de los siguientes efectos de la terapia prolongada con glucocorticoides es posible en este
paciente?

A. Producción de anticuerpos
B.formación de colágeno .
C. disfunción de fibroblastos
D. migración de células inflamatorias
E. alteración en proceso de cicatrización

2. Paciente Femenina de 32 años acude a su oficina con hx de 6 meses de evacuaciones


líquidas, aproximadamente 8 por día. Hay presencia de sangre en ellas. Ha perdido 30 libras y
en las últimas 6 semanas ha tenido fiebre intermitentemente. No tiene historia de problemas
gastrointestinales, y no hay historia familiar de problemas gastrointestinales. Al examen,
paciente en distrés. BP 130/70. Pulso 108 regular. Hay dolor abdominal difuso, sin rebote. Una
sigmoidoscopía revela una mucosa friable con varios puntos de sangrado. Cuál es el
diagnóstico más probable de este paciente.

A. Colitis pseudomembranosa
B.sindrome de intestino irritable
C. Enfermedad de Crohn
D. Colitis ulcerativa

3. Se realiza una laparoscopía a un hombre de 25 años con diagnóstico presuntivo de


apendicitis, se encuentra una ileítis terminal que no está envolviendo el área cecal. La ileitis no
es obstructiva , el paso más apropiado a seguir es :
A. apendicectomía
B. cerrar al paciente
C. colocar un drenaje
D. resección del ileum

4. El timoma:
A. Es la neoplasia más común en el mediastino medio
B. La mayoría desarrollan miastenia gravis
C. Dan metástasis a distancia con frecuencia
D. El tratamiento es la cirugía

5. Masculino de 56 años acude a su medico familiar ya que su esposa lo nota "amarillo".


Paciente niega cualquier sintoma, excepto haber perdido peso en los últimos 4 meses
Antecedente: diabetes mellitus tipo 2. Laboratorios: BR totales 15 mg/dL, BR directa 10 mg/dL y
fosfatasa alcalina 560 IU/L. USG abdominal no muestra litos en vesícula o via biliar.
¿Cuál es el siguiente paso recomendado?

A. Resonancia magnética (colangioresonancia)


B. CPRE
C. HIDA Scan
D. CAT abdominal

6. El Ácido Tranexámico es un antifibrinolítico que se ha comprobado reduce la mortalidad en


trauma, tanto globales como por coagulopatía, ¿qué es cierto del mismo?:

A. Su efecto en la muerte por sangrado no depende de la Presión Arterial Sistólica


B. No disminuye la necesidad de transfusiones
C. Está indicado en pacientes que están con hipercoagulación
D. El mayor beneficio se observa al ser administrado a las 9 horas del trauma
E. Aumenta la tasa trombosis, específicamente el infarto agudo al miocardio

7. Respecto a la fisiologia del peritoneo señalar las opciones incorrecta:

A. el peritoneo puede absorber hasta 35 C.C. por hora por gradiente osmótico
B. la función plástica (creación de adherencias) depende de los fibroblastos del
peritoneo
C. los estímulos dolorosos del peritoneo parietal producen sobre todo contractura de los
músculos del abdomen
D. la distensión o tracción del peritoneo visceral puede producir bradicardia

8. Usted es el médico de turno y está valorando un paciente con diagnóstico de colitis


ulcerativa. El paciente presenta alteración del estado mental, fiebre, distensión abdominal. Cuál
de los siguientes hallazgos confirma el diagnóstico de megacolon Tóxico?
A. Bradicardia
B. ESplenomegalia
C. Leucocitosis
D. Constipación

9. La capa más fuerte de la pared intestinal es :


A. mucosa
B. submucosa
C. muscularis
D. serosa

10. Paciente de 39 años, masculino, en cuarto de urgencias con dolor abdominal 2 dias de
evolución. Antecedente de alcoholismo severo. Al examen físico con dolor abdominal y
equimosis periumbilical y flancos bilaterales. ¿Cuál es la mejor estrategia de tratamiento en
este caso?
A. CPRE
B. Unidad Cuidados Intensivos, reposo intestinal, hidratación endovenosa
C. Unidad de Cuidados Intensivos y clordiazepoxido
D. Endoscopia alta para varices esofágicas

11. Endpoint que indica reanimación adecuada en pacientes con shock es:
A. Clearence de lactado de 10% en 2 horas
B. Llenado Capilar de 3.5 segundos
C. Saturación Venosa Central de 60%
D. índice de Perfusión periférica de 1
E. Diurésis de 0.3 ml/kg/h

12. De los siguientes factores, cuál es parte de la denominada tríada de la muerte?


A. Hipotensión
B. Coagulopatía
C. Hipertermia maligna
D. Oliguria
E. Alcalosis

13. EI ABC score es útil para predecir la necesidad de transfusión masiva. El mismo incluye:

A. Tomografia com evidencia de sangrado activo.


B. Presión arterial sistolica en el salón de operaciones <90
C. Diurésis menor de 0.5cc/kg/h
D. Frecuencia cardíaca mayor de 120
E. Historia de trauma cerrado

14. Femenina de 47 años se presenta a Urgencias con 4 horas de dolor abdominal irradiado a
espalda y nauseas. Es casada, con 3 hijos y trabaja de secretaria. Niega consumo de alcohol,
tabaco o drogas. Al examen físico: febril y taquicárdica. Estudios de imagen muestran líquido
peripancreático sin necrosis y colelitiasis. Luego de su trtamiento inicial con líquidos
intravenosos, analgesia y reposo intestinal presenta mejoría completa y se programa el egreso.
¿Cuál es el siguiente paso más apropiado en su manejo?
A. Litotripsia
B. CPRE
C. Repetir CAT en 6 semanas
D. Colecistectomia laparoscópica

15. Cuál de las siguientes No es una manifestación extraintestinal de la Enfermedad de Crohn?


A. Artritis
B. Uveitis
C. eritema Nodoso
D. Dermatitis herpetiforme

16. Femenina de 37 años con múltiples heridas de proyectil de arma de fuego tórax,
abdomen,extremidades superiores y extremidades inferiores. Al llegar a urgencias: pálida,
sudorosa, poco cooperadora, FC 140, FR 38, no se auscultan ruidos respiratorios en el pulmón
derecho. tiene deformidad en el muslo izquierdo y pierna derecha. Para la reanimación de esta
paciente:
A. se debe transfundir GRE, PFC y plaquetas manteniendo proporción de 1:1:1 ó
1:1:2
B. Se debe transfundir GRE Y PFC de acuerdo a evolución y resultados de laboratorio
C. Se debe transfundir PFC y plaquetas manteniendo proporción 1:1 ó 1:2
D. Plaquetas son transfundidas de acuerdo a valor en hemograma y evaluación de
hematología
E. No es necesario transfundir

17. Una niña de 14 años tiene una masa firme, móvil y gomosa en el seno izquierdo que se
notó por primera vez hace 1 año y desde entonces ha crecido hasta tener 6 cm de diámetro.
qué es?
A. Fibroadenoma
B. Fibroadenoma juvenil gigante
C. Cystosarcoma phyllodes
D. quiste palpable en enfermedad fibroquística
E. papiloma intraductal

18 . En la bacteriología de las peritonitis agudas secundarias el germen más frecuente de los


siguientes es :
A. estreptococo fecal
B. clostridium
C. corynebacterium
D. bacteroides fragilis
E. estreptococo B hemofilico

19. Masculino 48 años quien toma cerveza una caja cada dia, desarrolla nauseas y vómitos.
Luego de esto, el paciente presenta dolor torácico severo y es traído al cuarto de urgencias. el
tiempo transcurrido desde su episodio de vómito hasta que llega al cuarto de urgencias 5
horas. Su PA 100/50 y su FC es 110, se le dan dos bolos de líquidos intravenosos . Se obtiene
una radiografía de tórax y muestra efusión izquierda y un neumotórax. Cuál es el estudio más
sensitivos para el diagnóstico la condición de este paciente.
A. Tomografía abdominal
B. tomografía de tórax
C. angiograma
D. trago con gastrografia, luego de hacer trago de bario

20. Masculino de 45 años con historia de cólico biliar que se presenta con vomito y dolor
abdominal irradiado a espalda. Febril, PA 100/78 mmHg y Fc 112/min. Dolor a la palpación de
epigastrio e hipocondrio derecho. Laboratorios: glóbulos blancos: 18 200/uL; fosfatasa alcalina
650U/L; bilirrubinas totales 2.5mg/dL, amilasa 500U/L y lipasa 1160 U/L. Cuál de los valores de
laboratorio está asociado a incremento en la mortalidad para este paciente.
A. Glóbulos blancos
B. fosfatasa alcalina
C. bilirrubina totales
D. amilasa
E. lipasa

21. Usted le está dando seguimiento a un paciente con diagnóstico de Enfermedad de Crohn
quien se está recuperando luego de una resección intestinal. Cuál de las siguientes
afirmaciones le indica que el paciente entiende el fondo de su enfermedad.
A. “Luego de la resección del intestino, la enfermedad está curada”
B. “la enfermedad puede reaparecer en otra parte del intestino”
C. “Puedo descontinuar la toma de los medicamentos”
D. “puedo desarrollar colitis ulcerativa porque falta un pedazo de intestino”

22. Un hombre VIH positivo de 55 años tiene una masa fungosa que crece fuera del ano y
ganglios linfáticos agrandados y duros como la roca en ambas ingles. Ha perdido mucho peso,
y se ve demacrado y enfermo. ¿Qué es?
A. Carcinoma de células escamosas del ano.
B. Realizar el diagnóstico adecuado con biopsias de la masa fungosa.
C. El protocolo Nigro de quimioterapia y radiación pre operatorias combinadas durante 5
semanas con una tasa de curación del 90%.
D. La cirugía se realiza solo si Nigro no cura el cáncer.
E. Todo lo anterior es cierto

23. Paciente femenina de 40 años afroamericana se presenta al departamento de urgencias


quejándose de confusión, dolor abdominal cólico, vómitos, fiebre, y escalofríos. Ha notado que
en los dos últimos días sus heces han estado más pálidas de lo normal. Su temperatura es
39.6C (103.3F), pulso 120/min, presión arterial es 85/55, fr 25/min. Al examen se encuentra en
distrés, escleras ictéricas, ruidos intestinales normales, abdomen está doloroso a la palpación
en el cuadrante superior derecho sin rebote. Laboratorios revelan: leu 18,000, bilirrubina total
4.5, AST 90 I/U, ALT 100 l/U, fosfatasa alcalina es 250 l/U. CAT revela vías biliares
intrahepàticas dilatadas, hay neumobilia y piedras en la vesícula. Cuál de las siguientes es una
prioridad en el manejo de este paciente?
A. acceso vascular y líquidos intravenosos
B. imagenología adicional de las vías biliares
C. descompresión de emergencia de vías biliares
D. colocar sonda nasogástrica
E. antibióticos orales empíricos

24 ¿Cuál es el manejo de elección en un pseudoquiste de páncreas?


A. Drenaje vía ultrasonido endoscópico
B. Realizar CPRE + esfinteroplastia
C. Cistogastrostomía
D. Observación

25. Un vólvulo de intestino delgado da lugar a una situación de:


A. İleo paralítico
B. Oclusión mecánica simple
C. Síndrome de asa ciega
D. Síndrome de intestino corto
E. Oclusión mecánica estrangulada

26 . Masculino de 48 años, alcoholismo severo, se presenta con historia de dolor abdominal


intenso desde hace 2 semanas. Se le realiza estudios de imágenes que muestran una lesión de
paredes delgadas en íntima relación con el páncreas que mide 4 cm y que se comunica con el
ducto pancreático principal. ¿Cuál es el diagnóstico más probable?
A. Pseudoquiste de páncreas
B. Colección peripancreática
C. Pancreatitis aguda tardía
D. Ruptura de ducto de Wirsung

27. Hombre de 66 años de edad con hipertensión, enfermedad úlcera péptica y melanoma, se
presenta al cuarto de urgencias con aparición súbita de dolor abdominal severo 9/10, en el
cuadrante superior izquierdo. Niega náuseas y vómitos, dice no tener apetito a pesar que su
última comida fue hace 6 horas. El dolor se irradia a su hombro izquierdo y espalda y ha ido
empeorando con el tiempo. Está acostado en la camilla con sus rodillas en su pecho. Tu
temperatura es 37 C (100.2F), presión arterial 152/88 mm Hg, pulso es 110/min, fr 18/min. Al
examen físico el abdomen está distendido, doloroso y rígido, el timpanismo aumentado. Ruidos
intestinales no se encontraron. El resto del examen físico no es contributorio. Radiografía
abdominal muestra aire libre intraperitoneal. Laboratorios relevantes muestran: Leu
14,000/mm3, Hemoglobina: 13.2 g/dl, Plaquetas: 303,000/mm3, Bilirrubina directa: 0.2 mg/dl,
Bilirrubina total: 0.9mg/dl, Amilasa 210 U/1, Lipasa 20 U/L, AST: 33 U/1, ALT 30 U/L, Fosfatasa
alcalina: 45 U/1. Cuál de las siguientes es el diagnóstico?
A. colecistitis
B. hepatitis
C. pancreatitis
D. úlcera péptica perforada
E. obstrucción intestinal

28. Los fenómenos generales de respuesta a una peritonitis pueden incluir los siguientes
EXCEPTO:
A. retención agua por activación de los sistemas aldosterona+ADH
B. alcalosis metabólica
C. hipoventilación
D. taquipnea
E. hipopotasemia

29. Ud. evalúa a un paciente de 77 años de edad, emaciado y cardiópata, al que se diagnostica
por imágenes, empiema crónico. ¿Cuál sería el mejor tratamiento para este paciente?
A. Toracotomía y decorticación
B. Solo antibióticos
C. Drenaje abierto y resección de costilla
D. toracocentesis y lo toracotomía cerrad

30. Una mujer de 34 años de edad ha estado teniendo secreción sanguinolenta del pezón
Previo derecho, de forma intermitente durante varios meses. No hay masas palpables. ¿Qué
es?
A. Fibroadenoma.
B. Fibroadenoma juvenil gigante
C. Cystosarcoma phyllodes
D. Quiste palpable en enfermedad fibroquística (mastitis quística, displasia mamaria).
E. Papiloma intraductal.

31. Cuando se produce neumotórax a repetición:


A. Se indica cirugía
B. Indica Enfermedad pulmonar obstructiva crónica.
C. El tratamiento de elección es la toracocentesis
D. El tratamiento inicial no fue el adecuado.

32 ¿Cuál de los siguientes órganos tiene capa serosa visceral?:


A. el apéndice
B. el recto
C. la cara posterior de la segunda porción duodenal
D. el esofago torácico

33. Una sospecha de obstrucción intestinal de intestino delgado se valorará en un primer


momento con:
A. Un tránsito intestinal
B. Una radiografía simple en decúbito prono
C. Una radiografía simple en bipedestación
D. Una ecografía
E. Un Enema Opaco
F.
34. Paciente de 25 años quien tiene dificultad para evacuar, tiene mucho dolor anal, sensación
de masa y al limpiarse mancha el papel de color rojo rutilante el diagnostico mas probable es
A. Fisura anal.
B. Fistula anal.
C. Absceso anal.
D. Hemorroides externas trombosadas.

35. En las peritonitis establecidas se producen los siguientes fenómenos, con excepción de:
A. ileo adinámico
B. aumento de la absorción de líquidos
C. hipovolemia por creación de tercer espacio
D. diapedesis de polimorfonucleares
E. exudado intraperitoneal rico en proteínas y fibrina
36. Desde un punto de vista clínico en una obstrucción intestinal encontraremos:
A. los vómitos suelen preceder al dolor cólico intestinal
B. en la exploración abdominal el meteorismo y el signo de Blumberg siempre coinciden
C. el tacto rectal siempre es doloroso al palpar el fondo de saco de Douglas
D. primero escucharemos borborigmos y ruidos metálicos y posteriormente silencio
abdominal

37. La incidencia de la apendicectomía por apendicitis aguda disminuyó en los Estados Unidos
hasta la década de 1990, momento en el cual la frecuencia de la apendicectomía por
apendicitis no perforada comenzó a aumentar. ¿Cuál es una explicación potencial para esta
observación?
A. Mayor uso de imágenes de diagnóstico y detección de apendicitis que de otro
modo se habrían resuelto.
B. Mayor incidencia de obesidad y el impacto de la grasa peri-apendicular en la
obstrucción luminal.
C. Aumento de la incidencia de la enfermedad inflamatoria intestinal y la mitigación
potencial de los síntomas de colitis ulcerosa observados con apendicectomía
D. Los patrones de reembolso han cambiado en los Estados Unidos, favoreciendo una
toma de decisiones quirúrgica agresiva.

38. Los fibroadenomas de mama durante el embarazo, es más probable que sufran:
A. Crecimiento.
B. Disminución de tamaño.
C. Degeneración quística.
D. Degeneración maligna.
E. Remisión.

39. Un absceso del fondo de saco de Douglas se puede valorar en la exploración clínica con:
A. la maniobra de Murphy
B. un tacto rectal
C. la maniobra de Blumberg
D. el signo de Rovsing
E. la maniobra de rebote

40. Paciente femenina de 40 años se presenta al cuarto de urgencias con 3 horas de dolor
abdominal cólico y múltiples episodios de vómito, siente leve mejoría luego de vomitar. Dice
que el vómito era verdoso. Su última evacuación fue hace 2 días, pero ha continuado pasando
flatus. Sus signos vitales son normales, excepto por el pulso en 115/1pm. Al examen fisico está
en leve distrés, su abdomen está distendido y dolor difuso a la palpación con ruidos
hidroaéreos aumentados, tiene múltiples cicatrices de cirugías previas en el abdomen.
Hemograma, urianálisis y niveles de electrolitos están normales. Rx de abdomen de pie y
acostado revela múltiples niveles hidroaéreos, hay aire en el colon. Cuál de las siguientes es el
manejo más apropiado?
A. admitir al hospital, líquidos intravenosos, realizar CPRE
B. admitir al hospital, nada por boca, colocar sonda nasogástrica, dar líquidos
intravenosos
C. admitir al hospital, colocar tubo rectal, administrar enemas, desimpactar heces
D. laparotomía de emergencia
E. nada por boca y observar en el cuarto de urgencias

41. En un paciente con empiema posneumónico en fase fibrinopurulenta:


A. El tratamiento de elección es el uso de fibrinolíticos
B. Los antibióticos y un drenaje pleural son el tratamiento de elección.
C. La tomografía computada del tórax no es útil en esta etapa del empiema.
D. La videotoracoscopia es el tratamiento indicado.

42. En una obstrucción intestinal a nivel de la unión recto-sigmoidea la zona que más se
dilatará será:
A. el recto
B. el sigma
C. el ciego
D. el ileon terminal
E. el yeyuno
43. Una mujer de 18 años tiene una masa firme y gomosa en el seno izquierdo que se mueve
fácilmente con la palpación. ¿Qué es?
A. Fibroadenoma.
B. Fibroadenoma juvenil gigante
C. Cystosarcoma phyllodes
D. Quiste palpable en enfermedad fibroquística (mastitis quística, displasia mamaria).
E. Papiloma intraductal.

44. Un hombre de 34 años se presenta en su clínica preguntando sobre una apendicectomia


electiva. No tiene antecedentes de apendicitis. ¿Cuáles son las posibles indicaciones de
apendicectomía electiva en este paciente?
A. Viaje planeado a un lugar remoto sin atención quirúrgica.
B. Pacientes con enfermedad de Crohn donde el ciego está libre de enfermedad grave.
C. Como parte del procedimiento de Ladd.
D. Todas las anteriores.

45. Una mujer de 56 años asiste al cuarto de urgencias con dolor abdominal severo. No tiene
con dolor abdominal severo No tiene otros antecedentes médicos. Toma licor ocasionalmente.
Los laboratorios muestran amilasa en 24.000 y lipasa en 7.000. Radiografía de abdomen no
muestra patología. Su Wbc : 17.000. Su PA es 80/40 y su FC es 120. Afebril. Al examen tiene
dolor en el epigastrio. El paso más apropiado a seguir es:
A. Iniciar líquidos intravenosos y conseguir ultrasonido
B. Iniciar líquidos intravenosos y conseguir tomografía
C. Iniciar líquidos intravenosos e ir al SOP
D. Iniciar líquidos intravenosos e ir a resonancia

46. Una mujer de 45 años se presenta con dolor en el CID. Se realiza una tomografia
computarizada. ¿Qué hallazgo en la tomografia computarizada es el más sugerente de linfoma
apendicular?
A. Diámetro apendicular> 2,5 cm o engrosamiento del tejido blando circundante
B. Falta de contraste llenando el apéndice
C. Masa de 1 cm en la base del apéndice.
D. Nódulos linfáticos aórticos prominentes

47. Femenina de 52 años se presenta al cuarto de urgencia con dolor abdominal severo de 6
horas de evolución. Resultados de laboratorios con amilasa y lipasa > 300. Antecedentes
personales de VIH, glaucoma y trastomo de ansiedad todos medicados. ¿Cual es la condición
cuyo tratamiento medicamentoso seria el más probable responsable del cuadro clínico?
A. Glaucoma
B. VIH
C. Trastorno de ansiedad
D. Todas las anteriores

48. Para confirmar el diagnóstico de presunción de una obstrucción intestinal a nivel de colon
descendente podemos indicar una de las siguientes exploraciones EXCEPTO
A. radiologia simple de abdomen
B. enema opaco
C. TAC abdominal
D. transito intestinal con bario
E. colonoscopia

49. Las hemorroides internas que se reducen solas son:


A. Grado I
B. Grado II
C. Grado III
D. Grado IV

50. Paciente Masculino de 25 años con historia de dolor abdominal crónico por 18 meses. Ha
visto a varios médicos, le han dicho que tiene problemas estomacales, sindrome de intestino
irritable y depresión. Asociado al dolor abdominal presenta diarrea, anorexia, pérdida de 20 Ib.
Al examen tiene dolor alrededor del ano, dolor abdominal difuso y eritema en el área peri rectal.
Cuál es el diagnóstico más probable?
A. Fisura anal
B. pancreatitis crónica
C. Enfermedad de Crohn
D. Enfermedad de Hirschsprung

51. Paciente presenta con diarrea crónica luego de gastrectomía total que le causa deficiencias
vitaminas y mucho picor en el ano, cuál es las siguientes se considera una alternativa de
tratamiento
A. interposición del segmento de ileon
B. colestiramina para mejorar la absorción de sales biliares
C. dar metoclopramida o cisaprida para mejorar peristalsis
D. recomendaciones de dieta alta en carbohidratos

52. Mientras revisa la patología de una apendicectomia laparoscópica reciente, observa que
además de la apendicitis aguda, el paciente tenía un tumor carcinoide de 1,5 cm ubicado en la
base del apéndice. El paciente está sano y se está recuperando bien de la cirugía. ¿Qué
recomendarías?
A. No se necesita terapia adicional.
B. Hemicolectomía derecha.
C. Apendicectomia radical.
D. Quimioterapia adyuvante.

53. ¿Con qué frecuencia deben de realizarse mamografias en el seguimiento de la paciente


que ha padecido cáncer de mama:
A. Trimestral.
B. Semestral.
C. Anual
D. Bianual.
E. Sólo si en la exploración clinica se aprecian anormalidades

54. Un hombre de 25 años de edad presenta dolor migratorio en el cuadrante inferior derecho
(CID), leucocitosis y una tomografia computarizada compatible con apendicitis aguda y sin
complicaciones. Es fisiológicamente normal y son las 2 de la mañana. Ud. está planeando una
apendicectomía, ¿qué diferencia podría esperarse en su resultado si su operación se retrasa
hasta la mañana siguiente?
A. Mayor riesgo de un absceso intraabdominal
B. Mayor riesgo de infección en el sitio quirúrgico.
C. Disminución del tiempo operatorio
D. Mayor riesgo de perforación.
E. No hay diferencia en las tasas de perforación, infección del sitio quirúrgico,
absceso, tasa de conversión o tiempo operatorio

55. Masculino de 56 años con alcoholismo crónico que acude con dolor abdominal epigástrico
10/10 de 5 dias de evolución. Se ordena CAT que revele liquido peripancreático inflamatorio
¿Cuál laboratorio es mas probable que este anormal en este paciente?
A. Transaminasas elevadas
B. Lipasa disminuida
C. Lipasa elevada
D. Ferritina disminuida

56. Paciente se le realizó una Billroth II luego de un tumor gástrico distal, presenta dos
semanas después de la cirugia al cuarto de urgencias con vómitos biliosos y dolor en el
cuadrante superior derecho. Luego de vomitar, siente gran alivio. Cuál es el diagnóstico más
probable?
A. Sindrome de estasis del Roux
B. Sindrome de Asa aferente
C. Sindrome de Dumping
D. reflujo alcalino y gastritis

57. Una inmigrante de 27 años de México tiene una masa de 12 x 10 x 7 cm en el seno


izquierdo. Ha estado presente durante 7 años, y ha ido creciendo lentamente a su tamaño
actual. La masa firme, gomosa, completamente móvil, no está unida a la pared del tórax ni a la
piel que lo recubre. No hay nodos axilares palpables. ¿Qué es?
A. Fibroadenoma
B. Fibroadenoma juvenil gigante
C. Cystosarcoma phyllodes
D. Quiste palpable en enfermedad fibroquística (mastitis quística, displasia mamaria)
E. Papiloma intraductal.

58. Acerca del cáncer mamario en el varón, señale la respuesta incorrecta:


A. Constituye aproximadamente 1% de los cánceres de mama.
B. Pronóstico mejor que en la mujer
C. Frecuente asociación con ginecomastia.
D. Se trata mediante mastectomía radical modificada.

59. Una mujer de 35 años de edad tiene una historia de 10 años de sensibilidad en ambos
senos, relacionada con el ciclo menstrual, con múltiples bultos en ambos senos que parecen "ir
y venir" en diferentes momentos del ciclo menstrual. Ahora tiene una masa firme y redonda de
2 cm que no desapareció durante 6 semanas. ¿Qué es?
A. Fibroadenoma
B. Fibroadenoma juvenil gigante
C. Cystosarcoma phyllodes
D. Quiste palpable en enfermedad fibroquística (mastitis quística, displasia mamaria)
E. Papiloma intraductal.

60. En una obstrucción mecánica de intestino delgado lo siguiente es cierto EXCEPTO:


A. se pueden emitir gases y heces en las primeras horas
B. el comienzo del dolor es debido a la distensión de las asas
C. no deben existir signos de irritación peritoneal en las primeras horas
D. el dolor iniciales de caracteristicas cólicas
E. la auscultación es de ruidos hidroaereos progresivos al comienzo.
61. Es el mejor líquido para la reanimación de pacientes luego de trauma es:
A. Cristaloides
B. Coloides
C. Sangre
D. soluciones hipertónicas
E. Ninguna de las anteriores

62. En la zona proximal a la obstrucción, la dilatación del intestino delgado tendrá como
consecuencias EXCEPTO
A. acumulo de gases como consecuencia de fenómenos de putrefacción
B. disminución de la absorción en los tramos dilatados
C. edema de la pared intestinal
D. aumento del peristaltismo de lucha en las zonas dilatadas
E. proliferación bacteriana

63. El método de enguantado abierto consiste en:


A. Colocarse los guantes estériles con vestimenta estéril
B. Colocarse los guantes estériles sin vestimenta estéril
C. El que se utiliza cuando la instrumentista nos viste en el salón de operaciones
D. Colocarse doble guantes
E. Ninguna de las anteriores

64. Paciente de 320 lbs fue sometido a un bypass gástrico laparoscópico, fue difícil intubar al
paciente antes de la cirugía, tiene dolor en las heridas, el pulso está en 120, piden evaluación
al médico ya que presenta fiebre de 38 grados, se le administra algo para el manejo del dolor y
se le hidrata adecuadamente. Aún permanece taquicardico y presenta saturación en 82% con
cánula nasal. Qué debemos evaluar en este paciente:
A. diuresis adecuada, puede estar deshidratado
B. si está haciendo ejercicios respiratorios, puede tener atelectasia
C. dupplex de extremidades, descartar trombosis venosa profunda
D. evaluar por fuga o leak de anastomosis, mediante estudio radiológico
E. radiografía de tórax descartar broncoaspiración

65. Un hombre de 62 años se queja de incomodidad perianal e informa que hay vetas fecales
que ensucian su ropa interior. Hace cuatro meses tuvo un absceso perirrectal drenado
quirúrgicamente. El examen fisico muestra una abertura perianal en la piel, y se puede palpar
un tramo en forma de cordón desde la abertura hacia el interior del canal anal. La secreción
purulenta de color marrón puede expresarse desde el tracto.
A. Lo más probable es que se trate de una fisura recidivante en el ano.
B. En este caso no hay que descartar el cáncer ya que los tumores necróticos es muy
difícil que drenen
C. Es imperativo programar lo antes posible la fistulotomia definitiva.
D. Esta entidad sigue la ley de Goodsall que dice que las fistulas anteriores son
rectas y que las posteriores son curvas y tienen doble orificio
E. Ninguna de las anteriores

66. El cáncer de pulmón:


A. Podríamos decir que se divide en células no pequeñas, de células pequeñas y de
células grandes
B. El mayor factor de riesgo es el asbesto.
C. El síntoma más común es la tos.
D. El mejor tratamiento es la cirugía en todas las variedades histológicas

67. En los tumores neurogénicos:


A. En los adultos más del 50% son malignos
B. Se encuentran en el surco paravertebral
C. Los tumores de dumbell son más frecuentes en niños.
D. El tratamiento es la quimioterapia y radioterapia

68. Un paciente de 70 años es referido a su consulta con diagnóstico de cáncer de pulmón de


lóbulo superior izquierdo. El diagnóstico histológico es escamoso, obtenido por una biopsia
bronquial. Además tiene ganglios positivos paratraqueales izquierdos. Resto de los estudios
negativos.
El paciente:
A. Está en un estadio IV.
B. Su mejor opción es la cirugía
C. Ud. recomendaría terapia de inducción
D. Está fuera de tratamiento.

69. Paciente masculino de 72 años con historia de 3 meses de pérdida de peso, dolor
epigastrico intermitente y heces blancas. Al examen fisico ictericia escleral y masa palpable en
hipocondrio derecho, sin dolor abdominal. Se palpa ganglio supraclavicular izquierdo.
¿Cuál es el diagnóstico más probable en este caso?
A. Cáncer de páncreas
B. Coledocolitiasis
C. Colangitis
D. Hepatocarcinoma

70. La causa más frecuente de obstrucción intestinal en una paciente operada es:
A. adherencias
B. cáncer de colon sigmoideo
C. cáncer de colon ascendente
D. hernia inguinal
E. hernia crural
71. Qué medicamentos son considerados efectivos en el manejo de la enfermedad inflamatoria
intestinal leve a moderada?
A. Prednisona
B. Sulfasalazina
C. Metronidazol
D. Azatioprina (imuran)

72. ¿Cuál de los siguientes signos fisicos se asocia con la definición correcta que sugiere
apendicitis aguda?
A. Signo de Rovsing: dolor en el CID al palpar el cuadrante inferior izquierdo
B. Signo de Dunphy: dolor en el CID con palpación a la izquierda
C. Signo de obturador: dolor con extensión de la pierna
D. Signo del psoas: dolor en la rotación interna de la cadera derecha

73. El espacio peritoneal más declive es:


A. la fosa de Morrisson
B. el espacio parietocólico derecho
C. el fondo de saco de Douglas
D. el espacio subfrénico izquierdo
E. la transcavidad de los epiplones

74. Un hombre de 55 años de edad tiene evidencia de TC de apendicitis complicada con un


absceso contenido en el CID. Está levemente taquicardico, afebril y normotenso, con
sensibilidad focal en el CID pero sin peritonitis. ¿Cuál es el enfoque óptimo para este paciente?
A. Laparotomía inmediata.
B. Exploración laparoscópica y drenaje de abscesos
C. Drenaje percutáneo, líquidos intravenosos (IV), reposo intestinal y antibióticos de
amplio espectro.
D. Liquidos intravenosos, reposo intestinal y antibióticos de amplio espectro.

75. Paciente masculino de 25 años. Acude al cuarto de urgencias con historia de Accidente
automovilistico tipo colisión frontal como pasajero delantero. Al examen físico: se encuentra
ansioso, preguntando por su cartera y con PA 130/80 FC: 98 FR: 24. El paciente esta en
shock?
A. Si
B. No
C. Necesitamos gases arteriales para determinarlo
D. Necesitamos conocer la diuresis para determinarlo
E. Ninguna de las anteriores

76. Una mujer de 23 años que está embarazada de 28 semanas presenta dolor abdominal en
el lado derecho, leucocitosis y una ecografia abdominal que no visualiza el apéndice. ¿Qué
intervención recomendarías?
A. Laparoscopia exploratoria.
B. TC abdominal.
C. Imagen de resonancia magnética abdominal (IRM).
D. Observaciones clínicas seriales.

77. Una mujer de 29 años presenta dolor en el CID, fiebre y leucocitosis. Antes de los estudios
de imagen, la puntuación de Alvarado se utiliza para determinar la probabilidad de que el
paciente tenga apendicitis. Todas las siguientes variables conforman la puntuación de Alvarado
EXCEPTO
A. Anorexia
B. Desplazamiento a la izquierda de los neutrófilos
C. Signo del iliopsoas
D. Dolor en el CID
E. Fiebre
78. En estudio radiológico simple la postura del paciente MENOS EFICAZ para valorar una
obstrucción intestinal de intestino delgado será:
A. Bipedestación con rayo horizontal
B. Decúbito lateral derecho con rayo horizontal
C. Decúbito lateral izquierdo con rayo horizontal
D. Decúbito supino con rayo vertical
E. Decúbito supino con rayo horizontal

79. Un hombre de 34 años se somete a una apendicectomía sin complicaciones por apendicitis
aguda no perforada. Las notas del informe de patología dicen: inflamación aguda con un
adenocarcinoma de 1 cm del apéndice medio. Este paciente debería tener
A. Sin tratamiento adicional
B. Quimioterapia
C. Radiación regional
D. Hemicolectomía derecha
E. Ileocecectomía

80. Un niño de 8 años se presenta al departamento de emergencias quejándose de dolor


abdominal generalizado durante las últimas 24 horas. Las pruebas de laboratorio revelan una
leucocitosis de 13,000 y él está sensible en el RLQ en el examen físico. Se le lleva al quirófano
para una apendicectomía laparoscópica. La eliminación del apéndice se ha asociado con un
efecto protector de cuál de los siguientes?
A. Colitis de Crohn
B. Colitis ulcerosa
C. Clostridium difficile
D. Carcinoide

81. Paciente femenina de 43 años se presenta con pérdida de peso, diarrea y dolor abdominal
difuso de 2 meses de evolución. El CAT muestra masa en cabeza de páncreas. ¿Cuál es el
hallazgo de laboratorio más probable en este caso?
A. Hemoglobina/hematocrito elevado
B. Alfa-feto proteína elevada
C. Ca 19-9 elevado
D. CEA elevado

82. Una mujer de 62 tiene una lesión eczematoide en la areola. Ha estado presente durante 3
meses y le parece que tiene “algún tipo de afección de la piel” que no mejoró ni desapareció con
una variedad de lociones y ungüentos.
A. Estas son todas presentaciones clásicas de cáncer de mama.
B. Las masas duras son probablemente adenocarcinoma de mama invasivo.
C. La piel roja y naranja es probablemente un cáncer de mama inflamatorio.
D. La lesión areolar del eccematoide es probablemente la enfermedad de Paget del
seno (una forma rara de cáncer de mama).

83. El mesotelioma:
A. Puede ser benigno o maligno.
B. El tratamiento es la cirugía y generalmente es curativa.
C. Generalmente es multicéntrico y su tratamiento es controversial.
D. En el estadio IV, el tratamiento de elección es la cirugía.

84. Paciente masculino 48 años, acude con herida por proyectil de arma de fuego en cuello,
tórax y abdomen. Glasgow 15/15 FC 130 FR 40 PA 90/70 Cuello con herida en zona II, con
hematoma. Pulmones con ruidos respiratorios ausentes en hemitórax derecho. En cuanto a la
Reanimación el paciente necesita :
A. Transfusión masiva
B. Realizar hemograma completo, creatinina, electrolitos y tiempos de coagulación
C. Transfundir plasma fresco congelado si los tiempos de coagulación están alterados
D. Evaluación de gases arteriales y Tromboelastografía para determinar transfusión
E. Dos canalizaciones y lactato ringer a chorro en cada una.

85. Dentro de los tumores de las células germinales


A. El teratoma produce muchos síntomas a pesar de ser benigno.
B. Los seminomas son los más comunes dentro de los tumores de células germinales y son
más frecuentes en mujeres..
C. Los no seminomatosos característicamente tienen los marcadores tumorales
elevados.
D. Se ubican por igual en cualquiera de los compartimentos del mediastino.

86. Paciente femenina de 40 años se presenta al cuarto de urgencias con 2 dias de dolor
abdominal moderado acompañado de náuseas y vómitos. No ha presentado evacuación o
pasado flatus desde hace 4 días. Su historia médica es relevante por síndrome de dolor crónico
para lo que toma opioides regularmente. Sus signos vitales son normales. Al examen, está en
leve distres, su abdomen está distendido y está doloroso difusamente a la palpación, ruidos
hidroaéreos están ausentes. No tiene ni defensa ni rebote. Hemograma, urinàlisis y panel
metabólico y otros laboratorios no presentan anormalidades. Radiografía de abdomen supino y
de pie revelan distensión general del intestino delgado y colon, hay aire a través de las asas.
Cuál de las siguientes es el diagnóstico más probable?
A. Apendicitis aguda
B. Colecistitis aguda
C. Obstrucción intestinal completa
D. Íleo paralítico
E. Obstrucción parcial de intestino delgado

87. ¿Qué hallazgo de imagen excluiría la apendicitis?


A. Una tomografía computarizada (TC) con un apéndice no visualizado.
B. Un enema de bario donde se identificó claramente un apéndice corto (2 cm).
C. Un estudio de ultrasonido con un apéndice compresible de menos de 5 mm de
diámetro.
D. Una tomografía computarizada que muestra un apéndice edematoso pero retrocecal.

88. En el neumotórax espontáneo:


A. La aparición de nuevos episodios es poco frecuente
B. Generalmente se produce en pacientes altos, delgados y jóvenes
C. El neumotórax catamenial se ve con más frecuencia en pacientes posmenopáusicas.
D. No se produce neumotórax a tensión

89. Un hombre de 44 años aparece en el servicio de urgencias a las 11 pm con un exquisito


dolor perianal. Él no puede sentarse, informa que las evacuaciones intestinales son muy
dolorosas, y ha tenido escalofríos y fiebre. El examen físico muestra una masa caliente,
sensible, roja y fluctuante entre el ano y la tuberosidad isquiática.
A. Absceso perianal
B. Hemorroides trombosada abscedada
C. Absceso isquiorrectal
D. Apendicitis retrocecal
E. Fístula perianal abscedada

90. Una mujer de 69 años tiene una masa dura de 4 cm en el seno derecho con bordes mal
definidos, movible desde la pared del pecho pero no movible dentro del seno. La piel que
recubre la masa se retrae y tiene una apariencia de “piel de naranja”.
A. Estas son todas presentaciones clásicas de cáncer de mama.
B. Las masas duras son probablemente adenocarcinoma de mama invasivo.
C. La piel roja y naranja es probablemente un cáncer de mama inflamatorio.
D. La lesión areolar del eccematoide es probablemente la enfermedad de Paget del seno
(una forma rara de cáncer de mama).

91. Para realizar el diagnóstico de shock en un paciente:


A. Presión sistólica menor de 90
B. Debe excluirse lesión intracerebral
C. Debe hacer acidosis en los gases arteriales
D. El paciente no responde a la administración de líquidos intravenosos
E. Debe haber signos clínicos de perfusión inadecuada de órganos

92. Una mujer de 72 años tiene un pecho rojo e hinchado. La piel sobre el área se parece a la
cáscara de naranja y es discutible si el área está caliente o no. Ella no tiene fiebre o leucocitosis.
A. Estas son todas presentaciones clásicas de cáncer de mama.
B. Las masas duras son probablemente adenocarcinoma de mama invasivo.
C. La piel roja y naranja es probablemente un cáncer de mama inflamatorio.
D. La lesión areolar del eccematoide es probablemente la enfermedad de Paget del seno
(una forma rara de cáncer de mama).

93. Los siguientes son síntomas comunes del dumping, excepto:


A. Diaforesis
B. Taquicardia
C. Debilidad/mareos
D. Vómitos biliosos

94. En una obstrucción intestinal de 12 horas de evolución encontraremos todo lo anterior.


EXCEPTO:
A. Hiperpotasemia
B. Hiponatremia
C. Hipocloremia
D. Oliguria

95. En la etiología de un cuadro de “abdomen agudo” NO estaría uno de los siguientes


procesos:
A. Hemorragia digestiva por úlcera duodenal
B. Apendicitis aguda
C. Obstrucción intestinal por cáncer de sigma
D. Diverticulitis de sigma perforada
E. Colecistitis aguda
!
2PT%CIRUGIA.!MED6!
!
4 a 5 días

6 horas

!
!
!
!
!

!
!
!
!
!
!
!
!
!
!
!
!
!
!
!
!
!
!
!
!
!
!
!
!
!
!
!
!
!
!
!
!
!
!
!
!
!
!
!
!
!
!
!
!
!
!
!
!
!
!
!
!
By: Donnie

80) Un varón diabético de


años
64 es sometido a hemicolectomía derecha por un adenocarcinoma del ciego. En la primera
noche del postoperatorio prese dia y en las últimas 8 horas fiebre de 39,3”C. Los únicos datos anormales
en la exploración física incluye o y secreción de color oscuro. CNO más apropiado es:
a. Administración de antibióticos de amplio espectro, paracetamol y mantas enfriadoras.
b. Abrir la herida e iniciar tratamiento con oxígeno hiperbárico.
Cc. Aplicar compresas tibias estériles sobre la incisión y cambiar el apósito.
d. Abrirla herida y enviar muestras para tinción de gram y realizar debridamiento amplio.
9
9 Un varón de/62años/de edad con nutrición parenteral total (TPN) que se ini
fístula enterocutánea postoperatoria desarrolló fiebre alta en espigas hasta de 39*C en las últimas 8 horas. El único dato
po expxloración física es la presencia de eritema e induración alrededor del catéter central. ¿Cuál es el
más apropiado?
a. Administración de antibióticos de amplio espectro y observación por 24 horas.
b. Solicitar cultivos de sangre obtenida a través del catéter, iniciar antibióticos de amplio espectro y esperar los
resultados de los cultivos.
Retirar el catéter, enviar la punta para cultivo y colocar un nuevo catéter sobre el alambre guía.
. Retirar el catéter, enviar la punta para cultivo y colocar el catéter en otro sitio.
10
1 8 Las hemorroides internas grado Ill son:
a) Las que no se reducen
b) Las que se reducen manualmente
c) Las que sangran
d) Las que se reducen espontáneamente

11
44 Laindicación más común para cirugía en un paciente con:enfermedad de Crohn:
a. Carcinoma
b. Hemorragia
c. Obstrucción
d. Absceso

12
1 2 Un varón de 26 años de edad acude con dolor abdominal y diarrea sanguinolenta. A la exploración física se observa
febrículay/dolormoderado a la palpación. Se realiza colonoscopía, la cual revela mucosa edematosa con
afectación contigua desde el recto al colon'izquierdo. Se realizan biopsias aleatorias que muestran inflamación
aguda y crónica de la mucosa y submucosa con múltiples abscesos en las criptas. No se observan granulomas. ¿Cuál de
las siguientes aseveraciones es verdadera con respecto al diagnóstico?
a. El paciente necesita cirugía
b. El uso de corticoesteroides IV está contraindicado
C. Las fístulas perianales son características de esta enfermedad
d Hay un incremento sustancial del riesgo a largo plazo de desarrollar cáncer de colon

13
1 3 Paciente de 25 años quien es operada delaparotomía.exploradora por una gran apendicitis aguda. Al día siguiente la enfermera
reporta que el paciente tiene 38 grados de temperatura y se queja de dolor abdominal 5/10:

a. Infección de vías urinarias

b. Infección por Clostridium perfinges

£.Atelectasia

d. Fiebre de origen por determinar

14 1 P Usted como estudiante de medicina, para comprobar su diagnóstico debe:

a. Urinalisis y urocultivo

b. Coprocultivo

c. Radi rafía de Tórax

d. Tomografía axial Computarizada

15
1 a) Para evitar esta complicación usted debe: C

a. Retirar pronto el catéter urinario

b. Dar antibióticos profilácticos.

c. Ejercicios Respiratorios y analgésicos

d. Se debe realizar apendicectomía laparoscópica.

16
1 (A manera de reducir la infección del sitio quirúrgico es: B

a. Antibioticos profilacticos
B

B
26
27

28

29

30

31

32

33
34
35

36

37

38

39

40

41

42
43
A

B
1 ¿Cuál de las siguientes afirmaciones es verdadera con respecto a la profilaxis del
tromboembolismo venoso (TEV) en pacientes quirúrgicos?

2 Paciente de 47 años, sin antecedentes patológicos de interés. Presenta un cuadro de fiebre y


dolor en punta de costado derecho y ha sido diagnosticado de neumonía adquirida en la
comunidad en su Centro de Salud. En una radiografía simple de control, realizada 3 días
después del inicio del tratamiento antibiótico, se objetiva un derrame pleural ipsilateral que
ocupa un tercio del hemitórax derecho. Señale la respuesta correcta:
3 Varón de 35 años, trasladado a urgencias tras un accidente de tráfico. A su ingreso se observa
una puntuación en la escala de Glasgow de 15, tensión arterial de 140/90 mmHg, frecuencia
respiratoria de 35 rpm y frecuencia cardiaca de 110 lpm, con una saturación de oxígeno basal
del 91 %. En la exploración hay hipofonesis torácica derecha y timpanismo a la percusión. ¿Cuál
es el diagnóstico de presunción? :

4 ¿Cuál de las siguientes modalidades es MENOS probable que ayude en la prevención de


complicaciones pulmonares posoperatorias en un hombre fumador de 65 años?
E
5 ¿Cuál es el momento más apropiado para la administración de la profilaxis antibiótica en la
cirugía abdominal?:
en los 30 minutos previos

6 Cinco días después de la cirugía por apendicitis perforada, las heces líquidas emanan de la
herida del cuadrante inferior derecho. ¿Cuál de las siguientes afirmaciones es verdadera sobre
esta afección?
7 na mujer de 42 años con lupus eritematoso sistémico (LES) de larga duración complicado por
nefritis lúpica y artritis debilitante está en la UCI después de una resección intestinal de urgencia
4 días antes. Durante las siguientes horas, se vuelve febril, hipotensa y se queja de dolor
abdominal. Se le administran bolos de líquidos, pero la presión arterial no responde. Su examen
abdominal es normal. Los valores de laboratorio revelan un recuento de leucocitos de 12.000
células / L con eosinofilia, Na sérico de 133 mEq / L, bicarbonato sérico de 20 mEq / L y K sérico
de 5,3 mEq / L. ¿Cuál de los siguientes representa el mejor manejo de esta afección?
laparotomia exploratoria
8 Pregunta vinculada a la imagen Joven de 16 años sin antecedentes previos, que es traído a
urgencias por sus familiares por dificultad respiratoria aguda y dolor torácico. A su llegada
muestra intenso trabajo respiratorio, frecuencia cardiaca 120 lpm, tensión arterial 75/40 mmHg y
SatO2 86 % con aire ambiente. Considerando la radiografía de tórax, ¿cuál es la medida a
tomar más apropiada?:
realizar tc toracica

9.Pregunta relacionada con la foto. Indique en qué localización se encuentra la lesión:


10. Mujer de 27 años con tos no productiva, fiebre y dolor pleurítico de un mes de evolución. En
la radiografía de tórax se objetiva un derrame pleural izquierdo. El análisis del líquido pleural
muestra un exudado linfocítico con pH 7,32, glucosa 66 mg/dL y adenosin-deaminasa 59 U/L.
¿Cuál es la etiología más probable?:
11. Durante una anestesia general la temperatura del paciente ha ido aumentando
progresivamente y usted llega a la conclusión de que sufre un cuadro de hipertermia maligna.
Referente a esta complicación, una de las siguientes afirmaciones es cierta:
R; A,, esta producida por el uso de anestesicos halogenos

12. Pregunta vinculada a la imagen Hombre de 56 años, fumador activo, no hipertenso ni


dislipémico, que consulta porque hace 7 días presentó malestar general con dolor centrotorácico
y disnea de esfuerzo, encontrándose asintomático en el momento de la consulta. ¿Cuál es el
diagnóstico más probable, según el trazado electrocardiográfico?:
R:Necrosis
13. ¿Cuál de los siguientes estudios preoperatorios está más fuertemente asociado con un
mayor riesgo de complicaciones posoperatorias relacionadas con el pulmón?
R: Espirometria
14. ¿Cuál de las siguientes afirmaciones es verdadera sobre la cicatrización de heridas?
R; A las 48 horas.

15. La imagen de TC sin administración de contraste es característica de:


16. Pregunta vinculada a la imagen Hombre de 56 años con antecedentes de hipertensión
arterial y cáncer de colon con metástasis hepáticas, actualmente en tratamiento quimioterápico.
Acude a urgencias por disnea. Constantes: frecuencia cardiaca 110 lpm, tensión arterial 115/55
mmHg, frecuencia respiratoria 29 rpm, SatO2 89 %. Exploración: uso de musculatura
respiratoria accesoria, murmullo vesicular conservado en todos los campos. La primera prueba
diagnóstica de la que se dispone es el electrocardiograma. ¿Cuál es la sospecha clínica más
probable?:
R: C

17. Un paciente en la unidad de cuidados intensivos quirúrgicos comenzó con alimentación


enteral en bolo hace aproximadamente 3 horas y ahora tiene dificultad respiratoria significativa.
Recientemente no se administraron medicamentos y el personal de enfermería no notó ningún
evento significativo. Después de la intubación, una gasometría arterial muestra una PaO2 de 70
mm Hg con una FiO2 del 75%. ¿Cuál de las siguientes afirmaciones es verdadera sobre esta
afección?

R: C
18. Varón diabético de 76 años ingresado en la unidad de cuidados intensivos quirúrgicos tras
una caída. Sus lesiones incluyen una fractura de cuello femoral derecho y hemorragia
subaracnoidea. Continúa teniendo elevación intermitente de la presión intracraneal y aún
necesita asistencia respiratoria después de 2 días. ¿Cuál de las siguientes afirmaciones es
verdadera con respecto a la suplementación nutricional en este paciente?

R: D

19 La radioterapia en el cáncer de mama NO está indicada en:


R: B

20 Hombre de 42 años con antecedente de cirrosis hepática compensada, traído a urgencias


por ictericia, fiebre, aumento del perímetro abdominal y deterioro significativo del estado general.
¿Cuál de los siguientes parámetros analíticos NO le aportaría información acerca del
pronóstico?:
R: C

21 Paciente de 40 años, con exploración física normal, en el que se ha puesto en evidencia en


la tomografía computarizada una masa mediastínica y un nódulo hepático, y en la analítica una
elevación sérica de lácticodeshidrogenasa (LDH), betagonadotropina coriónica humana
(beta-GCH) y alfafetoproteína. ¿Cuál es su sospecha diagnóstica?:
22 Un hombre de 45 años se presenta con náuseas, vómitos y disnea. La historia pasada es
significativa por una colisión de vehículo de motor 5 años antes por la cual estuvo hospitalizado
durante una semana, sin requerir cirugía. Hay disminución de los ruidos respiratorios en el
hemitorax izquierdo. La radiografía de tórax (CXR) muestra un hemidiafragma izquierdo elevado,
con borramiento del ángulo costofrénico, así como múltiples burbujas de gas en la parte inferior
izquierda del tórax. El recuento de leucocitos es de 18.000 células / μL. ¿Cuál de las siguientes
es la mejor recomendación de manejo?

23 Una mujer de 28 años se somete a adhesiolisis (lisis de adherencias)por una obstrucción


aguda del intestino delgado. Durante el curso de la cirugía, requiere una resección ileal
segmentaria con anastomosis primaria. En el sexto día posoperatorio, se observa que tiene un
líquido espeso de color biliar que emana de la línea media de la herida. Después de la
hidratación intravenosa, el siguiente paso en el manejo debe ser:
24 Una mujer de 22 años presenta una parada cardiaca súbita con un ritmo que se considera
susceptible de desfibrilación. Se están realizando compresiones torácicas y ventilaciones, se ha
dado una descarga con el desfibrilador y se ha canalizado una vía venosa periférica. ¿Cuál de
las siguientes actuaciones sería correcta a continuación?:

25) Para conocer si el uso habitual de ácido acetilsalicílico se asocia a un mayor riesgo de
hipertensión se selecciona un grupo de sujetos, se averigua cuántos están tomando ácido
acetilsalicílico y se les sigue durante 5 años para identificar los casos nuevos de hipertensión.
Señale la respuesta correcta sobre el diseño de este estudio
R: COHORTES
26 Hombre de 57 años que acude a urgencias por un dolor epigástrico de elevada intensidad,
irradiado a espalda y a hipocondrio derecho, junto a náuseas y vómitos de 12 horas de
evolución. En la exploración se encuentra afebril, estable hemodinámicamente, lúcido y con
buena perfusión. En la analítica destacan 18.000 leucocitos (80% neutrófilos), siendo el resto del
hemograma normal. La bioquímica es normal, incluyendo calcio, LDH y triglicéridos, excepto una
amilasa de 3.000 U/L. Señale la respuesta correcta:
R. Sugiere una colamgitis

27 ¿En cuál de las siguientes situaciones puede aparecer una acidosis metabólica sin anion gap
elevado?:
28 Un hombre de 18 años llega al servicio de urgencias del hospital trasladado por una unidad
de soporte vital básico. Ha sido atropellado por un automóvil cuando iba en bicicleta. A su
llegada tiene una frecuencia cardíaca de 115 lpm, una presión arterial de 110/75 mmHg, pulso
radial palpable, un relleno capilar de 2,5 segundos y una frecuencia respiratoria de 25 rpm.
¿Qué nos indican estos datos?:

29 En un paciente con cáncer de pulmón sin comorbilidad, la combinación de quimioterapia y


radioterapia es el tratamiento de primera elección en:
R. Cancer de célula pequeña
30 ¿ucon respecto al uso de betabloqueantes en el período perioperatorio para pacientes
sometidos a cirugía no cardíaca?

31 Un hombre de 35 años es trasladado al servicio de urgencias tras ser apuñalado en el


abdomen derecho. Se queja de dolor abdominal mínimo sin rebote ni defensa. Durante la
exploración local de la herida (LWE), parece que no se viola la fascia anterior. Sus signos vitales
son normales. ¿Cuál de los siguientes es el manejo más apropiado? C
32 Hombre de 57 años, exfumador, con antecedentes de pancreatitis aguda alitiásica, gastritis
crónica por AINES y disfunción eréctil en tratamiento con pantoprazol y sildenafilo. Consulta por
dolor torácico opresivo a las 7:30 h a.m. mientras se encontraba en reposo. PA 148/82 mmHg,
FC 85 lpm, Sat 98% basal. AC: rítmico, sin soplos. AP: murmullo vesicular conservado. No
edemas, pulsos presentes y simétricos. Se le realiza un ECG de superficie (imagen). ¿Cuál de
los siguientes fármacos está contraindicado en este paciente? c
34 ¿Cuál de las siguientes afirmaciones es verdadera con respecto a las Pruebas de Función
Pulmonar(PFT)? a

35 Un hombre de 29 años llega al servicio de urgencias después de una colision de vehiculo de


alta velocidad con una escala de coma de Glasgow (GCS) de 4. Tiene un collarín cervical que le
colocaron los servicios médicos de emergencia (EMS). Se lo intuba y se le realiza una
tomografía computarizada, que muestra una hemorragia subdural grande y una hemorragia
punteada difusa sin evidencia de lesión de la columna cervical. Está ingresado en la UCI. Con
respecto al manejo del collarín cervical, ¿cuál de los siguientes se recomienda? d
36 De los cinco momentos recomendados en la higiene de manos, señale la respuesta
INCORRECTA:
r. antess del contacto con el entorno

37 Un hombre de 67 años que se recupera de una exenteración pélvica secundaria a un cáncer


de recto localmente avanzado se inicia con nutrición parenteral total para el íleo prolongado a
través de un catéter central insertado periféricamente en el lado derecho (PICC). Varios días
después, la enfermera informa que no puede extraer sangre de su línea PICC. El brazo está
hinchado. La ecografía confirma un coágulo en las venas basílica y axilar. ¿Cuál es el manejo
adecuado de su condición?
r.compresas calientes
38 En este corte ecográfico longitudinal de línea axilar anterior derecha, ¿qué estructura NO se
identifica?
39 Hombre de 60 años con un cuadro de dolor abdominal y sangrado digestivo alto, en quien se
encuentra una tumoración abdominal relacionada con la pared gástrica. La histología muestra
un tumor de células fusiformes con escasas mitosis, positivas para CD117. El diagnóstico más
probable es:
tumor del estrom

40 En relación con las proteínas plasmáticas, señale la afirmación INCORRECTA:


41 Hombre de 73 años con antecedente de infarto agudo de miocardio tratado con angioplastia
coronaria transluminal percutánea hace 6 semanas. Acude a urgencias por dolor brusco, frialdad
y pérdida de motilidad de la extremidad inferior derecha. En la exploración está presente el pulso
femoral derecho, con ausencia del resto de los pulsos de esa extremidad. Conserva pulsos a
todos los niveles en la extremidad contralateral. Respecto al diagnóstico de sospecha, señale la
afirmación FALSA:
A. En caos de paralisis ...

42 Pregunta vinculada a la imagen Hombre de 37 años no fumador y sin antecedentes médicos


de interés que presenta un cuadro de más de tres semanas de evolución de tos seca
persistente, febrícula y dolores articulares erráticos. Entre las pruebas que le solicita su médico
se incluye una radiografía de tórax. ¿Cuál de los siguientes diagnósticos sospecha en primer
lugar?:
43 Hombre de 60 años, fumador de 40 paquetes/año, que consulta por dolor en borde cubital del
brazo izquierdo y ptosis palpebral izquierda desde hace un mes. La radiografía de tórax muestra
una masa en el lóbulo superior izquierdo y la TC confirma la lesión con invasión de la segunda
costilla. En relación con el diagnóstico de sospecha, el tratamiento más adecuado es:

44 ¿Cuáles de las siguientes microfotografías teñidas con hematoxilina-eosina corresponden a


mucosa de intestino delgado? NOSEEEE
45 ¿Cuál de las siguientes afirmaciones es verdadera con respecto a las heridas del pie
diabético?
C. La mayoria se debe a isquemica

El vasopresor de elección para mantener la presión de perfusión en un enfermo con shock


séptico es:
C. Noradrenalina
Ante un paciente con pericarditis aguda y derrame pericárdico grave ¿cuál de las siguientes
medidas es INCORRECTA?:

n paciente de 69 años con un tumor en la unión rectosigmoidea se somete a colectomía


sigmoidea laparoscópica. El dolor posoperatorio se controla bien con anestesia epidural torácica
controlada por el paciente. El día 1 del postoperatorio se inicia la anticoagulación profiláctica con
heparina de bajo peso molecular (HBPM). Se está drenando la vejiga con un catéter de Foley
permanente. ¿Cuál de las siguientes afirmaciones es verdadera con respecto a la anestesia
epidural?
Un hombre de 45 años es llevado al servicio de urgencias después de una herida por proyectil
de arma de fuego en la pierna derecha. Está hipotenso en el servicio de urgencias, con una gran
pérdida de sangre en la escena. Se inicia el protocolo de transfusión masiva. Después de un
injerto de vena de interposición para una sección de la arteria femoral superficial, es ingresado
en la UCI para observación. A la mañana siguiente, se encuentra oligúrico, tiene presiones
máximas en las vías respiratorias en aumento y tiene el abdomen distendido. ¿Cuál de los
siguientes se esperaría en este paciente?
ombre de 35 años, ingresado por traumatismo torácico grave con múltiples fracturas costales.
Tras responder favorablemente al tratamiento con analgésicos y oxígeno, comienza a presentar
hipoxemia grave. Señale cuál es la causa más probable de este deterioro:
B

¿Cuál de las siguientes enfermedades se relaciona correctamente con el tipo de colágeno


afectado?b
OA. Síndrome de Alport: colágeno tipo !!!

OB. b. Síndrome de Ehlers-Danlos: colágeno tipo VII

Oc. Epidermólisis ampollosa: colágeno tipo VII

Osteogénesis imperfecta: colágeno tipo Il

Penfigoide ampolloso: colágeno tipo |


o
1° PARCIAL CIRUGIA MED12 Topitos

Paciente masculino de 28 años, ingresa al hospital con quemaduras de 50% de superficie


corporal quemada por explosión de tanque de gas, hace 3 horas. Presentando FC: 100 x min
PA: 80/50 Hipotérmico, Lactato: 4.6mmol/L, diuresis: 20cc/h. Cuál es la correcta?: fase
hipometabolica.
La indicación quirúrgica de un quiste de colédoco se basa enPOTENCIAL DE MALIGNIDAD

De las glándulas gástricas, cuál tipo de células es la que produce histamina: enterocromafin.

La mejor estrategia para modular la repuesta inflamatoria en el trauma es: todas

Manejo de un paciente con colelitiasis asintomática: recomendaciones

La citoquina que modula la respuesta metabólica es: Factor de necrosis tumoral

Cuáles son las indicaciones más frecuentes para realizar intubación endotraqueal? TODAS

Las colecistopatías más frecuentemente son confundidas con: gastritis

La respuesta metabólica en el ayuno es diferente a la del trauma; Una diferencia es que el


ayuno encontramos excepto: neoglucogenesis aumentada

Una mujer de 45 años se presenta con dolor en el CID. Se realiza una tomografía
computarizada. ¿Qué hallazgo en la tomografía computarizada es el más sugerente de linfoma
apendicular?
A. DIÁMETRO APENDICULAR >2.5 cm O ENGROSAMIENTO DEL TEJIDO BLANDO
CIRCULANTE

El intestino juega un papel importante como defensa interna del cuerpo. El principal combustible
del enterocito es: GLUTAMINA

¿Qué hallazgo de imagen excluiría la apendicitis? UN ESTUDIO DE USG CON APÉNDICE


COMPRESIBLE DE MENOS DE 5 mm DE DIÁMETRO

Paciente de 70 años con presencia de pólipo vesicular de 12 mm sintomático, su manejo


adecuado. colescistectomia.

En la Evaluación del APACHE II, que variables son las que se consideran para obtener el
puntaje Edad + 12 variables fisiológicas + Enf previa

Rama de la arteria gastroduodenal que irriga la porción derecha de la curvatura mayor del
estómago:gastroepiploica derecha

De las glándulas gástricas, cuál tipo de células es la que produce histamina: celulas
enterocromafines
Respecto al concepto de sedación lo siguiente es cierto EXCEPTO: cuando px esta sedado esta
en estado de analgesia

El órgano inraabdominal que mayormente se lesiona en el trauma abdominal penetrante:


intestino delgado

Respecto a las heridas lo siguiente es cierto EXCEPTO una herida de “scalp” es por
arrancamiento en la pared torácica

Respecto a la cicatrización de las heridas lo siguiente es cierto EXCEPTO px con corticoides

Femenina de 28 años, diabética, obesa, es traída por su esposo con dolor severo 9/10 en
hipocondrio derecho. Al examen físico, Murphy positivo y febril (T 38.2). Resto de signos vitales
estables. USG abdominal colecistitis aguda. ¿Cuál es su plan de manejo a continuación? NPO,
LÍQUIDOS ENDOVENOSOS, ANALGESIA Y COLECISTECTOMÍA EN MENOS DE 72 HORAS

Las heridas por proyectil de armas militares tiene la siguientes características excepto: son de
baja velocidad

Respecto a los fármacos empleados en anestesia general señalar si las afirmaciones siguientes
son verdaderas excepto: LA INTUBACIÓN ENDOTRAQUEAL SE REALIZA SIEMPRE ANTES
DE ADMINISTRAR LOS FÁRMACOS HIPNÓTICOS Y RELAJANTES MUSCULARES

Hormona de la contrarregulacion anabólica INSULINA

Paciente masculino de 58 años presenta sepsis abdominal importante por perforación de colon,
luego se complica con ARDS. Cuales son las características del ARDS ? todas

La respuesta del anabolismo al trauma produce lo siguiente excepto: HORMONAS TIROIDEAS

Los pacientes en UCI pueden exhibir diferentes tipos de respuesta, cuando se presenta una
respuesta equilibrada entre la proinflamación y la antinflamacion y predomina la hemostasia. Se
le llama: MARS

En la cicatrización ocurre lo siguiente EXCEPTO: LA FASE INFLAMATORIA COMIENZA A LOS


TRES O CUATRO DÍAS DESPUÉS DE PRODUCIRSE LA HERIDA

Paciente masculino de 38 años acude a consulta con historia de dolor intermitente, no


incapacitante, en hipocondrio derecho. Al examen físico signos vitales estables, Murphy
negativo. Laboratorios normales. USG abdominal revela pólipo en vesícula biliar de 1.5 cm sin
datos de colecistitis. ¿Cuál es el siguiente paso recomendado en el manejo? colecistectomia
laparoscopica electiva

Le consultan de Cuidados Intensivos al equipo de Cirugia General por paciente masculino de 26


años admitido por quemaduras severas y sepsis secundaria que lleva 2 semanas en UCI. Ante
la persistencia de glóbulos blancos elevados e intolerancia a la dieta enteral le realizan USG
abdominal que reporta distensión de la vesícula biliar con engrosamiento de sus paredes en 5
mm. ¿Cuál es su diagnóstico más probable en este caso?: Colecistitis aguda acalculosa
El sodio no debe ser corregido rápidamente por el riego de Mielinosis Pontina Cerebral. La
velocidad de reposición sodio debe ser no mayor por hora de: 20 meq

Un paciente de 20 años que presenta colelitiasis, ahora se asocia con ictericia y coluria. Que
estudio ayudaría a su diagnóstico: colangioresonancia

En la fase inflamatoria del proceso de cicatrización las principales células responsables del
Factor de Crecimiento para las células endoteliales y la angiogénesis son: macrofagos

Una de las siguientes afirmaciones sobre la ventilación mecánica (VM) y la intubación es


FALSA: sistema de intercambio de gases

Paciente de 39 años, diabética, obesa, hipertensa, acude a Urgencias con disuria y dolor en
bajo vientre. Urinalisis con datos de infección de vías urinarias. Hemograma sin leucocitosis. El
plan es egreso con antibióticos ambulatorios, pero en el resultado de USG abdominal ordenado
se reporta Colelitiasis. La paciente no tiene ni ha tenido dolor en hipocondrio derecho ni datos de
dispepsia. Basado en esto, ¿Cuál es su recomendación de manejo?: ningun manejo adicional

La mejor manera de disminuir las muertes por trauma en el primer pico de mortalidad es:
divulgar medidas

La características de las heridas por arma blanca son: baja energia

Sr Eduardo ingresa a UCI con Gangrena de Fournier, shock séptico asociado a Diabetes
descompensada. Presenta un nivel de Procalcitonina(PCT) de 10ng/dl. Cuál es falsa? Altamente
positivo en infecciones virales

Todos los siguientes se consideran una vía aérea definitiva excepto: . CRICOTIROIDECTOMÍA
POR AGUJA

En shock séptico cuál es el vasopresor de primera elección: norepinefrina

La características de una vía aérea difícil esta dada por: TODAS

En Febrero de 2016 la ESICM y la SCCM publican nuevos consensos de sepsis relacionados a


criterios clínicos eliminando los términos SRIS y Sepsis Severa. Adoptándose nuevas
definiciones:
SEPSIS: DISFUNCIÓN ORGÁNICA QUE AMENAZA LA VIDA CAUSADA POR
UNA DISREGULACIÓN EN LA RESPUESTA DEL HUÉSPED A LA INFECCIÓN

La principal causa de muerte en el trauma es: No tener via aerea permeable

Un paciente de 85 años fue operado por divertículo perforado, la cirugía fue prolongada, se deja
el abdomen abierto, en ventilador y se pasa a UCI. Son todas causas de aumento de pérdidas
sensibles excepto: taquicardia

Las siguientes lesiones pueden causar la muerte entre minutos y horas excepto: LESION
COMPLETA MEDULAR A NIVEL C4
El sustrato para producir oxido nitroso es: arginina

Luego de una cena, acude a Urgencia paciente de 42 años con dolor intermitente en hipocondrio
derecho. Signos vitales estables. Murphy dudoso. Laboratorios sin alteraciones. USG abdominal
con litos en la vesícula. ¿Cuál es la causa más probable de su dolor?
distencion de la vesicula
Cuál de las siguientes situaciones de entre las que se consiguen durante una anestesia general
es FALSA: mantenimiento de los nociceptivos

Paciente de 30 años su automóvil choca contra un poste de luz, provocando un trauma torácico
cerrado importante, es trasladado por sus amigos al cuarto de urgencia. Al examen físico esta
quejumbroso, Con PA de 80/40, FC 120 x, FR 32x. sat 02 91%. Tiene ingurgitación yugular
disminución de los movimientos del tórax del lado derecho, ruidos respiratorios e
hiperresonancia y los ruidos cardíacos se auscultan taquicárdicos. El diagnóstico más probable
de este paciente es:
Neumotorax a tension

Paciente de 30 años su automóvil choca contra un poste de luz, provocando un trauma torácico
cerrado importante, es trasladado por sus amigos al cuarto de urgencia. Al examen físico esta
quejumbroso, Con PA de 80/40, FC 120 x, FR 32x. sat 02 91%. Tiene ingurgitación yugular
disminución de los movimientos del tórax del lado derecho, ruidos respiratorios e
hiperresonancia y los ruidos cardíacos se auscultan taquicárdicos. Para mejorar la oxigenación
del paciente debemos: COLOCAR MÁSCARA FACIAL CON RESERVORIO A FIO AL 100% Y
FLUJO 10 LPM

La piedra angular en el manejo de la colangitis es drenaje de la via biliar

Paciente operado de apendicitis aguda no complicada pesa 80 Kg. y mide 1.70 cm la cantidad
de agua que le corresponde en 24 horas es de: 2000

El patrón de trauma automovilístico posterior produce comunmente lesiones: cervicales

Señalar el puesto correcto para cada una de las actuaciones siguientes en la secuencia de la
inducción anestesica: MONITORIZACIÓN, ACCESO VENOSO, PREOXIGENACIÓN,
FÁRMACOS,
INTUBACIÓN

Son complicaciones de las úlceras pépticas EXCEPTO: dolor

Mientras revisa la patología de una apendicectomía laparoscópica reciente, observa que


además de la apendicitis aguda, el paciente tenía un tumor carcinoide de 1,5 cm ubicado en la
base del apéndice. El paciente está sano y se está recuperando bien de la cirugía. ¿Qué
recomendarías? Hemicolectomía derecha

Las células Principales del estómago secretan: Pepsina

Al colangiocarcinoma que se presenta en la confluencia de los radicales hepáticos se le conoce


como:
C. TUMOR DE KLATSKIN

Respecto a la cicatrización de las heridas lo siguiente es cierto EXCEPTO: los px con


tratamiento corticoideo tienen mejor cicatrizacion

Una mujer de 29 años presenta dolor en el CID, fiebre y leucocitosis. Antes de los estudios de
imagen, la puntuación de Alvarado se utiliza para determinar la probabilidad de que el paciente
tenga apendicitis. Todas las siguientes variables conforman la puntuación de Alvarado
EXCEPTO SIGNO DE ILIOPSOAS

Una mujer joven con antecedente de enfermedad convulsiva presenta convulsiones tónico
clónicas en la sala de urgencias. Se encuentra letárgica y a la exploración física no hay
focalización neurológica. Sus gases en sangre arterial muestran pH de 7.12, dióxido de carbono
de 48, Po2 de 86 y bicarbonato calculado de 16. ¿Cómo se clasificaría el trastorno acido
básico? acidosis mixta

En un paciente que presenta dolor recurrente en hipocondrio derecho, ¿qué estudio de imagen
nos ayudaría para el diagnóstico?. ULTRASONIDO

En la evaluación secundaria del trauma todo los siguiente es cierto excepto: LA DE PTES
INESTABLES

En un caso de hidrosadenitis supurada de la ingle el mejor tratamiento después de la excisión


de la lesión es: cierre por segunda intencion

Paciente con colangitis y deterioro neurológico, manejo adecuado:CPRE

El sodio no debe ser corregido rápidamente por el riego de Mielinosis Pontina Cerebral. La
velocidad de reposición sodio debe ser no mayor por hora de: 20meq

¿Cuál de los siguientes signos físicos se asocia con la definición correcta que sugiere
apendicitis aguda? signo de rovsing

Masculino de 66 años de edad tos productiva desde hace 8 días, debilidad general, dificultad
para respirar FR: 25 con PaO2: 40 recibiendo FIO2: 21% (190). Plaquetas: 200,000 Bilirrubinas:
1.0 PA: 88/45 Presión Arteria Media 59, Escala de Glasgow: 13, Creatinina: 2.5 TPT: 60seg
Cuántos criterios de Falla Orgánica (SOFA) presenta: Cuatro

Sr Oscar sufre atropello por un vehículo al cruzar la calle, múltiples traumas incluyendo Trauma
Craneoncefálico, presentando: FC: 96 PA: 127/68 SatO2: 99% No abre los ojos ni al dolor,
flexión anormal de extremidades y al interrogatorio esta confuso. Que nivel de Glasgow tendrá y
que hacemos? galsgow 7
La características de las heridas por arma blanca son: baja energia

La metas del tratamiento inicial en el trauma son: TODAS LAS ANTERIORES (son las mismas)

Luego de una cena, acude a Urgencia paciente de 42 años con dolor


intermitente en hipocondrio derecho. Signos vitales estables. Murphy dudoso.
Laboratorios sin alteraciones. USG abdominal con litos en la vesícula. ¿Cuál es
la causa más probable de su dolor?
A. DISTENSIÓN DE LA VESÍCULA BILIAR

En las siguientes sentencias sobre los procedimientos anestésicos, todas son verdaderas
EXCEPTO: anestesia local periferica….mama

Sra. Carlota presenta shock hipovolémico con FC: 110 PA:80/54 FR:21 Sat: O2: 95%. Cuál es la
mejor forma para administrar rápidamente mayores volumen de líquido ? atraves de cateter
periferico ya que mas corto mas flujo

Un hombre de 25 años de edad presenta dolor migratorio en el cuadrante inferior derecho (CID),
leucocitosis y una tomografía computarizada compatible con apendicitis aguda y sin
complicaciones. Es fisiológicamente normal y son las 2 de la mañana. Ud. está planeando una
apendicectomía, ¿qué diferencia podría esperarse en su resultado si su operación se retrasa
hasta la mañana siguiente? no hay diferencia en las tasas de perforación, infección del sitio
quirurgico, absceso, tasa de conversion o tiempo operatorio

Manejo de un paciente con colelitiasis asintomática: recomendaciones

La indicación quirúrgica de un quiste de colédoco se basa en: pOTENCIAL DE MALIGNIDAD

Una mujer de 23 años que está embarazada de 28 semanas presenta dolor abdominal en el
lado derecho, leucocitosis y una ecografía abdominal que no visualiza el apéndice. ¿Qué
intervención recomendarías? IRM

Un varón de 43 años de edad con SIDA refiere disnea y diarrea cada vez más intensa. Su
temperatura es de 36.6°C, frecuencia respiratoria de 26/min, frecuencia cardíaca de 100 lpm y
presión arterial de 100/70 mmHg. La exploración del área cardíaca y campos pulmonares es
normal. La gasometría con inhalación de aire ambiental muestra pH de 7.10; Pco2 de 5, Po2 de
130; bicarbonato calculado de 6. ¿Cuál es el trastorno acidobásico primario?
ACIDOSIS MET

Femenina de 35 años, obesa, acude solicitando una prueba de embarazo de


urgencia. Signos vitales estables, prueba de embarazo resulta positiva y se
ordena USG abdominal que reporta colelitiasis. La paciente niega dolor
abdominal. ¿Cuál es el manejo recomendado?
D. EGRESO CON RECOMENDACIONES

¿Cuál de las siguientes heridas se puede suturar directamente sin necesidad de esperar una
cicatrización por “segunda intención”? HERIDA INCISA POR ARMA BLANCA EN LA CARA

Sr. Carlos tiene disnea progresiva y falla respiratoria y un médico le dice que
tiene disminuido el Contenido arterial de O2. Cómo se calcula este contenido ?
C. CaO2=1.34(Hb)%SatO2+0.0031 PaO2

Femenina de 31 años acude a urgencias con dolor abdominal luego de ingesta de comida grasa.
Al examen físico febril, taquicardica y febril (T 38.0). ¿Cuál es el paso a seguir en su
manejo?USG ABDOMINAL, ANTIBIÓTICOS IV Y COLECISTECTOMÍA
Femenina de 44 años acude al cuarto de urgencias con dolor severo intermitente en hipocondrio
derecho. Antecedente de Diabetes mellitus tipo 2, ansiedad y colon irritable. Al examen físico es
obesa y con dolor a la palpación profunda de hipocondrio derecho. Signos vitales estables con
laboratorios AST 100, ALT 105, Amilasa 30, Fosfatasa alcalina 125, BR total 2.8 BR directa 2.1.
Se realiza ultrasonido abdominal demostrando ausencia de litos en la vesícula biliar, leve
distensión de la misma sin signos inflamatorios evidentes. ¿Cuál es el siguiente paso
recomendado en el manejo? CPRE

La hipokalemia se relaciona con lo siguiente excepto: onda t elevada

Radiográficamente en una úlcera péptica perforada esperamos encontrar: aire subdiafragma

Paciente masculino de 58 años presenta sepsis abdominal importante por perforación de colon,
luego se complica con ARDS. Cuales son las características del A: TODAS

La respuesta metabólica en un paciente en la Fase Flow, por una respuesta inflamatoria


sistémica (SRIS) el cual es articulado principalmente por cuál eje nervioso – endocrino ?
hipotalamo-hipofisis-adrenak

En la Evaluación de la Escala SOFA, que variable sí corresponde al órgano o sistema evaluado?


Cardiovascular- presion arterial media

Cuál de las siguientes heridas incisas, tendrá debido a la tensión de las líneas
de Langer, MENOR tendencia a la separación de sus bordes?
C. HERIDA DE UNA LAPAROTOMÍA SUBCOSTAL DERECHA

Todo lo siguiente podemos considerar en relación con un trauma pélvico: todas

La citoquina que produce el daño pulmonar es: IL-8

En la evaluación inicial del trauma se evalua todo lo siguiente del estado neurológico, excepto:
REFLEJOS OSTEOTENDINOSOS

Paciente de 30 años su automóvil choca contra un poste de luz, provocando un trauma torácico
cerrado importante, es trasladado por sus amigos al cuarto de urgencia. Al examen físico esta
quejumbroso, Con PA de 80/40, FC 120 x, FR 32x. sat 02 91%. Tiene ingurgitación yugular
disminución de los movimientos del tórax del lado derecho, ruidos respiratorios e
hiperresonancia y los ruidos cardíacos se auscultan taquicárdicos. En la evaluación inicial de
este paciente es importante: USG FAST

Paciente de 30 años su automóvil choca contra un poste de luz, provocando un trauma torácico
cerrado importante, es trasladado por sus amigos al cuarto de urgencia. Al examen físico está
quejumbroso, con PA de 80/40, FC 120 x, FR 32x. sat 02 91%. Tiene ingurgitación yugular
disminución de los Movimientos del tórax del lado derecho, ruidos respiratorios e
hiperresonancia y los ruidos cardíacos se auscultan taquicárdicos. Para mejorar la ventilación
del paciente debemos: COLOCACIÓN DE AGUJA EN EL SEGUNDO ESPACIO INTERCOSTAL,
LÍNEA MEDIO CLAVICULAR
.Los inhibidores de la colinesterasa se emplean: EN LA FASE DE DESPERTAR PARA
REVERTIR EL EFECTO DE LOS RELAJANTES

Rama de la arteria gastroduodenal que irriga la porción derecha de la curvatura mayor del
estómago: GASTROEPIPLOICA DERECHA

Sr Oscar sufre atropello por un vehículo al cruzar la calle, múltiples traumas incluyendo Trauma
Craneoencefálico, presentando: FC: 96 PA: 127/68 SatO2:99% No abre los ojos ni al dolor,
flexión anormal de extremidades y al interrogatorio está confuso. ¿Qué nivel de Glasgow tendrá
y qué hacemos? GLASGOW 7 REQUIERE INTUBACIÓN ENDOTRAQUEAL

El órgano intraabdominal que mayormente se lesiona en el trauma abdominal cerrado es: bazo

Paciente masculino de 68 años ingresa al hospital con FC: 94, PA: 110/73.
SatO2: 96%, FR:24, Lactato: 8.0 mmol/L, GB:8,000, “moretones” en las piernas, con
llenado capilar lento (>5 seg). Cuál es su más probable diagnóstico:
Está en shock

En la Evaluación de la Escala SOFA, que variable sí corresponde al órgano o sistema evaluado?

La clasificación de la Sociedad Americana de Anestesiólogos (ASA) para valorar


el riesgo quirúrgico se basa en:
C. SITUACIÓN GENERAL DEL PACIENTE Y ENFERMEDADES ASOCIADAS

Paciente de 92 años es traído de hogar de ancianos en mal estado general con


sepsis urinaria y admitido a UCI para manejo. Luego de varios días en UCI y
debido a su deterioro, se le realiza USG abdominal que reporta cálculo biliar
enclavada en cístico con distensión de vesícula biliar y líquido pericolecístico.
¿Cuál es su recomendación de manejo a seguir?
A. COLECISTOSTOMÍA PERCUTÁNEA (PUNCIÓN CATÉTER)

Cuáles son sus características principales de la fase Flow en su 4 día?: elevacion de


catecolaminas y gluco,,

Las lesiones laterales por trauma automovilístico produce comúnmente lesiones


de
D. A (PELVIS) y C (DIAFRAGMÁTICAS

Respecto al concepto de sedación lo siguiente es cierto EXCEPTO: cuando esta sedado hay
analgesia

En un paciente que presenta dolor recurrente en hipocondrio derecho, qué estudio de imagen
nos ayudaría para el diagnóstico?.
Ultrasonido

Respecto a la monitorización anestésica señalar cuál de las siguientes


sentencias es falsa:
E. LOS CATÉTERES VENOSOS CENTRALES TIENEN MUY POCA UTILIDAD EN LA
MONITORIZACIÓN ANESTÉSICA
Paciente de 50 años con dolor abdominal crónico en hemiabdomen superior se le realiza CAT
abdominal reportando distensión de vesícula biliar con engrosamiento calcificado de sus
paredes (vesícula de porcelana). ¿Qué condición de riesgo tiene esta paciente asociada con
este hallazgo de imagen? ca de vesicula bliar

Un hombre de 55 años de edad tiene evidencia de TC de apendicitis complicada


con un absceso contenido en el CID. Está levemente taquicárdico, afebril y
normotenso, con sensibilidad focal en el CID pero sin peritonitis. ¿Cuál es el
enfoque óptimo para este paciente?
C. DRENAJE PERCUTÁNEO, LÍQUIDOS IV, REPOSO INTESTINAL Y
ANTIBIÓTICOS DE AMPLIO ESPECTRO.

Cuáles son las indicaciones más frecuentes para realizar intubación endotraqueal? todas

Respecto a los fármacos empleados en anestesia general señalar si las afirmaciones siguientes
son verdaderas excepto: LA INTUBACIÓN ENDOTRAQUEAL SE REALIZA SIEMPRE ANTES
DE ADMINISTRAR LOS FÁRMACOS HIPNÓTICOS Y RELAJANTES MUSCULARES

En la evaluación secundaria del trauma todo los siguiente es cierto excepto:


D. SE DEBE REALIZAR EN PACIENTES INESTABLES

Una mujer de 49 años de edad con antecedentes de migraña reporta seis días de evolución con
cefalea persistente, náuseas y vómito recurrente. a la exploración física la paciente se encuentra
de pie. Los electrólitos muestran bicarbonato de 42 meq/L y la medición de gases en sangre
arterial muestra un pH de 7.53, dióxido de carbono de 53, y Po2 de 85. ¿Cuál es la anomalía
acido básica subyacente?
Alcalosis metabólica

La citoquina que modula la respuesta metabólica es:


A. TNF

Las siguientes lesiones pueden causar la muerte entre minutos y horas excepto
D. LESIÓN COMPLETA MEDULAR A NIVEL DE C4

En la evaluación inicial del trauma se evalua todo lo siguiente del estado neurológico, excepto:
Reflejos osteotendinosos

Paciente femenina de 62 años que acude vía cuarto de urgencias, traída por su
esposo, refiere que en horas de la mañana presentó evacuaciones melénicas en
3 ocasiones y vómitos en borras de café en 1 ocasión, sus signos vitales son FC
90/min, FR: 18/min, PA: 100/60 mmHg, el mejor diagnóstico es:
D. SANGRADO DIGESTIVO ALTO

Hormona de la contrarregulación anabólica:


R: INSULINA

Paciente con sospecha de neoplasia de periampular. Marcador tumoral más utilizado es:
A. CA 19-9
PRIMER PARCIAL DE CIRUGÍA MED-11
LA FLOGOSIS

1. ¿​Cuál de las siguientes heridas se puede suturar directamente sin


necesidad de esperar una cicatrización por “segunda intención”?
B. HERIDA INCISA POR ARMA BLANCA EN LA CARA

2. Todo lo siguiente podemos considerar en relación con un trauma pélvico:


D. TODAS LAS ANTERIORES (SON FUENTE DE HEMORRAGIA IMPORTANTE, SI
HAY FRACTURA SE DEBE DESCARTAR LESIÓN INTRAABDOMINAL Y TIENE
ALTA MORBIMORTALIDAD)

3. Paciente de 39 años, diabética, obesa, hipertensa, acude a Urgencias con


disuria y dolor en bajo vientre. Urinalisis con datos de infección de vías urinarias.
Hemograma sin leucocitosis. El plan es egreso con antibióticos ambulatorios,
pero en el resultado de USG abdominal ordenado se reporta Colelitiasis. La
paciente no tiene ni ha tenido dolor en hipocondrio derecho ni datos de
dispepsia. Basado en esto, ¿Cuál es su recomendación de manejo?
C. NINGÚN MANEJO ADICIONAL

4. Manejo de un paciente con colelitiasis asintomática


A. RECOMENDACIONES

5. Paciente operado de apendicitis aguda no complicada pesa 80 Kg. y mide 1.70


cm la cantidad de agua que le corresponde en 24 horas es de:
R: 2400 cc

6. De las glándulas gástricas, cuál tipo de células es la que produce histamina:


B. CÉLULAS ENTEROCROMAFINES

7. Respecto a las heridas lo siguiente es cierto EXCEPTO


A. UNA HERIDA POR ARMA BLANCA EN EL TÓRAX QUE AFECTA A LA PLEURA
PARIETAL ES UNA HERIDA PENETRANTE
B. UNA HERIDA POR ARMA BLANCA EN EL ABDOMEN QUE AFECTA AL
PERITONEO PARIETAL ES UNA HERIDA PENETRANTE
C. UNA HERIDA POR EMPALAMIENTO AFECTA EL ANO Y EL PERINÉ
D. UNA HERIDA POR ARMA DE FUEGO QUE AFECTA A LA PARED GÁSTRICA ES
PERFORANTE
E. UNA HERIDA DE “SCALP” ES POR ARRANCAMIENTO EN LA PARED
TORÁCICA
8. Los pacientes en UCI pueden exhibir diferentes tipos de respuesta, cuando se
presenta una respuesta equilibrada entre la proinflamación y la antiinflamación y
predomina la hemostasia. Se le llama:
C. MARS

9. Radiográficamente en una úlcera péptica perforada esperamos encontrar:


D. AIRE LIBRE SUBDIAFRAGMÁTICO

10. Paciente de 70 años con presencia de pólipo vesicular de 12 mm sintomático, su


manejo adecuado.
A. COLECISTECTOMÍA

11. ¿Cuáles son sus características principales de la fase Flow en su 4 día?:


R: ELEVACIÓN DE CATECOLAMINAS Y GLUCOCORTICOIDES

12. Señalar el puesto correcto para cada una de las actuaciones siguientes en la
secuencia de la inducción anestésica:
B. MONITORIZACIÓN, ACCESO VENOSO, PREOXIGENACIÓN, FÁRMACOS,
INTUBACIÓN

13. Todos los siguientes se consideran una vía aérea definitiva excepto:
A. INTUBACIÓN ENDOTRAQUEAL
B. CRICOIDECTOMÍA Qx
C. CRICOTIROIDECTOMÍA POR AGUJA
D. INTUBACIÓN NASOTRAQUEAL

14. La hipokalemia se relaciona con lo siguiente excepto:


A. DEBILIDAD
B. ÍLEO PARALÍTICO
C. HIPOTENSIÓN ORTOSTÁTICA
D. ONDA T ELEVADA EN EKG

15. Femenina de 44 años acude al cuarto de urgencias con dolor severo intermitente
en hipocondrio derecho. Antecedente de Diabetes mellitus tipo 2, ansiedad y
colon irritable. Al examen físico es obesa y con dolor a la palpación profunda de
hipocondrio derecho. Signos vitales estables con laboratorios AST 100, ALT 105,
Amilasa 30, Fosfatasa alcalina 125, BR total 2.8 BR directa 2.1. Se realiza
ultrasonido abdominal demostrando ausencia de litos en la vesícula biliar, leve
distensión de la misma sin signos inflamatorios evidentes. ¿Cuál es el siguiente
paso recomendado en el manejo?
B. CPRE

16. Sr. Carlos tiene disnea progresiva y falla respiratoria y un médico le dice que
tiene disminuido el Contenido arterial de O2. Cómo se calcula este contenido ?
C. CaO2=1.34(Hb)%SatO2+0.0031 PaO2

17. La indicación quirúrgica de un quiste de colédoco se basa en:


B. POTENCIAL DE MALIGNIDAD

18. La principal causa de muerte en el trauma es:


A. NO TENER UNA VÍA AÉREA PERMEABLE

19. El sustrato para producir óxido nitroso es:


D. ARGININA

20. Una mujer joven con antecedente de enfermedad convulsiva presenta


convulsiones tónico clónicas en la sala de urgencias. Se encuentra letárgica y a
la exploración física no hay focalización neurológica. Sus gases en sangre
arterial muestran pH de 7.12, dióxido de carbono de 48, Po2 de 86 y bicarbonato
calculado de 16. ¿Cómo se clasificaría el trastorno ácido básico?
B. ACIDOSIS MIXTA, METABÓLICA Y RESPIRATORIA

21. Una mujer de 45 años se presenta con dolor en el CID. Se realiza una
tomografía computarizada. ¿Qué hallazgo en la tomografía computarizada es el
más sugerente de linfoma apendicular?
A. DIÁMETRO APENDICULAR >2.5 cm O ENGROSAMIENTO DEL TEJIDO BLANDO
CIRCULANTE

22. Sr Oscar sufre atropello por un vehículo al cruzar la calle, múltiples traumas
incluyendo Trauma Craneoencefálico, presentando: FC: 96 PA: 127/68 SatO2:
99% No abre los ojos ni al dolor, flexión anormal de extremidades y al
interrogatorio está confuso. ¿Qué nivel de Glasgow tendrá y qué hacemos?
E. GLASGOW 7 REQUIERE INTUBACIÓN ENDOTRAQUEAL

23. Un paciente de 20 años que presenta colelitiasis, ahora se asocia con ictericia y
coluria. Que estudio ayudaría a su diagnóstico
D. COLANGIORESONANCIA

24. En Febrero de 2016 la ESICM y la SCCM publican nuevos consensos de sepsis


relacionados a criterios clínicos eliminando los términos SRIS y Sepsis Severa.
Adoptándose nuevas definiciones:
D. SEPSIS: DISFUNCIÓN ORGÁNICA QUE AMENAZA LA VIDA CAUSADA POR
UNA DISREGULACIÓN EN LA RESPUESTA DEL HUÉSPED A LA INFECCIÓN

25. Rama de la arteria gastroduodenal que irriga la porción derecha de la curvatura


mayor del estómago:
D. GASTROEPIPLOICA DERECHA

26. La piedra angular en el manejo de la colangitis es


B. DRENAJE BILIAR

27. Paciente masculino de 58 años presenta sepsis abdominal importante por


perforación de colon, luego se complica con ARDS. ¿Cuáles son las
características del ARDS ?
E. TODAS LAS ANTERIORES (DISNEA AGUDA Y TAQUIPNEA, HIPOXEMIA
REFRACTARIA AL AUMENTO DE O2, INFILTRADOS EN PARCHE EN Rx DE
TÓRAX, NO EVIDENCIA DE FALLA CARDÍACA)

28. En shock séptico cuál es el vasopresor de primera elección:


B. NOREPINEFRINA

29. En un caso de hidrosadenitis supurada de la ingle el mejor tratamiento después


de la excisión de la lesión es:
A. CIERRE POR SEGUNDA INTENCIÓN

30. ¿​Cuál de las siguientes situaciones de entre las que se consiguen durante una
anestesia general es FALSA:
A. AMNESIA ANTERÓGRADA
B. PÉRDIDA DE CONCIENCIA
C. MANTENIMIENTO DE LOS ESTÍMULOS NOCICEPTIVOS
D. REDUCCIÓN DE LA ACTIVIDAD REFLEJA
E. INHIBICIÓN DEL SISTEMA SIMPÁTICO

31. Le consultan de Cuidados Intensivos al equipo de Cirugía General por paciente


masculino de 26 años admitido por quemaduras severas y sepsis secundaria
que lleva 2 semanas en UCI. Ante la persistencia de glóbulos blancos elevados
e intolerancia a la dieta enteral le realizan USG abdominal que reporta distensión
de la vesícula biliar con engrosamiento de sus paredes en 5 mm. ¿Cuál es su
diagnóstico más probable en este caso?
R: COLECISTITIS AGUDA ACALCULOSA
32. Mientras revisa la patología de una apendicectomía laparoscópica reciente,
observa que además de la apendicitis aguda, el paciente tenía un tumor
carcinoide de 1,5 cm ubicado en la base del apéndice. El paciente está sano y
se está recuperando bien de la cirugía. ¿Qué recomendarías?
R: HEMICOLECTOMÍA DERECHA

33. Respecto al concepto de sedación lo siguiente es cierto EXCEPTO:


A. ES UN ESTADO PREVIO A LA HIPNOSIS Y A LA ANESTESIA
B. EL PACIENTE SEDADO SE ENCUENTRA EN ESTADO DE INDIFERENCIA
PSICOMOTORA
C. EL PACIENTE SE ENCUENTRA REACTIVO Y PUEDE SER DESPERTADO CON
FACILIDAD
D. CUANDO UN PACIENTE ESTÁ SEDADO ESTÁ EN ESTADO DE ANALGESIA
E. DURANTE LA SEDACIÓN EXISTE UNA ABOLICIÓN DEL COMPONENTE
PSÍQUICO DEL DOLOR

34. Paciente masculino de 68 años ingresa al hospital con FC: 94, PA: 110/73,
SatO2: 96%, FR: 24, Lactato: 8.0 mmol/L, GB: 8,000. “moretones” en la piernas,
con llenado capilar lento (> 5 seg). Cuál es su más probable diagnóstico?:
B. ESTA EN SHOCK

35. Paciente con colangitis y deterioro neurológico, manejo adecuado:


C. CPRE

36. La respuesta del anabolismo al trauma produce lo siguiente excepto:


A. BALANCE NITROGENADO POSITIVO
B. CICATRIZACIÓN DE LAS HERIDAS
C. DISMINUCIÓN DE HORMONAS TIROIDEAS
D. PREPARAR EL SISTEMA INMUNE

37. Las siguientes lesiones pueden causar la muerte entre minutos y horas excepto:
A. NEUMOTÓRAX A TENSIÓN
B. TAPONAMIENTO PERICÁRDICO
C. NEUMOTÓRAX SIMPLE
D. LESIÓN COMPLETA MEDULAR A NIVEL DE C4

38. Paciente masculino de 38 años acude a consulta con historia de dolor


intermitente, no incapacitante, en hipocondrio derecho. Al examen físico signos
vitales estables, Murphy negativo. Laboratorios normales. USG abdominal revela
pólipo en vesícula biliar de 1.5 cm sin datos de colecistitis. ¿Cuál es el siguiente
paso recomendado en el manejo?:
B. COLECISTECTOMÍA LAPAROSCÓPICA ELECTIVA

39. Respecto a los fármacos empleados en anestesia general señalar si las


afirmaciones siguientes son verdaderas excepto:
A. LOS SEDANTES ANSIOLÍTICOS TIPO BENZODIACEPINAS SE PUEDEN
UTILIZAR EN LA FASE DE PREINDUCCIÓN ANESTÉSICA
B. LA INTUBACIÓN ENDOTRAQUEAL SE REALIZA SIEMPRE ANTES DE
ADMINISTRAR LOS FÁRMACOS HIPNÓTICOS Y RELAJANTES MUSCULARES
C. LOS OPIÁCEOS TIPO FENTANILO SE UTILIZAN DURANTE LA FASE DE
MANTENIMIENTO ANESTÉSICO
D. LA NEOSTIGMINA Y NALOXONA SON FÁRMACOS REVERSORES QUE SE
EMPLEAN EN LA FASE DE EDUCCIÓN
E. EL MANTENIMIENTO HIPNÓTICO SE PUEDE REALIZAR TANTO CON
FÁRMACOS INHALATORIOS COMO INTRAVENOSOS

40. Un hombre de 55 años de edad tiene evidencia de TC de apendicitis complicada


con un absceso contenido en el CID. Está levemente taquicárdico, afebril y
normotenso, con sensibilidad focal en el CID pero sin peritonitis. ¿Cuál es el
enfoque óptimo para este paciente?
C. DRENAJE PERCUTÁNEO, LÍQUIDOS IV, REPOSO INTESTINAL Y
ANTIBIÓTICOS DE AMPLIO ESPECTRO.

41. La citoquina que produce el daño pulmonar es:


D. IL-8

42. Los inhibidores de la colinesterasa se emplean:


C. EN LA FASE DE DESPERTAR PARA REVERTIR EL EFECTO DE LOS
RELAJANTES

43. El intestino juega un papel importante como defensa interna del cuerpo. El
principal combustible del enterocito es:
B. GLUTAMINA

44. Femenina de 35 años, obesa, acude solicitando una prueba de embarazo de


urgencia. Signos vitales estables, prueba de embarazo resulta positiva y se
ordena USG abdominal que reporta colelitiasis. La paciente niega dolor
abdominal. ¿Cuál es el manejo recomendado?
D. EGRESO CON RECOMENDACIONES

45. Respecto a la monitorización anestésica señalar cuál de las siguientes


sentencias es falsa:
A. LA PULSIOXIMETRÍA ES UN MÉTODO ESPECTROFOTOMÉTRICO QUE
PERMITE MEDIR LA SATURACIÓN DE O2 DE FORMA NO INVASIVA
B. EL CAPNÓGRAFO MIDE LOS NIVELES DE CO2 Y ES EL MEJOR MÉTODO NO
INVASIVO DE MONITORIZACIÓN DE LA VENTILACIÓN
C. LA INDICACIÓN DE MONITORIZAR LA DIURESIS DEPENDE, ENTRE OTROS
FACTORES, DE LA DURACIÓN DE LA INTERVENCIÓN
D. EL CATÉTER DE SWAN-GANZ SE COLOCA EN LA ARTERIA PULMONAR*
E. LOS CATÉTERES VENOSOS CENTRALES TIENEN MUY POCA UTILIDAD EN LA
MONITORIZACIÓN ANESTÉSICA

46. Paciente de 30 años su automóvil choca contra un poste de luz, provocando un


trauma torácico cerrado importante, es trasladado por sus amigos al cuarto de
urgencia. Al examen físico está quejumbroso, con PA de 80/40, FC 120 x, FR
32x. sáb 02 91%. Tiene ingurgitación yugular disminución de los movimientos
del tórax del lado derecho, ruidos respiratorios e hiperresonancia y los ruidos
cardíacos se auscultan taquicárdicos. El diagnóstico más probable de este
paciente es:
R: NEUMOTÓRAX A TENSIÓN

47. En la evaluación inicial del trauma se evalúa todo lo siguiente del estado
neurológico, excepto
A. GLASGOW COMA SCORE
B. REFLEJOS OSTEOTENDINOSOS
C. PUPILAS
D. SIGNOS DE LATERALIZACIÓN

48. Al colangiocarcinoma que se presenta en la confluencia de los radicales


hepáticos se le conoce como
C. TUMOR DE KLATSKIN

49. El sodio no debe ser corregido rápidamente por el riesgo de Mielinosis Pontina
Cerebral. La velocidad de reposición sodio debe ser no mayor por hora de
D. 20 meq

50. Las lesiones laterales por trauma automovilístico produce comúnmente lesiones
de
D. A (PELVIS) y C (DIAFRAGMÁTICAS)

51. El órgano intraabdominal que mayormente se lesiona en el trauma abdominal


penetrante:
D. INTESTINO DELGADO
52. Paciente de 92 años es traído de hogar de ancianos en mal estado general con
sepsis urinaria y admitido a UCI para manejo. Luego de varios días en UCI y
debido a su deterioro, se le realiza USG abdominal que reporta cálculo biliar
enclavada en cístico con distensión de vesícula biliar y líquido pericolecístico.
¿Cuál es su recomendación de manejo a seguir?
A. COLECISTOSTOMÍA PERCUTÁNEA (PUNCIÓN CATÉTER)

53. Una de las siguientes afirmaciones sobre la ventilación mecánica (VM) y la


intubación es FALSA:
A. LA VM ES UN SISTEMA DE INTERCAMBIO DE GASES A NIVEL DE LA
MEMBRANA ALVEOLO-CAPILAR
B. LA VM CONTROLADA POR VOLUMEN ES LA QUE MÁS SE UTILIZA, CON TUBO
ENDOTRAQUEAL, DURANTE LA ANESTESIA GENERAL
C. DEBIDO A LA DIFERENTE DISTENSIBILIDAD DE LOS ALVÉOLOS SEGÚN SU
NIVEL DE PERFUSIÓN, LA VM TIENDE A HIPOVENTILAR LAS ZONAS
PULMONARES MÁS PERFUNDIDAS Y A HIPERVENTILAR LAS ZONAS
PULMONARES DE ESPACIO MUERTO
D. LA INTUBACIÓN ENDOTRAQUEAL ES EL MÉTODO MÁS SEGURO PARA
MANTENER LA PERMEABILIDAD Y CONTROL DE LA VÍA AÉREA
E. LAS CÁNULAS DE GUEDEL SON LAS CÁNULAS OROFARÍNGEAS, QUE SE
EMPLEAN PARA EVITAR QUE LA BASE DE LA LENGUA OBSTRUYA LA VÍA AÉREA

54. Hormona de la contrarregulación anabólica:


R: INSULINA

55. ¿Cuál de los siguientes signos físicos se asocia con la definición correcta que
sugiere apendicitis aguda?
A. SIGNO DE ROVSING: DOLOR EN EL CID AL PALPAR EL CUADRANTE
INFERIOR IZQUIERDO

56. ¿​Cuáles son las indicaciones más frecuentes para realizar intubación
endotraqueal?
E. TODAS LAS ANTERIORES (GLASGOW < 8, SHOCK, ATELECTASIA MASIVA Y
OBSTRUCCIÓN DE LA VÍA AÉREA)

57. Un hombre de 25 años de edad presenta dolor migratorio en el cuadrante inferior


derecho (CID), leucocitosis y una tomografía computarizada compatible con
apendicitis aguda y sin complicaciones. Es fisiológicamente normal y son las 2
de la mañana. Ud. está planeando una apendicectomía, ¿qué diferencia podría
esperarse en su resultado si su operación se retrasa hasta la mañana siguiente?
E. NO HAY DIFERENCIA EN LAS TASAS DE PERFORACIÓN, INFECCIÓN DEL
SITIO Qx, ABSCESO, TASA DE CONVERSIÓN O TIEMPO OPERATORIO

58. Son complicaciones de las úlceras pépticas EXCEPTO:


A. DOLOR
B. SANGRADO
C. PERFORACIÓN
D. OBSTRUCCIÓN

59. El órgano intraabdominal que mayormente se lesiona en el trauma abdominal


cerrado es:
C. BAZO

60. Un varón de 43 años de edad con SIDA refiere disnea y diarrea cada vez más
intensa. Su temperatura es de 36.6°C, frecuencia respiratoria de 26/min,
frecuencia cardíaca de 100 lpm y presión arterial de 100/70 mmHg. La
exploración del área cardíaca y campos pulmonares es normal. La gasometría
con inhalación de aire ambiental muestra pH de 7.10; Pco2 de 5, Po2 de 130;
bicarbonato calculado de 6. ¿Cuál es el trastorno ácido básico primario?
C. ACIDOSIS METABÓLICA

61. Paciente de 50 años con dolor abdominal crónico en hemiabdomen superior se


le realiza CAT abdominal reportando distensión de vesícula biliar con
engrosamiento calcificado de sus paredes (vesícula de porcelana). ¿Qué
condición de riesgo tiene esta paciente asociada con este hallazgo de imagen?
C. CÁNCER DE VESÍCULA BILIAR

62. En la cicatrización ocurre lo siguiente EXCEPTO:


A. LA FASE INFLAMATORIA COMIENZA A LOS TRES O CUATRO DÍAS DESPUÉS
DE PRODUCIRSE LA HERIDA
B. LOS MACRÓFAGOS SEGREGAN EL FACTOR DE CRECIMIENTO DE LOS
FIBROBLASTOS
C. LA EPITELIZACIÓN SE PRODUCE SIEMPRE SOBRE EL TEJIDO DE
GRANULACIÓN Y DESDE LOS BORDES DE LA HERIDA
D. EL HIERRO Y LA VIT C SON INDISPENSABLES PARA LA SÍNTESIS DE
COLÁGENO
E. EL FENÓMENO DE CONTRACCIÓN DE LA HERIDA SE PRODUCE
APROXIMADAMENTE A LAS 48 HRS DE PRODUCIRSE ESTA

63. Respecto a la cicatrización de las heridas lo siguiente es cierto EXCEPTO


A. SE LLAMA ÚLCERA DE MARJOLIN A LA APARICIÓN DE UN CARCINOMA
ESPINO CELULAR IMPLANTADO SOBRE LA CICATRIZ DE UNA QUEMADURA
B. EN LA CICATRIZ QUELOIDEA LA MASA DE COLÁGENO SOBREPASA LOS
BORDES DE LA HERIDA
C. LAS CICATRICES HIPERTRÓFICAS GENERALMENTE NO NECESITAN
TRATAMIENTO PORQUE MEJORAN CON EL TIEMPO
D. LOS PACIENTES EN TRATAMIENTO CORTICOIDEO TIENEN MEJOR
CICATRIZACIÓN POR SU EFECTO ANTIINFLAMATORIO
E. LAS DROGAS CITOTÓXICAS AFECTAN A LA SÍNTESIS DEL COLÁGENO

64. En la fase inflamatoria del proceso de cicatrización las principales células


responsables del Factor de Crecimiento para las células endoteliales y la
angiogénesis son:
B. MACRÓFAGOS

65. Paciente femenina de 62 años que acude vía cuarto de urgencias, traída por su
esposo, refiere que en horas de la mañana presentó evacuaciones melénicas en
3 ocasiones y vómitos en borras de café en 1 ocasión, sus signos vitales son FC
90/min, FR: 18/min, PA: 100/60 mmHg, el mejor diagnóstico es:
D. SANGRADO DIGESTIVO ALTO

66. En la Evaluación de la Escala SOFA, que variable sí corresponde al órgano o


sistema evaluado :
D. CARDIOVASCULAR-PRESIÓN ARTERIAL MEDIA

67. La mejor manera de disminuir las muertes por trauma en el primer pico de
mortalidad es:
R: DIVULGAR MEDIDAS DE PREVENCIÓN SOBRE EL TRAUMA

68. Las colecistopatías más frecuentemente son confundidas con:


D. GASTRITIS

69. La citoquina que modula la respuesta metabólica es:


A. TNF

70. Paciente con sospecha de neoplasia de periampular. Marcador tumoral más


utilizado es
A. CA 19-9
71. Un paciente de 85 años fue operado por divertículo perforado, la cirugía fue
prolongada, se deja el abdomen abierto, en ventilador y se pasa a UCI. Son
todas causas de aumento de pérdidas sensibles excepto:
A. EVAPORACIÓN
B. TAQUICARDIA
C. FIEBRE
D. PÉRDIDA AL TERCER ESPACIO

72. La mejor estrategia para modular la respuesta inflamatoria en el trauma es:


D. TODAS LAS ANTERIORES (OXIGENACIÓN ADECUADA, REANIMACIÓN
ADECUADA, ANALGESIA TEMPRANA)

73. Las células Principales del estómago secretan:


R: PEPSINA

74. Sra. Carlota presenta shock hipovolémico con FC: 110 PA:80/54 FR:21 Sat. O2:
95%. Cuál es la mejor forma para administrar rápidamente mayores volumen de
líquido ?
B. A TRAVÉS DE CATÉTERES PERIFÉRICOS YA QUE MÁS CORTOS PASA MÁS
FLUJO

75. En la evaluación secundaria del trauma todo los siguiente es cierto excepto:
A. SE EVITA MÁS DAÑOS DE LA LESIÓN INICIAL
B. SE ENCUENTRA LESIONES QUE PUEDEN PASAR DESAPERCIBIDAS EN LA
EVALUACIÓN INICIAL
C. SE DEBE REALIZAR UN EXAMEN DE PIES A CABEZA
D. SE DEBE REALIZAR EN PACIENTES INESTABLES

76. Paciente de 30 años su automóvil choca contra un poste de luz, provocando un


trauma torácico cerrado importante, es trasladado por sus amigos al cuarto de
urgencia. Al examen físico está quejumbroso, con PA de 80/40, FC 120x, FR
32x. sat 02 91%. Tiene ingurgitación yugular disminución de los movimientos del
tórax del lado derecho, ruidos respiratorios e hiperresonancia y los ruidos
cardíacos se auscultan taquicárdicos. E​n la evaluación inicial de este paciente
es importante:
B. USG FAST

77. La respuesta metabólica en un paciente en la Fase Flow, por una respuesta


inflamatoria sistémica (SRIS) el cual es articulado principalmente por cuál eje
nervioso – endocrino ?
C. HIPOTÁLAMO-HIPÓFISIS-ADRENAL

78. Sr Eduardo ingresa a UCI con Gangrena de Fournier, shock séptico asociado a
Diabetes descompensada. Presenta un nivel de Procalcitonina(PCT) de 10 ng/dl.
Cuál es falsa?:
A. PCT SE ELEVA EN INFLAMACIÓN E INFECCIONES
B. BIOMARCADOR IMPORTANTE PARA EL Dx DE SEPSIS
C. ALTAMENTE POSITIVO EN INFECCIONES VIRALES
D. VALORES>0.5ng/dl ALTO RIESGO DE SEPSIS
E. ES UNA HORMONA DE LA PROCALCITONINA

79. Masculino de 66 años de edad tos productiva desde hace 8 días, debilidad
general, dificultad para respirar FR: 25 con PaO2: 40 recibiendo FIO2: 21%
(190). Plaquetas: 200,000 Bilirrubinas: 1.0 PA: 88/45 Presión Arterial Media 59,
Escala de Glasgow: 13, Creatinina: 2.5 TPT: 60 seg Cuántos criterios de Falla
Orgánica (SOFA) presenta:
D. CUATRO

80. Una mujer de 29 años presenta dolor en el CID, fiebre y leucocitosis. Antes de
los estudios de imagen, la puntuación de Alvarado se utiliza para determinar la
probabilidad de que el paciente tenga apendicitis. Todas las siguientes variables
conforman la puntuación de Alvarado EXCEPTO
A. ANOREXIA
B. DESPLAZAMIENTO A LA IZQUIERDA DE LOS NEUTRÓFILOS
C. SIGNO DE ILIOPSOAS
D. DOLOR EN EL CID
E. FIEBRE

81. La clasificación de la Sociedad Americana de Anestesiólogos (ASA) para valorar


el riesgo quirúrgico se basa en:
C. SITUACIÓN GENERAL DEL PACIENTE Y ENFERMEDADES ASOCIADAS

82. En la Evaluación del APACHE II, qué variables son las que se consideran para
obtener el puntaje
B. EDAD + 12 VARIABLES FISIOLÓGICAS + ENFERMEDAD PREVIA

83. ¿​Cuál de las siguientes heridas incisas, tendrá debido a la tensión de las líneas
de Langer, MENOR tendencia a la separación de sus bordes?
C. HERIDA DE UNA LAPAROTOMÍA SUBCOSTAL DERECHA
84. Femenina de 31 años acude a urgencias con dolor abdominal luego de ingesta
de comida grasa. Al examen físico febril, taquicárdica y febril (T 38.0). ¿Cuál es
el paso a seguir en su manejo?
A. USG ABDOMINAL, ANTIBIÓTICOS IV Y COLECISTECTOMÍA

85. En un paciente que presenta dolor recurrente en hipocondrio derecho, ¿qué


estudio de imagen nos ayudaría para el diagnóstico?.
C. ULTRASONIDO

86. En las siguientes sentencias sobre los procedimientos anestésicos, todas son
verdaderas EXCEPTO:
A. EL HALOTANO ES UN ANESTÉSICO INHALADO
B. LA ANESTESIA RAQUÍDEA ES LA QUE SE REALIZA MEDIANTE INYECCIÓN DE
UN ANESTÉSICO LOCAL EN EL ESPACIO SUBARACNOIDEO DE LA MÉDULA
ESPINAL
C. LA ANESTESIA INTRAVENOSA TOTAL PRECISA SIEMPRE LA UTILIZACIÓN DE
UN AGENTE ANESTÉSICO HIPNÓTICO, UN ANALGÉSICO (GENERALMENTE
OPIÁCEO) Y UN BLOQUEANTE NEUROMUSCULAR
D. LA ANESTESIA REGIONAL IV PERIFÉRICA ESTÁ ESPECIALMENTE INDICADA
PARA LA Cx DE LA MAMA
E. LA ANESTESIA EPIDURAL SE REALIZA INYECTANDO UN ANESTÉSICO LOCAL
EN EL ESPACIO EXTRADURAL DEBAJO DEL LIGAMENTO AMARILLO

87. ¿Qué hallazgo de imagen excluiría la apendicitis?


C. UN ESTUDIO DE USG CON APÉNDICE COMPRESIBLE DE MENOS DE 5 mm DE
DIÁMETRO

88. Una mujer de 23 años que está embarazada de 28 semanas presenta dolor
abdominal en el lado derecho, leucocitosis y una ecografía abdominal que no
visualiza el apéndice. ¿Qué intervención recomendarías?
C. IMAGEN DE RESONANCIA MAGNÉTICA ABDOMINAL (IRM)

89. Femenina de 28 años, diabética, obesa, es traída por su esposo con dolor
severo 9/10 en hipocondrio derecho. Al examen físico, Murphy positivo y febril (T
38.2). Resto de signos vitales estables. USG abdominal colecistitis aguda. ¿Cuál
es su plan de manejo a continuación?
D. NPO, LÍQUIDOS ENDOVENOSOS, ANALGESIA Y COLECISTECTOMÍA EN
MENOS DE 72 HORAS
90. Las heridas por proyectil de armas militares tiene la siguientes características,
excepto:
A. SON DE BAJA VELOCIDAD
B. TIENEN UNA ENERGÍA DE CAVITACIÓN ALTA
C. PRODUCEN CAVITACIÓN PERMANENTE SIN NECESIDAD DE PENETRAR A LA
CAVIDAD ABDOMINAL
D. TIENEN UNA ENERGÍA CINÉTICA ALTA

91. Paciente de 30 años su automóvil choca contra un poste de luz, provocando un


trauma torácico cerrado importante, es trasladado por sus amigos al cuarto de
urgencia. Al examen físico está quejumbroso, con PA de 80/40, FC 120 x, FR
32x. sat 02 91%. Tiene ingurgitación yugular disminución de los movimientos del
tórax del lado derecho, ruidos respiratorios e hiperresonancia y los ruidos
cardíacos se auscultan taquicárdicos. ​Para mejorar la ventilación del paciente
debemos:
D. COLOCACIÓN DE AGUJA EN EL SEGUNDO ESPACIO INTERCOSTAL , LÍNEA
MEDIO CLAVICULAR

92. Paciente de 30 años su automóvil choca contra un poste de luz, provocando un


trauma torácico cerrado importante, es trasladado por sus amigos al cuarto de
urgencia. Al examen físico está quejumbroso, con PA de 80/40, FC 120 x, FR
32x. sat 02 91%. Tiene ingurgitación yugular disminución de los movimientos del
tórax del lado derecho, ruidos respiratorios e hiperresonancia y los ruidos
cardíacos se auscultan taquicárdicos. Para mejorar la oxigenación del paciente
debemos:
D. COLOCAR MÁSCARA FACIAL CON RESERVORIO A FIO AL 100% Y FLUJO 10
LPM

93. Las características de las heridas por arma blanca son:


B. BAJA ENERGÍA

94. El patrón de trauma automovilístico posterior produce comúnmente lesiones:


C. CERVICALES

95. Las características de una vía aérea difícil está dada por:
R: TODAS LAS ANTERIORES (EL TIPO DE LESIÓN, CUELLO CORTO,
OBSTRUCCIÓN DE LA VÍA AÉREA)

96. Paciente masculino de 28 años, ingresa al hospital con quemaduras de 50% de


superficie corporal quemada por explosión de tanque de gas, hace 3 horas.
Presentando FC: 100 x min PA: 80/50 Hipotérmico, Lactato: 4.6mmol / L,
diuresis: 20cc / h. ¿Cuál es la correcta ?:
R: ESTÁ EN UNA FASE HIPOMETABÓLICA

97. La respuesta metabólica en el ayuno es diferente a la del trauma; Una diferencia


es que el ayuno encontramos excepto:
A. UTILIZACIÓN DE CUERPOS CETÓNICOS COMO FUENTE ENERGÉTICA
B. HAY POCO AUMENTO DE LAS HORMONAS DE LA CONTRARREGULACIÓN
C. NEOGLUCOGÉNESIS AUMENTADA
D. TIENDE A ADAPTARSE

98. Una mujer de 49 años de edad con antecedentes de migraña reporta seis de
evolución con cefalea persistente, náuseas y vómito recurrente. a la exploración
física la paciente se encuentra de pie. Los electrolitos muestran bicarbonato de
42 meq / L y la medición de gases en sangre arterial muestra un pH de 7.53,
dióxido de carbono de 53, y Po2 de 85. ¿Cuál es la anomalía ácido básica
subyacente?
R: ALCALOSIS METABÓLICA

99. La metas del tratamiento inicial en el trauma son:


D. TODAS LAS ANTERIORES (EVALUACIÓN RÁPIDA, SEGUIR UN ORDEN
FISIOLÓGICO, EL Dx DE LA LESIÓN ES LO MENOS IMPORTANTE)

100. Luego de una cena, acude a Urgencia paciente de 42 años con dolor
intermitente en hipocondrio derecho. Signos vitales estables. Murphy dudoso.
Laboratorios sin alteraciones. USG abdominal con litos en la vesícula. ¿Cuál es
la causa más probable de su dolor?
A. DISTENSIÓN DE LA VESÍCULA BILIAR
Primer Parcial de Cirugía
MED-8

1. Masculino de 24 años, ingresa con quemaduras de 50% de su superficie


corporal por explosión de tanque de gas, hace 3 horas presentó FC 100xmin,
PA 30/50, hipotérmico, lactato en 4 y diéresis en 20, la correcta es:
D. Está en fase hipometabólica
2. Paciente 4 días en UCI, el paciente ya estaba en Fase Flow, cuáles son sus
características principales:
B. Elevación de catecolaminas y glucocorticoides
3. La respuesta metabólica en este paciente quemado se caracteriza en la Fase
Flow, por una respuesta inflamatoria sistémica, el cual está articulado por el
siguiente eje nervioso-endocrino:
C. Hipotálamo-Hipófisis-Adrenal
4. Estos pacientes pueden exhibir diferentes tipos de respuesta, cuando se
presenta una respuesta equilibrada entre respuesta proinflamatoria y
antiinflamatoria, donde predomina la hemostasia, se le llama:
C. MARS
5. Masculino de 66 años de edad, tos productiva desde hace 8 días, debilidad
general, dificultad para respirar, FR 25 con PaO2 40 recibiendo FIO2 21%
(190), Plaquetas 200,000. Bilirrubinas: 1:0 PA: 88/45 Presión Arterial Media
59. Escala de Glasgow: 13, Creatinina: 2.5 TPT: 60 seg. Cuántos criterios de
Falla Orgánica (SOFA) presenta:
A. uno
B. dos
C. tres
D. cuatro
E. cinco

6. En febrero de 2016 la ESICM y la SCCM publican nuevos consensos de sepsis


relacionados a criterios clínicos eliminando los términos SRIS y Sepsis Severa.
Adoptándose nuevas definiciones:
A. Sepsis: respuesta inflamatoria severa por infección
B. Sepsis: exagerada respuesta inflamatoria del huésped a la infección
C. Sepsis: SRIS por una infección demostrada
D. Sepsis: disfunción orgánica que amenaza la vida causada por una
disregulación en la respuesta del huésped a la infección
E. Sepsis: disfunción orgánica causada por una disregulación en la respuesta
del huésped a la infección.

7. En la Evaluación SOFA, que variable si corresponde al órgano o sistema


evaluado?
A. Respiratorio- Frecuencia respiratoria
B. Coagulación- TP. TPT, Fibrinógeno
C. Hígado- Transaminasas
D. Cardiovascular- Presión arterial media
E. Renal- Nitrógeno de Urea
8. En la Evaluación del APACHE II, qué variables son las que se consideran para
obtener el puntaje
A. Edad + 16 variables fisiológicas + Enf previa
B. Edad + 12 variables fisiológicas + Enf previa
C. Edad + SOFA + Escala de Glasgow
D. Edad + Enf crónicas + falla renal
E. Edad + Enf crónicas + falla hepática
9. Paciente masculino de 68 años ingresa al hospital con FC: 94, PA: 110/73.
SatO2: 96%, FR:24, Lactato: 8.0 mmol/L, GB:8,000, “moretones” en las piernas, con
llenado capilar lento (>5 seg). Cuál es su más probable diagnóstico:
A. Estado de PreShock
B. Está en shock
C. Probablemente en estado de hipoperfusión
D. Tiene glucólisis anaeróbica
E. Shock compensado
F. Hacen falta más datos
10. En shock séptico cuál es el vasopresor de primera elección:
A. Dobutamina
B. Norepinefrina
C. Epinefrina
D. Dopamina
E. Levosimendan
11. La Señora Encarnación tiene 69 años, ingresa a UCI a los 6 días de cirugía
pancreatoduodenectomía p(no se lee lo siguiente) de Vater; falla ventilatoria
requiriendo ventilación mecánica por hipoxemia. Su cirujano dice que “​está en
restricción de líquidos y requiere diuréticos”​ . Los intensivistas dicen: “lo que tiene es
un ARDS severo”. ¿Cuál es la causa de ARDS?
A. Broncoaspiración
B. Sepsis
C. Infección de las heridas
D. Infección urinaria
E. Infección por catéter
12. El cuadro de ARDS que pone en peligro la vida de la Sra Encarnación, podría
resolverse con:
A. Más aporte de diuréticos para “secar” el pulmón
B. Colocarla en Presión control con el ventilador
C. Colocarla en posición prono para mejorar la oxigenación
D. Resolver el cuadro séptico abdominal
E. Cambiar los antibióticos según el cultivo
13. El Sr. Eduardo ingresa a UCI con Gangrena de Fournier, shock séptico asociado
a Diabetes descompensada, un nivel de Procalcitonina (PCT) de 10ng/dl. Cuál es
falsa:
A. PCT se eleva en inflamación e infecciones
B. Biomarcador importante para el Dx de sepsis
C. Altamente positivo en infecciones virales
D. Valores >0.5ng/dl alto riesgo de sepsis
E. Es una prohormona de la calcitonina
14. NO ESTÁ
15. Cuáles son las indicaciones más frecuentes para realizar intubación
endotraqueal?
A. Escala de Coma de Glasgow <8
B. Shock
C. Atelectasia Masiva
D. Obstrucción de la vía aérea
E. Todas las anteriores
16. Paciente masculino de 58 años presenta sepsis abdominal importante por
perforación de colon, ARDS. ¿Cuáles son las características del ARDS?
A. Disnea aguda y taquipnea
B. Hipoxemia refractaria al aumento de O2
C. Infiltrados en parche en Rx Tórax
D. No evidencia de falla cardíaca
E. Todas las anteriores
17. El Sr. Carlos tiene disnea progresiva y falla respiratoria y un médico le dice que
tiene disminuido el O2. Cómo se calcula este contenido?
A. CaO2= 1.30(Hb)%SatO2 + 0.031PaO2
B. CaO2= 1.32(Hb)%SatO2 + 0.0038PaO2
C. CaO2= 1.34(Hb)%SatO2 + 0.0031PaO2
D. CaO2= 1.24(Hb)%SatO2 + 0.31PaO2
18. La Sra. Carlota presenta shock hipovolémico con FC: 110 PA:80/54 FR:21
SatO2: 95%. Cuál es la mejor forma para administrar rápidamente mayores volumen
de líquido?
A. A través de un catéter corto periférico, pues entre más delgado logra más
B. A través de catéteres periféricos ya que entre más cortos pasta más fluido.
C. A través de un catéter central, pues además puedo pasar vasopresores
D. A través de un catéter central, ya que el flujo es más lento
19. Paciente femenina de 51 años de edad, a la que se le realiza
Pancreatoduodenectomía (Cirugía de Whipple) por CA de papila de Vater- En el
postoperatorio que consecuencias fisiopatológicas tendrá si no aplicamos
PROTOCOLO ERAS/ACERTO correctamente?
A. Resistencia periférica a la insulina
B. Proteólisis y catabolismo proteico
C. Glucólisis e hiperglucemia
D. Lipólisis y aumento de ácidos grasos libres
E. Todas las anteriores
20. Para evitar complicaciones en el posoperatorio y lograr reducir la resistencia a la
insulina. ¿Qué se debió hacer preoperatoriamente?
A. Inmunonutrición 2 días antes de la cirugía
B. Aplicar Fentanyl siempre para el dolor
C. Mantener N x B desde la noche anterior
D. Carga de maltodrextina 50g 200cc 2h antes de la inducción anestésica
E. Antibióticos profilácticos 3 horas antes de la incisión
21. Para evitar complicaciones en el postoperatorio como náuseas, vómitos y que el
paciente no pueda comer vía oral. Qué se debió hacer transoperatoriamente?
A. Usar Fentanyl de rutina
B. Aplicar 1000 SSN en caso de hipotensión
C. Manejo de líquidos Guiados por Metas y de forma restrictiva
D. Monitoreo estricto de la diuresis para guiar el manejo de líquidos
E. Favorecer hipotermia
22. Evitar complicaciones en el postoperatorio. ¿Qué se debe hacer
postoperatoriamente?
A. Esperar Ruidos intestinales para iniciar dieta oral
B. Mantener líquidos intravenosos por 3-4 días
C. Mantener sonda urinaria de rutina varios días
D. Usar Fentanyl para dolor como primera opción
E. No usar sonda nasogástrica de rutina
23. El concepto de Prehabilitación dentro del Protocolo ERAS en qué consiste? Cuál
es la incorrecta?
A. Ejercicio físico óptimo
B. Dejar de fumar
C. Preparar Nutricionalmente 3 días antes
D. Apoyo psicológico y manejo de estrés
E. Consumir proteínas mínimo 1.5g/Kg/día
24. A la Sra. Eloísa le van a realizar una gastrectomía por CA de estómago, no
puede ingerir alimentos vía oral desde hace más de un mes. Ingresa al hospital el
lunes para ser operada dos días después. La nutricionista de la sala entra el primer
día y encuentra que está malnutrida severa. Cuál debe ser el paso siguiente?
A. Colocarle D/A 5% 80cc/h urgentemente para darle algo de calorías y nutrición
B. Que la cirugía sea lo más corta posible para que no afecte la cicatrización
C. Colocarle sonda entreral vía endoscopia y nutrirla por 5 días
D. Retrasar la cirugía por 4 días para darle proteínas a 0.8g/kg/día
E. Omitir la cirugía, Nutrición parenteral por 10 días mínimo y después operarla
25. Si la Sra Eloísa pesa 60Kg y el cirujano decide colocarle D/A 5% a 80cc/h para
“prepararla” nutricionalmente (no sale) de calorías y nutrición. ¿Cuánto representa
por día el aporte de calorías y proteínas?
A. Recibe 86g de CHO por día- 2.6g/Kg/día- 7.44Kcal/Kg/día + 8g proteínas
B. Recibe 96g de CHO por día- 1.6g/Kg/día- 5.44Kcal/Kg/día + No recibe
proteínas
C. Recibe 106g de CHO por día- 4.6g/Kg/día- 7.44Kcal/Kg/día + No recibe
proteínas
D. Recibe 96g de CHO por día- 1.6g/Kg/día- 5.44Kcal/Kg/día + 16g proteínas
E. Recibe 116g de CHO por día- 7.6g/Kg/día- 5.44Kcal/Kg/día + No recibe
proteínas

26.

27. La Sra Eloisa que debe quedarse en ayuno desde las 8:00 pm pues su cirugía
será a las 11:00 am. ¿Qué efectos nocivos esto tiene?
A. Aumenta el catabolismo proteico
B. Se sintetiza normalmente el receptor Glut 4
C. Catecolaminas y citoquinas empeoran la resistencia a insulinas
D. Sólo a y c
E. Sólo a y b

28. Durante la gastrectomía de la Sra Eloisa, la pérdida de líquidos totales fue de


aproximadamente 3 litros en 4 horas de cirugía. Sin embargo, el balance de líquidos
resultó ser en 5,000cc. ¿Qué efectos podrá tener este balance tan positivo?
A. Edema de los tejidos
B. Náuseas y vómitos
C. No se afecta la oxigenación
D. Retraso en la cicatrización
E. Sólo a, b y d
29. Luego de la gastrectomía + gastroyeyuno anastomosis, existe controversia para
indicarle la dieta oral y cuando. Cuál es la incorrecta?
A. Las complicaciones generales son menores al inicio precoz de la vía oral
B. Se debe esperar escuchar ruidos hidroaéreos en el abdomen o que canalice
gases​.
C. Las fugas anastomóticas disminuyen con dieta precoz
D. Hay menos días de hospitalización con la dieta precoz
E. En anastomosis esófago-yeyuno, por gastrectomía total, también se puede
iniciar temprano la dieta vía oral.
30. El Sr. Agustin está en preparación para una hemicolectomía izquierda por Ca de
colon, requiere adecuado aporte de proteínas. Cuanto debe ser este aporte?
A. 0.3-0.4 g/kg/día
B. 0.5-1.0g/kg/día
C. 1.0-1.2g/kg/día
D. 1.5-2.0g/kg/día
E. 3.0-4.0g/kg/día
31. El Sr. Agustín presenta obstrucción intestinal total, con distensión abdominal y
alto drenaje “fecaloide” por el tubo de levine. Su estado de malnutrición es severo.
¿Qué se debe hacer?
A. Hemicolectomía izquierda + colostomía terminal
B. Hemicolectomía izquierda + anastomosis colo-rectal
C. Retrasar la cirugía por 2 semanas hasta mejorar el estado nutricional
D. hemicolectomía derecha con íleo-transverso anastomosis
E. Colocar el levine a alta succión, darle inmunonutrición y después operarlo
32. ¿Cuáles son los efectos que tendrá el Sr. Agustín en su anastomosis intestinal si
es operado así malnutrido?
A. Pobre depósito de colágeno
B. Mayor depósito de proteína
C. Dehiscencia de la anastomosis
D. Menor riesgo de sepsis abdominal
E. Sólo a y c
33. ¿Qué farmaconutrientes (inmunonutrientes) debe recibir un paciente quirúrgico?
A. Arginina
B. Glutamina
C. Omega-3
D. Antioxidantes
E. Todas las anteriores
34. ¿Cuál es la dosis incorrecta de los siguientes farmaconutrientes para un
paciente grave y séptico?
A. Arginina 20-30g/día
B. Glutamina 0.3-0.5g/kg/día
C. Omega 3: 20 g /día
D. Selenio 1000ug/día
E. Proteínas 2g/kg/día
35. En pacientes críticos la terapia nutricional es tan importante como los
antibióticos, la ventilación mecánica y vasopresores en caso de hipotensión.
¿Cuales son las dosis correctas para pacientes críticos?
A. CHON: 1.0-1.2g/kg/día
B. CHO 5-6g/kg/día
C. Lípidos: >3.0g/kg/día
D. CHO 1-2 g/kg/día
E. CHON 1.5-2.0 g/kg/día
F. Lípidos 5-6 g/kg/día
36. Órgano inmunológico más grande del cuerpo:
A. Hígado
B. intestino
C. riñón
D. pulmón
E. tejido linfático
37. Ventajas que ofrece la Glutamina en el contexto del paciente críticamente
enfermo, EXCEPTO:
A. Fuente para síntesis de proteínas en la mucosa intestinal
B. Precursor de Gluthation
C. Protege contra la apoptosis inducida por oxidantes y citoquinas
D. Cataboliza proteínas de Shock térmico
E. Importante efecto antiinflamatorio
38. Componentes activos de los ácidos grasos omega-3 que deben recibir los
pacientes quirúrgicos para inflamación posoperatoria:
A. ácido eicosapentaenoico y ácido docosahexaenoico
B. ácido eicosatetraenoico y ácido docosahexaenoico
C. ácido linoléico y ácido araquidónico
D. ácido alphalinoleíco y ácido gammalinolenico
E. ácido docosapentaenoico y ácido docosahexaenoico
39. Sobre el uso de la inmunonutrición en pacientes de cirugías mayores, los
mejores resultados se han visto en
A. inmunonutrición hiperproteica
B. Inmunonutrición parenteral preoperatoria
C. inmunonutrición preoperatoria
D. inmunonutrición perioperatoria
E. inmunonutrición posoperatoria
40. El óxido nítrico tiene importantes funciones en mejorar la.. antibacteriano. Por
tanto es útil mantener óptimos niveles en el paciente quirúrgico.
A. glutamina
B. arginina
C. metionina
D. alanina
E. glicina
41. Las proteínas de shock térmico son importantes proteínas citoplasmáticas que
reparan el daño celular, además bloquean la expresión del FNKB logrando un efecto
antiinflamatorio importante en el paciente quirúrgico. ¿Cuál es su único precursor?
A. glutamina
B. arginina
C. metionina
D. alanina
E. glicina
42. Paciente masculino de 68 años, en su segundo día posoperatorio,
hemicolectomía izquierda + colo-recto anastomosis por CA de sigmoides.
Presentando: PA 110/60, disnea progresiva con FR: 30-32 x min + Sat: 89% +
PaO2: 60 mmHg + PaCO2: 30 mmHg, leucocitosis en 21,000, lactato en 4.0
Abdomen distendido y doloroso, a medida que pasan las horas la disnea empeora
con aleteo nasal y utilizando los músculos accesorios de la respiración. Rx de Tórax:
infiltrado… parche de ambos campos pulmonares. Usted como médico tratante del
caso ¿que debe hacer a este momento?
A. colocarle O2 por máscara
B. colocarle O2 por puntas nasales
C. intubarlo y colocarlo en ventilador
D. aplicar ventilación no invasiva inicialmente
E. agregar antibióticos
43.¿Cuál es el diagnóstico más probable de lo que está ocurriendo?
A. síndrome de distress respiratorio agudo (ARDS)
B. dehiscencia de la anastomosis y sepsis abdominal
C. infección severa
D. neumonía severa
E. edema pulmonar
44.¿Cuál es la causa más probable de lo que está ocurriendo?
A. síndrome de distress respiratorio agudo (ARDS)
B. dehiscencia de la anastomosis y sepsis abdominal
C. infección severa
D. neumonía severa
E. edema pulmonar
45.¿Cuál es la causa más frecuente de ARDS?
A. infección pulmonar difusa
B. broncoaspiración
C. shock
D. sepsis
E. embolia grasa
46.
47.
48. Un paciente de 85 años fue operado por divertículo perforado, la cirugía fue
prolongada, se deja el abdomen abierto, en … y se pasa a UCI. Son todas causas
de aumento de pérdidas sensibles excepto:
A. evaporación
B. taquicardia
C. fiebre
D. pérdida al tercer espacio
49. un litro de lactato ringer contiene de sodio:
A. 100 meq.
B. 120 meq.
C. 130 meq.
D. 154 meq.
50. Un cc de solucion salina hipertonica de sodio al 23,4% posee de Cloro:
A. 2 meq.
B. 3 meq.
C. 4 meq.
D. 5 meq.
51. Una mujer de 49 años de edad con antecedentes de migraña reporta seis días
de evolución con cefalea persistente, náuseas y vómitos recurrentes, a la
exploración física la paciente se encuentra de pie. Los electrolitos muestran
bicarbonato de 42 meg/L y la medición de gases en sangre arterial muestra un pH
de 7.53, dióxido de carbono de 53 y PO2 de 85. ¿Cuál es la anomalía ácido base
subyacente?
a. Acidosis metabólica
b. Alcalosis metabólica
c. Acidosis respiratoria
d. Alcalosis respiratoria

52. Una mujer joven con antecedentes de enfermedad convulsiva presenta


convulsiones tónico clónicas en la sala de urgencias. Se encuentra letargia y a la
exploración física no hay focalización neurológica. Sus gases en sangre arterial
muestra pH de 7.12, dióxido de carbono de 48, PO2 de 86 y bicarbonato calculado
de 16. ¿Cómo se clasifica el trastorno ácido base?
a. Acidosis respiratoria
b. Acidosis mixta, metabólica y respiratoria
c. Acidosis metabólica con alcalosis respiratoria
d. Alcalosis metabólica con acidosis respiratoria

53. Un varón de 43 años de edad con SIDA, refiere disnea y diarrea cada vez más
intensa. Su temperatura es de 36.6 C, frecuencia respiratoria de 26/min, frecuencia
cardiaca de 100 lpm y presión arterial de 100/70 mmHg. La exploración del área
cardiaca y campos pulmonares es normal. La gasometría con inhalación de aire
ambiental muestra pH de 7.10; PCO2 de 5, PO2 de 130; bicarbonato calculado de 6.
¿Cual es el trastorno acido básico primario?
a. Acidosis respiratoria
b. Alcalosis respiratoria
c. Acidosis metabólica
d. Alcalosis metabólica

54. La hipokalemia se relacion con lo siguiente excepto:


a. debilidad
b. ileo paralítico
c. hipotensión ortostática
d. onda t elevada en EKG

55. El sodio no debe ser corregido rápidamente por el riesgo de Mielinosis Pontina
Cerebral. La velocidad de reposición sodio debe ser no mayor por hora de:
a. 5 meq
b. 10 meq
c. 15 meq
d. 20 meq

56. Un hombre de 55 años fue arrollado por un auto. En el cuarto de urgencias,


después de 2 litros de LR su presión arterial es de 60/40, La Radiografía del Tórax
es normal, GCS 9/15 . No hay fracturas evidentes de huesos largos. El próximo
paso a seguir sería
a. Laparotomía
b. Dos unidades de GRE y reevaluación
c. TAC abdominal de urgencia
d. TAC cerebral de urgencia

57. Una dama de 40 años estuvo involucrada en un choque frontal con otro
vehículo, su esposo murió en la escena. Su presión arterial es de 100/90 FC
100/min. Después de dos litros de LR la presión arterial es de 115/90 y su FC
90/min. Se queja de leve dolor abdominal generalizado. El primer paso a seguir
sería:
a. Laparotomía
b. Hospitalizar
c. Ultrasonido de abdomen
d. Prueba de alcoholemia

58. Un hombre de 28 años recibió un golpe con un bate en una riña en el cuadrante
superior derecho del abdomen. su presión arterial es de 110/60 y su FC es de
120/min. Refiere dolor en el hemiabdomen derecho. Todas las siguientes podrían
ser medidas de manejo apropiadas excepto:
a. Dos litros de LR a chorro
b. Laparotomía
c. Observación y exámenes físicos señalados
d. Ultrasonido

59. Todas las siguientes son ciertas con respecto a la TAC de abdomen en el
diagnóstico del trauma abdominal cerrado, excepto:
a. Está indicada en pacientes estables con dolor abdominal
b. Está indicada en pacientes estables comatosos sin dolor abdominal
c. Está indicada en pacientes estables parapléjicos
d. Es altamente sensible para detectar lesiones diafragmaticas

60. El ultrasonido en trauma abdominal


a. Está indicado en pacientes con shock y dolor abdominal
b. Es más sensible que el TAC para diagnosticar lesiones renales
c. Está indicado en pacientes estables con dolor abdominal
d. Requiere la administración de contraste endovenoso

61. Cual de las siguientes heridas incisas, tendrá debido a la tensión de las líneas
de Langer. ¿MENOR tendencia a la separación de sus bordes?
a. Herida vertical en cara anterior del cuello
b. Herida vertical en región escapular
c. Herida de una laparotomía subcostal derecha
d. Herida longitudinal en cara anterior de muslo
e. Herida longitudinal sobre la cara anterior de la rodilla

62. En la fase inflamatoria del proceso de cicatrización las principales células


responsables del Factor de Crecimiento para las células endoteliales y la
angiogénesis son:
a. leucocitos
b. macrofagos
c. fibroblastos
d. neutrófilos
e. células epiteliales

63. Respecto a las heridas lo siguiente es cierto EXCEPTO:


a. una herida por arma blanca en el tórax que afecta la pleura parietal es una
herida penetrante
b. una herida por arma blanca en el abdomen que afecta el peritoneo parietal es
una herida penetrante
c. una herida por empalamiento afecta el no y el periné
d. una herida por arma de fuego que afecta a la pared gástrica es perforante
e. una herida de “scalp” es por arrancamiento en la pared torácica

64. ¿Cual de las siguientes heridas se puede retirar directamente sin necesidad de
esperar una cicatrización por “ segunda intencion”?
a. mordedura de perro en una pierna en varón joven
b. herida incisa por arma blanca en la cara
c. herida incisa de más de seis horas en una pierna en un accidente de tráfico
d. herida incisa en periné
e. todas se pueden suturar directamente

65. En la cicatrización ocurre lo siguiente EXCEPTO:


a. la fase inflamatoria comienza a los tres o cuatro días después de producirse
la herida
b. los macrofagos segregan el factor de crecimiento de fibroblastos
c. la epitelización se produce siempre sobre el tejido de granulación y desde los
bordes de la herida
d. el hierro y la vitamina C son indispensables para la sintesis de colageno
e. el fenómeno de contracción de la herida se produce aproximadamente a las
48 horas de producirse esta

66. Respecto a la cicatrización de las heridas lo siguiente es cierto EXCEPTO


a. se llama úlcera de Marjolin a la aparición de un carcinoma espino celular
implantado sobre la cicatriz de una quemadura
b. en la cicatriz queloidea la masa de colágeno sobrepasa los bordes de la
herida
c. las cicatrices hipertróficas generalmente no necesitan tratamiento porque
mejoran con el tiempo
d. los pacientes en tratamiento corticoideo tiene mejor cicatrización por su
efecto antiinflamatorio
e. las drogas citotóxicas afectan a la síntesis del colágeno

67. ¿Cuál de los siguientes mecanismos es el más característico en la formación de


un queloide?
a. una relación alterada entre la formación de colágeno y la actividad de la
colagenasa
b. una fase inflamatoria excesivamente larga
c. una síntesis excesiva de la enzima colagenasa
d. una actividad aumentada de las células endoteliales
e. una actividad aumentada del fenómeno de angiogénesis

68. En un caso de hidrosadenitis supurada de la ingle el mejor tratamiento despues


de la aparición de la lesión es:
a. cierre por segunda intención
b. cierre primario de la herida quirúrgica
c. cierre primario retardado
d. injerto de piel de espesor parcial
e. realizar un colgajo desde el abdomen para tapar el defecto

69. Sobre la fase proliferativa de la cicatrización cual de las siguientes afirmaciones


es FALSA:
a. las células implicadas en esta fase son los fibroblastos
b. la hidroxiprolina es una de los aminoácidos lleva en la síntesis de colageno
c. el hierro y la vitamina C son importantes en la cicatrización por su necesidad
de la sintesis del colageno
d. la tensión máxima de las fibras de colágeno se consigue como mínimo a los
treinta días de la producción de la herida
e. el movimiento es un buen estímulo para la correcta orientación de las fibras
de colágeno de la herida

70. Sobre el fenómeno de contracción de la herida una de las siguientes


afirmaciones es FALSA:
a. la contracción de la herida comienza antes de 48 horas
b. los miofibroblastos son las células responsables de la contracción de la
herida
c. la formación de colágeno no interviene en la contracción de la herida
d. cuando las células epiteliales de ambos bordes de la herida contactan se
crean anclajes definitivos aunque sean estirpes celulares distintas
e. la citosina es un factor de crecimiento plasmático que estimula el dinamismo
celular de la contracción de la herida

71. Cuales son las células más importantes que participan en la cicatrización de las
heridas:
a. Polimorfonucleares
b. Macrofagos
c. Plaquetas
d. Linfocitos

72. Acerca de la mitosis difusa clostridial (gangrena gaseosa), señale la respuesta


incorrecta:
a. Se inicia habitualmente en los 3 días que siguen a la lesión y progresa
rápidamente
b. lo más habitual es se deba a un retraso o desbridamiento adecuado de
tejidos después de una lesión desvalorizante
c. siempre hay crepitación
d. las manifestaciones locales consisten en dolor en la herida, con exudado
seropurulento parduzco y edema de superficie, necrosis y cambio de color
que son generalmente menos extensos que la necrosis muscular subyacente
e. la toxemia es profunda e incluye taquicardia grave, delirio e ictericia
hemolítica

73. La celulitis plantea diagnóstico diferencial con


a. tromboflebitis
b. alergia intensa por contacto
c. inflamación química por inyección de fármacos
d. fascitis necrosante
e. todas

74. Señala que medida de las que se citan es menos importantes para disminuir la
incidencia de infecciones hospitalarias
a. baño preoperatorio del paciente con jabón antiséptico
b. lavado de manos por parte del personal sanitario
c. aplicación de luz ultravioleta a la sala de operaciones
d. reducción al mínimo imprescindible de la ambulación y el hablar en la sala de
operaciones
e. evitar que intervienen cirujanos con infecciones cutáneas o de vías
respiratorias

75. La embolia gaseosa se manifiesta por:


a. taquipnea subita
b. hipotensión
c. soplo de rueda dentada
d. todos
e. ninguno

76. Señala que factor no aumenta la incidencia de infección de la herida quirurgica:


A. Exceso de catecolaminas
B. Exceso de vasopresina
C. Diabetes bien controlada
D. Exceso de angiotensina
E. Tejidos mal vascularizados

77. Se define la oliguria en el paciente adulto como un gasto urinario inferior a:


A. 0.1 cc/kg/h
B. 0.2 cc/kg/h
C. 0.3 cc/kg/h
D. 0.4 cc/kg/h
E. 0.5 cc/kg/h

78.las siguientes situaciones se asocian con una mayor incidencia de evisceración


posoperatoria EXCEPTO:
A. Malnutrición
B. Infección
C. Anemia
D. Incision paramedian
E. Isquemia

79. La cicatrización de las heridas es un proceso vital prioritario que en los humanos
se lleva a cabo mediante el depósito de colágeno. la complicación más
frecuentemente observada es la infección. De los siguientes enunciados cuál NO ES
CIERTO:
A. La clasificación de la herida es un factor pronóstico de la infección
B. Durante la fase de Respuesta Inflamatoria Aguda la recuperación de fuerza
tensil es mínima.
C. La profilaxis antibiótica debe hacerse en todas las cirugías desde 15 minutos
antes del procedimiento y mantenerla por 48 horas
D. El signo o síntoma clínico más precoz de infección de la herida quirúrgica es
el excesivo dolor sobre ella
E. La deposición de colágeno comienza alrededor del 5to día postoperatorio

80. Mujer obesa de 56 años, varicosa, en tratamiento a base de prednisona, 60 mg


por dia desde hace un año. Ingresa para cirugía abdominal mayor. Hemicolectomía
derecha por CA de colon
A. Tiene alto riesgo de tromboembolismo venoso
B. Con el colon bien preparado y si no hay complicaciones transgresiones a la
técnica, su cirugía será una cirugía limpia
C. Requiere anticoagulación plena con heparina por bomba de infusión
D. Supresión de la vía oral desde dos días antes de la cirugía
E. Todas son verdaderas

81. Femenina de 44 años acude al cuarto de urgencias con dolor severo


intermitente en hipocondrio derecho. Antecedentes de DM2, ansiedad y colon
irritable. Al examen físico es obesa y con dolor a la palpación profunda de
hipocondrio derecho. Signos vitales estables con laboratorios AST 100, ALT 105,
Amilasa 30, Fosfatasa alcalina 236, BR total 2.8, BR directa 2.1. Se realiza
ultrasonido abdominal demostrando ausencia de litos en vesícula biliar, leve
dilatación de la misma sin signos inflamatorios evidentes. ¿Cual es el siguiente paso
recomendado en el manejo?
A. Colecistectomía Laparoscópica
B. CPRE
C. Analgesicos y manejo conservador
D. Resonancia magnética abdominal

82. Femenina de 35 años, obesa, acude solicitando una prueba de embarazo de


urgencia. Signos vitales estables, prueba de embarazo resulta positiva y se ordena
USG abdominal que reporta colelitiasis. La paciente niega dolor abdominal. ¿Cual
es el manejo recomendado?
A. CAT abdominal
B. HIDA scan
C. Colecistectomía laparoscópica electiva
D. Egreso con recomendaciones

83. Luego de una cena, acude a Urgencia paciente de 42 años con dolor en
hipocondrio derecho. Signos vitales estables. Murphy dudoso. Laboratorios sin
alteraciones. USG abdominal con litos en la vesícula. ¿ Cuál es la causa más
probable de su dolor?
A. Distensión de la vesícula biliar
B. Inflamación de las paredes de la vesícula biliar
C. Distensión del conducto cístico
D. Inflamación de la cápsula de Glisson (perihepática)

84. Paciente de 39 años, diabética, obesa, hipertensa, acude a Urgencia con disuria
y dolor en bajo vientre. Urinalisis con datos de infección de vías urinarias.
Hemograma sin leucocitosis. El plan es egreso con antibióticos ambulatorios, pero
en el resultado de USG abdominal ordenado se reporta colelitiasis. La paciente no
tiene ni ha tenido dolor en hipocondrio derecho ni datos de dispepsia. Basado en
esto ¿ Cuál es su recomendación de manejo?
A. Colecistectomía Laparoscópica
B. Acido ursodeoxicolico
C. Ningún manejo adicional
D. CPRE

85. Paciente de 50 años con dolor abdominal crónico en hemiabdomen superior se


le realiza TAC abdominal reportando distensión de la vesícula biliar con
engrosamiento de sus paredes (vesícula de porcelana) ¿Que condición de riesgo
tiene esta paciente asociada con este hallazgo de imagen?
A. Cancer de pancreas
B. Ruptura de aneurisma
C. Cancer de vesicula biliar
D. Ruptura de quiste hidatídico

86. femenina de 28 años, diabética, obesa, es traída por su esposo con dolor 9/10
en hipocondrio derecho. Al examen físico Murphy positivo y febril (T 38.2). Resto de
signos vitales estables. USG abdominal colecistitis aguda. ¿Cúal es su plan de
manejo a continuación?
A. NPO, líquidos endovenosos, analgesia
B. NPO, líquidos endovenosos, analgesia y colecistectomía urgente
C. NPO, líquidos endovenosos, analgesia y colecistectomía en 24 h
D. NPO, líquidos endovenosos, analgesia y colecistectomía en menos de 72 h

87. Paciente de 92 años es traído de hogar de ancianos en mal estado general con
sepsis urinaria y admitido a UCI para manejo. Luego de varios días en UCI y debido
a su deterioro, se le realiza USG abdominal que reporta cálculo biliar enclavada en
cístico con distensión de vesícula biliar y líquido pericolecístico ¿Cúal es su
recomendación de manejo a seguir?
A. Colecistostomía percutánea (punción catéter)
B. Colecistectomía de urgencia
C. CAT abdominal
D. Manejo en UCI con antibióticos de amplio espectro

88. Paciente masculino de 38 años acude con historia de dolor intermitente, no


discapacitante, en hipocondrio derecho. Al examen físico signos vitales estables,
Murphy negativo. Laboratorios normales. USG abdominal revela polipo en vesícula
biliar de 1.5 mm sin datos de colecistitis. ¿Cuál es el siguiente paso recomendado
en el manejo?
A. Egreso con recomendaciones de dieta
B. Colecistectomía laparoscópica electiva
C. Vigilancia de pólipo vesicular 4-6 meses
D. Resonancia magnética abdominal

89. Le consultan de Cuidados Intensivos al equipo de cirugía general por paciente


masculino de 26 años admitido por quemaduras severas y sepsis secundaria que
lleva 2 semanas en UCI. Ante la persistencia de glóbulos blancos elevados e
intolerancia a la dieta enteral le realizan USG abdominal que reporta disfunción de
vesícula biliar con engrosamiento de sus paredes de 5 mm. ¿Cuál es su diagnóstico
más probable en este caso?
A. Colecistitis (sepsis severa)
B. Colecistitis aguda litiásica
C. Colecistitis aguda acalculosa
D. Pancreatitis aguda

90. Femenina de 31 años acude a urgencias con dolor abdominal luego de ingesta
de comida grasa. Al examen físico febril (T 38.0), taquicardia. ¿Cúal es el paso a
seguir en su manejo?
A. USG abdominal, antibióticos IV y colecistectomia
B. CAT abdominal contrastado, antibióticos y colecistectomia
C. PET scan, antibióticos y reevaluar
D. Resonancia magnética, antibióticos y reevaluar
91. En un paciente que presenta dolor recurrente en hipocondrio derecho, ¿qué
estudio de imagen nos ayudaría para el diagnóstico?
A. Rayos x de abdomen
B. Tomografía
C. Ultrasonido
D. Resonancia nuclear

92. Un paciente de 20 años que presenta colelitiasis, ahora se asocia con ictericia y
coluria. Que estudio ayudaría a su diagnóstico.
A. Ultrasonido
B. Tomografia
C. Centelleo
D. Colangioresonancia

93. Paciente con sospecha de neoplasia de periampular. Marcador tumoral más


utilizado es:
A. CA 19-9
B. CEA
C. CA 125
D. PSA

94. La piedra angular en el manejo de la colangitis es:


A. Hidratación
B. Drenaje de vía biliar
C. Antibióticos
D. Colecistectomía

95. Paciente de 70 años con presencia de pólipo vesicular de 12 mm sintomático, su


manejo adecuado:
A. Colecistectomía
B. Manejo conservador
C. Antiespasmódicos
D. CPRE

96. Paciente con colangitis y deterioro neurológico, manejo adecuado:


A. Colangiografía
B. Antiespasmódicos
C. CPRE
D. HIDA

97. indicación quirúrgica de un quiste de colédoco se basa en:


A. Sintomatología
B. Potencial de malignidad
C. Probabilidad de perforación
D. Riesgo de infección

98. Manejo de un paciente con colelitiasis asintomática:


A. Recomendaciones
B. CPRE
C. Colecistectomía de urgencia
D. Antiespasmódicos

99. Las colecistopatías más frecuentemente son confundidas con:


A. Gastroenteritis
B. Pancreatitis
C. Apendicitis
D. Gastritis

100. Al colangiocarcinoma que se presente en la confluencia de los radicales


hepático se le conoce:
A. Neoplasia periampular
B. Carcinoide
C. Tumor de klatskin
D. Carcinoma intrahepatico
Parcial 1 de Cirugía
MED 3

Paciente masculino de 68 años en su segundo día postoperatorio, hemocolectomía


izquierda + colo-recto anastomosis por CA de sigmoides. Presentando: PA: 110/60,
disnea progresiva con Fr: 30-32 x min + SO2 89%, PaO2: 60 mmHg, PaCO2: 30 mmHg.
Leucocitosis en 21000, Lactato 4.0. Abdomen distendido y doloroso. A medida que pasan
las horas la disnea empeora con aleteo nasal y utlizando los músculos accesorios de la
respiración. Rx de Tórax: infiltrados “en parche” de ambos campos pulmonares.

1. Usted como médico tratande del caso que debe hacer en este momento?
c. Intubarlo y colocarlo en ventilador

2. Cuál es el diagnóstico más probable de lo que está ocurriendo?


c. Síndrome de Distrés Respiratorio Agudo (ARDS)

3. Cual es la causa más probable de lo que está ocurriendo?


d. Deshicencia de la anastomosis y sepsis abdominal

4. Cuál es la causa más frecuente de ARDS?


b. Sepsis

5. Cómo está el transporte de oxígeno en este paciente?


b. Disminuido

6. Posteriormente el paciente presenta hipotensión: PA: 90/50, Fc: 120 x min, PVC: 4
mmHg asociado a hipovolemia, requiere aporte inmediato de líquidos intravenosos.
Qué vía utilizaría?
c. Acceso a través de dos venas periféricas

7. De las glándulas gástricas, cuál es el tipo de células que produce histamina:


b. Células enterocromafines

8. Órgano inmunológico más grande del cuerpo:


c. Intestino

9. Cuál de los siguientes aspectos anaomofisiológicos explican la mayor suceptibilidad


de la muscosa intestinal a los estados de shock e inestabilidad hemodinámica?
a. Cortocircuitos arterio-venosos en las vellosidades intestinales
b. Vasos arteriales entran a la vellosidad en ángulo recto
c. Esfínteres precapilares simpáticos
d. Vasodilatación parasimpática en yeyuno – íleon
e. Todas excepto la D
10. Translocación bacteriana es:
a. Cruce de bacterias y sus productos hacia el flujo sanguíneo y linfático
b. Cruce de bacterias, pero no de hongos, a la circulación sanguínea
c. Cruce de bacterias, pero no de virus, a la circulación sanguínea
d. Siempre lleva a SRIS, sepsis y shock séptico
e. Todas las anteriores

11. Durante un estado de shock o hipoperfusión sistémica, ocurre redistribución del flujo
sanguíneo:
d. Sistema Nervioso Central

12. Fórmulas inmunomoduladoras recomendadas en pacientes críticos y quirúrgicos de


alto riesgo deben estar enriquecidas con cual farmaconutriente:
a. Arginina
b. Glutamina
c. Ácidos grasos omega-3 de aceite de pescado
d. Aminoácidos ramificados
e. Todas las anteriores

13. En pacientes con agresiones severas como quemaduras, trauma, sepsis y cirugías
mayores puede ocurrir lo siguiente:
c. Hipercatabolismo

14. En pacientes críticos quirúrgicos cuál de las siguientes es la recomendación diana:


d. Proteínas 1.5 – 2.0 g/Kg/día

15. Cuál de los siguientes farmaconutrientes contribuyen al control de la respuesta


inflamatoria en pacientes quirúrgicos críticos:
a. Selenio
b. Glutamina
c. Ácidos grasos omega-3
d. Arginina
e. Todas las anteriores

16. Cuál de los siguientes es necesario en altas dosis para lograr mejor cicatrización de
heridas mayores:
a. Selenio
b. Glutamina
c. Ácidos grasos omega-3
d. Arginina
e. Todas las anteriores

17. Músculo detrás del cuál se ubica a la glándula tiroides:


c. Esternotiroídeo

18. Funciones de las hormonas tiroideas:


a. Generación de calor
b. La T3 incrementa el consumo de oxígeno en mucho tejidos
c. Desarrollo normal del SNC en el feto
d. Funcionamiento normal del SNC
e. Todas las anteriores

19. Mujer de 36 años hace dos semanas con infección respiratoria viral ahora se queja de
fiebre, dolor en la glándula tiroides, con piel eritematosa. Se trato con antiinflamatorios
y corticoides con excelente mejoría.
b. Tiroiditis subaguda

20. Paciente femenina de 57 años con antecedentes de fátiga, pérdida de peso, diarreas,
taquicardia y nerviosismo. Hace un mes con protrusión de ambos globos oculares y
bocio difuso. Cuál es su probable diagnóstico:
c. Enfermedad de Graves-Basedow

21. Paciente femenina de 45 años con nódulo tiroideo frío (0.8 cm) al centelleo en el lóbulo
derecho del tiroides, la biopsia por aguja fina confirma CA papilar, ausencia de datos
de mal pronóstico o metástasis.
Cual procedimiento quirúrgico requiere:
a. Tiroidectomia total bilateral
b. Lobectomía izquierda + istmectomía
c. Lobectomía bilateral
d. Radioterapia y después tiroidectomía
e. Ninguna de las anteriores

22. Sobre el nódulo tiroideo solitario:


b. 30% de los nódulos que sangran son malignos

23. En la unión esofogogástrica, cuál es el nombre al vértice del ángulo de His que
protruye en la cara interna del estómago y asciende cuando la presión intragástrica
aumenta:
c. Válvula de Gubaroff

24. Rama de la arteria gastroduodenal que irriga la porción derecha de la curvatura mayor
del estómago:
d. Gastroepiploica derecha

25. De las glándulas gástricas, cuál tipo de célula es la que produce histamina:
b. Célula enterocromafín
Paciente de 25 años de edad trasladada al servicio de reanimación del cuarto de
urgencia, tras accidente de tránsito. Al ingreso se encuentra hipotenso, taquicárdico y
taquipneíco.
Radiografía muestra: fracturas costales derecha e imagen de contusión pulmonar
derecha y de pelvis inestable.

26. La mejor conducta a realizar es:


b. Realizar ecografía abdominal en el cuarto de urgencias – FAST

27. Mientras exploramos a este paciente bruzamente presenta alteración de su control


hemodinámico y aumento de la taquipnea. Se observa desviación traqueal hacia la
izquierda, venas yugulares distendidas y ausencia de ruidos respiratorios en
hemitórax derecho, el diagnóstico más probable es:
c. Neumotórax a tensión

Paciente femenina de 65 años ingesta crónica de Mortin por artritis degenerativa de la


rodila. Se queja de dolor epigástrico episódico y que empeora al comer, toma omeprazol
desde hace un año pero siente mejoría episódica. Hasta que desde anoche siente mucho
dolor persistente en todo el abdomen el cual se alivia si no se mueve, al presionar todo
el abdomen se palpa muy duro. Al hundir la mano en el abdomen y retirarla rápido la
paciente grita de dolor.

28. Cuál es la causa de la patología inicial de esta paciente?


c. Úlcera gástrica

29. Cúal puede ser el origen de esta patología?


c. Helicobacter pylori

30. A que se debe probablemente el aumento del dolor gastrico generalizado en el


abdomen?
b. Perforación

31. En caso de confirmarse la sospecha de perforación de víscera hueca, que


procedimiento quirúrgico es el indicado?
c. LPE + biopsia de la úlcera + cierre de Graham

32. Paciente femenina de 34 años con vómitos de sangre roja mezclados con comida,
antecedentes de consumo de Ranitidina y Maalox, cuál puede ser la causa probable?
c. Úlcera péptica

33. La paciente continua con la hematemesis y se agrega melena. Se descubre que


consume aspirina para dolores epigástricos. Cuál es la conducta inmediata?
b. Transfundir plaquetas
34. Después de transfusiones de hemoderivados y endoscopía supuestamente
“satisfactoria” la paciente presenta un cuadro de vómito de sangre abundante,
presenta lipotimia, gran debilidad, Fc: 120 x min, PA: 80/58 y SO2: 98%.
Que procede ahora?
a. Nueva endoscopia terapéutica
b. Probable LPE y resección del sitio sangrante
c. Corregir el probable trastorno de la hemostasia
d. Reanimación hemodinámica
e. Todas las anteriores

Paciente de 25 años de edad quien colisiona su automóvil contra un oste de luz al llegar
al cuarto de urgencias los paramédicos refieren que tuvieron un retraso para extraerlo
del carro de 30 minutos y con deformidad severa del auto. Al llegar al cuarto de urgencias
el paciente grita que no puede respirar. PA 90/50 mmHg, Fc: 120 cpm, Fr: 32 cpm, SO2:
65%.
Presencia de equimosis del cinturón en tórax. Fracturas costales múltiples del lado
izquierdo, ruido abolidos del mismo lado, poca movilidad del hemitórax izquierdo.

35. En la evaluación inicial este paciente tiene un problema grave en:


b. La respiración

36. En la reanimación inicial de este paciente, usted debe:


c. Utilizar máscara facial con reservorio a 10 L/min y SO2 al 100%

37. El paciente no mejora y continua gritando, usted debe sospecha:


d. Neumotórax a tensión

38. Luego de haber hecho el diagnóstico inicial, usted debe primero:


d. Colocar una aguja gruesa en el segundo espacio intercostal izquierdo en
la línea medio clavicular izquierda.

39. El 16 de octubre se reconoce como día mundial del anestesiólogo, porque:


d. Ese día en 1846, en Boston Massachusetts, se operó por primera vez
en la historia a un paciente bajo los efectos del éter.

40. Los anestésicos locales:


c. Enlentecen la despolarización de la membrana a nivel de los canales
de sodio

41. Paciente de 19 años presenta diagnóstico de apendicitis aguda, va para SOP,


usted le hace la historia clínica, la paciente refiere sufrir ASMA BRONQUIAL
en la niñez y no presenta crisis desde hace 10 años. Usted se acuerda de su
clase de anestesiología y la clasificaría como:
b. Clase ASA II
42. Usted está en el cuarto de urgencias, va a suturar una paciente de 65 kg con
una herida extensa perforante, le aplicaría 20 Ml de lidocaína al 2%. ¿Cuál es
la dosis máxima de lidocaína que usted puede usar?
d. 5mg/kg

43. La hormona anabólica más importante de la respuesta a la contraregulación en


trauma es:
d. Insulina

44. En la respuesta metabólica por trauma, son todas acciones del TNF, excepto:
b. Es la productora del aumento de temperatura en trauma (falso, la que
causa esto es la interleucina 1)

45. El metabolismo de los carbohidratos se ve afectado por el trauma en:


c. Aumento de la resistencia periférica a la insulina.

46. La diferencia fundamental de la respuesta metabólica en ayuno vs el trauma,


lo constituye:
d. En el ayuno hay una fase de adaptación

47. El tratamiento adecuado para vencer la respuesta inflamatoria en el trauma es:


a. Manejo agresivo del choque
b. Intubación temprana si está indicada
c. Cirugía temprana
d. Todas las anteriores

48. La forma más rápida para evaluar la vía aérea de un paciente de trauma es:
a. Posicionar, estimular y evaluar la respuesta

49. La celulitis plantea diagnóstico diferencial con:


a. Tromboflebitis
b. Alergia por contacto
c. …
d. Todas las anteriores
UNIVERSIDAD DE PANAMA
FACUTAD DE MEDICNA
I EXAMEN PARCIAL DE CIRUGIA - SEPTIEMBRE 2016

Escoja la respuesta correcta

Caso 1 (pregunta 1 a 6)
Paciente masculino de 68 años, en su segundo día postoperatorio, hemicolectomia izquierda + colo-recto
anastomosis por CA de sigmoides. Presentando PA 110/60, disnea progresiva con Fr 30-32/min + Sat
89% + PaO2 60 mmHg + PaCO2 30 mmHg. Leucocitosis en 21 000, Lactato 4.0
Abdomen distendido y doloroso. A medida que pasan las horas la disnea empeora con aleteo nasal y
utilizando los músculos accesorios de la respiración. Rx de tórax: infiltrados en parche de ambos
campos pulmonares.

1- Usted como médico tratante del caso. Que debe hacer en este momento:
a) Colocarle O2 por mascara
b) Colocarle O2 por puntas nasales
c) Intubarlo y colocarlo en ventilador
d) Aplicarle ventilación no invasiva inicialmente
e) Agregarle antibióticos

2- Cuál es el diagnóstico más probable de lo que está ocurriendo:


a) Síndrome de Diestres Respiratorio Agudo (ARDS)
b) Dehiscencia de la anastomosis y sepsis abdominal
c) Infección severa
d) Neumonía severa
e) Edema pulmonar

3- Cuál es la causa más probable de lo que está ocurriendo:


a) Síndrome de Diestres Respiratorio Agudo (ARDS)
b) Dehiscencia de la anastomosis y sepsis abdominal
c) Infección severa
d) Neumonía severa
e) Edema pulmonar

4- Cual es la causa más frecuente de ARDS:


a) Infección pulmonar difusa
b) Broncoaspiración
c) Shock
d) Sepsis
e) Embolia grasa

5- Como está el transporte de oxígeno en este paciente:


a) Normal pues la PaO2 está bien
b) Aumentado
c) Disminuido
d) Evidente datos de hipoperfusión
e) Con APACHE II alto
6- Posteriormente el paciente presenta hipotensión PA 90/50, Fc 120/min, PVC 4 mmHg asociado
a hipovolemia, requiere aporte inmediato de líquidos intravenosos. Que vía utilizaría:
a) Acceso atravesó de un catéter central
b) Acceso a través de dos venas periféricas
c) Acceso intraóseo
d) Usar D/A 5% a chorro
e) Acceso por vena subclavia

7- De las glándulas gástricas cual tipo de células es la que produce Histamina:


a) Célula parietal
b) Célula enterocromafines
c) Célula G
d) Célula mucosa
e) Célula D

8- Órgano inmunológico más grande del cuerpo:


a) Hígado
b) Bazo
c) Intestino
d) Estomago
e) Pulmón

9- Cuál de los siguientes aspectos anatomofisiologicos explican la mayor susceptibilidad de la


mucosa intestinal a los estados de shock e inestabilidad hemodinámica:
a) Cortocircuito arterio-venoso en las vellosidades intestinales
b) Vasos arteriales entran a la vellosidad en ángulo recto
c) Esfínteres precapilares simpáticos
d) Vasodilatación parasimpática en yeyuno-ilion
e) Todas excepto D

10- Translocación bacteriana es:


a) Cruce de bacterias y sus productos hacia el flujo sanguíneo y linfático
b) Cruce de bacterias pero no de hongos a la circulación sanguínea
c) Cruce de bacterias pero no de virus a la circulación sanguínea
d) Siempre lleva a SRIS, sepsis y shock séptico
e) Todas las anteriores

11- Durante un estado de shock o hipoperfusión sistémica ocurre redistribución del flujo sanguíneo
hacia:
a) Sistema muscular
b) Sistema renal
c) Sistema gastrointestinal
d) Sistema nervioso central
e) Sistema óseo

12- Las formulas inmunomoduladoras recomendadas en pacientes críticos y quirúrgicos de alto


riesgo deben estar enriquecidas con cales farmaconutrientes:
a) Arginina
b) Glutamina
c) Ácidos grasos omega 3 de aceite de pescado
d) Aminoácidos ramificados
e) Todos los anteriores
13- En pacientes con agresiones severas como quemaduras, trauma, sepsis y cirugías mayores puede
ocurrir lo siguiente:
a) Mayor síntesis proteica
b) Mayor masa muscular
c) Hipercatabolismo
d) Mejoría inmunológica
e) Menor riesgo de infecciones

14- En pacientes críticos quirúrgicos cuál de los siguientes es la recomendación diaria:


a) Carbohidratos 1-2 g/Kg/día
b) Lípidos 0.8-1.5 g/Kg/día
c) Selenio 200-500ug/día
d) Proteína 1.5-2.0 g/Kg/día
e) Energía 30-35 Kcal/Kg/día

15- Cuál de las siguientes farmaconutrientes contribuyen al control de las respuesta inflamatoria en
pacientes quirúrgicos críticos:
a) Selenio
b) Glutamina
c) Ácidos grasos omega 3
d) Arginina
e) Todas las anteriores

16- Cuál de las siguientes es necesario en altas para lograr mejor cicatrización de herida mayores:
a) Selenio
b) Glutamina
c) Ácidos grasos omega 3
d) Arginina
e) Todas las anteriores

17- Musculo detrás del cual se ubica la glándula tiroides:


a) Digastrico (vientre anterior y posterior)
b) Estilohioideo
c) Esternotiroideo
d) Milohioideo
e) Genihioideo

18- Funciones de la glándula tiroides:


a) Generación de calcio
b) La T3 incrementa el consumo de oxígeno en muchos tejidos
c) Desarrollo normal del sistema nervioso central en el feto
d) Funcionamiento normal del sistema nervioso central
e) Todas las anteriores

19- Mujer de 36 años, hace dos semanas con infección viral, ahora se queja de fiebre, dolor en
glándula tiroides con piel eritematosa. Se trató con antiinflamatorios y corticoides con excelente
mejoría, cual puede ser el probable diagnóstico:
a) Tiroiditis aguda
b) Tiroiditis subaguda
c) Tiroiditis de Hashimoto
d) Enfermedad de Graves-Basedow
e) Struma linfomatosa
20- Paciente femenina de 57 años con antecedentes de fatiga, pérdida de peso, diarreas, taquicardia
y nerviosismo. Hace un mes con protrusión de ambos globos oculares y bocio difuso. Cuál es su
probable diagnóstico:
a) Enfermedad de Plummer
b) Tiroiditis de Quervain
c) Enfermedad de Graves-Basedow
d) Tormenta tiroidea
e) Bocio toxico

21- Paciente femenina de 45 años con nódulo tiroideo frio (0.8cm) al centelleo en el lóbulo derecho
del tiroides, la biopsia por aguja fina confirma CA papilar, ausencia de datos de mal pronóstico
o metástasis. Cual procedimiento quirúrgico requiere:
a) Tiroidectomía total bilateral
b) Lobectomía izquierda + istmectomia
c) Lobectomía bilateral
d) Radioterapia y después tiroidectomía
e) Ninguna de las anteriores

22- Sobre el nódulo tiroideo solitario:


a) Aumento rápido de tamaño sugiere hemorragia en nódulo existente
b) 30% de los nódulos que sangran son malignos
c) 10% de los nodulos que no sangran son malignos
d) Crecimiento rápido (semanas) descarta de malignidad
e) Malignidad es el doble en mujeres que en hombres

23- En la unión esofagogastrica, cual es el nombre al vértice del ángulo de His que protruye en la
cara interna del estómago y asciende cuando la presión intragastrica aumenta:
a) Ligamento frenogastrico
b) Peritoneo visceral del estomago
c) Válvula de Gubaroff
d) Roseta mucosa esofágica
e) Lamina Leimer

24- Rama de la arteria gastroduodenal que irriga la porción derecha de la curvatura mayor del
estómago:
a) Gastroduodenal inferior derecha
b) Pancreatoduodenal superior derecha
c) Rama posteroinferior de la gástrica derecha
d) Gastroepiploica derecha
e) Coronaria estomaquica

25- De las glándulas gástricas cual tipo de células es la que produce Histamina:
a) Célula parietal
b) Célula enterocromafines
c) Célula G
d) Célula mucosa
e) Célula D
Caso 2 (pregunta 26 y 27)
Paciente de 25 años de edad trasladada al servicio de reanimación del servicio de urgencias tras
accidente de tráfico. A su ingreso esta hipotenso, taquicardico y taquipneico. La radiología portátil
muestra fracturas costales derecha e imagen de contusión pulmonar derecha y la pelvis es la que se
adjunta.

26- Cuál es la mejor conducta a continuación:


a) Realizar ecografía abdominal en el cuarto de urgencias (USG-FAST)
b) Solicitar TAC de tórax
c) Solicitar angiografía pélvica
d) Solicitar TAC de abdomen y pelvis
e) Ingresar inmediatamente al paciente en la unidad de cuidados intensivos

27- Mientas exploramos a este paciente bruscamente presenta alteración de su control


hemodinámico y aumento de la taquipnea. Se observa desviación traqueal hacia la izquierda,
venas yugulares distendidas y ausencia de ruidos respiratorios en hemitórax derecho. El
diagnóstico más probable es:
a) Progresión de la contusión pulmonar derecha
b) Lesión vascular cervical derecha expansiva
c) Taponamiento cardiaco
d) Hemotórax derecho
e) Neumotórax a tensión
Caso 3 (pregunta 28 a 31)
Paciente femenina de 65 años ingesta crónica de Motrin por artritis degenerativa de la rodilla. Se queja
de dolor epigástrico episódico y que empeora al comer. Toma omeprazol desde hace un año pero sinte
mejoría episódica. Hasta que desde anoche siente mucho dolor persistente en todo el abdomen, el cual
se alivia si no se mueve, al presionar todo el abdomen se palpa muy duro. Al hundir en el abdomen y
retirarla rápido la paciente grita del dolor.

28- Cuál es la causa de la patología inicial de esta paciente:


a) Apendicitis aguda
b) Pancreatitis aguda
c) Ulcera gástrica
d) Ulcera duodenal
e) Gastritis crónica

29- Cuál puede ser el origen de esta patología:


a) Obstrucción de la luz del apéndice cecal
b) Obstrucción del ducto pancreático con algún calculo biliar
c) Helicobacter pylori
d) Antiinflamatorio no esteroideo
e) Hipergastrinemia

30- A qué se debe, probablemente, ahora el aumento del dolor generalizado en el abdomen:
a) Obstrucción
b) Perforación
c) Sangrado
d) Inflamación
e) Ca gástrico

31- En caso de confirmarse la sospecha de perforación de víscera hueca. Que procedimiento


quirúrgico es el más indicado:
a) LPE + apendicectomia
b) LPE + necrosectomia
c) LPE + biopsia de la ulcera + cierre de Graham
d) LPE + cierre de Zelanie-Jones
e) LPE + gastrectomía subtotal

Caso 4 (pregunta 32 a 35)


Paciente femenina de 34 años con vómitos de sangre roja mezclados con comida. Antecedentes de
consumo de Ranitidina y Malox.

32- Cuál puede ser la causa probable:


a) Varices esofágicas
b) Síndrome de Mallory-Weiss
c) Reflujo gastroesofágico
d) Ulcera péptica
e) Gastritis severa

La paciente continúa con la hematemesis y se agrega melena. Se descubre que consume Aspirina para
los dolores epigástricos.
33- Cuál es la conducta inmediata:
a) Hacer endoscopia lata
b) Transfundir plaquetas
c) Transfundir plasma fresco congelado
d) Transfundir glóbulos rojos empacados (GRE)
e) Hacer lavado gástrico
Después de transfusiones de hemoderivados y endoscopia supuestamente satisfactoria la paciente
presenta un cuarto vomito de sangre abundante, presenta lipotimia, gran debilidad, Fc 120/min, Pa
80/58, Sat 98%.
34- Que procede ahora:
a) Nueva endoscopia terapéutica
b) Probable LPE y resección del sitio sangrante
c) Corregir el probable trastorno de hemostasia
d) Reanimación hemodinámica
e) Todas las anteriores

La reanimación hemodinámica de esta paciente, consiste en administrar volumen (cristaloides +


albumina) a chorro hasta lograr adecuada normovolemia.
35- Porque el medico escogió dos venas gruesas periféricas para dicha reanimación:
a) Flujo inversamente proporcional al grosor del catéter
b) Flujo inversamente proporcional a la longitud del catéter
c) Flujo del catéter periférico es mayor porque es más delgado
d) Flujo es directamente proporcional a la viscosidad de la solución
e) Flujo no tienen que ver con la longitud del catéter

Caso 5 (pregunta 36 a 39)


Paciente de 25 años de edad quien colisiona su automóvil contra un poste de luz, al llegar al cuarto de
urgencias los paramédicos refieren que tuvieron un retraso en extraerlo del carro de 30 minutos y con
deformidad severa dl auto. Al llegar al cuarto de urgencias el paciente grita que no puede respirar. PA
90/50, Fc 120x´, Fr 32x´, Sat 75%. Presenta equimosis de cinturón en tórax, fracturas costales múltiples
del lado derecho, ruidos abolidos del mismo lado y poca movilidad del hemitórax izquierdo.
36- En la evaluación inicial este paciente un grave problema en:
a) Vía aérea
b) Ventilación
c) Circulación
d) Neurológico

37- En la reanimación inicial de este paciente usted debe:


a) Colocar cánula nasal con oxígeno a 10l/min
b) Colocar una cánula orofarinfea
c) Colocar broncodilatadores
d) Mascara facial con reservorio a 10l/min y FIO2 al 100%

38- El paciente no mejora y continua gritando usted debe sospechar:


a) Obstrucción de la lengua
b) Neumotórax a tensión
c) Ruptura de la aorta
d) Ruptura de bazo

39- Luego de haber hecho el diagnóstico inicial usted debe primero:


a) Canalizar dos venas y 2 litros de lactato a chorro
b) Intubar al paciente
c) Colocar una aguja gruesa en el segundo espacio intercostal línea medio clavicular izquierda
d) Ultrasonido FAST
40- El 16 de octubre se reconoce como el día mundial del anestesiólogo porque:
a) Ese día en 1846 en Boston Masachusset se operó por primera vez en la historia un paciente bajo
los efectos del cloroformo
b) Ese día en 1846 en Boston Masachusset se operó por primera vez en la historia un paciente bajo
los efectos del sevoflurane
c) Ese día en 1846 en Boston Masachusset se operó por primera vez en la historia un paciente
bajo los efectos del éter
d) Ese día en 1846 en Boston Masachusset se operó por primera vez en la historia un paciente bajo
los efectos del óxido nitroso

41- Los anestésicos locales enlentecen la despolarización de la membrana a nivel de:


a) Canales de Na+
b) Receptores de GABA
c) Receptores µ
d) Receptores de NMDA

42- Usted según la clase de anestesiología que recibió la clasificaría como:


a) Clase ASA 1
b) Clase ASA 2
c) Clase ASA 3
d) Clase ASA 4

43- Si usted está en el cuarto de urgencias y va a suturar a un paciente de 65 Kg con herida extensa
perforante, le aplicara 20 ml de Lidocaína al 2%. Cuál es la dosis máxima de Lidocaína que
usted puede usar:
a) 3 mg/Kg
b) 5 mg/Kg
c) 10 mg/Kg
d) 15 mg/Kg

44- La hormona anabólica más importante de la respuesta a la contraregulación en trauma es:


a) Glucagón
b) Cortisol
c) Adrenalina
d) Insulina

45- En la respuesta metabólica por trauma son todas acciones de TNFα excepto:
a) Inestabilidad hemodinámica
b) Sirve de estímulo a la aparición de nuevas citoquinas
c) Bloquea la aparición de linfocitos helper
d) Es la productora de aumento de temperatura en el paciente con trauma

46- El metabolismo de los carbohidratos se ve afectado por el trauma en:


a) Aumento de la producción de glucógeno
b) Aumento de la resistencia periférica a la insulina
c) Hipoglicemia
d) Aumento de cuerpos cetónicos

47- La diferencia fundamental entre la respuesta metabólica al ayuno versus el trauma lo constituye:
a) En el ayuno hay una fase de adaptación
b) La energía en el trauma es proveniente de los cuerpos cetónicos
c) En el ayuno hay un incremento importante de nitrógeno urinario
d) En el trauma hay una disminución de las proteínas de fase aguda
48- El tratamiento adecuado para atenuar la respuesta inflamatoria aumentada en el trauma es:
a) Manejo agresivo del choque
b) Intubación temprana si está indicada
c) Cirugía temprana
d) Todas las anteriores

49- La forma más rápida de evaluar la vía aérea de un paciente de trauma es:
a) Colocar oximetro de pulso
b) Escuchar el murmullo vesicular pulmonar normal
c) Llamarlo estimulándolo y evaluar la respuesta
d) Elevar la mandíbula

50- La celulitis plantea diagnóstico diferencial con:


a) Tromboflebitis
b) Alergia intensa por contacto
c) Inflamación química por inyección de fármacos
d) Fascitis necrotizante
e) Todas
PRIMER PARCIAL CIRUGIA
ABRIL 2007 MD-27

La composición electrolítica del lactato ringer, recuerda la composición electrolítica de:


a. Orina en un paciente normalmente hidratado
b. Pérdida insensible de líquidos
c. Líquido intracelular
d. Líquido extracelular

El desarrollo de hiponatremia post-operatoria es generalmente causada por:


a. Enfermedad renal
b. Administración inadecuada de líquidos
c. Hipoadrenalismo
d. Diabetes insípida oculta
e. Succión gástrica

Un paciente con hipernatremia postoperatoria, que ocasionalmente se presenta después de habérsele


administrado algunos agentes anestésicos, debe ser tratado con la aplicación de:
a. Cortisol
b. Vasopresina
c. Ácido etacrínico
d. Dextrosa en agua al 5%

En relación a la glutamina, podemos decir que es un:


a. Aminoácido no esencial (no, según Fuentes)
b. Ácido graso no esencial
c. Aminoácido esencial (según Matos que dio la clase, es no esencial que se vuelve esencial
durante el trauma)
d. Ácido graso esencial

Entre las complicaciones por nutrición parenteral se incluyen todas las siguientes, EXCEPTO:
a. Incremento en el riesgo de infección
b. Hiperglicemia
c. Infiltración grasa del hígado
d. Producción excesiva de CO2

Usted es llamado a la sala por un paciente de 80 años que fue sometido a cirugía abdominal. La
enfermera le informa que está en su 3er día post-operatorio y que luego de haber cenado presenta
dificultad respiratoria, a la auscultación se escucha sibilancias difusas en ambos campos pulmonares y
a la oximetría resultó saturación baja de oxígeno. Su diagnóstico más probable es:
a. neumotórax
b. broncoaspiración
c. embolismo pulmonar

Paciente varón de 75 años al cual se le practicó cirugía reparadora por hernia inguinal bajo anestesia
epidural. La cirugía fue prolongada y dificultosa. En el post-operatorio desarrolla una masa en bajo
vientre. La conducta correcta es:
a. llevarlo nuevamente al salón de operaciones
b. realizarle un ultrasonido de urgencia
c. colocarle sonda urinaria y luego retirarla
d. realizarle CAT abdominal y de pelvis
Todas las siguientes incrementan la tasa de infecciones en una cirugía mayor, EXCEPTO:
a. paciente mayor de 70 años
b. infecciones alejadas del sitio operatorio
c. diabetes mellitus controlada
d. ingesta de corticoides

La fuente de bacterias que causan infecciones en las heridas limpias más comúnmente es:
a. las manos del cirujano
b. el salón de operaciones
c. instrumental inadecuadamente esterilizado
d. el paciente

Un método práctico recomendado para el balance hídrico de un paciente en el post-operatorio es:


a. estimar el agua del cuerpo
b. presión arterial
c. frecuencia de pulso
d. pesarlo diariamente

Un paciente de 45 años estaba trabajando en un taller y solicito a su ayudante que arrancara el camión
para evaluar el motor, el ayudante poco ágil en el manejo perdió el control y apresiono a su compañero
contra la pared. Este es llevado al cuarto de urgencias el paciente presenta shock, escoriaciones en
extremidad y aumento en ambos muslos. El estado de shock que presenta el paciente es explicado:
a. por la perdida sanguínea asociada a fractura femoral bilateral.

Un adulto y un menor viajan en un vehiculo pequeño a una velocidad de 80 millas/hora. De ocurrir un


accidente, el factor predominante causante de las lesiones corporales seria:
a. la composición del objeto fijo
b. el peso y la masa de cada individuo
c. la velocidad y la aceleración que iban viajando
d. el peso y la masa del vehículo

Un hombre de 29 años de edad llega al servicio de urgencias luego de haber sufrido un accidente
automovilístico (colisión) se encuentra cianótico, tiene insuficiencia respiratoria a una escala de coma
glasgow de 6, no hay trauma facial significativo, su traquea se encuentra en línea media, tiene una
desviación crónica del tabique nasal que impide la intubación nasotraqueal, tiene abundante barba que
dificulta la fijación de la mascara de oxigeno, el paso siguiente apropiado es:
a. intentar una intubacion orotraqueal utilizando dos personas para mantener la columna alineada

Usted esta de turno en el servicio de urgencia y cuando le llega un paciente acompañado de TUM quien
le informa que el paciente recibió golpes con un garrote en la cabeza, el paciente esta comatoso, a la
palpación presenta una fractura deprimida de cráneo, la cara esta edematisada, y equimotica , su
respiración gorgoriante, y se encuentran restos de vomito. El paso mas apropiado a seguir:
a. succionar la orofaringe

Un hombre de 40 años sufre un traumatismo cerrado en el tórax al compactar el timón en un accidente


a alta velocidad con un objeto fijo, el paciente tiene:
a. neumotórax a tensión
Un motociclista de 20 años sufre una lesión importante en la cara al estrellar su motocicleta contra una
camioneta. El paciente llega a urgencias, usted es el medico, el paciente tiene presion arterial de
150/60, FC: 90x, arrítmico, Glasgow de 7. Usted intenta varias veces intubarlo tiene intubacion
orotraqueal manteniendo columna cerebral sin éxito, presencia de sangrado y anatomía distorsionada.
El mejor paso a seguir es:
a. Cricoidectomia con aguja e insuflación a presión

Hasta que se pruebe lo contrario usted debe considerar a un paciente de trauma que este belicoso,
combativo y no cooperador como:
a. diabético
b. intoxicado
c. impertinente
d. hipóxico

Un hombre de 62 años se produce una herida cortante en las manos con una botella de cerveza
cuando estaba de paseo en la playa. Acude a usted para suturarlo y refiere que tiene todas sus
vacunas desde la niñez y que hace 14 años le inyectaron toxoide tetánico. ¿Cuál sería su indicación
acerca de la prevención de infección y tétano?
a. Inmunoglobulina tetánica más toxoide tetánico
b. Toxoide tetánico y lavado con agua estéril o solución salina
c. Inmunoglobulina tetánica
d. Antibióticos más toxoide tetánico en solución Gaudiane

Después de un traumatismo quirúrgico mayor, a los 2 o 4 días podemos tener cada una de las
siguientes situaciones, EXCEPTO:
a. Aumento de la excreción de los esteroides adrenocorticales
b. Oliguria relativa
c. Disminución de la excreción de nitrógeno
d. Retención de sodio y potasio

Objetivos en la intubación endotraqueal:


R:/ Todas las anteriores.

Durante la revisión primaria cual de los siguientes hallazgos en un adulto indica manejo inmediato:
a. Abdomen distendido
b. Escala de coma de Glasgow de 11
c. Temperatura de 36.5ºC
d. Frecuencia cardiaca de 120 lpm
e. FR de 40 cpm

Los exámenes radiológicos más importantes que debemos obtener en el paciente traumatizado severo
son:
a. Cráneo, tórax y abdomen
b. Tórax, abdomen y pelvis
c. Cráneo, columna cervical y tórax
d. columna cervical, tórax y pelvis
e. columna cervical, tórax y abdomen
El timoma:
a. Es la neoplasia más común en el mediastino posterior
b. Puede desarrollar miastenia gravis
c. Dan metástasis a distancia con gran frecuencia
d. El tratamiento es la quimioterapia

La contusión pulmonar:
a. Es la lesión potencialmente mortal más común en la evaluación inicial
b. Siempre requiere de manejo con ventilación mecánica
c. Una vez realizada la reanimación, los líquidos deben ser administrados juiciosamente
d. El tratamiento incluye la fijación de la caja torácica

La peritonitis química ocurre debido a todos los siguientes, EXCEPTO:


a. Úlcera péptica perforada
b. Perforación de la vesícula
c. Hemoperitoneo
d. Lesión de cisterna quilosa

El mejor tipo de estudio de rayos X para localizar aire libre intraabdominal dudosa es una radiografía:
a. AP de tórax
b. Abdomen de pie y acostado
c. Decúbito lateral con rayos horizontales con el lado derecho hacia arriba
d. Tomografía computarizada de abdomen

La causa más común del abdomen agudo hemorrágico es:


a. Trauma abdominal
b. Hemorragia digestiva alta
c. Embarazo ectópico roto
d. Aneurisma intraabdominal roto

La característica clínica más importante para reconocer una quemadura de 2º grado es:
a. Venas trombosadas
b. Edema
c. Prurito
d. Vesículas

En las perforaciones del esófago:


a. La tomografía es el diagnóstico de elección
b. Actualmente la morbi-mortalidad es igual independiente del tiempo de evolución
c. Las perforaciones por trauma cerrado ocurre más frecuente
d. La morbi-mortalidad es menor en las lesiones del esófago cervical

CIERTO Y FALSO

F En el paciente de trauma, la hiperextensión del cuello de la cabeza facilita la maniobra de


intubación endotraqueal
F El shock cardiogénico es el más común por lesiones que se le producen al miocardio en las
heridas penetrantes
F Todo paciente de trauma hipotenso y frío está en shock hasta que se pruebe lo contrario
C En la hemorragia clase 1 presenta signos clínicos mínimos porque el organismo generalmente lo
compensa
F La sola permeabilidad de la vía aérea asegura una adecuada ventilación
C El nivel de conciencia alterado en trauma, es la causa más común de compromiso de las vías
aéreas
C Durante las primeras horas post-operatorias la causa más común de fiebre es la atelectasia
F El diagnóstico de neumotórax a tensión es radiológico
F En los tumores del mediastino posterior es particularmente importante la determinación de
marcadores tumorales
F La primera acción a tomar con paciente que se presenta con neumotórax abierto es la sutura
C La broncoscopía es la modalidad invasiva más valiosa en el diagnóstico de enfermedad
pulmonar
F Los tumores neurogénicos típicamente se encuentran en el mediastino anterior
C El tabaco es el principal responsable del cáncer de pulmón
F En el paciente de trauma al que se va intubar, la maniobra de alinear la columna cervical con
extensión de la cabeza es la indicada
F La escogencia entre el tratamiento quirúrgico, la quimio y radioterapia en el cáncer pulmonar se
basa en la edad del paciente
F El objetivo de la evaluación inicial es detectar lesiones que puedan ser potencialmente letales
C Una de las manifestaciones clínicas de intoxicación hídrica son las convulsiones
F El timoma es el tumor más frecuente de mediastino posterior

LLENA ESPACIOS

El desequilibrio ácido base se debe a perfusión y reanimación inadecuada. El tratamiento sería:


Oxigenar y ventilar
Administrar líquidos IV y sangre

En el manejo de vía aérea la primera prioridad es:


Asegurar vía aérea
Ventilar y oxigenar

Mencione 3 síndromes paraneoplásicos del cáncer broncogénico:


Síndrome de Cushing
Síndrome de secreción inapropiada de ADH
Hipercalcemia

La tríada de Beck está compuesta por:


Hipotensión arterial
Ingurgitación yugular
Disminución de los ruidos cardiacos

Mencione 2 funciones de líquidos en nuestro organismo:


Transporte de nutrientes y desechos desde y hacia la célula
Medio de reacciones bioquímicas

RECOMENDACIÓN: repasar los profetas de Fuentes… “ajustar” las respuestas a lo que él piense…
ABDOMEN  AGUDO   años   pero   no   sabe   qué   se   le   practicó.   Enumere   las   tres   posibilidades  
intestinales  que  puedan  obligar  a  una  intervención  quirúrgica:  
1.   La  causa  más  común  del  abdomen  agudo  hemorrágico  es:   a.   APENDICITIS  AGUDA  
a.   Trauma  abdominal   b.   DIVERTICULITIS  
b.   Hemorragia  digestiva  alta   c.   DIVERTICULO  DE  MECKEL  
c.   Embarazo  ectópico  roto   8.   __C__   el   origen   principal   de   los   gases   en   la   obstrucción   intestinal  
d.   Aneurisma  intraabdominal  roto   mecánica  es  el  aire  tragado  
2.   En   la   úlcera   péptica   perforada,   el   abdomen   en   tabla   aparece  
inicialmente  por:   ANESTESIA  
a.   Irritación  peritoneal  química  
b.   Abscesos  peritoneales   1.   Para   anestesia   local   por   infiltración   con   xilocaina,   la   dosis   máxima  
recomendada  es:  
c.   Irritación  peritoneal  bacteriana  
d.   DHE   a.   100  mgs  
3.   La   mayor   parte   del   gas   contenido   en   las   asas   intestinales   obstruidas   b.   200  mgs  
c.   300  mgs  
proviene  de:  
d.   400  mgs  
a.   Aire  que  traga  el  paciente  
b.   Fermentación  digestiva   e.   500  mgs  
2.   Los   factores  que   afectan  la  concentración   espiratoria   de   un   anestésico  
c.   Putrefacción  bacteriana  
d.   Todas   inhalado  son:  
4.   La   causa   más   común   de   abdomen   agudo   hemorrágico   es:   EMBARAZO   a.    volumen   del   circuito   respiratorio,   velocidad   del   flujo   de   gas  
ECTÓPICO.   fresco  
5.   Enumere  cuatro  imágenes  en  la  radiografía  simple  de  abdomen  agudo:   3.   Anestésico  más  comúnmente  usado:  
a.   AIRE  SUBDIAFRAGMÁTICO   a.    Sevoflurano.  
4.   La  potencia  de  los  anestésicos  inhalados  es  medida  por:  
b.   ASAS  INTESTINALES  DISTENDIDAS  
c.   NIVELES  HIDROAEREOS   a.      concentración  alveolar  mínima  
d.   FALTA  DE  AIRE  EN  EL  COLON   5.   Paciente   con   muerte   cerebral     para   transplante   de   órganos   es  
6.   Enumere  cuatro  signos  especiales  a  la  exploración  en  el  abdomen  agudo   clasificado  como:  
a.   SIGNO  DEL  PSOAS   a.   ASA  VI.  
b.   SIGNO  DEL  OBTURADOR   6.   Durante  la  anestesia  la  diseminación  del  anéstesico  depende  de:  
a.   Fuerza  con  que  se  inyectó  
c.   SIGNO  DE  McBURNEY  
d.   SIGNO  DE  ROVSING   b.   Gravedad    
7.   Una  señora  de  35  años  presenta  fiebre,  leucocitosis,  malestar  y  nauseas.   c.   Volumen  inyectado  
Tiene   una   cicatriz   antigua   sobre   fosa   iliaca   derecha   donde   presenta   d.   Lugar  de  la  inyección  
dolor,  defensa  y  rebote.  La  operación  se  llevó  a  cabo  cuando  tenía  16   e.   Concentración  del  anestésico    
7.   La  hipotensión  en  la  anestesia  raquídea  se  debe  a:  
a.   Bloqueo  neuromuscular   c.   CERRADO  
b.   Bloqueo  simpático     15.  Tipos  de  relajantes  musculares  
c.   Depresión  miocárdica   a.   DESPOLARIZANTES  
d.   hipovolemia   b.   NO  DESPOLARIZANTES  
8.   Cuál  de  las  siguientes  afirmaciones  es  cierta  en  relación  a  la  colocación   16.  Señale  los  receptores  opioides  
de  anestesia  local  para  una  cirugía  menor:   a.   Mu  
a.   Se  utiliza  marcaína  al  1-­‐2%   b.   Delta  
b.   Es   útil   el   uso   de   un   vasoconstrictor   (adrenalina   1:100   000)   en   c.   Kappa  
áreas  como  dedos  o  nariz   d.   Sigma  
c.   Se   administra   en   forma   subcutánea   desde   el   interior   hasta   el   17.  Escala  post-­‐operatorio  de  Aldrete,  indique  los  cinco  aspectos  que  toma  
exterior  de  la  herida   en  cuenta  
d.   El   anestésico  local  se  puede  utilizar   a   una   dosis  máxima   de  20   a.   RESPIRACIÓN  
mg/kg  de  peso   b.   CIRCULACIÓN  
e.   Es   muy   importante   interrogar   al   paciente   sobre   alergia   a   los   c.   ESTADO  DE  ALERTA  
anestésicos   d.   MOVILIDAD  
9.   La   hipotensión   en   la   anestesia   raquídea   se   debe   a:   BLOQUEO   e.   COLOR  
SIMPÁTICO  
10.  Señale  los  cuatro  componentes  de  la  anestesia  general   CANCER  DE  PIEL  
a.   INCONSCIENCIA  REVERSIBLE  
1.   Un   tumor   maligno   que   tiene   tendencia   a   presentar   regresión  
b.   AMNESIA  
espontánea  es:  
c.   ANALGESIA  
a.   Melanoma  maligno.  
d.   RELAJACIÓN  MUSCULAR  
b.   Carcinoma  broncogénico.  
11.  Tipos  de  anestésicos  locales  
c.   Hipernefroma.  
a.   AMIDAS    
d.   Tumor  de  Wilms.  
b.   ESTERES  
e.   CA  de  vejiga.  
12.  Señale  la  dosis  máxima  en  miligramos  (mg)  de  un  anestésico  local:  400  
2.   Paciente   de   40   años   que   practica   la   espeleología   como   profesión,  
mg    
presenta  una  lesión  color   café  con  leche  en  la  región  plantar  derecha.  
13.  La   absorción,   transporte   y   distribución   de   un   agente   anestésico  
Informa  que  está  aumentando  de  tamaño,  con  prurito  y  cambiando  su  
depende  de  tres  factores  
color  a  un  tono  más  oscuro.  Su  diagnóstico  sería:  
a.   SOLUBILIDAD  DEL  ANESTESICO  
a.   Tinea  pedis.  
b.   FLUJO  SANGUINEO  PULMONAR  
b.   CA  basocelular.  
c.   PRESIÓN  PARCIAL  DEL  ANESTÉSICO  
c.   CA  escamoso.  
14.  Indique  tres  tipos  de  sistema  de  anestesia  
d.   Nevus  de  superficie.  
a.   ABIERTO  
e.   Melanoma  maligno.  
b.   SEMIABIERTO  
3.   El  carcinoma  escamoso  es  más  frecuente  su  aparición  en:   1.   La   ayuda   diagnóstica   más   importante   para   el   diagnóstico   del   ano  
a.   Cualquier  parte  del  cuerpo   imperforado  es:  
4.   Cuándo  es  más  frecuente  el  carcinoma  basocelular:   a.   Serie  gastrointestinal.  
a.   Tercera  edad   b.   Radiografía  de  tórax.  
5.   En  qué  región  del  cuerpo  es  más  frecuente  el  carcinoma  basocelular:   c.   Serie  de  abdomen  agudo.  
a.   Cabeza  y  cuello   d.   Radiografía  de  abdomen  con  el  niño  en  posición  invertida  
6.   Cuál   de  los   cánceres   está   más   relacionado   con   el   estado   inmunológico   e.   Sonografía.  
del  paciente:   2.   Un  paciente  de  2  años  de  edad  procedente  de  San  miguelito  con  cuadro  
a.   Miosarcoma   de  obstrucción  intestinal,  la  causa  más  frecuente  es  
b.   Linfomas   a.   Bridas  
c.   Melanomas   b.   Hernia  inguinal  
7.   El  melanoma  maligno  en  mujeres  es  más  frecuente  en:   c.   Parásitos  intestinales  
a.   Extremidades  inferiores   d.   Intususepción  intestinal  
b.   Melanoma  visceral   3.   El  lugar  más  común  de  presentar  un  pólipo  en  un  niño  es:  
c.   Cabeza  y  cuello   a.   Ciego.  
8.   Melanoma  subungueal  en  la  clasificación  de  Clark  corresponde   b.   Colon  transverso.  
a.   Clark  II   c.   Ileon.  
b.   Clark  III   d.   Rectosigmoide.  
c.   Clark  V   e.   Yeyuno.  
9.   Una   señora   de   40   años   se   queja   de   ulceración   reciente   de   un   nevus   4.   La  causa  más  común  de  sangramiento  rectal  en  un  niño  es:  
desde   la   niñez   y   está   en   buena   salud.   El   tratamiento   inicial   más   a.   Colitis  ulcerativa.  
apropiado  sería:   b.   Fisura  anal.  
a.   Biopsia   c.   Intususcepción.  
b.   Fluorouracilo  local   d.   Divertículo  de  Meckel.  
c.   Radioterapia   e.   Pólipo  rectal.  
d.   Observación  con  corticoide  tópico   5.   Neonato   de   5   días,   vómitos   biliosos   y   marcada   distención   abdominal,  
10.  __C__el   carcinoma   basecelular   y   escamoso   son   más   frecuentes   en   la   sospecha  de  diagnóstico  excepto:  
piel  expuesta  al  sol   a.   Malrotación  intestinal  y  atresia  intestinal  ¿??  
11.  Entre  los  tumores  cancerosos,  el  más  común  es:  LA  PIEL   6.   Paciente   que   se   sospecha   de   Agangliosidosis   intestinal   congénita,   el  
12.  Ante  lo  signos  de  malignidad  de  un  nevus,  cuál  sería  el  procedimiento  a   mejor  método  diagnóstico  es:  
seguir:  BIOPSIA  EXCISIONAL   a.   Biopsia  rectal  
13.  El  melanoma  maligno  tiene  su  origen  en:  MELANOCITOS   7.   Paciente   escolar   masculino   que   presenta   súbitamente   dolor   a   nivel  
escrotal  con  cambio  de  coloración  violáceo,  lo  más  probable  es  que  se  
CIRUGÍA  PEDIÁTRICA   trate  de:  
a.   Torsión  testicular  
8.   Paciente   lactante   masculino   que   presenta   evacuaciones   con   sangre,   24.  La   edad   ideal   para   hacer   orquideopexia   en   un   testículo   que   no  
vómitos  biliosos,  su  diagnóstico  más  probable  sería:   descendió  es:  1  A  2  AÑOS  DE  EDAD  
a.   Invaginación  intestinal   25.  Cuáles  son  las  causas  de  sangrado  digestivo  bajo  más  comunes  en  niños  
9.   Método  estándar  terapéutico  en  el  manejo  de  invaginación  intestinal:   a.   INTUSUSCEPCIÓN    
a.   Reducción  hidrostática  con  control  tomográfico   b.   DIVERTICULO  DE  MECKEL  
10.  Si   el   paciente   presenta   Dextrocardia   y   dificultad   respiratoria,   el  
diagnóstico  más  probable  es:  HERNIA  DIAFRAGMÁTICA   CIRUGÍA  AMBULATORIA  
11.  Un  paciente  de  2  años  de  edad  procedente  de  San  miguelito  con  cuadro  
1.   La  biopsia  incisional  
de   obstrucción   intestinal,   la   causa   más   frecuente   es:   INTUSUSEPCIÓN  
a.   Se  aplica  a  lesiones  de  “grandes  dimensiones”,  dependiendo  de  
INTESTINAL  
la  ubicación  
12.  Neumonía  química  y  bacteriana  en  el  recién  nacido,  el  diagnóstico  más  
b.   Se  usa  una  incisión  en  rombo  que  debe  comprender  parte  de  la  
probable  es:  ATRESIA  ESOFÁGICA  Y  FÍSTULA  TRAQUEOESOFÁGICA  
lesión  y  parte  del  tejido  sano  vecino  
13.  Si  el  lactante  presenta      dolor  abdominal  intenso  y  masa  abdominal,  el  
c.   Debe  realizarse  adecuada  hemostasia  y  sutura  sin  tensión  
diagnóstico  más  probable  es:    MALROTACIÓN  INTESTINAL    
d.   Enviar  la  pieza  en  formol  10%  o  en  fresco  y  eventuales  cultivos  
14.  Si  el  lactante  presenta    vómitos  intestinales  que  se  inician  en  la  segunda  
e.   Son  cierta  todas  las  anteriores  
semana  de  vida,  el  diagnóstico  más  probable  es:  HIPERTROFIA  PILORICA  
2.   Son  algunas  de  las  ventajas  de  la  cirugía  ambulatoria,  excepto  
15.  Si   el   lactante   presenta   constipación   crónica   y   ampolla   rectal   vacía   el  
a.   Permite  aumentar  la  actividad  asistencial,  sin  estar  limitada  por  
diagnóstico  más  probable  es:  MEGACOLON  CONGÉNITO  
la  disponibilidad  de  camas.  
16.  Recién  nacida  con  un  solo  orificio  perineal,  el  diagnóstico  más  probable  
b.   Ahorro  de  estancia  y  camas  
es:  CLOACA  
c.   Los  recursos  ahorrados  se  pueden  destinar  a  otros  pacientes  
17.  Recién  nacido,  síndrome  de  Down  y  vómitos  verdes,  el  diagnóstico  más  
d.   Contribuye  a  disminuir  las  estancias  en  los  pacientes  ingresados  
probable  es:  OBSTRUCCIÓN  INTESTINAL  
e.   Al   incrementar   en   cantidad   disminuye   la   eficiencia   de   los  
18.  Recién   nacido   que   no   pasa   meconio   en   24   horas,   el   diagnóstico   más  
hospitales  
probable  es:  MEGACOLON  CONGENITO  
19.  Medida  inicial  en  paciente  con  invaginación  intestinal:  COLOCAR  SONDA  
ESTOMAGO,  INTESTINOS,  APENDICE,  COLON,  ANORECTAL  
NASOGASTRICA  
20.  Método   inicial   con   sospecha   de   invaginación   intestinal:   COLON   POR   1.   En  una  paciente  con  apendicitis  retrocecal  no  perforada:  
ENEMA  CON  BARIO   NO   Invariablemente   el   dolor   cambia   de  la   región   periumbilical  
21.  Vómitos   biliares   a   los   días   de   vida   después   de   ingesta   oral   normal:   al  CID.  
MALROTACIÓN  INTESTINAL   NO   Habitualmente  hay  hiperestesia  notable  en  el  CID.  
SÍ   Puede  presentarse  hematuria  microscópica.  
22.  Método  estándar  terapéutico  en  el  manejo  de  la  invaginación  intestinal:  
NO   La   cuenta   de   leucocitos   generalmente   está   por   debajo   de  
REDUCCIÓN  HIDROSTÁTICA  CON  CONTROL  TOMOGRÁFICO   10,000/mm3.  
23.  La  ayuda  diagnóstica  más  importante  en  el  manejo  del  ano  imperforado   NO   La  emesis  es  poco  habitual.  
es:  RADIOGRAFIA  DE  ABDOMEN  CON  EL  NIÑO  EN  POSICIÓN  INVERTIDA   2.   En   la   actualidad,   los   enfoques   propuestos   para   el   tratamiento   de  
pacientes  con  síndrome  de  Zollinger-­‐Ellison,  incluyen:  
SÍ   Administrar  Cimetidina.   NO   La   mortalidad   fetal   global   relacionada   con   apendicitis  
SÍ   Administrar  Omeprazol.   materna   es   de   10%   aproximadamente,   pero   puede  
SÍ   Vagotomía  para  células  parietales.   reducirse  empleando  progesterona.  
NO   Gastrectomía  parcial.   6.   Relacionado  con  el  fibroadenoma,  son  afirmaciones:  
SÍ   Gastrectomía  total.   SÍ   Puede   observarse   lesión   única   de   2   centímetros,   que  
3.   Son  afirmaciones  verdaderas  relacionadas  con  apendicitis  aguda:   clínicamente  corresponde  a  fibroadenoma  en  las  mamas  de  
NO     La   infección   es   el   factor   dominante   en   el   desarrollo   de   una  mujer  de  20  años  de  edad.  
apendicitis  aguda.   NO   Cuando   se   observa   una   lesión   que   clínicamente  
NO   La  pared  del  apéndice  normal  es  estéril.   corresponde  a  fibroadenoma  en  las  mamas  de  una  mujer  de  
SÍ   El   líquido   periapendicular   en   una   apendicitis   aguda   no   23  años  de  edad,  se  debe  practicar  mamografía.  
perforada,  generalmente  es  estéril.   SÍ   A   veces   no   es   posible   distinguir   entre   un   fibroadenoma  
SÍ   Se  pueden  cultivar  múltiples  microorganismos  en  el  líquido   juvenil  voluminoso  de  un  tumor  filodes.  
que  rodea  el  apéndice  perforado.   NO   La  superficie  de  corte  del  fibroadenoma  típico  se  retrae  a  lo  
SÍ   Citomegalovirus   puede   relacionarse   con   apendicitis   en   largo  de  la  línea  de  corte.  
pacientes  con  SIDA.   SÍ   Un   fibroadenoma   quizás   sea   doloroso   e   hiperestésico  
4.   En  la  apendicitis  aguda  en  niños  en  edad  pre-­‐escolar:   durante  el  embarazo.  
SÍ   Es  difícil  establecer  el  diagnóstico.   7.   Entre   los   trastornos   relacionados   con   mayor   incidencia   de   cáncer  
NO   Se  puede  desencadenar  un  ataque  cuando  se  administra  un   colorrectal  se  incluyen:  
catártico  fuerte  a  un  niño  estreñido.   SÍ   Poliposis  familiar.  
NO   Un   período   de   observación   cuidadoso   debe   preceder   a   la   SÍ   Síndrome  de  Gardner.  
decisión  de  operar  si  el  diagnóstico  es  dudoso.   NO   Enterocolitis  pseudomembranosa.  
SÍ   Una   complicación   frecuente   es   la   rotura   temprana   del   NO   Pólipos  juveniles.  
apéndice.   NO   Disentería  amebiana  crónica  
NO   Las  paredes  del  epiplón  mayor  se  encuentran  bastante  lejos   8.   Cuál   de   las   siguientes   características   de   úlcera   gástrica   constituyen  
del  área  y  por  lo  tanto,  la  peritonitis  generalizada  es  rara.   indicación  quirúrgica:  
5.   Cuáles   de   las   siguientes   afirmaciones   relacionadas   con   apendicitis   SÍ   Cuadro  clínico  con  hemorragia.  
durante  el  embarazo  son  correctas:   SÍ   Localizadas  en  la  curvatura  mayor.  
NO   La   incidencia   de   apendicitis   es   más   elevada   en   mujeres   SÍ   Acompañada  de  úlcera  duodenal.  
embarazadas   que   en   la   población   de   mujeres   no   grávidas   NO   Recurrencia  después  de  cicatrización  inicial.  
del  mismo  grupo  de  edad.   NO   Menos   de   50%   de   cicatrización   con   3   semanas   de  
NO   Se   puede   lograr   menor   mortalidad   fetal   tratando   la   tratamiento  médico.  
apendicitis   aguda   sin   operación,  con  líquidos  y  antibióticos   9.   Entre  los  síntomas  típicos  de  hemorroides  internas  se  incluyen:  
por  vía  intravenosa.   SÍ   Sangrado  sin  deposición.  
NO   En   la   mujer   embarazada,   la   cifra   elevada   de   leucocitos   es   NO   Sangre  mezclada  con  las  heces.  
útil  para  establecer  el  diagnóstico  de  apendicitis.   SÍ   Producción  de  moco.  
Sí   En   la   mujer   embarazada   se   puede   usar   laparoscopia   para   NO   Dolor.  
establecer  el   diagnóstico  de  apendicitis  en  casos  atípicos  y   SÍ     Prolapso.  
evitar  la  laparotomía.   10.  Cuáles   de   las   siguientes   afirmaciones   relacionadas   con   divertículo   de  
Meckel  son  correctas:  
SÍ   Es   un   verdadero   divertículo,   remanente   del   conducto   c.   Gastroscopia  
onfalomesentérico.   d.   Trago  de  Bario  
SÍ   Habitualmente   muestra   entrada   amplia   y   se   sitúa   sobre   el   15.  _A_  El  síndrome  de  Zollinger-­‐Ellison  incluye  
borde  antimesentérico    del  intestino  delgado.   a.   Hipersecreción  gástrica  
SÍ   En   general,   el   sangrado   se   observa   en   mucosa   gástrica   b.   Úlcera  péptica  intratable  
heterotópica.   c.   Tumor  de  las  células  no  beta  del  páncreas  
NO   Una  complicación  común  es  la  úlcera  del  divertículo.   16.  _B_  Las  causas  más  comunes  de  sangrado  digestivo  bajo  son:  
NO   Si   durante   una   operación   abdominal   se   observa   a.   Cáncer  colorrectal,  fisura  anal  y  hemorroides  
incidentalmente  un  divertículo  siempre  debe  extirparse.   b.   Hemorroides,  fístulas  y  fisuras  
11.  Entre   las   medidas   terapéuticas   eficaces   para   prevenir   o   controlar   la   c.   Diverticulosis,  cáncer  colorrectal  y  angiodisplasias  del  colon  
hemorragia   en   el   tracto   digestivo   alto   debido   a   gastroenteropatía   por   d.   Ninguna  de  las  anteriores  
estress:   17.  _D_   centelleo   con   glóbulos   rojos   marcados   es   positivo   cuando   el  
SI   Mantener  una  nutrición  adecuada   sangrado  es  de:  
NO   Alcalinización  gástrica   a.   5  MM/Minuto  
NO   Lavartelenol  intragástrico   b.   1  MM/Minuto  
SI   Reanimación  eficaz  durante  el  choque   c.   0.5  MM/Minuto  
SI   Vasopresina  intra-­‐arterial   d.   0.1  MM/Minuto  
  18.  _A_  El  tratamiento  inicial  de  la  diverticulitis  aguda  consiste  en:    
Seleccione  su  respuesta  en  la  siguiente  forma:   a.   Tubo  de  Levin  
  b.   Fluidos  IV  
A    si  1,  2  y  3  son  correctas   c.   Antibióticos  sistémicos  
B    si  1  y  3  son  correctas   d.   Sigmoidectomía    
C    si  2  y  4  son  correctas    
D    si  solo  4  es  correcta   19.  Usted   es   el   interno   de   un   hospital   y   es   llamado   para   evaluar   a   un  
E    si  todas  son  correctas   paciente  varón  de  23  años  con  historia  de  un  dolor  abdominal  que  en  la  
  últimas  12  horas  ha  aumentado  en  forma  progresiva  y  localizado  en  fosa  
12.  _A_  Durante  la  perforación  de  úlcera  péptica  ocurre:   iliaca   derecha.   Originalmente,   el   dolor   fue   periumbilical.   No   tiene  
a.   Inicio  súbito   hambre  y  comió  la  última  vez  hace  12  horas.  Presentó  nauseas  pero  no  
b.   Dolor  abdominal   ha  vomitado.  Anteriormente  ha  tenido  buena  salud.  Su  temperatura  es  
c.   Abdomen  en  tabla   de   37.8°C.   Su   presión   arterial   es   de   120/80,   pulso   regular   de   86.   Al  
d.   Si  sólo  4  es  correcta   examen,   el   corazón   y   los   pulmones   están   bien.   Al   examen   abdominal  
e.   Si  todas  son  correctas   había   dolor   a   la   palpación   en   fosa   iliaca   derecha   con   defensa.   No   se  
13.  _A_   Las   indicaciones   para   el   tratamiento   quirúrgico   de   la   úlcera   palpan  masas.  Los  ruidos  hidroaéreos  estaban  hipoactivos.  Se  quejó  de  
duodenal  son:   dolor  al  tacto  rectal  y  las  heces  no  mostraron  sangre  oculta.  La  cuenta  
a.   Perforación   de  blancos  era  de  13.200  con  70%  de  neutrófilos.  El  hematocrito  era  de  
b.   Hemorragia  masiva   43%.  El  diagnóstico  diferencial  debe  ser  entre:  
c.   Obstrucción   a.   APENDICITIS  AGUDA  
14.  _A_  El  manejo  inmediato  de  la  úlcera  duodenal  sangrante  consiste  en:   b.   DIVERTICULITIS  DEL  CIEGO  
a.   Trasfusión  de  sangre   c.   HERNIA  INGUINAL  
b.   Tubo  de  Levin  
20.  De  estos  exámenes,  cuáles  son  los  tres  que  usted  ordenaría  en  orden  de   A.   Rectosigmoidoscopía  flexible  
importancia.   B.   Colocar  tubo  nasogástrico  
A.   Centelleo  hepático   C.   Hacer  CAT  abdominal  
B.   Ultrasonografía  abdominal   D.   Colocar  Sonda  vesical  
C.   Rayos  X  de  tórax   E.   Solicitar  enema  de  Bario  
D.   Enema  de  Bario   F.   Administrar  lactato  Ringer  IV  
E.   Gases  arteriales   G.   Dar  corticoesteroide  
F.   Centelleo  con  Galio   H.   Antibióticos  por  boca  
G.   Amilasemia   I.   Radiografía  de  abdomen  
H.   Pielograma  endovenoso   J.   Enema  evacuante  
I.   Radiografía  de  abdomen  simple   K.   Radiografía  de  tórax  
J.   Serie  gastrointestinal   __B__  
K.   Urinalisis   __F__  
L.   CAT  Abdominal   __D__  
M.   Lavado  peritoneal   __I__  
__K__   __C__  
__I__   24.  Se   sabe   que   todos   los   siguientes   padecimientos   tienen   un   patrón  
__B__   familiar  relacionado  con  predisposición  a  cáncer,  excepto:  
21.  Qué  indicaría  usted  en  este  momento?   a.   Poliposis  de  colon.  
A.   Llamar  al  endoscopista  para  gastroesofagoscopia   b.   Cáncer  mamario.  
B.   Admitir  al  paciente  al  hospital  y  colocar  tubo  nasogástrico   c.   Síndrome  de  Peutz-­‐Jeghers  =  poliposis  intestinal  generalizada  y  
C.   Llamar  al  cirujano  de  turno  para  celiotomía  inmediata   máculas  pigmentadas.  
D.   Administrar  antibióticos  de  amplio  espectro  vía  oral   d.   Síndrome   de   Gardner   =   poliposis   familiar   autosómica  
E.   Observación  por  24  horas  en  casa   dominante.  
F.   Admitir  al  paciente  en  el  hospital  para  observación   e.   Retinoblastoma.  
__C__   25.  En   cuál   de   las   siguientes   circunstancias   puede   ser   útil   determinar  
  antígeno  carcinoembrionario  (ACE):  
22.  El  paciente  recibió  su  tratamiento  adecuado  y  se  le  dio  de  alta.  Dos  años   a.   Prueba  para  detección  masiva  de  cáncer  colorrectal.  
después,   acude   donde   usted   por   haberlo   tratado   muy   bien   y   haberle   b.   Como  indicador  general  de  la  extensión  de  cáncer  colorrectal.  
dado  muy  buena  atención  con  dolor  abdominal  que  no  puede  localizar   c.   Estudio   seriado   para   indicar   recurrencia   post-­‐operatoria   de  
con   precisión  y  se  siente  distendido.  Pasó  gases  y  heces  líquidas  en  la   cáncer  colorrectal.  
mañana.   Al   examen,   el   corazón   y   pulmones   normales.   Abdomen   d.   Predicción   de   la   respuesta   de   cáncer   colorrectal   a  
distendido.  Al  examen  rectal  se  queja  de  dolor.  No  hay  fiebre.  Los  ruidos   quimioterapia.  
hidroaéreos  están  hiperactivos.  La  cuenta  de  blancos  y  electrolitos  están   e.   Indicación  del  éxito  de  la  resección  de  tumor  colorrectal.  
dentro  de  los  límites  normales.  Cuáles  son  los  diagnósticos  diferenciales   26.  La  causa  más  común  de  hemorragia  masiva  en  tubo  digestivo  bajo  es:  
que  usted  haría:   a.   Carcinoma.  
a.   OBSTRUCCIÓN  INTESTINAL  POSTOPERATORIA   b.   Diverticulosis.  
b.   HERNIA  INTESTINAL   c.   Diverticulitis.  
23.  De   estos   procedimientos   o   estudios,   cuáles   recomendaría   hacer   en   d.   Pólipos.  
orden  de  importancia:   e.   Colitis  ulcerosa.  
27.  Un  hombre  de  25  años  de  edad  acude  a  la  sala  de  urgencias  por  haber   e.   Anemia  hemolítica  adquirida.  
tragado   6   horas   antes   2   alfileres   de   seguridad   abiertos.   La   radiografía   32.  El  carcinoma  del  estómago  ocurre  más  frecuentemente  en:  
muestra  los  alfileres  abiertos  en  intestino  delgado.  En  este  momento  el   a.   Fondo.  
tratamiento  más  apropiado  sería:   b.   Cardias.  
a.   Administración  IV  de  antibióticos  de  amplio  espectro.   c.   Cuerpo.  
b.   Un  ciclo  de  10  días  con  Metronidazol  oral.   d.   Antro.  
c.   Administración  de  250  mL  de   citrato  de  magnesio  para  inducir   e.   Unión  gastroesofágica.  
catarsis  e  incrementar  rapidez  del  tránsito  intestinal.   33.  Pólipos   juveniles   del   colon   y   recto   pueden   producir   todo   lo   siguiente  
d.   Seguimiento  con  radiografías  en  serie  y  examen  abdominal.   EXCEPTO:  
e.   Operación  inmediata.   a.   Raro  por  debajo  de  los  45  años.  
28.  Una   mujer   de   65   años   de   edad   presenta   melena  masiva   e   hipotensión   b.   Las  lesiones  son  múltiples  generalmente.  
acompañante,   que   se   corrigen   con   transfusión   sanguínea.   Luego   de   c.   Producen  anomalías  electrolíticas.  
endoscopia   gastrointestinal   alta   negativa,   el   siguiente   procedimiento   d.   Al  examen  histológico  se  observa  malignidad  con  frecuencia.  
que  debe  ejecutarse  para  definir  el  origen  del  sangrado  intestinal  debe   34.  La  peritonitis  química  ocurre  debido  a  todos  los  siguientes,  EXCEPTO:  
ser:   a.   Úlcera  péptica  perforada  
a.   Colonoscopia.   b.   Perforación  de  la  vesícula  
b.   Sigmoidoscopia.   c.   Hemoperitoneo  
c.   Arteriografía  selectiva.   d.   Lesión  de  cisterna  quilosa  
d.   Imagen  con  radionúclidos.   35.  Todas   las   siguientes   manifestaciones   extra   intestinales   de   la   colitis  
e.   Enema  de  bario  en  tubo  digestivo  bajo.   ulcerativa  remiten  con  la  resección  del  colon,  Excepto:  
29.  La  duplicación  del  tubo  digestivo  generalmente  ocurre  en:   a.   Eritema  nodoso  
a.   Esófago.   b.   Uveítis  
b.   Estómago.   c.   Artritis  
c.   Duodeno.   d.   Colangitis  esclerosante  
d.   Ileon.   36.  En  la  enfermedad  de  Crohn  todas  son  correctas,  Excepto:  
e.   Colon.   a.   La  relación  hombre/mujer  es  de  1:1  
30.  Todo   lo   siguiente   es   cierto   con   respecto   a   los   pólipos   del   estómago,   b.   Afecta  sólo  la  mucosa  del  intestino  
EXCEPTO:   c.   Puede  afectar  desde  la  boca  hasta  el  ano  
a.   Ocurren  en  una  mucosa  atrófica.   d.   Se  asocia  a  fistulas  en  el  ano  
b.   Aparecen  después  de  la  adolescencia.   37.  En  la  colitis  ulcerosa,  son  todas  correctas  excepto:  
c.   Pueden  ser  parte  de  una  enfermedad  familiar.   a.   La  Relación  hombre/mujer  10:1  
d.   Rara  vez  se  malignizan.   38.  Manifestaciones   extraintestinales   de   la   enfermedad   de   Crohn   incluye  
31.  Una   mujer   de   50   años   fue   admitida   con   historia   de   pérdida   de   peso   y  
todo  MENOS:  
anemia.   Una   SGD   reveló   un   defecto   de   llenado   en   el   antro   gástrico  
compatible  con  un  cáncer.  ¿a  qué  otro  tipo  de  anemia  está  relacionada   a.   Riesgo  de  malignidad  
ésta  lesión?   39.  La  enfermedad  de  Crohn  se  caracteriza  por:  
a.   Esferocitosis  hereditaria.   a.   Afecta  solamente  la  mucosa  
b.   Anemia  falciforme.   b.   Puede  afectar  todas  las  capas  del  intestino  
c.   Anemia  perniciosa.   c.   Solo  afecta  el  ileon  distal  (Ileitis  Terminal)  
d.   Anemia  mediterránea.  
d.   Ninguna  de  las  anteriores     a.   Salida  de  sangre  por  el  ano  
40.  Los  síntomas  más  común  de  la  enfermedad  de  Chron  son:   b.   Vómitos  de  sangre  
a.   Evacuaciones  mucosas  sanguinolentas   c.   Acción  del  ácido  clorhídrico  sobre  la  sangre  en  el  estómago  
b.   Dolor  y  masa  en  cuadrante  inferior  derecho   d.   Ninguna  
c.   Fiebre,  taquicardia  y  diarrea  con  sangre   47.    Según   la   estadística,   en   qué   nivel   se   encuentra   el   cáncer   colorrectal  
d.   Todas  las  anteriores   como  causa  de  muerte:  
41.  Las  causas  más  comunes  de  sangrado  digestivo  alto  son:   a.   Segunda  
a.   Gastritis  erosiva,  cáncer  gástrico,  várices  esofágicas   b.   Primera  
b.   Gastritis  erosiva,  enfermedad  ulceropéptica,  várices  esofágicas   c.   Tercera  
c.   Esofagitis,  cáncer  gástrico,  úlcera  duodenal   48.  Cuál  es  la  sintomatología  que  NO  es  característica  de  cáncer  de  recto:  
d.   Ninguna   a.   Cambio  de  hábito  de  defecación  
42.  El  estudio  de  elección  para  el  sangrado  digestivo  alto  es:   b.   Disminución  en  diámetro  de  heces  
a.   Serie  esofagogastroduodenal   c.   Intolerancia  a  las  grasas  
b.   Centelleo  con  glóbulos  rojos  marcados   49.  Cuál  es  la  principal  sintomatología  de  cáncer  de  colon  derecho:  
c.   Endoscopía  digestiva  alta   a.   Tumor  palpable  y  anemia  
d.   Todas   b.   Dolor  epigástrico  
43.  El  centelleo  con  glóbulos  rojos  marcados  es  positivo  cuando  el  sangrado   c.   Vesícula  biliar  palpable  e  ictericia  
es:   50.  Cuál  es  la  dieta  principal  para  evitar  el  cáncer  colorrectal  
a.   5  mm/min   a.   Rica  en  carbohidratos  
b.   1  mm/min   b.   Rica  en  grasa  animal  
c.   0.5  mm/min   c.   Rica  en  fibra  vegetal  
d.   0.1  mm/min   51.  Cuál  es  la  característica  del  cólico  biliar  
44.  Las  causas  más  comunes  de  sangrado  digestivo  bajo  son:   a.   Dolor  en  hipocondrio  derecho,  sin  fiebre  y  sin  leucocitosis  
a.   Cáncer  colorectal,  fisura  anal  y  hemorroides   b.   Ictericia  y  dolor  en  hipocondrio  derecho  
b.   Hemorroides,  fístulas  y  fisuras   c.   Dolor  en  hipocondrio  derecho  con  fiebre  y  leucocitosis  
c.   Divertoculosis,  cáncer  colorrectal  y  angiodisplasia  del  colon   52.  El  cáncer  colorrectal  es  más  frecuente  en  los  segmentos:  
d.   Ninguna   a.   Descendente  
45.  La   teoría   corriente  sobre  la  etiología  de  la   enfermedad   diverticular  del   b.   Transverso  
colon  es   c.   Recto-­‐Sigmoides  
a.   La  ingestión  frecuente  de  carnes   53.  En   la   úlcera   duodenal   complicada   con   estenosis,   el   síntoma   que  
b.   El  exceso  de  harina  refinada  en  la  dieta   prevalece  es:  
c.   Disminución  de  fibras  en  la  dieta   a.   Vómitos  biliosos  
d.   Factores  de  degeneración  de  la  pared  intestinal  con  la  edad   b.   Vómitos  postprandiales  
e.   Factores  hereditarios   c.   Vómitos  tardíos  
46.  El  término  melanomesis  corresponde  a:   d.   Vómitos  en  proyectil  
54.  En  un  paciente  de  70  años  con  cuadro  clínico  de  obstrucción  intestinal,   a.   Molestias  epigástricas  
la  causa  más  frecuente  es:   b.   Molestias  peri-­‐umbilicales  
a.   Bridas   c.   Molestas  en  la  fosa  iliaca  derecha  
b.   Hernias  inguinales   d.   Ninguna  
c.   Intususepción   e.   Todas  
d.   Neoplasia  de  colon   60.  La  apendicitis  aguda  puede  ser  producida  por:  
55.  Paciente   femenina   de   19   años   de   edad,   procedente   de   Penonomé,   a.   Fecalitos  
operada   hace   6   días   de   apendicitis   aguda   perforada,   cursa   con   b.   Inflamación  de  los  linfáticos  del  apéndice  
distensión   abdominal,   diarrea,   disuria,   fiebre   y   leucocitosis,   el   c.   Tumor  carcinoide  
diagnóstico  es:   d.   Ninguno  
a.   Obstrucción  intestinal   e.   Todos  
b.   Absceso  subfrénico   61.  En  El  factor  más  importante  en  la  etiología  de  la  apendicitis  aguda  es:  
c.   Absceso  pélvico   a.   Infección  bacteriana  
d.   Absceso  interasas   b.   Obstrucción  mecánica  
56.    Cuál  de  estos  hallazgos  es  el  que  usted  considera  MÁS  importante  en  la   c.   Insuficiencia  vascular  
apendicitis  aguda:     d.   Factores  químicos  
a.   Vómitos   e.   Tejido  linfoide  del  apéndice    
b.   Fiebre   62.  una   paciente   de   30   años   de   edad,   procedente   de   Darién,   con  
c.   Cuenta  de  blancos  elevada   obstrucción   intestinal,   deshidratación   grave,   fiebre   y   presencia   de  
d.   Defensa  en  la  FID     válvulas   conniventes   visibles   en   radiografía   simple   de   abdomen.   Su  
e.   Diarrea     manejo  es:  
57.  La   manifestación   clínica   más   importante   para   el   diagnóstico   de   la   a.   Tratamiento  médico  por  13  horas  
apendicitis  aguda  es:   b.   Sonda  nasogástrica,  hidratación  y  cirugía  en  4  horas  
a.   Rebote   c.   Sonda  nasogástrica,  hidratación  y  antibióticos  por  24  horas  
b.   Dolor  en  fosa  iliaca  derecha   d.   Tratamiento  médico  por  8  horas  
c.   Defensa  muscular   63.  El  tubo  de  Baker  o  Miller  Abott  está  indicado  en  la  obstrucción  intestinal  
d.   Ninguna   cuando  existe:  
e.   Todas   a.   Cáncer  de  colon  
58.  El  diagnóstico  de  apendicitis  aguda  puede  confundirse  con:   b.   Intususepción  intestinal  
a.   Inflamación  pélvica   c.   Recurrencia  de  obstrucción  
b.   Ileitis  regional   d.   Enfermedad  de  Crohn  
c.   Diverticulitis  del  ciego   64.  Paciente  femenina  de  20  años  de  edad,  procedente   de  aguadulce,  fue  
d.   Ninguna   operada  por  apendicitis  aguda  perforada  hace  una  semana,  ingresa  con  
e.   Todas   fiebre  alta,  disnea,  hipotensión  y  leucocitosis.  Su  diagnóstico  es:  
59.  Generalmente,  la  apendicitis  comienza  con:   a.   Deshiscencia  del  muñón  apendicular  
b.   Infección  de  la  herida  quirúrgica   71.  Una   mujer   de   21   años,   previamente   sana,   se   presenta   con   dolor  
c.   Sepsis  intrabdominal   abdominal   de   48   horas   de   evolución.   El   dolor   fue   inicialmente  
d.   Absceso  subfrénico   periumbilical,   y   posteriormente   se   localizó   en   cuadrante   inferior  
65.  Paciente  masculino  de  alto  riesgo  quirúrgico  que  presenta  perforación   derecho.   Tuvo   náuseas   y   disminución   del   apetito.   Niega   disuria.   Su  
de  úlcera  péptica  de  24  a  36  horas  de  evolución,  se  le  realizó  un  estudio   última   menstruación   fue   dos   semanas   antes.   Examen   físico:   febril  
con   gastrografía   que   no   reveló   extravasación   del   medio   a   la   cavidad   (38.2°C),  dolor  en  cuadrante  inferior  izquierda  y  con  defensa  muscular.  
peritoneal,   no   presenta   datos   de   peritonitis   generalizada,   se   decide   Tacto  rectal  normal.  Leucocitosis  moderada.  Diagnóstico  más  probable?  
realizar   manejo   no   quirúrgico   de   la   úlcera   perforada,   que   incluye,   a.   Gastroenteritis  
excepto:   b.   Pancreatitis  aguda  
a.   Succión  con  sonda  nasogástrica   c.   Apendicitis  aguda  
b.   Bloqueadores  H2  E.V.   d.   Enteritis  regional  
c.   Antibióticos   72.  En   la   obstrucción   intestinal   del   intestino   delgado,   cuál   es   el   síntoma  
d.   Octeotride   predominante:  
66.  En  relación  al  sangrado  por  enfermedad  diverticular  del  colon,  podemos   a.   Distención  abdominal  
afirmar:   b.   Cólico  abdominal  
a.   Es  la  causa  más  común  de  sangre  oculta  en  heces   c.   Vómitos  
b.   La  mayoría  de  los  pacientes  requieren  resección  del  colon  para   d.   falta  de  eliminación  de  heces  y  gases  
prevenir  resangrado   73.  La  causa  más  frecuente  de  la  obstrucción  intestinal  
c.   Cuando  el  sangrado  persiste,  la  gammagrafía  con  glóbulos  rojos   a.   Adherencias  
marcados  es  útil  para  el  diagnóstico   b.   Estrecheses  
d.   Rara  vez  se  resuelve  espontáneamente   c.   Vólvulo  
67.  Todo  lo  siguiente  es  cierto  en  lo  referente  a  hemorroides  trombosadas,   d.   Bezoar  
EXCEPTO:   e.   Intususcepsión  
a.   Se  cura  espontáneamente  en  24  horas   74.  Cuándo  aparece  el  signo  radiológico  de  pilas  de  monedas  o  signo  de  la  
68.  Tratamiento  principal  del  ileo  paralítico   escalera   en   la   radiografía   de   abdomen   en   cuadro   de   obstrucción  
a.   Fluidos  IV,  prostigmina,  Tubo  de  Levin   intestinal:  
69.  En  obstrucción  intestinal,  la  depleción  de  líquidos  es  debido  a:   a.   Cuando  hay  signos  de  perforación  
a.   Líquidos  retenidos  en  el  lumen  intestinal,  líquidos  perdidos  por   b.   Cuando  hay  edema  de  la  válvula  ileocecal  y  está  continente  
vómitos,  Succión  gástrica   c.   Cuando  hay  edema  de  las  válvulas  conniventes  
70.  La   incidencia   de   cáncer   colorectal   está   correlacionada   positivamente   d.   Cuando  hay  signos  de  isquemia  
con,  EXCEPTO   75.  Cuál  es  el  manejo  médico  inmediato  en  el  tratamiento  de  la  obstrucción  
a.   Ingesta  aumentada  de  carne   intestinal  
b.   Edad   a.   Venoclisis  con  lactato  
c.   Fumar   b.   Colocación  de  sonda  vesical  
d.   Ingesta  aumentada  de  grasas   c.   Colocación  de  sonda  nasogástrica  
d.   Realizar  tipaje,  Th,  prueba  cruzada  y  trasfundir  GRE   c.   Dolor  abdominal  en  cuadrante  inferior  derecho  
76.  Después   del   drenaje   de   un   absceso   perianal,   qué   debe   informarle   al   d.   Todas  las  anteriores  
paciente   82.  La  enfermedad  diverticular  es  más  frecuente  en:  
a.   Que  va  a  sangrar   a.   Duodeno  
b.   Que  va  a  tener  dificultad  para  evacuar   b.   Yeyuno  
c.   Que  posteriormente  va  a  tener  una  fistula  perianal   c.   Colon  transverso  
d.   Que  va  a  sentir  mucho  dolor   d.   Ciego  
77.  El  sitio  que  menos  presenta  cáncer  de  la  vía  digestiva  es:   e.   Sigmoides    
a.   Esófago   83.  Las  complicaciones  del  divertículo  de  Meckel  son  todas,  MENOS:  
b.   Estómago   a.   Sangramiento  
c.   Intestino  delgado   b.   Inflamación  
d.   Intestino  grueso   c.   Perforación  
e.   Ano-­‐recto   d.   Obstrucción  
78.  El  cáncer  gástrico  está  asociado  con:   e.   Malignización    
a.   Hipoacidez   84.  Los  factores  que  contribuyen  a  la  enfermedad  diverticular  incluye  todo  
b.   Alto  índice  de  incurabilidad   menos:  
c.   Alta  mortalidad  con  gastrectomía  subtotal   a.   Constipación  crónica  
d.   A  y  B   b.   La  edad  
e.   A  y  C   c.   Heces  con  gran  residuo  
f.   A,  B  y  C   d.   Contracciones  circulares  del  intestino  
79.  El  recurso  más  efectivo  en  el  diagnóstico  de  cáncer   gástrico  temprano   e.   Predisposición  heredada  
es:   85.  La  enfermedad  diverticular  inicia  en:  
a.   Serie  esofagogastroduodenal  con  doble  contraste   a.   Quinta  década  
b.   Tomografía  de  epigastrio   86.  En  relación  al  sangrado  por  enfermedad  diverticular  del  colon:  
c.   Ultrasonografía   a.   Cuando  el  sangrado  persiste,  la  gammagrafía  con  GR  marcados  
d.   Endoscopia   es  útil  para  el  diagnóstico  y  tratamiento  
80.  Un  paciente  con  historia  de  polipomatosis  familiar  se  le  hace  biopsia  de   87.  Momento  en  que  se  escuchan  los  ruidos  intestinales  de  lucha:  
uno  de  los  pólipos.  Qué  tipo  de  pólipos  se  espera  encontrar:   a.   Durante  el  dolor  y  el  cólico  
a.   Adenoma  velloso   88.  __C__  El  adenoma  velloso  se  considera  como  un  tumor  premaligno  
b.   Pseudopólipo   89.  __F__  La  apendicitis  aguda  no  debe  ser  diagnosticada  a  menos  que  se  
c.   Polipo  hiperplásico   halle  rigidez  en  la  fosa  iliaca  derecha  
d.   Polipo  adenomatoso   90.  __F__  la  sobrevida  de  casos  con  cáncer  gástrico  se  ha  mejorado  por  el  
81.  Los  síntomas  más  comunes  de  la  colitis  ulcerativa  son:   uso  más  generalizado  de  cirugía  radical  
a.   Evacuaciones  mucosas  sanguinolentas   91.  __F__  La  estrechez  de  un  segmento  del  ileo  terminal  con  obstrucción  en  
b.   Plastrón  en  cuadrante  inferior  derecho   radiografía  es  indicativo  de  cáncer  del  delgado  
92.  __F__   La   apendicitis   no   se   debe   diagnosticar   a   menos   que   presente   105.   Principal  sintomatología  del  cáncer  de  colon  transverso:  
dolor  y  defensa  en  fosa  iliaca  derecha.   a.   CAMBIOS  EN  EL  HÁBITO  DE  DEFECACIÓN  
93.  __C__  El  antígeno  carcino-­‐embrionario  (CEA)  es  útil  en  el  seguimiento  y   b.   TUMORACIÓN  PALPABLE  
manejo  de  casos  operados  de  cáncer  de  colon   106.   Principal  sintomatología  en  la  cáncer  de  colon  izquierdo:  
94.  __C__  El  megacolon  tóxico  no  es  indicación  para  cirugía  de  urgencia   a.   CAMBIOS  EN  EL  HÁBITO  DE  DEFECACIÓN  
95.  __F__  Los  pseudopólipos  en  caso  de  colitis  ulcerativa  son  premalignos   b.   DISTENSIÓN  ABDOMINAL  
96.  Un   paciente   de   60   años   con   historia   de   sangrado   y   dolor   durante   la   c.   DISMINUCIÓN  DEL  DIÁMETRO  DE  LAS  HECES  
evacuación,  mencione  dos  diagnósticos  diferenciales   107.   Sintomatología  del  cáncer  de  recto:  
a.   FISURA  ANAL   a.   CAMBIOS  EN  EL  HÁBITO  DE  DEFECACIÓN  
b.   CÁNCER  COLORRECTAL   b.   SANGRADO  TRANSANAL  
97.  Historia   de   epigastralgias   y   pirosis   crónica,   ahora   con   hemorragia   c.   DISMINUCIÓN  DEL  DIÁMETRO  DE  LAS  HECES  
digestiva  alta:  ULCERA  GASTRODUODENAL   108.   Cuál  es  la  dieta  principal  para  evitar  el  cáncer  colo-­‐rectal:  ALTA  EN  
98.  El   síntoma   más   común   de   la   enfermedad   de   Crohn   es:   FÍSTULA   FIBRAS  Y  BAJA  EN  GRASAS  
PERIANAL   109.   La   incidencia   de   cáncer   colorrectal   está   correlacionada  
99.  El   síntoma   más   común   de   la   colitis   ulcerativa   es:   HECES   positivamente  con:  
SANGUINOLENTAS   a.   EDAD  
a.   Las  causas  más  comunes  de  sangrado  digestivo  alto  son:     b.   INGESTA  AUMENTADA  DE  CARNE  
b.   GASTRITIS  ATRÓFICA   c.   INGESTA  AUMENTADA  DE  GRASAS  
c.   ENFERMEDAD  ACIDO  PÉPTICA   110.   El   centelleo   con   glóbulos   rojos   marcados   es   positivo   cuando   el  
d.   VÁRICES  ESOFÁGICAS   sangrado  es  de:  0.1  ML/MIN  
100.   El  Síndrome  de  Zollinger  Ellison  incluye   111.   La   angiografía   mesentérica   puede   indicar   el   sitio   de   hemorragia  
a.   HIPERSECRECIÓN  GÁSTRICA   (80%)  cuando  el  sangrado  excede:  0.5  ML/MIN    
b.   ÚLCERA  PÉPTICA  INTRATABLE   112.   Paciente  de  35  años,  femenina  que  informa  dolor  antes  y  después  
c.   TUMOR  DE  LAS  CÉLULAS  NO  BETA  DEL  PANCREAS   de   evacuar,   desde   hace   1   mes.   Presenta   sangrado   rojo   después   de  
101.   El   estudio   de   elección   para   evaluar   el   sangrado   digestivo   alto   es:   evacuar.   Dice   tener   una   tetita   de   carne   en   el   ano,   antecedentes   de  
ENDOSCOPÍA  DEL  TRACTO  DIGESTIVO  ALTO   estreñimiento   desde   hace   2   -­‐   3   días   y   que   tiene   2   familiares   con  
almorranas.  ¿Cuál  es  su  diagnóstico  y  a  donde  lo  refiere?  
102.   La  causa  principal  de  obstrucción  intestinal  en  pacientes  adultos  es:  
a.   FISURA  ANAL  
ADHERENCIAS   b.   COLOPROCTOLOGÍA  
103.   Las  causas  más  comunes  del  sangrado  digestivo  bajo  son:     113.   Enumere   3   causas   más   frecuentes   de   la   isquemia   arterial  
a.   CARCINOMA  COLORRECTAL   obstructiva  
b.   ANGIODISPLASIA  DEL  RECTO   a.   TRAUMA  
c.   DIVERTICULOSIS   b.   TROMBOS  
104.   Cuál  es  la  primera  sintomatología  del  cáncer  de  colon  derecho:     c.   EMBOLISMO  
a.   ANEMIA   114.   Masculino   de   65   años   con   historia   de   diarreas   intercaladas   con  
b.   TUMOR  PAPLABLE   estreñimiento  dese  hace  un  año.  Ha  tenido  varios  episodios  de  dolor  en  
fosa   iliaca   derecha   y   fiebre.   Se   ha   tratado   con   antibióticos   y   ha   122.   Hematuria  y  fecaluria  en  paciente  de  45  años,  le  hace  sospechar  de  
mejorado.  Niega  sangrados  vía  rectal,  niega  pérdida  de  peso.  Indique  3   fístula  enterovesical  por:  DIVERTICULOSIS  
diagnósticos  diferenciales   123.   La  clasificación  de  Dukes  es  útil  para  evaluar  el  pronóstico  de  cáncer  
a.   DIVERTICULOSIS   de:  COLON  
b.   CÁNCER  DE  COLON   124.   Dos  entidades  quirúrgicas  que  suelen  mejorar  con  una  ingesta  alta  
c.   COLON  IRRITABLE   en  fibras  
115.   Paciente   de   19   años   operada   hace   4   días   de   apendicitis   aguda   a.   CONTIPACIÓN  
perforada,   cursa   con   fiebre   y   leucocitosis,   no   vómitos,   no   diarrea,   no   b.   ENFERMEDAD  DIVERTICULAR  
dolor  abdominal.  Diagnóstico  y  manejo:   125.   Un   tumor   doloroso   que   se   presenta   súbitamente   en   la   región   anal  
a.   INFECCIÓN  DE  HERIDA  QUIRURGICA   sería  probablemente:  HEMORROIDES  TROMBOSADA  
b.   DEBRIDAR  HERIDA  QUIRURGICA??=   126.   La  sintomatología  de  la  hemorroides  trombosada  es:  DOLOR  
116.   Paciente  femenina  de  19  años  de  edad,  procedente  de  Penonomé,   127.   Patologías  del  ano  en  las  cuales  puede  aparecer  masa  dolorosa  en  
operada   hace   6   días   de   apendicitis   aguda   perforada,   cursa   con   24  horas  
distensión   abdominal,   diarrea,   disuria,   fiebre   y   leucocitosis,   el   a.   HEMORROIDES  INTERNA  TROMBOSADA  
diagnóstico  es:  ABSCESO  PÉLVICO   b.   ABSCESO  PERIANAL  
117.   Paciente  masculino  de  40  años  con  diagnóstico  de  síndrome  pilórico   128.   Dolor  de  la  fisura  anal  post  evacuación  es  debido  a:  HIPERTONÍA  
por   úlcera   duodenal,   acude   a   urgencias   con   historia   de   vómitos   de   7   129.   Hemorragia   digestiva   baja   en   un   anciano   sin   previa   historia   de  
días  de  evolución  y  que  hoy  ha  orinado  poco.  Informa  pérdida  de  peso.   enfermedad  y  asintomático,  actualmente  su  causa  sería:  DIVERTICULO  
Examen  físico:  signos  evidentes  de  deshidratación.   130.   Enumere  cuatro  síntomas  de  la  fístula  anorectal  
a.   Trastorno  metabólico:  ALCALOSIS  METABÓLICA   a.   SALIDA  DE  SECRECIÓN  PURULENTA  
b.   Qué  solución  parenteral  administraría:  LACTATO  RINGER   b.   DOLOR  ANORRECTAL  
c.   Manejo  posterior:  CIRUGÍA   c.   FIEBRE  Y  ESCALOFRIOS  
118.   una   paciente   de   30   años   de   edad,   procedente   de   Darién,   con   d.   POSICIÓN  ANTALGICA  
obstrucción   intestinal,   deshidratación   grave,   fiebre   y   presencia   de   131.   Enumere  cuatro  factores  desencadenantes  del  prolapso  rectal  
válvulas   conniventes   visibles   en   radiografía   simple   de   abdomen.   Su   a.   PARASITOS  EN  NIÑOS  (TRICOCEFALO)  
manejo  es:  SONDA  NASOGÁSTRICA,  HIDRATACIÓN  Y  CIRUGÍA.   b.   RELAJACIÓN  DEL  PISO  PÉLVICO  
119.   Paciente  femenina  de  20  años  de  edad,  procedente   de  aguadulce,   c.   MUJERES  CON  HISTORIA  DE  HISTERECTOMÍA  
fue  operada  por  apendicitis  aguda  perforada  hace  una  semana,  ingresa   d.   NO  FIJACIÓN  DEL  RECTO  AL  SACRO  
con   fiebre   alta,   disnea,   hipotensión   y   leucocitosis.   Su   diagnóstico   es:   132.   Diagnóstico  de  CUCI:  
SEPSIS  INTRABDOMINAL     a.   ANOSCOPIA  
120.   En  qué  resulta  la  enfermedad  de  Menetrier:  HIPOPROTEINEMIA   b.   COLONOSCOPIA  
121.   La  enfermedad  hemorroidal  interna  de  III  grado  se  caracteriza  por:   c.   RECTOSIGMOIDOSCOPIA  
a.   SANGRADO   133.   El  síntoma  principal  de  CUCI  es:  DIARREA  CON  SANGRE  
b.   PROLAPSO  QUE  SE  REDUCE  MANUALMENTE   134.   Cuando  el  colon  parece  un  tubo  de  plomo  en  el  enema  de  bario,  el  
diagnóstico  más  aceptable  es:  COLITIS  ULCERATIVA  
135.   El  síntoma  más  común  de  la  enfermedad  de  Crohn  es:  DOLOR   a.   La  tomografía  es  el  diagnóstico  de  elección  
136.   El   síntoma   más   común   de   la   colitis   ulcerativa   es:   DIARREA   CON   b.   Actualmente   la   morbi-­‐mortalidad   es   igual   independiente   del  
MOCO  Y  SANGRE   tiempo  de  evolución  
c.   Las  perforaciones  por  trauma  cerrado  ocurre  más  frecuente  
137.   El   síntoma   más   frecuente   en   la   obstrucción   intestinal   es:   DOLOR  
d.   La   morbi-­‐mortalidad   es   menor   en   las   lesiones   del   esófago  
COLICO   cervical    
138.   Qué  encuentras  en  una  punción  abdominal:   3.   Es  causa  de  esofagitis  por  Reflujo  Gastro  Esofágico  
a.   ERITROCITO   a.   Hipotonía  del  EEI  
b.   LEUCOCITOS   b.   Vaciamiento  gástrico  anormal  
c.   BILIS   c.   Motilidad  anormal  de  la  parte  inferior  del  esófago  
d.   AMILASA   d.   Todas    
e.   Ninguna  
e.   FIBRAS  O  RESIUDO  
4.   A   vista   de   los   conocimientos   actuales,   el   esófago   de   Barrett   es  
f.   BACTERIAS   probablemente  debido  a  :  
139.   En   la   úlcera   duodenal   complicada   con   estenosis,   el   síntoma   que   a.   Daño  de  la  mucosa  esofágica  por  reflujo  gastroesofágico  
prevalece:  VÓMITOS  TARDIOS   b.   Daño  de  la  mucosa  esofágica  por  ingesta  de  cáusticos  
c.   Daño   de   la   mucosa   esofágica   por   ingesta   de   alimentos   muy  
A.   Colitis  Ulcerosa   D   Mejora  con  dieta  alta  en  residuos   calientes  
B.   Enfermedad  de  Cronh   A   Puede  (n)  causar  megacolon  tóxico   d.   Animalias  del  desarrollo  (fenómeno  congénito)  
C.   Se  sospecha  cuando  hay  fístulas  peri   e.   Cualquiera  de  las  anteriores  
Colitis  Amebiana   B  
anal  complicada   5.   Los  divertículos  epifrenicos  suelen  ser  debidos  a:  
D.   Ninguna   de   las   antes   a.   Anomalías  del  desarrollo  embrionario  
C   Afecta  solo  la  mucosa  del  colon  
mencionadas   b.   Debilidad  de  la  pared  esofágica  
E.   Todas   las   antes   c.   Factores  dietéticos  
C   No  hay  degeneración  maligna  
mencionadas   d.   Obstrucción  distal  del  esófago  causada  por  espasmo  o  acalasia  
  e.   Tracción  por  procesos  cicatriciales  de  vecindad  
6.   La  acalasia  esofágica  es:  
ESOFAGO   a.   Un  tumor  maligno  del  esófago  
b.   Un  trastorno  de  la  motilidad  esofágica  
1.   Cuál   de   las   siguientes   afirmaciones,   respecto   al   síndrome   de   Mallory-­‐ c.   Resultado  del  reflujo  gastroesofágico  
Weiss  son  correctas:   d.   Proceso  hereditario  
NO   El   principal   dato   patológico   es   la   perforación   espontánea   del   e.   Síntomas  del  divertículo  esofágico  
esófago.   7.   La  patología  del  síndrome  de  Mallory  Weiss  es:  
SÍ   El  principal  síntoma  es  hemorragia  digestiva  después  del  vómito.   a.   Laceración  o  rasgadura  de  la  unión  gastroesofágica  
SÍ   Comúnmente   se   relaciona   con   ingestión   crónica   de   bebidas   8.   La  etiopatogenia   de   la   acalasia   es:   TRASTORNO   DE   LA   MOTILIDAD   DEL  
alcohólicas.   ESÓFAGO  
NO   La  endoscopia  está  contraindicada  debido  al  peligro  de  perforación.  
9.   Vómito   de   contenido   alimentario   y   de   jugo   gástrico   seguido   por  
NO   Tratamiento  quirúrgico  con  prontitud  es  el  tratamiento  preferido.  
  hemorragia  digestiva:  SINDROME  DE  MALLORY  WEISS  
2.   En  las  perforaciones  del  esófago:  
10.  La  ruptura  de  la  mucosa  en  la  unión  gastroesofágica  produce  sangrado   mesogastrio   e   irradiado   a   ambos   cuadrantes   superiores   y   hacia   la  
denominado:  SINDROME  DE  MALLORY  WEISS   espalda  en  ambos  lados.  En  los  dos  últimos  años  había  tenido  cuadros  
11.  Los  divertículos  epifrénicos  suelen  ser  debidos  a:  OBSTRUCCIÓN  DISTAL   similares.  Estos  se  asociaban  a  la  ingesta  de  comidas  grasientas.  La  orina  
DEL  ESÓFAGO  CAUSADA  POR  ESPASMO  O  ACALASIA   se   le   oscurece   en   este   último   episodio.   Sin   cambios   en   los   hábitos  
12.  Enumere  2  causas  de  esofagitis  por  RGE:   evacuacionales.    Al  examen  físico  se  encuentra  una  mujer  bien  nutrida.  
a.   MOTILIDAD  ANORMAL  DE  LA  PARTE  INFERIOR  DEL  ESÓFAGO  
Su  presión  arterial  normal,  temperatura  de  38.9  °C.  Las  escleras  estaban  
b.   VACIAMIENTO  GÁSTRICO  ANORMAL  
c.   HIPOTONÍA  DEL  EEI   ictéricas   y   el   abdomen   marcadamente   distendido   con   dolor   a   la  
13.  A   la   vista   de   los   conocimientos   actuales,   el   esófago   de   Barrett   es   un   palpación   en   ambos   cuadrantes   superiores,   sin   masas.   Ligero   rebote.  
fenómeno  probablemente  debido  a:  DAÑO  DE  LA  MUCOSA  ESOFÁGICA   Examen   rectal   normal   con   heces   chocolates.   Los   estudios   iniciales  
POR  RGE   fueron   11.300   glóbulos   blancos   /cu   mm.   Con   88%   de   neutrófilos;   Hb  
14.  La   mucosa   esofágica   está   compuesta   por   una   capa   de   epitelio:   12.5   gh/100   ml;   electrolitos   normales.   Bilis   positiva   en   orina.   Su  
ESCAMOSO  ESTRATIFICADO  NO  QUERATINIZADO   diagnóstico  diferencial:  
15.  La  pared  del  esófago  carece  de:  SEROSA  
A.   Hepatitis  viral  
16.  Cuál   debe   ser  la   secuencia   diagnóstica   para   el   estudio   de   un   trastorno  
motor  esofágico:  PH-­‐METRIA  Y  TOMA  DE  BARIO   B.   Colecistitis  aguda  
  C.   Pancreatitis  aguda  
HEPATICAS  Y  BILIAR   D.   Gastritis  aguda  
E.   Obstrucción  intestinal  
Seleccione  su  respuesta  en  la  siguiente  forma:   F.   Coledocolititasis  
  G.   Crisis  hemolítica  
A    si  1,  2  y  3  son  correctas   1  __F__  
B    si  1  y  3  son  correctas  
2__B__  
C    si  2  y  4  son  correctas  
D    si  solo  4  es  correcta   3__C__  
E    si  todas  son  correctas   4.   Cuál  de  estos  exámenes  de  laboratorio  pediría  en  orden  de  importancia  
  A.   Examen  de  heces  
1.   _A_  El  ileo  por  cálculo  biliar  se  caracteriza  por   B.   Nitrógeno  de  urea  
a.   Aire  en  el  árbol  biliar   C.   Lipasa  en  suero  
b.   Dilatación  en  el  intestino  delgado   D.   Amilasa  en  suero  
c.   Sombra  de  cálculo  en  la  fosa  iliaca  derecha   E.   Dehidrogenasa  láctica  
2.   _A_  Los  síntomas  de  la  coledocolitiasis  son:   F.   Bilirrubinemia  
a.   Ictericia   G.   Glicemia  
b.   Cólico  biliar   H.   Tiempo  de  protrombina  
c.   Fiebre   __F__  
3.   Usted  es  llamado  para  atender  a  una  mujer  de  39  años  con  historia  de   __C__  
24  horas  de  dolor  agudo  y  severo  en  el  abdomen  superior  acompañado   __D__  
de   náuseas   y   vómitos.   El   dolor   era   tipo   cólico   localizado   en   el  
5.   Qué   estudios   radiográficos   pediría,   de   los   anunciados,   en   orden   de   d.   Rastreo  con  TC.  
importancia   e.   Estudio  con  radionúclidos.  
A.   Colecistografía  endovenosa   8.   Cuál  es  el  tratamiento  quirúrgico  de  la  colecistitis  aguda  
B.   Enema  de  bario   a.   Colecistostomía  
C.   Radiografía  de  tórax   b.   Colecistectomía  
D.   Centelleo  hepático   c.   Exploración  de  la  vía  biliar  
E.   Radiografía  simple  de  abdomen   9.   Cuál  es  la  principal  causa  de  colangitis:  
F.   Colecistografía  oral   a.   Parasitosis  
G.   Ultrasonografía  de  abdomen  superior   b.   Litiasis  
H.   Arteriografía  mesentérica   c.   Cáncer  
__G__   10.  En  una  paciente  diabética  de  65  años  con  cuadro  de  colecistitis  aguda  el  
_____   tratamiento  es:  
__E__   a.   Antibiótico  terapia  
6.   De  cuál  de  estos  consideraría  usted  como  tratamiento  inicial   b.   Manejo  de  diabetes  
A.   Colocar  tubo  nasogástrico   c.   Colocar   sonda   naso   gástrica,   antibiótico   terapia   y   manejo   de  
B.   Monitorear  la  presión  venosa  central   diabetes  
C.   Lactato  Ringer  100cc  por  hora   d.   Manejo  médico  inmediato  y  ante  todo  quirúrgico  
D.   Dar  antibióticos   11.  Los  niveles  de  alfa  feto  proteína  están  más  elevados  en:  
E.   Dar  Vit  K  I.M.   a.   Hepatoma.  
F.   Colocar  catéter  de  Foley   b.   CA  de  colon.  
G.   Dar  un  laxativo  oral   c.   Enfermedad  de  Crohn.  
H.   Balance  hídrico  horario   d.   CA  de  páncreas.  
I.   Preparar  para  una  pancreatografía   e.   CA  de  vesícula.  
J.   Solicitar  una  colangiografía  endoscópica   12.  Un   paciente   con   ictericia   en   la   piel   y   mucosas,   con   urobilinogeno  
__C__   negativo  en  la  orina.  La  ictericia  es  de  origen:  
__F__   a.   Hepatocelular  
__H__   b.   Hemolítica  
__D__   c.   Obstructiva  
__J__   d.   Secundaria  a  Sepsis  
  13.  El   cuadro   clínico   del   ileo   biliar   incluye   los   siguientes   parámetros,  
7.   El  procedimiento  inicial  más  adecuado  para  definir  la  causa  de  ictericia   EXCEPTO:  
obstructiva  en  un  hombre  de  75  años  de  edad  es:   a.   Aire  en  las  vías  biliares  
a.   Colangiopancreatografía  endoscópica  retrógrada  (CPRE).   b.   Obstrucción  del  intestino  delgado  
b.   Colangiografía  transhepática  percutánea  (CTP).   c.   El  cálculo  en  encuentra  en  el  sitio  de  la  obstrucción  
c.   Ultrasonografía.   d.   Ictericia  obstructiva  
14.  La   cirrosis   hepática   de   Laenec   o   Alcohólica,   según   la   clasificación   del   c.   Remover  el  cálculo,  si  está  presente  
síndrome  de  hipertensión  portal  es:   d.   Descomprimir  la  vía  biliar  
a.   Pre-­‐sinusoidal  extrahepática   21.  __F__   La   presencia   de   ictericia   en   un   caso   de   colecistitis   aguda   es  
b.   Pre-­‐sinusoidal  intrahepática   patonogmónico  de  coledocolitiasis.  
c.   Post-­‐sinusoidal  intrahepática   22.  __C__  la  fosfatasa  alcalina  y  la  bilirrubina  directa  tienden  a  elevarse  en  
d.   Post-­‐sinusoidal  extrahepática   ictericia  obstructiva.    
15.  Cuál  es  la  consecuencia  de  la  hipertensión  portal   23.  Un   señor   de   58   años   con   historia   de   tomar   licor   diariamente   por  
a.   Síndrome  de  Mallory  Weiss   muchos   años   llega   con   hemoglobina   de   5   g/dL,   historia   de   melena   y  
b.   Obstrucción  intestinal   vómitos   con   sangre,   ascitis,   ictericia   marcada   y   anasarca.   El   primer  
c.   Várices  esofágicas   estudio   diagnóstico   de   la   causa   hemorrágica   será:   ENDOSCOPIA  
d.   Ulcera  péptica  perforada   ESOFÁGICA.  En  caso  de  que  sea  cirrótica,  la  unicaría  en  la  clasificación  
16.  Recomendación  para  paciente  con  cirrosis  hepática  Child  B:   CHIDS  como:  CHIDS  C  
a.   Ligadura  de  várices  esofágicas   24.  Un   paciente   con   ictericia   en   la   piel   y   mucosas,   con   urobilinogeno  
b.   Esplenectomía   negativo  en  la  orina.  La  ictericia  es  de  origen:  HEMOLÍTICA  
c.   Trasplante  de  hígado   25.  En  un  paciente  diabético  de  65  años  de  edad  con  cuadro  de  colecistitis  
d.   Tratamiento  médico   aguda,  el  tratamiento  es:  QUIRURGICO  
17.  Lo  que  aparece  más  frecuentemente  en  la  hipertensión  portal  es:   26.  Cuáles  son  las  características  que  se  presentan  en  el  cólico  biliar:  
a.   Esplenomegalia   a.   DOLOR  EN  EL  HIPOCONDRIO  DERECHO  
b.   Hepatomegalia   b.   SIN  LEUCOCITOSIS  
c.   Varices  esofágicas   c.   SIN  FIEBRE  
d.   Ascitis   27.  Cuál  es  la  causa  de  colangitis:  LITIASIS  
18.  La  anemia,  leucopenia  y  plaquetopenia  en  hipertensión  portal  se  debe:   28.  Síntomas  de  la  coledocolitiasis  
a.   Sangrado  digestivo   a.   DOLOR  BILIAR  
b.   Hiperesplenismo   b.   ICTERICIA  
c.   Desnutrición   c.   FIEBRE  
d.   Infección   29.  El  ileo  por  cálculo  biliar  se  caracteriza  por:  
19.  En   un   paciente   con   cirrosis   hepática   que   desarrolla   un   hepatoma,   la   a.   PRESENCIA  DE  AIRE  EN  LAS  VIAS  BILIARES  
prueba  más  sensitiva  es:   b.   DILATACIÓN  DEL  INTESTINO  DELGADO  
a.   Alfafetoproteina   c.   CALCULO  A  NIVEL  DEL  PSOAS  (FID)  
b.   Alanina  Aminotrasnferasa   30.  En   un   caso   de   ictericia   obstructiva   severa,   el   procedimiento   más  
c.   Glutamil  Trasnpeptidasa   indicado  para  el  diagnóstico  de  colelitiasis  y/o  coledocolitiasis  sería  
d.   Aspartato  aminotrasnferasa   a.   ULTRASONIDO  
20.  El  objetivo  inicial  en  la  terapia  de  la  colangitis  aguda  es:   b.   COLANGIOPANCREATOGRAFIA  RETROGRADA  ENDOSCOPICA  
a.   Aliviar  la  ictericia  y  prevenir  el  daño  hepático   31.  Objetivo  inicial  de  la  terapia  en  la   colangitis  aguda  es:  DESCOMPRIMIR  
b.   Prevenir  el  desarrollo  de  pancreatitis   LAS  VIAS  BILIARES  
32.  La   Cirrosis   hepática   es   causante   de   una   hipertensión   portal:   POST   c.   Fibroblastos  
SINUSOIDAL  INTRAHEPATICA   d.   Capilares  sanguíneos  
33.  Cuál   es   la   recomendación   que   usted   le   da   a   un   Paciente   con   cirrosis   e.   Ninguno  de  los  de  arriba  
hepática  CHILD  B:  TRASPLANTE  DE  HIGADO   5.   Los   factores  que   contribuyen   a   la   dehiscencia  de   la   herida   se   incluyen  
34.  La  clasificación  de  CHILD  toma  en  cuenta  los  siguientes  criterios:   todos,  MENOS:  
a.   BILIRRUBINA  (mg/dL)   a.   Tos  
b.   ALBUMINA   b.   Ancianidad  
c.   PRESENCIA  DE  ASCITIS   c.   Hipoproteinemia  
d.   PRESENCIA  DE  ENCEFALOPATÍA   d.   Anemia  
e.   PRESENCIA  DE  MALNUTRICIÓN   e.   Malignidad  
6.   Mencione  los  5  factores  de  crecimiento  en  la  reparación  de  la  herida  
HERIDAS   a.   FACTOR  DE  CRECIMIENTO  DERIVADO  DE  PLAQUETAS  
b.   FACTOR  DE  CRECIMIENTO  EPIDERMICO  
1.   Una  herida  abierta  se  contrae  por:  
c.   FACTOR  ALFA  TRANSFORMANTE  DE  CRECIMIENTO  
a.   Acercamiento  de  la  piel  alrededor  
b.   Por  injerto  de  piel   d.   FACTOR  BETA  TRANSFORMANTE  DE  CRECIMIENTO  
c.   Por  proliferación  miofibroblástica   e.   FACTOR  DE  CRECIMIENTO  FIBROBLASTICO  
d.   Por  acción  de  los  linfocitos   7.   Enumere  4  fases  de  la  reparación  de  una  herida  
2.   Cuál   de   las   siguientes   es   absolutamente   esencial   en   la   cicatriz   de   las   a.   FIBROPLASIA  
heridas  quirúrgicas:   b.   EPITELIZACIÓN  
a.   Vitamina  D   c.   CONTRACCIÓN  
b.   Carbohidratos  
d.   FUERZA  DE  LA  HERIDA  
c.   Ingesta  calórica  
d.   Vitamina  C   8.   Enumerar   la   clasificación   de   las   heridas   de   acuerdo   con   la  
e.   Dieta  hipoproteica   estimación   clínica   de   la   contaminación   microbiana   y   al   riesgo   de  
3.   Un   hombre   de   62   años   se   produce   una   herida   cortante   en   las   manos   infección  subsiguiente  
con  una  botella  de  cerveza  cuando  estaba  de  paseo  en  la  playa.  Acude  a   a.   HERIDA  LIMPIA,  1.5  –  5.1%  
usted   para   suturarlo   y   refiere   que   tiene   todas   sus   vacunas   desde   la   b.   HERIDA  LIMPIA  CONTAMINADA,  7.7  –  10.8%  
niñez  y  que  hace  14  años  le  inyectaron  toxoide  tetánico.  ¿Cuál  sería  su   c.   HERIDA  CONTAMINADA,  15.3  –  16.2%  
indicación  acerca  de  la  prevención  de  infección  y  tétano?   d.   HERIDA  SUCIA,  28  –  40%  
a.   Inmunoglobulina  tetánica  más  toxoide  tetánico   9.   En  el  manejo  de  las  heridas  contaminadas,  el  factor  más  importante  
b.   Toxoide  tetánico  y  lavado  con  agua  estéril  o  solución  salina   para  evitar  las  infecciones  es:  DESBRIDAMIENTO  DE  TODO  EL  TEJIDO  
c.   Inmunoglobulina  tetánica   NECRÓTICO  
d.   Antibióticos  más  toxoide  tetánico  en  solución  Gaudiane   10.  Factores  involucrados  en  la  reparación  de  las  heridas  
4.   Durante  el  proceso  de  cicatrización  el  colágeno  es  producido  por:   a.   FIBROBLASTOS  
a.   Células  epiteliales   b.   PLAQUETAS  
b.   Células  endoteliales   c.   FACTOR  DE  CRECIMIENTO  TRANSFORMANTE  ALFA  Y  BETA  
11.  Fibroplasia/Patofisiología  de  la  reparación  de  heridas   para   caminar,   no   refiere   vómitos   o   cólicos   abdominal.   El   Diagnóstico  
a.   INFLAMACIÓN   más  probable  es:  
b.   PROLIFERACIÓN   a.   Testículo  criptorquídico  
c.   MADURACIÓN   b.   Quiste  de  cordón  inguinal  
c.   Hidrocele  
HERNIAS   d.   Hernia  inguinoescrotal  
6.   El  saco  herniario  en  la  hernia  inguinal  indirecta,  realiza  un  recorrido  por:  
1.   En  relación  al  conducto  inguinal,  cuál  de  los  conceptos  es  verdadero:  
EL  CORDON  INGUINAL  
a.   El   límite   superior   lo   constituye   la   aponeurosis   del   músculo  
7.   En   relación   al   conducto   inguinal,   escriba   su   límite   lateral:   LIGAMENTO  
transverso  del  abdomen  
INGUINAL  
b.   El  límite  inferior  es  el  ligamento  de  Cooper  
8.   Escriba   el   límite   medial   del   agujero   femoral:   LIGAMENTO   DE  
c.   En  su  contenido  se  encuentra  el  nervio  genito-­‐femoral  
GIMBERNAT  O  LACUNAR  
d.   El  límite  lateral  lo  constituye  el  ligamento  inguinal  
9.   En  relación  a  los  límites  del  agujero  femoral:  
2.   En   relación   a   los   límites   del   agujero   femoral,   cuál   de   los   siguientes  
a.   Límite  anterior:  LIGAMENTO  INGUINAL  (POUPART)  
conceptos  es  verdadero  
b.   Límite  posterior:  LIGAMENTO  PECTINEO  (COOPER)  
a.   El  límite  medial  lo  constituye  el  ligamento  de  Cooper  
c.   Límite  medial:  LIGAMENTO  LACUNAR  (GIMBERNAT)  
b.   El  límite  lateral  lo  constituye  el  ligamento  inguinal  
d.   Límite  lateral:  VASOS  FEMORALES  
c.   El  límite  anterior  lo  constituye  la  cintilla  iliopectinea  
10.  Cuando  una  de  las   paredes  del  órgano  contenido  en   una  hernia  forma  
d.   El  límite  medial  lo  constituye  el  ligamento  de  Gimbernat  
parte   y   está   adherido   al   saco   herniario,   a   la   hernia   se   le   denomina:  
3.   El  saco  herniario  en  la  hernia  inguinal  indirecta,  realiza  su  recorrido  por:  
HERNIA  DESLIZANTE  
a.   Conducto  inguinal  
11.  Fisiopatología   de   la   hernia   inguinal   indirecta:   DEBILIDAD   DE   LA   PARED  
b.   Cordón  inguinal  
POSTERIOR  O  FASCIA  TRANSVERSALIS  
c.   Fascia  transversa  
12.  Hernia  donde  parte  del  saco  herniario  es  parte  de  la  pared  intestinal  se  
d.   Anillo  inguinal  
llama:  HERNIA  DE  RICHTER  
4.   Cuando  una  de  las   paredes  del  órgano  contenido  en  una  hernia  forma  
13.  Hernia   que   produce   encarcelamiento   sin   causar   obstrucción:   HERNIA  
parte  y  está  adherido  al  saco  herniario,  la  hernia  se  denomina:  
DESLIZANTE  
a.   Hernia  de  Richter  
14.  Paciente  de  35  años  de  edad,  procedente  de  Colón  con  historia  de  masa  
b.   Hernia  en  pantalón  
en  la  región  inguino  escrotal  izquierda.  Hace  5  años  que  dicha  masa  no  
c.   Hernia  de  Litre  
aumenta  ni  disminuye  con  los  esfuerzos,  no  se  reduce.  El  Paciente  tiene  
d.   Hernia  deslizante  
la  sensación  de  que  el  testículo  ha  desaparecido,  presenta  incomodidad  
5.   Paciente  de  35  años  de  edad,  procedente  de  Colón  con  historia  de  masa  
para   caminar,   no   refiere   vómitos   o   cólicos   abdominal.   El   Diagnóstico  
en  la  región  inguinoescrotal  izquierda.  Hace  5  años  que  dicha  masa  no  
más  probable  es:  HIDROCELE  
aumenta  ni  disminuye  con  los  esfuerzos,  no  se  reduce.  El  paciente  tiene  
15.  Paciente  de  sexo  masculino  de   35  años  de  edad,  que  hace  6  semanas  
la  sensación  de  que  el  testículo  ha  desaparecido,  presenta  incomodidad  
observó  una  masa  en  la  región  inguinal  derecha.  Escriba  2  Diagnósticos  
diferenciales:  
a.   HERNIA  INGUINAL   d.   Dejar  hematomas  en  la  herida  
b.   ADENOSIS  INGUINAL   e.   Aproximar  fuertemente  las  suturas  sobre  la  piel  
16.  En  la  obstrucción  intestinal  por  hernia  inguinal,  ¿Como  usted  diferencia   3.   Con  respecto  a  los  antibióticos  profilácticos  todas  son  ciertas  EXCEPTO:  
una  hernia  inguinal  extrangulada  de  una  encarcelada?   a.   Deben  iniciarse  el  día  antes  de  la  cirugía  
a.   LA   HERNIA   INGUINAL   EXTRANGULADA   PRESENTA   NECRÓSIS   Y   b.   Están  indicados  si  las  probabilidades  de  infección  son  altas  
COMPROMETE  EL  SISTEMA  VASCULAR.  
c.   Están   indicadas   si   las   probabilidades   de   la   infección  son   bajas,  
b.   LA   HERNIA   ENCARCELADA   NO   COMPROMETE   EL   SISTEMA  
VASCULAR  Y  NO  REDUCE   pero  una  infección  podría  ser  fatal  
17.  Paciente  masculino  de  35  años  desde  hace  6  semanas  observa  masa  en   d.   Se  requiere  niveles  séricos  óptimos  al  hacer  la  incisión  
región  inguinal  derecha.  Diagnóstico  diferencial:   4.   Todo  lo  siguiente  aumenta  la  tasa  de  infecciones  en   una  cirugía  mayor  
a.   HERNIA  INGUINAL   excepto:  
b.   HIDROCELE   a.   Paciente  mayor  de  70  años  
c.   ADENITIS  INGUINAL   b.   Infecciones  alejadas  del  sitio  operatorio  
18.  La   hernia   inguinoescrotal   que   no   causa   obstrucción   y   no   se   reduce:   c.   Diabetes  mellitus  controlada  
ENCARCERADA   d.   Ingesta  de  corticoides  
19.  Dentro   de   las   hernias   inguinales,   cuál   es   la   más   frecuente:   HERNIA   e.   No  hay  excepto,  todas  aumentan  la  tasa  de  infecciones  
INGUINAL  INDIRECTA   5.   La  celulitis  plantea  diagnóstico  diferencial  con:  
20.  En  la   obstrucción  intestinal  por  hernia  inguinal,  cómo  usted   diferencia   a.   Tromboflebitis  
una  her  nia  inguinal  estrangulada  de  una  encarcelada   b.   Alergia  intensa  por  contacto  
a.   EN   LA   ENCARCELADA   NO   SE   PUEDE   REDUCIR   Y   NO   HAY   c.   Inflamación  química  por  inyección  de  fármacos  
COMPROMISO  VASCULAR   d.   Fascitis  necrosante  
b.   EN  LA  ESTRANGULADA  SI  HAY  COMPROMISO  VASCULAR   e.   Todas  
6.   Acerca  de  la  miositis  clostridial  (gangrena  gaseosa),  señale  la  respuesta  
INFECCIONES  Y  COMPLICACIONES  EN  CIRUGÍA   INCORRECTA  
a.   Se  inicia  habitualmente  en  los  tres  días  que  siguen  a  la  lesión  y  
1.   La  fuente  de  bacterias  que  causan  infecciones  en  las  heridas  limpias  más   progresa  rápidamente  
comúnmente  es:   b.   Lo  más  habitual  es  que  se  deba  a  un  retraso  desvascularizante  
a.   las  manos  del  cirujano   c.   Siempre  hay  crepitación  
b.   el  salón  de  operaciones   d.   Las  manifestaciones  locales  consisten  en  dolor  en  la  herida  con  
c.   instrumental  inadecuadamente  esterilizado   exudado   seropurulento   parduzco   y   edema   de   superficie,  
d.   el  paciente   necrosis   y   cambios   de   color   que   son   generalmente   menos  
2.   Todas   las   siguientes   situaciones   aumentan   el   riesgo   de   infecciones   del   extensos  que  la  necrosis  muscular  subyacente  
sitio  quirúrgico  luego  de  una  herniorrafia  electiva  excepto   e.   La   toxemia   es   profunda   e   incluye   taquicardia   grave,   delirio   e  
a.   Rasurar  el  sitio  operatorio  el  día  anterior  de  la  cirugía   ictericia  hemolítica  
b.   Colocar  drenaje  en  la  herida   7.   La   infección   nosocomial   adquirida   más   frecuentemente   dentro   de   un  
c.   No  continua  con  los  antibióticos  profilácticos  más  de  24  horas   hospital  es:  
a.   Herida  quirúrgica   15.  La  celulitis  plantea  diagnóstico  diferencial  con:  
b.   Tracto  respiratorio   a.   TROMBOFLEBITIS  
c.   Tracto  urinario   b.   ALERGIA  INTENSA  POR  CONTACTO  
d.   Lugar  de  inyección  intravenosa   c.   INFLAMACIÓN  QUÍMICA  POR  INYECCIÓN  DE  FÁRMACOS  
8.   Señale   qué   medidas   de   las   que   se   citan     es   menos   importante   para   d.   FASCITIS  NECROTIZANTE  
disminuir  la  incidencia  de  infecciones  hospitalarias   16.  Embolia  gaseosa  se  manifiesta  por:  
a.   Baño  pre-­‐operatorio  del  paciente  con  jabón  antiséptico   a.   TAQUIPNEA  SÚBITA  
b.   Lavado  de  manos  par  parte  del  personal  sanitario   b.   HIPOTENSIÓN  
c.   Aplicación  de  luz  ultravioleta  a  la  sala  de  operaciones   c.   SOPLO  DE  RUEDA  DENTADA    
d.   Reducción   al   mínimo   imprescindible   de   la   ambulación   y   el   17.  Los  signos  clásicos  de  flogosis  que  pueden  estar  presente  en:  
hablar  en  la  sala  de  operaciones.   a.   SEROMA  
9.   La  embolia  gaseosa  se  manifiesta  por:   b.   DEHISCENCIA  DE  LA  HERIDA  
a.   Taquicardia  súbita   c.   HEMATOMA  
b.   Hipotensión   d.   INFECCIÓN  DEL  SITIO  QUIRÚRGICO    
c.   Soplo  de  ruega  dentada   18.  Paciente   a   quién   se   le   infiltró   anestesia   local,   presenta   cuadro   de  
d.   Todos   convulsión.  El  anestésico  probablemente  se  utilizó  en  dosis  inadecuadas  
e.   Ninguno   fue:    BUPIVACAINA  
10.  Todas  menos  una  puede  ser  causa  de  fiebre  pos-­‐operatoria  en  un  caso  
electivo   MANEJO  DEL  PACIENTE  QUIRURGICO  
a.   Atelectasia  
1.   Usted  es  llamado  a  la  sala  por  un  paciente  de  80  años  que  fue  sometido  
b.   Infección  en  la  herida  
a   cirugía   abdominal.   La   enfermera   le   informa   que   está   en   su   3er   día  
c.   Metabolismo  basal  aumentado  
post-­‐operatorio   y   que   luego   de   haber   cenado   presenta   dificultad  
d.   Antibióticos  
respiratoria,   a   la   auscultación   se   escucha   sibilancias   difusas   en   ambos  
11.  Todos  aumentan  el  riesgo  de  infección  de  herida  quirúrgica  menos:  
campos  pulmonares  y  a  la  oximetría  resultó  saturación  baja  de  oxígeno.  
a.   Confianza   injustificada   en   la   eficacia   terapéutica   antibiótica  
Su  diagnóstico  más  probable  es:  
profiláctica.  
a.   neumotórax  
12.  Cuando   usted   tiene   un   paciente   con   una   infección   establecido   cuyo  
b.   broncoaspiración  
agente   patógeno   es   conocido,   el   paso   más   importante   para   tratar   al  
c.   embolismo  pulmonar  
paciente  es:  DAR  ANTIBIÓTICOS  INMEDIATAMENTE  
2.   Paciente  varón  de  75  años  al  cual  se  le  practicó   cirugía  reparadora  por  
13.  Con   respecto   a   los   antibióticos   profilácticos,   enumere   dos   condiciones  
hernia   inguinal   bajo   anestesia   epidural.   La   cirugía   fue   prolongada   y  
pre-­‐operatorias  en  que  esté  indicado  para  su  uso  
dificultosa.  En  el  post-­‐operatorio  desarrolla  una  masa  en  bajo  vientre.  La  
a.   POSIBILIDAD  DE  INFECCIONES  SON  ALTAS  
conducta  correcta  es:  
b.   INICIARSE  EL  DIA  ANTES  DE  LA  CIRUGÍA  
a.   llevarlo  nuevamente  al  salón  de  operaciones  
14.  Acerca   de   la   miositis   difusa   clostridial   (gangrena   gaseosa),   señale   el  
b.   realizarle  un  ultrasonido  de  urgencia  
signo  constante  en  la  gangrena  gaseosa:  CREPITACIONES  
c.   colocarle  sonda  urinaria  y  luego  retirarla   SÍ   Puede  tratarse  sin  toracostomía  con  sonda  cuando  los  síntomas  son  
d.   realizarle  CAT  abdominal  y  de  pelvis   leves.  
3.   En  un  paciente  politraumatizado  séptico  comienza  a  drenar  sangre  roja   NO   Habitualmente   tiene   una   causa   subyacente   grave   que   debe  
definirse.  
por   el   tubo   nasográstrico   y   evacuar   melena   en   el   sexto   día  
SÍ   La   probabilidad   de   recurrencia   es   de   50%   después   del   primer  
postoperatorio.  La  causa  más  frecuente  sería:   episodio.  
a.   Ulcera  de  Stress   SÍ   Habitualmente  el  único  tratamiento  es  toracotomía  con  soda.  
b.   Ulcera  de  la  mucosa  esofágica  gástrica   SÍ   Debe   considerarse   como   indicación   quirúrgica   si   la   fuga   de   aire  
c.   Presencia  de  cáncer  gástrico  no  reconocido   persiste  más  de    una  semana.  
d.   Presencia  de  úlcera  péptica  no  reconocida    
e.   Reacción  a  los  medicamentos   2.   El  timoma:  
4.   Lo  más  útil  para  evaluar  a  un  paciente  con  problemas  de  coagulación:   a.   Es  la  neoplasia  más  común  en  el  mediastino  posterior  
a.   Tiempo  de  sangría   b.   Puede  desarrollar  miastenia  gravis  
b.   Tiempo  de  coagulación   c.   Dan  metástasis  a  distancia  con  gran  frecuencia  
c.   Cuenta  de  plaquetas   d.   El  tratamiento  es  la  quimioterapia  
d.   Tiempo  parcial  de  tromboplastina   3.   Entre  los  tumores  del  mediastino  superior,  el  más  común  es:  
e.   Leucocitosis     a.   Tumor  de  células  germinales  
5.   Es  verdadero  en  relación  a  los  requerimientos  de  potasio  en  el  paciente   b.   Bocio  endotoráxico  
operado   c.   Timoma  
a.   Durante  las  primeras  24  horas  no  se  requiere  potasio.   4.   Cuáles  son  los  3  tumores    más  comunes  del  mediastino  anterior:  
6.   La  morbilidad  y  mortalidad  peri  operatoria  depende  de  la  interacción  de   a.   TIMOMA  
los  siguientes  factores:   b.   TERATOMA  
a.    Todos.   c.   BOCIO  INTRATORÁCICO  
7.   Temperatura   de   38°C   la   noche   de   haberse   operado   de   hernia   inguinal   5.   __F__   en   los   tumores   de   mediastino   posterior   es   particularmente  
sin   mayores   complicaciones.   La   causa   más   probable   de   la   fiebre:   importante  la  determinación  de  marcadores  tumores  
ATELECTASIA  PULMONAR   6.   __F__   los   tumores   neurogénicos   típicamente   se   encuentran   el  
8.   Causas  de  fiebre  postoperatoria   mediastino  anterior  
a.   ATELECTASIA  PULMONAR   7.   __F__  El  timoma  es  el  tumor  más  frecuente  de  mediastino  posterior  
b.   INFECCIÓN  DE  LA  HERIDA  QUIRURGICA    
c.   METABOLISMO  BASAL  AUMENTADO   8.   La   escogencia   entre   el   tratamiento   quirúrgico,   la   quimio   y   la  
9.   __C__  Durante  las  primeras  horas  post-­‐operatorias  la  causa  principal  de   radioterapia  en  el  tratamiento  del  cáncer  pulmonar  se  basa  en:  
fiebre  es  la  atelectasia   a.   Disponibilidad  de  estos  tratamientos  
b.   La  edad  del  paciente  
MEDIASTINO  Y  PATOLOGÍA  PULMONAR   c.   El  debido  estadiamiento  del  tumor  canceroso  
9.   El  Mesotelioma:  
1.   Afirmaciones  ciertas  respecto  a  neumotórax  espontáneo  incluyen  que:   a.   Puede  ser  maligno  o  benigno  
b.   El  tratamiento  es  cirugía  y  generalmente  es  curativa   c.   Neumonía  neumocócica.  
c.   Generalmente   es   multicéntrico   y   su   tratamiento   es   d.   Neumonía  por  Pneumocystis.  
controversial   e.   Tuberculosis.  
d.   En  el  estadio  IV  el  tratamiento  de  elección  es  la  cirugía  
16.  Mencione  3  síndromes  paraneoplásicos  del  cáncer  broncogénico:  
10.  En  un  paciente  con  empiema  post-­‐neumónico  en  fase  fibrino-­‐purulenta:  
a.   El  tratamiento  de  elección  es  el  uso  de  fibrinolíticos   a.   SÍNDROME  DE  CUSHING  
b.   Los   antibióticos   y   un   drenaje   pleural   son   el   tratamiento   de   b.   SÍNDROME  DE  SECRECIÓN  INAPROPIADA  DE  ADH  
elección   c.   HIPERCALCEMIA  
c.   La  tomografía  computada  de  tórax  no  es  útil  en  esta  etapa  del   17.  __C__   La   broncoscopia   es   la   modalidad   invasiva   más   valiosa   en   el  
empiema     diagnóstico  de  la  enfermedad  pulmonar  
d.   La  video  toracoscopía  es  el  tratamiento  indicado   18.  __C__  el  tabaco  es  el  principal  responsable  del  cáncer  de  pulmón  
11.  Usted  evalúa  un  paciente  de  77  años  de  edad,  emaciado  y  cardiópata,  al  
19.  __F__La   escogencia   entre   le   tratamiento   quirúrgico,   la   quimio   y   la  
que   se   le   diagnostica   por   imágenes   empiema   crónico.   ¿Cuál   sería   el  
mejor  tratamiento  para  este  paciente?   radioterapia  en  el  tratamiento  del  cáncer  pulmonar  se  basa  en  la  edad  
a.   Toracotomía  y  decortización   del  paciente  
b.   Solo  antibióticos  
c.   Drenaje  abierto  y  resección  de  costilla   MORDEDURAS  
d.   Toracosentésis  y/o  toracotomía  cerrada  
12.  Un  señor  previamente  en  buen  estado  de  salud,  súbitamente  presenta   1.   Todo  lo  anterior  es  cierto  con  respecto  o  a  las  lesiones  por  mordeduras  
cianosis,  afasia  y  estridor.  Cuál  es  el  diagnóstico  más  probable   humanas  excepto:  
a.   Espasmo  bronquial   a.   En  las  manos  tienen  secuelas  graves  como  infección,  pérdida  de  
b.   Taquicardia  con  isquemia  cerebral   función  y  amputación  si  no  se  diagnostican  y  se  tratan.  
c.   Cuerpo  extraño  en  las  vías  aéreas  
b.   Sospecharla   siempre   que   hay   lesión   en   el   dorso   de   la   región  
d.   Neumotórax  espontaneo  
13.  En  el  neumotórax  espontáneo:   metacarpofalangica  del  puño  
a.   La  aparición  de  nuevos  episodios  es  poco  frecuente   c.   La  automordedura   en   la   lengua   no  se   considera   de  este   grupo  
b.   Generalmente   se   presenta   en   pacientes   altos,   delgados   y   ya  que  es  la  flora  bacteriana  propia  del  huésped  
jóvenes   d.   En   la   lesión   genuina   el   agresor   clava   sus   dientes   en   la   víctima  
c.   El   neumotórax   catamenial   se   ve   con   más   frecuencia   en   produciendo   heridas   produciendo   heridas   por   punción,  
pacientes  post-­‐menopáusicas   desgarros  o  desprendimiento  de  tejidos  
d.   No  se  produce  neumotórax  a  tensión  
e.   Las   lesiones   por   puñetazos   deben   ser   consideradas   como  
14.  Cuando  se  produce  neumotórax  a  repetición:  
a.   Se  indica  cirugía   mordeduras  
b.   Indica  EPOC   2.   Las  mordeduras  humanas  son  
c.   El  tratamiento  de  elección  es  toracosentésis   a.   Son  más  frecuentes  que  las  mordeduras  por  perros  
d.   El  tratamiento  inicial  no  fue  adecuado     b.   Se  consideran  más  infectadas  que  las  mordeduras  de  gato  
15.  La  causa  más  común  de  absceso  pulmonar  es:   c.   Más  frecuentes  en  personas  mayores  de  50  años  
a.   Aspiración.   d.   Se  consideran  heridas  contuso-­‐punzantes  
b.   Obstrucción  bronquial  por  tumor.   e.   Se  localizan  más  frecuentemente  en  las  extremidades  inferiores  
3.   Todo   lo   anterior   es   cierto   en   relación   al   riesgo   de   infección   de   las   OFIDIOS  
lesiones  por  mordedura  humana,  EXCEPTO  
a.   Tiene   alto   riesgo   de   infección   cuando   compromete   1.   Un  paciente  de  45  años  en  el  cuarto  de  urgencias,  12  horas  después  de  
articulaciones  como  la  metacarpofalangica   accidente  de  ofidio  (Verrugosa)  presenta:  
b.   Edad  mayor  de  50  años,  diabetes,  alcoholismo,  son  factores  de   a.   Petequias,  hematuria,  hemoptisis  
riesgo  asociados  a  mayor  riesgo   2.   En   la   evaluación   de  la   severidad   de   un   cuadro   clínico   de   un   accidente  
c.   Tiempo  de  evolución  mayor  de  12  horas  está  asociado  a  mayor   ofídico  de  III  grado  son:  
riesgo   a.   SE  OBSERVAN  MARCAS  DE  COLMILLOS  
d.   Un   75%   de   las   mismas   ocurren   en   actos   agresivos   y   b.   EDEMA  Y  ERITEAMA  EN  EL  AREA  DE  LA  LESIÓN  (MAYOR  DE  25  
principalmente  en  adolescentes  y  adultos  jóvenes   CM)  
e.   Son  ciertas  todas  las  anteriores   c.   DOLR  MARCADO  EN  EL  ÁREA  DE  LA  LESIÓN  
4.   El  antibiótico  profiláctico  de  elección  para  las  mordeduras  humanas,  de   d.   COMPROMISO  SISTÉMICO:  PETEQUIAS,  EQUIMOSIS,  SANGRADO  
no  existir  alergias  es  el  siguiente   GENERALIZADOS,  NAUSEAS  
a.   Clindamicina  +  Ciprofloxacina   e.   ALTERACIONES   EN   LAS   PRUEBAS   DE   LABORATORIO:   AUMENTA  
b.   Trimetropin  +  Sulfamotoxazol   TP,  TPT,  TIEMPO  DE  SANGRIA  Y  DISMINUYEN  PLAQUETAS.  
c.   Amixicilina  +  ácido  clavulánico   3.   Las   manifestaciones   clínicas   y   la   gravedad   del   envenenamiento   por  
d.   Moxifloxacina   ofidios  (Botrops)  depende  de  cuatro  factores  principales  como:  
e.   Doxiciclina   a.   CANTIDAD  DE  VENENO  INOCULADO  
5.   Todo   lo   anterior   es   cierto   en   relación   a   las   mordeduras   humanas   b.   EDAD  Y  TAMAÑO  DE  LA  VÍCTIMA  
excepto   c.   SENSIBILIDAD  DE  LA  VÍCTIMA  AL  VENENO  
a.   Las   mordeduras   en   las   manos   con   menos   de   6   horas   de   d.   TIEMPO   DE   EVOLUCIÓN   DESDE   EL   ACCIDENTE   HASTA   LA  
evolución   se   suturan   previa   irrigación   con   SSN   y   con   su   ATENCIÓN  
respectiva  terapia  antibiótica   4.   En  Panamá  tenemos  3  familias    de  ofidios.  Enumere  estas  tres  familias  y  
b.   Siempre  se  les  debe  realizar  radiografías   un  ejemplo  “Genero”  de  cada  uno  
c.   Siempre  se  debe  administrar  antibióticos  profilácticos   a.   ELAPIEDAE,  Micrurus  
d.   Las  mordeduras  en  las  manos  no  se  suturan   b.   HIDROPHIIDAE,    
e.   El   método   preferido   para   cerrar   una   herida   de   la   mano   es   el   c.   VIPERIDAE,  Botrops  asper  
cierre  primario  diferido  o  la  cicatrización  por  segunda  intención.   5.   El  ministerio  de  salud  de  Panamá  recomienda  el  siguiente  esquema  de  
6.   Tres   días   después   de   haber   sido   mordido   por   un   perro   que   tiene   sus   tratamiento  en  accidente  ofídico:  
vacunas  completas,  la  mano  del  niño  está  edematosa  y  roja,  con  severa   a.   GRADI  I:  4  –  5  VIALES  (40  –  50  ml)  
celulitis,  tiene  además,  un  drenaje  purulento  por  la  herida.  Al  momento   b.   GRADO  II:  6  –  8  VIALES  (60  –  80  ml)  
del   examen   hay   algo   de   linfangitis   y   linfadenopatías   epitrocleares   y   c.   GRADO  III:  9  –  12  VIALES  (90  –  120  ml)  
axilares   dolorosas.   En   combinación   con   el   drenaje   quirúrgico,   qué   d.   GRADO  IV:  MAS  DE  12  VIALES  (MAS  DE  120  ml)  
antibiótico   probablemente   sería   de   primera   elección:   AMOXICILINA  
CLAVULANATO  
PACIENTE   QUIRURGICO   EN   UCI,   NUTRICIÓN,   LIQUIDOS   Y   d.   Gas  bajo  el  diafragma  
6.   _C_  La  hipokalemia  en  la  obstrucción  duodenal  con  vómitos  se  debe  a:  
ELECTROLITOS   a.   Pérdida  de  saliva  
b.   Pérdida  de  jugo  gástrico  
Seleccione  su  respuesta  en  la  siguiente  forma:   c.   Pérdida  de  jugo  pancreático    
  d.   Sustitución  del  K+  por  H+  en  el  riñón    
A    si  1,  2  y  3  son  correctas    
B    si  1  y  3  son  correctas   7.   Las  causas  de  hipokalemia  clínica  incluyen  
C    si  2  y  4  son  correctas  
a.   Vómitos  
D    si  solo  4  es  correcta  
E    si  todas  son  correctas   b.   Ileo  
  c.   Uso  prolongado  de  diuréticos  a  base  de  Clorotiazida  
1.   _A_   Es   verdadero   en   relación   a   los   requerimientos   de   potasio   en   el   d.   Constipación  
paciente  post-­‐operado:     e.   A,  B  y  C  
a.   Durante  las  primeras  24  horas  no  requiere  potasio   8.   La  composición  electrolítica  del  lactato  ringer,  recuerda  la  composición  
b.   La  función  renal  debe  ser  adecuada  antes  de  administrar  K+   electrolítica  de:  
c.   De  30  a  40  mEq  son  los  requerimientos  diarios  
a.   Orina  en  un  paciente  normalmente  hidratado  
d.   Estimulación   a   la   eliminación   de   K+   es   necesario   antes   de  
administrarlo   b.   Pérdida  insensible  de  líquidos  
2.   _B_  El  calcio  juega  un  papel  importante  en:   c.   Líquido  intracelular  
a.   Excitabilidad  de  la  función  nerviosa     d.   Líquido  extracelular  
b.   En   la   contractilidad   del   músculo   tanto   esquelético   como   9.   El  desarrollo  de  hiponatremia  post-­‐operatoria  es  generalmente  causada  
cardíaco     por:  
c.   Tiene  función  en  los  orgánulos  y  membranas  celulares  
a.   Enfermedad  renal  
d.   Ayuda  en  la  liberación  de  hormonas  
3.   _E_  Se  observa  como  complicación  de  la  alimentación  parenteral:   b.   Administración  inadecuada  de  líquidos  
a.   Hiperglicemia   c.   Hipoadrenalismo  
b.   Glucosuria   d.   Diabetes  insípida  oculta  
c.   Coma  hiperosmolar   e.   Succión  gástrica  
d.   Septicemia   10.  Un  paciente  con  hipernatremia  postoperatoria,  que  ocasionalmente  se  
4.   _E_  Son  características  de  la  obstrucción  intestinal:   presenta   después   de   habérsele   administrado   algunos   agentes  
a.   Dolor  tipo  cólico  
anestésicos,  debe  ser  tratado  con  la  aplicación  de:  
b.   Vómitos  
c.   Distensión  intestinal   a.   Cortisol  
d.   No  pasaje  de  gas  o  heces  por  el  ano   b.   Vasopresina  
5.   _A_   En   el   estudio   radiográfico   en   el   abdomen   en   la   obstrucción   c.   Ácido  etacrínico  
intestinal:   d.   Dextrosa  en  agua  al  5%  
a.   Hay  gas  en  el  intestino  delgado   11.  En  relación  a  la  glutamina,  podemos  decir  que  es  un:  
b.   Niveles  líquidos   a.   Aminoácido  no  esencial  (no,  según  Fuentes)  
c.   Ausencia  de  gas  en  el  colon  
b.   Ácido  graso  no  esencial  
c.   Aminoácido   esencial   (según   Matos   que   dio   la   clase,   es   no   operación   es   muy   probable   que   el   paciente   presente   una   de   las  
esencial  que  se  vuelve  esencial  durante  el  trauma)   siguientes  anomalías  metabólicas:  
d.   Ácido  graso  esencial   a.   Hipopotasemia.  
12.  Entre   las   complicaciones  por  nutrición  parenteral   se  incluyen   todas  las   a.   Hiponatremia.  
siguientes,  EXCEPTO:   b.   Hipernatremia.  
a.   Incremento  en  el  riesgo  de  infección   c.   Hipocloremia.  
b.   Hiperglicemia   d.   Hipoglicemia.  
c.   Infiltración  grasa  del  hígado   17.  El   paciente   tiene   mayor   probabilidad   de   presentar   ésta   anomalía  
d.   Producción  excesiva  de  CO2   metabólica  debido:  
13.  Un   método   práctico   recomendado   para   el   balance   hídrico   de   un   a.   Suministro  inadecuado  de  dextrosa.  
paciente  en  el  post-­‐operatorio  es:   b.   Administración  de  líquidos  hipotónicos.  
a.   estimar  el  agua  del  cuerpo   c.   Secreción  inapropiada  de  hormona  antidiurética.  
b.   presión  arterial   d.   Pérdida  de  colon.  
c.   frecuencia  de  pulso   e.   Secreción  de  aldosterone  
d.   pesarlo  diariamente   18.  Luego  de   inyectar   2000  mL   de   líquidos  IV,   después   de   traumatismo,  la  
14.  La  guía  más  útil  para  determinar  el  reemplazo  adecuado  de  líquidos  es   presión  arterial  del  paciente  es  110/70  mmHg.  ¿Cuál  de  los  siguientes  
a.   La  diuresis   datos   sería   la   mejor   guía   clínica   para   una   reanimación   adecuada   con  
b.   El  retorno  de  la  conciencia   líquidos?  
c.   El  llenado  capilar  de  las  uñas   a.   Excreción  urinaria  >  de  30  mL/h.  
d.   Gases  arteriales  normales   b.   Reducción  de  la  frecuencia  de  pulso  <  lpm.  
e.   El  desarrollo  de  leve  edema  pulmonar   c.   Retorno  de  la  temperatura  normal  de  la  piel.  
15.  En  un  hombre  de  70  Kg.  El  volumen  de  agua  intracelular  es  de:   d.   Retorno  de  la  precepción  sensorial  normal.  
a.   3.5  L   e.   Desaparición  de  la  hipotensión  ortostática.  
b.   5  L   19.  En   la   terapéutica   del   choque   hemorrágico,   el   mejor   signo   clínico   para  
c.   10.5  L   juzgar  si  la  reposición  de  líquido  tiene  éxito  es:  
d.   28  L   a.   Incremento  de  presión  arterial  
e.   42  L   b.   Incremento  en  la  excreción  de  orina  
16.  Un  hombre  de  65  años  de  edad,  sufre  una  resección  abdominoperineal   c.   Incremento  de  oxigenación  arterial  
por  carcinoma  rectosigmoideo  sin  complicaciones  durante  la  operación.   d.   Disminución  de  la  sed  
La  sangre  perdida  se  repuso  volumen  a  volumen  y  la  excreción  de  orina   e.   Reducción  de  la  taquicardia  
se  mantuvo  en  20  a  30  mL/h.  durante  el  post-­‐operatorio  se  administró   20.  Cuál   de   las   siguientes   medidas   se   utilizan   en   el   tratamiento   de   los  
al   paciente   líquido   en   cantidad   apropiada   para   restituir   jugo   gástrico   y   pacientes  con  hiperpotasemia  aguda  
conservarlo  bien  hidratado,  también  se  le  suministró  dextrosa  al  5%  en   a.   Administración  de  glucosa  e  insulina  
SSN  al  0.45%  a  una  tasa  de  150  mL/h.  veinticuatro  horas  después  de  la   b.   Administración  de  Lactato  Ringer  
c.   Administración  de  resinas  de  intercambio  iónico  por   vía  oral  o   c.   Falla  renal  
rectal   d.   Vómitos  
d.   Infusión  de  soluciones  hipotónicas  de  Sodio   e.   Fístula  del  intestino  delgado  
e.   A  y  C   27.  La  hipomagnesemia  se  observa  en  todo  lo  siguiente,  MENOS:  
21.  la  alcalosis  metabólica  está  asociada  con:   a.   Alcoholismo  crónico  
a.   Hiperventilación   b.   Hipoparatiroidismo  
22.  Consecuencia  de  administrar  exceso  de  proteínas   c.   Pancreatitis  
a.   Hiperamonemia   d.   Quemaduras  
b.   Elevación  de  creatinina  y  BUN.   e.   Cetosis  diabética  
c.   Oliguria   28.  No  está  indicada  la  nutrición  parenteral  total  en:  
d.   Falla  pulmonar   a.   Paciente  en  coma  y  ECV  
e.   Esteatosis  hepática   b.   Paciente  con  fístula  enterocutanea  o  pérdida  de  60  cc  
23.  Entre   las   complicaciones   por   alimentación   calórica   excesiva   en   el   c.   Pacientes  con  síndrome  de  intestino  corto  
paciente  nutrición  parenteral  se  incluyen  todas  las  siguientes,  excepto   d.   Paciente  con  cáncer  gástrico  obstructivo  
a.   Incrementa  el  riesgo  de  infección   e.   Paciente  con  colitis  ulcerativa  
b.   Hiperglicemia   29.  Ejemplo  de  oligoelementos:  
c.   Infiltración  grasa  del  hígado   a.   Cobre  
d.   Producción  excesiva  de  CO2   30.  Ventaja  de  nutrición  enteral  sobre  parenteral:  
24.  Consecuencia  de  administrar  exceso  de  carbohidratos   a.   mantiene  la  función  inmunológica.  
a.   Hiperglicemia   31.  Indicación  de  nutrición  enteral  
b.   Hipercapnia   a.   Intestino  viable.  
c.   Hígado  graso   32.  Contraindicación  de  terapia  nutricional  
d.   Hipertrigliceridemia   a.   Paciente  politraumatizado  con  FC:  120x’,  PA:  82/40.  
e.   Todas  las  anteriores   33.  Paciente  masculino  de  70  kg  recibe  dextrosa  al  5%  a  100  cc  para  darle  
25.  Las   medidas   para   tratar   la   Hiperkalemia   incluye   todo   lo   siguiente,   algo  de  calorías.  En  realidad  cuantas  calorías  recibe?:  
MENOS:   a.   408  calorías.  
a.   Evitar  la  administración  de  potasio  exógeno   34.  Paciente   de   60   kg,   con   gastrectomía   hace   6   días,   tiene   peristaltismo  
b.   Administrar  glucosa  e  insulina   intestinal,  decide  dar  nutrición  enteral  con  formula  enteral:  100  cc,  20  g  
c.   Kayexalate   de  proteínas,  10  g  de  lípidos,  30  g  de   carbohidratos   está  a   100  cc  por  
d.   Administración  de  Mg   hora,   recibe   nutrición   5:1.   Que   volumen   de   la   formula     recibe   en   24  
e.   Diálisis     horas?  
26.  Las   causas   de   Acidosis   metabólicas   incluyen   todas   las   siguientes,   a.   2000  cc  a  40.  
MENOS:   35.  Cuantos  gramos  de  N2  representa  esas  proteínas  ,  y  cuantos  gramos  de  
a.   Diabetes   carbohidratos?.  
b.   Inanición   a.   N2  a  6.4  CHO  60.  
36.  Llega  el  resultado  de  nitrógeno  egresado  en  2  4horas,  10  gramos,  como   51.  __C__   proteína   ligadora   de   retinol   con   valor   de   2.1   mg/dL   es  
esta  ese  balance?   desnutrición  severa  
a.   Negativo.   52.  __F__  la  desnutrición  hospitalaria  en  promedio  corresponde  a  30%  
37.  Cuál   es   el   método   que   usted   considera   más   práctico   para   el   balance  
hídrico  de  un  paciente  pos-­‐operado:  PESAR  AL  PACIENTE  DIARIAMENTE   PANCREAS  
38.  Enumere   tres   complicaciones   por   alimentación   calórica   excesiva   en   el  
1.   En   la   pancreatitis   aguda,   cuál   de   los   siguientes   factores   identificado  
paciente  con  soporte  nutricional  
desde   el   momento   de   la   admisión   o   en   las   primeras   48   horas   de  
a.   HIPERGLICEMIA   hospitalización  indica  ataque  grave  y  diagnóstico  sombrío:  
b.   INCREMENTO  EN  EL  RIESGO  DE  INFECCIÓN   SÍ   Déficit  de  base  de  6  mEq/L.  
c.   PRODUCCIÓN  EXCESIVA  DE  CO2   SÍ   Leucocitosis  de  20,000/mm3.  
39.  Mencione  2  funciones  de  líquidos  en  nuestro  organismo:   SÍ   Concentración  de  glucosa  en  plasma  de  250mg/100  mL.  
a.   TRANSPORTE   DE   NUTRIENTES   Y   DESECHOS   DESDE   Y   HACIA   LA   SÍ   Concentración  sérica  de  calcio  de  7  mg/100  mL.  
CÉLULA   SÍ   PO2  arterial  de  55  mmHg  
 
b.   MEDIO  DE  REACCIONES  BIOQUÍMICAS  
Seleccione  su  respuesta  en  la  siguiente  forma:  
40.  Cite   2   funciones   terapéuticas   en   la   prescripción   de   soluciones    
parenterales  para  el  tratamiento  de  alteraciones  hidroelectrolíticas   A    si  1,  2  y  3  son  correctas  
a.   ¿???   B    si  1  y  3  son  correctas  
b.   ¿???   C    si  2  y  4  son  correctas  
41.  Cite  una  función  principal  del  líquido  en  nuestro  organismo   D    si  solo  4  es  correcta  
E    si  todas  son  correctas  
a.   ¿???  
 
42.  __F__  Una  de  las  manifestaciones  clínicas  de  intoxicación  hídrica  son  las   2.   _A_  En  el  diagnóstico  de  la  pancreatitis,  es  importante:  
convulsiones  
a.   Historia  de  alcoholismo  
43.  __C__   La   pérdida   de   peso   pre-­‐operatorio   indica   alto   riesgo   post-­‐ b.   Dolor  en  epigastrio  difuso  en  cinturón  
operatorio   c.   Elevación  de  las  amilasas  
44.  __C__  Pacientes  desnutridos  presentan  más  infecciones  nosocomiales  
3.   _A_  En  la  pancreatitis  aguda  se  encuentra  elevada:  
45.  __C__  La  desnutrición  se  puede  definir  como  pérdida  de  peso  mayor  de   a.   Amilasa  en  suero  
10%  en  6  meses   b.   Amilasa  urinaria  
46.  __C__  La  desnutrición  se  puede  definir  como  pérdida  de  peso  mayor  de  
c.   Lipasa  en  suero  
5%  en  1  mes    
47.  __C__   perder   entre   el   10   al   20%   del   peso   se   considera   desnutrición   4.   Cuál  de  los  siguientes  indica  un  pobre  pronóstico  en  pancreatitis?  
moderada  
a.   Los  niveles  de  amilasa  en  el  suero.  
48.  __C__  valor  normal  de  albúmina  sérica  es  de  3.5-­‐5  mg/dL   b.   Hiperglicemia  y  glucosuria.  
49.  __F__  La  pre  albumina  es  un  precursor  de  la  albumina   c.   Niveles  elevados  de  amilasa  en  orina.  
50.  __C__  la  vida  media  de  la  albumina  es  de  20  días   d.   Tiempo  de  coagulación  elevados.  
e.   Disminución  de  la  calcemia.   e.   SGOT  
5.   En  Panamá,  la  primera  causa  de  pancreatitis   10.  Todo   lo   anterior   es   cierto   acerca   de   la   pancreatitis   necrotizante,  
a.   Parásitos   excepto:  
b.   Biliar   a.   Es  una  enfermedad  grave  caracterizada  por  necrosis  glandular  y  
c.   Alcoholismo   una  respuesta  inflamatoria  sistémica  destructiva  
6.   Ante  un  caso  de  pancreatitis  aguda  (PA),  ¿en  qué  situación  consideraría   b.   El  tratamiento  precoz  es  quirúrgico  en  primera  instancia  y  es  de  
indicada  la  CEPRE?:   menor   importancia   las   medidas   enérgicas   de   reanimación   y  
a.   Pancreatitis  aguda  severa  en  fase  aguda   soporte  
b.   Pancreatitis  aguda  recurrente  de  origen  no  esclarecido   c.   Los   resultados   están   determinados   de   manera   primaria   por   la  
c.   Pancreatitis  aguda  persistente  durante  más  de  una  semana   presencia   de   infección   bacteriana   secundaria   tardía   de   la  
d.   Todas   glándula  necrótica  
7.   Un   varón   de   40   años   de   edad   con   antecedentes   de   abuso   de   d.   Los   antibióticos   empíricos   tempranos   y   la   necrosectomía  
alcoholismo  se  presenta  después  de  una  borrachera,  se  queja  de  dolor   quirúrgica   tardía   en   el   contexto   apropiado   son   claves   para   el  
en  el  epigastrio  con  irradiación  a  la  espalda  acompañado  de  náuseas  y   tratamiento  de  estos  pacientes  graves  
vómitos.  A  la  exploración  física  tiene  marcado  dolor  en  el  epigastrio  con   e.   Con   terapia   apropiada   se   puede   minimizar   la   mortalidad   y   es  
defensa,  disminución  de  los  ruidos   aéreos   y   distención   abdominal.   Las   posible  restaurar  la  calidad  de  vida  a  largo  plazo  
placas   del   abdomen   demuestran   dilatado   intestinal   del   hemiabdomen   11.  Permite  establecer  el  pronóstico  en  la  pancreatitis  aguda,  excepto  
superior.  El  diagnóstico  más  probable  es:   a.   Criterios  de  Ranson  
a.   Úlcera  péptica  perforada   b.   Sistema  ASA  
b.   Colecistitis  aguda   c.   Criterios  de  Glasgow  
c.   Pancreatitis  aguda   d.   APACHE  II  
d.   Divertículo  del  sigmoides  perforado   e.   Criterios  de  Baltasar  
8.   El   fármaco   que   se   ha   mostrado   eficaz   en   pacientes   con   pancreatitis   12.  __C__   El   pseudoquiste   de   páncreas   no   debe   ser   drenado   cuando   es  
aguda:   diagnosticado  
a.   Somatostatina   13.  Un   varón   de   40   años   de   edad   con   antecedentes   de   abuso   de  
b.   Anticolinérgicos   alcoholismo  se  presenta  después  de  una  borrachera,  se  queja  de  dolor  
c.   Glucagon   en  el  epigastrio  con  irradiación  a  la  espalda  acompañado  de  náuseas  y  
d.   Morfina   vómitos.  A  la  exploración  física  tiene  marcado  dolor  en  el  epigastrio  con  
e.   Ninguno   defensa,  disminución  de  los  ruidos  aéreos  y  distención  abdominal.  Las  
9.   De  los  signos  pronósticos  de  Ranson  para  valoración  de  la  gravedad  de   placas   del   abdomen   demuestran   dilatado   intestinal   del   hemiabdomen  
pancreattis  aguda,  cuál  se  documenta  en  el  momento  del  ingreso?   superior.  El  diagnóstico  más  probable  es:  PANCREATITIS  AGUDA    
a.   BUN   14.  En  panamá,  cuál  es  la  primera  causa  de  pancreatitis:  BILIAR  
b.   Calcemia   15.  ¿Qué   es   lo   que   indica   un   pobre   pronóstico   en   una   Pancreatitis?  
c.   PaO2   DISMINUCIÓN  DE  LA  CALCEMIA  A  MENOS  DE  8.  
d.   Déficit  de  Base  
16.  La  presencia  de  ictericia  y  vesícula  biliar  palpable  diferencia  al  cáncer  de   NO   En  mujeres  de  30  años  de  edad  debe  practicarse  un  estudio  basal.  
la:  CABEZA  Y  COLA  DE  PANCREAS   NO   En   ese   estudio   se   emplea   menor   radiación   que   en   una   radiografía  
17.  Pancreatitis  aguda:   de  tórax.  
a.   IDIOPÁTICA   NO   La   mamografía   es   un   sustituto   eficiente   de   la   biopsia   en   lesiones  
b.   BILIOSA   sospechosas.  
c.   ALCOHOLICA   NO   Técnicas  termográficas  mejoradas  representan  un  sustituto  seguro  y  
18.  En  el  diagnóstico  de  la  Pancreatitis  en  importante:   eficiente  para  la  detección  mamográfica.  
a.   HISTORIA  DE  ALCOHOLISMO.   SÍ   El   estudio   debe   ser   parte   de   la   vigilancia   regular   de   la   paciente  
b.   DOLOR  EPIGÁSTRICO  DIFUSO  EN  CINTURÓN.   después  de  terapéutica  por  cáncer  mamario  unilateral.  
c.   ELEVACIÓN  DE  LA  AMILASA.    
19.  Un   paciente   de   50   años   se   presenta   al   hospital   con   fiebre   alta,  
3.   Afirmaciones  correctas  respecto  al  cáncer  de  mama,  incluyen:  
escalofríos,   tos   y   dolor   en   el   hipocondrio   derecho   e   ictericia,   acolia   y  
coluria.  Cuáles  serían  sus  diagnósticos  diferenciales   SÍ   La  incidencia  de  cáncer  mamario  en  mujeres  ha  crecido  lentamente.  
a.   COLEDOCOLITIASIS   SÍ   En  la  actualidad,  el  cáncer  de  mama  es  la  principal  causa  de  muerte  
b.   COLANGITIS   relacionada  con  cáncer  en  las  mujeres  panameñas.  
NO   Cifras   de   sobrevivencia   para   etapas   comparables   de   cáncer  
c.   CÁNCER  DE  CABEZA  DE  PANCREAS  
mamario   son   más   altas     en   nuestras   mujeres   mulatas   si   se  
comparan  con  nuestras  mujeres  blancas.  
PATOLOGÍA  DE  LA  MAMA   NO   Personas   con   dietas   ricas   en   grasas   insaturadas   muestran   riesgos  
significativos  mayores  de  padecer  carcinoma  mamario.  
1.   Relacionado  con  el  fibroadenoma,  son  afirmaciones:  
SÍ   Las   monjas   presentan   incidencia   mayor   de   contraer   cáncer  
SÍ   Puede   observarse   lesión   única   de   2   centímetros,   que   clínicamente   mamario.  
corresponde   a   fibroadenoma   en   las   mamas   de   una   mujer   de   20    
años  de  edad.   4.   Afirmaciones  verdaderas  respecto  a  la  mastitis  aguda,  incluyen:  
NO   Cuando   se   observa   una   lesión   que   clínicamente   corresponde   a  
fibroadenoma  en  las  mamas  de   una  mujer  de  23  años  de  edad,  se   SÍ   Ocurre  con  mayor  frecuenta  en  mujeres  en  lactación.  
debe  practicar  mamografía.   SÍ   Es  más  común  en  clima  caluroso.  
NO   Es  extremadamente  dolorosa.  
SÍ   A   veces   no   es   posible   distinguir   entre   un   fibroadenoma   juvenil  
SÍ   El  agente  causal  más  común  es  Staphylococcus  aureus.  
voluminoso  de  un  tumor  filodes.   SÍ   El   tratamiento   inicial   debe   ser   con   Penicilina   o  
NO   La  superficie  de  corte  del  fibroadenoma  típico  se  retrae  a  lo  largo  de   Cefalosporina.  
la  línea  de  corte.   5.   Usted   examina   a   una   de   45   años,   quien   se   queja   de   una   masa   no  
SÍ   Un   fibroadenoma   quizás   sea   doloroso   e   hiperestésico   durante   el   dolorosa   en   la   mama   izquierda.   Cuál   de   estas   cosas   le   gustaría  
embarazo.   preguntar   en   la   historia   clínica   de   esta   paciente   en   orden   de  
  importancia  
2.   Cuáles   de   las   siguientes   afirmaciones   acerca   de   la   mamografía   son   A.   Previas  masas  en  las  mamas  
correctas:   B.   Descarga  por  el  pezón  
C.   Cambio  de  peso   tricomoniasis   y   procede   a   tratarla   con   Flagyl;   además   le   solicita   una  
D.   Trauma  en  mamas   mamografía   de   rutina.   ¿Considera   usted   que   está   indicada   la  
E.   Preñeces   mamografía  de  rutina?  
F.   Historia  familiar   a.   Sí.  
G.   Anticonceptivos   b.   No.  
H.   Historia  menstrual   9.   ¿Cuáles   serían   los   resultados   más   probables   en   esta   mamografía,   en  
I.   Menaquia   relación  con  las  fases  del  ciclo  mestrual?  
J.   Operaciones  previas   a.   Fase  luteínica:  
__F__   i.   Mayor  número  de  mamografía  falso  negativo.  
__G__   ii.   Menor  número  de  mamografía  falso  negativo.  
__B__   b.   Fase  folicular:  
__C__   i.   Menor  número  de  mamografía  falso  negativo.  
6.   Al   examen   físico   se   encuentra   una   masa   de   2   cm   en   el   cuadrante   ii.   Mayor  número  de  mamografía  falso  negativo.  
superoexterno   de   la   mama  izquierda.   El   movible   y  la   piel   sobre   ella   es   10.  El  riesgo  de  cáncer  de  mama,  en  la  mama  restante  es  mayor  en:  
normal.  No  se  palpan  nódulos  en  la  axila  ni  en  el  cuello.  La  otra  mama  es   a.   Comedocarcinoma.  
normal.  No  se  palpa  el  hígado.  Cuál  de  los  siguientes  usted  le  ordenaría   b.   Carcinoma  inflamatorio.  
en  orden  de  importancia?   c.   Carcinoma  lobular.  
A.   Ordena  una  mamografía   d.   Enfermedad  de  Paget.  
B.   Le  pide  regresar  pasado  el  periodo  menstrual   e.   Carcinoma  ductal.  
C.   Obtiene  una  biopsia  por  aguja   11.  El  tipo  más  frecuente  de  cáncer  de  mama  es:  
D.   Inicia  tratamiento  con  Bromocriptina  (Parloled)   a.   Carcinoma  papilar  infiltrante.  
E.   Ordena  ultrasonografía  de  la  mama  izquierda   b.   Carcinoma  ductal  infiltrante.  
F.   Le  dice  a  la  paciente  que  es  una  mama  benigna   c.   Carcinoma  medular.  
__C__   d.   Carcinoma  coloide.  
7.   En  este  momento,  cuáles  de  las  siguientes  considerar  pedir   e.   Carcinoma  lobular.  
A.   Centelleo  óseo   12.  El  área  donde  se  observa  más  frecuentemente  el  cáncer  mamario  es:  
B.   Hemograma  completo   a.   Cuadrante  superior  interno  
C.   Electrolitos   b.   Cuadrante  superior  externo  
D.   Radiografía  de  Tórax   c.   Cuadrante  inferior  interno  
E.   Centelleo  hepático   d.   Cuadrante  inferior  externo  
F.   Calcemia   13.  Sangramiento  por  el  pezón  
__D__   a.   Está  asociado  a  mailignidad  en  10  al  30%  de  los  casos  
__A__   b.   Puede  ser  debido  a  un  papiloma  intraductal  o  carcinoma  ductal  
8.   Paciente  femenina  de  44     años   de   edad   que   hace   uso   de   estrógenos,   c.   Puede  ser  debido  a  múltiples  lesiones  
consulta  a  su  ginecólogo  por  secreción  vaginal.  Su  médico  le  diagnostica   d.   Se  trata  con  hormonas  
e.   Se  trata  con  mastectomía   21.  Enumere  cuatro  factores  en  el  desarrollo  del  cáncer  de  mama:  
14.  La  mastitis  aguda  ocurre  con  frecuencia  en   a.   HIPERPLASIA  
a.   La  pubertad   b.   BRCA1  y  BRCA2  
b.   La  preñez   c.   ENFERMEDAD  FIBROQUISTICA  
c.   En  la  menopausia   d.   FIBROADENOMA  
d.   Lactancia   22.  Factores  de  riesgo  para  el  desarrollo  de  Cáncer  de  Mama:  
e.   Al  nacer   a.   EDAD  
15.  La   mamografía   de   una   señora   de   53   años   es   sospechosa   de   lesión   b.   FACTORES  HEREDITARIOS  
maligna  cuando  muestra:   c.   ENFERMEDAD  MAMARIA  BENIGNA  
a.   Aumento  de  la  densidad   d.   FACTORES  ENDOCRINOS  ENDOGENOS  
b.   Calcificaciones  de  los  vasos   e.   ANTICONCEPTIVOS  ORALES  
c.   Linfadenopatía  axilar   f.   NULIPARIDAD  
d.   Calcificaciones  finas   g.   TRATAMIENTO   SUSTITUTIVO   POSTMENOPAUSICO   CON  
e.   Calcificaciones  redondas   ESTRÓGENOS  
16.  Cuál   de   los   siguientes   hallazgos   es   más   indicativo   de   cáncer   en   una   23.  Actualmente  se  ha   descubierto  que   el   cromosoma   13  está  asociado   al  
mamografía  tomada  en  una  señora  de  70  años:   cáncer  de  mama.  Mencione  los  dos  genes  responsables  
a.   Calcificaciones  finas   a.   BRCA-­‐1  
17.  Enumere  el  %  de  cáncer  en  la  mama  que  ocurre  entre  los  45  y  59  años,   b.   BRCA-­‐2  
antes  de  los  20  años  y  antes  de  los  30  años.   24.  __F__  Mujeres  de  38  años  debe  practicarse  estudio  basal.  
a.   67  –  75%   25.  __F__   los   factores   de   riesgo   están   presentes   en   menos   de   la   mitad   d  
b.   0.08%   elos  casos  de  los  pacientes  que  son  diagnosticados  por  primera  vez  de  
c.   1.8%   cáncer  de  mama  
18.  Métodos   adecuados   para   el   Dx   del   cáncer   de   la   mama   con   el   fin   de   26.  __F__   Hay   mayor   riesgo   de   contraer   cáncer   mamario   con   múltiples  
detectar  el  trastorno  en  su  fase  pre-­‐clínica   embarazos  que  una  nulípara.  
a.   MAMOGRAFIA   27.  __C__  En  este  estudio  (mamografía)  se  emplea  menor  radiación  que  en  
b.   AUTO   EXAMEN   FÍSICO   DE   LA   MAMA   Y   PROMOVER   EL   AUTO   una  radiografía  de  torax.  
EXAMEN  DE  MAMA.   28.  __F__   La   mamografía     es   un     sustituto   de   biopsia   en   lesiones  
19.  Cuál   es   el   tipo   histológico   de   mayor   riesgo   de   cáncer   de   la   mama   sospechosas  
restante:  CANCER  DE  MAMA  LOBULAR   29.  __F__   Técnicas   tecnograficas   mejoradas   representan   un   sustituto  
20.  El  diagnóstico  de  Cáncer  de  Mama  está  basado  en:   seguro  suficiente  para  diagnóstico.  
a.   AUTOEXAMEN  DE  LA  MAMA   30.  __C__  El   estudio   este   parte   de   la   vigilancia   regular   de   un   paciente   de  
b.   HISTORIA  CLINICA  Y  EXAMEN  FISICO   falla  terapéutica  de  cáncer  mamario  unilateral  
c.   MAMOGRAFIA   31.  __C__   La   incidencia   de   cáncer   de   mama   en   mujeres   desciende  
d.   ECOGRAFIA   lentamente  
e.   BIOPSIA  
32.  __C__   En   la   actualidad   el   cáncer   de   mama   es   la   principal   causas   de   a.   Se  hace  traqueostomía  ante  la  primera  sospecha.  
muerte  relacionado  con  cancer  en  mujeres  panameñas.   b.   La  saturación  de  oxígeno  es  la  mejor  guía  para  el  tratamiento.  
33.  __F__  Cifras  de   supervivencia   para  etapas   comparables  para  el   cáncer   c.   Puede  haber  vibrisas  quemadas.  
mamario  son  más  altas  en  mujeres  mulatas  que  en  mujeres  blancas   d.   Es  frecuente  en  los  diabéticos.  
34.  __C__  Las  monjas  presentan  incidencia  de  mayor  cáncer  mamario   5.   Los  efectos  locales  de  una  quemadura  incluyen  los  siguientes,  MENOS:  
35.  Las  mamografías  falsas  negativas  son  más  frecuentes  en:     a.   Aumento  de  la  evaporación  
b.   Pérdida  de  calor  por  la  evaporación    
a.   __F__Mamas  eutróficas   c.   Invasión  al  área  por  microorganismos  
b.   __F__Fase  folicular  
d.   Trombosis  vascular  y  necrosis  tisular  
c.   __C__Fase  luteinica    
d.   __C__En   mujeres   que   estan   en   terapia   de   remplazo   e.   Septicemia  por  Gram  negativos  
hormonal     6.   De   acuerdo   con   la   regla   de   los   “9”   el   área   de   la   cabeza   y   cuello  
e.   __C__En  mujeres  que  toman  estrógenos    de  forma  cíclica     corresponde  a:  
  a.   9%  
QUEMADURAS   b.   18%  
  c.   27%  
1.   La   característica   más   importante   de   reconocer   una   quemadura   de   2º   d.   1%  
grado  es:   e.   36%  
a.   Venas  trombosadas.   7.   Porcentaje   de   superficie   corporal   que   incluye   miembros   superiores,  
b.   Edema.   tórax  y  hemiabdomen  superior:  
c.   Prurito.   a.   27%  
d.   Vesícula  o  ampolla.   8.   La  mejor  forma  de  hacer  el  diagnóstico  de  ruptura  vesical  después  de  un  
2.   El   periodo   en   que   se   escapan   las   proteínas   del   espacio   intravascular   trauma  pélvico  es:  
durante  una  quemadura  es  de:   a.   Citoscopía  
a.   Primeras  12  horas.   b.   Pielograma  IV  
b.   24  a  48  horas.   c.   Cistograma  
c.   Hasta  las  96  horas  desde  la  quemadura.   d.   Radiografía  apical  lord  ótica  
d.   No  ocurre  escape  de  proteínas.   e.   Sangre  en  la  orina  
3.   La  fórmula  de  Baxter  dice  que  el  reemplazo  de  líquido  durante  el  primer   9.   Qué  porcentaje  de  superficie  corporal  representa  una  quemadura  de  los  
día  es  de:   miembros  superiores  y  tórax  anterior:  27%  
a.   cc  por  cc  con  solución  coloide.   10.  El   periodo   en   que   se   escapan   las   proteínas   del   espacio   intravascular  
b.   L/R  3  cc/Kg  x  porcentaje  de  quemadura.   durante  la  quemadura  es  de:  24  A  48  HORAS  
c.   D/A  %%  1000  cc  +  KCl  mEq.  
d.   Equivalente  al  volumen  de  la  diuresis  con  Lactato  Ringer.  
TIROIDES  
4.   En  el  daño  por  inhalación:    
1.   En   una   mujer   de   20   años   de   edad   con   una   masa   asintomática   de   6   NO   El   tratamiento   definitivo   de   la   causa   subyacente   de   un  
centímetros  en  el  polo  inferior  del  lóbulo  tiroideo  derecho,  la  maniobra   neumotórax  a  tensión    requiere  toracostomía  abierta.  
inicial  más  apropiada  es:   2.   Es   llamado   al   cuarto   de   urgencias   para   atender   a   un   obrero   de   la  
a.   Biopsia  excisional.   construcción  de  28  años,  que  es  admitido  después  de  caer  de  9  metros  
b.   Biopsia  incisional.   de  altura  en  su  trabajo.  El  paramédico  de  la  ambulancia  informó  que  el  
c.   Aspiración  con  aguja  para  citología.   paciente  quedó  inconsciente  después  del  su  accidente,  pero  recobró  al  
d.   Estudio  de  captación  de  yodo  radiactivo.   consciencia  en  el  transcurso  al  hospital,  volviéndose  cada  vez  más  alerta  
e.   Examen  de  tiroides  con  ultrasonido   y  quejándose  de  dolor  en  el  muslo  derecho.  Al  llegar,  su  presión  arterial  
2.   El  cáncer  de  tiroides  más  común  es:   era   de   148/86,   pulso   de   100,   y   la   respiración   era   de   18   por   minuto.  
a.   Carcinoma  folicular.   Tenía   una   férula   colocada   en   su   extremidad   derecha   la   cual   se  
b.   Carcinoma  papilar.   observaba  edematosa.  Al  examen  físico,  mostraba  una  abrasión  de  4  cm  
c.   Carcinoma  anaplásico.   en   la  región  temporal,  leve  dolor  al  palpar  el  abdomen  y  dificultad  en  
d.   Carcinoma  medular.   mover   la   pierna   derecha,   la   cual   le   dolía   al   tocarla   en   el   muslo.   Sus  
e.   Cáncer  secundario.   reflejos  plantares  y  pupilares  están  bien.  
3.   Un   don   de   55   años   presenta   nódulo   tiroideo   de   4cms,   solitario   e   Cuál  de  lo  siguiente  haría  usted  en  los  primeros  5  a  10  minutos  en  orden  
indoloro.   T4   es   normal,   centelleo   no   demuestra   captación.   La   de  importancia:  
ultrasonografía  indica  que  es  sólido.  De  las  siguientes  estaría  indicado:   A.   Transfundir  una  unidad  de  sangre  
a.   Biopsia  con  aguja  fina  y  posible  cirugía   B.   Colocar  un  tubo  endotraqueal  
b.   Tratamiento  de  tiroides  y  solución  de  lugol   C.   Infundir  Lactato  Ringer  a  100  cc  por  hora  
c.   Yodo  radiactivo   D.   Infundir  dextrosa  al  5%  en  agua  
d.   Propanolol   E.   Infundir  SSN  
e.   Irradiación  externa     F.   Transfundir  una  unidad  de  células  rojas  empacados  
G.   Colocar  un  catéter  Foley  
TRAUMA   H.   No  dar  líquidos  IV  por  posible  trauma  cerebral  
  I.   Administrar  O2  al  100%  
1.   Cuáles   son   las   siguientes   afirmaciones   caracterizan   correctamente   el   __I__  
neumotórax  a  tensión:   __C__  
3.   Después   de   lo   anterior,   cuáles   de   estas   medidas   considera   usted,   en  
NO   Un   neumotórax   espontáneo   sin   tratamiento   habitualmente   orden  de  importancia:  
evoluciona  hacia    neumotórax  a  tensión.  
A.   Punción  lumbar  
NO   Los  datos  físicos  incluyen  desviación  de  la  tráquea  hacia  el  lado  
B.   Hemograma  completo  
afectado.  
NO   Cuando  un  paciente  presenta  neumotórax  a  tensión  mientras  se   C.   Electrolitos  del  suero  
le  administra  ventilación  con  presión  positiva  debe  suspenderse   D.   Gases  arteriales  
de  inmediato  la  presión  positiva.   E.   Urinalisis  
F.   EKG  
G.   Tipaje  y  cruce  de  una  unidad  de  sangre   6.   Una   mujer   de   18   años   de   edad   ingresa   a   una   sala   de   urgencias   en  
H.   Lavado  peritoneal   choque   hemorrágico   luego   de   un   accidente   automovilístico.   Luego   de  
__G__   administrar  2  L  de  L/R,  presión  arterial  y  pulso  permanecen  inestables  y  
__B__   la   paciente   se   queja   de   dolor   en   abdomen   alto.   En   el   examen   se  
__D__   observan   hiperestesia   y   notable   contractura   involuntaria   en   el   CSI   del  
4.   Cuales   procedimientos   radiográficos   de   los   siguientes   solicitaría,   en   abdomen.   Radiografía   torácica,   hematocrito   y   análisis   de   orina   son  
orden  de  importancia:   normales.   Cuál   de   los   siguientes   procedimientos   se   considera  
a.   Arteriografía  cerebral   indispensable  para  el  tratamiento  de  esta  paciente?  
b.   Radiografía  lateral  de  la  columna  cervical   a.   Tomografía  computarizada  de  abdomen.  
c.   US  abdominal   b.   Lavado  peritoneal.  
d.   Radiografía  de  fémur  derecho   c.   Sonografía  abdominal.  
e.   Aortograma  abdominal   d.   Arteriografía  celíaca  selectiva.  
f.   Pielograma  IV   e.   Laparotomía  exploratoria.  
g.   Radiografía  de  Tórax   7.   Un  evento  metabólico  en  trauma,  excepto:  
h.   Centelleo  esplénico   a.   Fuente  de  glucosa  para  glucogenolisis  
i.   CAT  de  la  cabeza   8.   Todo  lo  siguiente  es  cierto  acerca  de  la  respuesta  metabólica  al  trauma  
__B__   excepto:  
__G__   a.   La  alanina  es  el  principal  aminoácido  utilizado  en  el  ciclo  de  Cori  
__I__   para  obtención  de  energía  
5.   Un   hombre   de   18   años   de   edad   ingresa   al   departamento   de   urgencias   b.   La  principal  fuente  de  aminoácidos  es  proveniente  del  músculo  
después   de   sufrir   un   accidente   automovilístico.   Se   encuentra   en   un   c.   El  glicerol  puede  ser  transformado  en  glucosa  
estado  de  coma  (7  en  la  escala  de  Glasgow).  El  pulso,  con  frecuencia  de   d.   Las   respuestas   metabólicas   en   el   trauma   es   similar   a   los  
140  latidos  por  minuto  es  casi  impalpable  y  la  presión  arterial  es  60/0.   pacientes  en  ayuno  
La   respiración   es   rápida   y   superficial,   ventilando   ambos   campos   9.   En   un  paciente  politraumatizado,  con  enfisema  subcutáneo   de   toda   la  
pulmonares.  El  abdomen  se  encuentra  moderadamente  distendido,  sin   pared  torácica  y  con  signos  de  falla  respiratoria,  lo  correcto  es:  
peristalsis  audible.  Se  detectan  fracturas  cerradas  en  antebrazo  derecho   a.   Hacer  una  radiografía  de  tórax  
y  parte  inferior  de  pierna  izquierda.  Luego  de  administración  IV  de   2   L   b.   Realizar  una  broncoscopía  
de   L/R   en   extremidades   superiores,   el   pulso   es   de   130   y   la   presión   c.   Colocar  un  tubo  pleural  en  cada  hemitórax    
arterial  70/0.  El  siguiente  paso  inmediato  sería:   10.  Un   joven   de   30   años   fue   traído   al   cuarto   de   urgencias   después   de   un  
a.   Practicar   radiografía   lateral   de   columna   cervical   a   través   de   la   accidente   de   tránsito.   Los   signos   vitales   eran:   PA   110/90,   Pulso  
mesa.   110/minuto.  La  radiografía  de  tórax  reveló  leve  neumotórax  (menor  de  
b.   Practicar  TC  de  cabeza  y  abdomen.   1%).   Tenía   signos   de   abdomen   agudo.   Cuál   es   el   próximo   paso   en   el  
c.   Practicar  radiografía  de  abdomen  en  decúbito  supino  y  lateral.   manejo  de  este  paciente:  
d.   Practicar  aortografía  de  cayado.   a.   Administrar  oxígeno  con  máscara  
e.   Explorar  el  abdomen.   b.   Llevarlo  al  salón  por  su  abdomen  agudo  
c.   Colocarle  un  tubo  de  toracostomía   a.   Tratar  el  Shock  hipovolémico  
d.   Hacerle  una  entubación  endotraqueal   b.   -­‐-­‐-­‐-­‐-­‐  neurogénico  
e.   Hacerle  una  toracotomía   c.   Manejo  quirurgico  del  collar  cervical  ¿=  
11.  En   un   individuo   previamente   sano,   la   pérdida   de   15%   a   30%   del   d.   Intubación  nasotraqueal  
volumen  sanguíneo:   16.  Un  joven  herido  en  accidente  moto  ciclístico  llega  al  cuarto  de  urgencia  
a.   Se  distingue  por  hipotensión  profunda   con  presión  arterial  70/40,  pulso  120,  con  tiraje,  y  cianosis  debido  a  un  
b.   Usualmente   produce   todos   los   signos   y   síntomas   clásicos   del   neumotórax  a  tensión  izquierda,  el  abdomen  doloroso,  y  defendido,  con  
Shock   fractura   expuesta   del   fémur   derecho.   Usted   es   la   única   persona   en   el  
c.   Requiere  reemplazo  por  anuria   cuarto  de  recibo.  Indique  el  orden  de  prioridad  que  usted  instalaría:  
d.   Se   manifiesta   por   taquicardia,   taquipnea   y   reducción   de   la   1.   Reducir  la  fractura  y  férula  de  yeso  luego  de  limpieza  
presión  del  pulso  sin  otros  síntomas  de  shock   2.   Canalizar  una  vena  con  aguja  14  o  16  y  dar  L/R  
12.  La  primera  medida  que  se  debe  tomar  en  un  poli  traumatizado  que  llega   3.   Tubo  intra  pleural  izquierdo  
inconsciente  y  PA  90/60  es:   4.   Laparotomía  exploradora  después  de  dar  sangre  
a.   Canalizar  una  vena  con  una  aguja  gruesa  14  0  16   5.   Hipertet,  toxoide  antitetánico,  según  su  criterio  
b.   Llamar  urgente  a  neurocirugía   a.   3,2,4,1,5  
c.   Control  de  la  hemorragia  con  torniquetes   b.   2,5,3,4,1  
d.   Hipertet  IM  y  refuerzo  con  toxoide     c.   4,3,2,1,5  
e.   Asegurar  la  permeabilidad  de  las  vías  aéreas         d.   2,4,3,1,5  
13.  Signo  para  hacer  el  diagnóstico  diferencial  de  taponamiento  cardiaco  y   e.   3,2,1,4,5  
neumotórax  a  tensión   17.  El  tratamiento  inmediato  de  un  paciente  con  múltiples  fracturas  y  shock  
a.   Ruidos  cardiacos   debe  ser:  
b.   Presión  arterial  baja   a.   Transfusión  de  sangre  
c.   Ruidos  respiratorios   b.   Salina  normal  
d.   Taquicardia   c.   Plasma  
14.  Manejo  en  un  centro  de  salud   d.   Lactato  Ringer  
a.   Llamar  al  personal  para  que  le  tome  radiografía   e.   Albúmina    
b.   Pericardiocentesis   18.  Una   joven   de   20   años   es   traída   al   hospital   después   de   accidente   de  
c.   Enviar  a  cuarto  de  urgencias  de  un  hospital   tránsito.   Está   consciente   y   sangra   profusamente   por   una   laceración  
d.   Referir   grande  del  cuero  cabelludo  y  de  herida  abierta  del  muslo  derecho.  Está  
e.   Nada   aprehensiva   y   sedienta.   El   pulso   es   130,   la   presión   es   de   70/40.   Dice  
15.  Manejo   de   femenina   que   sufre   accidente   automovilístico   y   presenta   pesar  110  libras.  Se  calcula  la  pérdida  de  sangre  en:  
disminución   de   la   presión   arterial,   aumento   de   la   frecuencia   cardiaca,   a.   500  ml  
parálisis  de  las  4  extremidades,  está  consciente,  buena  ventilación,  vías   b.   1000  ml  
aéreas  permeables,  ella  está  pálida  y  fría,  y  refiere  tener  sed  y  no  poder   c.   1500  ml  
respirar:   d.   2000  ml  
e.   2500  ml   d.   Es  más  importante  en  el  paciente  inconsciente  que  el  paciente  
19.  En  el  adulto  traumatizado,  los  volúmenes  ventilatorios  deben:     consciente  que  mueve  activamente  las  extremidades  
a.   Ser  por  lo  menos  25  ml/kg  de  peso  estimado   e.   Debe   enfocar   sobre   el   proceso   odontoides   y   los   cuerpos  
b.   Ser  800  a  1200  ml  por  respiración   vertebrales  
c.   Ser  menos  que  el  volumen  corriente  para  evitar  neumotórax   24.  Un   joven   de   20   años   de   edad   recibe   tratamiento   para   lesiones  
d.   Nunca  exceder  10  ml/kg  de  peso  estimado   incurridas  en  un  accidente.  No  hay  sangre  en  el  banco,  solo  se  dispone  
e.   Ser   considerado   como   de   importancia   secundaria   en   de  líquidos  intravenosos  no  sanguíneos.  Su  hemoglobina  es  de  4.5  gms.  
comparación  a  las  presiones  ventilatorias     La  cianosis  es:    
20.  En   la   primera   hora   después   de   trauma   cerrado   lesiones   del   a.   Inevitable  
retroneumoperitoneo:     b.   Probable  
a.   Se  caracteriza  por  sepsis  anaeróbica   c.   Imposible  
b.   Están  asociadas  a  hemorragias  exanguinantes     d.   Evitable  
c.   Son  difíciles  de  diagnosticar   e.   Deseable    
d.   Se  descartan  por  lavado  peritoneal   25.  El   aspecto   más   importante   del   tratamiento   inicial   de   la   acidosis  
e.   Se  sospechan  por  los  signos  y  síntomas  que  presentan   metabólica  en  los  traumas  severos  es:  
21.  La  razón  por  el  monitoreo  de  la  excreción  urinaria  en  la  víctima  de  Shock   a.   El  balance  químico  del  pH  arterial  
es  porque  refleja  con  acierto:   b.   Establecer  una  perfusión  tisular  adecuada  
a.   La  sobrecarga  de  líquido   c.   Reemplazo  del  volumen  perdido  con  electrolitos  
b.   El  transporte  de  agua  libre   d.   Hiperventilación  
c.   El  sodio  en  el  suero   26.  Cuando   hay   muchas   víctimas   de   trauma   en   el   cuarto   de   urgencias,   la  
d.   La  perfusión  renal   prioridad  más  alta  de  manejo  se  le  deba  asignar  al  que  sufre  de:  
e.   Los  niveles  de  catecolamina     a.   Fractura  expuesta  de  ambos  fémures  
22.  Antes   de   intentar   una   de   las   maniobras   de   ventilación   en   el   paciente   b.   Hemorragia  severa  con  heridas  abiertas  
con   trauma   severo   de   la   cabeza   y   cuello,   debe   ser   explorado   por   c.   Trauma  craneoencefálico  severo  
fractura  cervical.  El  mejor  método  diagnóstico  sería:   d.   Obstrucción  de  las  vías  aéreas  
a.   Exploración  digital  del  cuello   e.   Tórax  batiente  
b.   Historia  de  dolor  en  la  columna  cervical   27.  Las   hemorragias   que   producen   pérdidas   del   15%   del   volumen  
c.   Radiografía  posteroanterior  del  cuello   sanguíneo:  
d.   Radiografía  lateral  del  cuello   a.   Pueden  producir  shock  hasta  en  un  tercio  de  individuos  sanos  
e.   Signos  neurológicos   b.   Es  precedido  por  oliguria  
23.  Una   vista   lateral   de   las   vértebras   cervicales   con   el   paciente   acostado   c.   Causa  desmayos,  otras  cosas  poco  
sobre  la  mesa:   d.   Se  pueden  tratar  sin  transfusión  de  sangre  
a.   Descarta  lesiones  serias  de  la  columna  cervical   e.   Usualmente  causan  hipotensión  
b.   Debe  proceder  la  entubación  endotraqueal  
c.   Es  inaceptable  a  menos  que  se  visualicen  las  7  cervicales  
28.  En   el   paciente   con   trauma   cerrado   de   tórax,   y   que   presenta   imagen   31.  Un   mecánico   de   45   años   de   edad   estaba   trabajando   en   su   taller   y  
hiperluscente   en   el   hemitórax   izquierdo,   además   de   un   neumotórax   solicitó  a  su  ayudante  que  arrancara  el  camión  para  evaluar  el  motor.  El  
debe  descartarse   ayudante   poco   experto   en   manejo   pierde   el   control   y   aprisiona   a   su  
a.   Hemotorax   compañero  contra  la  pared.  Este  es  llevado  al  servicio  de  urgencias  de  
b.   Contusión  pulmonar   su   hospital.   El   paciente   presenta   un   estado   evidente   de   shock   con  
c.   Hernia  diafragmática   excoriaciones,  deformidad  y  aumento  de  volumen  en  ambos  muslos.  El  
29.  Un  hombre  de  29  años  de  edad  llega  al  servicio  de  urgencias  luego  de   estado  de  shock  que  presenta  el  paciente:  
haber   sufrido   un   accidente   automovilístico-­‐colisión.   Se   encuentra   a.   No  puede  ser  explicado  a  menos  que  exista  una  fractura  pélvica  
cianótico,   tiene   insuficiencia   respiratoria,   y   una   escala   de   coma   de   asociada  
Glasgow   de   6.   No   hay   trauma   facial   significativo;   su   tráquea   se   b.   No   puede   ser   revertido   si   se   aplican   férulas   de   tracción   en  
encuentra   en   la   línea   media;   tiene   una   desviación   crónica   del   tabique   ambos  miembros  inferiores  
nasal   que   impide   la   intubación   nasotraqueal.   Tiene   abundante   barba   c.   Es   explicado   porque   significa   que   el   paciente   ha   tenido   una  
que  dificulta  la  fijación  de  la  máscara  de  oxígeno.  El  paso  siguiente  más   pérdida   de   aproximadamente   15%   de   su   volumen   sanguíneo  
apropiado  es:   circulante.  
a.   Realizar  una  cricotiroidotomía  quirúrgica.   d.   Solo  se  explica  si  hay  lesión  de  una  arteria  importante  
b.   Forzar  un  tubo  nasotraqueal  a  través  del  tabique  nasal  desviado   e.   Es   explicado   por   pérdida   sanguínea   asociada   a   fracturas  
c.   Intentar  una  intubación  orotraqueal  usando  dos  personas  para   femorales  bilaterales.  
mantener  la  columna  alineada.   32.  Son  objetivos  de  la  intubación  endotraqueal:  
d.   Ventilar  al  paciente  con  mascarilla  con  válvula  y  reservorio  hasta   a.   Establecer  vía  aérea  
que  se  le  pueda  afeitar  la  barba  para  lograr  colocar  una  máscara   b.   Administrar  oxígeno  
de  oxígeno.   c.   Apoyar  la  ventilación  
30.  Usted   está   de   turno   en   el   servicio   de   urgencias   cuando   le   llega   un   d.   Prevenir  la  broncoaspiración  
paciente   acompañado   de   TUM’s   quienes   le   informan   que   el   paciente   e.   Todas  las  anteriores  
recibió  golpes  con  un  garrote  en  la  cabeza  y  en  la  cara.  El  paciente  usted   33.  Un   hombre   de   40   años   sufre   un   traumatismo   cerrado   de   tórax   al  
lo  encuentra  comatoso  y  a  la  palpación  presenta  una  fractura  deprimida   estrellarse   contra   el   timón   del   vehículo   en   un   accidente   en   el   que  
de   cráneo.   La   cara   está   edematizada   y   equimótica,   su   respiración   es   impactó   a   alta   velocidad   un   objeto   fijo.   En   el   servicio   de   urgencias  
gorgoreante  y  se  encuentran  restos  de  vomito  en  la  cara  y  en  la  ropa.  El   observa   que   el   paciente   presenta   distensión   de   las   venas   del   cuello,  
paso   más   apropiado   a   seguir,   luego   de   la   administración   de   oxígeno   desvío  de  la  tráquea  hacia  la  izquierda,  una  perfusión  periférica  pobre  y  
suplementario  y  de  elevarle  la  mandíbula  es:   ausencia  de  ruidos  respiratorios  en  el  hemotórax  derecho.  La  causa  más  
a.   Succionar  la  orofaringe.   probable  de  estos  hallazgos  es:  
b.   Insertar  un  tubo  gástrico   a.   Lesión  aórtica  
c.   Solicitar  una  tomografía  computarizada   b.   Una  contusión  cardiaca  
d.   Obtener  una  radiografía  lateral  de  la  columna  cervical   c.   Neumotórax  a  tensión  
e.   Ventilar   al   paciente   utilizando   una   mascarilla   con   válvula   y   d.   Síndrome  de  vena  cava  superior  
reservorio-­‐AMBU  
34.  Un  motociclista  de  20  años  sufre  una  lesión  muy  importante  en  la  cara   b.   el  peso  y  la  masa  de  cada  individuo  
al  estrellar  su  motocicleta  de  frente  contra  una   camioneta.  El  paciente   c.   la  velocidad  y  la  aceleración  que  iban  viajando  
es   llevado   por   los   TUM’s   al   servicio   de   urgencias   completamente   d.   el  peso  y  la  masa  del  vehículo  
inmovilizado  en  una  tabla  espinal  larga  y  con  un  collar  cervical.  Usted  es   38.  Un   hombre   de   40   años   sufre   un   traumatismo   cerrado   en   el   tórax   al  
el   médico   que   recibe   este   paciente.   Al   examen   tiene   PA   de   150/60   compactar  el  timón  en  un  accidente  a  alta  velocidad  con  un  objeto  fijo,  
mmHg,  la  FC=  90/min.  Rítmico,  la  FR=  26/min.  El  paciente  presenta  una   el  paciente  tiene:  
respiración  laboriosa  y  es  ruidosa.  Su  escala  de  coma  de  Glasgow  es  de   a.   neumotórax  a  tensión  
7.   Usted   efectúa   varios   intentos   de   intubación   orotraqueal     39.  Al  colocar  un   catéter   subclavio,   el   paciente   presenta   disnea,   cianosis   y  
manteniendo   la   columna   cervical   alineada,   sin   éxito   debido   a   la   dolor  torácico.  Probablemente  ha  ocurrido:  
presencia   de   sangrado   y   una   anatomía   distorsionada.   Súbitamente   a.   Sepsis  
presenta  apnea.  El  mejor  procedimiento  para  el  manejo  temporal  de  la   b.   Punción  de  la  arteria  
vía  aérea  en  esta  situación  es:   c.   Trombosis  de  la  vena  
a.   Intubación  nasotraqueal   d.   Hidro  o  neumotórax  
b.   Traqueostomía  de  urgencia   e.   Punción  del  miocardio  
c.   Colocación  de  una  cánula  orofaríngea   40.  Cuál  de  los  casos  siguientes  tendría  prioridad  estando  solo  en  el  cuarto  
d.   Colocación  de  una  cánula  nasofaríngea   de  urgencias  
e.   Cricotiroidotomía  con  aguja  e  insuflación  a  presión   a.   Trauma  cerrado  de  tórax,  disneico  con  tiraje  
35.  Durante   la   revisión   primaria   ¿Cuál   de   los   siguientes   hallazgos   en   un   b.   Trauma  cerrado  de  abdomen,  vomita  sangre  
adulto  indica  un  manejo  inmediato?   c.   Herida  de  bala  sobre  la  tetilla  izquierda,  PA  100/70,  Pulso  de  
a.   Abdomen  distendido   100  
b.   Escala  de  coma  de  Glasgow  de  11   d.   Fractura  expuesta  de  fémur,  sangrado  de  la  herida    
c.   Temperatura  de  36.5ºC   41.  Cuál   de   los   siguientes   indica   tamponamiento   cardiaco   en   una   herida  
d.   Frecuencia  cardiaca  de  120  latidos  por  minuto   punzante  en  el  pericardio  (hemopericardio)  
e.   Frecuencia  respiratoria  de  40  respiraciones  por  minuto   a.   Ruidos  cardiacos  apagados  o  distantes  
36.  Los  exámenes  radiológicos  más  importantes  que  debemos  obtener  en  el   b.   Presión  venosa  de  2  cm  de  agua  
paciente  traumatizado  severo  son:   c.   Hipotensión  sistólica  
a.   Cráneo,  tórax  y  abdomen   d.   Todas  
b.   Tórax,  abdomen  y  pelvis   e.   Solamente  A  y  C  
c.   Cráneo,  columna  cervical  y  tórax   42.  Un  joven  de  18  años  recibe  una  bala  en  el  abdomen  durante  un  asalto.  
d.   Columna  cervical,  tórax  y  pelvis   La  herida  de  la  bala  está  por  debajo  del  ombligo  y  otra  herida  de  10  cms  
e.   Columna  cervical,  tórax  y  abdomen   por   debajo   del   reborde   costal   en   el   lado   derecho   de   la   línea   axilar  
37.  Un  adulto  y  un  menor  viajan  en  un    vehiculo  pequeño  a  una  velocidad   media.  El  pulso  es  de  90  por  minuto  y  la  PA  de  110/60.  El  paciente  debe:  
de   80   millas/hora.   De   ocurrir   un   accidente,   el   factor   predominante   a.   Ser   sometido   a   exploración   de   las   heridas   bajo   anestesia  
causante  de  las  lesiones  corporales  seria:   local  
a.   la  composición  del  objeto  fijo   b.   Ser  sometido  a  un  lavado  peritoneal  diagnóstico  
c.   Ser  observado  cuidadosamente  por  hemorragia  o  peritonitis   b.   Vitamina  K  
d.   Ser  sometido  a  laparotomía  lo  antes  posible   c.   Plasma  fresco  congelado  
e.   Inyectar  las  heridas  con  medio  de  contraste  en  el  sitio  de  la   d.   Crioprecipitados  
lesión   48.  Un   señor   de   65   años,   gran   fumador,   con   enfermedad   obstructiva  
43.  Hasta   que  se   pruebe   lo   contrario   usted   debe   considerar   a   un   paciente   crónica,   se   cae   y   se   fractura   cuatro   costillas   del   lado   derecho.   Se  
de  trauma  que  este  belicoso,  combativo  y  no  cooperador  como:   descarta   otra   lesión   del   lado   derecho   y   otra   lesión   concomitante.   El  
a.   diabético   tratamiento  apropiado  podría  ser  todos  los  siguientes  menos:  
b.   intoxicado   a.   Colocar  cinta  adhesiva  alrededor  del  tórax  para  inmovilización  
c.   impertinente   b.   Bloqueo  de  nervios  intercostales  
d.   hipóxico   c.   Paracentesis  abdominal  
44.  Después  de  un  traumatismo  quirúrgico  mayor,  a  los  2  o  4  días  podemos   d.   Radiografía  de  tórax  
tener  cada  una  de  las  siguientes  situaciones,  EXCEPTO:   e.   Consulta  a  neumólogo  
a.   Aumento  de  la  excreción  de  los  esteroides  adrenocorticales   49.  Cuál   de   los   siguientes   probablemente   NO   será   detectado   durante   la  
b.   Oliguria  relativa   evaluación  inicial  de  un  accidentado  de  transito:  
c.   Disminución  de  la  excreción  de  nitrógeno   a.   Obstrucción  de  la  laringe  
d.   Retención  de  sodio  y  potasio   b.   Desgarro  total  de  la  aorta  torácica  
45.  La  contusión  pulmonar:   c.   Neumotórax  abierto  
a.   Es  la  lesión  potencialmente  mortal  más  común  en  la  evaluación   d.   Neumotórax  a  tensión  
inicial   e.   Tórax  batiente  importante  con  insuficiencia  respiratoria  
b.   Siempre  requiere  de  manejo  con  ventilación  mecánica   50.  Todos  lo  siguientes  sugieren  lesión  traumática  de  la  uretra,  EXCPETO:  
c.   Una   vez   realizada   la   reanimación,   los   líquidos   deben   ser   a.   Sangre  en  el  meato  urinario  
administrados  juiciosamente   b.   Hematoma  del  escroto  
d.   El  tratamiento  incluye  la  fijación  de  la  caja  torácica   c.   Ausencia  de  la  próstata  palpable    al  tacto  rectal  
46.  El   mejor   tipo   de   estudio   de   rayos   X   para   localizar   aire   libre   d.   Próstata  elevada  al  tacto  rectal  
intraabdominal  dudosa  es  una  radiografía:   e.   Priapismo  
a.   Una  radiografía  AP  de  tórax   51.  Cuando   se   lesiona   un   vaso   la   respuesta   inmediata   para   producir  
b.   Una  radiografía  de  de  pie  y  acostado  del  abdomen   hemostasia  es:  
c.   Una  radiografía  en  decúbito  lateral  con  rayos  horizontales,  con   a.   Adherencia  plaquetaria  en  el  lugar  de  la  lesión  
el  lado  derecho  hacia  arriba   b.   Vasoconstricción        
d.   Tomografía  computarizada  de  abdomen   c.   Liberación  de  un  trombo  
47.  Un  joven  sano  recibió  10  unidades  de  sangre  luego  de  sufrir  un  trauma   d.   Ayuda  de  los  fibroblastos  
en  una  colisión  de  un  vehículo  a  motor.  Su  temperatura  corporal  es  de   52.  Enumere  cuatro  signos  OBJETIVOS  de  compromiso  de  la  vía  aérea:  
37°C,   plaquetas   75.000.   Presenta   sangrado   difuso.   El   tratamiento   más   a.   AGITADO  
efectivo  deberá  ser:   b.   OBNUBILADO  
a.   Infusión  de  plaquetas   c.   CIANOSIS  
d.   TIRAJE   a.   ASEGURAR  VÍA  AÉREA    
e.   ESTRIDOR  LARINGEO   b.   VENTILAR  Y  OXIGENAR  
53.  El  paciente  politraumatizado  crítico  muere  por:   62.  Signos  objetivos  de  paciente  en  Shock:  
a.   HIPOTERMIA   a.   FRIO  
b.   ACIDOSIS  METABOLICA   b.   TAQUICARDIA  
c.   COAGULOPATÍA   c.   ¿?==?  
54.  Un   joven   herido   en   accidente   automovilístico   llega   al   servicio   de   63.  Vía  aérea  definitiva:  
urgencia   con   PA   70/40,   pulso   120,   sin   signos   externos   de   hemorragia   a.   VÍA  AÉREA  INSEGURA  
severa,  disnea  severa,  asimetría  de  tórax,  tráquea  desviada  a  la  derecha,   b.   APNEA  
el   abdomen   doloroso   y   defendido,   con   fractura   expuesta   del   fémur.   c.   LESIÓN  CRANEOENCEFÁLICA  CERRADA  
Usted   es   la   única   persona   en   el   servicio.   Indique   en   qué   orden   de   d.   COMPROMISO  INMINENTE  DE  LA  VÍA  AÉREA  
prioridad  usted  atendería  a  este  paciente   e.   OXIGENACIÓN  POBRE  
a.   TUBO  PLEURAL  DERECHO   f.   RIESGO  DE  ASPIRACIÓN  
b.   CANALIZACION  DE  2  VENAS  PARA  LACTATO  RINGER  A  CHORRO   64.  __F__   La   primera   medida   que   se   toma   en   un   paro   cardiaco   es   la  
c.   LAPAROTOMIA     inyección  de  adrenalina  
d.   LIMPIEZA  Y  REDUCCIÓN  DE  FRACTURA  EXPUESTA   65.  __F__  El  diagnóstico  de  neumotórax  a  tensión  es  radiológico  
e.   TOXOIDE  TETÁNICO     66.  __F__   La   primera   acción   a   tomar   con   paciente   que   se   presenta   con  
55.  Cuando   se   lesiona   un   vaso   la   respuesta   inmediata   para   producir   neumotórax  abierto  es  la  sutura  
hemostasia  es:  VASOCONSTRICCIÓN   67.  __F__  En   el  paciente  de  trauma  al  que  se  va  a  intubar  la  maniobra  de  
56.  Cuando   el   Paciente   es   sometido   a   ventilación   mecánica   con   presión   alinear  la  columna  cervical  con  extensión  de  la  cabeza  es  la  indicada  
positiva,   la   causa   más   frecuente   de   neumotórax   a   tensión   es:   EL   68.  __F__  En  el  paciente  de  trauma,  la  hiperextensión  de  la  cabeza  facilita  la  
BAROTRAUMA   maniobra  de  intubación  orotraqueal.  
57.  Historia   de   golpe   en   1/3   inferior   del   hemitórax   izquierdo   asociado   a  
69.  __F__   En   el   paciente   de   trauma,   la   hiperextensión   del   cuello   de   la  
dolor  en  el  hombro  izquierdo  más  defensa  abdominal  indica:  RUPTURA  
DE  BAZO   cabeza  facilita  la  maniobra  de  intubación  endotraqueal  
58.  El   tratamiento   inicial   del   neumotórax   abierto   es:   CUBRIR   LA   HERIDA   70.  __F__   El   shock   cardiogénico   es   el   más   común   por   lesiones     que   se  
CON  APÓSITOS  ESTÉRILES  Y  FIJARLO  EN  3  LADOS.   producen  al  miocardio  en  las  heridas  penetrantes.  
59.  La  tríada  de  Beck  está  constituida  por:   71.  __F__  Todo  paciente  hipotenso  y  frio  está  en  shock  hasta  que  se  pruebe  
a.   HIPOTENSIÓN   lo  contrario  
b.   DISMINUCIÓN  DE  LOS  RUIDOS  CARDÍACOS  A  LA  AUSCULTACIÓN   72.  __C__  En  la  hemorragia  Clase  I  presenta  signos    clínicos  mínimo  porque  
c.   INGURGITACIÓN  YUGULAR  
el  organismo  generalmente  lo  compensa.  
60.  El   desequilibrio   ácido   base   se   debe   a   perfusión   y   reanimación  
73.  __F__   La   sola   permeabilidad   de   la   vía   aérea   asegura   una   adecuada  
inadecuada.  El  tratamiento  sería:  
ventilación  
a.   OXIGENAR  Y  VENTILAR  
74.  __C__  El  nivel  de  conciencia  alterado  en  trauma  es  la  causa  más  común  
b.   ADMINISTRAR  LÍQUIDOS  IV  Y  SANGRE  
de  compromiso  de  la  vía  aérea.  
61.  En  el  manejo  de  vía  aérea  la  primera  prioridad  es:  
75.  __F__   El   objetivo   de   la   evaluación   inicial   es   detectar   lesiones   que   SÍ   La  abstinencia  de  fumar  ayuda  a  impedir  la  gangrena.  
pueden  ser  potencialmente  letales.   SÍ   Una  operación  profiláctica  ayuda  a  impedir  la  gangrena.  
76.  __C__  Un  evento  metabólico  que  ocurre  en  el  paciente  con  trauma  es  la    
excreción  urinaria  de  nitrógeno.   2.   En   la   siguiente   lista   son   síntomas   y   signos   de   la   isquemia   arterial  
77.  __C__   En   el   shock   hipovolémico,   la   resistencia   periférica   está   obstructiva  aguda  
aumentada   Si    Parálisis  
78.  __C__  una  vista  lateral  de  las  vértebras  cervicales  es  recomendable  en   No    Edema  
todo  caso  de  lesión  severa  por  arriba  de  las  claviculas   Si    Palidez  
79.  __C__   En   los   jóvenes,   el   desplazamiento   del   mediastino   en   casos   de   Si    Ausencia  de  pulso  
neumotórax  masivo  es  mayor  que  en  los  de  mayor  edad.   Si    Dolor  
80.  __C__  Una  indicación  para  el  lavado  peritoneal  es  el  estado  de  coma  en    
trauma  cerrado  en  shock  hipovolémico   3.   La  tromboflebitis  de  venas  profundas  de  las  extremidades:  
81.  __C__   La   extirpación   del   bazo   en   jóvenes   expone   al   peligro   de   a.   Se  maneja  con  anticoagulantes  a  largo  plazo  
infecciones  altamente  peligrosas   4.   __C__  La  tromboflebitis  superficial  se  maneja  sin  cirugía  
5.   Manifestación  clínica  de  la  isquemia  crónica:  DOLOR  EN  REPOSO    
TUMOR  DE  CABEZA  Y  CUELLO   6.   Manifestación   clínica   de   la   isquemia   arterial   aguda   en   los   miembros  
  inferiores  
1.   Entre   los   tumores   intraorales   tenemos   las   lesiones   premalignas,   a.   DOLOR  
mencione  3:   b.   AUSENCIA  DE  PULSO  
a.   HIPERQUERATOSIS  
c.   PARALISIS,  PARESTESIA  Y  PALIDEZ  
b.   HIPERPLASIA  PSEUDOEPITELIOMATOSA  
c.   LEUCOPLASIA   7.   Manifestación  clínica  de  la  isquemia  arteria  crónica  
2.   Dentro  de  la  etiología  de  los  tumores  malignos  intraorales  mencione  3:   a.   CLAUDICACIÓN  INTERMITENTE  
a.   IRRITACIÓN  CRÓNICA   b.   DOLOR  EN  REPOSO  
b.   TABAQUISMO   c.   GANGRENA  
c.   ALCOHOL   8.   En  el  síndrome  post-­‐flebítico,  lo  más  importante  es:  PREVENCIÓN    
3.   Entre   los   tumores   malignos   intraorales   cuál   es   el   tipo   histológico   que   9.   Las   complicaciones   más   importantes   de   la   trombosis   venosa   profunda  
predomina:  ESCAMOSO    
son:  
  a.   EMBOLISMO  PULMONAR  
VASCULAR  PERIFÉRICA   b.   SINDROME  POSTFLEBITICO  
 
1.   Cuáles   de   las   siguientes   afirmaciones   relacionadas   con   pacientes   que    
padecen  claudicación  intermitente  son  ciertas:  

SÍ   2/3  de  ellos  pueden  tratarse  sin  intervención  quirúrgica.  


NO   1/3  presentará  gangrena.  
SÍ   En  ausencia  de  cambios  tróficos,  rara  vez  aparece  gangrena.  
GRAN  COMPENDIO  DE  CIRUGÍA  
Manejo  Inicial  del  Paciente  con  Trauma  
 
1.   Paciente  de  24  años   de  edad  quien  al  dejar  una   fiesta  colisiona  contra   otro  vehículo  frontalmente  a  más  o  
menos   120   km/h.   Estaba   ingiriendo   licor.   Durante   el   transporte   se   mantuvo   hipotenso   80   mmHg   a   la  
palpación.   Sus   signos   vitales   al   llegar   al   cuarto   de   urgencias   Pa:110/88,   frecuencia   cardiaca   en   120   X,  
frecuencia  respiratoria  32  X  y  temperatura  35,6;  se  palpa  frío,  diaforético  y  responde  frases  incomprensibles  
al  estimularlo.  Presenta  deformidad  de  la  extremidad  inferior  con  exposición  de  hueso  y  sangrado  activo.  La  
primera  prioridad  de  este  paciente  es:  
a.   Radiografía  lateral  de  columna  cervical  
b.   Intubación  orotraqueal  
c.   Controlar  la  hemorragia  de  la  extremidad  inferior  
d.   Monograma  abdominal  de  urgencias  
 
2.   Después  de  la  resucitación  la  frecuencia  cardiaca  se  encuentra  en  100  X,  su  piel  está  caliente  y  su  presión  es  
ahora  de  118/90  mmHg.  Su  GCS  es  de  8.  En  la  placa  de  tórax  se  observa  un  ensanchamiento  del  mediastino.  El  
USG  FAST  no  se  observa  líquido  en  cavidad  abdominal.  Todo  lo  siguiente  se  debe  realizar  en  este  paciente,  
excepto:  
a.   Consulta  a  neurocirugía  
b.   Celiotomía  
c.   Tomografía  de  tórax  
d.   Consulta  a  ortopedia  
 
3.    Paciente  de  30  años   quien  es  atropellado   por   un  autobús.  Ella  fue  intubada  en  la  escena  y  se  le  indicaron  
líquidos  intavenosos.  Al  arribar  al  cuarto  de  urgencias  su  GSC  es  de  8,  pulso  120  X  y  su  presión  arterial  es  de  
100/70.  Los  paramédicos  le  administraron  oxígeno  por  AMBU  con  reservorio  a  más  o  menos  20  respiraciones  
por   minuto.   Su   oxigenación   es   de   90%.   A   la   auscultación   no   se   escuchan   los   ruidos   respiratorios   del   lado  
izquierdo  y  no  se  observan  las  venas  del  cuello  distendidas  o  desviación  de  la  tráquea.  El  tratamiento  debe  
consistir  primero  en:  
a.   Inserción  de  una  aguja  número  14  F  en  el  segundo  EII  
b.   Colocación  de  un  tubo  32  F  en  el  quinto  EII  
c.   Observación   de   las   marcas   del   tubo   endotraqueal,   succionar   el   tubo   y   lentamente   extraerlo   mientras   se  
ausculta  
d.   Colocarlo  en  decúbito  lateral  derecho  para  mejorar  la  ventilación  
 
4.      Un   varón   de   45   años   es   traído   al   servicio   de   urgencias   después   de   sufrir   un   accidente   automovilístico.   El  
paciente   está   alerta   y   orientado,   con   examen   neurológico   normal.   Su   fx   respiratoria   es   de   20   x,   pulmones  
limpios  bien  ventilados,  fx  cardiaca  en  120  x  y  presión  arterial  de  6..mmHg.  A  la  exploración  física  se  nota  con  
distensión,   establece   una   línea   intravenosa   y   el   paciente   recibe   rapidamente   2   L   de   L/R,   sin   cambios   en   la  
frecuencia  del  pulso  ni  en  la  Pa.  El  siguiente  paso  y  el  más  apropiado  en  su  Tx  es:  
a.   Radiografía  de  columna  cervical  
b.   Inserción  de  catéter  central  
c.   Laparotomía  exploradora  
d.   CAT  de  abdomen  
e.   Lavado  peritoneal  
 
5.    Según  el  comité  de  trauma  del  colegio  americano  de  cirugía  para  decisión  del  triade  de  los  pacientes,  lo  más  
importante  a  tomar  en  cuenta  es:  
a.   Sitio  anatómico  de  la  lesión  
b.   Estados  co-­‐morbidos  del  paciente  
c.   Mecanismo  de  la  lesión  
d.   Signos  fisiológicos  anormales  
 
6.    La  primera  causa  de  mortalidad  en  pacientes  menores  de  45  años  en  occidente  es:  
a.   Las  enfermedades  infecciosas  
b.   Las  enfermedades  cardiovasculares  
c.   Las  enfermedades  cardiovasculares  
d.   Las  enfermedades  traumáticas  
 
7.    La  valoración  inicial  del  paciente  traumatizado,  según  la  guía  de  soporte  vital  avanzado  en  trauma  (ATLS)  del  
colegio  americano  de  cirujanos,  se  divide  en  4  fases:  
a.   Valoración  pre  hospitalaria,  transporte  del  paciente,  cirugía  urgente  y  cuidados  intensivos  
b.   Acceso  inmediato  al  sistema,  cuidados  pre  hospitalarios,  cuidados  hospitalarios  y  rehabilitación  
c.   Reconocimiento  primario,  resucitación,  transporte  y  cuidados  hospitalarios  
d.   Reconocimiento  primario,  resucitación  inmediata,  reconocimiento  secundario  y  tx  definitivo  
 
8.    Un   joven   de   28   años   sufre   accidente   al   caer   de   su   moto.   No   tenía   casco   protector.   Al   llegar   al   cuarto   de  
urgencias  llega  en  severo  distres  respiratorio,  Pa  80/50,  cianosis,  con  sangrado  protuso  por  la  nariz  y  fractura  
expuesta  del  fémur.  Los  ruidos  ventilatorios  se  encuentran  disminuídos  del  lado  derecho.  El  Tx  prioritario  es:  
a.   Control  de  la  hemorragia  con  empaque  nasal  anterior  y  posterior  
b.   Rx  lateral  del  cuello  
c.   Intubación  endotraqueal  con  inmovilización  neutra  del  cuello  
d.   Toracotomía  cerrada  derecha  
e.   Canalizar  2  venas  y  cruce  de  sangre  
 
9.    Al  paciente  anterior  se  le  suministran  3L  de  L/R  en  10  minutos  y  continúa  hipotenso,  usted  cree  que  necesita  
sangre  inmediatamente,  usted  transfundiría:  
a.   Sangre  tipo  específico  cruzada  
b.   Sangre  O  negativo  
c.   Sangre  O  positivo  
d.   Sangre  tipo  específico  sin  cruzar  
e.   Ninguna  de  las  anteriores  
 
10.    Después  de  una  laparotomía  por  herida  de  bala  un  hombre  de  20  años  tiene  Hb  8.5g    Pa  100/60  Fc  76x  usted  
:  
a.   Le  transfunde  1  u  de  GRE  
b.   Le  transfunde  de  1  u  de  sangre  completa  
c.   Repite  la  prueba  de  Hg  antes  de  transfundir  
d.   No  lo  transfunde  
 
11.    Una  complicación  común  en  pacientes  traumatizados  que  hay  que  controlar  precozmente  por  ser  nociva  para  
la  coagulación  y  hemodinámica  del  paciente  es  :  
a.   Hipoglucemia  
b.   Hiponatremia  
c.   Agitación  
d.   Hipotermia  
 
12.    Una  mujer  de  25  años  de  edad  se  ve  involucrada  en  un  choque  de  vehículo  de  motor  y  sufre  una  importante  
lesión  cerrada  de  cráneo,  una  contusión  pulmonar  y  fractura  de  pelvis.  Se  encuentra  sin  respuesta  a  estímulos  
externos  y  con  ventilación  en  la  unidad  de  cuidados  intensivos.  El  mejor  enfoque  para  el  Tx  de  las  necesidades  
nutricionales  de  esta  paciente  es:  
a.   Inserción  de  catéter  subclavio  e  inicio  de  hiperalimentación  central  intravenosa  
b.   Esperar  para  la  extubación  y  mejora  de  su  estado  neurológico,  que  permita  instituír  ingresos  calórico  oral  
c.   Instalar  tempranamente  una  sonda  nasogástrica  o  nasoyeyunal  para  alimentación  con  fórmula  elemental  
d.   Esperar   que   se   resuelva   el   ileo   gastrointestinal,   seguido   por   el   inicio   tardío   de   alimentación   por   sonda  
nasogástrica  con  una  fórmula  hipercalórica  
e.   Hiperalimentación  por  vía  venosa  periférica  
 
13.  Paciente  de  27  años  de  edad  presenta  lesiones  en  ambas  extremidades  superiores  al  caer  de  una  motocicleta  
a  alta  velocidad  en  la  autopista.  El  aspecto  más  importante  en  el  manejo  de  este  paciente  debe  ser:  
a.   Enfriar  las  partes  amputadas  o  isquémicas  
b.   Dx  y  Tx  de  las  injurias  que  amenacen  la  vida  del  paciente  
c.   Asegurar  una  adecuada  perfusión  
d.   Dx  y  reconocer  tempranamente  el  síndrome  compartamental  
 
14.  En  pacientes  con  traumatismos  craneoencefálicos  graves,  la  exploración  neurológica  se  realizará:  
a.   Antes  de  la  valoración  de  la  vía  aérea,  ventilación  y  circulación  
b.   Después  de  la  resucitación  inicial  
c.   Nunca  antes  de  la  llegada  a  un  hospital  y  debe  ser  realizada  por  un  neurocirujano  
d.   Después  de  la  cirugía  urgente  
e.   Después  de  la  valoración  de  la  vía  aérea  y  de  la  ventilación  y  antes  de  la  valoración  de  la  circulación  
 
15.    La  valoración  neurológica  durante  el  reconocimiento  primario  constará  de:  
a.   Valorar  nivel  de  conciencia  mediante  escala  de  glassgow  
b.   Valorar  nivel  de  conciencia  mediante  escala  de  Glasgow,  respuesta  pupilar  y  respuesta  motora    
c.   Realizar  escáner  cerebral  
d.   Valorar  nivel  de  conciencia,  reflejos  de  tronco,  miotomas  y  dermatomas  
 
16.  Paciente  de  50  años  es  arrollado  por  un  bus  y  presenta  múltiples  fracturas  costales  en  el  hemotórax  derecho,  
presenta  movimiento  paradójico  del  tórax  y  una  saturación  de  oxígeno  de  60%  con  máscara  facial  a  flujo  de  
10  L/min  y  FIO2  del  100%.  Todo  lo  siguientes  es  cierto  del  trauma  torácico  cerrado,  excepto:  
a.   Este  paciente  tiene  alta  probabilidad  de  tener  una  contusión  pulmonar  
b.   Es  recomendable  que  se  fijen  las  fracturas  costales  para  mejorar  la  ventilación    
c.   Se  le  debe  colocar  un  catéter  epidural  para  el  manejo  del  dolor  
d.   Debe  ser  enviado  a  UCI  y  considerar  entubación  endotraqueal,  luego  de  realizar  toracotomía  cerrada  
 
17.  El  paciente  anterior  es  evaluado  por  el  residente  de  cirugía  y  por  la  severidad  de  la  lesión  torácica  sospecha  
de  contusión  cardíaca  a  pesar  de  que  tiene  un  EKG  normal:  
a.   Se  le  deben  tomar  enzimas  cardiacas  seriadas  
b.   Se  debe  monitorizar  el  ritmo  cardíaco  al  paciente  y  se  trataran  las  arritmias  
c.   Se  debe  tomar  EKG  cada  hora  
d.   Se  debe  colocar  un  catéter  de  Swan  Ganz  
 
18.  Todas  las  siguientes  consideraciones  sugieren  una  lesión  uretral  excepto:  
a.   Hematoma  escrotal  
b.   Sangre  en  la  luz  del  recto  
c.   Ausencia  de  próstata  palpable  
d.   Sangre  en  el  meato  uretral  
 
19.  Después   de   sufrir   un   impacto   automovilístico   llevando   puesto   es   cinturón   de   seguridad   un   paciente   de   26  
años   de   edad   es   trasladado   al   hospital.   A   la   revisión   primaria   no   existe   evidencia   de   lesión   grave   y   solo  
manifiesta  un  discreto  dolorimiento  difuso  en  todo  el  abdomen.  Los  ruidos  intestinales  son  hipoactivos  y  la  
matidez  hepática  parece  estar  ausente.  Las  Rx  de  abdomen  revelan  aire  libre.  El  paciente  debe  de:  
a.   Ser  sometido  a  lavado  peritoneal  
b.   Ser  sometido  a  laparatomia  de  inmediato  
c.   Realizar  estudios  radiológicos  del  trato  GI  con  medio  de  contraste  
d.   Debe  ser  cuidadosamente  observado  para  descartar  evidencia  futura    
de  lesiones  intraabdominales  
 
20.  La  principal  causa  de  acidosis    en  paciente  con  traumas  múltiples  es:  
a.   Hipovolemia  
b.   Hipotermia  
c.   Descarga  adrenérgica  
d.   Rabdomiolisis  
 
21.    La  causa  menos  probable  de  depresión  del  estado  de  conciencia  en  una  paciente  con  traumas  múltiples  es:  
a.   Choque  hipovolémico  
b.   Trauma  craneal  
c.   Oxigenación  disminuida  
d.   Alcohol  y  drogas  
 
22.    Cuál  de  los  siguientes  hallazgos  debe  ser  inmediatamente  tratado  en  la  evaluación  primaria  del  paciente  de  
trauma:  
a.   Abdomen  distendido  y  doloroso  
b.   GCS  de  8  
c.   Fx  respiratoria  de  32  X  (Ventilación  asistida)  
d.   Pulso  de  130  X  
 
23.    Cuál  de  los  siguientes  elementos  es  de  mayor  ayuda  en  la  evaluación  de  un  paciente  para  prepararlo  para  
cirugía  rutinaria:  
a.   Historia  
b.   Rx  de  tórax  
c.   Examen  Físico  
d.   EKG  
 
24.    La  mortalidad  por  trauma  es  trimodal,  el  segundo  pico  corresponde  a  la  mortalidad  precoz  (4-­‐6  horas)  tras  el  
accidente,  habitualmente  en  el  ingreso  hospitalario  siendo  la  causa  más  frecuente:  
a.   El  traumatismo  craneoncefálico  y/o  exanguinación  
b.   Obstrucción  de  la  vía  aérea  y  neumotórax  a  tensión  
c.   Traumatismo  abdominal  y  /o  pélvico  
d.   Rotura  de  aorta  y  /o  cardíaca  
e.   Ninguna  de  las  anteriores  
 
25.    La  lesión  de  la  columna  cervical:  
a.   Es  excluida  por  un  examen  neurológico  normal  
b.   Puede  ser  detectada  al  flexionar  y  extender  cuidadosamente  el  cuello  en  la  evaluación  inicial  
c.   Puede  ser  excluida  al  tomarle  una  placa  lateral  de  la  columna  cervical  
d.   Puede  manifestarse  primariamente  luego  de  mover  el  cuello  sin  ningún  cuidado  
 
26.  Pareo  

a.   Laparotomía  exploradora   Paciente  con  herida  por  arma  de  fuego  en  abdomen  y  dolor  Abdominal  
A  
b.   US  FAST   D   Paciente  de  65  años  hemodinamicamente  estable  con  dolor  Abdominal  
Paciente  hipotenso  luego  de  accidente  automovilistico  sin  dolor  
c.   Laparoscopía  Dx  
B   Abdominal  
Paciente  de  20  años  con  trauma  abdominal  cerrado,  
d.   CAT   hemodinamicamente  estable,  con  USG  FAST  positivo  porlíquido  y  lesión  
  esplénica  
e.   Angiografía      
f.   Lavado  peritoneal      
 
27.  La  intubación  naso-­‐traqueal:  
a.   Se  debe  realizar  en  pacientes  inconscientes  que  no  presenten  lesión  de  columna  cervical.  
b.   Se  debe  realizar  en  pacientes  en  que  se  sospeche  lesión  cervical  y  que  estén  inconscientes.  
c.   Esta  contraindicado  en  pacientes  que  respiren  espontáneamente.  
d.   Se  debe  realizar  en  pacientes  conscientes  con  sospecha  de  lesión  cervical.  
 
28.  Varios  minutos  después  de  haberse  caído  desde  una  ventana,  un  niño  de  9  años  es  traído  por  sus  padres  al  
departamento  de  emergencias.    El  niño  esta  sangrando  profusamente  a  través  de  una  de  herida  de  10  cm  en  
su  muslo  derecho    el  tratamiento  inmediato  de  la  herida  debería  consistir  de:    
a.   Aplicación  de  torniquete.  
b.   Colocación  de  anestesia    y  punto  hemostáticos  
c.   Presión  directa  sobre  la  herida  
d.   Aplicación  de  un  apósito  de  gas  estéril.  
 
29.  Paciente   de   67   años   resulta   severamente   lesionado   al   colisionar   su   automóvil   contra   objeto   fijo,   no   tenía  
colocado   el   cinturón   de   seguridad.   Presenta   deformidad   de   la   pelvis   y   el   fémur   derecho   se   encuentra  
fracturado.     La   extracción   del   vehículo   fue   difícil   y   dura   aproximadamente   30   minutos.     Una   infusión   de  
lactato   ringer   fue   iniciada   en   la   escena   del   accidente   hasta   su   llegada   al   cuarto   de   urgencias.   Su   presión  
arterial   se   eleva   de   70/40   mm   Hg   a   130/80   mm   Hg.   A   la   hora   de   la   resucitación   el   paciente   se   encuentra  
hemodinámicamente  estable  pero  a  través  de  la  sonda  urinaria  no  presenta  diuresis.    El  paso  inicial  para  el  
tratamiento  de  esta  anuria  debe  incluir:  
a.   Furosemida  100  mg  en  bolo.  
b.   Manitol  12,5  gr.  En  bolo.  
c.   Lactato  Ringer  a  chorro.  
d.   Tomografía  axial  computarizada  de  pelvis  y  abdomen  contrastado.  
e.   Ninguna  de  las  anteriores.  
 
 
30.  Un   hombre   de   39   de   edad   es   admitido   al   cuarto   de   urgencias   luego   de   haber     sufrido   una   colisión   es   su  
automóvil.     El   paciente   se   encuentra   cianótico,   tiene   insuficiencia   respiratoria   y   una   escala   de   Glasgow   de  
5/15.  No  hay  trauma  facial  significante;  su  traque  se  encuentra  en  la  línea  media;  y  tiene  desviación  crónica  
del  tabique  nasal  que  impide  la  intubación  naso  traqueal.    La  abundante  barba  del  paciente  dificulta  la  fijación  
de  la  máscara  de  oxígeno.  El  siguiente  paso  más  apropiado  es:  
a)   Realizar  una  cricotiroidotomía  quirúrgica.  
b)   Forzar  un  naso  traqueal  a  través  del  tabique  nasal.  
c)   Intentar  una  intubación  oro  traqueal  usando  a  2  personas  para  mantener  la  columna  alineada.  
d)   Ventilarlo  con  una  mascarilla  con  válvula  y  reservorio  hasta  que  se  descarte  una  lesión  de  la  columna  
cervical.  

Trauma  Cervical  y  Traqueal  


 
31.    Paciente   de   25   años   quién   recibe   herida   por   arma   blanca   en   zona   II   del   cuello,   se   encuentra  
hemodinamicamente  estable,  no  disfonía  o  disfagia.  Todos  los  siguientes  estudios  se  encuentran   indicados,  
excepto:  
a.   Esofagoscopía  Flexible  
b.   Broncoscopía  
c.   Arteriografía  
d.   CAT  de  cuello  
 
32.  El  Tx  inicial  de  un  trauma  laríngeo  severo  es:  
a.   Traqueotomía  
b.   Cricotiroidotomia  por  punción  
c.   Intubación  orotraqueal  
d.   Oxígeno  por  cánula  nasal  
 
33.  Todos   los   siguientes   son   factores   de   riesgos   asociados   al   desarrollo   de   malignidades   de   cabeza   y   cuello,  
excepto:  
a.   Tabaquismo  
b.   Ingesta  de  agua  sin  flùor  
c.   Ingestión  de  alcohol  
d.   Exposición  a  radiación  
e.   Ninguna  de  las  anteriores  
 
34.    Un   niño   de   10   años   de   edad   se   presenta   al   cuarto   de   urgencias   luego   de   ingerir   una   bateria   de   una  
calculadora.  La  rx  de  tórax  demuestra  que  la  bateria  se  encuentra  en  el  esófago.  Cual  es  el  mejor  manejo  del  
paciente:  
a.   Observación  
b.   Endoscopia  
c.   Darle  un  vaso  de  leche  a  tomar  
d.   Administración  de  IV  de  glucagón  
e.   Administración  sublingual  de  nifedipina  
 
35.    Cual  de  los  siguientes  pacientes  debe  ser  transportado  prioritariamente    
             al  hospital:  
a.    Paciente   de   50   años   de   edad   que   cayó   de   una   altura   de   8   pies   con   fractura   de   cadera   y   signos   vitales  
normales.  
b.   Paciente  de  15  años  con  trauma  craneal  cerrado  con  Glasgow  de  13.  
c.   Paciente  de  23  años,  mujer  quien  fue  victima  de  asalto,  con  herida  cortante  en  la  espalda,  con  signos  vitales  
normales  y  sin  distress  respiratorio.  
d.   Paciente  de  3  años  de  edad.  Quien  era  pasajero  y  el  auto    en  que  viajaba  colisionó  contra  objeto  fijo  en  la  
autopista  con  signos  vitales  normales,  con  signos  de  contusión  a  nivel  de  la  pared  abdominal.  
 
36.  ¿Cuál  de  los  siguientes  enunciados  en  falso  a  cerca  de  los  traumas  craneoencefálicos?  
a.   La  mayoría  de  los  muertos  de  los  accidentes  de  autos  son  debidas  a  lesiones  craneales.  
b.   Una  rápida  y  completa  evaluación  neurológica  forma  parte  de  la  evaluación  inicial  de  trauma.  
c.   Optimizar  la  oxigenación  arterial  es  parte  fundamental  de  la  terapia  inicial  de  trauma.  
d.   En   pacientes   hipovolémicos   la   resucitación   con   líquidos   debe   ser   muy   lenta   para   no   aumentar   el   edema  
cerebral.  
 

37.  Paciente   de   25   años   quien   recibe   herida   por   arma   Blanca   en   zona   II   del   cuello,   se   encuentra  
hemodinamicamente  estable,  no  disfonía  o  disfagia.  Todos  los  siguientes  estudios  se  encuentran   indicados,  
excepto:  
a.   Esofagoscopia  flexible  
b.   Broncoscopía  
c.   Ateriografía  
d.   Tomografía  computarizada  el  cuello  
 
Trauma  Torácico  
 
38.  Paciente   de   19   años   de   edad   se   encuentra   hemodinamicamente   estable,   alerta   y   bien   orientado   luego   de  
haber  recibido  una  herida  por  arma  blanca  en  la  región  toraco  abdominal  izquierda.  La  herida  se  encuentra  
por   debajo   del   pezón,   por   arriba   del   márgen   costal   y   en   la   línea   medio   clavicular.   La   radiografía   de   tórax  
revela  un  neumotórax  de  30%.  Después  de  la  evaluación  inicial  es  completada,  el  más  apropiado  Tx  es:  
a.   Exploración  local  de  la  herida  
b.   Colocación  de  tubo  pleural  y  laparatomía  
c.   Dejar  en  observación  y  tomar  una  radiografía  control  en  6  horas  
d.   Pericardiocentesis  
 
39.  Después  de  que  ocurre  una  lesión  torácica  la  primera  maniobra  para  mejorar  la  oxigenación  es:  
a.   Intubar  al  paciente  
b.   Evaluación  de  los  gases  arteriales  
c.   Administración  de  oxígeno  suplementario  
d.   Obtener  una  Rx  lateral  de  la  columna  cervical  
 
40.  Paciente  de  18  años  de  edad  recibe  herida  por  arma  de  fuego  en  hemitórax  izquierdo  presenta  Pa  80/50,  Fc  
130   x   y   las   venas   yugulares   distendidas.   El   tratamiento   inicial   de   este   paciente   puede   consistir   en   todo   lo  
siguiente,  excepto:  
a.   L/R  2L  a  chorro  
b.   Administrar  oxígeno  suplementario  con  una  mascara  facial  con  reservorio  
c.   Toracotomía  cerrada  
                 d.  Toracotomía  abierta  y  pinzamiento  de  la  aorta  torácica  
 
 
 
41.    Todas  las  siguientes  complicaciones  del  IAM  son  indicaciones  para  cirugía,  excepto:  
a.   Extrasítoles  ventriculares  prematuras  
b.   Aneurisma  ventricular  
c.   Ruptura  ventricular  
               d.  Ruptura  del  septum  interventricular    
 
42.  Paciente  de  18  años  de  edad  recibe  herida  por  arma  de  fuego  en  hemitórax  izquierdo  presenta  Pa  80/50,  Fc  
130   x   y   las   venas   yugulares   distendidas.   El   tratamiento   inicial   de   este   paciente   puede   consistir   en   todo   lo  
siguiente,  excepto:  
a.   L/R  2L  a  chorro  
b.   Pericardiocentesis  
c.   Toracotomía  cerrada  
d.  Lavado  Peritoneal  
 
43.  Un  varón  de  26  años  de  edad  fue  atrapado  bajo  una  grúa  en  una  
           construcción.  Después  de  una   prolongada  liberación,  es  traído  al  servicio  de  urgencias,  inmovilizado  en  una  
tabla   posterior   y   recibiendo   oxígeno   al   100%   por   máscara.   Está   alerta   y   se   queja   de   dolor   en   el   pecho.   Tiene  
saturación   de   oxígeno   al   90%,   fX   respiratoria   de   35   X,   fX   cardíaca   en   120   X   y   Pa   en   85/60.   La   tráquea   está  
desviada   a   la   derecha,   hay   crepitación   a   la   palpación   del   tórax   y   disminución   de   los   ruidos   respiratorios   en   el  
hemitórax  izquierdo.    
Cuál  es  el  siguiente  paso  y  el  más  apropiado  en  la  valoración  inicial  y  el  Tx  en  este  paciente:  
a.   Sentar  al  paciente  bien  erguido  para  mejorar  el  esfuerzo  ventilatorio  y  administrar  analgésicos  para  aliviar  las  
molestias  relacionadas  
b.   Intubar  inmediatamente  y  ventilación  asistida  
c.   Tomar  Rayos  X  de  tórax  portátil  
d.   Descompresión  del  hemitórax  izquierdo  con  aguja,  seguido  de  la  inserción  de  un  tubo  de  toracostomía  
e.   Toracotomía  en  el  servicio  de  urgencias  
 
44.    Todo  es  cierto  acerca  del  trauma  cardiaco  penetrante  excepto:  
a.   El  EKG  muestra  cambios  inespecíficos  
b.   La  triada  de  Beck  se  observa  en  menos  del  40%  de  los  pacientes  con  heridas  cardiacas  
c.   Las  heridas  por  arma  de  fuego  tienen  una  mayor  mortalidad  
d.   La  presión  venosa  central  es  de  ayuda  para  descartar  la  lesión  
e.   Las  enzimas  cardiacas  están  elevadas  
 
45.  Paciente  de  25  años  quien  recibe  HPAF  en  región  de  hipocondrio  izquierdo  a  nivel  del  5  EI  y  otra  en  región  
occipital   de   la   cabeza.   El   paciente   está   intubado,   Glasgow   de   3   Pa   90/60   Fc   100   X   Fr   28   X   Rx   de   tórax   hay  
opacidad  en  todo  el  hemitórax  izquierdo,  no  se  observa  el  proyectil.  El  próximo  paso  a  seguir  en  este  paciente  
será:  
a.   Envíar  inmediatamente  CAT  cerebral  
b.   Realizar  toracotomía  cerrada  
c.   Transfundir  inmediatamente  sangre  
d.   Colocar  tubo  nasogástrico  y  sonda  urinario  
 
46.  Paciente  de  18  de  edad  recibe  herida  por  arma  de  fuego  en  hemitorax    
         derecho.   Presenta   presión   de   80/40   mm   Hg,   frecuencia   cardíaca   130   x’,   frecuencia   respiratoria   28   x´   GCS  
14/15.     Al   examen   físico   se   encuentra   intranquilo   diciendo   que   se   va   a   morir.     Los   ruidos   respiratorios   se  
encuentran   abolidos   del   lado   derecho   y   los   ruidos   cardíacos   están   presentes,   además   presentan   ingurgitación  
yugular  bilateral.    La  prioridad  en  la  atención  de  este  paciente  debe  enfocarse  en:  
a.   La  vía  aérea  
b.   La  ventilación    
c.   La  circulación  
d.   El  estado  neurológico  
 
 
47.  La  primera  medida  a  tomar  sería:  
a.   Intubación  endotraqueal  inmediata  
b.   Pericardiocentesis.  
c.   Canalizar  2  venas  periféricas  y  administrar  2  L  de  Lactato  Ringer  a  chorro.  
d.   Colocar  tubo  pleural  derecho  
 
 
48.  Con  respecto  a  la  contusión  miocárdica  causadas  luego  de  un  trauma  toráxico,  todas  son  ciertas  excepto:  
a.   Son  diagnosticadas  por  estudios  cardíacos  invasivos.  
b.   Ocurren  en  menos  del  5%  de  los  casos.  
c.   La  medición  de  enzimas  cardíacos  no  ayuda  en  el  diagnóstico.  
d.   El  tratamiento  es  conservador.  
 
49.  Después  de  un  impacto  vehicular  contra  una  columna  de  cemento  de  un  puente,  un  hombre  de  25  años  es  
trasladado  al  hospital.  El  hombre  está  intoxicado  tiene  un  nivel  de  coma  de  Glasgow  de  13  y  se  queja  de  dolor  
abdominal.     Al   momento   de   ser   admitido   en   el   hospital,   su   presión   sanguínea   era   de   80   sistólica   a   la  
palpación,   pero   con   la   administración   de   fluidos   intravenosos,   está   rápidamente   aumentó   a     110   /70   .   La  
frecuencia  cardíaca  es  ahora  de  120   palpitaciones  por  minuto.    Las  radiografías  del  tórax  muestran  pérdida  
del   botón   aórtico,   ensanchamiento   del   mediastino,     no   tiene   fracturas   costales   o   hemoneumotorax.   El  
diagnóstico  más  probable  sería:  
a.   Hemonotórax  masivo  
b.   Neumotórax  a  tensión  
c.   Ruptura  de  Aorta  
d.   Taponamiento  Cardíaco.  

50.  Un   hombre   de   55   años   fue   arrollado   por   un   auto.   En   el   cuarto   de   urgencias,   después   de   2   litros   de   LR.   Su  
presión  arterial  es  de  60/40.  la  radiografía  del  tórax  es  normal.  GCS  9/15  el  próximo  paso  a  seguir  sería:  
a.   Craneostomía  de  urgencia  
b.   Laparotomía  
c.   Dos  (2)  unidades  de  GRE  y  reevaluación  
d.   CAT  abdominal  de  urgencias  
e.   CAT  cerebral  de  urgencia.  
 
51.  Una  dama  de  40  años  estuvo  involucrada  en  un   choque   frontal  con  otro  vehículo.  Su  presión  arterial  es  de  
100/90  FC  100  xmin.  Después  de  dos  (2)  litrod  de  LR  la  presión  arterial  es  de  115/80  u  si  FC  96  min.  El  próximo  
paso  a  seguir  sería:  
a.   Laparotomía  
b.   Sangre  
c.   Observación  
d.   CAT  abdominal  
e.   Prueba  de  alcoholemia  
 
52.  Un   hombre   de   28   años   recibió   una   puñalada   en   el   cuadrante   inferior   izquierdo   del   abdomen.   Su   presión  
arterial   es   de   110/60   y   su   FC   es   de   80xmin.   Refiere   dolor   en   hemiabdomen   izquierdo.   Todas   las   siguientes  
serían  medidas  de  manejo  apropiadas.  Excepto:  (buscar  respuestas)  
a.   Dos  litros  de  LR  a  chorro  
b.   Laparotomía  
c.   Observación  
d.   Ultrasonido  Abdominal  
e.   CAT  abdominal  
 
53.  Todas   las   siguientes   son   ciertas   respecto   a   la   TAC   de   abdomen   en   el   diagnóstico   del   trauma   abdominal  
cerrado,  excepto:  
a.   Esta  indicada  en  pacientes  estables  con  dolor  abdominal  
b.   Está  indicada  en  pacientes  estables  comatosos  sin  dolor  abdominal  
c.   Está  indicada  en  pacientes  parapléjicos  
d.   Es  altamente  sensible  para  detectar  lesiones  hepáticas  
e.   Es  altamente  sensible  para  detectar  lesiones  diafragmáticas  
 
54.    El  ultrasonido  en  el  trauma  abdominal:  (buscar  respuestas)  
a.   Está  indicado  en  pacientes  con  Shock  
b.   Es  más  sensible  que  el  TAC  para  diagnosticar  lesión  de  viscera  huéca.  
c.   Está  indicado  en  pacientes  estables  con  dolor  abdominal  
d.   Requiere  la  administración  de  contraste  endovenoso  
e.   Sólo  está  indicado  cuado  no  se  puede  hacer  CAT  
 
55.  Un  hombre  de  22  años  recibió  una  puñalada  en  el  hemotórax  izquierdo.  Su  presión  arterial  es  de  60/40  y  su  
FC  140x  min.  Las  venas  yugulares  están  ingurgitadas  a  30º.  Su  diagnóstico:  
a.   Hemotórax  
b.   Lesión  de  arteria  coronaria  
c.   Fistula  broncopleural  
d.   Neumotórax    
e.   Taponamiento  cardíaco  
 
 
 
 
 
56.  Un  hombre  de  30  años  recibió  una  puñalada  en  el  hemotórax  izquierdo.  Su  presión  arterial  es  de  60/40  y  su  
FC   es   de   120   x   min.   Los   ruidos   pulmonares   están   abolidos   en   hemotórax   izquierdo   con   hiperresonancia.  
Ruidos  cardíacos  normales  taquirrítmicos.  El  próximo  paso  a  seguir  sería:  
a.   Descompresióncon  aguja  fina  
b.   Toracotomía  abierta  
c.   Toracotomía  cerrada  
d.   Ultrasonido  
e.   Ventana  pericárdica.  
 
57.  Un  hombre  de  22  años  de  edad  presenta  una  herida  de  bala  en  el  hemitorax  izquierdo.    Se  coloca  un  tubo  
pleural  y  se  evacuó  700  cc  de  sangre.    Diez  minutos  después  de  llegar  al  Cuarto  de  Urgencia  su  presión  arterial  
disminuye  a  80/60  y  la  frecuencia  cardíaca  aumenta  a  136  x’.    El  siguiente  paso  a  seguir  es:  (buscar  respuesta  
b  o  d)  
a.   Pinzar  el  tubo  torácico.  
b.   Programar  toracotomía  de  urgencia.  
c.   2  L  de  LR  a  chorro.  
d.   Repetir  la  evaluación  primaria  y  proceder  con  el  traslado.  
 
58.  Un   hombre   de   60   años   de   edad   sufre   una   puñalada   en   el   costado   posterior   derecho.     Los   testigos   del  
incidente  indican  que  el  arma  era  un  pequeño  cuchillo.    La  frecuencia  cardíaca  del  paciente  es  de  90  x  ‘,    la  
presión  arterial  es  de  120/72  y  la  frecuencia  respiratoria  es  de  24x’.  la  acción  más  apropiada  es:  
a.   Realizar  ultrasonido  abdominal.  
b.   Realizar  PIV  
c.   Realizar  una  evaluación  física  seriada.  
d.   Suturar  la  herida  y  enviar  al  paciente  a  casa.  
 
59.    Un  atleta  de  22  años  de  edad  recibe  una  puñalada  en  el  tórax  izquierdo  a  nivel  del  tercer  espacio  intercostal  
en  la  línea  axilar  anterior.    Al  ser  admitida  al  área  de  urgencias,  se  encuentra  alerta  y  despierto.    La  frecuencia  
cardíaca  es  de  100  x’  ,  la  presión  arterial  es  80/50  y  la  frecuencia  respiratoria  es  de  20  x’  .    Una  radiografía  de  
tórax  revela  un  velamiento  completo  del  hemitorax  izquierdo.  Se  coloca  un  tubo  pleural  y  se  drena  1600  cc  de  
sangre.    El  siguiente  paso  en  el  manejo  de  esta  paciente  debe  consistir  en:  
a.   Realizar  toracoscopia.  
b.   Insertar  otro  tubo  toráxico.  
c.   Prepararía  para  toracotomía  abierta.  
d.   Angiografía  y  embolización  arterial.  
 

Lesión  por  Quemaduras  


 
 
60.  Durante   una   explosión   una   mujer   de   52   años   de   edad   sufre   quemaduras   de   sol   del   50%   de   la   superficie  
corporal.   La   paciente   presenta   quemaduras   alrededor   del   tórax   y   en   ambas   extremidades   superiores.   La  
resucitación  inicial  es  realizada  en  forma  adecuada,  la  intubación  se  efectúa  por  vía  nasotraqueal  y  se  inicia  
una  ventilación  mecánica.  Los  niveles  de  carboxihemoglobina  son  del  10%,  sus  análisis  de  gases  arteriales  en  
sangre   muestran   una   PaO2   de   40   mmHg;   PaCO2   de   60   mmHg   y   pH   de   7.25.   El   manejo   inmediato   de   esta  
paciente  debe  incluír:  
a.   Aumentar  los  líquidos  
b.   Administrar  narcóticos  endovenosos  en  pequeñas  cantidades  
c.   Reevaluar  la  vía  aérea  
d.   Aplicar  presión  positiva  al  final  de  la  expiración  
 
61.    La  mejor  guía  de  adecuada  resucitación  de  los  líquidos  en  el  paciente  quemado  es:  
a.   Gasto  urinario  
b.   Normalización  de  la  frecuencia  cardiaca  
c.   4cc/kg%  de  superficie  corporal  quemada  en  24  horas  (fórmula  de  Parkland)  
d.   Normalización  de  la  presión  venosa  central  
 
62.  Paciente   de   75   años   de   edad   quien   sufre   quemaduras   secundario   a   explosión   de   un   tanque   de   gas,   la  
descripción   de   las   quemaduras   es   la   siguiente,   quemadura   de   primer   grado   en   la   cara,   toda   la   extremidad  
superior  derecha  y  miembro  inferior  derecho,  quemadura  de  segundo  grado  en  la  parte  anterior  del  dorso,  
miembro   inferior   izquierdo,   quemadura   de   3   grado   en   todo   miembro   superior   izquierdo;   el   %   de   la  
quemadura:  
a.   49.5%  
b.   45%  
c.   76.55%  
d.   58.55  
 
63.    El  tx  inicial  de  este  paciente  es:  
a.   Oxigeno  suplementario  
b.   Traslado  inmediatamente  a  un  centro  de  quemados  
c.   Administrar  analgésicos  
d.   Administrar  antibióticos  
e.   Realizar  faciotomía  
 
64.    Paciente  de  45  años  de  edad  quien  se  durmió  en  su  casa  con  un  cigarrillo  encendido  provocando  un  incendio.  
Llega   al   hospital   de   las   tablas   con   quemaduras   de   2   grado   de   50%   de   SCT,   esputo   carbonaceo   y   dolor   de  
garganta.  El  cirujano  de  turno  le  dice  que  lo  traslade  al  HST.  Lo  más  importante  antes  de  trasladarlo  es:  
a.   Canalizar  2  venas  de  grueso  calibre  y  L/R  a  chorro  
b.   Oxígeno  con  máscara  con  reservorio  a  FIO  40%  y  flujo  de  10  LPM  
c.   Entubar  al  paciente  
d.   Cubrir  las  heridas  con  vendas  húmedas  y  aplicar  sulfadizina  de  plata  
 
65.  Varón   de   22   años   previamente   sano   de   80   kg   fue   rescatado   de   una   casa   en   llamas,   después   de   quedar  
atrapado   en   una   pequeña   recámara   de   la   casa   durante   varias   horas.   Cuando   es   llevado   al   servicio   de  
urgencias  se  le  nota  combativo  y  desorientado;  sus  pulmones  están  claros  en  la  auscultación  bilateral.  La  fx  
respiratoria  es  de  30  X,  Pa  100/70  y  fX  cardiaca  de  115  X.  Sufrió  quemaduras  de  2  y  3  grado  en  15%  de  la  STC  
en  extremidades  inferiores,  con  lesión  circunferencial  de  grosor  completo  por  debajo  de  la  rodilla  derecha.  El  
paso  más  apropiado  en  el  Tx  es:  
a.   Sedación  con  midazolam  IV  
b.   Morfina  IV  para  analgesia  
c.   Administración  de  O2  al  100%  con  mascarilla  facial  
d.   Bolo  de  líquidos  IV  a  razón  de  20  cc/kg  
 
 
 
 
 
66.  El  líquido  apropiado  de  reanimación  sería:  
a.   Dextrosa  IV  en  SSN  0.5  a  razón  de  150  cc/h  
b.   L/R  a  tasa  de  mantenimiento  por  hora,  con  bolos  de  albúmina  según  sea  necesario  
c.   L/R  en  volumen  doble  de  la  tasa  de  mantenimiento  por  hora  
d.   4800  cc  de  L/R  ,  administrado  a  200  cc/h  
e.   Líquidos  de  mantenimiento  IV,  además  de  4800  cc  adicionales  de  L/R,  la  mitad  administrada  en  las  primeras  
8  hrs  y  el  resto  en  las  siguientes  16  hrs  
 
67.    Dentro  de  las  primeras  horas  de  ingreso  el  paciente  se  queja  de  dolor  en  su  pie  derecho.  Se  nota  tumefacción  
del   pie   y   del   tobillo,   disminución   del   llenado   capilar   de   los   dedos   del   pie   derecho   y   disminución   de   la  
sensibilidad.  El  Tx  de  elección  debería  ser:  
a.   Elevación  de  la  extremidad  afectada  
b.   Iniciar  antibióticos  
c.   O2  hiperbárico  
d.   Escarotomia  
 
68.  Paciente  de  24  años  de  edad  quien  es  encontrada  por  los  bomberos  dentro  de  una  casa  en  llamas,  alrededor  
de  10  minutos.  
Le   colocan   oxigeno   por   mascara   y   reservorio   a   10   LPM.   Al   llegar   al   cuarto   de   urgencias   se   encuentra  
conciente,  signos  vitales  dentro  de  límites  normales.  Presenta  desaparición  de  los  vellos  de  la  nariz  y  cejas  y  
tiene  cenizas  en  la  nariz.  Todo  lo  siguiente  es  cierto  acerca  del  tratamiento  de  esta  paciente  excepto:  
 
a.   En  los  pacientes  quemados  se  trata  inicialmente  igual  que  los  otros  tipos  de  trauma.  
b.   Si  los  pacientes  no  tienen  compromiso  de  la  vía  aérea  no  es  necesario  intubarlos.  
c.   En  pacientes  concientes  que  necesitan  tener  una  vía  aérea  definitiva  deben  de  ser  sedados.  
d.   Pacientes  quemados  desarrollan  un  Síndrome  de  Respuesta  Inflamatoria  Sistémica.    
 
69.  Paciente   quien   presenta   quemaduras   de   tercer   grado   circunferenciales   en   ambos   miembros   inferiores.   Se  
queja  de  adormecimiento  y  aumento  de  volumen  importante.  El  tratamiento  de  inmediato  debe  ser:  
a.   Elevación  de  las  piernas.  
b.   Escarotomia.  
c.   Diuréticos.  
d.   Analgésicos  
 
70.  Manejo  inicial  de  quemaduras  de  segundo  grado  y  tercer  grado:  
a.   Curación  y  antibióticos  
b.   Llevar  al  SOP  y  luego  a  cuidados  intensivos  
c.   Reanimación  ABC,  Lactato  Ringer  según  cálculo  por  superficie  corporal  quemada.  
d.   T/A  
 
71.  El  tratamiento  de  los  pacientes  por  quemaduras  eléctricas  difiere  en  las  quemaduras  por  fuego  con  respecto  a  
la  necesidad  de:  (verificar)    
a.    Fasciotomia.  
b.    Hemodiálisis.  
c.   Amputación.  
d.   Utilización  de  Oximetria  de  pulso.  
e.   Resucitación  prehospitalaria.  
 
72.  La   cantidad   de   fluido   intravenoso   en   24   horas   que   debe   recibir   una   mujer   de   60   Kg.   de   30   años   de   edad  
victima  de  quemaduras  por  flama  con  80  %  de  superficie  quemada  es  de:    
a.   19.2  litros  de  L/R  en  5%  de  D/A  .  
b.   14.4  litros  de  L/R.  
c.   9.6  litros  de  salina  hipertónica.  
d.   7.2  litros  de  L/R.  
 
73.  Es  causa  de  síndrome  de  respuesta  inflamatoria  sistémica:  
a.   Pancreatitis  aguda.  
b.   Quemaduras  de  segundo  grado  30%  de  SCT.  
c.   Infección  por  CMV.  
d.   Herida  por  arma  de  fuego  en  abdomen.  
e.   Todas  las  anteriores.  
 
Clasificación  de  Heridas  e  Infecciones  Nosocomiales  
74.    Son  señales  indirectas  de  infección  post  operatoria  todas,  excepto;  
a.   Aumento  en  el  requerimiento  de  líquidos  
b.   Ileo  adinámico  
c.   Plaquetopenia  
d.   Intolerancia  a  la  glucosa  
 
75.  Todo  lo  siguiente  incrementa  la  tasa  de  infecciones,  en  una  cirugía  mayor;  excepto:  
a.   Paciente  mayor  de  70  años  
b.   Infecciones  alejadas  del  sitio  operatorio  
c.   DM  controlada  con  glucosa  menor  de  200mg/dl  
d.   Ingesta  de  corticoides  
 
76.  Todo  lo  siguiente  incrementa  la  tasa  de  infecciones,  en  una  cirugía  mayor;  excepto:  
a.   Las  mordeduras  de  gatos  y  monos  se  debridan  y  se  dejan  abiertas  si  el  evento  es  reciente  
b.   Toda  mordedura  de  animal  que  no  ha  recibido  atención  luego  de  24  horas,  se  debe  dejar  abierta  
c.   Se  deben  lavar  con  abundante  agua  y  jabón  
d.   Las  mordeduras  tempranas  de  perro  son  debridadas  y  se  dejan  abiertas    
 
77.    La  causa  más  común  de  fiebre  en  las  primeras  48  horas  post  operatoria  
a.   Infección  de  la  herida  
b.   Colitis  por  antibióticos  
c.   Infección  asociada  a  catéteres  intra-­‐vasculares  
d.   Atelectasias  pulmonares  
 
78.    La  principal  causa  de  la  disminución  de  las  infecciones  quierugicas  se  debe  a:  
a.   Seguir  los  preceptos  de  lister  
b.   Utilización  de  antibióticos  profilácticos  
c.   Dejar  las  heridas  abiertas  
d.   Utilizar  ropas  estériles  
e.   Irrigar  las  heridas  con  desinfectantes  
 
79.    La  CDC  de  Atlanta  define  infección  del  sitio  quirurgico  como  una  infeccion  que  ocurre  en  el  sitio  de  insición  
dentro  de  los:    
a.   30  días  post  operatorio  o  dentro  de  12  meses  si  un  implsnte  protésico  a  sido  colocado  
 
 
 
 
80.  Un   hombre   de   30   años   fue   operado   por   apendicitis   aguda   perforada   con   cierre   primario   de   la   herida  
quirúrgica.  Tres  días  después  tiene  T  de  39C,  está  oligúrico  e  hipotenso,  hay  flictenas,  crepitación  y  eritema  
extenso  alrededor  de  la  herida  quirúrgica.  El  manejo  inmediato  después  de  la  resucitación  debe  ser:  
a.   Exploración  de  la  herida  con  limpieza  y  debridamiento  en  salón  de  operaciones  
b.   Laparotomía  exploratoria  
c.   Cámara  hiperbárica  
d.   Abrir  y  curar  la  herida  en  sala  
 
81.    Son   las   2   de   la   mañana   y   usted   es   llamado   por   la   enfermera   de   la   sala   de   cirugía   porque   el   paciente   fue  
operado  a  las  8  am  de  ayer  (colecistotomia  abierta),  se  queja  de  mucho  dolor.  Su  enfoque  de  Dx  y  manejo  
inicial  debe  ser:  
a.   Hx  clínica  y  examen  físico  
b.   Rx  de  tórax  
c.   Hb  completo  
d.   Urinálisis  
 
82.    Una  paciente  tiene  un  catéter  venoso  central  desde  hace  una  semana.  Hoy  cursa  con  T  38.8  C  y  se  queja  de  
dolor  en  el  sitio  de  inserción  el  cual  se  aprecia  con  escasa  secreción  mucoide.  El  siguiente  paso  a  seguir  sería:  
a.   Retirar  el  catéter  y  colocar  otro  en  el  mismo  sitio  
b.   Retirar  el  catéter  
c.   Hemocultivo  a  través  del  catéter  
d.   Vancomicina  IV  
 
83.  Un  hombre  de  62  años  se  produce  una  herida  cortante  en  la  mano  con  una  botella  de  cerveza,  cuando  estaba  
de  paseo  en  la  playa.  Acude  a  usted  para  suturarlo  y  refiere  que  tiene  todas  sus  vacunas  de  la  niñez  y  que  
hace  14  años  atrás  le  inyectaron  toxoide  tetánico.  Cuál  seria  su  decisión  acerca  de  la  prevención  de  infección  
y  tétano:  
a.   Inmunoglobulina  tetánica  más  toxoide  tetánico  
b.   Toxoide  tetánico  y  lavado  con  agua  estéril  o  solución  salina  
c.   Inmunoglobulina  tetánica  
d.   Antibióticos  más  toxoides  tetánicos  y  solución  gaudiano  
e.   Antibióticos  más  inmunoglobulina  tetánica  
 
 
84.  Un  hombre  de  62  años  se  produce  una  herida  cortante  en  la  mano  con  una  botella  de  cerveza,  cuando  estaba  
de  paseo  en  la  playa.  Acude  a  usted  para  suturarlo  y  refiere  que  tiene  todas  sus  vacunas  de  la  niñez  y  que  
hace  14  años  atrás  le  inyectaron  toxoide  tetánico.  Cuál  seria  su  decisión  acerca  de  la  prevención  de  infección  
y  tétano:  
a.   Inmunoglobulina  tetánica  más  toxoide  tetánico  
b.   Toxoide  tetánico  
c.   Inmunoglobulina  tetánica  
d.   Antibióticos  más  toxoide  tetánico  
e.   Antibióticos  más  inmunoglobulina  tetánica  
 
 
85.    Cada  una  de  las  siguientes  preguntas  escoja  una  de  las  siguientes  opciones  
 
a)   Infecciones  nosocomiales  en  las  primeras  48  hrs  
b)   Infecciones  nosocomiales  posteriores  a  48  hrs  
c)   Ambas  
d)   Ninguna  
 
       C        Hiperglicemia  
       A        Streptococo  Beta  hemolítico  
       B        Diarrea  
       C      Requerimiento  aumentado  de  líquidos  
       C    Ileo  adinámico  
 
86.  Paciente  de  50  años  de  edad  quien  desde  hace  5  días  presenta  dolor  en  epigastrio  y  en  fosa  iliaca  derecha  
asociada  a  nauseas,  vómitos  y  diarreas  liquidas.  Ha  consultado  en  tres  ocasiones,  al  cuarto  de  urgencias  y  lo  
han  tratado  con  antibióticos  y  analgésicos  sin  mejoría.  Ahora  presenta  dolor  en  fosa  iliaca  derecha  y  sensación  
de  masa  en  esta  área  y  defensa,  además  de  fiebre  y  leucocitosis  con  desviación  hacia  la  izquierda.  Es  llevado  
al  salón  de  operaciones  y  se  encuentra  una  apendicitis  aguda  perforada  con  absceso  en  fosa  iliaca  derecha  y  
pelvis.  
 
1.   Según  la  clasificación  de  las  heridas  este  paciente  tiene  una  herida:  
a-­‐  Limpia.  
b-­‐  Limpia  contaminada.  
c-­‐  Contaminada.  
d-­‐  Sucia.  
87.  Los  cuidados  postoperatorios  de  este  paciente  debe  incluir  excepto:  (verificar)  
a-­‐  Antibióticos  intravenosos  y  curación  de  la  herida  con  clorhexidina  a  7%.    
b-­‐  Ejercicios  respiratorios.  
c-­‐  Analgesia.  
d-­‐  Reposición  de  líquidos  y  electrolíticos.  
 
88.  Al    quinto  día  postoperatorio  el  paciente  cursa  con  hipotensión,  taquicardia  y  fiebre.  La  diuresis  es  menor  de  
10   c.c.   por   hora.   Además   de   distensión   abdominal.   Se   revisan   los   líquidos   administrados   y   el   balance   se  
encuentra  positivo.  Este  paciente  se  le  debe  realizar:  
a-­‐  Serie  de  abdomen  agudo.  
b-­‐  Ultrasonido.  
c-­‐  Tomografía  axial  de  abdomen  superior.  
d-­‐  Colon  por  enema.    
 
89.  El  manejo  de  este  paciente  debe  incluir  excepto:  
a-­‐  Utilización  de  inotropicos  
b-­‐  Colocación  de  catéter  central,  para  medir  presión  venosa  central.  
c-­‐  Traslado  a  la  unidad  de  cuidados  intensivos.  
d-­‐  Aumentar  los  líquidos  y  utilizar  diuréticos.  
 
90.    Todas   las   siguientes   situaciones   aumentan   el   riesgo   de   infecciones   del   sitio   quirúrgico   luego   de   una  
Herniorrafia  electiva  excepto:  
             a.        Rasurar  el  sitio  operatorio  el  día  anterior  a  la  cirugía.  
             b.        Colocar  drenajes  en  la  herida.  
             c.          No  continuar  los  antibióticos  profilácticos  más  de  24  horas.  
             d.          Dejar  hematomas  en  la  herida.  
             e.          Aproximar  fuertemente  las  suturas  sobre  la  piel.  
 
91.  Con  respecto  a  la  preparación  pre-­‐operatoria  del  paciente:  
a.   El  rasurado  debe  ser  preciso  y  meticuloso  
b.   En  casos  electivos  se  debe  lavar  al  paciente  con  Clorhexidina.  
c.   La  preparación  de  Yodo  es  bactericida,  fungicida  y  virucida.  
d.   La  tintura  de  Yodo  es  un  iodoforo  altamente  efectivo  
 
92.  Son  señales  indirectas  de  infección  post-­‐operatoria  todas,  excepto:  
a.   Aumento  en  el  requerimiento  de  líquidos  
b.   Ileo  adinámico  
c.   Hiperkalemia  
d.   Intolerancia  a  la  glucosa  
 
93.  Ud.  es  llamada  (o)  a  las  2  am    de  la  sala  de  cirugía  porque  el  paciente  que  se  le  realizó  una  colecistectomia  
abierta    a  las  8  am  del  día  anterior  se  queja  de  mucho  dolor.  
Su  enfoque  de  Dx  y  manejo  inicial  incluye  todas  las  siguientes  excepto:  
a.   Historia  Clínica  y  Examen  Físico  
b.   R-­‐X  de  Tórax  
c.   Hemograma  completo  
d.   Urinalisis  
 
94.  Todo  lo  siguiente  incrementa  la  tasa  de  infecciones,  en  una  cirugía  mayor  excepto:  (verificar  o  es  esa  o  la  E)  
a-­‐    Paciente  mayor  de  70  años.  
b-­‐    Infecciones  alejadas  del  sitio  operatorio.  
c-­‐    Diabetes  mellitus  controlada.  
d-­‐    Ingesta  de  corticoides.  
e-­‐    No  hay  excepto,  todas  aumentan  la  tasa  de  infecciones.  
 
95.  Con  respecto  a  los  antibióticos  profilácticos  todas  son  ciertas  excepto:  
a-­‐   Deben  iniciarse  el  día  antes  de  la  cirugía.  
b-­‐   Están  indicados  si  las  probabilidades  de  infección  son  altas.  
c-­‐   Están  indicados  si  las  probabilidades  de  infección  son  bajas  pero  una  infección  podría  ser  fatal.  
d-­‐   Se  requieren  niveles  séricos  óptimos  al  hacer  la  incisión.  
 
96.  Una  paciente  tiene  un  cateter  venoso  central  desde  hace  1  semana.  Hoy  cursa  con  Tº  38.8  y  se  queja  de  dolor  
en  el  sitio  de  la  inserción  el  cual  aprecia  con  escasa  secreción  mucoide.  
El  siguiente  paso  a  seguir  sería:  
a-­‐   Retirar  el  catéter  y  colocar  otro  en  el  mismo  sitio  
b-­‐   Retirar  el  catéter  
c-­‐   Hemocultivo  a  través  del  cateter  
d-­‐   Vancomicina  IV  
 
97.  El  cultivo  semi-­‐cuantitativo  de  la  punta  del  catéter  venoso  central:  (verificar)  
a-­‐   No  es  necesario  si  hay  signos  evidentes  de  infección  en  el  sitio  de  entrada  del  catéter.  
b-­‐   Es  positivo  si  se  reportan  más  de  15  colonias.  
c-­‐   Se  realiza  en  medio  de  tioglicolato.  
d-­‐   También  se  realiza  en  casos  de  sospecha  de  infección  de  vías  urinaria.  
 
98.  Todos  los  siguientes  datos  son  ciertos  acerca  del  tratamiento  de  las  mordeduras  de  animales  excepto:  
a-­‐   Se  deben  lavar  con  abundate  agua  y  jabón  
b-­‐   Las  mordeduras  de  gatos  y  monos  se  debridan  y  se  dejan  abiertas  si  el  evento  es  reciente.  
c-­‐   Toda  mordedura  de  animal  que  no  ha  recibido  atención  luego  de  24  horas,  se  debe  dejar  abierta  
d-­‐   Las  mordeduras  tempranas  de  perro  son  debridadas  y  se  dejan  abiertas.  
 
99.  Un   hombre   de   30   años   sufrió   heridas   varias   en   las   piernas   con   las   cuchillas   de   una   cortadora   de   césped  
mientras  trabajaba.  (verificar)  
El  manejo  inicial  debe  consistir  en:  
a-­‐   Toxina  tetánica  y  antiglobulina  
b-­‐   Limpieza  y  sutura  en  el  salón  de  operaciones  
c-­‐   Morfina  15  mg  IV  en  el  Servicio  de  Urgencias  
d-­‐   T/A  
 
100.   Las  tres  causas  más  probables  de  fiebre  >  38.5°  C,  36  horas  después  de  una  colectomía  son:  
a.   Atelectasia  pulmonar  
b.   Escapes  de  viscera  hueca  
c.   Infección  necrotizante  de  tejidos  blandos  
 
101.   Un   niño   es   mordido   en   la   mano   por   un   perro.   Tiene   edema   y   secreción   purulenta.   Hay   ganglios  
epitrocleares  y  un  cordón  rojo  en  el  antebrazo.  Cual  de  los  siguientes  sería  el  antibiótico  indicado:  
a-­‐   Eritromicina  
b-­‐   Ampicilina  +  sulbactan  o  Ac.  Clavulónico.  
c-­‐   Clindamicina  
d-­‐   Vancomicina  
 
102.   Con  respecto  a  los  antibióticos  profilácticos  todas  son  ciertas  excepto:  
e-­‐   Deben  iniciarse  el  día  antes  de  la  cirugía.  
f-­‐   Están  indicados  si  las  probabilidades  de  infección  son  altas.  
g-­‐   Están  indicados  si  las  probabilidades  de  infección  son  bajas  pero  una  infección  podría  ser  fatal.  
h-­‐   Se  requieren  niveles  séricos  óptimos  al  hacer  la  incisión.  
i-­‐   El  antibiótico  a  utilizar  se  escogerá  según  la  operación  a  realizar.  
 
103.   La  causa  más  común  de  fiebre  en  las  primeras  48  hrs.  Post-­‐operatorias.  
a-­‐   Infección  de  la  herida  
b-­‐   Colitis  por  antibióticos  
c-­‐   Infección  asociada  a  catéteres  intra-­‐vasculares  
d-­‐   Atelectasias  pulmonares  
 
104.   Es  causa  posible  de  infecciones  quirúrgicas  después  de  72  horas  postoperatorias:  
a-­‐   Sinusitis.  
b-­‐   Sepsis  asociada  al  catéter  venoso  central.  
c-­‐   Absceso  intra-­‐abdominal.  
d-­‐   Infección  de  la  herida  quirúrgica.  
e-­‐   Todas  las  anteriores.  
 
105.   Un   hombre   de   50   años   tiene   fiebre   38.5   y   está   distendido   5   días   después   de   una   apendicectomía   por  
apendicitis  perforada.  GB  19,000  con  85%  Neutrófilos.  El  próximo  paso  a  seguir  sería:  
a-­‐   Laparotomía  
b-­‐   TAC  de  abdomen  
c-­‐   Cambio  de  antibióticos  
d-­‐   Seguir  igual  
 
106.   Un   paciente   de   40   años   fue   operado   por   apendicitis   aguda   perforada   con   cierre   primario   de   la   herida  
quirúrgica.   Tres   días   después   tiene   temperatura   de   39   grados,   esta   oligurico   e   hipotenso   hay   flictenas,  
crepitación  y  eritema  extenso  alrededor  de  herida  quirúrgica.  
El  manejo  inmediato  después  de  la  resucitación  debe  ser:  
a-­‐   Exploración  de  la  herida  con  limpieza  y  debridamiento  extenso  en  el  SOP  
b-­‐   Laparotomía  exploradora  
c-­‐   Cámara  hiperbárica  
d-­‐   Abrir  y  curar  la  herida  en  sala  
e-­‐   Se  debe  realizar  luego  de  exámen  endoscópico  de  la  boca,  laringe,  esófago  y  tráquea.  
 
107.   La  CDC  (centro  de  control  de  infecciones  de  Atlanta)  define  una  infección   del  sitio  quirúrgico  como  una  
infección  que  ocurre  en  el  sitio  de  infección  dentro  de  los    _30    días  postoperatorios  o  dentro  de    12      meses  si  
un  implante  protésico  ha  sido  colocado.  
 
Sangrado  Digestivo  
108.    Un  hombre  de  57  años  tiene  Hb  de  8.5  g  después  de  una  hemorragia  digestiva  alta  que  ha  cesado.  Tuvo  
un  infarto  hace  3  meses  y  está  en  lista  de  espera  para  cirugía  de  re-­‐vascularización  miocárdica.  Usted:  
a.   Le  transfunde  2  u  de  GRE  
b.   Le  transfunde  2  u  de  sangre  completa  
c.   No  le  transfunde  
d.   Repite  la  prueba  de  Hb  antes  de  transfundir  
 
109.    Con  respecto  al  sangrado  por  varices  esofágicas,  excepto:  
a.   El  propanolol  es  útil  en  la  hemorragia  activa  
b.   El  octeortide  es  el  medicamento  de  elección  en  el  manejo  farmacológico  inicial  
c.   La  ligadura  o  escleropatía  de  las  varices  son  parte  del  manejo  rutinario  inicial  
d.   TIPS  significa  shunt  porto  sistémico  intra-­‐hepático  transyugular  
 
110.    Con  respecto  al  sangrado  digestivo  bajo:  
a.   Los  divertículos  son  la  causa  más  común  
b.   50%  de  los  pacientes  con  diverticulos  presentarán  episodio  de  SD  bajo  secundario  a  los  mismos  
c.   La  hemorragia  generalmente  es  de  origen  venoso  
d.   Si  el  sangrado  es  por  diverticulos  en  más  del  50%  de  las  veces  habrá  que  operar  
 
111.   Con  respecto  al  SD  todas  son  ciertas,  excepto:  
a.   Los  SD  altos  son  mucho  más  frecuentes  que  los  SD  bajos  
b.   La  primera  medida  en  el  Dx  consiste  en  pasar  un  tubo  nasogástrico  
c.   Las  plaquetas  y  el  plasma  fresco  son  utilizados  como  parte  usual  del  Tx  
d.   La  hematoquesia  ocasionalmente  puede  originarse  en  TGI  alto  
e.   La  taquicardia  es  un  signo  más  temprano  que  la  hipotensión  
 
112.   Un  hombre  de  70  años  consulta  por  hematoquezia.  Su  Pa  100/60  Fc  110  x.  Para  establecer  el  dx  todas  son  
ciertas,  excepto:  
a.   Para  poder  localizar  el  sitio  de  sangrado  la  angiografía  necesita  una  tasa  de  sangrado  superior  al  centelleo  con  
glóbulos  rojos  marcados  
b.   La  angiografía  da  una  localización  precisa  del  sitio  de  sangrado  
c.   La  colonoscopia  es  el  primer  estudio  a  realizar  
d.   El  scan  con  glóbulos  rojos  marcados  tiene  una  severidad  del  80%  
 
113.   Un    Paciente  cirrótico    acude  por  hematemesis  masiva.  Su  presión  arterial  es  de  90/70  y  su  FC  110  x  min.    
Tiene  várices  esofágicas  diagnosticadas  con  anterioridad.    El  próximo  paso  a  seguir,  sería:  
a.   Gastroscopia  
b.   Derivación  porto  cava  de  urgencia.  
c.   Observación  
d.   Análogo  de  somatostatina  IV.  
e.   Vasopresina  IV  
114.   Una  Dama  de  85  años,  consulta  por  sangrado  transrectal  color  rojo  vino.    Su  Presión  arterial  es  de  110  /  80  
y  su  FC  90  x  min.    Todos  los  siguientes  serían  conductas  apropiadas,  excepto:  
a.   Angiografía  
b.   Centelleo  con  glóbulos  rojos  marcados.  
c.   Colonoscopia  
d.   Hemostasia  quirúrgica.  
e.   Resucitación  y  estabilización.  

115.   Todos  los  siguientes  nos  hacen  sospechar  de  isquemia  intestinal,  excepto:  
a.   Arritmias  cardíacas  
b.   Fiebre  
c.   Leucocitosis  
d.   Taquicardia  
e.   Vómitos  incontrolables  
 
116.   Un   hombre   de   55   años   consulta   por   debilidad   y   evacuaciones   negras.   El   próximo   paso   a   seguir   sería:  
(buscar)  
a.   Colonoscopía  
b.   Colon  por  enema  
c.   CPRE  
d.   Gastroscopía  
e.   Serie  esófago  gastroduodenal  
 
117.   Un   hombre   de   62   años,   se   le   realizó   una   endoscopía   que   mostró   una   úlcera   duodenal   con   un   coágulo  
fresco  en  el  fondo.  El  próximo  paso  a  seguir:  
a.   Cirugía  
b.   Hemostasia  endoscópica  
c.   Observación  
d.   Somatostatina  IV  
e.   Vasopresina  IV  
 
SANGRADO  DIGESTIVO  BAJO  
 
118.   En  relación  a  la  hemorragia  digestiva  baja  el  uso  de  centelleo  con  sulfuro  coloidal  marcado  con  tecnecio:  

a.   es  depurado  por  el  sistema  reticulo  endotelial  


b.   tiene  una  vida  media  de  3  minutos  
c.   requiere  elocidad  de  sangrado  de  0.5  ml  por  minuto  
d.      todas  las  anteriores  
 
119.   Paciente  de  5  años  de  edad  es  traido  al  cuarto  de  urgancias  del  hospital  del  niño  por  presentar  sangrado  
transrectal  abundante  con  palidez  de  tegumento  y  dolor  en  FID  y  flanco  derecho.  La  causa  mas  frecuente  es:  
a.   diverticulo  de  meckel  
b.   intolerancia  a  la  leche  
c.   parasitosis  intestinal  
d.   angiodisplasia  
 
120.   El  agente  etiológico  de  la  colitis  seudomembranosa  es:  
a-­‐   E.  Coli.  
b-­‐   Enterobacter  fecalis.  
c-­‐   Klebsiella.  
d-­‐   Clostridium  difficile.  
e-­‐   Pseudomona.  
 
121.   Todos  los  postulados  en  relación  con  el  divertículo  de  Meckel  son  verdaderos  excepto:  
a-­‐   El  divertículo  de  Meckel  aparece  en  el  ileon  a  mas  o  menos  90  cm.  de  la  válvula  ileocecal.  
b-­‐   El  divertículo  de  Meckel  resulta  de  la  falla  de  obliterarse  el  ducto  vitelino.  
c-­‐   La  incidencia  del  divertículo  de  Meckel  en  la  población  general  es  del  15%.  
d-­‐   La  mucosa  gástrica  es  el  tejido  ectopico  más  comúnmente  encontrado  en  el  divertículo  de  Meckel.  
 
122.   La  complicación  más  frecuente  del  divertículo  de  Meckel  es:  
a.   Sangrado.  
b.   Diverticulitis  
c.   Obstrucción  
d.   Dolor  abdominal  intermitente  
 
123.   Los  siguientes  genes  participan  en  la  secuencia  pólipo,  adenoma  ,carcinoma  excepto:  

A.   DDC(Delete  in  colorectal  cancer)  


B.   K-­‐ras  
C.   HMSH2  
D.   APC  
 
124.   El  síndrome  de  Lynch  todas  son  ciertas  excepto:  
a.   Se  asocia  a  múltiples  pólipos  en  el  colon  
b.   El  cáncer  de  colon  se  presenta  a  edad  temprana  
c.   El  cáncer  es  más  común  en  lado  derecho  
d.   Suele  encontrarse  múltiples  primarios  
 
e.   Todas  son  ciertas  respecto  a  la  enfermedad  diverticular  del  colon  excepto:  
e.   Esta  asociada  a  dietas  bajas  en  fibras  
f.   Son  pseudo  divertículos  
g.   Se  presentan  principalmente  en  el  colon  izquierdo  
h.   Cuando  hay  sangrado  masivo  usualmente  es  del  colon  izquierdo  
i.   Ninguna  de  las  anteriores  
 
125.   Un  hombre  de  68  años  acude  por  dolor  abdominal  inespecífico  en  bajo  vientre.  Ha  perdido  20  libras  en  los  
últimos   7   meses.     Hemoglobina   en   9,8   g%  y   refiere   que   desde   hace   3   meses   viene  teniendo   problemas   de  
constipación  .  El  estudio  mas  efectivo  para  Diagnosticar  su  problema  es:  
a-­‐   SEGD.  
b-­‐   Radiografìa  simple  de  abdomen  
c-­‐   Colonoscopia.  
d-­‐   CAT.  
e-­‐   Antígeno  carcinoembrionario.  
 
126.   Sus  signos  vitales  son  PA  130/80,  FC  76  x',  con  respecto  a  la  anemia  de  este  pte  todos  son  ciertos  excepto:  
a-­‐   No  requiere  transfusión,  si  no  tiene  cardiopatía  isquemica.  
b-­‐   El  examen  de  sangre  oculta  en  heces  debe  ser  positivo.  
c-­‐   Las  transfusiones  de  sangre  perioperatorias  empeoran  el  pronóstico.  
d-­‐   Debe  ser  transfundido  los  más  pronto  posible  para  evitar  complicaciones.  
 
127.   El  mejor  examen  para  evaluar  la  posibilidad  de  metástasis  hepáticas  y  pélvicas  sería                    CAT  
 
Posterior  a  la  cirugía  este  pte  debe  ser  seguido  con  determinaciones  séricas  de:  
a-­‐   Alfa  feto  proteína.  
b-­‐   Antígeno  carcinoembrionario.  
c-­‐   Determinaciones  de  C  A    19-­‐9.  
d-­‐   Gastrina.  
 
128.   Una  mujer  de  75  años  consulta  por  dolor  en  Cuadrante  inferior  izq.  (CII)  de  48  horas  de  evolución.    T=  38.3  
C,    GB=15.9  con  90%  N,  hay  defensa  y  rebote  en  CII.    El  próximo  paso  a  seguir  sería:  
 
a.   Antibióticos  IV,  hidratación  IV.  
b.   Laparotomía  
c.   Colonoscopía  
d.   Colon  por  enema                                                
e.    Ninguna  de  las  anteriores  
 
129.   Un  hombre  de  70  años  esta  siendo  tratado  por  un  episodio  de  diverticulitis  desde  hace  4  días.    Su  T  es  de  
38.5°  C,  G.B.  19.0  con  85%N.    El  próximo  paso  a  seguir  sería:  
a.   Colonoscopía  
b.   Colon  por  enema  
c.   Laparotomía  
d.   Seguir  igual  
e.   Ninguno  de  los  anteriores  
 
Un  hombre  de  70  años  esta  siendo  tratado  por  un  episodio  de  diverticulitis  desde  hace  4  días.    Su  T  es  de  38.5°  C,  
G.B.  19.0  con  85%N.    El  próximo  paso  a  seguir  sería:  
f.   Colonoscopía  
g.   Colon  por  enema  
h.   TAC  abdomino-­‐pelvico  
i.   Seguir  igual  
Ninguno  de  los  anteriores  
 
Un   hombre   de   55   años   de   edad,   con   antecedentes   de   diverticulosis,   se   presenta   con   cuadro   de   2   semanas   de  
evolución   con   disuria,   urgencia   y   neumaturia.   Además,   refiere   un   dolor   vago,   intermitente,   localizado   en  
cuadrante   inferior   izquierdo   del   abdomen,   de   tres   meses   de   evolución   y   hábitos   intestinales   irregulares.   A   la  
exploración   física,   tiene   sensación   de     plenitud   a   la   palpación   en   el   cuadrante   inferior   izquierdo,   el   cultivo   de  
orina  muestra  una  infección  polimicrobiana.  En  la  tomografía  por  computadora  se  aprecia  una  masa  inflamatoria  
en  el  lado  izquierdo  de  la  pelvis.  
130.   El  estudio  que  más  probablemente  ayuda  para  hacer  el  diagnóstico  es:  
a-­‐  Cistouretrografía  de  vaciamiento.  
b-­‐  Cistoscopia.  
c-­‐    Enema  de  bario  con  contraste  aéreo.  
d-­‐  Colonoscopia.  
e-­‐   Laparoscopia.  
 
131.   Después  de  confirmar  el  diagnóstico,  los  pasos  iniciales  más  apropiados  en  el  tratamiento  de  este  paciente  
incluyen:  
 
a-­‐  Antibióticos  de  amplio  espectro  e  inserción  de  sonda  permanente  de  vejiga.  
b-­‐  Tratar  al  enfermo  como  ambulatorio,  antibióticos  orales  y  suavizadores  del  excremento.  
c-­‐  Líquidos  claros,  citrato  de  magnesio  y  antibióticos  orales.  
d-­‐  Laparotomía  urgente.  
e-­‐  Colostomia  proximal  para  desfuncionalizar.  
 
132.   Un   hombre   de   55   años   consulta   por   fecaluria,   neumaturia   e   IVU   a   repetición.     En   la   evaluación   de   su  
padecimiento  los  siguientes  pasos  a  seguir  serían:  
a.   Colon  por  enema  y  cistoscopía  
b.   TAC  y  Urocultivo  
c.   Informarle  de  la  necesidad  de  ser  operado  
d.   Todas  las  anteriores  
e.   Ninguna  de  las  anteriores  
 
133.   El  tratamiento  del  vólvulo  del  sigmoide  es:  
a.   Rectosigmoidoscopía  inicialmente  
b.   Laparotomía  si  hay  signos  de  gangrena  
c.   Resección  y  colostomía  si  hay  que  operar  
d.   Todas  las  anteriores  
e.   Ninguna  de  las  anteriores  
 
134.   Son  indicaciones  de   cirugía  en  la  enfermedad   de  Crohn  ¿¿¿excepto?(   En   un   profeta  dice  excepto  y  otro  
no):  
                       a-­‐        Perforación.  
                       b-­‐        Sangrado.  
                       c-­‐        Obstrucción.  
                       d-­‐        Debilidad  y  desnutrición.  
 
135.   El  agente  etiológico  de  la  colitis  seudomembranosa  es:  
a-­‐   E.  Coli.  
b-­‐   Enterobacter  fecalis.  
c-­‐   Klebsiella.  
d-­‐   Clostridium  difficile.  
e-­‐Pseudomona  
 
136.   El  antibiótico  de  elección  es  para  tratar  la  colitis  seudomembranosa  es:  
a-­‐  Ampicilina.  
b-­‐  Vancomicina.    (Luego  de  Metronidazol)  
c-­‐  Cefoxitina.  
f-­‐   Gentamicina.  
 
SANGRADO  DIGESTIVO  ALTO    
137.   En  relación  al  tratamiento  de  sangrado  digestivo  alto:  
a.   el  80%  se  detiene  solo  
b.   el  50%  requiere  tratamiento  de  urgencias  
c.   el  45%  requiere  tratamiento  endoscopico  
d.   solamente  el  20%  se  detiene  solo.  
 
138.   La  principal  causa  de  sangrado  digestivo  al  en  nuestro  medio  es:  
a.   ulcera  duodenal  cronica  activa  
b.   sindrome  de  mallory  –weiss  
c.   varices  esofàgicas  
d.   leimioma  gastrico  
 
139.   el  angiograma  selectivo  determina  el  sitio  de  sangramiento  agudo  cuando  este  es  mayor  de    
a.   0,1  ml  por  min  
b.   0,5  ml  por  min  
c.   1  ml  por  min  
d.   5  ml  por  minuto  
 
140.   En  relacion  a  la  hemorragia  digestiva  alta.  Usted  tiene  una  pte  que  acude  cuando  con  historia  de  haber  
vomitado  sangre  la  noche  anterior  esta  hemodinamicamente  estable,  le  realiza  una  endoscopia  digestiva  alta  
de  urgencia  y  el   hallazgo  endoscópico  es   una  ùlcera  duodenal  activa  con  vaso  visible,  sin  sangre  digerida  o  
fresca  en  el  estomago  usted.  
a.   lo  envia  a  casa  porqu  eno  hay  sangre  endoscopica.  
b.   Lo  envia  a  casa  con  recomendaciones  de  peptobismol,  ranitidina  y  motronidazol  
c.   Lo  hopitaliza  pues  esta  es  una  lesion  de  alto  riesgo  de  resangrado  
d.   Lo  envia  a  ca  pues  al  igual  que  la  ulcer  e  fondo  limpio  este  hallazgo  no  se  asocia  con  resangrado.  
 
Apendicitis  
 
141.    Paciente  femenina  de  18  años  acude  al  cuarto  de  urgencias  por  dolor  abdominal  de  aparición  súbita  en  
bajo  vientre  de  3  horas  de  evolución.  Al  Examen  físico  el  dolor  es  difuso  pero  de  predominio  en   Fosa  iliaca  
derecha,  sin  signos  peritoneales  GB  9000  con  75%  de  neutrófilos.  Cuál  de  las  siguientes  afirmaciones  es  cierta:  
a.   La  ausencia  de  signos  peritoneales  excluye  un  cuadro  quirúrgico  
b.   El  hemograma  normal  excluye  un  cuadro  quirúrgico  
c.   El  USG  pélvico  no  está  indicado  
d.   La  paciente  debe  ser  re-­‐evaluada  en  4-­‐5  horas  
 
142.    Con  respecto  al  caso  anterior  todas  son  ciertas,  excepto:  
a.   Se  deben  iniciar  antibióticos  inmediatamente  
b.   Se  debe  solicitar  un  urinálisis  
c.   La  historia  menstrual  de  la  paciente  es  muy  importante  
d.   El  dolor  difuso  es  típico  del  dolor  viceral  
 
143.   Una  mujer  de  18  años  tiene  historia  de  24  horas  de  evolución  de  dolor  de  inicio  en  mesogastrio  y  luego  
irradiado  a  bajo  vientre  T  38.5  C,  GGB  13.5  con  96%  de  neutrófilos.  FUM  hace  3  semanas.  Examen  fisico  hay  
signos  de  irritación  peritoneal  en  bajo  vientre,  principalmente  en  fosa  iliaca  derecha.  El  examen  ginecológico  
es  normal,  orto  negativo.  USG  pélvico  es  normal.  El  próximo  paso  a  seguir  sería:  
a.   Observación  por  12  horas  
b.   Apendicectomía  
c.   Laparascopía  diagnóstica  
d.   Colon  por  enema  
 
Dolor  Abdominal  
 
144.    Con  respecto  al  dolor  abdominal  todas  son  ciertas;  excepto:  
a.   La  apendicitis  aguda  es  el  Dx  quirúrgico  más  común  
b.   Los  mayores  de  50  años  deben  ser  estudiados  aún  cuando  desaparezca  el  dolor  
c.   El  dolor  parietal  es  localizado  
d.   El  primer  síntoma  de  una  obstrucción  intestinal  es  la  distensión  
   
145.    Hombre  de  35  años  con  dolor  en  fosa  iliaca  derecha  de  3  días  de  evolución  T  37.5C  GB  9.0  con  75%  de  
neutrófilos,  dolor  y  defensa  en  fosa  ilíaca  derecha.  Varios  episodios  similares  en  las  últimas  dos  semanas.  Al  
examen  físico  hay  aparente  masa  en  fosa  iliaca  derecha.  El  próximo  paso  a  seguir  sería:  
a.   US  abdominal  
b.   TAC  
c.   Laparotomía  
d.   Laparascopía  
 
146.    Después   de   tres   días   de   antibióticos   IV   su   es   de   T   38.5   C,   GB   22   con   82%   neutrófilos.   El   dolor   a  
aumentado.  El  próximo  paso  a  seguir  será:  
a.   Nuevo  estudio  de  imagenología  
b.   Colonoscopía  
c.   Colon  por  enema  
d.   Laparotomía  
 
147.    El  Tx  inicial  de  la  obstrucción  intestinal  incluye  todas,  excepto:  
a.   Reposición  de  líquidos  y  electrólitos  
b.   Colocación  de  sondas  y  monitorización  de  signos  vitales  
c.   Radiografía  de  tórax  y  abdomen  
d.   Cirugía  inmediata  
 
148.    Todas  son  causas  comunes  de  obstrucción  mecánica  del  colon,  excepto:  
a.   vólvulo  del  sigmoides  
b.   Cáncer  de  colon  
c.   Diverticulitis  sigmoidea  
d.   Síndrome  de  Ogilvie  
 
149.   La  causa  más  común  de  obstrucción  intestinal  es:  
a.   Cáncer  de  colon  
b.   Divertículo  de  Meckel  
c.   Adherencia  post  operatorias  
d.   Diverticulitis  sigmoideas  
 
150.    En  la  obstrucción  intestinal  son  signos  de  sufrimiento  todos,  excepto:  
a.   Leucocitosis  
b.   Fiebre  
c.   Oliguria  
d.   Dolor  localizado  
 
151.    El  síntoma  más  temprano  en  la  obstrucción  intestinal  es:  
a.   Distensión  abdominal  
b.   Obstipación  
c.   Dolor  
d.   Vómitos  
 
152.    Hombre  de  65  años  acude  por  hx  de  48  horas  de  evolución  caracterizada  por  dolor  cólico,  obstinación  y  
distensión  abdominal,  tiene  una  laparatomia  previa,  la  rx  muestra  niveles  hidroaéreos  en  intestino  delgado  y  
gas   en   colon   derecho   y   transverso,   no   se   ve   gas   en   el   colon   izquierdo,   ni   en   la   ampolla   rectal,   cual   de   las  
siguientes  es  cierta:  
a.   La  obstrucción  del  colon  por  adherencia  es  común  
b.   La  paciente  tiene  una  obstrucción  total  del  intestino  delgado  
c.   La  obstrucción  intestinal  de  este  paciente  es  poco  probable  que  se  resuelva  con  medidas  conservadoras  
d.   Si  el  ciego  mide  más  de  5  cm  es  urgente  operarlo  
e.   Ninguna  de  las  anteriores  
 
153.    El  TAC  en  el  manejo  del  trauma  abdominal  puede  no  detectar  lesiones  de:  
a.   Diafragma  e  intestino  delgado  
b.   Hígado  y  riñones  
c.   Bazo  y  páncreas  
d.   No  tiene  utilidad  en  el  manejo  del  trauma  abdominal  
e.   Detecta  todas  las  lesiones  intra  abdominales  
 
154.   E  n  el  grupo  etario  entre  20  –  40  años  la  causa  más  común  de  dolor  abdominal  que  requiere  cirugía  es:  
a.    Obstrucción  intestinal.  
b.   Trombosis  mesentérica.  
c.   Apendicitis  aguda.  
d.   Diverticulitis  colónicas.  
 

155.   Paciente  femenina  de  18  años  acude  al  cuarto  de  urgencias  por  dolor    
                       abdominal   de   aparición   súbita   en   bajo   vientre   de   3   hrs   de   evolución.     Al   EF   el   dolor   es   difuso   pero   de  
predominio  en  FID  sin  signos  peritoneales.    GB  9000  con  75%  neu.    
1.    Cuál  de  las  afirmaciones  es  cierta:  
a.   La  ausencia  de  signos  peritoneales  excluye  un  cuadro  quirúrgicos.  
b.   El  hemograma  normal  excluye  un  cuadro  quirúrgico.  
c.   El  ultrasonido  pélvico  no  esta  indicado.  
d.   La  paciente  debe  ser  re-­‐evaluada  en  unas  4-­‐5  hrs.  
 
2.   Con  respecto  al  caso  anterior  todas  son  ciertas  excepto:  
a.   Se  deben  iniciar  antibióticos  inmediatamente.  
b.   Se  debe  solicitar  un  urinálisis.  
c.   La  historia  menstrual  de  la  paciente  es  muy  importante.  
d.   El  dolor  difuso  es  típico  del  dolor  visceral.  
 
156.   Con  respecto  al  dolor  abdominal  todas  son  ciertas  excepto:  
a.   La  apendicitis  aguda  es  e  DX.  Quirúrgico  más  común.  
b.   Los  mayores  de  50  años  deben  ser  estudiados  aún  cuando  desaparezca  el  dolor.  
c.   El  dolor  parietal  es  localizado.  
d.   El  primer  síntoma  de  una  obstrucción  intestinal      es    la  distensión.  
 
157.   Hombre  de  65  años  acude  por  historia  de  48  hrs  de  evolución  caracterizada  por  dolor  cólico,  constipación  
y   distensión   abdominal.     Tiene   una   laparotomía   previa.     RX   muestras   niveles   hidro-­‐aéreos   en   intestino  
delgado  y  gas  en  colon  derecho  y  transverso  no  se  ve  gas  en  colon  izq.  Ni  en  la  ampolla  rectal.    Cual  de  las  
siguientes  es  cierta:  
a.   La  obstrucción  del  colon  por  adherencias  en  común  
b.   El  paciente  tiene  una  obstrucción  total  del  intestino  delgado  
c.   La   obstrucción   intestinal   de   este   paciente   es   poco   probable   que   se   resuelva   con   medicinas  
conservadores  
d.   Si  el  ciego  mide  más  de  5  cms  es  urgente  operarlo  
e.   Ninguna  de  las  anteriores  
 
158.   Mujer   de   18   años   tiene   historia   de   24   horas   de   evolución   de   dolor   de   inicio   en   mesogastrio   y   luego  
irradiado  a  bajo  vientre.  T  38.5  ºC,  GB  13.5  con  96%  de  neutrófilos.  FUM  hace  3  semanas.  (buscar  respuesta)  
a.   Observación  por  12  horas  
b.   Apendicectomía  
c.   Laparoscopía  diagnóstica  
d.   Colon  por  Enema  
e.   Ninguno  de  las  anteriores  
 
159.   Hombre  de  65  años  con  dolor  en  FID  de  3  días  de  evolución  T  37.5  ºC  GB  9.0  con  75%  neu.  Dolor  y  defensa  
en  FID.    Varios  episodios  similares  en  las  últimas  dos  semanas.    Al  E.F.  hay  aparente  masa  en  FID.    El  próximo  
paso  a  seguir  sería  (buscar  respuesta)  
a.   US  abdominal  
b.   TAC  
c.   Laparotomía  
d.   Laparoscopía  
e.   Ninguna  de  las  anteriores  
 
160.   Después  de  tres  días  de  antibióticos  IV  su  Tº  es  de  38.5  ºC  GB  22  con  82%  neu.    El  dolor  ha  aumentado,    El  
próximo  paso  a  seguir  sería:  
a.   Nuevo  estudio  de  imagenología  
b.   Colonoscopía  
c.   Colon  por  enema  
d.   Laparotomía  
e.   Ninguna  de  las  anteriores.  
 
161.   En  el  diagnóstico  de  dolor  abdominal,  el  factor  más  importante  a        
                         considerar  es:  (buscar  respuesta)  
Edad  del  paciente  
Estado  socioeconómico  

a.   Ocupación  del  paciente  


b.   Sexo  
c.   Raza  
 
162.   Una  dama  de  22  años  consulta  por  dolor  abdominal  de  más  de  12  horas  de  evolución.  GB  8,000  con  66%  
neutrófilos.  Hay  defensa  y  rebote  en  todo  el  hemiabdomen  inferior.  El  próximo  paso  a  seguir  sería:  (buscar  
respuesta)  
a.   Apendicectomía  
b.   Laparoscopía  
c.   Observación  
d.   Ultrasonido  
e.   TAC  abdominal  
 
163.   Un  hombre  de   20  años  consulta  por  dolor  abdominal  de  12  horas  de  evolución.  Al  EF  no  hay  signos  de  
irritación   peritoneal.   Sólo   dolor   leve   al   a   palpación   profunda   en   FID   GB   16,000   con   85%   de   neutrófilos.   El  
diagnóstico  más  probable.  
a.   Adenitis  mesentérica  
b.   Apendicitis  aguda  
c.   Gastroenteritis  
d.   Colecititis  
e.   Pancreatitis  
 
164.   Un  hombre  de  70  años,  consulta  por  dolor  abdominal,  de  más  o  menos  24  horas  de  evolución.  El  dolor  es  
tipo  cólico.  No  hay  signos  de  irritación  peritoneal,  pero  si  hay  dolor  leve  en  fosa  ilíaca  izquierda.  GB  10,5000  N  
78%  Hgb  11.2  g.  Hay  escasos  niveles  hidroaéreos  del  intestino  delgado.  Hay  historia  de  pérdida  de  peso  no  
cuantificada  durante  los  últimos  dos  meses.  El  diagnóstico  más  probable  sería:  
a.   Apendicitis  guda  
b.   Diverticulitis  aguda  
c.   Isquemia  intestinal  
d.   Obstrucción  intestinal  
e.   Tiflitis  
 
165.   Un  hombre  de  25  añosconsulta  por  dolor  abdominal  tipo  cólico  de  más  o  menos  6  horas  de  evolución  y  
vómitos.  Tiene  una  laparotomía  previa  hace  1  año  por  trauma  penetrante.  No  esta  distendido,  no  hay  nivles  
hidroáreos.  GB  8,700  con  78%  de  neutrófilos.  El  diagnóstico  más  probable:  
a.   Apendicitis  aguda  
b.   Gastritis  
c.   Litiasis  urinaria  
d.   Obstrucción  intestinal  
 
 
Cáncer  de  Colon  
 
166.    Cuál  de  las  siguientes  se  consideran  lesiones  premalignas  del  colon:  
a.   Pólipos  inflamatorios  
b.   Pólipos  adenomatosos  
c.   Pólipos  hiperplásicos  
d.   Pólipos  hamartomatosos  
 
167.    La  causa  más  común  de  hipercalcemia  es:  
a.   Cáncer  
b.   Hiperparatiroidismo  primario  
c.   Insuficiencia  renal  
d.   Deshidratación  
 
168.   Con  respecto  al  Cancer    Colorectal  todos  son  ciertas  excepto:  
a.   Solo  los  pólipos  adenomatosos  tienen  potencial  de  malignización  
b.   Las  mutaciones  k-­‐ras  están  implicadas  en  la  transformación  de  un  adenoma  en  adenocarcinoma.  
c.   La  detección  y  remoción  de  pólipos  no  ha  logrado  reducir  la  incidencia  de  cáncer  de  colon.  
d.   El  factor  pronóstico  mas  importante  es  la  presencia  de  adenopatías  metastásicas.  
e.   Las  dietas  bajas  en  fibras  y  altas  en  grasas  aumentan  la  incidencia  de  cancer  de  colon.  
 
169.   Con  respecto  al  cáncer  colorectal  todas  son  ciertas  excepto:  
a.   En  15%  de  los  casos  hay  una  historia  familiar  positiva.  
b.   En  75%  de  los  casos  no  hay  factores  predisponentes  identificables.  
c.   Los  exámenes  de  pesquiza  se  deben  iniciar  a  los  60  años.  
d.   La   determinación   de   sangre   oculta   anualmente   mas   sigmoidoscopía   flexible   c/5   años   es   un  protocolo  
aceptable  de  pesquiza.  
e.   Colonoscopía  c/10  años  es  un  protocolo  aceptable  de  pesquiza.  
 
170.   Con  respecto  al  cancer    colorectal  todas  son  ciertas  excepto:  (a  o  c)  
 
a.   El  cancer  colorectal  es  sintomático  desde  muy  temprano  en  su  evolución.  
b.   El  tenesmo  es  un  síntoma  de  cancer  rectal.  
c.   Los  cambios  en  los  hábitos  de  defecación  son  comunes  en  el  cancer  del  colon  pero  no  del  recto.  
d.   La  anemia  es  mas  frecuente  en  canceres  de  colon  derecho.  
e.   Casi  el  70%  de  los  casos  se  presentan  al  alcance  del  sigmoidoscopio  flexible.  
 
171.   Con  respecto  al  cancer  colorectal  todas  son  ciertas  excepto:    
e.   El  antígeno  carcino-­‐embrionario  tiene  mas  utilidad  en  el  seguimiento  que  en  el  tratamiento  inicial.  
f.   La  quimioterapia  adyuvante  siempre  está  indicado  en  los  Estadios  III.  
g.   La  Quimio-­‐radiación  neo  adyuvante  no  mejora  la  sobrevida    en  el  cancer  rectal  estadío  III.  
h.   En  la  evaluación  pre-­‐operatoria  de  los  pacientes  electivos  son  importantes  la  TAC  y  las  PFH.  
i.   Si  el  paciente  esta  obstruído  los  examenes  pre-­‐operatorios  no  son  necesario  
 
172.   El  cáncer  colorectal  el  gen  p53:  
a.   se  asocia  a  mutacion  del  gen  5q21  
b.   participa  en  la  muerte  celular  programada  
c.   sensibiliza  las  cèlulas  a  los  factores  de  crecimiento  
d.   es  un  oncogen  estimulador  de  cancer    
 
173.   cual  es  la  localizacion  màs  frecunte  del  ca  de  colonico:  
a.   colon  ascendente    
b.   recto  
c.   colon  transverso  
d.   colon  descendente  
e.        sigmoides  
 
174.   mujer  anciana  admitida  por  debilidad,  asma  perdida  de  peso,  y    masa  palpable  abdominal  Ella  tiene   un  
carcinoma  de  recto.  El  sitio  anatomico  es:  
a.   recto  
b.   colon  sigmoides  
c.   colon  izquierdo  
d.   colon  transverso  
e.   colon  drecho  
 
175.   Los  pólipos  de  colon  que  se  consideran  potencialmente  malignos  son:  
a-­‐   Pólipos  inflamatorios.  
b-­‐   Pólipos  hiperplasicos.  
c-­‐   Pólipos  hamartomatosos.  
d-­‐   Pólipos  adenomatosos.  
e-­‐   Todos  los  anteriores  
 
176.   En  la  poliposis  adenomatosa  familiar  el  riesgo  de  cancer  de  colon  es;  
a.   1%  sin  tratamiento  
b.   10%  sin  tratamiento  
c.   100%  sin  tratamiento  
d.   no  hay  riesgo  de  cancer  de  colon  
 
177.   A  un  hombre  de  65  años  se  le  realizó  una  colectomía  por  un  Adeno  Carcinoma  de  sigmoides  hace  6  días.    
Actualmente  su  estado  general  es  bueno  pero  tiene  T  de  38.8°C  desde  hace  un  día.    GB=15.3  con  80%  N.    Con  
respecto  a  la  evaluación  de  la  infección  post-­‐operatoria  de  este  paciente  todas  son  ciertas  excepto:  
a.   Una   TAC   de   abdomen   es   el   mejor   método   para   investigar   la   presencia   de   infecciones   de   órganos   y  
espacios  del  sitio  operatorio.  
b.   La   presencia   de   infecciones   relacionadas   a   dispositivos   IV   solo   requiere   de   un   cultivo   cualitativo   de   la  
punta  del  catéter.  
c.   En  este  periodo  las  infecciones  de  la  herida  por  Streptococos    hemolíticos    no  son  frecuentes.  
d.   La  toxina  del  Clostridium  dificcile  es  la  responsable  de  los  síntomas  de  infecciones  entéricas.  
 
178.   según  la  clasificacion  de  dukes  para  el  cancer  de  colorectal  el  tumor  que  se  extiende  hasta  la  muscular,  se  
refiere  al  estadio:  
a.   A  
b.   B1  
c.   B2  
d.   C  
e.   D  
 
179.   cuando  el  tumor  de  colon  afecta  la  serosa  ,  según  la  clasificación  de  dukes  se  clasifica  como:  
a.   dukes  a  
b.   dukes  b2  
c.   dukes  c  
d.   dukes  d  
e.   ninguna  de  las  anteriores  
 
180.   el  síntoma  más  caracteristico  de  sospecha  de  cáncer  de  colon  izquierdo:  
a.   obstrucción  
b.   dolor  còlico  
c.   anemia  
d.   perdida  de  peso  
e.   ninguna  de  las  anteriores  
 
181.   la  prueba  mas  utilizada  para  detección  temprana  de  cáncer  de  colon  es:  
a.   rectoscopia  
b.   colon  por  enema  
c.   sangre  en  heces  
d.   colonoscopia  
e.   ninguna  de  las  anteriores  
 
Anomalías  Anorectales  
 
182.   El  examen  más  preciso  en  la  evaluación  del  cáncer  de  recto  es:  
a.   El  tacto  rectal  
b.   La  tomografía  axial  computarizada  
c.   El  ultrasonido  endorectal  
d.   La  resonancia  magnética  nuclear  
 
183.   En  la  fisura  anal  la  tríada  de  Brodie  se  caracteriza  por:  
a.   Sangrado,  dolor  y  masa  anal  
b.   Fisura  anal  lateral,  prurito  e  ictericia  
c.   Fisura  anal,  colgajo  cutáneo  y  papila  hipertrófica  
d.   Hemorroides  grado  III,  dolor  y  papila  hipertrófica.  
 
184.   Un  hombre  de  35  años  acude  con  dolor  perianal  que  no  le  permite  sentarse,  fiebre  y  diarrea  de  4  días  
de   evolución.     Al   E.F.   la   región   perianal   es   normal   excepto   por   un   endurecimiento   muy   doloroso   del   lado  
derecho  que  hace  muy  difícil  el  examen.    El  manejo  debe  ser:  
a-­‐  Incisión  y  drenaje  del  absceso  perianal  
b-­‐  Baños  de  asiento  por  hemorroides  internas.  
c-­‐  Antibióticos  por  su  gastroenteritis.  
d-­‐  Laxantes  por  su  fisura  anal.  
a-­‐   Ninguna  de  las  anteriores  
 
185.   En  la  fisura  anal  el  dolor  post-­‐defecatorio  se  debe  a:  
a.   Apertura  de  la  fisura  anal  
b.   Espasmo  del  esfínter  externo  
c.   Espasmo  del  esfínter  interno  
d.   Espasmo  del  elevador  del  ano  
 
 
186.   Las  causas  más  comunes  de  dolor  anal  son:  
a.   Fístula,  hemorroides  sangrante  y  proctalgia  fugax  
b.   Abscesos,  fístulas  y  hemorroides  internas  
c.   Cáncer  de  ano,  hemorroides  y  fisuras  
d.   Abscesos,  fisuras  y  trombosis  hemorroidal  externa  
 
187.   Las  hemorroides  internas  grado  II    son:  
a.   Las  que  sangran  sin  prolapsarse  
b.   Las  que  se  prolapsan  y  se  reducen  manualmente  
c.   Las  que  se  prolapsan  y  se  reducen  espontáneamente  
d.   Las  que  se  prolapsan  y  no  se  pueden  reducir  
 
188.   hay  una  indicación  quirurgica  clara  en  pacientes  con  enfermedad  hemorroidal:  
e.   si  las  hemorroides  son  grado  I  
f.   Si  las  hemorroides  estan  trombosadas  
g.   Si  la  paciente  esta  embarazada  y  tiene  hemorroides  grado  III  
h.   Nunca  
b.   Siempre  
 
189.   Las  hemorroides  internas  de  tercer  grado  se  caracterizan  por:  
a.   prolapso  hemorroidal  que  se  reduce  espontàneamente  
b.   prolapso  hemorroidal  que  se  reduce  manualmente  
c.   prolapso  hemorroidal  encarcelado  
d.   ninguna  de  las  anteriores.  
190.   El  Tratamiento  de  los  abscesos  anales  consiste  en  :    
a.   antibioticos  
b.   baños  de  asciento  
c.incisión  y  drenaje  
d.   anti-­‐inflamatorios  
 
191.   El  tratamiento  del  absceso  anal  siempre  es  drenaje,  excepto  en:  
a.   Ancianos  
b.   Inmunosuprimidos  
c.   Pacientes  con  insuficiencia  renal  
d.   Pacientes  leucémicos  
 
192.   En  los  abscesos  anales  los  antibióticos  están  indicados  en  :  
.   diabéticos  
.   inmunosuprimidos  
.   leucémicos  
.   todos  los  anteriores  
.   ninguno  de  los  anteriores  
 
193.   La  complicacion  mas  severa  del  abseco  anorectal  no  tratado  es:  
a.   gangrena  meleney  
b.   fascitis  necrotizante  
c.   mionecrosis  
d.   gangrena  de  fournier  
 
194.   Las  fístulas  anales  se  caracterizan  por  
a.   Orificio  perianal  con  descarga  de  pus  intermitente  
b.   Sangrado  al  evacuar  
c.   Masa  pediculada  en  el  ano  luego  de  un  absceso  
d.   Eritema  perianal  asociado  a  prurito  
 
195.   Fiebre  y  dolor  perianal  son  caracterìsticos  de:  
a.   hemorroides  grado  IV  
b.   Abseso  perianal  
c.   Fisura  anal  crònica  
d.   Quiste  pilonidal  
e.   oxiuriasis  
 
 
Cirugía  Pediátrica  
 
196.    Paciente  de  6  meses  de  edad  la  madre  refiere  que  se  encuentra  intranquilo,  no  quiere  la  leche,  vómitos  
verdosos.   Al   examen   físico   se   encuentra   intranquilo   e   impresiona   masa   dura   en   flanco   derecho.   El   dx   más  
probable  que  tenga  este  paciente  es:  
a.   Intususcepción  
b.   Plastrón  apendicular  
c.   Estenosis  pilórica  
d.   Malrotación  intestinal  
 
197.    Paciente  de  2  días  de  nacido  quién  refiere  la  madre,  que  al  llorar  presenta  masa  en  la  región  inguinal  y  
luego  se  desaparece.  El  diagnóstico  más  probable  de  este  paciente  es:  
a.   Hernia  Inguinal  
b.   Quiste  del  cordón  
c.   Hidrocele  
d.   Tumor  testicular  
     
 
198.    Una   niña   de   4   años   de   edad   previamente   sana,   es   traída   al  servicio   de   urgencias   con   un   cuadro   de   24  
horas   de   evolución   caracterizado   por   sangrado   rectal   y   mareo,   sin   síntomas   GI.   A   la   exploración   física   se  
encuentra  pálida,  Fc  140,  presenta  hipotensión  postural.  Su  abdomen  no  está  distendido  ni  es  doloroso  y  al  
examen  rectal  hay  sangre  fresca  y  coágulos  en  el  ámpula.  El  diagnóstico  más  probable  es:  
a.   Divertículo  de  Meckel  sangrante  
b.   Pólipo  rectal  juvenil  
c.   Hemorroides  
d.   Fisura  anal  
e.   Intusucepción  
 
 
199.   Paciente   de   24   horas   de   nacido,   se   observa   taquipneico,   al   examen   físico   se   encuentra   un   abdomen  
excavado  y  el  murmullo  vesicular  disminuido  en  lado  derecho  y  matidez.    El  diagnóstico  más  probable  es:  
a.   Atresia  esofágica.  
b.   Ruptura  de  bula  enfisematosa  congénita.  
c.   Hernia  diafragmática  
d.   Neumotórax  por  barotrauma.  
 
 
200.   Paciente  de  2  años  de  edad,  es  traído  por  su  madre  al  cuarto  de  urgencias  por  presentarse  irritable  con  
vómitos  biliosos,  además  presenta  heces  diarreicas  rojas  con  moco  y  espesas.  Al  examen  físico  se  palpa  una  
tumoración  en  fosa  y  flanco  derecho    el  diagnostico  más  probable  de  este  paciente  es:    
a.   Mal  rotación  intestinal  
b.   Plastrón  apendicular  
c.   Invaginación  intestinal  
d.   Megacolon  congénito.  

 
Hernias  e  Hidrocele  
 
201.    Paciente   de   25   años   de   edad,   se   queja   que   presenta   una   masa   no   reducible   del   testículo   derecho   al  
examen   físico   no   se   reduce,   hay   transiluminación   y   no   se   palpa   el   testículo   derecho.   El   diagnóstico   más  
probable  es:  
a.   Hernia  inguinal  
b.   Quiste  del  cordón  
c.   Hidrocele  
d.   Tumor  testicular  
 
202.    Un   hombre   obeso   de   58   años   acude   por   presentar   una   tumoración   inguinal   dolorosa   con   signos  
inflamatorios  leves  que  no  desaparece  desde  hace  24  horas.  Dice  que  tiene  una  hernia  del  mismo  lado  pero  
que  normalmente  se  la  puede  reducir.  Usted:  
a.   Usted  le  ordena  AINES  para  re-­‐evaluarlo  en  24  horas  
b.   Le  punciona  la  masa  
c.   Le  dice  que  necesita  cirugía  urgente  
d.   Trata  de  ver  si  se  puede  reducir  la  masa  
 
203.    Un   hombre   obeso   de   58   años   acude   por   presentar   una   tumoración   inguinal   dolorosa   con   signos  
inflamatorios  leves  que  no  desaparece  desde  hace  24  horas.  Dice  que  tiene  una  hernia  del  mismo  lado  pero  
que  normalmente  se  la  puede  reducir.  Usted:  
a.   Usted  le  ordena  AINES  para  re-­‐evaluarlo  en  24  horas  
b.   Le  punciona  la  masa  
c.   Le  dice  que  necesita  cirugía  urgente  
d.   Trata  de  ver  si  se  puede  reducir  la  masa  
e.   Le  prescribe  antibióticos  
 
204.   Cuando  un  órgano  forma  parte  del  saco  herniario,  esta  hernia  recibe  el  nombre  de:  
a.   Hernia  deslizante  
b.   Hernia  de  Ritcher  
c.   Hernia  Encarcelada  
d.   Hernia  Inguinal  indirecta  
e.   Hernia  en  pantalón  
 
205.    La  persistencia  del  proceso  vaginalis  muchas  veces  resulta  en:  
a.   Hidrocele  del  cordón  espermático  
b.   Hernia  inguinal  indirecta  
c.   Hernia  inguinal  directa  
d.   Hernia  Femoral  
e.   A  y  b  
f.   A,b,c  
 
206.    El  músculo  cremaster  que  recubre  el  cordón  inguinal  es  la  prolongación  de:  
a.   Músculo  oblicuo  externo  o  mayor  
b.   Músculo  oblicuo  interno  o  menor  
c.   Músculo  transverso  del  abdomen  
d.   Fascia  transversales  
e.   Músculo  Recto  Abdominal  
 
207.    La  tasa  más  baja  de  recurrencias  en  operaciones  por  hernias  inguinales  se  observa  con:  
a.   Herniorrafia  tipo  Bassini  
b.   Herniorrafia  tipo  Mc  Vay  
c.   Herniorrafia  abierta  con  malla  de  polipropileno  
d.   Todas  tienen  tasas  de  recurrencias  similares  
 
208.    La  hernia  inguinal  directa:  
a.   Sale  por  el  triángulo  de  Hasselbach  
b.   Es  adquirida  
c.   Está  relacionada  a  esfuerzos  físicos  
d.   Todas  las  anteriores  
 
209.    La  hernia  inguinal  indirecta:  
a.   Es  congénita  
b.   Sale  por  el  anillo  profundo  
c.   Está  rodeada  por  el  músculo  cremaster  
d.   Todas  las  anteriores  
 
Tiroides  y  Paratiroides  
 
210.    El  síntoma  más  temprano  de  hipocalcemia  post  operatoria  es:  
a.   Signo  de  Chvostek  
b.   Signo  de  trousseau  
c.   Espasmo  carpo-­‐pedal  
d.   Parestesias  peri-­‐orales  
 
211.    Son  signos  de  mal  pronóstico  en  un  paciente  con  nódulo  tiroideo  todos  menos:  
a.   Tamaño  mayor  de  3  cm  
b.   Sexo  masculino  
c.   Exposición  a  radiación  
d.   Dieta  baja  en  yodo  
e.   Edad  mayor  de  40  años  
 
212.    Una  mujer  de  60  años  asintomática  tiene  un  nódulo  tiroideo  de  3  cm  de  diámetro.  El  primer  estudio  Dx  a  
realizar  debe  ser:  
a.   Punción  con  aguja  fina  
b.   T4  
c.   Centelleo    de  tiroides  
d.   Ultrasonido  de  tiroides  
e.   CAT  de  cuello  
 
213.    Paciente  de  15  años  que  presenta  masa  en  la  región  cervical  del  cuello,  dolorosa  y  eritematosa,  al  elevar  
la  lengua  la  masa  se  mueve  hacia  arriba.  .  El  Dx  más  probable  es:  
a.   Higroma  quístico  
b.   Adenopatía  
c.   Neoplasia  de  tiroides  
d.   Quiste  tirogloso  
 
214.    Cuando  la  biopsia  de  una  masa  de  cuello  en  pacientes  con  riesgo  de  cáncer  debe  ser  realizada:  
a.   Inmediatamente  luego  de  ser  descubierta  
b.   Inmediatamente  luego  de  antibióticos  
c.   Cuando  no  hay  factores  de  malignidad  
d.   Si  la  masa  no  está  presenta  desde  la  infancia  
e.   Se  debe  realizar  luego  de  examen  endoscopico  de  la  boca,  laringe,  esófago  y  tráquea  
 
215.    Paciente  de  65  años  de  edad,  alcohólico  y  fumador  crónico,  además  de  vómitos  post  pandriales  y  pérdida  
de   peso   no   cuantificada.   Presenta   una   masa   en   región   supraesternal   izquierda   del   cuello,   el   manejo   más  
apropiado  sería:  
a.   Antibióticos  y  anti  inflamatorios  
b.   Observación  
c.   Biopsia  luego  de  endoscopia,  laringoscopía  y  broncoscopia  
d.   Biopsia  inmediata  
 
216.    Son  complicaciones  de  las  tiroidectomías  todas,  excepto:  
a.   hipocalcemia  
b.   Lesión  del  nervio  laríngeo  recurrente  
c.   Hipofosfatemia  
d.   Lesión  del  nervio  laríngeo  superior  
e.   Hemorragia  
   
217.    El  tumor  de  parótida  benigna  más  común  es  el:  
a.   Adenoma  pleomórfico  
b.   Lipoma  
c.   Adenocarcinoma  
d.   Linfoma  
e.   Ninguna  de  las  anteriores  
 
218.    La  causa  más  común  de  sodio  sérico  de  125  mEq/L  en  un  paciente  
                             post  operado  es:  
a.   Déficit  de  agua  
b.   Exceso  de  agua  
c.   Déficit  de  sodio  
d.   Exceso  de  sodio  
e.   Ninguna  de  las  anteriores  
 
219.   La  pseudohiponatremia  ocurre  secundariamente  en  la  siguiente  
                     Patología:  
a.   Enfermedades  del  SNC  
b.   Enfermedad  de  Addison  
c.   Acidosis  tubular  renal  
d.   Hiperglicemia  
e.   Hipopituitarismo  
 
Una  mujer  39  años  de  edad  se  presenta  al  consultorio  del  médico  para  la  evaluación  de  un  nódulo  palpable  en  el  
cuello  de  2  años  de  evolución.  Con  antecedente  de  enfermedad  de  Hashimoto  diagnosticada  hace  5  años,  para  la  
que   toma   hormona   tiroidea.   La   paciente   tiene   antecedente   de   radioterapia   en   tórax   de   dosis   baja   por   un  
crecimiento   del   timo   durante   la   infancia.   En   la   exploración   física   se   palpa   un   nódulo   de   2,5   cm   en   el   lóbulo  
izquierdo  de  la  tiroides,  firme  y  no  doloroso.  
220.   ¿Cuál  de  sus  antecedentes  aumenta  el  riesgo  para  cáncer  tiroideo?  
a-­‐    Grupo  de  edad  de  20-­‐40  años.  
b-­‐    Género  femenino.  
c-­‐      Radiación  de  dosis  bajas  durante  la  infancia.  
d-­‐      Antecedentes  de  la  enfermedad  de  Hashimoto.  
e-­‐      Todas  las  anteriores.  
   
221.   ¿Cuál  de  las  siguientes  es  el  mejor  paso  a  seguir.  
a-­‐  Ultrasonido  del  cuello.  
b-­‐  Centelleo  de  tiroides.  
c-­‐  Tomografía  de  cuello  y  tórax.  
d-­‐  Aspiración  del  nódulo  con  aguja  fina.  
e-­‐  Todas  las  anteriores.  
   
222.   El  hipertiroidismo  puede  ser  causado  por  las  siguientes  enfermedades  excepto:  
a.    Enfermedad  de  Graves.  
b.    Enfermedad  de  Hashimoto.  
c-­‐    Enfermedad  de  Plumier.  
d-­‐    Carcinoma  medular  de  tiroides.  
 
223.   Un  hombre  de  55  años  tiene  un  nódulo  en  el  polo  inferior  del  lóbulo  derecho   de  la  tiroides.      El  nódulo  
mide  4  cms  la    PAAF    fue  reportada  como  sospechosa.  Usted  le  recomienda.  
   
a-­‐                                  Cirugía  
b-­‐                                Iodo  radioactivo  
c-­‐                                  Observación  
d-­‐                                Otra  punción  
e-­‐                                  Tratamiento  supresivo  
   
224.   Un  hombre  de  55  años    tiene  un  nódulo  en  el  polo  inferior   del  lóbulo  derecho  de  la  tiroides.  El   nódulo  
mide  4  cm  y  la  PAAF  fue  reportada  como  sospechosa.  Usted  le  recomienda:  
a.            Cirugía  
b.            Yodo  radioactivo  
c.              Observación  
d.            Otra  punción  
e.              Tratamiento  supresivo  
     
225.   El  procedimiento  diagnostico  de  tamizaje  para  diferenciar  un  nódulo  tiroideo  maligno  de  uno  benigno:  
a.    USG  de  tiroides.  
b.    Centelleo  de  tiroides.  
c.    Biopsia  por  aspiración  con  aguja  fina.  
d.    Prueba  de  supresión  de  hormona  tiroidea.  a.  Antibióticos  de  amplio  espectro  
b.  TAC  en  las  primeras  48  horas  
c.  Atención  en  la  unidad  de  cuidados  intensivos  
d.  Laparotomía  por  necrosis  peri-­‐pancreáticas  
e.  Monitoreo  hemodinámica  invasivo  
 
   
226.   La  ronquera  secundaria  a  carcinoma  broncogénico  es  usualmente  debido  a          invasión  del  tumor  a:  
a.  Cuerdas  vocales  
b.  Nervio  Laringeo  superior  
c.  Nervio  laringeo  recurrente  
d.  Laringe  
 
Ulceras    
227.    Un   hombre   de   65   años   que   toma   AINES   diariamente   por   dolores   articulares   acude   por   historia   de  
hematemesis,  taquicardia  y  diaforesis.  La  endoscopía  alta  revela  una  úlcera  en  el  bulbo  duodenal  sin  sangrado  
activo  pero  con  vaso  visible.  El  paso  a  seguir  sería:  
a.   A  sala  con  infusión  de  omeprazole  
b.   Vaguectomía  más  piloroplastía  
c.   Aplicar  electrocauterio  o  clips  endoscopicamente  al  vaso  visible  
d.   Balón  de  compresión  intra  gástrico  
e.   Lavados  gástricos  contínuos  
 
228.    Un   sujeto   de   55   años   de   edad   asiste   al   consultorio   del   médico,   se   queja   de   dolor   en   la   parte   alta   del  
abdomen   de   2   meses   de   evolución.   El   dolor   se   describe   como   retorcijón,   localizado   en   hipogastrio,  
acompañado  de  náuseas,  el  dolor  se  exacerba  con  los  alimentos  y  pérdida  de  peso   de   10  kg,  los  últimos  2  
meses.   Tiene   antecedentes   de   tabaquismo,   ingesta   de   alcohol   ocasional,   así   como   Dx   de   úlcera   gástrica  
benigna.  Cuál  de  las  pruebas  siguientes  es  el  método  más  fidedigno  para  Dx  una  úlcera  gástrica  benigna:  
a.   Serie  esofagogastroduodenal  con  bario  
b.   Endoscopia  con  fibra  óptica  superior  
c.   CAT  abdominal  
d.   USG  endoscópico  
 
229.    Es  indicación  para  cirugía  en  la  enfermedad  úlcero  péptica:  
a.   Las  úlceras  no  curan  luego  de  12-­‐15  semanas  de  tx  
b.   Cuando  hay  recurrencia  de  la  úlcera,  a  pesar  de  su  tx  médico  
c.   Cuando  ocurre  una  complicación  
d.   Todas  las  anteriores  
e.   Ninguna  de  las  anteriores  
 
230.    Las  úlceras  gástricas  tipo  III  se  caracterizan  por:  
a.   Se  encuentran  en  la  curvatura  menor  
b.   Están  siempre  asociadas  a  una  úlcera  duodenal  
c.   Son  úlceras  gástricas  prepilóricas  
d.   Cerca  de  un  tercio  se  encuentran  asociadas  a  la  ingestión  de  aspirinas  o  AINES  
e.   Ninguna  de  las  anteriores  
 
231.    En  este  paciente  se  encontró  una  úlcera  gástrica  benigna  y  se  instaló  el  Tx  con  un  inhibidor  de  la  bomba  
de   protones   y   3   esquema   de   antibióticos   para   Helycobacter   pylori,   el   paciente   regresó   al   consultorio   del  
médico  3  meses  después  con  la  misma  sintomatología  y  en  la  revaloración  se  encontró  que  la  úlcera  gástrica  
persistía.  Cuál  de  las  siguientes  opciones  es  la  mejor  para  continuar  el  Tx:  
a.   Un  segundo  Tx  de  inhibidores  de  la  bomba  de  protones  con  triple  esquema  de  antibiótico  y  reevaluación  en  2  
meses  
b.   Un  Tx  de  prueba  con  sucralfalto  y  reevaluación  en  2  meses  
c.   Tx  quirúrgico  
d.   Un  Tx  con  prostaglandinas  y  revaloración    en  2  meses  
 
232.    Un   hombre   de   65   años   que   toma   AINES   diariamente   por   dolores   articulares   acude   por   hx   de  
hematemesis,  taquicardia  y  diaforesis.  La  endoscopia  alta  revela  una  úlcera  en  el  bulbo  duodenal  sin  sangrado  
activo  pero  con  vaso  visible,  el  paso  siguiente  a  seguir  será:  
a.   A  sala  con  infusión  de  omeprazol  
b.   Vaguectomía  más  piloroplatía  
c.   Aplicar  electrocauterio  o  clips  endoscopicamente  al  vaso  visible  
d.   Balón  de  compresión  intra  gástrico  
e.   Lavados  gástricos  contínuos  
 
233.   La  primera  línea  de  la  terapia  de  la  enfermedad  ulcero  péptica  incluye:  
a.   Vaguectomia  +  piloroplastia  
b.   Antrectomia  
c.   Determinación  de  gastrina  
d.   Descartar  Helicobacter  pylori  
e.   Anti  H2  
 
Cáncer  de  Mama  
 
234.    Paciente  de  60  años  con  cáncer  intraductal  mamario,  tumor  de  2  cm.  Sin  ganglios  palpables  en  axila,  el  
mejor  Tx  a  seguir  es:  
a.   Mastectomía  radical  modificada  
b.   Lumpectomía  más  vaciamiento  axilar  sin  radio  Tx  post  operatorio  
c.   Lumpectomía  más  biopsia  del  ganglio  centinela  con  radio  Tx  post  operatorio  
d.   Lumpectomía  solamente  
 
235.   Una  paciente  tiene  un  tumor  de  6  cm  en  la  mama.  Una  biopsia  con  aguja    
             Dx  carcinoma  ductal  infiltrante.  El  mejor  Tx  inicial  a  seguir  será:  
a.   Quimioterapia  pre  operatoria  
b.   Mastectomía  radical  modificada  
c.   Lumpectomía  más  vaciamiento  axilar  con  radioterapia  post  operatoria  
d.   Lumpectomía  más  vaciamiento  axilar  sin  radioterapia  post  operat  
e.   Ninguna  de  las  anteriores  
 
236.    Todo  lo  siguiente  es  cierto  acerca  de  cáncer  de  mama  no  invasivo,  excepto:  
a.   Se  define  como  aquel  cáncer  de  mama  que  no  avanza  más  allá  de  la  membrana  basal  
b.   Puede  ser  ductal  o  lobular  
c.   El  lobular  nunca  se  palpa  
d.   Ambos  tienen  buen  pronóstico  y  se  observan  como  microcalcificaciones  en  la  mamografía  
     
237.    Paciente  de  60  años  con  cáncer  intraductal  mamario,  tumor  de  2  cm,  sin  ganglios  palpables  en  axila,  el  
mejor  tx  a  seguir  es:  
a.   Mastectomía  radical  modificada  
b.   Lumpectomía  más  vaciamiento  axilar  sin  radio  tx  post  operatorio  
c.   Lumpectomía  más  biopsia  del  ganglio  centinela  con  radio  tx  post  operatorio  
d.   Lumpectomía  solamente  
e.   Ninguna  de  las  anteriores  
 
238.   Una   mujer   de   62   años   se   le   ha   diagnosticado   un   carcinoma   ductal   infiltrante   de   la   mama   T1   N0   M0  
receptores  estrogénicos  negativos  después  de  lumpectomía  y  biopsia  del  ganglio  centinela.    El  próximo  paso  a  
seguir  sería.  
a.   Mastectomía  radical  modificada  
b.   Quimioterapia  
c.   Radioterapia  
d.   Seguimiento  en  3  meses  
e.   Tamoxifen  
 
239.   Una  mujer  de  62  años  con  un  carcinoma  ductal  infiltrante  T2  N0  M0  receptores  estrogénicos  positivos  le  
pregunta  con  respecto  al  Tx  con  Tamoxifen.    Usted  le  aconseja  que:  (verificar  puede  ser  opción  c)  
a.   El  Tamoxifen  no  está  indicado  en  mujeres  post-­‐menopáusicas.  
b.   Debe  tomar  Tamoxifen  por  cinco  años.  
c.   Debe  tomar  Tamoxifen  por  tres  años  
d.   Si  los  ganglios  son  negativos  no  necesita  tomar  Tamoxifen.  
e.   Antes  de  tomar  una  decisión  se  debe  esperar  la  determinación  del  marcador  Her-­‐2  neu  
 
240.   Mujer  de  60  años  acude  por  presentar  una  masa  en  CSE  de  la  mama  derecha,  tiene  mamografías  que  es  
reportada  como  UNAM  asa  de  6  cms  sin  signos  mamográficos  de  malignidad.  El  próximo  paso  a  seguir  sería:  
a.   Biopsia  
b.   Nueva  mamografía  en  3  meses  
c.   Mastectomía  
d.   AINES  y  re-­‐evaluación  en  15  días  
e.   Biopsia  solo  si  tiene  factores  de  riesgo.  
 
241.   Una  paciente  de  45  años  se  le  ha  diagnosticado  un  carcinoma  ductal  infiltrante  T3  N1  M0  el  próximo  paso  
a  seguir  sería.  
 
a-­‐   Mastectomía  radical  modificada  
b-­‐   Quimioterapia  neoadyuvante  
c-­‐   Radioterapia  
d-­‐   Seguimiento  en  1  mes  
e-­‐   Tamoxifen  
 
242.   Una   Dama   de   45   años,   le   consulta   por   presentar   un   hallazgo   mamográfico   caracterizado   por  
microcalcificaciones   en   acúmulo,   con   sospecha   de   tumoración.     Categorizada   como   BIRADS   IV.   Al   examen  
físico    no  hay  tumoración  palpable.    El  próximo  paso  a  seguir  sería:  
   
a.   Biopsia  
b.   Estudios  genéticos  
c.   Observación  
d.   Resonancia  magnética  de  la  mama  
e.   Ultrasonido  mamario  
 
243.   A  una  mujer  de  58  años  se  le  detecta  un  tumor  de  2  cms  que  es  resecado  y  reportado  como  carcinoma  
ductal  infiltrante  con  márgenes  libres.    El  próximo  paso  a  seguir  sería:  
 
a-­‐   Biopsia  de  ganglio  centinela  
b-­‐   Mastectomía  radical  modificada  
c-­‐   Quimioterapia  neo-­‐adyuvante  
d-­‐   Radioterapia  
e-­‐   Tamoxifen  
 
244.   Las  ventajas  de  la  Mastectomía  radical  modificada  sobre  la  terapia  con  cirugía  conservadora  incluyen.    
 
a-­‐   Mayor  sobre  vida  
b-­‐   Menores  recurrencias  locales  
c-­‐   Menor  incidencia  de  metástasis  a  distancia  
d-­‐   Menor  morbilidad  
e-­‐   Ninguna  de  las  anteriores.  
 
245.   Una  mujer  de  55  años  tiene  una  tumoración  en  CSE  de  la  mama  derecha  de  4  cms  de  diámetro,  es  nodular  
cauchosa,  indolora  y  bien  definida,  tiene  6  meses  de  evolución.  Usted  le  recomienda:  
a-­‐   Mamografía  y  biopsia  
b-­‐   Mamografía  y  biopsia  solo  si  sospecha  malignidad  
c-­‐   No  hacer  nada,  es  un  fibroadenoma  
d-­‐   Mamografía  ahora  y  repetirla  en  tres  meses  para  ver  si  hay  cambios.  
e-­‐   Re-­‐evaluar  en  dos  semanas.  
 
246.   Un   Hombre   alcohólico,   de   65   años,   consulta   por   un   área   indurada   de   reciente   aparición   en   la   mama  
izquierda.    Al  examen  físico  usted  diagnostica    ginecomastia  izquierda  con  una  masa  dominante  en  el  C  S  E  y  
una  adenopatía  endurecida  en  la  axila  ipsilateral.        El  próximo  paso  a  seguir  será:  
a.   Biopsia  
b.   Estudios  genéticos  
c.   Evaluación  psiquiátrica  para  que  de  deje  de  tomar  
d.   Resonancia  magnética  de  la  mama  
e.   Ultrasonido  mamario  
 
247.   Pueden  ser  causas  de  ginecomastia  todas  excepto:  
a-­‐   Marihuana  
b-­‐   Cimetidina  
c-­‐   Cirrosis  hepática  
d-­‐   Ibuprofeno  
 
248.   Todas  las  mujeres  deben  realizarse  mamografías  de  pesquiza  anuales:  
a-­‐   A  partir  de  los  20  años  
b-­‐   A  partir  de  los  30  años  
c-­‐   A  partir  de  los  40  años  
d-­‐   A  partir  de  los  50  años    
 
249.   Una  muchacha  de  16  años,  consulta  por  una  tumoración  cauchosa  móvil,  lobulada  de  más  o  menos  3  cms.  
de  diámetro  en  el  CSE  de  la  mama  izquierda,  de  más  o  menos  seis  (6)  meses  de  evolución.  
Todos  los  siguientes  serán  apropiados  en  el  manejo  de  esta  Paciente  excepto:  
a.   Biopsia  
b.   Punción  con  aguja  fina  
c.   Reevaluación  en  tres  (3)  meses.  
d.   Resonancia  magnética  de  la  mama  
e.   Ultrasonido  mamario.  
 
250.   Una  adolescente  de  16  años  tiene  una  tumoración  bien  definida  de  5  cm.  de  diámetro,  en  el  cuadrante  
infero-­‐interno  de  la  mama  D,  es  móvil,  lobulado,  cauchosa  e  indolora.  El  tratamiento  debe  consistir  en:  
a.   Ultrasonido  y  excisión  del  fibroadenoma.  
b.   Mamografía.  
c.   Incisión  y  drenaje  de  absceso.  
d.   AINES.  
 
251.   Al  evaluar  una  masa  mamaria  son  signos  de  mal  pronostico  todos  excepto:  
a.   Edad  mayor  de  40  años.  
b.   Biopsia  previa  que  evidenció  cambios  proliferativos.  
c.   Cáncer  de  mama  en  una  hermana.  
d.   Secreción  verdosa  a  través  del  pezón.  
 
252.   Según   la   Sociedad   Americana   de   Cáncer   las   guías   a   seguir   para   el   tamizaje   del   cáncer   de   mama   son  
excepto:  
a-­‐  Toda  paciente  mayor  de  40  años  se  debe  realizar  mamografía  anual.  
b-­‐  Pacientes  menores  de  20  años  se  deben  hacer  auto  examen  de  la  mama  mensualmente.  
c-­‐  Pacientes  menores  de  38  años  deben  ser  examinadas  las  mamas  anualmente  por  un  médico.  
d-­‐   Pacientes  de  30  años  con  incremento  del  riesgo  de  cáncer  de  mama  se  debe  realizar  cada  6  meses  
mamografía  y  ultrasonido  de  mama.  
 
253.   Todo  es  cierto  acerca  de  la  mamografía  excepto:  
a-­‐  Es  útil  en  la  mujer  asintomática.  
b-­‐  El  10-­‐15%  de  los  canceres  de  mama  no  se  observan  en  la  mamografía.  
c-­‐  La  mayoría  de  los  canceres  de  mama  no  se  palpan.  
d-­‐  Es  útil  para  biopsias  dirigidas.  
 
 
254.   En  una  mujer  de  23  años  todo  los  siguientes  son  ciertos  excepto:  
a-­‐   Debe  efectuarse  un  auto  examen  manual  mensual  de  las  mamas  
b-­‐   Sus  mamas  deben  ser  examinadas  por  un  médico  por  lo  menos  una  vez  al  año.  
c-­‐   La  mamografía  es  de  poca  utilidad  porque  hay  que  irradiar  mucho  a  las  pacientes.  
d-­‐   El  auto  examen  manual  se  debe  realizar  después  de  la  menstruación  
 
255.   Paciente  de  25  años  de  edad  consulta  por  presentar  fiebre,  aumento  de  volumen  de  la  mama  derecha  y  
dolor.  Refiere  que  esta  amamantando  a  su  niña  que  tiene  1  año.  Al  examen  la  mama  se  encuentra  roja  y  dura.  
Usted  debe:  
a-­‐  Enviar  al  SOP  para  incisión  y  drenaje.  
b-­‐  Mamografía.  
c-­‐  Antibióticos,  calor  local  y  AINES.  
d-­‐Tomar  biopsia.  
   
 
256.   Pte  de  60  años  con  cancer  intraductal  mamario,  tumor  de  2  cm.  Sin  ganglios  palpables  en  axila,  el  mejor  
tratamiento  a  seguir  es:  
a-­‐   Mastectomía  radical  modificada.  
b-­‐   Lumpectomía  más  vaciamiento  axilar  sin  radio  Tx  post-­‐op.  
c-­‐   Lumpectomía  más  biopsia  del  ganglio  centinela  con  radio  Tx  post.op  
d-­‐   Lumpectomía  solamente.  
e-­‐   Ninguna  de  las  anteriores  
 
257.   Son  signos  de  mal  pronóstico  en  el  cáncer  de  mama.  Todos  excepto:  
a-­‐   Tumor  mayor  de  4  cms  
b-­‐   Ganglios  positivos  en  axila  
c-­‐   Carcinoma  lobular  in  situ  
d-­‐   Receptores  estrogénicos  negativos  
e-­‐   No  hay  exceptos,  todos  son  signos  de  mal  pronóstico.  
 
258.   Una  paciente  tiene  un  tumor  de  6  cms  en  la  mama.    Una  biopsia  con  aguja    diagnostica    carcinoma  ductal  
infiltrante.    El  mejor  Tx  inicial  a  seguir  sería:  
 
a-­‐   Quimio  terapia  pre-­‐operatoria.  
b-­‐   Mastectomía  radical  modificada.  
c-­‐   Lumpectomía  más  vaciamiento  axilar  con  radio  terapia  post-­‐operatoria.  
d-­‐   Lumpectomía  más  vaciamiento  axilar  sin  radio  terapia  post.op.  
e-­‐   Ninguna  de  las  anteriores  
 
259.   Para  cada  una  de  las  siguientes  preguntas  escoja  una  de  las  siguientes  opciones  
 
a)   Carcinoma  ductal  in  situ  
b)   Carcinoma  lobular  in  situ  
c)   Ambos  
d)   Ninguno  
 
     B    Más  común  en  mujeres  premenopáusicas  
     B      Nunca  es  palpable  
     A    Hallazgos  mamográficos  significativos  
     C      Riesgo  de  cáncer  invasivo  mas  o  menos  30%  
     B      Igual  riesgo  de  cáncer  invasivos  en  ambas  mamas  
     A    Cáncer  invasivo  se  presenta  en  el  mismo  sitio  del  cáncer                
                                     in  situ  
 
260.   Para  cada  una  de  las  siguientes  preguntas  escoja  una  de  las  siguientes  opciones  
 
a.   Carcinoma  lobular  in  situ  
b.   Carcinoma  ductual  in  situ  
c.   Ambos  
d.   Ninguno  
 
B        rara  vez  es  palpable  
A        Nunca  es  palpable  
A        Es  un  factor  de  riesgo  
B        Puede  avanzar  a  carcinoma  invasor  
B        Lumpectomía  sin  vaciamiento  axilar  
A        Observación  
A        Mastectomía  profiláctica  
D        Quimioterapia  pre-­‐operatoria  
B        Quimioterapia  post-­‐operatoria  
B        Microcalcificaciones  
 
 
Balance  Hidroelectrolítico  
261.    Usted   está   en   su   primer   día   de   internado   y   el   residente   le   pide   que   le   corrija   la   hiponatremia   a   un  
paciente.  Usted  debe  tomar  en  cuenta  lo  siguiente:  
a.   Hacer  los  cálculos  del  déficit  y  suministrar  la  mitad  en  8  horas  
b.   La  reposición  no  debe  ser  mayor  d  20  mEq/h  
c.   Examinar  al  paciente  y  revisar  el  expediente  clínico  
d.   Darle  diurético  al  paciente  
 
262.    Usted   está   en   cuidados   intensivos   de   cirugía   y   es   su   primer   día   de   internado.   La   enfermera   lo   llama  
porque  el  paciente  operado  de  bypass  coronario  no  orina,  luego  de  haber  leído  el  expediente,  que  debe  hacer  
usted:  
a.   Aumentar  los  líquidos  intravenosos  
b.   Darle  diurético  al  paciente  
c.   Decirle  a  la  enfermera  que  lo  evalúe  en  una  hora  más  
d.   Revisar  la  sonda  foley  en  otros  dice  colocar  
 
263.    Usted  está  en  sala  9  y  es  su  primer  día  de  internado.  La  enfermera  lo  llama  porque  el  paciente  operado  de  
colecistectomía  no  orina,  luego  de  haber  leído  el  expediente,  que  debe  hacer  usted:  
a.   Aumentar  los  líquidos  intravenosos  
b.   Darle  diurético  al  paciente  
c.   Decirle  a  la  enfermera  que  lo  evalúe  en  una  hora  más  
d.   Revisar  la  sonda  foley  
 
264.    Causa  de  brecha  aniónica  elevada:  
a.   Hipoperfusión  
b.   Diarrea  
c.   Fístula  pancreática  
d.   Excesiva  administración  de  ácidos  
 
265.    La  cantidad  de  mEq/L  de  sodio  que  contiene  un  L  lactato  ringer  es  de    
a.   130  
b.  154  
b.   200  
c.   145  
d.   Ninguna  de  las  anteriores  
 
266.   Paciente   de   80   años   hospitalizado   en   la   unidad   de   cuidados   intensivos   revela   los   siguientes   gases  
arteriales  ph  7.25  PCO2  92  mmHg  bicarbonato  14  mEq/L  El  trastorno  ácido  base  que  esta  padeciendo  es:  
a.   Acidosis  respiratoria  
b.   Alcalosis  respiratoria  
c.   Acidosis  metabólica  
d.   Alcalosis  metabólica  
 
 243.   Paciente   masculino   de   30   años   de   edad   que   recibe   múltiples   impacto   de   bala   en   la   región   abdominal   es  
llevado   al   salón   de   operaciones   realizándole   laparotomía   exploradora   encontrando   devascularización   del  
intestino   delgado.     Se   le   realizó   resección   de   más   o   menos   2   metros   de   intestino   delgado   y   anastomosis.       La  
cirugía  tuvo  una  duración  de  4  horas  y  se  le  transfundieron  4  unidades  de  GRE.    En  el  postoperatorio  se  encuentra  
muy  distendido  con  un  drenaje  aumentando  a  través  del  tubo  de  naso  gástrico  
 Tiene  un  peso  de  80  Kg.    
 Sus  electrolitos  son:  Na  125  meq,  K  3,0  meq,  Cl  80  meq.    Creatinina  2,5.  Bun  48.  Glc  180.    
Gases  arteriales  7,4.    HCO3  30  meq,  PCO2  56  mm  Hg.  PO2  96  mm  Hg  y    Sat  de  oxígeno  100  %.  Exceso  de  Base  más  
4.  
 
267.    La  diuresis  horaria  de  este  paciente  no  debe  ser  menor  de:  (verificar  a,c)  
a.   30  cc  por  hora  
b.   50  cc  por  hora  
c.   40  cc  por  hora  
d.   10cc  por  hora  
 
268.   La  causa  de  la  hiponatremia  es  de  tipo:  
a.   Hipotónica  –  Hipovolemica  
b.   Hipotónica    -­‐  Isovolémica.  
c.   Hipotónica    -­‐  Hipervplemica.  
d.   Isotónica.  
 
269.    La  osmolaridad  calculada  de  este  paciente  es:  
a.   200  mosm  
b.   280  mosm  
c.   150  mosm  
d.   260  mosm  
 
270.   El  déficit  de  sodio  calculado  es:  
a.   600  meq.  
b.   560meq.  
c.   720  meq  
d.   920  meq  
271.   El  desequilibrio  ácido  –  base  que  presenta  es:  
a.   Alcalosis  metabólica  compensada.  
b.   Alcalosis  metabólica  descompensada.  
 
272.    El  déficit  de  potasio  de  este  paciente  es:  
a.   Menos  de  100  meq  
b.   150  meq  
c.   250  meq  
d.   Mayor  de  200  meq.  
 
273.   En  sala  tiene  un  paciente,  el  cual  diagnostica  una  hiponatremia  y  el  déficit  de  sodio  calculado  es  de  200  
meq  .    los  líquidos  que  usted  debe  ordenar  es:  
a.   SSN  500  cc  +  NaCl  a  23,4%          4    cc.  Pp  en  10  hrs.  
b.   L/R  1000  cc    +  NaCl  a  23,4%    18    cc.  Pp  en  10  hrs.  
c.   SSN  1000  cc  +  NaCl  a  23,4%    60  cc  pp  en  10  hrs.  
d.   SSN  1000  cc  +  NaCl  a  23,4%    10    cc    p  p  en  10  hrs.  

 
Shock  
 
274.    En  cuanto  a  la  relación  a  la  patogénesis  del  shock  séptico  y  sepsis  podemos  decir  que:  
a.   El  SIRS  se  puede  producir  solo  en  pacientes  de  trauma  
b.   La  sepsis  se  define  como  un  paciente  que  presenta  SIRS,  foco  infeccioso  e  hipotensión  irreversible  
c.   La   isquemia   intestinal   producida   en   el   shock   promueve   la   aparición   de   MODS   por   aumento   de   la  
translocación  bacteriana  
d.   La   nutrición   enteral   y   parenteral   no   están   indicados   en   pacientes   con   sepsis   por   el   riesgo   de   resistencia  
periférica  a  la  insulina  
 
   
275.   El  signo  más  temprano  de  hipovolemia  en  un  paciente  es:  
a.   Hipotensión  
b.   Taquicardia  
c.   Mucosa  seca  
d.   Disminución  de  la  diuresis  
e.   Estado  de  conciencia  
 
276.    En  cuanto  a  la  relación  del  intestino  en  la  patogénesis  del  shock    
                               séptico  y  sepsis  podemos  decir  que:  
a.   La  recontaminación  intestinal  selectiva  con  antibióticos  orales  a  disminuido  las  infecciones  y  la  mortalidad  
b.   La  utilización  de  fármacos  anti-­‐inflamatorios  ha  logrado  modular  la  respuesta  inflamatoria  en  el  síndrome  de  
disfunción  multiorgánica  
c.   La   isquemia   intestinal   producida   en   el   shock   promueve   la   aparición   de   MODS   por   aumento   de   la  
translocación  bacteriana  
d.   La  nutrición  parenteral  preserva  las  vellosidades  intestinales    
 
277.    Según  American  C..  y  Care  Medicine  1991,  se  define  SIRS  como:  
a.   Signos  inespecíficos  como  temperatura  >  38C  o  <  36,  leucocitosis  o  leucopenia,  taquipnea  y  taquicardia  
b.   Signos  inespecíficos  como  fiebre,  leucocitosis,  taquicardia,  taquipnea  y  gérmenes  detectados  en  hemocultivo  
c.   Situación  de  Shock  asociada  a  signos  clínicos  inespecíficos  como  temperatura>  38C  o  <36C,  leucocitosis   o  
leucopenia,  taquipnea  y  taquicardia  
d.   Infección  diagnosticada  microbiologicamente  acompañada  de  hipotensión  y  oliguria  
e.   Hipotensión  <90  mmHg  mantenida  a  pesar  de  perfusión  de  drogas  inotrópicas  
 
278.    Según  American  C..  y  Care  Medicine  1991,  se  define  SEPSIS  como:  
a.   Signos  inespecíficos  como  temperatura  >  38C  o  <  36,  leucocitosis  o  leucopenia,  taquipnea  y  taquicardia  
b.   Signos   inespecíficos   como   fiebre,   leucocitosis,   taquicardia,   taquipnea   y   gérmenes   detectados   en  
hemocultivo  
c.   Situación   de   Shock   asociada   a   signos   clínicos   inespecíficos   como   temperatura>   38C   o   <36C,   leucocitosis   o  
leucopenia,  taquipnea  y  taquicardia  
d.   Infección  diagnosticada  microbiologicamente  acompañada  de  hipotensión  y  oliguria  
e.   Hipotensión  <90  mmHg  mantenida  a  pesar  de  perfusión  de  drogas  inotrópicas  
 
279.    Paciente  de  35  años  el  cual  es  admitido  al  SOP  por  haber  sido  atropellado  por  un  autobús  con  múltiples  
lesiones  abdominales.  Transoperatoriamente  se  trato  agresivamente  el  shock  hipovolémico  y  la  cirugía  tuvo  
una   duración   prolongada   aproximadamente   de   6   horas.   Luego   de   48   horas   usted   está   en   la   unidad   de  
cuidados  intensivos  de  turno  y  la  enfermera  le  dice  que  no  está  orinando  a  pesar  que  tiene  buena  presión  y  la  
PVC  es  normal.  Usted  luego  de  revisar  la  sonda  urinaria  decide  colocar  un  diurético  pero  no  presenta  diuresis.  
a.   Este  paciente  probablemente  tiene  una  trombosis  de  la  arteria  renal  
b.   Posiblemente  su  presión  abdominal  es  mayor  de  15  mmHg  
c.   Se  debe  iniciar  diálisis  renal  
d.   Aumenta  el  goteo  de  los  líquidos  intravenosos  
 
280.   El  tx  inmediato  del  paciente  anterior  debe  ser:  
a.   Heparina  y  estreptoquinasa  
b.   Medir  la  presión  intraabdominal  
c.   Realización  de  fístula  vascular  para  hemodiálisis  
d.   Colasión  de  filtro  en  la  vena  cava  
 
281.   Los  líquidos  utilizados  en  la  resucitación  del  paciente  en  un  shock    
                               son  los  siguientes,  excepto:  
a.   Cristaloides  
b.   Coloides  
c.   Hemoderivados  
d.   Dextrosa  al  5%  
 
282.   Están  aumentados  en  el  shock  todos,  excepto:  
a.   Vasopresina  
b.   Epinefrina  
c.   pCo2  
d.   Cortisol  
e.   Acido  Láctico  
 
 
Heridas  y  Suturas  
283.    Para  el  tx  con  antibióticos  de  las  heridas  se  debe  tomar  en  cuenta  lo  siguiente:  
a.   Tipo  de  herida  
b.   Factores  del  paciente  
c.   Mecanismo  de  lesión  
d.   Todas  las  anteriores  
 
284.    Con  respecto  a  la  preparación    pre-­‐operatoria  del  paciente:  
a.   El  rasurado  debe  ser  extenso  y  meticuloso  
b.   El  rasurado  se  debe  realizar  la  noche  anterior  a  la  cirugía    
c.   La  preparación  con  yodo  es  bactericida,  fungicida  y  virucida  
d.   La  tintura  de  yodo  es  un  iodoforo  altamente  efectivo  
 
285.    En  cuanto  a  las  suturas  quirúrgicas  todas  son  ciertas,  excepto:  
a.   Las  suturas  absorvibles  producen  menos  reacción  tisular  que  las  no  reabsorvibles  
b.   Las  suturas  polifilamentosas  tienden  a  favorecer  las  infecciones    
c.   Una  sutura  3-­‐0  es  más  delgada  que  una  2-­‐0  
d.   El  ácido  poliglicólico  es  una  sutura  que  se  absorve  por  hidrólisis  
 
286.      Diga  2  ejemplos  de  herida  según  clasificación  por  el  riego  de  infección:  
a.   Limpia:    
b.   Limpia  Contaminada:  
c.   Contaminada:  
d.   Sucia:  
 
287.    En  cuanto  a  los  antisépticos  y  el  tx  de  las  heridas  todo  es  cierto,  
       excepto:  
a.   Los  antisépticos  retrasan  el  proceso  de  cicatrización  
b.   No  se  deben  utilizar  en  heridas  limpias  
c.   Su  penetración  es  superficial  
d.   El  peróxido  de  hidrógeno  es  un  potente  bactericida  
 
288.    Con  respecto  a  los  antibióticos  profilacticos  todo  es  cierto,  
       excepto:  
a.   Están  indicados  en  colecistotomias  sin  factores  de  riesgo  
b.   Están  indicados  en  apendicectomias  
c.   Están  indicados  en  cirugías  por  obstrucción  pilórica    
d.   Deben  ser  administrados  justo  antes  de  iniciar  la  operación  
 
Neoplasias  Gástricas  
289.    Todos  los  siguientes  son  signos  de  neoplasia  gástrica  avanzada:  
a.   Nódulo  de  Wichow  
b.   Nódulo  de  sister  mary  Joseph  
c.   Signo  de  Blumer  
d.   Todas  las  anteriores  
e.   Solo  a  y  c  
 
290.   Factor  etiológico  de  mayor  importancia  en  el  desarrollo  del  Cáncer  Gástrico  es:  
a-­‐   Bajo  consumo  de  vegetales.  
b-­‐   Alto  consumo  de  almidones.  
c-­‐    Alto  consumo  de  alimentos  salados  y  ahumados    
d-­‐   Infestación  por  el  Helicobacter  pylori.  
e-­‐   Ninguna  de  las  anteriores.  
 
291.   Se  debe  sospechar  de  Gastrinoma  o  Síndrome  de  Zollinger-­‐Ellison  en  el  siguiente  caso:  
a-­‐   Ulceras  resistentes  a  tratamiento  antisecretorio.  
b-­‐   Múltiples  ulceras  duodenales.  
c-­‐   Ulceras  que  recurren  luego  de  procedimiento  quirúrgicos  adecuados.  
d-­‐   Todas  las  anteriores.  
e-­‐   Ninguna  de  las  anteriores.  
 
292.   según  la  clasificaciòn  de  Ca  Gastrico  de  Borman  la  linitis  plàstica  es  el  tipo:  
a.tipo  I  
b.   tipo  II  
c.  tipo  III  
d.   tipo  IV  
e.tipo  V  
 
293.   todos  los  siguientes  son  signos  de  neoplasia  gàstrica  avanzada,  excepto:  
a.nodulo  de  vircow  
b.   nodulo  de  sister  mary  joseph  
c.  signo  de  blummer  
d.   signo  del  Moro  
e.ninguna  de  las  anteriores  
 
294.   Pte  de  75  años  el  cual  es  traído  al  cuarto  de  urgencias  del  HST  luego  de  haber  sido  tratado  en  el  centro  de  
Salud  con  antiácidos  y  metoclopramida  con  el  diagnostico  clínico  de  gastritis.  Los  familiares  informan  que  el  
señor  ha  perdido  peso  y  que  desde  hace  un  año  lo  estaban  llevando  al  centro  de  salud  porque  se  quejaba  de  
dolor  en  epigastrio  además  de  presentar  vómitos  post-­‐prandriales.  También  han  notado  la  aparición  de  una  
masa  sobre  la  clavícula  izquierda.  El  estudio  de  elección  de  este  paciente  inicialmente  debe  ser:  
           a.    Tomografía  axial  computarizada.  
           b.    Endoscopia  digestiva  alta.  
           c.    Colonoscopia.    
d.  Biopsia  de  la  masa  clavicular  izquierda.  
 
295.   Hombre  de  68  años  se  presenta  a  su  consultorio  quejándose  de  disfagia  progresiva  durante  los  últimos  
tres  meses,  relacionada  con  incomodidad  moderada  en  el  pecho  ,  el  paciente  refiere  una  pérdida  de  peso  de  
7,5   Kg.   ,   tiene   antecedente   de   tabaquismo   positivo   durante   30   años,   e   ingesta   de   alcohol   ocasional.   A   la  
exploración   física   sin   datos   de   interés,   incluso   los   signos   vitales,   una   radiografía   de   tórax   es   normal   y   el  
esofagograma   con   bario   muestra   un   defecto   de   llenado   irregular   en   el   tercio   distal   del   esófago   con  
deformación  y  estrechamiento  de  la  luz:  
             ¿Cuál  de  los  siguientes  diagnósticos  es  el  más  probable?  
                         a-­‐    Esofagitis  por  estenosis.  
                         b-­‐  Carcinoma  de  esófago.  
                         c-­‐    Carcinoma  pulmonar  con  invasión  a  esófago.  
                         d-­‐    Divertículo  de  Zenker.  
 
296.   El  carcinoma  típico  que  se  desarrolla  en  asociación  con  el  esófago  de  Barrett  es:  
a-­‐   Epidermiode.  
b-­‐   Mucoepidermoide.  
c-­‐   Células  pequeñas.  
d-­‐   Adenocarcinoma.  
e-­‐   Células  escamosas.  
 
297.   ¿Cual  es  la  afirmación  correcta  acerca  del  cáncer  gástrico?  
a-­‐  El  cáncer  gástrico  distal  es  el  más  común  actualmente.  
b-­‐  Los  cánceres  gástricos  tempranos  están  confinados  a  la  mucosa  y  submucosa.  
c-­‐  Los  tumores  gástricos  tipos  difusos  tiene  mejor  pronósticos  que  el  tipo  intestinal.  
               d-­‐  Todas  las  anteriores  son  ciertas.  
 
298.   su   tia   de   60   años   que   desde   hace   4   semanaas   se   vanìa   sintiendo   con   una   sensaciòn   de   llenura   post-­‐
prandial  y  dolor  ocasional  en  FID.  Usted  le  recomienda  que:  
a.   se  haga  mamografìa  
b.   se  haca  una  colonoscopia  
c.   se  haga  un  Pap  cervicouterino  
d.   todas  las  anteriores.  
 
 
 
 
299.   Son  todas  características  de  la  motilina,  excepto:  
a.   Es  un  péptido  intestinal  
b.   Es  secretado  por  las  células  enterocromafines  del  intestino  delgado  
c.   La  acidficación  del  duodeno  causa  aumento  de  su  liberación  
d.   La  eritromicina  inhibe  su  liberación  
 
Enfermedades  del  Hígado  
 
300.    Lesión  benigna  más  común  encontrada  en  el  hígado:  
a.   Metástasis  neoplásicas  
b.   Hepatocarcinoma  
c.   Hemangioma  
d.   Abcesos  hepáticos  
 
301.    La  ecuación  de  Harris  Benedict  nos  sirve  para:  
a.   Cuantificar  las  proteínas  totales  
b.   Cuantificar  los  carbohidratos  
c.   Nos  proporciona  el  gasto  energético  basal  
d.   Cuantificar  los  lípidos  
e.   Todas  las  anteriores  
 
302.    La  irrigación  del  hígado  esta  dada  por:  
a.   Arteria  hepática  25%,  vena  porta  85%  
b.   Arteria  hepática  75%,  vena  porta  25%  
c.   Arteria  hepática  25%,  vena  porta  75%  
d.   Arteria  hepática  50%,  vena  porta  50%  
 
Enfermedades  del  Bazo  
 
303.    Primer  signo  que  sugiere  rotura  traumática  del  bazo:  
a.   Inconciencia  
b.   Dolor  abdominal  
c.   Hipotensión,  palidez  
d.   Hematemesis,  taquicardia,  hipotensión  
 
ERGE  
304.    El  esófago  inferior  de  un  paciente  es  inadvertidamente  perforado  durante  una  endoscopia.  Un  trago  de  
bario   hecho   inmediatamente   luego   de   la   perforación   demuestra   salida   de   bario   hacia   el   espacio   pleural  
izquierdo.  Cuál  es  el  tx  adecuado  más  aceptado:  
a.   Observación  
b.   Observación  más  antibióticos  
c.   Toracotomía  cerrada  izquierda  
d.   Intubación  esofágica  prolongada  
e.   Drenaje  y  reparación  quirúrgica  de  la  lesión  
 
Anestésicos  
305.    Una  paciente  de  70  kg  que  llega  al  cuarto  de  urgencias  con  una  herida  de  cuello  amplia  y  que  usted  va  a  
suturar,  se  debe  considerar:  
a.   La  dosis  máxima  de  xilocaína  que  debe  infiltrarse  es  350  mg  
b.   La  dosis  máxima  de  xilocaína  que  debe  infiltrarse  es  de  250  mg  
c.   La  xilocaína  viene  en  concentración  de  50  mg/cc  
d.   La  vida  media  de  la  xilocaína  es  aprox  4  horas  
 
306.    Cuantos  mg  de  lidocaína  contienen  30  cc  de  lidocaína  al  2%:  
a.   200  mg  
b.   300  mg  
c.   400  mg  
d.   500  mg  
e.   600  mg  
 
307.    La  clasificación  de  ASA  nos  orienta  sobre:  
a.   El  riesgo  de  mortalidad  operatorio  
b.   El  riesgo  de  complicaciones  peri  operatorias  
c.   El  riesgo  cardiológico  peri  operatorio  del  paciente  
d.   Ninguna  de  las  anteriores  
 
308.      La  ventaja  de  la  analgesia  controlada  por  el  paciente  es:  
a.   El  paciente  tiene  control  sobre  su  medicamento  
b.   Independencia  de  la  enfermera  
c.   Inmediata  medicación  
d.   Rápida  analgesia  
e.   Todas  las  anteriores  
 
     
309.   Durante  la  etapa  pre  anestésica  podemos  decir  que:  
a.   La  clasificación  de  ASA  correlaciona  el  estado  general  del  paciente  y  el  riesgo  anestésico  
b.   La  clasificación  de  Mallmpati  correlaciona  el  estado  general  del  paciente  y  el  riesgo  anestésico  
c.   Los  pacientes  ASA  V,  son  siempre  de  intubación  difícil  
d.   Los  pacientes  Mallampati  gradi  I,  son  siempre  de  intubación  dificil  
e.   Los  pacientes  ASA  I  corrientemente  presentan  problemas  durante  el  acto  anestésico  
 
310.    El  orden  de  aparición  clínica  de  signos  de  intoxicación  por  anetésicos  locales  es:  
a.   Adormecimiento  de  la  lengua,  confusión,  paro  cardíaco,  paro  respiratorio  
b.   Adormecimiento  de  la  lengua,  confusión,  paro  respiratorio,  paro  cardiaco  
c.   Confusión,  adormecimiento  de  la  lengua,  paro  respiratorio,  paro  cardiaco  
d.   Confusión,  adormecimiento,  paro  cardiaco,  para  respiratorio  
 
Insuficiencia…  (circulación)  
311.    La  triada  de  Virchow  consta  de:  
a.   Estasis,  edema  y  hipercoaguabilidad  
b.   Estasis,  enrojecimeinto  cutáneo  y  edema  
c.   Estasis,  hipercoaguabilidad,  lesión  vascular  
d.   Ninguna  de  las  anteriores  
 
312.   Paciente  con  riesgo  aumentado  de  trombosis  venosa  elevado,  excepto:  
a.   Encamados  
b.   Uso  de  anticonceptivos  orales  
c.   DM  
d.   Insuficiencia  cardiaca  
 
313.    Todo  es  cierto  de  los  aneurismas  abdominales,  excepto:  
a.   Los  aneurismas  mayores  de  5cm  tienen  mayor  riesgo  de  ruptura  
b.   Su  etiología  es  multifactorial  
c.   Es  una  enfermedad  de  ancianos  
d.   La  mayoría  de  los  aneurismas  son  asintomáticos  
 
   
314.   El  factor  de  mal  pronóstico  más  importante  en  el  shock  hemorrágico  por  traumatismo  es:  
a.   El  aporte  masivo  de  volumen  en  la  atención  pre  hospitalaria  
b.   El  tiempo  transcurrido  desde  el  impacto  hasta  la  cirugía  
c.   La  atención  realizada  en  el  hospital  de  referencia  
d.   La  realización  de  las  pruebas  dx  apropiadas  
e.   Existencia  de  lesiones  ortopédicas  
 
315.    Usted  tiene  una  paciente  que  tiene  dx  de  TEP,  se  le  inicia  la    
     Heparinización  con  un  bolo  de  80  unidades  y  una  infusión  de  18  
                         unidades  por  hora.  Se  toma  un  control  de  TPT  a  las  4  horas  y  es  el    
                           siguiente  TPT    P35  TPTC  35,  usted  debe:  
a.   Bolo  de  80  unidades  de  heparina  
b.   Bolo  de  40  unidades  de  heparina  
c.   Disminuir  la  infusión  y  esperar  2  horas  
d.   Bolo  de  100  u  de  heparina  
 
316.    Subitamente   el   paciente   anterior,   luego   de   haberle   administrado   5000   u   de   heparina   comienza   a  
presentar  hematemesis  y  por  los  sitios  de  punción  hay  sangrado  espontáneo,  se  realiza  un  control  de  TPT  y  es  
informado  así,  TPTP  es  >1  minuto.  Usted  debe:  
a.   Detener  la  heparina  
b.   Detener  la  heparina  +  50  mg  de  sulfato  de  protamina  
c.   Detener  heparina  +  4  u  de  plasma  fresco  congelado  +  50  mg  de  sulfato  de  protamina  
d.   Detener  la  heparina  y  trnsfundir  3  u  de  plasma  congelado  
 
   
317.   Todo  los  siguientes  son  síntomas  de  la  insuficiencia  arterial,  excepto:  
a.   Atrofia  muscular  
b.   Edema  de  miembros  inferiores  
c.   Vello  escaso  
d.   Rubor  
 
Obesidad  Morbida  
318.    El  índice  de  masa  corporal  de  un  paciente  que  pesa  180  kg  y  mide  
                               1.70  metros  es  de:  
a.   105  
b.   62  
c.   181  
d.   0.009  
e.   Ninguna  de  las  anteriores  
 
Patología  Colónica  
319.    Son  indicaciones  para  colocación  de  drenajes,  excepto:  
a.   Luego  de  una  sutura  hepática  secundaria  a  un  traumatismo  
b.   Luego  de  una  coledocotomia  para  explorar  la  vía  biliar  
c.   Luego  de  una  mastectomía  radical  modificada  
d.   Luego  de  una  laparotomía  exploradora  por  peritonitis  generalizada  sin  encontrar  el  foco  infeccioso  
 
320.   Mujer   de   70   años   cardiopata   acude   por   dolor   abdominal,   distensión   abdominal   y   obstinación,   la   rx  
muestra  distensión  del  colon,  con  poco  gas  en  intestino  delgado,  el  ciego  mide  11  cm  no  se  ve  gas  en  el  recto,  
todas  son  cierta,  excepto:  
a.   Tiene  una  obstrucción  en  asa  ciega  
b.   La  causa  más  probable  de  su  problema  es  una  neoplasia  del  sigmoides  
c.   La  perforación  colónica  es  inminente  
d.   Necesita  una  colonoscopia  urgente  
 
321.    Entre  los  cuidados  de  los  drenajes  se  debe  considerar:  
a.   Se  deben  cambiar  periódicamente  los  apósitos  
b.   Se  debe  calcular  cuanto  es  lo  drenado  periódicamente  
c.   Se  debe  reponer  en  los  líquidos  administrados  al  paciente  lo  drenado  
d.   Todas  las  anteriores  
e.   Solo  b  y  c  
 
322.   Un  hombre  de  35  años  acude  por  dolor  perianal  que  no  le  permite  sentarse,  fiebre  y  diarrea  de  4  días  de  
evolución,   al  examen   físico   la   región   perianal   es  normal,   excepto   por   un   endurecimiento   doloroso   del   lado  
derecho  que  hace  muy  difícil  el  examen,  el  manejo  debe  ser:  
a.   Baños  de  asiento  por  hemorroide  izqueirda  
b.   Antibiótico  por  su  gastroenteritis  
c.   Incisión  y  drenaje  del  abceso  perianal  
d.   Laxantes  por  su  fisura  anal  
e.   Ninguna  de  las  anteriores  
 
323.   Paciente  de  80  años  operado  por  enfermedad  diverticular  
perforada  del  colon,  se  le  realiza  hemicolectomía  izquierda  y  colostomia  terminal.  A  los  5  días  presenta  
febrícula,  aumento  de  necesidad  de  líquidos  para  mantener  diuresis,  además  hiperglicemias  y  distensión  
abdominal.  Sería  recomendable  en  este  paciente  efectuar:  
a.   Hemocultivo  
b.   USG  abdominal  
c.   CAT  de  abdomen  
d.   Centelleo  con  leucocitos  marcado  con  tecnecio  99  
   
Pie  Diabético  
 
324.   La  forma  más  efectiva  de  prevenir  la  aparición    de  ulceras  en  los  pies  de  los  pacientes  diabéticos  es:  (C  o  
D)  
a.   Arteriografía  temprana.  
b.   Medición  de  la  oximetría  del  pie  
c.   Detección  temprana  de  pérdida  de  la  sensibilidad.  
d.   Utilización  de  calzados  especiales.  
 
325.   Paciente   de   50   años   de   edad   diabético   tratado   con   hipoglucemiantes   orales,   quien   desde   hace   una  
semana  presenta  fiebre  y  aumento  de  volumen  del  pie  derecho.    Refiere  que  hace  un  mes  presentaba  ulcera  
plantar  en  pie  derecho.    Todas  las  siguientes  son  causas  intrínsecas  de  ulceración  excepto:  (a  o  d)  
a.   Neuropatía.  
b.   Ceguera  
c.   Pobre  conocimiento  de  la  diabetes.  
d.   Ulcera  previa.  
 
Perforación  Intestinal  
326.   La  causa  más  común  de  perforación  intestinal  en  pacientes  con  SIDA  es:  
a.   Linfoma  
b.   Sarcoma  de  Kaposi.  
c.   Gastroenteritis  x  CMV  
d.   Infecciones  por  Mycobacterias.  
 
Enfermedad  De  Hirchprung  
327.   Todo  lo  mencionado  para  la  Enfermedad  de  Hirchsprung  es  cierto,  excepto:  
a-­‐   Se  caracteriza  por  ausencia  de  células  ganglionares.  
b-­‐   El  segmento  más  afectado  en  el  colon  ascendente  e  íleon  terminal.  
c-­‐   El  enema  baritado  y  la  biopsia  rectal  son  fundamentales  para  el  diagnóstico.  
d-­‐   Sospechamos  diagnóstico  en  el  neonato  que  no  expulsa  meconio  en  las  primeras  24  horas  de  vida.  
 
Respuesta  Inflamatoria  Sistémica  
328.   El  síndrome  de  respuesta  inflamatoria  sistémica  se  define  como  dos  o  mas  de  los  siguientes  enunciados  
excepto:  
a.   Temperatura  mayor  de  38º.  
b.   Frecuencia  cardiaca  mayor  de  90  x  
c.   Frecuencia  respiratoria  mayor  de  20  x  
d.   Leucocitos  mayor  de  12  000.  
e.   %  de  bandas  mayor  de  4%.  
 
329.   Es  causa  de  síndrome  de  respuesta  inflamatoria  sistémica:  
f.   Pancreatitis  aguda.  
g.   Quemaduras  de  segundo  grado  30%  de  SCT.  
h.   Infección  por  CMV.  
i.   Herida  por  arma  de  fuego  en  abdomen.  
j.   Todas  las  anteriores.  
 
330.   la  citoquina  que  se  cree  que  es  la  màs  importante  mediadora  del  SRIS  SDMO  es:  
1.   TNF  
2.   IL-­‐1  
3.   IL-­‐6  
4.   IL-­‐8  
5.   Ninguna  de  las  anteriores  
 
331.   El  tratamiento  de  SDMO  para  mejorar  la  cinètica  del  oxigeno  dee  consistir  en:    
a.   mojorar  oxigenacion  
b.   optimizar  la  relacion  DO2/VO2  
c.   Corregir  la  anemia  
d.   Debridamiento  de  tejido  necròtico  
e.   Control  de  hipertermia  
f.   Todas  las  anteriores  
g.   Ninguna  de  las  anteriores  
 
 
Pareo.    
a.   Lynch                  _____        Fiebre,  dolor  con  ictericia    
b.   Ogilvie              _____        Fiebre,  dolor,  ictericia,    obnulación    
                                           e  hipotensión.        
c.   Klatskin            _____        Tumor  de  la  confluencia  de  los  conductos  hepáticos.    
d.   Reynolds        _____    Pseudo  obstrucción  colónica    
e.   Charcot              _____        Cancer  colorectal  hereditario  no  polipósico    
 
332.   Diga  5  características  del  dolor    que  siempre  deben  ser  constatadas  como  parte  del  interrogatorio  ante  un  
cuadro  de  dolor  abdominal  .  
 
a.   Inicio  
b.   Migracion  
c.   Tipo  
d.   Que  lo  mejora  
e.   Que  lo  empeora  
 
 
CICATRIZACIÓN  
 
333.   Todo  lo  siguiente  es  cierto  acerca  de    la  cicatrización  de  las  heridas  quirúrgicas  excepto:  
a-­‐   Luego  de  24  horas  no  es  necesario  cubrir  la  herida.  
b-­‐   En  la  fase  de  inflamación  hay  migración  de  leucocitos,  monocitos  y  macrófagos  hacia  la  herida.  
c-­‐   El  cierre  de  la  herida  por  segunda  intención  se  sutura  luego  de  5  días  cuando  no  hay  signos  de  
infección.  
d-­‐   La  fase  de  contracción  solo  ocurre  en  heridas  grandes.  
 
334.   La  cicatrización  por  segunda  intención  es:  
a.   cuando  el  tejido  es  icidido  y  suturado  
b.   cuando  el  tejido  es  incidido,  debridado  y  suturado.  
c.   cuando  ocurre  en  heridas  abiertas  y  hay  formación  de  tejido  de  granulacion  con  recubrimiento  final  de  la  
herida.  
d.   Cuando  la  herida  cure  abierta  durante  unos  5  dias  y  despues  se  cierra.    
e.   Ninguna  de  las  anteriores    
 
335.   la  respues  de  cicatrizacion    se  inicia:  
a.   al  momento  de  la  lesion  
b.   cuando  la  sangre  es  expuesta  a  la  colágena  
c.   cuando  se  inician  las  curaciones  
d.   a  las  24  horas  después  de  la  lesión  
 
336.   los  principales  componentes  celulares  de  la  cicatrización  son  :  
a.   granulocitos,  linfocitos  y  plaquetas.  
b.   Linfaticos  elementos  trazas  y  enzimas  
c.   Macrófagos  y  fibroblasto  
d.   Todas  las  anteriores.  
e.   Solo  a  y  c  
 
VESICULA  Y  VIAS  BILIARES  /  ICTERICIA  
 
337.      El  objetivo  inicial  de  la  terapia  de  la  colangitis  aguda  es:  
a.   Aliviar  la  ictericia  y  prevenir  el  daño  hepático  
b.   Prevenir  el  desarrollo  de  la  pancreatitis  
c.   Remover  el  cálculo  si  está  presente  
d.   Descomprimir  la  vía  biliar  
 
338.    Se  le  realiza  a  una  paciente  colecistectomía  y  exploración  de  vías  biliares,  2  cálculos  fueron  removidos  y  
se  colocó  un  tubo  en  T  en  el  colédoco,  a  los  9  días  post  operatorios,  se  le  elevan  los  niveles  de  bilirrubina  y  
tiene  dolor  en  hipocondrioderecho  irradiado  a  su  espalda.  Cuál  debe  ser  el  próximo  paso  en  el  manejo  de  esta  
paciente:  
a.   Iniciar  terapia  con  drogas  que  disuelvan  los  cálculos  
b.   Remover  el  tubo  T  
c.   Pinzar  el  tubo  en  T  
d.   Operar  a  la  paciente  
e.   Realizar  colangiografía  por  tubo  en  T  
 
339.   El  dolor  típico  de  la  colelitiasis  es  
a.   En  epigastrio  o  hipocondrio  derecho,  cólico,  menos  de  una  hora  de  duración.  
b.   En  epigastrio  o  hipocondrio  derecho,  constante,  minutos  de  duración.  
c.   En  epigastrio  o  hipocondrio  derecho,  cólico,  minutos  de  duración.  
d.   En  epigastrio  o  hipocondrio  derecho,  constante  más  de  una  hora  de  duración.  
e.   En  epigastrio  y  espalda,  constante,  menos  de  una  hora  de  duración.  
 
340.   Paciente   de   65   años   de   edad   se   presenta   por   presentar   molestias   abdominales   e   ictericia   durante   las  
últimas  tres  semanas.  Tiene  antecedentes  de  tabaquismo  positivo  por  30  años,  e  ingesta  de  alcohol  ocasional,  
y  un  melanoma   ulcerado  de  5,5mm  que  fue  retirado  de  su  espalda   hace   dos  años  y  medio.  La  exploración  
física   mostró   un   paciente   ligeramente   ictérico,   con   los   signos   vitales   normales   y   el   abdomen   ligeramente  
distendido   con   dolor   en   el   cuadrante   superior   derecho   y   hepatomegalia   importante.   El   diagnostico   más  
probable  es:  
a.   Hepatitis  A.    
b.   Metástasis  hepáticas.    
c.   Pancreatitis.    
d.   Hepatoma.  
 
341.   La  evaluación  del  paciente  anterior  debe  incluir:  
a.   Anti  A  IgM.  
b.   Amilasa  y  lipasa  serica.  
c.   Tomografía  axial  computarizada.  
d.   Alfa  feto  proteína.  
 
342.   Para  cada  una  de  las  siguientes  preguntas  escoja  una  de  las  siguientes  opciones:  
a.   Pancreatitis  aguda  
b.   Pancreatitis  crónica  
c.   Ambas  
d.   Ninguna  
 
D    Shock  
C      Dolor  abdominal  
B    Diabetes  y  esteatorrea  
B    Es  necesaria  cirugía  en  más  del  70%  de  los  casos  
C      Pseudoquistes  
 
 
343.   Una  paciente  de  80  años  acude  al  Servicio  de  Urgencias  con  fiebre  de  39°C,  Ictericia,  G.B  20,000  con  85%  
neutrófilos.  
El  manejo  inicial  de  esta  paciente  debe  incluir  todos  excepto.  
 
a.   Antibióticos  
b.   Laparoscopia  
c.   Perfil  de  coagulación  
d.   Reanimación  cardiovascular                                    
e.   Ultrasonido  
 
344.   Una  mujer  de  78  años  consulta  por  fiebre  con  escalofríos,  dolor  abdominal  e  ictericia.    Usted  diagnostica  
colangitis  aguda.    La  PA  es  de  80/70  FC  IIS  x’.    El  tratamiento  a  seguir  es.  
a.   Antibiótico  terapia  por  48  hrs.  
b.   Drenaje  biliar  urgente      
c.   Catéter  de  Swan-­‐Ganz  para  establecer  la  causa  del  shock.  
d.   Tomografía  computarizada  
e.   Tratamiento  en  Unidad  de  Cuidados  Intensivos.    
 
Para  cada  una  de  las  siguientes  preguntas  escoja  una  de  las  siguientes  opciones  
 
a.   ERCP  
b.   Colangio  Resonancia  
c.   TAC  
d.   Ninguno  
 
345.   _B_  Bilirrubinas  totales  8  mg  %  F.  Alcalina  100  (N  <  125)  
a.   US:  vías  biliares  9  mm    (N  <  6  mm),  sin  coledocolitiasis    
 
346.   _A_  Bilirrubinas  totales  8  mg%    F.  Alcalina  200  (N  <  125)    
a.   US:  vías  biliares  9  mm  muestra  coledocolitiasis  
 
347.   _C_  Bilirrubinas  totales  15  mg.%  F.  alcalina  500  (N  <  125)    
a.   US:  vías  biliares  12  mm    masa  en  cabeza  de  páncreas  y  colelitiasis  
 
348.   _D_  Bilirrubinas  totales  10  mg  %  F.  alcalina  100  (N  <  125)      
a.   US:  vías  biliares  5  mm  (  N  <  6  mm)  sin  colelitaisis  
 
349.   Con  respecto  a  la  colelitiasis  todas  son  ciertas  excepto:  
a.   60%  de  pacientes  con  colelitiasis  nunca  tendrán  síntomas  
b.   La  dispepsia  y  mareos  son  indicaciones  de  cirugía  en  pacientes  con  colelitiasis  
c.   2%  de  los  pacientes  con  colelitiasis  tendrán  como  primer  síntoma  una  complicación  
d.   Ninguna  de  las  anteriores.  
 
350.   Con  respecto  a  la  coledocolitiasis  todas  son  ciertas  excepto:  
a.   Su  incidencia  es  el  10-­‐15%  en  pacientes  con  colelitiasis  
b.   La  mayor  parte  de  los  cálculos  son  primarios  del  propio  colédoco  
c.   La  prueba  funcional  hepática  mas  sensible  es  la  fosfatosa  alcalina  
d.   Es  una  causa  frecuente  de  colangitis  
 
351.   Los  pacientes  con  colangitis  pueden  presentar  todos  los  siguientes  excepto:  
a.   Fiebre  con  escalofríos  
b.   Ictericia  
c.   Dolor  en  cuadrante  superior  derecho  
d.   Obstrucción  intestinal  
 
352.   Una   paciente   de   80   años   tiene   un   cuadro   de   colangitis   aguda   de   36   horas   de   evolución   por  
coledocolitiasis.    Un  intento    de  descomprimir    la  vía  biliar  por  CPRE  ha  fallado.    El  radiólogo  intervencionista  
está  fuera  de  la  ciudad.    Su  PA  es  80/40,    Fc  120x'.  Recibe  antibióticos  apropiados   desde  hace  36  horas.  El  
próximo  paso  a  seguir  será.  
a.   Cirugía  
b.   Cambio  de  antibióticos  
c.   Otro  intento  de  CPRE  dentro  de  24  horas  
d.   Esperar  que  regrese  el  radiólogo  intervencionista  en  24  horas  
 
353.   Describa  un  "  Cólico  biliar  "  típico  
a.   Dolor  de  Ubicación  en:      HCD  o    epigastrico  
b.   Constante  o  Espasmódico(  escriba  cual):  CONSTANTE  
c.   Duración  en  horas  mas  de  UNA  y  menos  de  24HORAS  
d.   Las  náuseas  son  frecuentes  si  ó  no:  SI  
e.   Siempre  está  asociado  a  la  ingesta  de  alimentos  si  ó  no:  PUEDE  O  NO  ESTAR  ASOCIADO  A  LA  INGESTA  DE  
ALIMENTOS  
 
354.   La  vena  porta  tiene  su  origen  de  la  confluencia  de:  
a.   La  vena  esplènica  y  la  vena  mesentérica  inferior  
b.   La  vena  mesentérica  superior  y  la  vena  esplénica  
c.   La  vena  cava  inferior  y  la  vena  esplénica  
d.   Ninguna  de  las  anteriores  
 
 
355.   Un   hombre   de   60   años,   diabético   consulta   por   historia   de   mas   o   menos   24   horas   de   evolución  
caracterizada  por  dolor  en   HCD,  fiebre  de  38.5C,  G.B.  19.0  con  95%   N.    El  US  es  compatible  con   colecistitis  
aguda  calculosa.    Todas  las  siguientes  son  ciertas  excepto:  
a.   Requiere  cirugía  en  las  próximas  24  horas  
b.   Las  probabilidades  de  que  la  vesícula  esté  perforado  son  superiores  a  lo  usual  
c.   La  colecistectomía  laparoscópica  está  contraindicada  
d.   Inicialmente  la  colecistitis  es  estéril  
e.   La  etiología  del  cuadro  es  obstructiva  
 
356.   Una  mujer  de  55  años  está  en  su  cuarto  día  de  hospitalización  por  un  cuadro  de  pancretitis  aguda  biliar.    
En  la  TAC  de  hace  24  hrs.  Se  aprecia  necorosis  pancreática  extensa  con  gas  en  el  retroperitoneo.    El  próximo  
paso  a  seguir  sería:  
e.   Cultivo  de  la  necrosis  pancreática  por  punción  percutánea  
f.   Intubación  endotraqueal  y  ventilación  mecánica  
g.   Cirugía  
h.   Continuar  el  manejo  conservador  
 
357.   Todas  las  siguientes  son  ciertas  respecto  a  la  pancreatitis  aguda  biliar  excepto:  
a.   La  colecistectomía  es  parte  fundamental  del  tratamiento  integral  
b.   La  CPRE    siempre  está  indicada  durante  el  episodio  agudo  
c.   Un  cálculo  obstruyendo  la  papila  es  el  evento  desencadenante  
d.   La  TAC  es  el  estudio  de  imagenología  de  elección  
 
358.   Ud.  es  llamada  (o)  a  las  2  am    de  la  sala  de  cirugía  porque  el  paciente  que  se  le  realizó  una  colecistectomia  
abierta    a  las  8  am  del  día  anterior  se  queja  de  mucho  dolor.  
Su  enfoque  de  Dx  y  manejo  inicial  incluye  todas  las  siguientes  excepto:  
a.   Historia  Clínica  y  Exámen  Físico  
b.   R-­‐X  de  Tórax  
c.   Hemograma  completo  
d.   Urinálisis  
 
359.   Varón  de  75  años  de  edad  es  traído  al  servicio  de  urgencias  por  su  familia  para  evaluación  por  ictericia.  El  
paciente  se  queja  de  prurito  de  2  semanas  de  evolución  y  pérdida  de  peso  reciente  de  5  Kg.  A  la  exploración  
física  se  encuentra  ictericia  intensa  y  tiene  una  masa  globosa  en  el  cuadrante  superior  derecho  del  abdomen  
que  se  mueve  con  la  respiración.  El  diagnostico  más  probable  que  tenga  este  paciente  es:  
a.   Cirrosis  hepática.  
b.   Hepatitis  viral.  
c.   Neoplasia  de  la  cabeza  de  páncreas.  
d.   Neoplasia  de  la  cola  del  páncreas  
 
360.   Todas  las  siguientes  son  ciertas  respecto  a  la  pancreatitis  aguda  biliar  excepto:  
a.   La  colecistectomia  es  parte  fundamental  del  tratamiento  integral  
b.   La  CPRE    siempre  está  indicada  durante  el  episodio  agudo  
c.   Un  cálculo  obstruyendo  la  papila  es  el  evento  desencadenante  
d.   La  TAC  es  el  estudio  de  imagenología  de  elección  
 
361.   Son  funciones  de  la  colecistoquinina  excepto:  
a.   Retrasa  el  vaciamiento  gástrico.  
b.   Estimula  la  contracción  de  la  vesícula  biliar.  
c.   Relaja  el  esfínter  anal  promoviendo  la  defecación..  
d.   Estimula  la  secreción  pancreática.  
e.   Juega  un  papel  importante  en  la  regulación  de  la  saciedad  
 
362.   Un  hombre  de  59  años  consulta  por  ictericia  y  prurito.  El  US  muestra  las  vías  biliares  en  9  mm  (Normal  <  
de  6  mm)  y  colelitiasis.    Todos  los  siguientes  serían  adecuados  en  el  manejo  inicial  de  este  paciente  excepto.  
 
a.   Colangio  resonancia  
b.   ERCP  
c.   Laparotomía  
d.   Perfil  de  coagulación  
e.   TAC  de  Abdomen.  
   
363.   En  el  manejo  inicial  de  la  pancreatitis  severa  están  indicados  todos,    
   excepto:  
a.  Antibióticos  de  ampli  espectro  
b.  TAC  en  las  primeras  48  horas  
c.  Atención  en  la  unidad  de  cuidados  intensivos  
d.  Laparotomía  por  necrosis  peri-­‐pancreáticas  
e.  Monitoreo  hemodinámica  invasivo  
 
Piel  
 
364.   Indicaciones  para  la  excisión  de  un  nevo  pigmentado  incluyen:  
a.  Cambio  en  el  color,  tamaño  y  forma  
b.  Desarrollo  de  dolor  de  la  lesión  
c.  La  aparición  de  lesiones  satélites  en  el  aérea  de  un  nevo    previamente  existente  
d.  Adenopatía  regional  inexplicable  
e.  Todas  las  anteriores  
 
365.   Todos  los  siguientes  enunciados  con  respecto  a  las  quemaduras  electricas  son  ciertas  excepto:    
a.  Ellas  son  usualmente  mas  profundas  y  mas  severas  que  lo  que  aparentan  
b.  Los  vasos  sanguíneos  los  nervios  y  los  huesos  tienen  una  resistencia  baja  a  la  corriente  eléctrica  
c.  La  necrosis  muscular  puede  estar  sobreestimada  
d.  Todos  los  pacientes  con  lesiones  eléctricas  necesitan  monitoreo  continuo  
e.  El  manitol  puede  ser  utilizado  para  mantener  una  diuresis  adecuada  para  prevenir  el  daño  renal  secundaria  a  la  
mioglobinuria      
 
Pareo  
a.   Bupivacaína  
b.   Lidocaína  
c.   Ambas  
d.   Ninguna  
 B      Dosis  máxima  de  infiltración  300  mg  
 A    Dosis  máxima  de  infiltración  175  mg  
   B    Efecto  dura  hasta  dos  horas  
 A    Efecto  dura  hasta  ocho  horas  
 B    La  dosis  máxima  de  infiltración  se  puede  incrementar  si  se  usa  adrenalina  
 
a.   Secretina  
b.   Colecistoquinina  
c.   Ambas  
d.   Ninguna  
___Secreción  de  gránulos  de  Zimógeno  
___Secretada  en  el  duodeno  
___Secreción  de  agua  y  electrolitos  
___Secreción  disminuida  por  la  administración  de  inhibidores  de  la  bomba  de  protones  
___Secreción  disminuida  por  resección  pancreático  duodenal  
 
a.   Carcinoma  lobular  in  situ  
b.   Carcinoma  ductal  in  situ  
c.   Ambos  
d.   Ninguno  
 B          Rara  vez  es  palpable  
 A        Nunca  es  palpable  
 A        Es  un  factor  de  riesgo  
 C        Puede  avanzar  a  carcinoma  invasor  
 B        Lumpectomía    sin  vaciamiento  axilar  
 A        Observación  
 A        Mastectomía  profiláctica  
D            Quimioterapia  pre-­‐operatoria  
 B            quimioterapia  post-­‐operatoria  
 B            Microcalcificaciones  
 

Para  Futuras  Generaciones  


24  de  Marzo  de  2008  
ABDOMEN AGUDO. PATOLOGIA DEL PERITONEO. OBSTRUCCION
INTESTINAL. (respuestas)

1.- En la etiología de un cuadro de “abdomen agudo” NO estaría uno de los siguientes


procesos:
*a.- hemorragia digestiva por ulcera duodenal
b.- apendicitis aguda
c.- obstrucción intestinal por cáncer de sigma
d.- diverticulitis de sigma perforada
e.- colecistitis aguda

2.- Respecto a las afirmaciones siguientes sobre el término de “abdomen agudo”


señalar lo correcto:
a.- un paciente con abdomen agudo siempre tendrá alteraciones, por exceso o defecto,
del transito intestinal *(si) (no)
b.- un paciente con un abdomen agudo siempre debe ser tratado con criterio de
urgencia
*(si) (no)
c.- pueden existir pacientes con cuadro clínico de abdomen agudo que no tengan dolor
abdominal (si) *(no)
d.- existen patologías extraabdominales que producen un abdomen agudo *(si) (no)
e.- todos los procesos etiológicos que producen un abdomen agudo acaban con el
tiempo afectando a la serosa peritoneal *(si) (no)

3.- Respecto al dolor del abdomen agudo señalar lo correcto:


a.- el signo de Blumberg determina una situación de irritación peritoneal *(si) (no)
b.- el dolor cólico con origen en la vesícula biliar tiene una irradiación hacia
mesogastrio (si) *(no)
c.- el dolor de origen pancreático tiene una irradiación en cinturón *(si) (no)
d.- la distensión de las asas de yeyuno produce un dolor de tipo visceral *(si) (no)
e.- la distensión de la cápsula de Glisson por un hematoma hepático es indolora
(si) *(no)

4.- Un absceso del fondo de saco de Douglas se puede valorar en la exploración clínica
con:
a.- la maniobra de Murphy
*b.- un tacto rectal
c.- la maniobra de Blumberg
d.- el signo de Rovsing
e.- la maniobra de rebote

5.- ¿Los siguientes órganos tienen capa serosa visceral?:


a.- el apéndice *(si) (no)
b.- el recto (si) *(no)
c.- el primer asa yeyunal a nivel del ángulo de Treitz *(si) (no)
d.- la cara posterior de la segunda porción duodenal (si) *(no)
e.- el esófago torácico (si) *(no)

6.- Respecto a la fisiología del peritoneo señalar las opciones correctas:


a.- el peritoneo puede absorber hasta 35 c.c. por hora por gradiente osmotico *(si) (no)
b.- la función plástica (creación de adherencias) depende de los fibroblastos del
peritoneo (si) *(no)
c.- los estímulos dolorosos del peritoneo parietal producen sobre todo contractura de los
músculos del abdomen *(si) (no)
d.- la distensión o tracción del peritoneo visceral puede producir bradicardia *(si) (no)
e.- la absorción de bacterias por el peritoneo se hace por vía linfática especialmente
*(si) (no)

7.- El espacio peritoneal más declive es:


a.- la fosa de Morrisson
b.- el espacio parietocólico derecho
*c.- el fondo de saco de Douglas
d.- el espacio subfrénico izquierdo
e.- la trascavidad de los epiplones

8.- ¿En las peritonitis establecidas se producen los siguientes fenómenos?:


a.- ileo adinamico *(si) (no)
b.- aumento de la absorción de líquidos (si) *(no)
c.- hipovolemia por creación de tercer espacio *(si) (no)
d.- diapedesis de polimorfonucleares *(si) (no)
e.- exudado intraperitoneal rico en proteínas y fibrina *(si) (no)

9.- Los fenómenos generales de respuesta a una peritonitis pueden incluir los siguientes
EXCEPTO:
a.- retención agua por activación de los sistemas aldosterona+ADH
*b.- alcalosis metabólica
c.- hipoventilación
d.- taquipnea
e.- hipopotasemia

10.- Entre los signos clínicos de las peritonitis encontraremos:


a.- maniobra de rebote positiva *(si) (no)
b.- tacto rectal doloroso al palpar el fondo de saco de Douglas *(si) (no)
c.- en la percusión matidez al principio y timpanismo en fase establecida (si) *(no)
d.- hiperperistaltismo en la auscultación (si) *(no)
e.- paciente generalmente inmóvil *(si) (no)

11.- Cual de las siguientes situaciones NO debe razonablemente plantear un


diagnóstico diferencial con una peritonitis:
a.- herpes zoster de la pared abdominal
b.- porfiria
*c.- hemorragia digestiva por úlcera de la segunda porción duodenal
d.- un síndrome de abstinencia a heroína
e.- una obstrucción intestinal

12.- Respecto a las peritonitis primarias señalar las opciones correctas:


a.- la contaminación peritoneal se produce por vía hemática desde un foco
extraabdominal *(si) (no)
b.- son más frecuentes en niños *(si) (no)
c.- son siempre polimicrobianas (si) *(no)
d.- el germen más frecuente es el bacteroides fragilis (si) *(no)
e.- no deben precisar tratamiento quirúrgico si el tratamiento antibiótico está bien
dirigido *(si) (no)

13.- En la bacteriología de las peritonitis agudas secundarias el germen más frecuente de


los siguientes es:
a.- estreptococo fecal
b.- clostridio
c.- corynebacteria
*d.- bacteroides fragilis
e.- estreptococo B hemofilico

14.- Un paciente ha sido intervenido por una peritonitis difusa secundaria a una
perforación de un asa yeyunal por una espina de pescado. Se retiraron los drenajes a los
siete días. A los quince días de la intervención el paciente tolera bien la alimentación
oral sin dolor abdominal pero persiste un cuadro febril en picos y tiene la siguiente
radiografía de tórax

a.- se debe sospechar una neumonía basal derecha y tratarse con antibióticos (si) *(no)
b.- se debe realizar un TAC para descartar un absceso intraabdominal *(si) (no)
c.- se debe intervenir al paciente sin más exploraciones porque la fiebre en picos
significa que tiene una peritonitis difusa recidivada (si) *(no)
d.- se puede sospechar con esta radiografía un absceso subfrenico derecho *(si) (no)
e.- se debe mantener una actitud de observación sin tratamiento durante al menos siete
días más porque puede ser una evolución normal postoperatoria (si) *(no)

15.- Un paciente ha sido intervenido por una peritonitis difusa secundaria a una
perforación de un asa yeyunal por una espina de pescado. Se retiraron los drenajes a los
siete días. A los quince días de la intervención el paciente tiene el abdomen blando y
tolera bien la alimentación oral, pero aparece un cuadro febril en picos y se realiza un
TAC que muestra la imagen siguiente:
a.- se trata de un absceso subfrénico que debe ser drenado percutaneamente con control
radiológico o ecográfico *(si) (no)
b.- se debe tratar con amoxicilina y clavulanico siete días y después repetir el TAC
(si) *(no)
c.- se dede hacer una punción para tomar un cultivo y tratar con el antibiótico adecuado
durante siete días (si) *(no)
d.- lo más probable es que la colección se reabsorba espontáneamente con tratamiento
exclusivamente antiinflamatorio (si) *(no)
e.- por tener aire en la parte superior de la imagen (ver flecha superior) se debe
descartar el absceso y valorar otro diagnóstico (angioma, metástasis etc.) (si) *(no)

16.- En una obstrucción mecánica de intestino delgado lo siguiente es cierto EXCEPTO:


a.- se pueden emitir gases y heces en las primeras horas
*b.- el comienzo del dolor es debido a la distensión de las asas
c.- no deben existir signos de irritación peritoneal en las primeras horas
d.- el dolor inicial es de características cólicas
e.- la auscultación es de ruidos hidroaereos progresivos al comienzo.

17.- La causa más frecuente de obstrucción intestinal en una paciente operada es:
*a.- adherencias
b.- cáncer de colon sigmoideo
c.- cáncer de colon ascendente
d.- hernia inguinal
e.- hernia crural

18.- Un vólvulo de intestino delgado da lugar a una situación de:


a.- Íleo paralítico
b.- Oclusión mecánica simple
c.- Síndrome de asa ciega
d.- Síndrome de intestino corto
*e.- Oclusión mecánica estrangulada

19.- Respecto a las obstrucciones intestinales funcionales (ileo paralítico) señalar la


opción correcta:
a.- son obstrucciones intestinales con luz permeable *(si) (no)
b..- después de un laparotomía se produce siempre un cierto nivel de ileo paralítico
*(si) (no)
c.- un ileo paralítico reflejo por un cólico ureteral comenzará siempre a nivel yeyunal
con “hiperperistaltismo de lucha”. (si) *(no)
d.- el síndrome de Ogilvie es un ileo paralítico localizado en el intestino grueso *(si)
(no)
e.- una invaginación intestinal es una causa de ileo paralítico (si) *(no)

20.- Desde un punto de vista clínico en una obstrucción intestinal encontraremos:


a.- los vómitos suelen preceder al dolor cólico intestinal (si) *(no)
b.- en la exploración abdominal el meteorismo y el signo de Blumberg siempre
coinciden (si) *(no)
c.- el tacto rectal siempre es doloroso al palpar el fondo de saco de Douglas (si) *(no)
d.- primero escucharemos borborigmos y ruidos metálicos y posteriormente silencio
abdominal *(si) (no)
e.- la irritación peritoneal debe hacer sospechar una situación de obstrucción
estrangulada *(si) (no)

21..- En una obstrucción intestinal de 12 horas de evolución encontraremos:


a.- hiperpotasemia (si) *(no)
b.- hiponatremia *(si) (no)
c.- hipocloremia *(si) (no)
d.- oliguria *(si) (no)
e.- bradicardia (si) *(no)

22.- Una sospecha de obstrucción intestinal de intestino delgado se valorará en un


primer momento con :
a.- Un transito intestinal
b.- Una radiografía simple en decúbito prono
*c.- Una radiografía simple en bipedestación
d.- Una ecografía
e.- Un Enema Opaco

23.- En la zona proximal a la obstrucción, la dilatación del intestino delgado tendrá


como consecuencias EXCEPTO:
a.- acumulo de gases como consecuencia de fenómenos de putrefacción
b.- disminución de la absorción en los tramos dilatados
c.- edema de la pared intestinal
*d.- aumento del peristaltismo de lucha en las zonas dilatadas
e.- proliferación bacteriana

24.- En estudio radiológico simple la postura del paciente MENOS EFICAZ para
valorar una obstrucción intestinal de intestino delgado será:
a.- bipedestación con rayo horizontal
b..- decúbito lateral derecho con rayo horizontal
c.- decúbito lateral izquierdo con rayo horizontal
*d.- decúbito supino con rayo vertical
e.- decúbito supino con rayo horizontal

25.- Señalar la opción correcta (puede haber más de una), para cada una de las
situaciones siguientes en un paciente con diagnóstico de obstrucción intestinal:
ileo mecánico simple (A)
obstrucción intestinal con estrangulación (B)
ileo paralítico (C)
a.- dolor cólico moderado con poca distensión y signo de Blumberg negativo *(A) (B)
(C)
b.- dolor cólico de comienzo brusco con Blumberg positivo (A) *(B) (C)
c.- distensión abdominal difusa sin zonas de dolor focalizado (A) (B) *(C)
d.- radiología simple con predominio de la distensión y menos niveles hidroaereos (A)
(B) *(C)
e.- ruidos de hiperperistaltismo en la auscultación abdominal en las primeras horas *(A)
*(B) (C)

26.- En una obstrucción intestinal a nivel de la unión recto-sigmoidea la zona que más
se dilatará será:
a.- el recto
b.- el sigma
*c.- el ciego
d.- el ileon terminal
e.- el yeyuno

27.- Para confirmar el diagnóstico de presunción de una obstrucción intestinal a nivel de


colon descendente podremos indicar una de las siguientes exploraciones EXCEPTO:
a.- radiología simple de abdomen
b.- enema opaco
c.- TAC abdominal
*d.- transito intestinal con bario
e.- colonoscopia
Respuestas ANESTESIOLOGIA Y REANIMACIÓN.

1.- Respecto al concepto de sedación lo siguiente es cierto EXCEPTO:


a.- es un estado previo a la hipnosis y a la anestesia
b.- el paciente sedado se encuentra en estado de indiferencia psicomotora
c.- el paciente se encuentra reactivo y puede ser despertado con facilidad
* d.- cuando un paciente está sedado está en estado de analgesia
e.- durante la sedación existe una abolición del componente psíquico del dolor

2.- En las siguientes sentencias sobre los procedimientos anestésicos, señalar si la


sentencia es verdadera (V) o falsa (F):
a.- el halotano es un anestésico intravenoso (V) *(F)
b.- la anestesia raquídea es la que se realiza mediante inyección de un anestésico local
en el espacio subaracnoideo de la medula espinal *(V) (F)
c.- la anestesia intravenosa total precisa siempre la utilización de un agente anestésico
hipnótico, un analgésico (generalmente opiáceo) y un bloqueante neuromuscular.
*(V) (F)
d.- la anestesia regional intravenosa periférica está especialmente indicada para la
cirugía de la mama (V) *(F)
e.- la anestesia epidural se realiza inyectando un anestésico local en el espacio
extradural debajo del ligamento amarillo. *(V) (F)

3.- Cual de las siguientes situaciones de entre las que se consiguen durante una
anestesia general es FALSA:
a.- amnesia anterógrada
b.- pérdida de conciencia
*c.- mantenimiento de los estímulos nociceptivos
d.- reducción de la actividad refleja
e.- inhibición del sistema simpático

4.- Respecto a los fármacos empleados en anestesia general señalar si las afirmaciones
siguientes son verdaderas (V) o falsas (F):
a.- los sedantes ansiolíticos tipo benzodiacepinas se pueden utilizar en la fase de
preinducción anestésica *(V) (F)
b.- la intubación endotraqueal se realiza siempre antes de administrar los fármacos
hipnóticos y relajantes musculares (V) *(F)
c.- los opiáceos tipo fentanilo se utilizan durante la fase de mantenimiento anestésico
*(V) (F)
d.- la neostigmina y naloxona son fármacos reversores que se emplean en la fase de
educción *(si) (no)
e.- el mantenimiento hipnótico se puede realizar tanto con fármacos inhalatorios como
intravenosos *(V) (F)

5.- La clasificación de la Sociedad Americana de Anestesiólogos (ASA) para valorar el


riesgo quirúrgico se basa en:
a.- criterios analíticos
b.- criterios radiológicos
*c.- situación general del paciente y enfermedades asociadas
d.- situación cardiorrespiratoria del paciente
e.- conjunto de parámetros analíticos y radiológicos exclusivamente
6.- Respecto a la monitorización anestésica señalar si la sentencia es verdadera (V) o
falsa (F):
a.- la pulsioximetría es un método espectrofotométrico que permite medir la saturación
de oxigeno de forma NO invasiva *(V) (F)
b.- el capnógrafo mide los niveles de dióxido de carbono y es el mejor método no
invasivo de monitorización de la ventilación *(V) (F)
c.- la indicación de monitorizar la diuresis depende, entre otros factores, de la duración
de la intervención *(V) (F)
d.- el catéter de Swan-Ganz se coloca en la arteria pulmonar *(V) (F)
e.- los catéteres venosos centrales tienen muy poca utilidad en la monitorización
anestésica (V) *(F)

7.- Una de las siguientes afirmaciones sobre la ventilación mecánica (VM) y la


intubación es FALSA:
*a.- la VM es un sistema de intercambio de gases a nivel de la membrana alveolo-
capilar.
b.- la VM controlada por volumen es la que más se utiliza, con tubo endotraqueal,
durante la anestesia general.
c.- Debido a la diferente distensibilidad de los alvéolos según su nivel de perfusión, la
VM tiende a hipoventilar las zonas pulmonares más perfundidas y a hiperventilar las
zonas pulmonares de espacio muerto.
d.- la intubación endotraqueal es el método más seguro para mantener la permeabilidad
y control de la vía aérea
e.- las cánulas de Guedel son las cánulas orofaríngeas, que se emplean para evitar que la
base de la lengua obstruya la vía aérea.

8.- Señalar el puesto correcto para cada una de las actuaciones siguientes en la
secuencia de la inducción anestesica:
a.- intubación 1 2 3 4 *5
b.- acceso venoso 1 *2 3 4 5
c.- fármacos (opiáceos, hipnóticos, relajantes) 1 2 3 *4 5
d.- preoxigenación 1 2 *3 4 5
e.- monitorización *1 2 3 4 5

9.- Los inhibidores de la colinesterasa se emplean:


a.- en la fase de inducción anestésica
b.- como droga vasoactiva en la fase de mantenimiento
*c.- en la fase de despertar para revertir el efecto de los relajantes.
d.- como droga para contrarrestar a los opiáceos
e.- como analgésico en el postoperatorio inmediato
10.- Con respecto a la anestesia general señalar si las siguientes afirmaciones son
verdaderas (V) o falsas (F)
a.- La permeabilidad de la vía aérea es el factor de mayor importancia en el curso de una
anestesia general. *(V) (F)
b.- Durante el estado de inconsciencia, el paciente depende por completo del
anestesiólogo y sus instrumentos. *(V) (F)
c.- Las técnicas de mantenimiento anestésico que se pueden emplear son: la inhalatoria
pura, la balanceada (bloqueantes neuromusculares, opiáceo, gases y/o vapores) o
anestesia intravenosa total (agente hipnótico intravenoso, opiáceo y bloqueante
neuromuscular). *(V) (F)
d.- Los objetivos durante la fase de mantenimiento de la anestesia general son: el
mantenimiento de un adecuado plano anestésico acorde con el estímulo quirúrgico, el
mantenimiento de una adecuada perfusión tisular y el mantenimiento de un buen
transporte de oxígeno. *(V) (F)
e.- Después de una anestesia general las complicaciones que podemos encontrar son:
Hipotensión arterial, hipertensión arterial y arritmias cardíacas. *(V) (F)

11.- Con respecto a la anestesia regional es CIERTO que:


a.- En la anestesia epidural el anestésico se deposita en el espacio comprendido entre el
ligamento amarillo y la duramadre. *(V) (F)
b.- En la anestesia raquídea el anestésico se deposita en el espacio subaracnoideo
. *(V) (F)
c.- En la anestesia de plexos nerviosos el anestésico se inyecta en las vainas de los
plexos *(V) (F)
d.- El frío con cloruro de etilo esta indicado especialmente en la anestesia de plexo
nervioso cervical (V) *(F)
e.- La lidocaína se puede utilizar como anestésico regional intravenosos *(V) (F)

12.- Con respecto al dolor postoperatorio señalar lo correcto:


a.- Forma parte del sufrimiento que debe pagar todo paciente que es operado, por lo que
no hay que tratarlo.(si) *(no)
b.- El dolor postoperatorio puede causar trastornos sistémicos importantes como:
hipoxia, inmunodepresión, espasmos musculares y depresión psiquica.
*(si) (no)
c.- Conviene que todo paciente tenga un poco de dolor a la hora de pautar un
tratamiento analgésico para evitar los efectos secundarios de los fármacos.(si) *(no)
d.- Siempre hay que prevenir y tratar el dolor postoperatorio considerando como
objetivo que el paciente no sufra ningún tipo de dolor o disconfort desde el momento en
que se despierta de la anestesia. *(si) (no)
e.- Un cuadro de taquicardia en el postoperatorio inmediato puede ser una respuesta al
dolor *(si) (no)

13.- Respecto a los estadios de la anestesia general señalar lo correcto:


a.- El estadio I, o de analgesia, es el mejor plano anestésico para realizar cualquier
intervención. (si) *(no)
b.- El estadio IV, o de intoxicación, es un plano previo al quirúrgico por el que todo
paciente tiene que pasar cuando es anestesiado. (si) *(no)
c.- El estadio III, o quirúrgico, puede dividirse en cuatro subestadios, siendo el III4 el
plano anestésico más profundo.*(si) (no)
d.- En el estadio III4 se produce una parálisis incipiente del tronco encefálico, hay
abolición del reflejo tusígeno y es el plano adecuado para la intubación endotraqueal y
la realización de todas las intervenciones de la cavidad abdominal.*(si) (no)

14.- Indicar cuál de los siguientes conceptos es FALSO:


a.- La anestesia es la ausencia de sensibilidad.
b.- La analgesia es la ausencia de dolor en respuesta a estímulos que normalmente lo
provocarían.
c.- La anestesia neuroaxial es un subtipo de la anestesia regional que incluye la
anestesia epidural y la subaracnoidea o raquídea.
*d.- Todo paciente inconsciente es incapaz de responder a estímulos nociceptivos, por
lo que está por definición analgesiado.
e.- El dolor es la percepción consciente de un estímulo nociceptivo, por lo que un
paciente inconsciente no se puede decir que sufra dolor.

15.- En relación con los conceptos generales de la anestesia señalar las afirmaciones
correctas
a.- La hipnosis es un estado de sueño en el que se pierde la capacidad de despertar frente
a estímulos externos. *(si) (no)
b.- La sedación se debe conseguir simultáneamente a la anestesia (si) *(no)
c.- Anestesiar es crear una situación en la que el individuo no responde a estímulos
nociceptivos *(si) (no)
d.- Los componentes corticales que debe conseguir la anestesia son la inconsciencia y la
amnesia. *(si) (no)
e.- Uno de los objetivos primordiales de la anestesia general es mantener la actividad
refleja (si) *(no)

16.- Durante la fase de mantenimiento de la anestesia general se pueden utilizar los


fármacos siguientes EXCEPTO:
a.- opiáceos
b.- bloqueantes neuromusculares
c.- hipnóticos intravenosos
d.- hipnóticos inhalatorios
*e.- agentes reversores como la neostigmina

17.- En la anestesia general se pueden utilizar fármacos con los siguientes efectos:
a.- anestésicos *(si) (no)
b.- hipnóticos *(si) (no)
c.- sedantes *(si) (no)
d.- amnésicos *(si) (no)
e.- analgésicos *(si) (no)

18.- Respecto a la pulsioximetría señalar las afirmaciones correctas:


a.- es un método de monitorización invasivo (si) *(no)
b.- mide la saturación de oxigeno arterial capilar *(si) (no)
c.- monitoriza la frecuencia cardiaca *(si) (no)
d.- es un método espectrofotométrico *(si) (no)
e.- la saturación capilar de oxigeno normal es del 50% (si) *(no)
19.- Pueden ser consecuencias del dolor postoperatorio EXCEPTO:
*a.- bradicardia
b.- hipertensión
c.- retención urinaria
d.- ileo paralítico
e.- atelectasias pulmonares

20.- En la fase de educción o despertar anestésico una de las siguientes afirmaciones es


FALSA:
a.- se debe procurar que perdure la analgesia
b.- los inhibidores de la colinesterasa sirven para recuperar la placa motora
*c.- se debe mantener lo más posible la situación de hipnosis
d.- cuando el paciente recupera la consciencia puede ser extubado
e.- en esta fase se recupera la actividad refleja
RESPUESTAS - CIRUGÍA ONCOLOGICA. MELANOMA.

1.- En 1935 se realizó la primera “duodenopancreatectomía cefálica” por el siguiente


autor:
a.- Miles
b.- Halsted
*c.- Whipple
d.- Billroth
e.- McBurney

2.- De los siguientes tumores malignos cual es el que produce en España, en los
hombres, la mayor mortalidad:
a.- estómago
*b.- pulmón
c.- colon y recto
d.- páncreas
e.- hígado

3.- El concepto de “cirugía profiláctica” puede aplicarse a las siguientes patologías


EXCEPTO:
a.- poliposis de colon
*b.- diverticulosis de colon
c.- colitis ulcerosa
d.- cáncer medular de tiroides familiar
e.- criptorquidia

4.- En la clasificación TNM la designación T0 (cero) significa:


a.- tumor “in situ”
*b.- tumor de localización desconocida
c.- tumor menor de 1 cm. de tamaño
d.- tumor sin metástasis ganglionares en el primer escalón
e.- tumor polipoideo

5.- Señalar lo correcto respecto a los siguientes objetivos de la determinación del


Antígeno Carcino Embrionario (CEA):
a.- es una prueba que se utiliza en el diagnóstico precoz del cáncer de colon (si) *(no)
b.- sirve en determinaciones seriadas para indicar recurrencia postoperatoria del cáncer
de colon *(si) (no)
c.- indica la sensibilidad de un tumor colo-rectal a la quimioterapia (si) *(no)
d.- sirve para valorar la resección completa de un tumor colo-rectal *(si) (no)
e.- tiene también valor diagnóstico en el cáncer de vías biliares (si) *(no)

6.- Señalar en cuales de estos tumores es probable que ocurra propagación linfática:
a.- carcinoma de próstata *(si) (no)
b.- astrocitoma (si) *(no)
c.- melanoma maligno *(si) (no)
d.- lipoma localizado en la espalda (si) *(no)
e.- carcinoma de células basales de la piel de la cara (si) *(no)
7.- Respecto a la estadificación (o estadiaje) de los tumores malignos señalar los
conceptos correctos:
a.- la biopsia del ganglio centinela tiene como objetivo estadiar un tumor evitando en lo
posible grandes linfadenectomías *(si) (no)
b.- la determinación del CEA es indispensable para el estadiaje correcto de un cáncer de
colon (si) *(no)
c.- la clasificación TNM definitiva de un tumor gástrico debe establecerse después de
haber intervenido al paciente *(si) (no)
d.- la clasificación TNM valora el tipo histológico del tumor primitivo (si) *(no)
e.- el estadiaje solamente tiene un valor diagnóstico (si) *(no)

8.- Se sabe que los siguientes estados presentan un patrón familiar relacionado a una
predisposición a padecer un cáncer EXCEPTO:
a.- poliposis de colon
b.- cáncer de mama
c.- síndrome de Gardner
*d.- síndrome de Peutz-Jeghers
e.- retinoblastoma

9.- Cual de los siguientes tumores se considera más radiosensible:


*a.- carcinoma de mama
b.- adenocarcinoma gástrico
c.- melanoma maligno
d.- seminoma
e.- liposarcoma

10.- Las siguientes situaciones precisan tratamiento urgente quirúrgico o radioterápico;


señalar la opción correcta:
a.- obstrucción de la vía respiratoria por un linfoma de tiroides (cirugía) *(radioterapia)
b.- obstrucción intestinal por un carcinoma de colon sigmoide *(cirugía) (radioterapia)
c.- compresión de la medula espinal por una metástasis de cáncer de próstata
(cirugía) *(radioterapia)
d.- obstrucción de la vena cava superior por un cáncer de pulmón
(cirugía) *(radioterapia)
e.- dolor vertebral por carcinoma de mama metastático (cirugía) *(radioterapia)

11.- Ante una lesión cutánea pigmentada conocida, que se hace pruriginosa, ¿cual sería
la actitud correcta?:
a.- Tratamiento tópico hasta que desaparezca el prurito, y evitar la luz solar
b.- Revisión anual, ya que es una posibilidad normal de las lesiones pigmentadas
c.- Extirpación y biopsia selectiva del ganglio centinela
*d.- Biopsia escisional
12.- De los referidos a continuación, ¿ cual el factor pronóstico mas importante del
melanoma ?
a.- espesor de Breslow
*b.- presencia de metástasis ganglionares
c.- ulceración y sangrado de una lesión pigmentada preexistente
d.- localización en el tronco

13.- Cual es el tratamiento de elección de un melanoma > de 1 mm, sin evidencia


clínica de afectación ganglionar:
a- biopsia escisional, mas interferón
b- extirpación amplia y linfadenectomía regional
c- extirpación amplia y radioterapia loco-regional
*d- extirpación amplia y biopsia selectiva del ganglio centinela

14.- Relacionar los conceptos de tratamiento oncológico siguientes con las situaciones
clínicas descritas:
paliativo (A)
coadyuvante (B)
neoadyuvante (C)
citorreductor (D)
curativo (E)

a.- resección quirúrgica de una metástasis hepática única de un cáncer de colon


intervenido hace tres años (A) (B) (C) (D) *(E)
b.- tratamiento quimio y radioterápico preoperatorios de un cáncer de recto T3N1M0
(A) (B) *(C) (D) (E)
c.- realización de una anastomosis yeyuno-cólica latero-lateral en un cáncer de colon
transverso irresecable *(A) (B) (C) (D) (E)
d.- resección del epiplon mayor en una carcinomatosis peritoneal por cáncer de ovario
(A) (B) (C) *(D) (E)
e.- radioterapia post-quirúrgica en un cáncer de mama T2N1M0 (A) *(B) (C) (D) (E)

15.- El término recurrencia se puede aplicar a las situaciones siguientes:


a.- aparición de enfermedad después de un periodo de remisión *(si) (no)
b.- aparición de tejido tumoral en el mismo sitio donde se extirpó un tumor (si) *(no)
c.- aparición de metástasis hepáticas de un cáncer de colon irresecable tratado de forma
paliativa con quimioterapia (si) *(no)
d.- aparición de una metástasis hepática un año después de resecar un cáncer de colon
T1N0M0 *(si) (no)
e.- aparición de ascitis en un cáncer de ovario con carcinomatosis peritoneal (si) *(no)
Equilibrio hidro-electrolítico en los pacientes quirúrgicos. Respuestas.

1.- Cual de los siguientes valores normales en plasma es anormal:


a.- sodio 135-145 mEq/l
*b.- potasio 6-8 mEq/l
c.- cloruros 100-106 mEq/l
d.- proteínas totales 7 grs./100 ml.
e.- calcio 8.5-10.5 mgrs./100 ml.

2.- Cuales de las siguientes causas pueden producir hiponatremia:


a.- pérdidas de liquido gastrointestinal por una fístula digestiva *(si) (no)
b.- insuficiencia renal severa *(si) (no)
c.- cirrosis hepática *(si) (no)
d.- administración excesiva de suero glucosado al 5% *(si) (no)
e.- insuficiencia suprarrenal *(si) (no)

3.- Sobre las alteraciones del Potasio en el paciente quirúrgico señalar las afirmaciones
válidas:
a.- la acidosis metabólica facilita la entrada de K en las células produciendo
hipopotasemia (si) *(no)
b.- las pérdidas de líquidos de origen digestivo (diarrea fístulas) producen
hipopotasemia *(si) (no)
c.- la insuficiencia renal cursa con hiperpotasemia *(si) (no)
d.- las alteraciones de las ondas T y del intervalo PR del ECG son características de la
hiperpotasemia *(si) (no)
e.- la insulina favorece la entrada de K en las células *(si) (no)

4.- Señalar a que trastorno ácido-básico corresponde cada uno de los patrones
siguientes:
acidosis metabólica (A) alcalosis metabólica (B) acidosis respiratoria (C)
alcalosis respiratoria (D)
a.- pH↑ y aumento de CO3 H‾ (A) *(B) (C) (D)
b.- pH↑ y disminución de PCO2 (A) (B) (C) *(D)
c.- pH↓ y aumento de PCO2 (A) (B) *(C) (D)
d.- pH↓ y disminución de CO3 H‾ * (A) (B) (C) (D)

5.- Son causas de acidosis respiratoria EXCEPTO:


a.- alteraciones de la relación perfusión-ventilación
*b.- situaciones de shock hipovolémico
c.- derrame pleural bilateral
d.- neumotórax traumático
e.- broncoespasmo

6.- Sobre la acidosis respiratoria cual de las siguientes afirmaciones es INCORRECTA :


a.- un signo clínico es la somnolencia
b.- la PCO2 se eleva por encima de 43 mm de Hg.
*c.- en el tratamiento se debe administrar siempre bicarbonato
d.- el K suele estar elevado
e.- es indispensable para corregirla ventilar correctamente al paciente.
7.- Pueden ser causa de acidosis metabólica las siguientes situaciones EXCEPTO:
a.- shock séptico
b.- shock cardiogénico
c.- insuficiencia renal severa
*d.- cuadro de vómitos por estenosis pilórica
e.- pérdidas bilio-pancreáticas por fístula digestiva a nivel de yeyuno

8.- En la alcalosis respiratoria una de las siguientes afirmaciones es FALSA:


a.- se manifiesta por taquipnea
b.- al disminuir el Ca iónico puede aparecer tetania
c.- el CO3H‾ puede estar normal o ligeramente aumentado dependiendo del grado de
compensación renal
d.- existe vasoconstricción cerebral por la disminución de CO2
*e.- se debe tratar con administración de ácidos.

9.- Cual de las situaciones siguientes NO producirá una alcalosis metabólica:


a.- vómitos con pérdida de jugo gástrico
b.- transfusiones masivas
*c.- fístulas biliares
d.- tratamientos con diuréticos
e.- hiperaldosteronismo

10.- En la alcalosis metabólica podemos encontrar EXCEPTO:


a.- hipoventilación
*b.- hiperpotasemia
c.- CO3H‾ por encima de 25 meq/l.
d.- astenia
e.- arritmias ventriculares
. HEMORRAGIA HEMOSTASIA EN CIRUGÍA. (RESPUESTAS)

1.- Cuando un tumor erosiona un vaso y se produce una hemorragia se llama


hemorragia por:
a.- dieresis
b.- rexis
*c.- diabrosis
d.- diapedesis
e.- en sabana

2.- Relacionar las siguientes situaciones con el tipo de hemorragia que pueden producir:
traumatismo esplénico (A)
úlcera duodenal de la segunda porción duodenal (B)
cáncer de sigma (C)
hemorroides (D)
varices esofágicas (E)

a.- melena (A) (B) *(C) (D) (E)


b.- rectorragia (A) (B) (C) *(D) (E)
c.- hematoquecia (A) (B) *(C) (D) (E)
d.- hematemesis (A) (B) (C) (D) *(E)
e.- hemoperitoneo *(A) (B) (C) (D) (E)

3.- En una hemorragia aguda importante aparecen los siguientes signos clínicos
a.- Palidez *(si) (no)
b.- Oliguria *(si) (no)
c.- Bradicardia (si) *(no)
d.- Ictericia (si) *(no)
e.- Hipotension *(si) (no)

4.- El primer hecho en la hemostasia natural es:


a.- formación del coágulo de fibrina
*b.- vasoconstricción del vaso lesionado
c.- formación del tapón de plaquetas
d.- fibrinolísis
e.- activación de los fosfolipidos plaquetarios

5.- En la hemostasia todos los siguientes factores son fisiológicos EXCEPTO:


a.- Fibrinolisis
b.- Formación de fibrina
c.- Formación de tapones plaquetarios
*d.- Elevación de la adrenalina circulante
e.- Constricción vascular

6.- Después de una lesión vascular el primer paso de la cascada de coagulación será:
a- *Unión del factor XII a la colágena endotelial
b- Desdoblamiento del factor XI en factor activo IX
c- Conversión de protrombina en trombina
d- Conversión de fibrinógeno en fibrina
e- Unión del factor IX al VIII en presencia de Ca.
7.- Respecto a la hemostasia primaria señalar lo correcto:
a.- el factor de Von Willebrand participa en la adhesión plaquetaria. *(si) (no)
b.- el tapón hemostático primario lo forman leucocitos y macrófagos (si) *(no)
c.- la adhesión plaquetaria se produce sobre estructuras subendoteliales *(si) (no)
d.- las fibras de colágeno, elastina y reticulina del subendotelio tienen una elevada
capacidad trombogénica al activar la adhesión plaquetaria *(si) (no)
e.- en la agregación de las plaquetas intervienen receptores de la membrana plaquetaria
que fijan el fibrinógeno formando puentes que “agregan” las plaquetas *(si) (no)

8.- Cuál de los siguientes factores se consume durante la coagulación, pudiendo ser
necesario su aporte si existe excesivo consumo?
*a.- Fibrinógeno (Factor I)
b.- Factor Christmas (IX)
c.- Factor Stuart (X)
d.- Tromboplastina (XI)
e.- Factor Hageman (XII)

9.- Una paciente de 70 años diagnosticada de colecistitis aguda y en tratamiento


antibiótico, sufre un empeoramiento del cuadro séptico, apareciendo además melenas y
petequias. Los estudios de laboratorio muestran trombocitopenia, tiempo de
protrombina prolongado y disminución del fibrinógeno. ¿Cuál es el mejor tratamiento
de la coagulopatía que está iniciando la paciente?
a.- Administrar heparina
*b.- Operar a la paciente y solucionar el cuadro séptico
c.- Administrar sangre completa
d.- Administrar plasma fresco congelado
e.- Administrar corticoides

10.- Las manifestaciones analíticas de un paciente con trombopenia serán (señalar lo


correcto)
a.- Tiempo de hemorragia *(aumentado) (disminuido) (sin alteración)
b.- Número de plaquetas (aumentado) *(disminuido) (sin alteración)
c.- Tiempo de protrombina (aumentado) (disminuido) *(sin alteración)
d.- Tiempo de cefalina (aumentado) (disminuido) *(sin alteración)
e.- Fibrinógeno (aumentado) (disminuido) *(sin alteración)

11.- El INR estará elevado en las siguientes situaciones EXCEPTO:


a.- hepatopatías crónicas
b.- pacientes en tratamiento con anticoagulantes orales
c.- en situación de coagulación intravascular diseminada
*d.- en la enfermedad de Von Willebrand
e.- en pacientes con defecto de fibrinógeno
12.- Relacionar los siguientes factores con su situación u objetivo en el proceso de
hemostasia-coagulación:
plasmina (A) trombina (B) tromboplastina (C) factor de Von Willebrand (D)
Vitamina K (E)

a.- lisis de fibrina *(A) (B) (C) (D) (E)


c.- paso de fibrinógeno a fibrina (A) *(B) (C) (D) (E)
d.- adhesión plaquetaria (A) (B) (C) *(D) (E)
d.- factor tisular del mecanismo extrínseco (A) (B) *(C) (D) (E)
e.- formación de protrombina (A) (B) (C) (D) *(E)

13.- Cual de los siguientes factores NO FORMA PARTE de los procesos fibrinolíticos
o mecanismos anticoagulantes:
a.- Flujo sanguíneo
*b.- Vitamina K
c.- Antitrombina III
d.- Proteína C
e.- Integridad del endotelio vascular

14.- Paciente que refiere heces melénicas desde hace 4 meses y analíticamente presenta
una cifra de hemoglobina de 6,5 gr% ¿qué le trasfundirías?
a.- Plasma fresco congelado
b.- Sangre total
*c.- Concentrados de hematíes
d.- Crioprecipitados
e.- Concentrados de plaquetas

15.- En cual de las siguientes situaciones el INR NO está alterado:


*a.- profilaxis con heparinas de bajo peso molecular
b.- hepatopatías crónicas
c.- coagulación intravascular diseminada
d.- tratamiento con Sintrom
e.- pacientes politransfundidos
HERIDAS, CICATRIZACION. (RESPUESTAS)

1.- Respecto a las contusiones señalar las opciones correctas:


a.- una equimosis es una contusión de segundo grado (si) *(no)
b.- una contusión de tercer grado con escara se tratará como una herida *(si) (no)
c.- una contusión con hematoma es de segundo grado *(si) (no)
d.- la impotencia funcional que produce una contusión debe tratarse siempre con
movilización forzada precoz (si) *(no)
e.- los fármacos analgésicos y anti-inflamatorios pueden estar indicados en las
contusiones *(si) (no)

2.- Cual de las siguientes heridas incisas, tendrá debido a la tensión de las líneas de
Langer, MENOR tendencia a la separación de sus bordes:
a.- herida vertical en cara anterior del cuello
b.- herida vertical en región escapular
*c.- herida de una laparotomía subcostal derecha
d.- herida longitudinal en cara anterior de muslo
e.- herida longitudinal sobre cara anterior de la rodilla

3.- En la fase inflamatoria del proceso de cicatrización las principales células


responsables del Factor de Crecimiento para las células endoteliales y la angiogénesis
son:
a.- leucocitos
* b.- macrófagos
c..- fibroblastos
d.- neutrófilos
e.- células epiteliales

4.- Respecto a las heridas lo siguiente es cierto EXCEPTO


a.- una herida por arma blanca en el tórax que afecta a la pleura parietal es una herida
penetrante
b.- una herida por arma blanca en el abdomen que afecta al peritoneo parietal es una
herida penetrante
c.- una herida por empalamiento afecta al ano y al periné
d.- una herida por arma de fuego que afecta a la pared gástrica es perforante
*e.- una herida de “scalp” es por arrancamiento en la pared torácica

5.- Cual de las siguientes heridas se puede suturar directamente sin necesidad de esperar
una cicatrización por “segunda intención”:
a.- mordedura de perro en una pierna en varón joven.
*b.- herida incisa por arma blanca en la cara
c.- herida incisa de más de seis horas en una pierna en un accidente de tráfico
d.- herida incisa en periné
e.- todas se pueden suturar directamente.
6.- En la cicatrización ocurre lo siguiente EXCEPTO:
*a.- la fase inflamatoria comienza a los tres o cuatro días después de producirse la herida
b.- los macrófagos segregan el factor de crecimiento de los fibroblastos
c.- la epitelización se produce siempre sobre el tejido de granulación y desde los bordes de
la herida.
d.- el hierro y la vitamina C son indispensables para la síntesis del colágeno
e.- el fenómeno de contracción de la herida se produce aproximadamente a las 48 horas de
producirse esta.

7.- Respecto a la cicatrización de las heridas lo siguiente es cierto EXCEPTO


a.- se llama ulcera de Marjolin a la aparición de un carcinoma espino celular implantado
sobre la cicatriz de una quemadura
b.- en la cicatriz queloidea la masa de colágeno sobrepasa los bordes de la herida
c.- las cicatrices hipertróficas generalmente no necesitan tratamiento porque mejoran
con el tiempo
*d.- los pacientes en tratamiento corticoideo tienen mejor cicatrización por su efecto
antiinflamatorio.
e.- las drogas citotóxicas afectan a la síntesis del colágeno.

8.- Sobre el proceso de cicatrización de las heridas señalar lo correcto:


a.- la vitamina C es indispensable en el proceso de síntesis del colágeno *(si) (no)
b.- la tensión máxima de las fibras de colágeno se consigue a los dos o tres días después
de producirse la herida (si) *(no)
c.- el pico máximo de número de fibroblastos se produce al segundo día, en la fase
inflamatoria de la cicatrización (si) *(no)
d.- la resistencia de la herida a la tensión depende del proceso de reorientación de las
fibras de colágeno *(si) (no)
e.- la epitelización se produce siempre sobre el tejido de granulación de forma
centrifuga desde el centro hacia los bordes de la herida (si) *(no)

9.- sobre el fenómeno de “contracción” de la herida señalar lo correcto:


a.- las células responsables de este fenómeno son los macrófagos (si) *(no)
b.- es un mecanismo que aproxima espontáneamente los bordes de la herida y
disminuye su superficie *(si) (no)
c.- las fibras de colágeno juegan un papel fundamental en la contracción de la herida
(si) *(no)
d.- la contracción de la herida comienza después de la fase de maduración de la cicatriz
(si) *(no)
e.- el fenómeno de contracción puede verse afectado en heridas en zonas isquémicas
*(si) (no)

10.- Cual de los siguientes mecanismos es el más característico en la formación de un


queloide:
*a.- una relación alterada entre la formación de colágeno y la actividad de la colagenasa
b.- una fase inflamatoria excesivamente larga
c.- una síntesis excesiva de la enzima colagenasa.
d.- una actividad aumentada de las células endoteliales
e.- una actividad aumentada del fenómeno de angiogénesis
11.- En relación a la influencia de los siguientes factores en la cicatrización señalar lo
correcto:
a.- una tensión de oxígeno baja sirve de estímulo a la actividad de los fibroblastos
(si) *(no)
b.- la síntesis del colágeno se estimula con la hipoxia (si) *(no)
c.- el hierro es necesario para la actividad de los fibroblastos *(si) (no)
d.- los corticosteroides mejoran la cicatrización por su efecto antiinflamatorio (si) *(no)
e.- la hipovitaminosis C afecta a la síntesis del colágeno *(si) (no)

12.- En un caso de hidrosadenitis supurada de la ingle el mejor tratamiento después de


la excisión de la lesión es:
*a.- cierre por segunda intención
b.- cierre primario de la herida quirúrgica
c.- cierre primario retardado
d.- injerto de piel de espesor parcial
e.- realizar un colgajo desde el abdomen para tapar el defecto.

13.-

El paciente de la fotografía ha sufrido un traumatismo en la


región frontal con una barra de hierro. La descripción de la lesión debe ser:
a.- contusión de primer grado con equimosis
b.- contusión de primer grado con sufusión
*c.- contusión de segundo grado con hematoma
d.- contusión de tercer grado
e.- herida contusa.

14.- Relacionar los siguientes mecanismos de producción con los tipos de herida:
arrancamiento (A) herida de bala en abdomen con afectación de la pared gástrica (B)
herida por arma blanca con afectación de la articulación de la rodilla (C)
herida que afecta al ano y perine por objeto punzante (D) mordedura de perro (E)

a.- inciso-contusa (A) (B) (C) (D) *(E)


b.- scalp *(A) (B) (C) (D) (E)
c.- penetrante (A) (B) *(C) (D) (E)
d.- perforante (A) *(B) (C) (D) (E)
e.- empalamiento (A) (B) (C) *(D) (E)
15.- En el proceso inicial de la cicatrización, señalar lo correcto:
a.- la fase inflamatoria en una herida no complicada suele durar entre cuatro y seis días
*(si) (no)
b.- las células implicadas en la fase inflamatoria son los fibroblastos (si) *(no)
c.- los leucocitos liberan la mayoría de los mediadores celulares y los radicales libres de
oxigeno bactericidas *(si) (no)
d.- el TNF y las prostaglandinas son mediadores inflamatorios producidos por los
fibroblastos (si) *(no)
e.- la epitelización comienza en el proceso inicial de la cicatrización. *(si) (no)

16.- Sobre la fase proliferativa de la cicatrización cual de las siguientes afirmaciones es


FALSA:
a.- las células implicadas en esta fase son los fibroblastos
b.- la hidroxiprolina es una de los aminoácidos llave en la síntesis del colágeno
c.- el hierro y la vitamina C son importantes en la cicatrización por su necesidad en la
síntesis del colágeno
d.- La tensión máxima de las fibras de colágeno se consigue como mínimo a los treinta
días de la producción de la herida
*e.- El movimiento es un buen estímulo para la correcta orientación de las fibras de
colágeno de la herida.

17.- Sobre el fenómeno de contracción de la herida una de las siguientes afirmaciones es


FALSA:
a.- la contracción de la herida comienza antes de 48 horas
b.- los miofibroblastos son las células responsables de la contracción de la herida
c.- la formación de colágeno no interviene en la contracción de la herida
*d.- cuando las células epiteliales de ambos bordes de la herida contactan se crean
anclajes definitivos aunque sean estirpes celulares distintas
e.- la citosina es un factor de crecimiento plasmático que estimula el dinamismo celular
de la contracción de la herida.

18.- Los factores siguientes afectan negativamente a la cicatrización:


hematomas (A) glucocorticoides (B) isquemia (C) drogas citotóxicas (D)
hipovitaminosis C (E)
Señalar a cual aspecto de la cicatrización afectan especialmente (señalar solamente un
factor en cada respuesta)
a.- facilita proliferación bacteriana *(A) (B) (C) (D) (E)
b.- afecta a la síntesis del colágeno (A) (B) (C) (D) *(E)
c.- disminuyen la respuesta inflamatoria (A) *(B) (C) (D) (E)
d.- inhiben la multiplicación de fibroblastos (A) (B) (C) *(D) (E)
e.- afecta a la neoangiogénesis (A) (B) *(C) (D) (E)

19.- Respecto a las cicatrices hipertróficas señalar lo correcto:


a.- la causa es generalmente una excesiva tensión en los bordes de la herida *(si) (no)
b.- suelen mejorar con el tiempo *(si) (no)
c.- no tienen preferencia por ninguna localización *(si) (no)
d.- el colágeno se produce en exceso pero no sobrepasa los bordes de la herida
*(si) (no)
e.- pueden mejorar con técnicas que disminuyan la tensión de los bordes de la herida
*(si) (no)
20.- Sobre la patología de la cicatriz señalar lo correcto:
a.- las cicatrices queloides dependen de una predisposición del individuo *(si) (no)
b.- las heridas con cicatrización tórpida pueden degenerar en adenocarcinomas
(si) *(no)
c.- las heridas con cicatrización tórpida pueden degenera en carcinomas epidermoides
*(si) (no)
d.- el mejor tratamiento de los queloides es la extirpación quirúrgica precoz (si) *(no)
e.- el método de Friedrich consiste en la extirpación en bloque de los bordes de la herida
*(si) (no)
RESPUESTAS TEST XVI. HERNIAS Y PARED ABDOMINAL.

1.- Respecto a las hernias inguinales lo siguiente es cierto EXCEPTO:


a.- una hernia estrangulada implica que exista compromiso vascular del
contenido del saco
*b.- una hernia es deslizante cuando el contenido del saco entra fácilmente
en la cavidad abdominal
c.- las hernias directas se producen a través de un defecto de la fascia
transversalis
d.- las hernias crurales son mas frecuentes en mujeres
e.- una hernia crural aparece siempre por debajo del arco crural

2.- El músculo cremaster es una derivación de una de las siguientes


estructuras anatómicas:
a.- aponeurosis oblicua externa
*b.- músculo oblicuo interno
c.- músculo transverso del abdomen
d.- fascia transversales
e.- peritoneo parietal

3.- El cordón espermático contiene todo lo siguiente EXCEPTO:


a.- fibras nerviosas autónomas
*b.- nervio genitofemoral
c.- vasos linfáticos
d.- arteria testicular
e.- conducto deferente

4.- Cuando existe una hernia inguinal las estructuras siguientes pueden
incluirse en el cordón espermático:
a.- saco de hernia directa (si) *(no)
b.- saco de hernia crural (si) *(no)
c.- saco de hernia indirecta *(si) (no)
d.- grasa preperitoneal (lipoma preherniario) *(si) (no)
e.- protusión de la facsia tranversalis (si) *(no)

5.- Respecto a las hernias deslizantes es CIERTO que:


a.- tienen una recurrencia mayor después de la corrección quirúrgica
*b.- el saco incluye siempre una estructura retroperitoneal
c.- son más frecuentes en la región inguinal izquierda
d.- aparecen de forma casi exclusiva en mujeres
e.- siempre precisan reparación a través de una laparotomía
6.- Cual de las siguientes estructuras anatómicas es más profunda en la
región inguinal:
a.- fascia de Scarpa
*b.- fascia transversalis
c.- fascia del oblicuo mayor
d.- cordón espermático
e.- músculo oblicuo mayor

7.- Respecto a las hernias crurales lo siguiente es cierto:


a.- aparecen siempre por encima del arco crural (si) *(no)
b.- son mas frecuentes en mujeres *(si) (no)
c.- pueden estar en relación con los vasos femorales *(si) (no)
d.- es frecuente que la vejiga se deslice con el saco (si) *(no)
e.- tienen relación con la vaginal del testículo (si) *(no)

8.- Uno o varios tipos de hernias corresponde a cada uno de los pacientes
siguientes. Indicar los diagnósticos más probables:

(1) hernia crural


(2) hernia estrangulada
(3) hernia incarcerada
(4) hernia de Spiegel
(5) hernia inguinal directa
(6) hernia inguinal indirecta
(7) eventración

a.- una mujer de 73 años, encontrándose previamente bien, refiere la


aparición brusca de una tumoración en región inguinal derecha, debajo del
ligamento inguinal, con motivo de un acceso de tos. Tiene dolor a dicho
nivel y dolores de características cólicas por todo el abdomen. Seleccionar
dos diagnósticos *(1) (2) *(3) (4) (5) (6) (7)
b.- un paciente de 75 años ha sido sometido a una hemicolectomía
izquierda por un cáncer de colon perforado hace tres años. Presenta una
tumoración blanda en mesogastrio. Seleccionar un diagnóstico
(1) (2) (3) (4) (5) (6) *(7)
c.- un varón de 35 años presenta una tumoración blanda inguino-escrotal,
que en la palpación aparece a través del orificio inguinal superficial.
Seleccionar dos diagnósticos. (1) (2) (3) (4) *(5) *(6) (7)
d.- Una paciente presenta signos de obstrucción intestinal y una masa en la
pared abdominal infraumbilical, derecha, lateral al músculo recto
Seleccionar dos diagnósticos (1) (2) *(3) *(4) (5) (6) (7)
e.- Un paciente de 50 años refiere la aparición brusca de una tumoración de
unos 5 cms. en región inguinal derecha, acompañándose de dolor intenso.
El cuadro le dura aproximadamente una hora y posteriormente la
tumoración desaparece cediendo el dolor local. Realiza sin embargo una
deposición con sangre tres horas después. Seleccionar tres diagnósticos.
(1)*(2) (3) (4) *(5) *(6) (7)

9.- Respecto a las evisceraciones y eventraciones cual de las siguientes


afirmaciones es FALSA
a.- las eventraciones siempre son posquirúrgicas
*b.- la evisceración siempre implica estrangulación de asas
c.- la evisceración puede ocurrir con motivo de una herida abdominal por
arma blanca
d.- en las eventraciones no hay solución de continuidad en el peritoneo
e.- en las evisceraciones siempre hay solución de continuidad en el
peritoneo.

10.- Cuales de las siguientes situaciones se pueden considerar como factor


de riesgo elevado de eventración:
a.- obesidad *(si) (no)
b.- utilización de material de sutura reabsorbible (si) *(no)
c.- paciente con peritonitis *(si) (no)
d.- paciente en tratamiento con corticoides *(si) (no)
e.- paciente con ascitis *(si) (no)

11.- Respecto a las hernias inguinales directas e indirectas señalar la opción


correcta:
a.- en las hernias indirectas el saco herniario sale en el orificio inguinal
profundo por fuera de los vasos epigástricos *(si) (no)
b.- en las hernias directas el saco herniario sale a través del cordón
espermático (si) *(no)
c.- las hernias inguinales directas son las más frecuentes y suponen el 80%
de las hernias inguinales *(si) (no)
d.- en las hernias directas de gran tamaño la vejiga puede estar deslizada en
el saco *(si) (no)
e.- las hernias directas protuyen a través de la fascia transversalis *(si) (no)

Respecto a las imágenes siguientes señalar las opciones correctas:


12.- Hernia umbilical en un niño de tres meses de edad menor de dos cms.
asintomática. ¿Requiere tratamiento quirúrgico antes de que el niño cumpla
un año?

(si) *(no)

13.- A que tipo de hernia corresponde esta imagen (la línea marca el
pliegue cutáneo inguinal):

a.- crural
*b.- crural incarcerada
c.- inguinal indirecta estrangulada
d.- inguinal directa incarcerada
e.- de Spiegel

14.- Paciente de 55 años con aparición progresiva en los últimos tres años
de las dos tumoraciones que aparecen en la imagen. Desaparecen en
posición de decúbito.

a.- se trata de hernias bilaterales


*(Inguinales directas)
(Crurales)
(Inguinoescrotales)
(Indirectas incarceradas)
b.- se deben intervenir con carácter
(Urgente)
*(Programado)
(Se debe esperar y valorar la evolución del tamaño)
c.- cuando se corrijan quirúrgicamente se deben corregir
*(Simultáneamente)
(En dos tiempos)
d.- mientras se decide la intervención se debe aconsejar la utilización de un
braguero
(si) *(no)
e.- la intervención que menores recidivas tiene en este caso, será:
(Intervención de Bassini)
*(Corrección con prótesis bilateral reticular)
(Corrección con plicatura simple de la fascia transversalis)
INFECCION respuestas

1.- Señalar la opción CORRECTA respecto a la clasificación del tipo de cirugía (limpia,
limpia-contaminada, contaminada, sucia) en la siguiente lista de intervenciones:
a.- colocación de prótesis de cadera
*(limpia) (limpia-contaminada) (contaminada) (sucia)
b.- apendicitis aguda supurada
(limpia) (limpia-contaminada) (contaminada) *(sucia)
c.- cirugía reglada de resección de un cáncer de colon sigmoide
(limpia) *(limpia-contaminada) (contaminada) (sucia)
d.- cirugía cardiaca, by-pass coronario.
*(limpia) (limpia-contaminada) (contaminada) (sucia)
e.- traumatismo abdominal con herida perforante de más de cuatro horas de evolución.
(limpia) (limpia-contaminada) (contaminada) *(sucia)

2.- La incidencia de la infección post-operatoria en una cirugía limpia debe ser menor
del:
*a.- 5%
b.- 10%
c.- 15%
d.- 20%
e.- 25%

3.- Sobre el uso de los antibióticos en la profilaxis quirúrgica señalar lo correcto:


a.- el momento más recomendable para administrar el antibiótico es durante la
inducción anestésica *(si) (no)
b.- la vía de elección es la oral (si) *(no)
c.- la dosis debe ser la terapéutica máxima *(si) (no)
d.- el antibiótico elegido debe tener un perfil farmacocinetico adecuado para difundirse
al tejido extravascular *(si) (no)
e.- se debe mantener la administración al menos durante dos días después de la
intervención (si) *(no)

4.- Señalar en cuales de las siguientes intervenciones debe utilizarse un antibiótico


profiláctico con capacidad anaerobicida:
a.- apendicitis aguda *(si) (no)
b.- corrección de una hernia inguinal con prótesis (si) *(no)
c.- cirugía a corazon abierto (si) *(no)
d.- cirugía no urgente de colon *(si) (no)
e.- craneotomía (si) *(no)

5.-¿En una intervención por hernia inguinal sin necesidad de colocar prótesis los
factores siguientes indican la profilaxis antibiótica?
a.- paciente en tratamiento quimioterapico por un cáncer de colon *(si) (no)
b.- paciente en tratamiento radioterapico por un cáncer de próstata *(si) (no)
c.- paciente mayor de 70 años (si) *(no)
d.- paciente VIH + *(si) (no)
e.- paciente con 4000 leucocitos /ml. (si) *(no)
6.- A un paciente de 50 años está previsto practicarle una hemicolectomia derecha por
un cáncer de ciego, no obstructivo y sin metástasis. Cual de las siguientes preparaciones
es la más CORRECTA:
a.- enemas de limpieza 24 h. antes de la intervención
b.- antibioterapia profiláctica con metronidazol y gentamicina solamente.
*c.- solución evacuante 24 h. antes de la intervención y antibioterapia profiláctica con
metronidazol y gentamicina
d.- antibioterapia profiláctica con amoxicilina+ clavulanico solamente
e.- en principio este paciente no precisa preparación especifica

7.- Los siguientes factores facilitan la flora nosocomial en un periodo postoperatorio


EXCEPTO:
a.- colonización de las vías respiratorias altas
b.- perdida de la barrera ácida gástrica
*c.- edad avanzada del paciente
d.- uso previo de antibióticos
e.- ileo paralítico postoperatorio

8.- En la cirugía gastroduodenal en cuales de las siguientes situaciones está indicada la


profilaxis antibiotica:
a.- uso de anti H2 durante varios meses *(si) (no)
b.- cáncer gástrico *(si) (no)
c.- estenosis pilórica *(si) (no)
d.- anemia perniciosa *(si) (no)
e.- cirugía de la obesidad *(si) (no)
Respuestas NUTRICIÓN PARENTERAL Y ENTERAL

1.- Respecto al concepto de desnutrición en el paciente quirúrgico señalar lo correcto:


a.- el paciente desnutrido mantiene su masa muscular (si) *(no)
b.- la desnutrición cursa con hiperpotasemia (si) *(no)
c.- la situación de desnutrición interfiere con los mecanismos endocrino- metabólicos
de adaptación al stress *(si) (no)
d.- las complicaciones posquirúrgicas aumentan hasta un 70% en los pacientes con
desnutrición grave *(si) (no)
e.- en la composición corporal del paciente desnutrido existe un exceso relativo de agua
*(si) (no)

2.- Cual de las siguientes determinaciones analíticas tiene más valor para determinar el
estado nutricional preoperatorio de un paciente:
a.- hemoglobina
b.- hematocrito
*c.- proteína ligada al retinol
d.- albúmina
e.- proteínas totales

3.- La fórmula de Harris-Benedict, en el paciente quirúrgico, es un método de cálculo


de:
*a.- las necesidades energéticas totales basales
b.- las necesidades proteicas del paciente postoperado
c.- el balance nitrogenado
d.- la distribución relativa de calorías entre hidratos de carbono, proteínas y grasas en
las necesidades postagresión.
e.- la relación entre consumo de O2 y producción de CO2

4.- Sobre las siguientes afirmaciones señalar lo correcto:


a.- la formula de Harris Benedict es el mejor método de medida de la situación
nutricional preoperatoria de un paciente (si) *(no).
b.- las necesidades calóricas basales de un paciente en reposo son de
25 a 35 /kcal./kgr./día *(si) (no)
c.- el GEB (gasto energético basal) depende del nivel de stress *(si) (no)
d.- la calorimetría indirecta relaciona el consumo de O2 y la producción de CO2
*(si) (no)
e.- la insuficiencia renal es una situación que aumenta los requerimientos energéticos de
un paciente *(si) (no)

5.- Respecto a los requerimientos energéticos del paciente quirúrgico señalar la


afirmación FALSA
a.- los lípidos proveen en general 9 Kcal./gr.
*b.- en una situación de stress alto las reservas de glucógeno se agotan a partir de la
primera semana
c.- las necesidades mínimas de carbohidratos son de 100-150 g/día.
d.- las necesidades mínimas de proteínas son 0.8-1 g/kg. de peso/día.
e.- las reservas de glucógeno del organismo se almacenan en hígado y músculo
principalmente.
6.- Respecto a la Glutamina lo siguiente es cierto EXCEPTO:
a.- es indispensable para el metabolismo de los enterocitos
b.- es el amino ácido más abundante en la sangre
*c.- es un aminoácido esencial
d.- puede ser necesario para la proliferación de linfocitos y macrófagos
e.- juega un papel importante en el mantenimiento de la mucosa intestinal durante el
stress.

7.- Señalar la opción correcta cuando nos referimos a la comparación entre nutrición
enteral y parenteral
1.- la incidencia y riesgo de sepsis son menores en la nutrición enteral *(si) (no)
2.- la nutrición parenteral es más barata que la nutrición enteral (si) *(no)
3.- la nutrición enteral tiene menos complicaciones graves *(si) (no)
4.- la nutrición parenteral disminuye la traslocación bacteriana más que la nutrición
enteral (si) *(no)
5.- la nutrición enteral mejora mas que la parenteral la función inmunitaria *(si) (no)

8.- En cual de las siguientes situaciones NO ESTA INDICADA la nutrición enteral


administrada por yeyunostomia:
a.- pancreatitis aguda
*b.- enfermedad inflamatoria intestinal en fase aguda
c.- paciente con adenocarcinoma de esófago
d.- paciente séptico
e.- paciente en coma neurológico

9.- Respecto a la nutrición parenteral una de las siguientes afirmaciones es FALSA:


a.- la aportación de proteínas debe realizarse en función del grado de stress del paciente
b.- la relación gr. de Nitrógeno proteico / calorías no proteicas en la solución aportada
debe ser de 1/ 100 aproximadamente.
c.- una solución parenteral que aporte al menos 2500 Kcal./día requiere siempre un
acceso venoso central
*d.- para aportar 3000 Kcal./día mediante soluciones parenterales es suficiente un
volumen de 1000 c.c.
e.- en la nutrición parenteral total las calorías se aportan con carbohidratos y grasas
principalmente.

10.- En las siguientes patologías puede estar indicado un soporte nutricional. Elegir la
forma (parenteral o enteral) más adecuada:
a.- pancreatitis aguda (parenteral) *(enteral)
b.- dehiscencia de sutura después de una resección por cáncer de colon
*(parenteral) (enteral)
c.- paciente con enfermedad de Crohn *(parenteral) (enteral)
d.- paciente quemado (parenteral) *(enteral)
e.- paciente en postoperatorio de una peritonitis por apendicitis aguda
*(parenteral) (enteral)
11.- Respecto a la nutrición enteral una de las siguientes afirmaciones es FALSA:
a.- la alimentación por sonda gástrica está contraindicada en pacientes con riesgo de
aspiración
b.- las dietas por yeyunostomia deben tener una presión osmótica baja.
c.- la alimentación por sonda gástrica permite la utilización de dietas naturales
*d.- en caso de obstrucción intestinal está indicada la vía nasoyeyunal
e.- la deshidratación por diarreas es una complicación de las dietas demasiado
hiperosmolares.

12.- Respecto a los carbohidratos en relación con los requerimientos energéticos de un


paciente quirúrgico señalar las opciones correctas
a.- generalmente aportan entre el 30-60% de las calorías totales *(si) (no)
b.- es necesario un mínimo de 150 grs./día de glucosa para las necesidades
indispensables de cerebro y células sanguíneas *(si) (no)
c.- solamente el 10% de la glucosa en forma de glucógeno se almacena en el músculo
esquelético (si) *(no)
d.- las reservas totales de glucógeno son de aproximadamente 5 kgrs. en un adulto de 70
kgrs de peso. (si) *(no)
e.- las reservas totales de glucógeno pueden servir para las necesidades de
aproximadamente 10 días en un postoperatorio normal (si) *(no)

13.- Respecto a los requerimientos de proteínas señalar lo correcto:


a.- las necesidades normales en el individuo sano son de 0.8-1 gr. /kg/día *(si) (no)
b.- los pacientes con insuficiencia renal requieren mas proteínas *(si) (no)
c.- en situación de encefalopatía hepática se debe aumentar el aporte proteico (si) *(no)
d.- el balance nitrogenado es la medida en bruto de consumo de proteínas *(si) (no)
e.- el soporte nutricional artificial está indicado cuando el balance nitrogenado es
positivo (si) *(no)

14.- Sobre los aminoácidos (AA) y el aporte nutricional artificial:


a.- Por lo menos el 20% del aporte proteico debe ser en forma de AA esenciales
*(si) (no)
b.- Los AA esenciales son los que puede producir el organismo (si) *(no)
c.- La glutamina es la principal fuente energética de los enterocitos *(si) (no)
d.- La leucina y la isoleucina son AA esenciales *(si) (no)
e.- La glutamina es un AA esencial (si) *(no)

15.- Cual de las siguientes afirmaciones es INCORRECTA sobre los lípidos en la


nutrición artificial:
a.- Los lípidos deben aportar al menos 25% del total de los requerimientos calóricos
durante la nutrición suplementaria
*b.- Los triglicéridos de cadena larga son los que se absorben directamente por los
enterocitos
c.- Las pancreatitis severas afectan a la absorción de triglicéridos
d.- El ácido linoleico es ácido graso esencial
e.- La falta de aporte correcto de ácidos grasos puede dar manifestaciones clínicas a
partir de la cuarta semana en un paciente con nutrición parenteral total.
PATOLOGIA QUIRÚRGICA DEL ESOFAGO.
RESPUESTAS.

1.- Un paciente de 70 años tiene un cuadro de disfagia, regurgitaciones de comida no


digerida y halitosis crónica. Presenta el esófagograma siguiente:

a.- ¿para establecer el diagnóstico de divertículo de Zenker es necesaria la manometría?


(si) *(no)
b.- el divertículo de Zenker es el divertículo más frecuente del esófago *(si) (no)
c.- generalmente aparece en edades entre 30 y 40 años (si) *(no)
d.- se localiza siempre por debajo del músculo cricofaringeo (si) *(no)
e.- es un divertículo de pulsión *(si) (no)

2.- Respecto al divertículo de Zenker lo siguiente es cierto EXCEPTO:


*a.- la pared del divertículo tiene todas las capas del esófago
b.- su complicación más frecuente son las neumonías por aspiración
c.- el tratamiento quirúrgico consiste en la miotomía del músculo cricofaringeo y la
diverticulectomía
d.- es consecuencia de una incoordinación cricofaringea
e.- suelen aparecer en la cara posterior del esófago.

3.- Cual de las siguientes situaciones manométricas y pHmetricas NO encontraremos en


un paciente con el diagnóstico de achalasia:
a.- esfínter esofágico inferior con presión basal elevada
b.- peristaltismo descoordinado en el cuerpo esofágico
c.- esfínter esofágico superior con presión normal
*d.- reflujo gastroesofágico de 24 h. elevado
e.- prueba de anticolinergicos positiva (test de Mecolil)
4.- Un paciente de 50 años refiere desde hace unos seis meses una historia de disfagia
para sólidos y líquidos así como regurgitaciones de comida sin digerir al acostarse. El
estudio baritado esofágico es el siguiente:

Se establece un diagnóstico de presunción de achalasia.


¿Qué consideraciones respecto a este paciente son correctas?

a.- la primera prueba complementaria que debe hacerse es una endoscopia *(si) (no)
b.- la manometría es necesaria aunque la radiología, endoscopia y clínica sean típicas
*(si) (no)
c.- el tratamiento médico con nitratos e inhibidores de los canales del Calcio es eficaz en
más del 50% de los casos (si) *(no)
d.- la dilatación endoscópica neumática es eficaz en más del 90% de los casos (si) *(no)
e.- sería lógico que este paciente refiriera también reflujo ácido gastroesofágico
(si) *(no)

5.- Las siguientes afirmaciones respecto a la achalasia son correctas EXCEPTO:


a.- en la mayoría de los casos las células ganglionares del cuerpo esofágico están
ausentes o han sufrido un proceso degenerativo
*b.- en el cuerpo esofágico la presión es tres veces menor que los valores normales
c.- la incidencia mayor es en personas entre 30 y 50 años.
d.- los alimentos fríos provocan mayor disfagia que los alimentos templados o calientes.
e.- la incidencia de cáncer de esófago es mayor en los pacientes con megaesofago

6.- La imagen siguiente corresponde a una miotomia de Heller realizada en un paciente


con diagnóstico de achalasia. Señalar las estructuras correspondientes a los números de
la imagen:

a.- mucosa esofagica (1) *(2) (3) (4) (5)


b.- pilar diafragmatico izquierdo *(1) (2) (3) (4) (5)
c.- borde de la capa muscular esofagica seccionada (1) (2) *(3) (4) (5)
d.- fundus gástrico (1) (2) (3) (4) *(5)
e.- nervio vago anterior (1) (2) (3)*(4) (5)

7.- Las siguientes estructuras están implicadas en la deglución EXCEPTO:


a.- músculo cricofaringeo
b.- epiglotis
c.- paladar blando
*d.- esfínter esofagico inferior
e.- lengua

8.- Los siguientes factores facilitan la relajación del esfínter esofagico inferior
EXCEPTO
a.- colecistikinina
b.- anticolinergicos
c.- etanol
*d.- gastrina
e.- nicotina

.- Preguntas 9 a 12.
Seleccionar para los casos clínicos que se exponen en cada una de las preguntas los
diagnósticos (se pueden proponer más de uno) más probables de la lista siguiente:
Enfermedad por reflujo gastroesofágico (a)
Leiomioma esófago (b)
Divertículo de Zenker (c)
Hernia de hiato por deslizamiento (d)
Hernia de hiato paraesofagica (e)
Cáncer de esófago (f)
Achalasia (g)
Estenosis peptica de esófago (h)

9.- Un paciente de 40 años con historia antigua de pirosis retroesternal, que no precisó
tratamiento, refiere desde hace un año disfagia para alimentos sólidos y desde hace dos
meses también para líquidos. El esofagograma muestra una estenosis (ver figura) que en
la biopsia ha resultado ser benigna.

Seleccionar dos diagnósticos para este paciente


*a, b, c, d, e, f, g,*h

10.- Un paciente varón de 58 años presenta una historia de disfagia para líquidos y
sólidos y regurgitaciones intermitentes de comida sin digerir
Seleccionar dos diagnósticos para este paciente
a, b,*c, d, e, f,*g, h

11.- Un paciente varón de 75 años de edad refiere episodios de disnea de esfuerzo con
sensación de opresión retroesternal. Los estudios cardiológicos han sido normales.
Presenta la radiografía de tórax lateral de la imagen siguiente .

Seleccionar un diagnóstico para este paciente.


a, b, c, d, *e, f, g, h

12.- Un paciente de 56 años presenta una historia de cuatro episodios de neumonía en


lóbulo inferior derecho en el ultimo año. Tambien refiere episodios de tos seca
nocturna. Los estudios pulmonares incluida broncoscopia han sido normales. No es
fumador. El estudio radiológico con bario del esófago ha sido normal.
Seleccionar un diagnóstico para este paciente *a, b, c, d, e, f, g, h

13.- Todas las afirmaciones siguientes son ciertas respecto al anillo de Schatzki
EXCEPTO:
a.-. en muchos pacientes existe una relación entre la existencia del anillo y la ingestión
de ciertos fármacos.
b.- el tratamiento del anillo de Schaztki, si no se precisa tratar un reflujo asociado es la
dilatación forzada endoscópica
*c.- el anillo de Schatzki es una lesión precancerosa
d.- el síntoma principal es la disfagia de corta duración
e.- el anillo de Schaztki es un anillo delgado submucoso en el tercio distal del esófago.

14.- La imagen siguiente corresponde a la corrección quirúrgica de una hernia hiatal


Señalar las opciones correctas:
a.- se trata de una intervención de Nissen con fundoplicatura de 360º *(si) (no)
b.- los pilares diafragmáticos se han aproximado por delante del esófago (si) *(no)
c.- uno de los objetivos es conseguir reponer en el abdomen un segmento de esófago lo
más amplio posible. *(si) (no)
d.- se debe asociar siempre una vagotomia y piloroplastia en este tipo de intervención
(si) *(no)
e.- la fundoplicatura puede producir disfagia si queda demasiado apretada *(si) (no)

15.- Entre las complicaciones de las hernias hiatales están :


a.- incarceración
b.- vólvulo gástrico
c.- estenosis péptica por reflujo
d.- hemorragia por ulcera intraherniaria
*e.- todo lo anterior puede ocurrir

16.- El síndrome de Mallory Weiss consiste en :


*a.- hemorragia por ulceras en el tercio distal del esófago
b.- incoordinacion motora del cuerpo esófagico y cardias
c.- mucosa gástrica metaplasica en tercio distal de esófago
d.- perforación espontanea del esófago
e.- estenosis péptica del esófago

17.- Un paciente con disfagia, dolor retroesternal y voz bitonal, probablemente tendrá :
a.- una achalasia
b.- un diverticulo de Zenker
*c.- un carcinoma de tercio medio esofágico
d.- una estenosis péptica
e.- un espasmo difuso esofágico

18.- Cuales de las siguientes afirmaciones respecto al carcinoma epidermoide de


esófago son ciertas:
a.- el síntoma inicial más frecuente es la disfagia *(si) (no)
b.- el tabaquismo se considera un factor etiologico importante *(si) (no)
c.- la localización más frecuente es el tercio medio del esófago *(si) (no)
d.- la capa serosa del esófago evita la propagación temprana del tumor hacia el
mediastino (si) *(no)
e.- suele asentar sobre un esófago de Barret (si) *(no)
19.- Cuales de las siguientes afirmaciones son correctas respecto al adenocarcinoma de
esófago:
a.- la mayoria de ellos se diagnostican precozmente (si) *(no)
b.- la quimioterapia y la radioterapia por si solas son más eficaces que la cirugía en su
tratamiento (si) *(no)
c.- los adenocarcinomas de tercio distal suelen metastatizar en los ganglios cervicales
(si) *(no)
d.- es excepcional que produzcan metástasis hepáticas (si) *(no)
e.- se considera que la mayoría de estas lesionas asientan sobre un esófago de Barret
*(si) (no)

20.- Un paciente de 47 años presenta un cuadro de disfagia moderada para sólidos desde
hace seis meses. No refiere perdida de peso ni anemia. En la endoscopia (ver imagen) se
aprecia una tumoración submucosa, no ulcerada a nivel de tercio medio, muy
sospechosa de leiomioma. Señalar las consideraciones correctas sobre este caso:

a.- antes de proceder al tratamiento quirúrgico es obligado realizar una biopsia


endoscópica (si) *(no)
b.- el tratamiento quirúrgico debe consistir en la enucleación del tumor sin necesidad de
resección esofágica. *(si) (no)
c.- generalmente son tumores múltiples en el 50% de los casos
d.- aunque fuera un hallazgo se debería resecar por su alto poder de malignización
(si) *(no)
e.- la disfagia es el síntoma principal *(si) (no)

21.- Las siguientes consideraciones se pueden aplicar al cáncer de esófago


EXCEPTO :
*a.- su localización habitual es el tercio superior
b.- la edad de máxima incidencia es 50-70 años
c.- la relación varón/hembra es 3/1 aproximadamente
d.- su síntoma principal es la disfagia
e.- en tercio medio y superior más del 80% son carcinomas epidermoides
PATOLOGIA QUIRÚRGICA DEL TÓRAX. Respuestas.

1.-En un paciente politraumatizado, diagnosticado de volet costal, señalar lo correcto:


a.- en pequeños defectos de la zona posterior del tórax, se puede mantener una actitud
conservadora, sin realizar fijación, dependiendo de la repercusión que produzcan.
*(si) (no)
b.- los volet torácicos anteriores son los que requieren con mayor frecuencia fijación
quirúrgica inmediata *(si) (no)
c.- dependiendo de la situación clínica del paciente la intubación con respiración
asistida es una alternativa válida a la fijación quirúrgica de urgencia *(si) (no)
d.- la característica clínica del volet costal es el movimiento paradójico de la parte de la
pared torácica afectada *(si) (no)
e.- en situación urgente la colocación de un tubo de tórax con aspiración soluciona
momentáneamente la insuficiencia respiratoria del paciente (si) *(no)

2.- Son indicaciones de toracotomía de urgencia en el contexto de un traumatismo


torácico:
a.- Traumatismos abiertos, como las heridas por arma de fuego *(si) (no)
b.- Cuando se asocian un volet costal con un hemo-neumotórax. *(si) (no)
c.- Cuando tras colocar tubo torácico, el drenaje inicial de sangre es superior
a los 1500 c.c. *(si) (no)
d.- Todos los hemotórax, independientemente de su cuantía, cuando no se conoce el
punto de sangrado. (si) *(no)
e.- Una situación de enfisema subcutáneo, con sospecha de rotura traqueal *(si) (no)

3.- Respecto al neumotórax espontáneo, señalar lo correcto:


a.- Cuando se produce en jóvenes, suele ser producido por la rotura espontánea de
"blebs" pleurales. *(si) (no)
b.- El neumotórax espontáneo tiene tendencia a la recidiva, y en ciertos casos es
necesario realizar una toracotomía o cirugía toracoscópica para un control definitivo de
la enfermedad. *(si) (no)
c.- El tratamiento inicial debe ser un tubo de drenaje torácico solamente con sellado
bajo agua, y que no debe llevar aspiración (si) *(no)
d.- Una de las formas de presentación más frecuentes en jóvenes, es el "neumotórax a
tensión", que se corrige con la colocación inmediata de un drenaje torácico *(si) (no)
e.- El neumotórax espontáneo secundario es el que ocurre en enfermos con patología
broncopulmonar previa y más frecuentemente se asocia a la presencia de bullas
pulmonares. *(si) (no)

4.- Respecto a los traumatismos torácicos, una de las siguientes respuestas es falsa:
*a.-Las fracturas esternales precisan siempre corrección quirúrgica de la deformidad.
b.-Los fragmentos de costillas pueden provocar dislaceraciones pulmonares y
neumotórax.
c.-La contusión cardiaca se asocia con frecuencia a fracturas esternales y es crítica la
observación hospitalaria, la vigilancia de alteraciones electrocardiográficas y en ciertos
casos la realización de ecocardiograma.
d.-La contusión pulmonar se manifiesta mediante la aparición en la radiografía de tórax
de infiltrados alveolares.
e.-La fractura de la primera costilla se suele asociar a traumatismos de gran energía y es
preciso descartar otras lesiones asociadas.
5.- Respecto a los drenajes torácicos, señalar lo correcto
a.- es necesario siempre colocarlos bajo anestesia general (si) *(no)
b.- se conectan a un dispositivo que básicamente consiste en un sello de agua o válvula
unidireccional que solo permite la salida de liquido o aire de la cavidad pleural
*(si) (no)
c.- cuando se colocan para drenar contenido liquido del espacio pleural se introducirán
entre el segundo y tercer espacio intercostal (si) *(no)
d.- es indispensable disponer de una radiografía de tórax para colocarlos (si) *(no)
e.- están contraindicados en los traumatismos abiertos de tórax *(si) (no)

6.- Respecto al mesotelioma difuso maligno señalar la afirmación CORRECTA:


*a.- Se relaciona con la inhalación de fibras de asbesto
b.- La forma clínica de presentación más frecuente es la insuficiencia respiratoria brusca
c.- Se alcanza solamente con cisplatino una supervivencia del 85% a los cinco años
d.- El tratamiento quirúrgico permite una supervivencia del 95 % a los tres años
e.- El diagnostico diferencial fundamental es con las bronquiectasias.

7.- Ante un cáncer bronquial T2 N0 M0 confirmado por mediastinoscopia se debe:


*a.- plantear el tratamiento quirúrgico directamente
b.- tratar solamente con quimioterapia
c.- tratar solamente con radioterapia
d.- tratar con quimioterapia neoadyuvante e intervenir posteriormente si cumple
criterios de operabilidad.
e.- tratar con radioterapia preoperatoria y posterior resección.

8.- La tricoptisis es patognomónica de:


a.- timoma
b.- teratoma
c.- linfoma
d.- neurinoma
e.- carcinoma bronquial.

9.- Cual de los siguientes hallazgos se puede encontrar en la radiografía de tórax de un


paciente con rotura de aorta torácica después de un accidente de tráfico (señalar lo
correcto):
a.- Ensanchamiento mediastínico (más de 8 cms. en su porción superior)
*(si) (no)
b.- Pérdida de nitidez del botón aórtico *(si) (no)
c.- Derrame pleural izquierdo *(si) (no)
d.- Neumomediastino (si) *(no)
e.- Casquete apical izquierdo *(si) (no)

10.- En un paciente con un carcinoma epidermoide de pulmón derecho el dato que


indica peor pronóstico y probable irresecabilidad quirúrgica es:
a.- tos crónica
b.- hemoptisis
*c.- ronquera y voz bitonal
d.- neumonitis obstructiva
e.- dolor pleurítico
11.- ¿Las manifestaciones siguientes de un carcinoma de pulmón indican la existencia
de metástasis e inoperabilidad ?
a.- hipercalcemia (si) *(no)
b.- síndrome cushingoide (si) *(no)
c.- síndrome similar a miastenia (si) *(no)
d.- ginecomastia moderada (si) *no)
e.- secreción alterada de hormona antidiurética (si) *(no)

12.- Cual de las siguientes afirmaciones es CORRECTA respecto al carcinoide


bronquial:
a.- es avascular
b.- se localiza generalmente en bronquiolos finos
*c.- es raro que provoque metástasis a distancia
d.- es muy radiosensible
e..- cuando se presenta el síndrome carcinoide es incurable

13.- En un paciente asintomático se encuentra un nódulo pulmonar solitario de 2 cms.


de diámetro. Los estudios radiológicos muestran que el nódulo tiene un núcleo central
de calcio. La actitud correcta con esta lesión es:
*a.- seguimiento periódico con estudios radiológicos.
b.- biopsia transbronquial
c.- biopsia percutánea dirigida con TAC
d.- tratamiento con anfotericina
e.- toracotomía y resección

14.- Una paciente con sarcoma osteogénico de fémur, extirpado dos años antes, presenta
dos lesiones metastásicas pequeñas en el pulmón derecho y otra en el pulmón izquierdo.
Si no se encuentran otras lesiones en ningún otro órgano el tratamiento más correcto
sería:
*a.- resección bilateral de las lesiones
b.- quimioterapia solamente
c.- radioterapia solamente
d.- radioterapia y quimioterapia conjuntamente
e.- observación durante tres meses para valorar evolución y decidir después el
tratamiento.
RESPUESTAS TEST XVI. HERNIAS Y PARED ABDOMINAL.

1.- Respecto a las hernias inguinales lo siguiente es cierto EXCEPTO:


a.- una hernia estrangulada implica que exista compromiso vascular del
contenido del saco
*b.- una hernia es deslizante cuando el contenido del saco entra fácilmente
en la cavidad abdominal
c.- las hernias directas se producen a través de un defecto de la fascia
transversalis
d.- las hernias crurales son mas frecuentes en mujeres
e.- una hernia crural aparece siempre por debajo del arco crural

2.- El músculo cremaster es una derivación de una de las siguientes


estructuras anatómicas:
a.- aponeurosis oblicua externa
*b.- músculo oblicuo interno
c.- músculo transverso del abdomen
d.- fascia transversales
e.- peritoneo parietal

3.- El cordón espermático contiene todo lo siguiente EXCEPTO:


a.- fibras nerviosas autónomas
*b.- nervio genitofemoral
c.- vasos linfáticos
d.- arteria testicular
e.- conducto deferente

4.- Cuando existe una hernia inguinal las estructuras siguientes pueden
incluirse en el cordón espermático:
a.- saco de hernia directa (si) *(no)
b.- saco de hernia crural (si) *(no)
c.- saco de hernia indirecta *(si) (no)
d.- grasa preperitoneal (lipoma preherniario) *(si) (no)
e.- protusión de la facsia tranversalis (si) *(no)

5.- Respecto a las hernias deslizantes es CIERTO que:


a.- tienen una recurrencia mayor después de la corrección quirúrgica
*b.- el saco incluye siempre una estructura retroperitoneal
c.- son más frecuentes en la región inguinal izquierda
d.- aparecen de forma casi exclusiva en mujeres
e.- siempre precisan reparación a través de una laparotomía
6.- Cual de las siguientes estructuras anatómicas es más profunda en la
región inguinal:
a.- fascia de Scarpa
*b.- fascia transversalis
c.- fascia del oblicuo mayor
d.- cordón espermático
e.- músculo oblicuo mayor

7.- Respecto a las hernias crurales lo siguiente es cierto:


a.- aparecen siempre por encima del arco crural (si) *(no)
b.- son mas frecuentes en mujeres *(si) (no)
c.- pueden estar en relación con los vasos femorales *(si) (no)
d.- es frecuente que la vejiga se deslice con el saco (si) *(no)
e.- tienen relación con la vaginal del testículo (si) *(no)

8.- Uno o varios tipos de hernias corresponde a cada uno de los pacientes
siguientes. Indicar los diagnósticos más probables:

(1) hernia crural


(2) hernia estrangulada
(3) hernia incarcerada
(4) hernia de Spiegel
(5) hernia inguinal directa
(6) hernia inguinal indirecta
(7) eventración

a.- una mujer de 73 años, encontrándose previamente bien, refiere la


aparición brusca de una tumoración en región inguinal derecha, debajo del
ligamento inguinal, con motivo de un acceso de tos. Tiene dolor a dicho
nivel y dolores de características cólicas por todo el abdomen. Seleccionar
dos diagnósticos *(1) (2) *(3) (4) (5) (6) (7)
b.- un paciente de 75 años ha sido sometido a una hemicolectomía
izquierda por un cáncer de colon perforado hace tres años. Presenta una
tumoración blanda en mesogastrio. Seleccionar un diagnóstico
(1) (2) (3) (4) (5) (6) *(7)
c.- un varón de 35 años presenta una tumoración blanda inguino-escrotal,
que en la palpación aparece a través del orificio inguinal superficial.
Seleccionar dos diagnósticos. (1) (2) (3) (4) *(5) *(6) (7)
d.- Una paciente presenta signos de obstrucción intestinal y una masa en la
pared abdominal infraumbilical, derecha, lateral al músculo recto
Seleccionar dos diagnósticos (1) (2) *(3) *(4) (5) (6) (7)
e.- Un paciente de 50 años refiere la aparición brusca de una tumoración de
unos 5 cms. en región inguinal derecha, acompañándose de dolor intenso.
El cuadro le dura aproximadamente una hora y posteriormente la
tumoración desaparece cediendo el dolor local. Realiza sin embargo una
deposición con sangre tres horas después. Seleccionar tres diagnósticos.
(1)*(2) (3) (4) *(5) *(6) (7)

9.- Respecto a las evisceraciones y eventraciones cual de las siguientes


afirmaciones es FALSA
a.- las eventraciones siempre son posquirúrgicas
*b.- la evisceración siempre implica estrangulación de asas
c.- la evisceración puede ocurrir con motivo de una herida abdominal por
arma blanca
d.- en las eventraciones no hay solución de continuidad en el peritoneo
e.- en las evisceraciones siempre hay solución de continuidad en el
peritoneo.

10.- Cuales de las siguientes situaciones se pueden considerar como factor


de riesgo elevado de eventración:
a.- obesidad *(si) (no)
b.- utilización de material de sutura reabsorbible (si) *(no)
c.- paciente con peritonitis *(si) (no)
d.- paciente en tratamiento con corticoides *(si) (no)
e.- paciente con ascitis *(si) (no)

11.- Respecto a las hernias inguinales directas e indirectas señalar la opción


correcta:
a.- en las hernias indirectas el saco herniario sale en el orificio inguinal
profundo por fuera de los vasos epigástricos *(si) (no)
b.- en las hernias directas el saco herniario sale a través del cordón
espermático (si) *(no)
c.- las hernias inguinales directas son las más frecuentes y suponen el 80%
de las hernias inguinales *(si) (no)
d.- en las hernias directas de gran tamaño la vejiga puede estar deslizada en
el saco *(si) (no)
e.- las hernias directas protuyen a través de la fascia transversalis *(si) (no)

Respecto a las imágenes siguientes señalar las opciones correctas:


12.- Hernia umbilical en un niño de tres meses de edad menor de dos cms.
asintomática. ¿Requiere tratamiento quirúrgico antes de que el niño cumpla
un año?

(si) *(no)

13.- A que tipo de hernia corresponde esta imagen (la línea marca el
pliegue cutáneo inguinal):

a.- crural
*b.- crural incarcerada
c.- inguinal indirecta estrangulada
d.- inguinal directa incarcerada
e.- de Spiegel

14.- Paciente de 55 años con aparición progresiva en los últimos tres años
de las dos tumoraciones que aparecen en la imagen. Desaparecen en
posición de decúbito.

a.- se trata de hernias bilaterales


*(Inguinales directas)
(Crurales)
(Inguinoescrotales)
(Indirectas incarceradas)
b.- se deben intervenir con carácter
(Urgente)
*(Programado)
(Se debe esperar y valorar la evolución del tamaño)
c.- cuando se corrijan quirúrgicamente se deben corregir
*(Simultáneamente)
(En dos tiempos)
d.- mientras se decide la intervención se debe aconsejar la utilización de un
braguero
(si) *(no)
e.- la intervención que menores recidivas tiene en este caso, será:
(Intervención de Bassini)
*(Corrección con prótesis bilateral reticular)
(Corrección con plicatura simple de la fascia transversalis)
INFECCION respuestas

1.- Señalar la opción CORRECTA respecto a la clasificación del tipo de cirugía (limpia,
limpia-contaminada, contaminada, sucia) en la siguiente lista de intervenciones:
a.- colocación de prótesis de cadera
*(limpia) (limpia-contaminada) (contaminada) (sucia)
b.- apendicitis aguda supurada
(limpia) (limpia-contaminada) (contaminada) *(sucia)
c.- cirugía reglada de resección de un cáncer de colon sigmoide
(limpia) *(limpia-contaminada) (contaminada) (sucia)
d.- cirugía cardiaca, by-pass coronario.
*(limpia) (limpia-contaminada) (contaminada) (sucia)
e.- traumatismo abdominal con herida perforante de más de cuatro horas de evolución.
(limpia) (limpia-contaminada) (contaminada) *(sucia)

2.- La incidencia de la infección post-operatoria en una cirugía limpia debe ser menor
del:
*a.- 5%
b.- 10%
c.- 15%
d.- 20%
e.- 25%

3.- Sobre el uso de los antibióticos en la profilaxis quirúrgica señalar lo correcto:


a.- el momento más recomendable para administrar el antibiótico es durante la
inducción anestésica *(si) (no)
b.- la vía de elección es la oral (si) *(no)
c.- la dosis debe ser la terapéutica máxima *(si) (no)
d.- el antibiótico elegido debe tener un perfil farmacocinetico adecuado para difundirse
al tejido extravascular *(si) (no)
e.- se debe mantener la administración al menos durante dos días después de la
intervención (si) *(no)

4.- Señalar en cuales de las siguientes intervenciones debe utilizarse un antibiótico


profiláctico con capacidad anaerobicida:
a.- apendicitis aguda *(si) (no)
b.- corrección de una hernia inguinal con prótesis (si) *(no)
c.- cirugía a corazon abierto (si) *(no)
d.- cirugía no urgente de colon *(si) (no)
e.- craneotomía (si) *(no)

5.-¿En una intervención por hernia inguinal sin necesidad de colocar prótesis los
factores siguientes indican la profilaxis antibiótica?
a.- paciente en tratamiento quimioterapico por un cáncer de colon *(si) (no)
b.- paciente en tratamiento radioterapico por un cáncer de próstata *(si) (no)
c.- paciente mayor de 70 años (si) *(no)
d.- paciente VIH + *(si) (no)
e.- paciente con 4000 leucocitos /ml. (si) *(no)
6.- A un paciente de 50 años está previsto practicarle una hemicolectomia derecha por
un cáncer de ciego, no obstructivo y sin metástasis. Cual de las siguientes preparaciones
es la más CORRECTA:
a.- enemas de limpieza 24 h. antes de la intervención
b.- antibioterapia profiláctica con metronidazol y gentamicina solamente.
*c.- solución evacuante 24 h. antes de la intervención y antibioterapia profiláctica con
metronidazol y gentamicina
d.- antibioterapia profiláctica con amoxicilina+ clavulanico solamente
e.- en principio este paciente no precisa preparación especifica

7.- Los siguientes factores facilitan la flora nosocomial en un periodo postoperatorio


EXCEPTO:
a.- colonización de las vías respiratorias altas
b.- perdida de la barrera ácida gástrica
*c.- edad avanzada del paciente
d.- uso previo de antibióticos
e.- ileo paralítico postoperatorio

8.- En la cirugía gastroduodenal en cuales de las siguientes situaciones está indicada la


profilaxis antibiotica:
a.- uso de anti H2 durante varios meses *(si) (no)
b.- cáncer gástrico *(si) (no)
c.- estenosis pilórica *(si) (no)
d.- anemia perniciosa *(si) (no)
e.- cirugía de la obesidad *(si) (no)
Respuestas NUTRICIÓN PARENTERAL Y ENTERAL

1.- Respecto al concepto de desnutrición en el paciente quirúrgico señalar lo correcto:


a.- el paciente desnutrido mantiene su masa muscular (si) *(no)
b.- la desnutrición cursa con hiperpotasemia (si) *(no)
c.- la situación de desnutrición interfiere con los mecanismos endocrino- metabólicos
de adaptación al stress *(si) (no)
d.- las complicaciones posquirúrgicas aumentan hasta un 70% en los pacientes con
desnutrición grave *(si) (no)
e.- en la composición corporal del paciente desnutrido existe un exceso relativo de agua
*(si) (no)

2.- Cual de las siguientes determinaciones analíticas tiene más valor para determinar el
estado nutricional preoperatorio de un paciente:
a.- hemoglobina
b.- hematocrito
*c.- proteína ligada al retinol
d.- albúmina
e.- proteínas totales

3.- La fórmula de Harris-Benedict, en el paciente quirúrgico, es un método de cálculo


de:
*a.- las necesidades energéticas totales basales
b.- las necesidades proteicas del paciente postoperado
c.- el balance nitrogenado
d.- la distribución relativa de calorías entre hidratos de carbono, proteínas y grasas en
las necesidades postagresión.
e.- la relación entre consumo de O2 y producción de CO2

4.- Sobre las siguientes afirmaciones señalar lo correcto:


a.- la formula de Harris Benedict es el mejor método de medida de la situación
nutricional preoperatoria de un paciente (si) *(no).
b.- las necesidades calóricas basales de un paciente en reposo son de
25 a 35 /kcal./kgr./día *(si) (no)
c.- el GEB (gasto energético basal) depende del nivel de stress *(si) (no)
d.- la calorimetría indirecta relaciona el consumo de O2 y la producción de CO2
*(si) (no)
e.- la insuficiencia renal es una situación que aumenta los requerimientos energéticos de
un paciente *(si) (no)

5.- Respecto a los requerimientos energéticos del paciente quirúrgico señalar la


afirmación FALSA
a.- los lípidos proveen en general 9 Kcal./gr.
*b.- en una situación de stress alto las reservas de glucógeno se agotan a partir de la
primera semana
c.- las necesidades mínimas de carbohidratos son de 100-150 g/día.
d.- las necesidades mínimas de proteínas son 0.8-1 g/kg. de peso/día.
e.- las reservas de glucógeno del organismo se almacenan en hígado y músculo
principalmente.
6.- Respecto a la Glutamina lo siguiente es cierto EXCEPTO:
a.- es indispensable para el metabolismo de los enterocitos
b.- es el amino ácido más abundante en la sangre
*c.- es un aminoácido esencial
d.- puede ser necesario para la proliferación de linfocitos y macrófagos
e.- juega un papel importante en el mantenimiento de la mucosa intestinal durante el
stress.

7.- Señalar la opción correcta cuando nos referimos a la comparación entre nutrición
enteral y parenteral
1.- la incidencia y riesgo de sepsis son menores en la nutrición enteral *(si) (no)
2.- la nutrición parenteral es más barata que la nutrición enteral (si) *(no)
3.- la nutrición enteral tiene menos complicaciones graves *(si) (no)
4.- la nutrición parenteral disminuye la traslocación bacteriana más que la nutrición
enteral (si) *(no)
5.- la nutrición enteral mejora mas que la parenteral la función inmunitaria *(si) (no)

8.- En cual de las siguientes situaciones NO ESTA INDICADA la nutrición enteral


administrada por yeyunostomia:
a.- pancreatitis aguda
*b.- enfermedad inflamatoria intestinal en fase aguda
c.- paciente con adenocarcinoma de esófago
d.- paciente séptico
e.- paciente en coma neurológico

9.- Respecto a la nutrición parenteral una de las siguientes afirmaciones es FALSA:


a.- la aportación de proteínas debe realizarse en función del grado de stress del paciente
b.- la relación gr. de Nitrógeno proteico / calorías no proteicas en la solución aportada
debe ser de 1/ 100 aproximadamente.
c.- una solución parenteral que aporte al menos 2500 Kcal./día requiere siempre un
acceso venoso central
*d.- para aportar 3000 Kcal./día mediante soluciones parenterales es suficiente un
volumen de 1000 c.c.
e.- en la nutrición parenteral total las calorías se aportan con carbohidratos y grasas
principalmente.

10.- En las siguientes patologías puede estar indicado un soporte nutricional. Elegir la
forma (parenteral o enteral) más adecuada:
a.- pancreatitis aguda (parenteral) *(enteral)
b.- dehiscencia de sutura después de una resección por cáncer de colon
*(parenteral) (enteral)
c.- paciente con enfermedad de Crohn *(parenteral) (enteral)
d.- paciente quemado (parenteral) *(enteral)
e.- paciente en postoperatorio de una peritonitis por apendicitis aguda
*(parenteral) (enteral)
11.- Respecto a la nutrición enteral una de las siguientes afirmaciones es FALSA:
a.- la alimentación por sonda gástrica está contraindicada en pacientes con riesgo de
aspiración
b.- las dietas por yeyunostomia deben tener una presión osmótica baja.
c.- la alimentación por sonda gástrica permite la utilización de dietas naturales
*d.- en caso de obstrucción intestinal está indicada la vía nasoyeyunal
e.- la deshidratación por diarreas es una complicación de las dietas demasiado
hiperosmolares.

12.- Respecto a los carbohidratos en relación con los requerimientos energéticos de un


paciente quirúrgico señalar las opciones correctas
a.- generalmente aportan entre el 30-60% de las calorías totales *(si) (no)
b.- es necesario un mínimo de 150 grs./día de glucosa para las necesidades
indispensables de cerebro y células sanguíneas *(si) (no)
c.- solamente el 10% de la glucosa en forma de glucógeno se almacena en el músculo
esquelético (si) *(no)
d.- las reservas totales de glucógeno son de aproximadamente 5 kgrs. en un adulto de 70
kgrs de peso. (si) *(no)
e.- las reservas totales de glucógeno pueden servir para las necesidades de
aproximadamente 10 días en un postoperatorio normal (si) *(no)

13.- Respecto a los requerimientos de proteínas señalar lo correcto:


a.- las necesidades normales en el individuo sano son de 0.8-1 gr. /kg/día *(si) (no)
b.- los pacientes con insuficiencia renal requieren mas proteínas *(si) (no)
c.- en situación de encefalopatía hepática se debe aumentar el aporte proteico (si) *(no)
d.- el balance nitrogenado es la medida en bruto de consumo de proteínas *(si) (no)
e.- el soporte nutricional artificial está indicado cuando el balance nitrogenado es
positivo (si) *(no)

14.- Sobre los aminoácidos (AA) y el aporte nutricional artificial:


a.- Por lo menos el 20% del aporte proteico debe ser en forma de AA esenciales
*(si) (no)
b.- Los AA esenciales son los que puede producir el organismo (si) *(no)
c.- La glutamina es la principal fuente energética de los enterocitos *(si) (no)
d.- La leucina y la isoleucina son AA esenciales *(si) (no)
e.- La glutamina es un AA esencial (si) *(no)

15.- Cual de las siguientes afirmaciones es INCORRECTA sobre los lípidos en la


nutrición artificial:
a.- Los lípidos deben aportar al menos 25% del total de los requerimientos calóricos
durante la nutrición suplementaria
*b.- Los triglicéridos de cadena larga son los que se absorben directamente por los
enterocitos
c.- Las pancreatitis severas afectan a la absorción de triglicéridos
d.- El ácido linoleico es ácido graso esencial
e.- La falta de aporte correcto de ácidos grasos puede dar manifestaciones clínicas a
partir de la cuarta semana en un paciente con nutrición parenteral total.
PATOLOGIA QUIRÚRGICA DEL ESOFAGO.
RESPUESTAS.

1.- Un paciente de 70 años tiene un cuadro de disfagia, regurgitaciones de comida no


digerida y halitosis crónica. Presenta el esófagograma siguiente:

a.- ¿para establecer el diagnóstico de divertículo de Zenker es necesaria la manometría?


(si) *(no)
b.- el divertículo de Zenker es el divertículo más frecuente del esófago *(si) (no)
c.- generalmente aparece en edades entre 30 y 40 años (si) *(no)
d.- se localiza siempre por debajo del músculo cricofaringeo (si) *(no)
e.- es un divertículo de pulsión *(si) (no)

2.- Respecto al divertículo de Zenker lo siguiente es cierto EXCEPTO:


*a.- la pared del divertículo tiene todas las capas del esófago
b.- su complicación más frecuente son las neumonías por aspiración
c.- el tratamiento quirúrgico consiste en la miotomía del músculo cricofaringeo y la
diverticulectomía
d.- es consecuencia de una incoordinación cricofaringea
e.- suelen aparecer en la cara posterior del esófago.

3.- Cual de las siguientes situaciones manométricas y pHmetricas NO encontraremos en


un paciente con el diagnóstico de achalasia:
a.- esfínter esofágico inferior con presión basal elevada
b.- peristaltismo descoordinado en el cuerpo esofágico
c.- esfínter esofágico superior con presión normal
*d.- reflujo gastroesofágico de 24 h. elevado
e.- prueba de anticolinergicos positiva (test de Mecolil)
4.- Un paciente de 50 años refiere desde hace unos seis meses una historia de disfagia
para sólidos y líquidos así como regurgitaciones de comida sin digerir al acostarse. El
estudio baritado esofágico es el siguiente:

Se establece un diagnóstico de presunción de achalasia.


¿Qué consideraciones respecto a este paciente son correctas?

a.- la primera prueba complementaria que debe hacerse es una endoscopia *(si) (no)
b.- la manometría es necesaria aunque la radiología, endoscopia y clínica sean típicas
*(si) (no)
c.- el tratamiento médico con nitratos e inhibidores de los canales del Calcio es eficaz en
más del 50% de los casos (si) *(no)
d.- la dilatación endoscópica neumática es eficaz en más del 90% de los casos (si) *(no)
e.- sería lógico que este paciente refiriera también reflujo ácido gastroesofágico
(si) *(no)

5.- Las siguientes afirmaciones respecto a la achalasia son correctas EXCEPTO:


a.- en la mayoría de los casos las células ganglionares del cuerpo esofágico están
ausentes o han sufrido un proceso degenerativo
*b.- en el cuerpo esofágico la presión es tres veces menor que los valores normales
c.- la incidencia mayor es en personas entre 30 y 50 años.
d.- los alimentos fríos provocan mayor disfagia que los alimentos templados o calientes.
e.- la incidencia de cáncer de esófago es mayor en los pacientes con megaesofago

6.- La imagen siguiente corresponde a una miotomia de Heller realizada en un paciente


con diagnóstico de achalasia. Señalar las estructuras correspondientes a los números de
la imagen:

a.- mucosa esofagica (1) *(2) (3) (4) (5)


b.- pilar diafragmatico izquierdo *(1) (2) (3) (4) (5)
c.- borde de la capa muscular esofagica seccionada (1) (2) *(3) (4) (5)
d.- fundus gástrico (1) (2) (3) (4) *(5)
e.- nervio vago anterior (1) (2) (3)*(4) (5)

7.- Las siguientes estructuras están implicadas en la deglución EXCEPTO:


a.- músculo cricofaringeo
b.- epiglotis
c.- paladar blando
*d.- esfínter esofagico inferior
e.- lengua

8.- Los siguientes factores facilitan la relajación del esfínter esofagico inferior
EXCEPTO
a.- colecistikinina
b.- anticolinergicos
c.- etanol
*d.- gastrina
e.- nicotina

.- Preguntas 9 a 12.
Seleccionar para los casos clínicos que se exponen en cada una de las preguntas los
diagnósticos (se pueden proponer más de uno) más probables de la lista siguiente:
Enfermedad por reflujo gastroesofágico (a)
Leiomioma esófago (b)
Divertículo de Zenker (c)
Hernia de hiato por deslizamiento (d)
Hernia de hiato paraesofagica (e)
Cáncer de esófago (f)
Achalasia (g)
Estenosis peptica de esófago (h)

9.- Un paciente de 40 años con historia antigua de pirosis retroesternal, que no precisó
tratamiento, refiere desde hace un año disfagia para alimentos sólidos y desde hace dos
meses también para líquidos. El esofagograma muestra una estenosis (ver figura) que en
la biopsia ha resultado ser benigna.

Seleccionar dos diagnósticos para este paciente


*a, b, c, d, e, f, g,*h

10.- Un paciente varón de 58 años presenta una historia de disfagia para líquidos y
sólidos y regurgitaciones intermitentes de comida sin digerir
Seleccionar dos diagnósticos para este paciente
a, b,*c, d, e, f,*g, h

11.- Un paciente varón de 75 años de edad refiere episodios de disnea de esfuerzo con
sensación de opresión retroesternal. Los estudios cardiológicos han sido normales.
Presenta la radiografía de tórax lateral de la imagen siguiente .

Seleccionar un diagnóstico para este paciente.


a, b, c, d, *e, f, g, h

12.- Un paciente de 56 años presenta una historia de cuatro episodios de neumonía en


lóbulo inferior derecho en el ultimo año. Tambien refiere episodios de tos seca
nocturna. Los estudios pulmonares incluida broncoscopia han sido normales. No es
fumador. El estudio radiológico con bario del esófago ha sido normal.
Seleccionar un diagnóstico para este paciente *a, b, c, d, e, f, g, h

13.- Todas las afirmaciones siguientes son ciertas respecto al anillo de Schatzki
EXCEPTO:
a.-. en muchos pacientes existe una relación entre la existencia del anillo y la ingestión
de ciertos fármacos.
b.- el tratamiento del anillo de Schaztki, si no se precisa tratar un reflujo asociado es la
dilatación forzada endoscópica
*c.- el anillo de Schatzki es una lesión precancerosa
d.- el síntoma principal es la disfagia de corta duración
e.- el anillo de Schaztki es un anillo delgado submucoso en el tercio distal del esófago.

14.- La imagen siguiente corresponde a la corrección quirúrgica de una hernia hiatal


Señalar las opciones correctas:
a.- se trata de una intervención de Nissen con fundoplicatura de 360º *(si) (no)
b.- los pilares diafragmáticos se han aproximado por delante del esófago (si) *(no)
c.- uno de los objetivos es conseguir reponer en el abdomen un segmento de esófago lo
más amplio posible. *(si) (no)
d.- se debe asociar siempre una vagotomia y piloroplastia en este tipo de intervención
(si) *(no)
e.- la fundoplicatura puede producir disfagia si queda demasiado apretada *(si) (no)

15.- Entre las complicaciones de las hernias hiatales están :


a.- incarceración
b.- vólvulo gástrico
c.- estenosis péptica por reflujo
d.- hemorragia por ulcera intraherniaria
*e.- todo lo anterior puede ocurrir

16.- El síndrome de Mallory Weiss consiste en :


*a.- hemorragia por ulceras en el tercio distal del esófago
b.- incoordinacion motora del cuerpo esófagico y cardias
c.- mucosa gástrica metaplasica en tercio distal de esófago
d.- perforación espontanea del esófago
e.- estenosis péptica del esófago

17.- Un paciente con disfagia, dolor retroesternal y voz bitonal, probablemente tendrá :
a.- una achalasia
b.- un diverticulo de Zenker
*c.- un carcinoma de tercio medio esofágico
d.- una estenosis péptica
e.- un espasmo difuso esofágico

18.- Cuales de las siguientes afirmaciones respecto al carcinoma epidermoide de


esófago son ciertas:
a.- el síntoma inicial más frecuente es la disfagia *(si) (no)
b.- el tabaquismo se considera un factor etiologico importante *(si) (no)
c.- la localización más frecuente es el tercio medio del esófago *(si) (no)
d.- la capa serosa del esófago evita la propagación temprana del tumor hacia el
mediastino (si) *(no)
e.- suele asentar sobre un esófago de Barret (si) *(no)
19.- Cuales de las siguientes afirmaciones son correctas respecto al adenocarcinoma de
esófago:
a.- la mayoria de ellos se diagnostican precozmente (si) *(no)
b.- la quimioterapia y la radioterapia por si solas son más eficaces que la cirugía en su
tratamiento (si) *(no)
c.- los adenocarcinomas de tercio distal suelen metastatizar en los ganglios cervicales
(si) *(no)
d.- es excepcional que produzcan metástasis hepáticas (si) *(no)
e.- se considera que la mayoría de estas lesionas asientan sobre un esófago de Barret
*(si) (no)

20.- Un paciente de 47 años presenta un cuadro de disfagia moderada para sólidos desde
hace seis meses. No refiere perdida de peso ni anemia. En la endoscopia (ver imagen) se
aprecia una tumoración submucosa, no ulcerada a nivel de tercio medio, muy
sospechosa de leiomioma. Señalar las consideraciones correctas sobre este caso:

a.- antes de proceder al tratamiento quirúrgico es obligado realizar una biopsia


endoscópica (si) *(no)
b.- el tratamiento quirúrgico debe consistir en la enucleación del tumor sin necesidad de
resección esofágica. *(si) (no)
c.- generalmente son tumores múltiples en el 50% de los casos
d.- aunque fuera un hallazgo se debería resecar por su alto poder de malignización
(si) *(no)
e.- la disfagia es el síntoma principal *(si) (no)

21.- Las siguientes consideraciones se pueden aplicar al cáncer de esófago


EXCEPTO :
*a.- su localización habitual es el tercio superior
b.- la edad de máxima incidencia es 50-70 años
c.- la relación varón/hembra es 3/1 aproximadamente
d.- su síntoma principal es la disfagia
e.- en tercio medio y superior más del 80% son carcinomas epidermoides
PATOLOGIA QUIRÚRGICA DEL TÓRAX. Respuestas.

1.-En un paciente politraumatizado, diagnosticado de volet costal, señalar lo correcto:


a.- en pequeños defectos de la zona posterior del tórax, se puede mantener una actitud
conservadora, sin realizar fijación, dependiendo de la repercusión que produzcan.
*(si) (no)
b.- los volet torácicos anteriores son los que requieren con mayor frecuencia fijación
quirúrgica inmediata *(si) (no)
c.- dependiendo de la situación clínica del paciente la intubación con respiración
asistida es una alternativa válida a la fijación quirúrgica de urgencia *(si) (no)
d.- la característica clínica del volet costal es el movimiento paradójico de la parte de la
pared torácica afectada *(si) (no)
e.- en situación urgente la colocación de un tubo de tórax con aspiración soluciona
momentáneamente la insuficiencia respiratoria del paciente (si) *(no)

2.- Son indicaciones de toracotomía de urgencia en el contexto de un traumatismo


torácico:
a.- Traumatismos abiertos, como las heridas por arma de fuego *(si) (no)
b.- Cuando se asocian un volet costal con un hemo-neumotórax. *(si) (no)
c.- Cuando tras colocar tubo torácico, el drenaje inicial de sangre es superior
a los 1500 c.c. *(si) (no)
d.- Todos los hemotórax, independientemente de su cuantía, cuando no se conoce el
punto de sangrado. (si) *(no)
e.- Una situación de enfisema subcutáneo, con sospecha de rotura traqueal *(si) (no)

3.- Respecto al neumotórax espontáneo, señalar lo correcto:


a.- Cuando se produce en jóvenes, suele ser producido por la rotura espontánea de
"blebs" pleurales. *(si) (no)
b.- El neumotórax espontáneo tiene tendencia a la recidiva, y en ciertos casos es
necesario realizar una toracotomía o cirugía toracoscópica para un control definitivo de
la enfermedad. *(si) (no)
c.- El tratamiento inicial debe ser un tubo de drenaje torácico solamente con sellado
bajo agua, y que no debe llevar aspiración (si) *(no)
d.- Una de las formas de presentación más frecuentes en jóvenes, es el "neumotórax a
tensión", que se corrige con la colocación inmediata de un drenaje torácico *(si) (no)
e.- El neumotórax espontáneo secundario es el que ocurre en enfermos con patología
broncopulmonar previa y más frecuentemente se asocia a la presencia de bullas
pulmonares. *(si) (no)

4.- Respecto a los traumatismos torácicos, una de las siguientes respuestas es falsa:
*a.-Las fracturas esternales precisan siempre corrección quirúrgica de la deformidad.
b.-Los fragmentos de costillas pueden provocar dislaceraciones pulmonares y
neumotórax.
c.-La contusión cardiaca se asocia con frecuencia a fracturas esternales y es crítica la
observación hospitalaria, la vigilancia de alteraciones electrocardiográficas y en ciertos
casos la realización de ecocardiograma.
d.-La contusión pulmonar se manifiesta mediante la aparición en la radiografía de tórax
de infiltrados alveolares.
e.-La fractura de la primera costilla se suele asociar a traumatismos de gran energía y es
preciso descartar otras lesiones asociadas.
5.- Respecto a los drenajes torácicos, señalar lo correcto
a.- es necesario siempre colocarlos bajo anestesia general (si) *(no)
b.- se conectan a un dispositivo que básicamente consiste en un sello de agua o válvula
unidireccional que solo permite la salida de liquido o aire de la cavidad pleural
*(si) (no)
c.- cuando se colocan para drenar contenido liquido del espacio pleural se introducirán
entre el segundo y tercer espacio intercostal (si) *(no)
d.- es indispensable disponer de una radiografía de tórax para colocarlos (si) *(no)
e.- están contraindicados en los traumatismos abiertos de tórax *(si) (no)

6.- Respecto al mesotelioma difuso maligno señalar la afirmación CORRECTA:


*a.- Se relaciona con la inhalación de fibras de asbesto
b.- La forma clínica de presentación más frecuente es la insuficiencia respiratoria brusca
c.- Se alcanza solamente con cisplatino una supervivencia del 85% a los cinco años
d.- El tratamiento quirúrgico permite una supervivencia del 95 % a los tres años
e.- El diagnostico diferencial fundamental es con las bronquiectasias.

7.- Ante un cáncer bronquial T2 N0 M0 confirmado por mediastinoscopia se debe:


*a.- plantear el tratamiento quirúrgico directamente
b.- tratar solamente con quimioterapia
c.- tratar solamente con radioterapia
d.- tratar con quimioterapia neoadyuvante e intervenir posteriormente si cumple
criterios de operabilidad.
e.- tratar con radioterapia preoperatoria y posterior resección.

8.- La tricoptisis es patognomónica de:


a.- timoma
b.- teratoma
c.- linfoma
d.- neurinoma
e.- carcinoma bronquial.

9.- Cual de los siguientes hallazgos se puede encontrar en la radiografía de tórax de un


paciente con rotura de aorta torácica después de un accidente de tráfico (señalar lo
correcto):
a.- Ensanchamiento mediastínico (más de 8 cms. en su porción superior)
*(si) (no)
b.- Pérdida de nitidez del botón aórtico *(si) (no)
c.- Derrame pleural izquierdo *(si) (no)
d.- Neumomediastino (si) *(no)
e.- Casquete apical izquierdo *(si) (no)

10.- En un paciente con un carcinoma epidermoide de pulmón derecho el dato que


indica peor pronóstico y probable irresecabilidad quirúrgica es:
a.- tos crónica
b.- hemoptisis
*c.- ronquera y voz bitonal
d.- neumonitis obstructiva
e.- dolor pleurítico
11.- ¿Las manifestaciones siguientes de un carcinoma de pulmón indican la existencia
de metástasis e inoperabilidad ?
a.- hipercalcemia (si) *(no)
b.- síndrome cushingoide (si) *(no)
c.- síndrome similar a miastenia (si) *(no)
d.- ginecomastia moderada (si) *no)
e.- secreción alterada de hormona antidiurética (si) *(no)

12.- Cual de las siguientes afirmaciones es CORRECTA respecto al carcinoide


bronquial:
a.- es avascular
b.- se localiza generalmente en bronquiolos finos
*c.- es raro que provoque metástasis a distancia
d.- es muy radiosensible
e..- cuando se presenta el síndrome carcinoide es incurable

13.- En un paciente asintomático se encuentra un nódulo pulmonar solitario de 2 cms.


de diámetro. Los estudios radiológicos muestran que el nódulo tiene un núcleo central
de calcio. La actitud correcta con esta lesión es:
*a.- seguimiento periódico con estudios radiológicos.
b.- biopsia transbronquial
c.- biopsia percutánea dirigida con TAC
d.- tratamiento con anfotericina
e.- toracotomía y resección

14.- Una paciente con sarcoma osteogénico de fémur, extirpado dos años antes, presenta
dos lesiones metastásicas pequeñas en el pulmón derecho y otra en el pulmón izquierdo.
Si no se encuentran otras lesiones en ningún otro órgano el tratamiento más correcto
sería:
*a.- resección bilateral de las lesiones
b.- quimioterapia solamente
c.- radioterapia solamente
d.- radioterapia y quimioterapia conjuntamente
e.- observación durante tres meses para valorar evolución y decidir después el
tratamiento.
RESPUESTAS
QUEMADURAS, LESIONES POR EFECTO EXPLOSIVO
SINDROME DE APLASTAMIENTO.

1.- Una quemadura que afecta a toda la parte anterior del tronco se puede calcular que
afecta a un porcentaje de la superficie corporal de:
a.- 9 %
*b.- 18 %
c.- 45 %
d.- 5 %
e.- 33 %

2.- Las siguientes características corresponden a una quemadura epidérmica:


a.- eritema *(si) (no)
b.- pigmentación melánica temporal *(si) (no)
c.- curación espontánea en 5-10 días *(si) (no)
d.- aparición de flictenas (si) *(no)
e.- cicatriz retráctil (si) *(no)

3.- Las siguientes características corresponden a una quemadura dérmica superficial:


a.- alcanzan la dermis con supervivencia de la capa basal *(si) (no)
b.- suelen tener una curación rápida en 10-15 días *(si) (no)
c.- son características las flictenas *(si) (no)
d.- dejan siempre una cicatriz retráctil (si) *(no)
e.- requieren siempre tratamiento quirúrgico (si) *(no)

4.- Las quemaduras subdérmicas o de tercer grado se caracterizan por:


a.- afectan a todo el espesor de la piel destruyendo todos los anejos cutáneos *(si) (no)
b.- es característico que se formen escaras *(si) (no)
c.- son indoloras por destrucción de las terminaciones nerviosas *(si) (no)
d.- dejan siempre cicatriz *(si) (no)
e.- requieren siempre tratamiento quirúrgico para su curación *(si) (no)

5.- Respecto a la fisiopatología de las quemaduras señalar lo correcto:


a.- en la zona quemada la destrucción de la piel produce una disminución de la
evaporación del agua corporal (si) *(no)
b.- la perdida de calor se calcula en unas 0,575 Kcal. por gr. de agua
evaporada *(si) (no)
c.- en la zona quemada se produce un aumento de la permeabilidad de los vasos
*(si) (no)
d.- se considera que una quemadura está infectada cuando el número de bacterias por
gr. de tejido es superior a 105 *(si) (no)
e.- entre las consecuencias locales se pueden producir trombosis vasculares *(si) (no)

6.- La rehidratación del paciente quemado tiene que tener en cuenta:


a.- Peso del paciente
b.- Superficie corporal quemada
c.- Tipo de quemadura
d.- Diuresis
*e.- Todas las anteriores
7.- En las lesiones por efecto explosivo señalar si las siguientes afirmaciones son
correctas:
a.- el choque de la onda expansiva produce lesiones por el siguiente orden:
1º lesiones de aceleración
2º lesiones de astillamiento
3º lesiones de implosión *(si) (no)
b.- las diferencias de presión que causa una onda expansiva producen las mayores
lesiones en las interfases aire/liquido de los tejidos *(si) (no)
c.- las lesiones de implosión producen los mayores daños en los tejidos sólidos.
(si) *(no)
d.- el llamado “pulmón de blast” se produce por rotura de la membrana alveolo-capilar
*(si) (no)
e.- se llaman lesiones terciarias a las producidas por los desplazamientos bruscos que
puede sufrir el cuerpo por causa de la onda expansiva *(si) (no)

8.- Respecto a las lesiones por efecto explosivo subacuaticas señalar la respuesta
INCORRECTA:
a.- con explosión equivalente, la presión en el agua es mayor que en el aire
b.- las lesiones de estallido de vísceras huecas abdominales son más frecuentes que en
las explosiones aéreas.
*c.- las ondas de choque acuáticas viajan a menor distancia que en el medio aéreo
d.- las roturas del tímpano son las lesiones más frecuentes.
e.- pueden existir, igual que en las lesiones por blast aéreo, lesiones de carácter
secundario.

9.- En el síndrome de aplastamiento señalar la afirmación INCORRECTA:


a.- se calcula un tiempo de cuatro horas para que se produzca el síndrome de
aplastamiento.
b.- la insuficiencia renal se debe a la liberación de mioglobina por el músculo lesionado
*c.- la insuficiencia renal se manifiesta siempre con poliuria
d.- el síndrome de aplastamiento se suele manifestar unas horas después de liberar el
miembro aplastado
e.- la actitud terapéutica de las zonas afectadas debe ser la descompresión progresiva.

10.- Señalar la opción correcta respecto a las alteraciones hidroelectroliticas del


síndrome de aplastamiento:
1.- hiperpotasemia *(si) (no)
2.- hipernatremia (si) *(no)
3.- elevación de creatinina superior a 3 mgrs % es signo de mal pronóstico *(si) (no)
4.- hipercalcemia (si) *(no)
5.- elevación de creatinfosfoquinasa *(si) (no)
SHOCK SRIS (Respuestas)

1.- En un shock cardiogénico NO se producirá:


a.- insuficiencia ventricular
* b.- resistencia vascular disminuida
c.- presión venosa central elevada
d.- menor gasto cardiaco
e.- hipoxia tisular

2.- Un paciente con una hemorragia digestiva de 1000 c.c. tendrá EXCEPTO:
* a.- bradicardia
b.- sudoración fría
c.- oliguria
d.- palidez
e.- taquipnea

3.- ¿Qué tienen en común todos los tipos de shock?


* a.- una mala oxigenación celular en tejidos vitales
b.- un fallo de bomba cardiaca
c.- unas resistencias periféricas elevadas
d.- una situación de sepsis
e.- una disminución de la presión venosa central

4.- Cuando ocurre una hemorragia aguda importante la precarga adecuada al corazón se
intenta mantener inicialmente por:
a.- desarrollo de taquicardia
b.- efectos hormonales de angiotensina
c.- efectos hormonales de hormona antidiurética
d.- efectos hormonales de renina
* e.- incremento de la resistencia vascular

5.- El shock neurogénico se caracteriza por la presencia de:


a.- piel fría y húmeda
b.- incremento del gasto cardiaco
* c.- disminución de la resistencia vascular periférica
d.- elevación de la presión venosa central
e.- hipertensión

6.- Los efectos sistémicos de los “mediadores inflamatorios” juegan un papel


fundamental en el shock …:
a.- cardiogénico
b.- hipovolémico
* c.- séptico
d.- neurogénico
e.- en todos los tipos de shock juegan un papel importante
7.- En un shock hipovolémico encontraremos (señalar la opción correcta)
a.- elevación de las resistencias vasculares periféricas *(si) (no)
b.- disminución del gasto cardiaco *(si) (no)
c.- elevación de la presión venosa central (si) *(no)
d.- hipertensión arterial (si) *(no)
e.- taquipnea *(si) (no)

8.- En un shock séptico encontraremos (señalar la opción correcta)


a.- elevación de las resistencias vasculares periféricas (si) *(no)
b.- disminución del gasto cardiaco (si) *(no)
c.- elevación de la presión venosa central (si) *(no)
d.- hipertensión arterial (si) *(no)
e.- taquipnea *(si) (no)

9.- En un shock neurogénico encontraremos (señalar la opción correcta)


a.- elevación de las resistencias vasculares periféricas (si) *(no)
b.- disminución del gasto cardiaco *(si) (no)
c.- elevación de la presión venosa central (si) *(no)
d.- hipertensión arterial (si) *(no)
e.- taquicardia *(si) (no)

10.- Un paciente de 19 años ingresa de urgencia con un traumatismo cerrado en


hemitórax izquierdo con fracturas costales, ocurrido dos horas antes. Está frío y su TA
es de 50/30 mm Hg. Seleccionar DOS opciones diagnósticas como las más probables
entre las siguientes:
* a.- shock hipovolémico por hemotórax
* b.- shock cardiogénico por taponamiento cardiaco
c.- shock séptico por infección de sangre en cavidad pleural
d.- shock neurológico por dolor
e.- shock cardiogénico por isquemia coronaria por bajo gasto cardiaco

11.- El shock es un síndrome precipitado por una alteración de la perfusión efectiva a


nivel tisular, pero sin embargo en uno de los tipos siguientes de shock la perfusión
global puede ser normal:
a.- shock cardiogénico
b.- traumático
* c.- séptico
d.- neurológico
e.- hemorrágico

12.- Cuales de las siguientes afirmaciones son correctas respecto a las consecuencias de
la hipoxia celular en el shock :
a.- aumento de los depósitos de ATP celulares (si) *(no)
b.- disminución de los depósitos de fosfocreatina *(si) (no)
c.- alteración de las bombas iónicas de membrana * (si) (no)
d.- presencia de edema intracelular *(si) (no)
e.- alteración del calcio intracelular *(si) (no)
13.- Cual de las siguientes situaciones puede causar un shock cardiogénico
extracardiaco por aumento de la postcarga:
a.- obstrucción de la vena cava inferior
b.- aumento de la presión intratorácica por un neumotórax
c.- taponamiento pericárdico
*d.- aneurisma disecante de la aorta torácica
e.- pericarditis constrictiva

14.- Cual de los siguientes tipos de shock NO se debe considerar como un “shock
distributivo”:
a.- séptico
b.- tóxico
c.- anafiláctico
*d.- hemorrágico
e.- neurogénico

15.- Cuales afirmaciones son correctas respecto a los mecanismos hacia la


irreversibilidad en la descompensación del shock:
a.- la vasodilatación de los esfínteres precapilares favorece la hipotensión *(si) (no)
b.- la liberación de prostaglandinas es un mecanismo vasoconstrictor (si) *(no)
c.- la inducción del oxido nítrico es un mecanismo vasodilatador *(si) (no)
d.- la histamina y bradikinina son mediadores que producen un incremento de la
permeabilidad capilar *(si) (no)
e.- los radicales libres de O2 son protectores del endotelio (si) *(no)

16.- Cuales de las siguientes afirmaciones son correctas respecto a los efectos del shock
sobre diferentes órganos:
a.- riñón: existe una insuficiencia renal prerenal en la primera fase del shock *(si) (no)
b.- hígado: disminución de bilirrubina, transaminasas y fosfatasa alcalina (si) *(no)
c.- hígado: patrón de necrosis centrolobulillar *(si) (no)
d.- tracto gastrointestinal: fenómenos isquémicos y traslocación bacteriana *(si) (no)
e.- pulmón: taquipnea e hipocapnia para compensar en la primera fase la acidosis
metabólica *(si) (no)

17.- Respecto a los criterios diagnósticos del síndrome de distress respiratorio señalar
las opciones correctas:
a.- una manifestación clínica es la taquipnea >25 rpm *(si) (no)
b.- es característica la ausencia de manifestaciones radiológicas (si) *(no)
c.- la PO2 arterial es < a 50 mm de Hg. *(si) (no)
d.- existe un aumento del shunt intrapulmonar y del espacio muerto *(si) (no)
e.- las lesiones anatomopatológicas características son atelectasias congestivas
*(si) (no)
18.- Señalar cuales de los siguientes signos clínicos
(a) taquicardia, (b)taquipnea, (c) oliguria, (d) frialdad de extremidades, (e) fiebre
suelen estar presentes en cada uno de los tipos de shock:
a.- hipovolémico *(a) *(b) *(c) *(d) (e)
b.- neurogénico *(a) *(b) *(c) *(d) (e)
c.- cardiogénico *(a) *(b) *(c) *(d) (e)
d.- séptico *(a) *(b) *(c) (d) *(e)
e.- anafiláctico *(a) *(b) *(c) *(d) *(e)

19.- Cuales de las siguientes pruebas complementarias tienen valor en la confirmación


del diagnóstico de shock:
a.- fórmula y recuento leucocitarios *(si) (no)
b.- gases arteriales *(si) (no)
c.- radiografía de tórax *(si) (no)
d.- creatinina *(si) (no)
e.- estudio de coagulación *(si) (no)

20.- Cual de los siguientes NO es un efecto esperado de la dopamina en el tratamiento


del shock.
a.- efecto inotrópico
b.- elevación de la presión arterial sistémica
c.- mejorar el gasto cardíaco
*d.- disminuir la diuresis
e.- moderado efecto dilatador

21.- En el SRIS (síndrome de respuesta inflamatoria sistémica) la generación de


trombina tendrá como consecuencias EXCEPTO:
a.- aumentar la adhesión leucocitaria
b.- activación de las plaquetas
*c.- aumentar la fibrinolísis
d.- producir trombosis microvascular
e.- generar una estado procoagulante

22.- La lesión del endotelio produce en el SRIS los siguientes hechos EXCEPTO:
a.- liberación de citokinas
b.- liberación de mediadores de vasodilatación
*c.- reabsorción de los coágulos de fibrina
d.- liberación de mediadores inflamatorios
e.- interacciones con los leucocitos

23.- ¿Cuál de los siguientes hechos encontraremos en el SRIS?


a.- retraso en la cicatrización de las heridas
b.- bradicardia
c.- hipotermia
*d.- hiperventilación
e.- plaquetopenia
24.- Respecto a las manifestaciones clínicas y analíticas del SRIS encontraremos:
a.- taquicardia *(si) (no)
b.- vasodilatación *(si) (no)
c.- poliuria *(si) (no)
d.- hipoglucemia *(si) (no)
e.- elevación de transaminasas hepáticas *(si) (no)

25.- ¿Cuáles de las siguientes afirmaciones son correctas en el SRIS?


a.- El Factor de Necrosis Tumoral es un potente mediador antiinflamatorio (si) *(no)
b.- Las interleuquinas 1,6 y 8 son citokinas proinflamatorias *(si) (no)
c.- El primer Estadio del SRIS consiste en la liberación de mediadores inflamatorios a
nivel local *(si) (no)
d.- La proteína C activada promueve la fibrinolisis *(si) (no)
e.- La proteína C se activa por la fijación de la trombina en la superficie endotelial *(si)
(no).
TRASPLANTE (Respuestas)

1.- Aplicar la terminología correcta a cada tipo de trasplante:


Autotrasplante (A) Isotrasplante (B) Alogénico (C) Ortotópico (D) Xenotrasplante (E)
a.- donante y receptor son el mismo individuo *(A) (B) (C) (D) (E)
b.- donante y receptor pertenecen a especies diferentes (A) (B) (C) (D) *(E)
c.- donante y receptor de la misma especie son genéticamente idénticos (A) *(B) (C)
(D) (E)
d.- el órgano del donante ocupa el mismo sitio anatómico que el del receptor
(A) (B) (C) *(D) (E)
e.- donante y receptor de la misma especie son genéticamente diferentes
(A) (B) *(C) (D) (E)

2.- El primer trasplante de hígado lo realizó:


a.- Carrel
*b.- Starzl
c.- Hardy
d.- Lillehei
e.- Borel

3.- Respecto a los antígenos de trasplante señalar lo correcto:


a.- El MHC esta localizado en el cromosoma 6 *(si) (no)
b.- El MHC codifica el HLA *(si) (no)
c.- El MHC II (HLA D) se encuentra en la superficie de todas las células nucleadas del
organismo (si) *(no)
d.- Los antígenos menores de histocompatibilidad no están codificados por los genes del
MHC *(si) (no)
e.- Los antígenos de clase I y II se pueden detectar con pruebas serológicas en
laboratorio *(si) (no)

4.- Respecto al fenómeno del rechazo:


a.- el rechazo agudo se inicia por las células T *(si) (no)
b.- un fenómeno de rechazo agudo es obligatorio que ocurra inmediatamente después
del trasplante (si) *(no)
c.- en el rechazo crónico se mantiene la estructura histológica del órgano trasplantado
(si) *(no)
d.- las infecciones recurrentes contribuyen al rechazo crónico *(si) (no)
e.- el rechazo hiperagudo depende de anticuerpos preformados *(si) (no)

5.- Cual de los siguientes objetivos del tratamiento del rechazo es INCORRECTO:
a.- suprimir las diferencias antigénicas
b.- bloquear las células efectoras de la reacción inmunitaria
*c.- facilitar los mecanismos de comunicación celular
d.- potenciar los mecanismos supresores naturales
e.- modificar la estructura antigénica del injerto
6.- Entre los efectos crónicos secundarios de los fármacos inmunosupresores
generalmente NO está:
*a.- hipotensión
b.- hiperglucemia
c.- nefrotoxicidad
d.- hepatotoxicidad
e.- osteoporosis

7.- Respecto a los fármacos inmunosupresores:


a.- los fármacos antineoplasicos como la azatioprina sirven para bloquear las células
efectoras de la reacción inmunitaria *(si) (no)
b.- los glucocorticoides bloquean los mecanismos de comunicación celular *(si) (no)
c.- la ciclosporina destruye selectivamente loa macrófagos (si) *(no)
d.- los glucorticoides se utilizan en el rechazo agudo y en la terapia de mantenimiento
*(si) (no)
e.- el OKT3 es un anticuerpo antilinfocitario que bloquea las células efectoras de la
reacción inmunitaria *(si) (no)

8.- Respecto al papel de las citoquinas en la reacción inmunológica:


a.- elevan la capacidad de expresión de los receptores celulares reguladores
b.- aumentan la función de las moléculas de adhesión
c.- elevan la susceptibilidad de las células diana a la lisis por los linfocitos T
*d.- inhiben las funciones de los macrófagos, neutrófilos y eosinófilos
e.- alteran la homeostasis metabólica de los tejidos adyacentes

9.- Establecer el orden correcto de los mecanismos de rechazo:


a.- activación de la respuesta inicial 1 *2 3 4
b.- reconocimiento antigénico *1 2 3 4
c.- destrucción del injerto 1 2 3 *4
d.- amplificación (proliferación/maduración) 1 2 *3 4

10.- Respecto a la conservación de órganos señalar la afirmación ERRONEA


a.- la hipotermia pretende paliar los efectos de la hipoxia del órgano
*b.- la congelación no provoca ningún daños celular
c.- el sistema más utilizado en la conservación es la perfusión hipotérmica.
d.- el hígado debe utilizarse en las primeras 24 horas. Como máximo.
e.- la simple conservación en frío tiene un límite más allá del cual el órgano deja de ser
viable.
ABDOMEN AGUDO. PATOLOGIA DEL PERITONEO. OBSTRUCCION
INTESTINAL. (respuestas)

1.- En la etiología de un cuadro de “abdomen agudo” NO estaría uno de los siguientes


procesos:
*a.- hemorragia digestiva por ulcera duodenal
b.- apendicitis aguda
c.- obstrucción intestinal por cáncer de sigma
d.- diverticulitis de sigma perforada
e.- colecistitis aguda

2.- Respecto a las afirmaciones siguientes sobre el término de “abdomen agudo”


señalar lo correcto:
a.- un paciente con abdomen agudo siempre tendrá alteraciones, por exceso o defecto,
del transito intestinal *(si) (no)
b.- un paciente con un abdomen agudo siempre debe ser tratado con criterio de
urgencia
*(si) (no)
c.- pueden existir pacientes con cuadro clínico de abdomen agudo que no tengan dolor
abdominal (si) *(no)
d.- existen patologías extraabdominales que producen un abdomen agudo *(si) (no)
e.- todos los procesos etiológicos que producen un abdomen agudo acaban con el
tiempo afectando a la serosa peritoneal *(si) (no)

3.- Respecto al dolor del abdomen agudo señalar lo correcto:


a.- el signo de Blumberg determina una situación de irritación peritoneal *(si) (no)
b.- el dolor cólico con origen en la vesícula biliar tiene una irradiación hacia
mesogastrio (si) *(no)
c.- el dolor de origen pancreático tiene una irradiación en cinturón *(si) (no)
d.- la distensión de las asas de yeyuno produce un dolor de tipo visceral *(si) (no)
e.- la distensión de la cápsula de Glisson por un hematoma hepático es indolora
(si) *(no)

4.- Un absceso del fondo de saco de Douglas se puede valorar en la exploración clínica
con:
a.- la maniobra de Murphy
*b.- un tacto rectal
c.- la maniobra de Blumberg
d.- el signo de Rovsing
e.- la maniobra de rebote

5.- ¿Los siguientes órganos tienen capa serosa visceral?:


a.- el apéndice *(si) (no)
b.- el recto (si) *(no)
c.- el primer asa yeyunal a nivel del ángulo de Treitz *(si) (no)
d.- la cara posterior de la segunda porción duodenal (si) *(no)
e.- el esófago torácico (si) *(no)

6.- Respecto a la fisiología del peritoneo señalar las opciones correctas:


a.- el peritoneo puede absorber hasta 35 c.c. por hora por gradiente osmotico *(si) (no)
b.- la función plástica (creación de adherencias) depende de los fibroblastos del
peritoneo (si) *(no)
c.- los estímulos dolorosos del peritoneo parietal producen sobre todo contractura de los
músculos del abdomen *(si) (no)
d.- la distensión o tracción del peritoneo visceral puede producir bradicardia *(si) (no)
e.- la absorción de bacterias por el peritoneo se hace por vía linfática especialmente
*(si) (no)

7.- El espacio peritoneal más declive es:


a.- la fosa de Morrisson
b.- el espacio parietocólico derecho
*c.- el fondo de saco de Douglas
d.- el espacio subfrénico izquierdo
e.- la trascavidad de los epiplones

8.- ¿En las peritonitis establecidas se producen los siguientes fenómenos?:


a.- ileo adinamico *(si) (no)
b.- aumento de la absorción de líquidos (si) *(no)
c.- hipovolemia por creación de tercer espacio *(si) (no)
d.- diapedesis de polimorfonucleares *(si) (no)
e.- exudado intraperitoneal rico en proteínas y fibrina *(si) (no)

9.- Los fenómenos generales de respuesta a una peritonitis pueden incluir los siguientes
EXCEPTO:
a.- retención agua por activación de los sistemas aldosterona+ADH
*b.- alcalosis metabólica
c.- hipoventilación
d.- taquipnea
e.- hipopotasemia

10.- Entre los signos clínicos de las peritonitis encontraremos:


a.- maniobra de rebote positiva *(si) (no)
b.- tacto rectal doloroso al palpar el fondo de saco de Douglas *(si) (no)
c.- en la percusión matidez al principio y timpanismo en fase establecida (si) *(no)
d.- hiperperistaltismo en la auscultación (si) *(no)
e.- paciente generalmente inmóvil *(si) (no)

11.- Cual de las siguientes situaciones NO debe razonablemente plantear un


diagnóstico diferencial con una peritonitis:
a.- herpes zoster de la pared abdominal
b.- porfiria
*c.- hemorragia digestiva por úlcera de la segunda porción duodenal
d.- un síndrome de abstinencia a heroína
e.- una obstrucción intestinal

12.- Respecto a las peritonitis primarias señalar las opciones correctas:


a.- la contaminación peritoneal se produce por vía hemática desde un foco
extraabdominal *(si) (no)
b.- son más frecuentes en niños *(si) (no)
c.- son siempre polimicrobianas (si) *(no)
d.- el germen más frecuente es el bacteroides fragilis (si) *(no)
e.- no deben precisar tratamiento quirúrgico si el tratamiento antibiótico está bien
dirigido *(si) (no)

13.- En la bacteriología de las peritonitis agudas secundarias el germen más frecuente de


los siguientes es:
a.- estreptococo fecal
b.- clostridio
c.- corynebacteria
*d.- bacteroides fragilis
e.- estreptococo B hemofilico

14.- Un paciente ha sido intervenido por una peritonitis difusa secundaria a una
perforación de un asa yeyunal por una espina de pescado. Se retiraron los drenajes a los
siete días. A los quince días de la intervención el paciente tolera bien la alimentación
oral sin dolor abdominal pero persiste un cuadro febril en picos y tiene la siguiente
radiografía de tórax

a.- se debe sospechar una neumonía basal derecha y tratarse con antibióticos (si) *(no)
b.- se debe realizar un TAC para descartar un absceso intraabdominal *(si) (no)
c.- se debe intervenir al paciente sin más exploraciones porque la fiebre en picos
significa que tiene una peritonitis difusa recidivada (si) *(no)
d.- se puede sospechar con esta radiografía un absceso subfrenico derecho *(si) (no)
e.- se debe mantener una actitud de observación sin tratamiento durante al menos siete
días más porque puede ser una evolución normal postoperatoria (si) *(no)

15.- Un paciente ha sido intervenido por una peritonitis difusa secundaria a una
perforación de un asa yeyunal por una espina de pescado. Se retiraron los drenajes a los
siete días. A los quince días de la intervención el paciente tiene el abdomen blando y
tolera bien la alimentación oral, pero aparece un cuadro febril en picos y se realiza un
TAC que muestra la imagen siguiente:
a.- se trata de un absceso subfrénico que debe ser drenado percutaneamente con control
radiológico o ecográfico *(si) (no)
b.- se debe tratar con amoxicilina y clavulanico siete días y después repetir el TAC
(si) *(no)
c.- se dede hacer una punción para tomar un cultivo y tratar con el antibiótico adecuado
durante siete días (si) *(no)
d.- lo más probable es que la colección se reabsorba espontáneamente con tratamiento
exclusivamente antiinflamatorio (si) *(no)
e.- por tener aire en la parte superior de la imagen (ver flecha superior) se debe
descartar el absceso y valorar otro diagnóstico (angioma, metástasis etc.) (si) *(no)

16.- En una obstrucción mecánica de intestino delgado lo siguiente es cierto EXCEPTO:


a.- se pueden emitir gases y heces en las primeras horas
*b.- el comienzo del dolor es debido a la distensión de las asas
c.- no deben existir signos de irritación peritoneal en las primeras horas
d.- el dolor inicial es de características cólicas
e.- la auscultación es de ruidos hidroaereos progresivos al comienzo.

17.- La causa más frecuente de obstrucción intestinal en una paciente operada es:
*a.- adherencias
b.- cáncer de colon sigmoideo
c.- cáncer de colon ascendente
d.- hernia inguinal
e.- hernia crural

18.- Un vólvulo de intestino delgado da lugar a una situación de:


a.- Íleo paralítico
b.- Oclusión mecánica simple
c.- Síndrome de asa ciega
d.- Síndrome de intestino corto
*e.- Oclusión mecánica estrangulada

19.- Respecto a las obstrucciones intestinales funcionales (ileo paralítico) señalar la


opción correcta:
a.- son obstrucciones intestinales con luz permeable *(si) (no)
b..- después de un laparotomía se produce siempre un cierto nivel de ileo paralítico
*(si) (no)
c.- un ileo paralítico reflejo por un cólico ureteral comenzará siempre a nivel yeyunal
con “hiperperistaltismo de lucha”. (si) *(no)
d.- el síndrome de Ogilvie es un ileo paralítico localizado en el intestino grueso *(si)
(no)
e.- una invaginación intestinal es una causa de ileo paralítico (si) *(no)

20.- Desde un punto de vista clínico en una obstrucción intestinal encontraremos:


a.- los vómitos suelen preceder al dolor cólico intestinal (si) *(no)
b.- en la exploración abdominal el meteorismo y el signo de Blumberg siempre
coinciden (si) *(no)
c.- el tacto rectal siempre es doloroso al palpar el fondo de saco de Douglas (si) *(no)
d.- primero escucharemos borborigmos y ruidos metálicos y posteriormente silencio
abdominal *(si) (no)
e.- la irritación peritoneal debe hacer sospechar una situación de obstrucción
estrangulada *(si) (no)

21..- En una obstrucción intestinal de 12 horas de evolución encontraremos:


a.- hiperpotasemia (si) *(no)
b.- hiponatremia *(si) (no)
c.- hipocloremia *(si) (no)
d.- oliguria *(si) (no)
e.- bradicardia (si) *(no)

22.- Una sospecha de obstrucción intestinal de intestino delgado se valorará en un


primer momento con :
a.- Un transito intestinal
b.- Una radiografía simple en decúbito prono
*c.- Una radiografía simple en bipedestación
d.- Una ecografía
e.- Un Enema Opaco

23.- En la zona proximal a la obstrucción, la dilatación del intestino delgado tendrá


como consecuencias EXCEPTO:
a.- acumulo de gases como consecuencia de fenómenos de putrefacción
b.- disminución de la absorción en los tramos dilatados
c.- edema de la pared intestinal
*d.- aumento del peristaltismo de lucha en las zonas dilatadas
e.- proliferación bacteriana

24.- En estudio radiológico simple la postura del paciente MENOS EFICAZ para
valorar una obstrucción intestinal de intestino delgado será:
a.- bipedestación con rayo horizontal
b..- decúbito lateral derecho con rayo horizontal
c.- decúbito lateral izquierdo con rayo horizontal
*d.- decúbito supino con rayo vertical
e.- decúbito supino con rayo horizontal

25.- Señalar la opción correcta (puede haber más de una), para cada una de las
situaciones siguientes en un paciente con diagnóstico de obstrucción intestinal:
ileo mecánico simple (A)
obstrucción intestinal con estrangulación (B)
ileo paralítico (C)
a.- dolor cólico moderado con poca distensión y signo de Blumberg negativo *(A) (B)
(C)
b.- dolor cólico de comienzo brusco con Blumberg positivo (A) *(B) (C)
c.- distensión abdominal difusa sin zonas de dolor focalizado (A) (B) *(C)
d.- radiología simple con predominio de la distensión y menos niveles hidroaereos (A)
(B) *(C)
e.- ruidos de hiperperistaltismo en la auscultación abdominal en las primeras horas *(A)
*(B) (C)

26.- En una obstrucción intestinal a nivel de la unión recto-sigmoidea la zona que más
se dilatará será:
a.- el recto
b.- el sigma
*c.- el ciego
d.- el ileon terminal
e.- el yeyuno

27.- Para confirmar el diagnóstico de presunción de una obstrucción intestinal a nivel de


colon descendente podremos indicar una de las siguientes exploraciones EXCEPTO:
a.- radiología simple de abdomen
b.- enema opaco
c.- TAC abdominal
*d.- transito intestinal con bario
e.- colonoscopia
Respuestas ANESTESIOLOGIA Y REANIMACIÓN.

1.- Respecto al concepto de sedación lo siguiente es cierto EXCEPTO:


a.- es un estado previo a la hipnosis y a la anestesia
b.- el paciente sedado se encuentra en estado de indiferencia psicomotora
c.- el paciente se encuentra reactivo y puede ser despertado con facilidad
* d.- cuando un paciente está sedado está en estado de analgesia
e.- durante la sedación existe una abolición del componente psíquico del dolor

2.- En las siguientes sentencias sobre los procedimientos anestésicos, señalar si la


sentencia es verdadera (V) o falsa (F):
a.- el halotano es un anestésico intravenoso (V) *(F)
b.- la anestesia raquídea es la que se realiza mediante inyección de un anestésico local
en el espacio subaracnoideo de la medula espinal *(V) (F)
c.- la anestesia intravenosa total precisa siempre la utilización de un agente anestésico
hipnótico, un analgésico (generalmente opiáceo) y un bloqueante neuromuscular.
*(V) (F)
d.- la anestesia regional intravenosa periférica está especialmente indicada para la
cirugía de la mama (V) *(F)
e.- la anestesia epidural se realiza inyectando un anestésico local en el espacio
extradural debajo del ligamento amarillo. *(V) (F)

3.- Cual de las siguientes situaciones de entre las que se consiguen durante una
anestesia general es FALSA:
a.- amnesia anterógrada
b.- pérdida de conciencia
*c.- mantenimiento de los estímulos nociceptivos
d.- reducción de la actividad refleja
e.- inhibición del sistema simpático

4.- Respecto a los fármacos empleados en anestesia general señalar si las afirmaciones
siguientes son verdaderas (V) o falsas (F):
a.- los sedantes ansiolíticos tipo benzodiacepinas se pueden utilizar en la fase de
preinducción anestésica *(V) (F)
b.- la intubación endotraqueal se realiza siempre antes de administrar los fármacos
hipnóticos y relajantes musculares (V) *(F)
c.- los opiáceos tipo fentanilo se utilizan durante la fase de mantenimiento anestésico
*(V) (F)
d.- la neostigmina y naloxona son fármacos reversores que se emplean en la fase de
educción *(si) (no)
e.- el mantenimiento hipnótico se puede realizar tanto con fármacos inhalatorios como
intravenosos *(V) (F)

5.- La clasificación de la Sociedad Americana de Anestesiólogos (ASA) para valorar el


riesgo quirúrgico se basa en:
a.- criterios analíticos
b.- criterios radiológicos
*c.- situación general del paciente y enfermedades asociadas
d.- situación cardiorrespiratoria del paciente
e.- conjunto de parámetros analíticos y radiológicos exclusivamente
6.- Respecto a la monitorización anestésica señalar si la sentencia es verdadera (V) o
falsa (F):
a.- la pulsioximetría es un método espectrofotométrico que permite medir la saturación
de oxigeno de forma NO invasiva *(V) (F)
b.- el capnógrafo mide los niveles de dióxido de carbono y es el mejor método no
invasivo de monitorización de la ventilación *(V) (F)
c.- la indicación de monitorizar la diuresis depende, entre otros factores, de la duración
de la intervención *(V) (F)
d.- el catéter de Swan-Ganz se coloca en la arteria pulmonar *(V) (F)
e.- los catéteres venosos centrales tienen muy poca utilidad en la monitorización
anestésica (V) *(F)

7.- Una de las siguientes afirmaciones sobre la ventilación mecánica (VM) y la


intubación es FALSA:
*a.- la VM es un sistema de intercambio de gases a nivel de la membrana alveolo-
capilar.
b.- la VM controlada por volumen es la que más se utiliza, con tubo endotraqueal,
durante la anestesia general.
c.- Debido a la diferente distensibilidad de los alvéolos según su nivel de perfusión, la
VM tiende a hipoventilar las zonas pulmonares más perfundidas y a hiperventilar las
zonas pulmonares de espacio muerto.
d.- la intubación endotraqueal es el método más seguro para mantener la permeabilidad
y control de la vía aérea
e.- las cánulas de Guedel son las cánulas orofaríngeas, que se emplean para evitar que la
base de la lengua obstruya la vía aérea.

8.- Señalar el puesto correcto para cada una de las actuaciones siguientes en la
secuencia de la inducción anestesica:
a.- intubación 1 2 3 4 *5
b.- acceso venoso 1 *2 3 4 5
c.- fármacos (opiáceos, hipnóticos, relajantes) 1 2 3 *4 5
d.- preoxigenación 1 2 *3 4 5
e.- monitorización *1 2 3 4 5

9.- Los inhibidores de la colinesterasa se emplean:


a.- en la fase de inducción anestésica
b.- como droga vasoactiva en la fase de mantenimiento
*c.- en la fase de despertar para revertir el efecto de los relajantes.
d.- como droga para contrarrestar a los opiáceos
e.- como analgésico en el postoperatorio inmediato
10.- Con respecto a la anestesia general señalar si las siguientes afirmaciones son
verdaderas (V) o falsas (F)
a.- La permeabilidad de la vía aérea es el factor de mayor importancia en el curso de una
anestesia general. *(V) (F)
b.- Durante el estado de inconsciencia, el paciente depende por completo del
anestesiólogo y sus instrumentos. *(V) (F)
c.- Las técnicas de mantenimiento anestésico que se pueden emplear son: la inhalatoria
pura, la balanceada (bloqueantes neuromusculares, opiáceo, gases y/o vapores) o
anestesia intravenosa total (agente hipnótico intravenoso, opiáceo y bloqueante
neuromuscular). *(V) (F)
d.- Los objetivos durante la fase de mantenimiento de la anestesia general son: el
mantenimiento de un adecuado plano anestésico acorde con el estímulo quirúrgico, el
mantenimiento de una adecuada perfusión tisular y el mantenimiento de un buen
transporte de oxígeno. *(V) (F)
e.- Después de una anestesia general las complicaciones que podemos encontrar son:
Hipotensión arterial, hipertensión arterial y arritmias cardíacas. *(V) (F)

11.- Con respecto a la anestesia regional es CIERTO que:


a.- En la anestesia epidural el anestésico se deposita en el espacio comprendido entre el
ligamento amarillo y la duramadre. *(V) (F)
b.- En la anestesia raquídea el anestésico se deposita en el espacio subaracnoideo
. *(V) (F)
c.- En la anestesia de plexos nerviosos el anestésico se inyecta en las vainas de los
plexos *(V) (F)
d.- El frío con cloruro de etilo esta indicado especialmente en la anestesia de plexo
nervioso cervical (V) *(F)
e.- La lidocaína se puede utilizar como anestésico regional intravenosos *(V) (F)

12.- Con respecto al dolor postoperatorio señalar lo correcto:


a.- Forma parte del sufrimiento que debe pagar todo paciente que es operado, por lo que
no hay que tratarlo.(si) *(no)
b.- El dolor postoperatorio puede causar trastornos sistémicos importantes como:
hipoxia, inmunodepresión, espasmos musculares y depresión psiquica.
*(si) (no)
c.- Conviene que todo paciente tenga un poco de dolor a la hora de pautar un
tratamiento analgésico para evitar los efectos secundarios de los fármacos.(si) *(no)
d.- Siempre hay que prevenir y tratar el dolor postoperatorio considerando como
objetivo que el paciente no sufra ningún tipo de dolor o disconfort desde el momento en
que se despierta de la anestesia. *(si) (no)
e.- Un cuadro de taquicardia en el postoperatorio inmediato puede ser una respuesta al
dolor *(si) (no)

13.- Respecto a los estadios de la anestesia general señalar lo correcto:


a.- El estadio I, o de analgesia, es el mejor plano anestésico para realizar cualquier
intervención. (si) *(no)
b.- El estadio IV, o de intoxicación, es un plano previo al quirúrgico por el que todo
paciente tiene que pasar cuando es anestesiado. (si) *(no)
c.- El estadio III, o quirúrgico, puede dividirse en cuatro subestadios, siendo el III4 el
plano anestésico más profundo.*(si) (no)
d.- En el estadio III4 se produce una parálisis incipiente del tronco encefálico, hay
abolición del reflejo tusígeno y es el plano adecuado para la intubación endotraqueal y
la realización de todas las intervenciones de la cavidad abdominal.*(si) (no)

14.- Indicar cuál de los siguientes conceptos es FALSO:


a.- La anestesia es la ausencia de sensibilidad.
b.- La analgesia es la ausencia de dolor en respuesta a estímulos que normalmente lo
provocarían.
c.- La anestesia neuroaxial es un subtipo de la anestesia regional que incluye la
anestesia epidural y la subaracnoidea o raquídea.
*d.- Todo paciente inconsciente es incapaz de responder a estímulos nociceptivos, por
lo que está por definición analgesiado.
e.- El dolor es la percepción consciente de un estímulo nociceptivo, por lo que un
paciente inconsciente no se puede decir que sufra dolor.

15.- En relación con los conceptos generales de la anestesia señalar las afirmaciones
correctas
a.- La hipnosis es un estado de sueño en el que se pierde la capacidad de despertar frente
a estímulos externos. *(si) (no)
b.- La sedación se debe conseguir simultáneamente a la anestesia (si) *(no)
c.- Anestesiar es crear una situación en la que el individuo no responde a estímulos
nociceptivos *(si) (no)
d.- Los componentes corticales que debe conseguir la anestesia son la inconsciencia y la
amnesia. *(si) (no)
e.- Uno de los objetivos primordiales de la anestesia general es mantener la actividad
refleja (si) *(no)

16.- Durante la fase de mantenimiento de la anestesia general se pueden utilizar los


fármacos siguientes EXCEPTO:
a.- opiáceos
b.- bloqueantes neuromusculares
c.- hipnóticos intravenosos
d.- hipnóticos inhalatorios
*e.- agentes reversores como la neostigmina

17.- En la anestesia general se pueden utilizar fármacos con los siguientes efectos:
a.- anestésicos *(si) (no)
b.- hipnóticos *(si) (no)
c.- sedantes *(si) (no)
d.- amnésicos *(si) (no)
e.- analgésicos *(si) (no)

18.- Respecto a la pulsioximetría señalar las afirmaciones correctas:


a.- es un método de monitorización invasivo (si) *(no)
b.- mide la saturación de oxigeno arterial capilar *(si) (no)
c.- monitoriza la frecuencia cardiaca *(si) (no)
d.- es un método espectrofotométrico *(si) (no)
e.- la saturación capilar de oxigeno normal es del 50% (si) *(no)
19.- Pueden ser consecuencias del dolor postoperatorio EXCEPTO:
*a.- bradicardia
b.- hipertensión
c.- retención urinaria
d.- ileo paralítico
e.- atelectasias pulmonares

20.- En la fase de educción o despertar anestésico una de las siguientes afirmaciones es


FALSA:
a.- se debe procurar que perdure la analgesia
b.- los inhibidores de la colinesterasa sirven para recuperar la placa motora
*c.- se debe mantener lo más posible la situación de hipnosis
d.- cuando el paciente recupera la consciencia puede ser extubado
e.- en esta fase se recupera la actividad refleja
RESPUESTAS - CIRUGÍA ONCOLOGICA. MELANOMA.

1.- En 1935 se realizó la primera “duodenopancreatectomía cefálica” por el siguiente


autor:
a.- Miles
b.- Halsted
*c.- Whipple
d.- Billroth
e.- McBurney

2.- De los siguientes tumores malignos cual es el que produce en España, en los
hombres, la mayor mortalidad:
a.- estómago
*b.- pulmón
c.- colon y recto
d.- páncreas
e.- hígado

3.- El concepto de “cirugía profiláctica” puede aplicarse a las siguientes patologías


EXCEPTO:
a.- poliposis de colon
*b.- diverticulosis de colon
c.- colitis ulcerosa
d.- cáncer medular de tiroides familiar
e.- criptorquidia

4.- En la clasificación TNM la designación T0 (cero) significa:


a.- tumor “in situ”
*b.- tumor de localización desconocida
c.- tumor menor de 1 cm. de tamaño
d.- tumor sin metástasis ganglionares en el primer escalón
e.- tumor polipoideo

5.- Señalar lo correcto respecto a los siguientes objetivos de la determinación del


Antígeno Carcino Embrionario (CEA):
a.- es una prueba que se utiliza en el diagnóstico precoz del cáncer de colon (si) *(no)
b.- sirve en determinaciones seriadas para indicar recurrencia postoperatoria del cáncer
de colon *(si) (no)
c.- indica la sensibilidad de un tumor colo-rectal a la quimioterapia (si) *(no)
d.- sirve para valorar la resección completa de un tumor colo-rectal *(si) (no)
e.- tiene también valor diagnóstico en el cáncer de vías biliares (si) *(no)

6.- Señalar en cuales de estos tumores es probable que ocurra propagación linfática:
a.- carcinoma de próstata *(si) (no)
b.- astrocitoma (si) *(no)
c.- melanoma maligno *(si) (no)
d.- lipoma localizado en la espalda (si) *(no)
e.- carcinoma de células basales de la piel de la cara (si) *(no)
7.- Respecto a la estadificación (o estadiaje) de los tumores malignos señalar los
conceptos correctos:
a.- la biopsia del ganglio centinela tiene como objetivo estadiar un tumor evitando en lo
posible grandes linfadenectomías *(si) (no)
b.- la determinación del CEA es indispensable para el estadiaje correcto de un cáncer de
colon (si) *(no)
c.- la clasificación TNM definitiva de un tumor gástrico debe establecerse después de
haber intervenido al paciente *(si) (no)
d.- la clasificación TNM valora el tipo histológico del tumor primitivo (si) *(no)
e.- el estadiaje solamente tiene un valor diagnóstico (si) *(no)

8.- Se sabe que los siguientes estados presentan un patrón familiar relacionado a una
predisposición a padecer un cáncer EXCEPTO:
a.- poliposis de colon
b.- cáncer de mama
c.- síndrome de Gardner
*d.- síndrome de Peutz-Jeghers
e.- retinoblastoma

9.- Cual de los siguientes tumores se considera más radiosensible:


*a.- carcinoma de mama
b.- adenocarcinoma gástrico
c.- melanoma maligno
d.- seminoma
e.- liposarcoma

10.- Las siguientes situaciones precisan tratamiento urgente quirúrgico o radioterápico;


señalar la opción correcta:
a.- obstrucción de la vía respiratoria por un linfoma de tiroides (cirugía) *(radioterapia)
b.- obstrucción intestinal por un carcinoma de colon sigmoide *(cirugía) (radioterapia)
c.- compresión de la medula espinal por una metástasis de cáncer de próstata
(cirugía) *(radioterapia)
d.- obstrucción de la vena cava superior por un cáncer de pulmón
(cirugía) *(radioterapia)
e.- dolor vertebral por carcinoma de mama metastático (cirugía) *(radioterapia)

11.- Ante una lesión cutánea pigmentada conocida, que se hace pruriginosa, ¿cual sería
la actitud correcta?:
a.- Tratamiento tópico hasta que desaparezca el prurito, y evitar la luz solar
b.- Revisión anual, ya que es una posibilidad normal de las lesiones pigmentadas
c.- Extirpación y biopsia selectiva del ganglio centinela
*d.- Biopsia escisional
12.- De los referidos a continuación, ¿ cual el factor pronóstico mas importante del
melanoma ?
a.- espesor de Breslow
*b.- presencia de metástasis ganglionares
c.- ulceración y sangrado de una lesión pigmentada preexistente
d.- localización en el tronco

13.- Cual es el tratamiento de elección de un melanoma > de 1 mm, sin evidencia


clínica de afectación ganglionar:
a- biopsia escisional, mas interferón
b- extirpación amplia y linfadenectomía regional
c- extirpación amplia y radioterapia loco-regional
*d- extirpación amplia y biopsia selectiva del ganglio centinela

14.- Relacionar los conceptos de tratamiento oncológico siguientes con las situaciones
clínicas descritas:
paliativo (A)
coadyuvante (B)
neoadyuvante (C)
citorreductor (D)
curativo (E)

a.- resección quirúrgica de una metástasis hepática única de un cáncer de colon


intervenido hace tres años (A) (B) (C) (D) *(E)
b.- tratamiento quimio y radioterápico preoperatorios de un cáncer de recto T3N1M0
(A) (B) *(C) (D) (E)
c.- realización de una anastomosis yeyuno-cólica latero-lateral en un cáncer de colon
transverso irresecable *(A) (B) (C) (D) (E)
d.- resección del epiplon mayor en una carcinomatosis peritoneal por cáncer de ovario
(A) (B) (C) *(D) (E)
e.- radioterapia post-quirúrgica en un cáncer de mama T2N1M0 (A) *(B) (C) (D) (E)

15.- El término recurrencia se puede aplicar a las situaciones siguientes:


a.- aparición de enfermedad después de un periodo de remisión *(si) (no)
b.- aparición de tejido tumoral en el mismo sitio donde se extirpó un tumor (si) *(no)
c.- aparición de metástasis hepáticas de un cáncer de colon irresecable tratado de forma
paliativa con quimioterapia (si) *(no)
d.- aparición de una metástasis hepática un año después de resecar un cáncer de colon
T1N0M0 *(si) (no)
e.- aparición de ascitis en un cáncer de ovario con carcinomatosis peritoneal (si) *(no)
Equilibrio hidro-electrolítico en los pacientes quirúrgicos. Respuestas.

1.- Cual de los siguientes valores normales en plasma es anormal:


a.- sodio 135-145 mEq/l
*b.- potasio 6-8 mEq/l
c.- cloruros 100-106 mEq/l
d.- proteínas totales 7 grs./100 ml.
e.- calcio 8.5-10.5 mgrs./100 ml.

2.- Cuales de las siguientes causas pueden producir hiponatremia:


a.- pérdidas de liquido gastrointestinal por una fístula digestiva *(si) (no)
b.- insuficiencia renal severa *(si) (no)
c.- cirrosis hepática *(si) (no)
d.- administración excesiva de suero glucosado al 5% *(si) (no)
e.- insuficiencia suprarrenal *(si) (no)

3.- Sobre las alteraciones del Potasio en el paciente quirúrgico señalar las afirmaciones
válidas:
a.- la acidosis metabólica facilita la entrada de K en las células produciendo
hipopotasemia (si) *(no)
b.- las pérdidas de líquidos de origen digestivo (diarrea fístulas) producen
hipopotasemia *(si) (no)
c.- la insuficiencia renal cursa con hiperpotasemia *(si) (no)
d.- las alteraciones de las ondas T y del intervalo PR del ECG son características de la
hiperpotasemia *(si) (no)
e.- la insulina favorece la entrada de K en las células *(si) (no)

4.- Señalar a que trastorno ácido-básico corresponde cada uno de los patrones
siguientes:
acidosis metabólica (A) alcalosis metabólica (B) acidosis respiratoria (C)
alcalosis respiratoria (D)
a.- pH↑ y aumento de CO3 H‾ (A) *(B) (C) (D)
b.- pH↑ y disminución de PCO2 (A) (B) (C) *(D)
c.- pH↓ y aumento de PCO2 (A) (B) *(C) (D)
d.- pH↓ y disminución de CO3 H‾ * (A) (B) (C) (D)

5.- Son causas de acidosis respiratoria EXCEPTO:


a.- alteraciones de la relación perfusión-ventilación
*b.- situaciones de shock hipovolémico
c.- derrame pleural bilateral
d.- neumotórax traumático
e.- broncoespasmo

6.- Sobre la acidosis respiratoria cual de las siguientes afirmaciones es INCORRECTA :


a.- un signo clínico es la somnolencia
b.- la PCO2 se eleva por encima de 43 mm de Hg.
*c.- en el tratamiento se debe administrar siempre bicarbonato
d.- el K suele estar elevado
e.- es indispensable para corregirla ventilar correctamente al paciente.
7.- Pueden ser causa de acidosis metabólica las siguientes situaciones EXCEPTO:
a.- shock séptico
b.- shock cardiogénico
c.- insuficiencia renal severa
*d.- cuadro de vómitos por estenosis pilórica
e.- pérdidas bilio-pancreáticas por fístula digestiva a nivel de yeyuno

8.- En la alcalosis respiratoria una de las siguientes afirmaciones es FALSA:


a.- se manifiesta por taquipnea
b.- al disminuir el Ca iónico puede aparecer tetania
c.- el CO3H‾ puede estar normal o ligeramente aumentado dependiendo del grado de
compensación renal
d.- existe vasoconstricción cerebral por la disminución de CO2
*e.- se debe tratar con administración de ácidos.

9.- Cual de las situaciones siguientes NO producirá una alcalosis metabólica:


a.- vómitos con pérdida de jugo gástrico
b.- transfusiones masivas
*c.- fístulas biliares
d.- tratamientos con diuréticos
e.- hiperaldosteronismo

10.- En la alcalosis metabólica podemos encontrar EXCEPTO:


a.- hipoventilación
*b.- hiperpotasemia
c.- CO3H‾ por encima de 25 meq/l.
d.- astenia
e.- arritmias ventriculares
. HEMORRAGIA HEMOSTASIA EN CIRUGÍA. (RESPUESTAS)

1.- Cuando un tumor erosiona un vaso y se produce una hemorragia se llama


hemorragia por:
a.- dieresis
b.- rexis
*c.- diabrosis
d.- diapedesis
e.- en sabana

2.- Relacionar las siguientes situaciones con el tipo de hemorragia que pueden producir:
traumatismo esplénico (A)
úlcera duodenal de la segunda porción duodenal (B)
cáncer de sigma (C)
hemorroides (D)
varices esofágicas (E)

a.- melena (A) (B) *(C) (D) (E)


b.- rectorragia (A) (B) (C) *(D) (E)
c.- hematoquecia (A) (B) *(C) (D) (E)
d.- hematemesis (A) (B) (C) (D) *(E)
e.- hemoperitoneo *(A) (B) (C) (D) (E)

3.- En una hemorragia aguda importante aparecen los siguientes signos clínicos
a.- Palidez *(si) (no)
b.- Oliguria *(si) (no)
c.- Bradicardia (si) *(no)
d.- Ictericia (si) *(no)
e.- Hipotension *(si) (no)

4.- El primer hecho en la hemostasia natural es:


a.- formación del coágulo de fibrina
*b.- vasoconstricción del vaso lesionado
c.- formación del tapón de plaquetas
d.- fibrinolísis
e.- activación de los fosfolipidos plaquetarios

5.- En la hemostasia todos los siguientes factores son fisiológicos EXCEPTO:


a.- Fibrinolisis
b.- Formación de fibrina
c.- Formación de tapones plaquetarios
*d.- Elevación de la adrenalina circulante
e.- Constricción vascular

6.- Después de una lesión vascular el primer paso de la cascada de coagulación será:
a- *Unión del factor XII a la colágena endotelial
b- Desdoblamiento del factor XI en factor activo IX
c- Conversión de protrombina en trombina
d- Conversión de fibrinógeno en fibrina
e- Unión del factor IX al VIII en presencia de Ca.
7.- Respecto a la hemostasia primaria señalar lo correcto:
a.- el factor de Von Willebrand participa en la adhesión plaquetaria. *(si) (no)
b.- el tapón hemostático primario lo forman leucocitos y macrófagos (si) *(no)
c.- la adhesión plaquetaria se produce sobre estructuras subendoteliales *(si) (no)
d.- las fibras de colágeno, elastina y reticulina del subendotelio tienen una elevada
capacidad trombogénica al activar la adhesión plaquetaria *(si) (no)
e.- en la agregación de las plaquetas intervienen receptores de la membrana plaquetaria
que fijan el fibrinógeno formando puentes que “agregan” las plaquetas *(si) (no)

8.- Cuál de los siguientes factores se consume durante la coagulación, pudiendo ser
necesario su aporte si existe excesivo consumo?
*a.- Fibrinógeno (Factor I)
b.- Factor Christmas (IX)
c.- Factor Stuart (X)
d.- Tromboplastina (XI)
e.- Factor Hageman (XII)

9.- Una paciente de 70 años diagnosticada de colecistitis aguda y en tratamiento


antibiótico, sufre un empeoramiento del cuadro séptico, apareciendo además melenas y
petequias. Los estudios de laboratorio muestran trombocitopenia, tiempo de
protrombina prolongado y disminución del fibrinógeno. ¿Cuál es el mejor tratamiento
de la coagulopatía que está iniciando la paciente?
a.- Administrar heparina
*b.- Operar a la paciente y solucionar el cuadro séptico
c.- Administrar sangre completa
d.- Administrar plasma fresco congelado
e.- Administrar corticoides

10.- Las manifestaciones analíticas de un paciente con trombopenia serán (señalar lo


correcto)
a.- Tiempo de hemorragia *(aumentado) (disminuido) (sin alteración)
b.- Número de plaquetas (aumentado) *(disminuido) (sin alteración)
c.- Tiempo de protrombina (aumentado) (disminuido) *(sin alteración)
d.- Tiempo de cefalina (aumentado) (disminuido) *(sin alteración)
e.- Fibrinógeno (aumentado) (disminuido) *(sin alteración)

11.- El INR estará elevado en las siguientes situaciones EXCEPTO:


a.- hepatopatías crónicas
b.- pacientes en tratamiento con anticoagulantes orales
c.- en situación de coagulación intravascular diseminada
*d.- en la enfermedad de Von Willebrand
e.- en pacientes con defecto de fibrinógeno
12.- Relacionar los siguientes factores con su situación u objetivo en el proceso de
hemostasia-coagulación:
plasmina (A) trombina (B) tromboplastina (C) factor de Von Willebrand (D)
Vitamina K (E)

a.- lisis de fibrina *(A) (B) (C) (D) (E)


c.- paso de fibrinógeno a fibrina (A) *(B) (C) (D) (E)
d.- adhesión plaquetaria (A) (B) (C) *(D) (E)
d.- factor tisular del mecanismo extrínseco (A) (B) *(C) (D) (E)
e.- formación de protrombina (A) (B) (C) (D) *(E)

13.- Cual de los siguientes factores NO FORMA PARTE de los procesos fibrinolíticos
o mecanismos anticoagulantes:
a.- Flujo sanguíneo
*b.- Vitamina K
c.- Antitrombina III
d.- Proteína C
e.- Integridad del endotelio vascular

14.- Paciente que refiere heces melénicas desde hace 4 meses y analíticamente presenta
una cifra de hemoglobina de 6,5 gr% ¿qué le trasfundirías?
a.- Plasma fresco congelado
b.- Sangre total
*c.- Concentrados de hematíes
d.- Crioprecipitados
e.- Concentrados de plaquetas

15.- En cual de las siguientes situaciones el INR NO está alterado:


*a.- profilaxis con heparinas de bajo peso molecular
b.- hepatopatías crónicas
c.- coagulación intravascular diseminada
d.- tratamiento con Sintrom
e.- pacientes politransfundidos
HERIDAS, CICATRIZACION. (RESPUESTAS)

1.- Respecto a las contusiones señalar las opciones correctas:


a.- una equimosis es una contusión de segundo grado (si) *(no)
b.- una contusión de tercer grado con escara se tratará como una herida *(si) (no)
c.- una contusión con hematoma es de segundo grado *(si) (no)
d.- la impotencia funcional que produce una contusión debe tratarse siempre con
movilización forzada precoz (si) *(no)
e.- los fármacos analgésicos y anti-inflamatorios pueden estar indicados en las
contusiones *(si) (no)

2.- Cual de las siguientes heridas incisas, tendrá debido a la tensión de las líneas de
Langer, MENOR tendencia a la separación de sus bordes:
a.- herida vertical en cara anterior del cuello
b.- herida vertical en región escapular
*c.- herida de una laparotomía subcostal derecha
d.- herida longitudinal en cara anterior de muslo
e.- herida longitudinal sobre cara anterior de la rodilla

3.- En la fase inflamatoria del proceso de cicatrización las principales células


responsables del Factor de Crecimiento para las células endoteliales y la angiogénesis
son:
a.- leucocitos
* b.- macrófagos
c..- fibroblastos
d.- neutrófilos
e.- células epiteliales

4.- Respecto a las heridas lo siguiente es cierto EXCEPTO


a.- una herida por arma blanca en el tórax que afecta a la pleura parietal es una herida
penetrante
b.- una herida por arma blanca en el abdomen que afecta al peritoneo parietal es una
herida penetrante
c.- una herida por empalamiento afecta al ano y al periné
d.- una herida por arma de fuego que afecta a la pared gástrica es perforante
*e.- una herida de “scalp” es por arrancamiento en la pared torácica

5.- Cual de las siguientes heridas se puede suturar directamente sin necesidad de esperar
una cicatrización por “segunda intención”:
a.- mordedura de perro en una pierna en varón joven.
*b.- herida incisa por arma blanca en la cara
c.- herida incisa de más de seis horas en una pierna en un accidente de tráfico
d.- herida incisa en periné
e.- todas se pueden suturar directamente.
6.- En la cicatrización ocurre lo siguiente EXCEPTO:
*a.- la fase inflamatoria comienza a los tres o cuatro días después de producirse la herida
b.- los macrófagos segregan el factor de crecimiento de los fibroblastos
c.- la epitelización se produce siempre sobre el tejido de granulación y desde los bordes de
la herida.
d.- el hierro y la vitamina C son indispensables para la síntesis del colágeno
e.- el fenómeno de contracción de la herida se produce aproximadamente a las 48 horas de
producirse esta.

7.- Respecto a la cicatrización de las heridas lo siguiente es cierto EXCEPTO


a.- se llama ulcera de Marjolin a la aparición de un carcinoma espino celular implantado
sobre la cicatriz de una quemadura
b.- en la cicatriz queloidea la masa de colágeno sobrepasa los bordes de la herida
c.- las cicatrices hipertróficas generalmente no necesitan tratamiento porque mejoran
con el tiempo
*d.- los pacientes en tratamiento corticoideo tienen mejor cicatrización por su efecto
antiinflamatorio.
e.- las drogas citotóxicas afectan a la síntesis del colágeno.

8.- Sobre el proceso de cicatrización de las heridas señalar lo correcto:


a.- la vitamina C es indispensable en el proceso de síntesis del colágeno *(si) (no)
b.- la tensión máxima de las fibras de colágeno se consigue a los dos o tres días después
de producirse la herida (si) *(no)
c.- el pico máximo de número de fibroblastos se produce al segundo día, en la fase
inflamatoria de la cicatrización (si) *(no)
d.- la resistencia de la herida a la tensión depende del proceso de reorientación de las
fibras de colágeno *(si) (no)
e.- la epitelización se produce siempre sobre el tejido de granulación de forma
centrifuga desde el centro hacia los bordes de la herida (si) *(no)

9.- sobre el fenómeno de “contracción” de la herida señalar lo correcto:


a.- las células responsables de este fenómeno son los macrófagos (si) *(no)
b.- es un mecanismo que aproxima espontáneamente los bordes de la herida y
disminuye su superficie *(si) (no)
c.- las fibras de colágeno juegan un papel fundamental en la contracción de la herida
(si) *(no)
d.- la contracción de la herida comienza después de la fase de maduración de la cicatriz
(si) *(no)
e.- el fenómeno de contracción puede verse afectado en heridas en zonas isquémicas
*(si) (no)

10.- Cual de los siguientes mecanismos es el más característico en la formación de un


queloide:
*a.- una relación alterada entre la formación de colágeno y la actividad de la colagenasa
b.- una fase inflamatoria excesivamente larga
c.- una síntesis excesiva de la enzima colagenasa.
d.- una actividad aumentada de las células endoteliales
e.- una actividad aumentada del fenómeno de angiogénesis
11.- En relación a la influencia de los siguientes factores en la cicatrización señalar lo
correcto:
a.- una tensión de oxígeno baja sirve de estímulo a la actividad de los fibroblastos
(si) *(no)
b.- la síntesis del colágeno se estimula con la hipoxia (si) *(no)
c.- el hierro es necesario para la actividad de los fibroblastos *(si) (no)
d.- los corticosteroides mejoran la cicatrización por su efecto antiinflamatorio (si) *(no)
e.- la hipovitaminosis C afecta a la síntesis del colágeno *(si) (no)

12.- En un caso de hidrosadenitis supurada de la ingle el mejor tratamiento después de


la excisión de la lesión es:
*a.- cierre por segunda intención
b.- cierre primario de la herida quirúrgica
c.- cierre primario retardado
d.- injerto de piel de espesor parcial
e.- realizar un colgajo desde el abdomen para tapar el defecto.

13.-

El paciente de la fotografía ha sufrido un traumatismo en la


región frontal con una barra de hierro. La descripción de la lesión debe ser:
a.- contusión de primer grado con equimosis
b.- contusión de primer grado con sufusión
*c.- contusión de segundo grado con hematoma
d.- contusión de tercer grado
e.- herida contusa.

14.- Relacionar los siguientes mecanismos de producción con los tipos de herida:
arrancamiento (A) herida de bala en abdomen con afectación de la pared gástrica (B)
herida por arma blanca con afectación de la articulación de la rodilla (C)
herida que afecta al ano y perine por objeto punzante (D) mordedura de perro (E)

a.- inciso-contusa (A) (B) (C) (D) *(E)


b.- scalp *(A) (B) (C) (D) (E)
c.- penetrante (A) (B) *(C) (D) (E)
d.- perforante (A) *(B) (C) (D) (E)
e.- empalamiento (A) (B) (C) *(D) (E)
15.- En el proceso inicial de la cicatrización, señalar lo correcto:
a.- la fase inflamatoria en una herida no complicada suele durar entre cuatro y seis días
*(si) (no)
b.- las células implicadas en la fase inflamatoria son los fibroblastos (si) *(no)
c.- los leucocitos liberan la mayoría de los mediadores celulares y los radicales libres de
oxigeno bactericidas *(si) (no)
d.- el TNF y las prostaglandinas son mediadores inflamatorios producidos por los
fibroblastos (si) *(no)
e.- la epitelización comienza en el proceso inicial de la cicatrización. *(si) (no)

16.- Sobre la fase proliferativa de la cicatrización cual de las siguientes afirmaciones es


FALSA:
a.- las células implicadas en esta fase son los fibroblastos
b.- la hidroxiprolina es una de los aminoácidos llave en la síntesis del colágeno
c.- el hierro y la vitamina C son importantes en la cicatrización por su necesidad en la
síntesis del colágeno
d.- La tensión máxima de las fibras de colágeno se consigue como mínimo a los treinta
días de la producción de la herida
*e.- El movimiento es un buen estímulo para la correcta orientación de las fibras de
colágeno de la herida.

17.- Sobre el fenómeno de contracción de la herida una de las siguientes afirmaciones es


FALSA:
a.- la contracción de la herida comienza antes de 48 horas
b.- los miofibroblastos son las células responsables de la contracción de la herida
c.- la formación de colágeno no interviene en la contracción de la herida
*d.- cuando las células epiteliales de ambos bordes de la herida contactan se crean
anclajes definitivos aunque sean estirpes celulares distintas
e.- la citosina es un factor de crecimiento plasmático que estimula el dinamismo celular
de la contracción de la herida.

18.- Los factores siguientes afectan negativamente a la cicatrización:


hematomas (A) glucocorticoides (B) isquemia (C) drogas citotóxicas (D)
hipovitaminosis C (E)
Señalar a cual aspecto de la cicatrización afectan especialmente (señalar solamente un
factor en cada respuesta)
a.- facilita proliferación bacteriana *(A) (B) (C) (D) (E)
b.- afecta a la síntesis del colágeno (A) (B) (C) (D) *(E)
c.- disminuyen la respuesta inflamatoria (A) *(B) (C) (D) (E)
d.- inhiben la multiplicación de fibroblastos (A) (B) (C) *(D) (E)
e.- afecta a la neoangiogénesis (A) (B) *(C) (D) (E)

19.- Respecto a las cicatrices hipertróficas señalar lo correcto:


a.- la causa es generalmente una excesiva tensión en los bordes de la herida *(si) (no)
b.- suelen mejorar con el tiempo *(si) (no)
c.- no tienen preferencia por ninguna localización *(si) (no)
d.- el colágeno se produce en exceso pero no sobrepasa los bordes de la herida
*(si) (no)
e.- pueden mejorar con técnicas que disminuyan la tensión de los bordes de la herida
*(si) (no)
20.- Sobre la patología de la cicatriz señalar lo correcto:
a.- las cicatrices queloides dependen de una predisposición del individuo *(si) (no)
b.- las heridas con cicatrización tórpida pueden degenerar en adenocarcinomas
(si) *(no)
c.- las heridas con cicatrización tórpida pueden degenera en carcinomas epidermoides
*(si) (no)
d.- el mejor tratamiento de los queloides es la extirpación quirúrgica precoz (si) *(no)
e.- el método de Friedrich consiste en la extirpación en bloque de los bordes de la herida
*(si) (no)
Preguntas de Blueprints

1. A 71-year-old man with sudden onset of severe abdominal and back pain is brought to the emergency
department for evaluation. He has a history of hypertension. He weighs 300 lb. He has a 45-pack-a-year history
of smoking. Physical examination reveals a pulsatile abdominal mass. Both lower extremities reveal pallor with
diminished pedal pulses. What is the most likely cause of this patient’s condition?
a. Atherosclerosis (aneurisma de aorta abdominal)
b. Marfan syndrome
c. Meningococcal infection
d. Syphilis
e. Trauma

2. A 78-year-old man is brought to the emergency department with a 12-hour history of abdominal pain,
diarrhea, and vomiting. He has a history of atrial fibrillation and was previously treated for congestive heart
failure with digoxin. Physical examination reveals a distended abdomen with significant guarding. Rectal
examination reveals guaiac positive stool in the vault. White blood cell count is 24,000/mL. Abdominal x-ray
reveals edema of the bowel wall. What is the most appropriate treatment for this patient?
a. Angiographic embolization
b. Antibiotic therapy with ampicillin and gentamicin
c. Antibiotic therapy with gentamicin
d. Heparinization followed by oral warfarin
e. Surgical exploration (isquemia mesentérica aguda) tratamiento incluye resección quirúrgica

3. A 20-year-old male tennis player crashes into a fence trying to chase a ball he thought he could catch up to
during an important match. His right knee sustains the brunt of injury. Physical examination reveals edema
and decreased range of motion of the knee in flexion and extension. Magnetic resonance imaging (MRI) is
performed and reveals dislocation of the joint. No pulse is palpable behind the knee joint. What is the most
likely explanation for this finding?
a. Anterior tibial artery rupture
b. Peroneal artery hematoma
c. Popliteal artery spasm
d. Posterior tibial artery hematoma
e. Superficial femoral artery spasm

4. A 25-year-old woman found a lump in her right breast on self-examination. She has no family history of breast
cancer. The lump is freely mobile and well circumscribed. What is the best option to evaluate a breast mass in
a young female?
a. Biopsy
b. Mammography
c. Testing for breast cancer (BRCA) gene
d. Ultrasound
e. Watchful waiting

5. A 19-year-old woman began breast-feeding for the first time. At first, it was difficult for her infant to feed.
Now, her breasts are red, warm, and sore. She has continued to breast-feed, despite the pain; however, she
has recently begun to use a breast pump instead of breastfeeding. She is begun on a course of oral antibiotics.
What condition is this patient at risk of developing?
a. Breast abscess
b. Fibrocystic disease
c. Inflammatory breast cancer
d. Prolactinoma
e. Tuberculosis

6. A 31-year-old premenopausal woman with a left breast mass undergoes a left modified radical mastectomy.
Pathology reveals infiltrating ductal carcinoma measuring 3 cm in size with negative lymph nodes. Estrogen
receptor status is negative. What is the most appropriate adjuvant therapy for this patient?
a. Chemotherapy (multiagent) terapia de eleccion en premenopausica con estadio 1 o 2 de cancer de mama size <1cm,
linfonodos negativos y estrógeno receptor negativo
b. External-beam radiotherapy
c. High-energy focused ultrasound therapy
d. Tamoxifen estrogeno receptor positive y >1cm
e. Watchful waiting

7. A 31-year-old woman complains of a 6-month history of bloody diarrhea, abdominal pain, and intermittent
fevers. She has a history of irritable bowel syndrome but has had a worsening of her symptoms during the
above time period. Her past medical history is unremarkable. Physical examination reveals abdominal
distension. Bowel sounds are present in all quadrants. Rectal examination reveals multiple anal fissures. What
is the most appropriate diagnostic testing for this patient?

a. Anoscopy
b. Colonoscopy → colitis ulcerativa
c. Flexible sigmoidoscopy
d. Rigid sigmoidoscopy
e. No further diagnostic testing is required for this patient.

8. A 71-year-old woman presents to her primary care physician complaining of rectal bleeding. She had some
mild left-sided abdominal cramps that subsided within a few minutes. She has never had a prior episode of
rectal bleeding. Physical examination reveals mild left lower quadrant abdominal pain without evidence of
guarding or rebound tenderness. Rectal examination reveals no fresh blood in the rectal vault. Colonoscopy
reveals several outpouchings of the sigmoid colon wall without evidence of bleeding or perforation. The
remainder of the colonoscopy is within normal limits. White blood cell count is normal. What is the most
appropriate treatment for this patient?
a. Antibiotic therapy with ampicillin and gentamicin
b. Left hemicolectomy
c. Right hemicolectomy
d. Subtotal colectomy
e. Watchful waiting → diverticulosis

9. An 85-year-old man is brought to the emergency department because of acute abdominal pain and
progressive abdominal distention. He is a resident of a local nursing home. He has not been eating because of
progressive nausea. Abdominal radiographs reveal a massively sigmoid colon. What is the initial treatment for
this patient?
a. Gastrografin enema→ esta también se puede considerar
b. High-fiber diet
c. Lactulose
d. Rectal tube decompression → vólvulo sigmoides (esto es lo ideal)
e. Surgical resection → si fuera calculo cecal

10. A 41-year-old woman complains of constant headaches for the past 6 months. She has also complained of
female infertility and has been unable to have children, despite having unprotected sexual intercourse with
her husband during the past 15 years. Physical examination reveals deficits in the extraocular movements
bilaterally. Breast examination reveals bilateral female gynecomastia. Which of the following laboratory tests
would be most useful in diagnosing this patient?
a. Ferritin
b. Hemoglobin
c. Hematocrit
d. Iron
e. Prolactin → prolactinoma

11. A 41-year-old woman with Crohn’s disease has undergone multiple surgical procedures. She has recently
undergone an ileostomy but still has evidence of some distal jejunal disease. Her current medications include
prednisone and aminosalicylic acid. Which of the following effects of prolonged therapy with glucocorticoids
are possible for this patient?
a. Antibody production
b. Collagen formation
c. Fibroblast dysfunction
d. Inflammatory cell migration
e. Impaired Wound healing
12. A 49-year-old obese man presents to his primary care physician for a follow-up examination. He has a history
of uncontrolled diabetes mellitus and bipolar disorder. His current medications include lithium and milk of
magnesium. Physical examination of the heart, lungs, and abdomen are within normal limits. Laboratory
studies reveal serum calcium of 14 mg/dL. What is the most likely explanation for these findings?
a. Dietary indiscretion
b. Medication overdose
c. Milk-alkali syndrome → esto es en pacientes que toman muchos antiacidos o beben excesiva cantidad de leche en
pacientes con ulcera gastrica que usan estas cosas para aliviar sus sintomas
d. Parathyroid adenoma
e. Parathyroid hiperplasia

13. A 41-year-old man has chronic gastroesophageal reflux. He is currently managed with an H2-blocker. Physical
examination of the heart, lungs, and abdomen are within normal limits. Which of the following factors would
be least protective of the esophagus in terms of the continued exposure induced by this condition?
a. Arcuate ligament
b. Gastric emptying ability
c. Gravitational effect
d. Salivary gland secretory products
e. Secondary peristaltic waves

14. A 40-year-old woman complains of chest pain and dysphagia to solids. She presents to a specialist for
evaluation. Esophageal manometric studies are performed and reveal high-amplitude contractions and
eventual normal relaxation of the lower esophageal sphincter. Barium swallow is normal. What is the most
likely diagnosis?
a. Cricopharyngeal muscle spasm
b. Diffuse esophageal spasm
c. Scleroderma
d. Tuberculosis
e. Psychogenic swallowing disorder

15. A 5-year-old boy is brought to the emergency department after ingesting liquid drain cleaner. The boy was left
unattended while his baby-sitter was on the telephone. The boy is hoarse and has obvious stridor. What is the
most appropriate initial treatment for this patient?
a. Antibiotics
b. Corticosteroids
c. Induction of vomiting with ipecac
d. Placement of nasogastric tube and lavage
e. Tracheostomy

16. A 76-year-old man with a history of vague right upper quadrant pain, a 25-lb weight loss, and anorexia
presents to his primary care physician for evaluation. Physical examination reveals scleral icterus. Abdominal
examination reveals a right upper quadrant mass. Kidney, ureter, and bladder (KUB) reveals a circular
calcification in the right upper quadrant. Exploratory laparotomy reveals a neoplastic process involving the
gallbladder and liver. What is the most likely pathology causing this condition?
a. Adenocarcinoma 80%
b. Sarcoma
c. Squamous cell carcinoma 5%. 10% anaplastic carcinoma
d. Transitional cell carcinoma
e. Tuberculosis granuloma

17. A 38-year-old woman presents to her primary care physician for evaluation of intermittent vague right
upper quadrant pain. She has a history of hypothyroidism and hypertension. Her current medications include synthetic
thyroid hormone replacement and a calcium channel blocker. Physical examination reveals mild right upper quadrant
pain to deep palpation. Ultrasound reveals a 3-cm gallstone. What is the most likely type of stone to be present in this
patient?
a. Black gallstone pequeñas multiples
b. Brown gallstone pequeñas multiples
c. Calcium oxalate gallstone: in the kidney
d. Type I cholesterol stone pequeñas, usually multiples
e. Type II cholesterol Stone 5-20% grandes
18. A 46-year-old woman presents to the emergency department complaining of right upper quadrant pain and a
fever to 102°F.Physical examination reveals scleral icterus and significant right upper quadrant pain to
palpation. Peritoneal signs are absent. Bowel sounds are present. Which of the following should be included in
the initial treatment of this patient?
a. Antibiotics → + fluid resuscitation is the first step (cholangitis porque tiene la triada)
b. Choledochojejunostomy → cuando el ducto biliar esta dilatado
c. Decompression with T-tube → second step also
d. Endoscopic sphincterotomy
e. Percutaneous transhepatic drainage → second step

19. A 17-year-old boy is brought to the emergency department after suffering from chest pain and dyspnea during
a pickup basketball game. Physical examination reveals a systolic crescendo-decrescendo murmur, heard best
at the second right intercostal space. The murmur radiates to the right carotid artery. Chest x-ray reveals a
normal heart size. Which of the following findings would be expected to be seen on an electrocardiogram in
this patient?
a. Inversion of T waves in leads V1–V4
b. Left ventricular hypertrophy (estenosis aortica)
c. Right bundle branch block
d. Right ventricular hypertrophy
e. Right atrial hypertrophy

20. A 72-year-old man collapses while walking in a shopping mall. He is pulseless and apneic. There is no history of
trauma. Cardiopulmonary resuscitation is started until a rescue squad arrives. Advanced cardiac life support
protocol is initiated. He is pronounced dead 40 minutes later. Autopsy reveals myocardial necrosis with
rupture of the left ventricle. Which of the following is the most likely risk factor that contributed to his death?
a. Family history of diabetes mellitus
b. Hypotension
c. Obesity
d. Sedentary lifestyle
e. Trauma

21. A 57-year-old man is brought to the emergency department complaining of dyspnea and chest pain. He also
admits to a 20-lb weight loss. He complains of fevers, chills, and night sweats. Physical examination reveals
supraclavicular adenopathy. Chest examination reveals distant heart sounds. Laboratory studies reveal a white
blood cell count of 170,000/ml. Chest x-ray and echocardiography reveal a pericardial effusion. What is the
most likely explanation of these findings?
a. Atrial myxoma
b. Atrial fibrillation
c. Lymphoma
d. Metastatic colorectal carcinoma
e. Pericarditis

22. A newborn male has an opening of the abdominal wall at the umbilicus. He has no other prior medical or
surgical history. Birth history was unremarkable. During the remainder of the physical examination and
diagnostic testing, which of the following findings are most likely?
a. Cleft lip
b. Cleft palate
c. Diaphragmatic hernia → onfalocele puede ser parte de la Pentalogia de Cantrell que se asocia con: hernia
diafragmática, esternón hendido, pericardio ausente, defectos intracardiacos, extrofia de la vejiga. Cierre incompleto de
los somatic folds en la pared abdominal anterior.
d. Pericardium
e. Urinary bladder in retroperitoneum

23. A 44-year-old male construction worker undergoes a right inguinal hernia repair. The surgical procedure is
uneventful. He has no prior medical or surgical history. He returns for follow-up on postoperative day 3 for a
wound check. The wound is clean, dry, and intact. What is the optimal convalescent period required before
returning to work for this patient?
a. 1 week
b. 4 weeks
c. 6 to 8 weeks
d. 12 weeks
e. Unknown

24. A 40-year-old woman undergoes repair of a right femoral hernia. During the procedure, the femoral canal is
dissected. The anatomic boundaries of the femoral canal include which of the following?
a. Cooper ligament. Bounded superiorly and medially by the iliopubic tract, inferior by cooper ligament, laterally by the
femoral vein.
b. Inguinal ligament
c. Ischial spine
d. Lacunar ligament
e. Nerve (femoral)

25. A 53-year-old man undergoes a radical prostatectomy for presumed organ-confined prostate cancer. The most
important factor in maintaining continence after radical prostatectomy is preservation of the:
a. Bladder neck
b. External urethral sphincter
c. Levator ani muscle complex
d. Nervi erigentes
e. Puboprostatic ligaments

26. A 27-year-old man has bulky retroperitoneal adenopathy after radical orchiectomy for a mixed germ cell
tumor. His chest x-ray is normal. Serum beta-human chorionic gonadotropin (B-hCG) and alpha-fetoprotein
(AFP) are markedly elevated. Liver enzymes are slightly elevated, and the patient relates a history of ethanol
excess. He receives three cycles of chemotherapy. Restaging reveals a 3-cm retroperitoneal mass, a normal
chest x-ray, and normal serum B-hCG. However, the serum AFP is 20 IU/mL (normal _ 0 to 9 IU/mL). What is
the next step in the management of this patient?
a. Computed tomography (CT)—guided needle biopsy
b. External-beam radiotherapy
c. Retroperitoneal lymph node dissection → hepatitis alcoholica
d. Salvage chemotherapy
e. Serial markers and CT scans

27. A 63-year-old man is disease-free two years after bacillus Calmette-Guerin therapy for carcinoma in situ and a
grade 2, stage T1 bladder cancer. In addition to physical examination, cystoscopy, and urinary cytology,
evaluation at this time should include:
a. Intravenous pyelogram
b. Prostatic urethral biopsy
c. Random biopsies of the bladder
d. Selective upper tract cytology
e. Urinary voided cytology, repeated three times

28. A 78-year-old man presents to the emergency department for evaluation of progressive right upper quadrant
pain, nausea, vomiting, and a 30-lb weight loss in the past 3 months. He has a prior medical history of
cholelithiasis, diabetes mellitus, hypertension, and dementia. Physical examination reveals scleral icterus
bilaterally. Abdominal examination reveals right upper quadrant tenderness and a palpable mass. Peritoneal
signs are absent. CT scan reveals pancreatic, duodenal, and choledochal lymph nodes. There is an asymmetric
thickening of the gallbladder. What is the most likely pathologic finding at exploratory laparotomy and biopsy?
a. Adenocarcinoma
b. Fibroma
c. Lipoma
d. Myxoma
e. Myoma

29. An 8-year-old boy undergoes a right upper quadrant ultrasound for persistent right upper quadrant
discomfort. He has no prior medical or surgical history. He has no known allergies and takes no medications.
His mother has a history of gallstones. Ultrasound findings include a fusiform dilation of the common bile duct.
What is the most likely explanation for these findings?
a. Type I choledochal cyst
b. Type II choledochal cyst
c. Type III choledochal cyst
d. Type IV choledochal cyst
e. Type V choledochal cyst

30. An 18-year-old man is stabbed in his abdomen multiple times by an assailant during an altercation involving
sale of illicit drugs. He is brought to the emergency department for evaluation. He has four stab wounds of the
abdomen—three are in the right upper quadrant, and one is in the left lower quadrant. Physical examination
of the abdomen reveals guarding and rebound tenderness. The patient is brought to surgery for an exploratory
laparotomy. A penetrating injury to the gallbladder is found. Which of the following associated viscera are
likely to be injured?
a. Aorta
b. Colon
c. Kidney
d. Liver
e. Urinary bladder

31. A 62-year-old woman presents to her primary care physician with a cough. She also complains of hemoptysis.
Social history reveals a 55-pack-a-year history of smoking. She is a recovering alcoholic. Physical examination
reveals bilateral wheezes. Cardiac, pulmonary, and abdominal examinations are unremarkable. Laboratory
values reveal serum calcium of 13 mg/dL. Serum protein electrophoresis shows no abnormal spikes. What is
the most likely diagnosis?
a. Goodpasture’s syndrome
b. Myeloma
c. Renal adenoma
d. Small-cell carcinoma of the lung
e. Squamous cell carcinoma of the lung

32. A 10-year-old boy is brought to his primary care physician for evaluation of persistent hoarseness. He has just
begun to participate with his school chorus and notes that his hoarseness worsens with singing. Physical
examination of the heart, lungs, and abdomen are unremarkable. Fiber-optic flexible laryngeal examination
reveals multiple lesions on his true vocal cords. What is the most likely diagnosis?
a. Gastroesophageal reflux
b. Granulomatous inflammation of the pharynx
c. Laryngeal papilloma
d. Singer’s nodule
e. Thyroid carcinoma

33. A 75-year-old man presents to his primary care physician because of hoarseness. He has a 60-pack-a-year
history of smoking. He also complains of a 25-lb weight loss over the past 4 months. Direct laryngoscopy
reveals a sessile mass on the high right vocal cord. He also has a palpable lymph node along the right anterior
cervical lymph node chain. If dysplasia is found on biopsy of the laryngeal lesion, what is the most likely
diagnosis?
a. Adenoma
b. Laryngeal polyp
c. Laryngitis
d. Mucoepidermoid cystic disease
e. Squamous cell carcinoma

34. A 21-year-old male college student presents to the outpatient clinic for a routine examination at the beginning
of the fall semester. He has a history of irritable bowel syndrome. Physical examination of the heart, lungs, and
abdomen are unremarkable. Genitourinary examination reveals that the testes are descended bilaterally. A
left grade 1 varicocele is present. There are no testicular masses. The penis is uncircumcised, and the foreskin
is unable to be retracted behind the glans. What is the most likely diagnosis?
a. Balanitis
b. Hypospadias
c. Epispadias
d. Paraphimosis
e. Phimosis
35. A 71-year-old white male presents to his primary care physician complaining of a 1-month history of nocturia,
polyuria, and difficulty starting and stopping his urinary stream. His American Urological Association Symptom
Score is 17/35. Physical examination of the prostate reveals an enlarged gland without masses. His testes are
descended bilaterally. He has a small right hydrocele that transilluminates. His prostate-specific antigen (PSA)
level is 6 ng/mL, and urinalysis is negative. The patient is begun on dutasteride 0.5 mg daily. What is a likely
result of taking this medication?
a. Ejaculatory dysfunction 5-8%
b. Maximal change in urinary flow rate
c. Prostate size decreases by 25%
d. Serum PSA increases by 50%
e. Symptom score remains unchanged

36. A 34-year-old white male has a painless enlargement of his right testis in the past 4 months. He is brought to
his primary care physician by his girlfriend, who urges him to seek evaluation. He has recently become
depressed because of this problem. He had a cryptorchid right testis as an infant, which was surgically
corrected. A scrotal ultrasound confirms the presence of a 3 cm hypoechoic right testicular mass .What is the
most likely diagnosis?
a. Choriocarcinoma
b. Embryonal (mixed germ cell) carcinoma
c. Endodermal (yolk sac) tumor
d. Seminoma
e. Teratoma

37. A 51-year-old man is found to have an intracranial mass and will undergo resection. The surgical procedure is
performed via a transoccipital approach. In this approach, the patient develops a cerebrospinal fluid (CSF) leak.
Which of the following statements is true regarding CSF?
a. Arachnoid villi act as two-way valves.
b. Arachnoid villi open at a pressure of 5 mm Hg.
c. CSF is absorbed through the spinal roots.
d. CSF enters through the foramen of Magendie.
e. Total CSF volume is 150 L.

38. A 19-year-old college student is driving under the influence of alcohol, despite recommendations from friends
not to drive. She is struck by another driver. The force of impact causes her to strike the temporal area of her
skull against the window. She develops a mild headache but does not lose consciousness. Several hours later,
she develops a severe headache with nausea and vomiting. Which is the most likely diagnosis?
a. Bacterial infection
b. Berry aneurysm
c. Epidural hematoma
d. Subarachnoid hematoma
e. Subdural hemorrhage

39. A 59-year-old man presents to his primary care physician complaining of progressive right-sided hearing loss
and gait unsteadiness. He states that when he uses the phone, he must use his left ear to listen instead of his
right ear. He has a past medical history of hypertension. His current medications include a calcium channel
blocker. Physical examination reveals loss of the right corneal reflex and facial weakness. Cardiac, pulmonary,
and abdominal examinations are within normal limits. What is the most appropriate next best step in the
diagnosis of this patient?
a. Audiometric testing
b. Brainstem-evoked potential testing
c. CT scan of the head without contrast
d. MRI of the head → neuroma acustico
e. Nystagmography

40. A 47-year-old man with a history of end-stage pulmonary disease of his right lung is scheduled for lung
transplantation. Preoperative cardiac function is good. He has no history of congenital defects. Which of the
following is the most appropriate surgical incision for this patient to have?
a. Chevron abdominal→ renal surgery
b. Lateral thoracotomy
c. Midline abdominal
d. Transverse anterior thoracotomy→ esta se usa si es doble transplante
e. Pfannenstiel

41. A 47-year-old man with multiple medical problems and end-stage pulmonary parenchymal disease undergoes
lung transplantation. He has a prior medical history of obstructive lung disease. He has an uncle with cystic
fibrosis. His father has restrictive lung disease, and his brother has pulmonary hypertension. Which of the
following portends the best survival after lung transplantation for this patient?
a. Bronchogenic carcinoma
b. Cystic fibrosis→ le sigue en supervivencia a EPOC
c. Obstructive lung disease
d. Pulmonary hypertension→ es la peor
e. Restrictive lung disease
42. A 4-year-old boy is on the waiting list for a liver transplant. He has end-stage hepatic disease and is currently
hospitalized for esophageal variceal hemorrhage. What is the most likely cause of liver failure in this patient?
a. Biliary atresia 50%
b. Hepatitis A
c. Primary biliary cirrhosis
d. Primary sclerosing cholangitis
e. Tuberculosis

43. A 23-year-old woman who complains of greasy and odorous stools, generalized weakness, and hair loss
presents to her primary care physician for evaluation. Physical examination of the heart, lungs, and abdomen
are unremarkable. She has no guarding or rebound tenderness. Bowel sounds are present in all quadrants.
Female pelvic examination was deferred at the patient’s request. What is the most likely explanation of these
findings?
a. Gastric ulcer with bleeding
b. Glucose malabsorption
c. Menstruation
d. Pancreatic insufficiency→ usualmente en pacientes con fibrosis quística se presenta con malabsorción y esteatorrea
severa
e. Pituitary tumor

44. A 27-year-old woman is 12 hours status after cadaveric pancreas transplantation and currently in the surgical
intensive care unit. She has a medical history of insulin-dependent diabetes since age 5. Her vital signs are
normal. Chest is clear to auscultation, and cardiac examination reveals a regular rate with a regular rhythm.
Wound dressing is clean, dry, and intact. Which of the following is the best method of monitoring the
transplanted pancreas?
a. Serum amylase level
b. Serum glucose level
c. Serum insulin level
d. Ultrasonography of the pancreatic vessels
e. Urinary amylase level

45. A 44-year-old man with recurrent pancreatitis is brought to the emergency department with another bout of
pancreatitis. Which of the following is the most reassuring factors regarding the severity of his condition?
a. Age
b. Blood glucose level of 300 mg/dL
c. Lactate dehydrogenase level of 400 IU/L
d. Serum calcium of 6 mg/dL
e. Serum hematocrit level of 29%

46. A 41-year-old man with a long history of renal stones and hypercalcemia is found to have an adenoma of the
right superior parathyroid gland. He is going to undergo surgical excision of this lesion. What is the best
surgical landmark for this lesion?
a. Bifurcation of the carotid arteries
b. Carotid sinus
c. Junction of the inferior thyroid artery and recurrent laryngeal nerve
d. Junction of the upper and middle third of the thyroid gland
e. Recurrent laryngeal nerve

47. Which of the following techniques is best used to define an enlarged parathyroid gland?
a. CT scan of the neck
b. Dual tracer imaging
c. MRI of the neck
d. Thyrocervical angiography
e. Ultrasonography

48. A 44-year-old man with end-stage renal disease successfully undergoes renal transplantation. He has a prior
medical history of hyperparathyroidism. Six months after renal transplantation, his serum calcium is still
13mg/d. Which of the following laboratory findings are possible in this patient?
a. Elevated serum phosphate
b. Elevated serum lactic acid dehydrogenase
c. Elevated urine calcium→ hiperparatiroidismo terciario que ocurre en pacientes que tienen ERC a pesar de un
transplante exitoso.
d. Elevated urine creatinine
e. Elevated urine protein

49. A 46-year-old man presents to his primary care physician for evaluation of a skin lesion. He complains of
hypopigmentation of the skin of his lower back. He has a prior medical history of eczema and basal cell
carcinoma. He is a farmer who spends a great deal of time outdoors. What cells are responsible for this
condition?
a. Adipocytes
b. Keratin-producing cells
c. Langerhans cells
d. Melanocytes
e. Merkel cells

50. A 69-year-old male presents to his dermatologist with a lesion present on his nose. He is a gardener who
spends a great deal of his time outdoors. He has a prior medical history of allergic rhinitis, hypertension, and
diabetes mellitus. His current medications include a beta-blocker and an oral hypoglycemic. Physical
examination of his nose reveals a raised, shiny, papular lesion with small blood vessels. What is the most likely
diagnosis?
a. Basal cell carcinoma
b. Histiocytosis X
c. Melanoma
d. Seborrheic keratosis
e. Squamous cell carcinoma

51. A 29-year-old Black woman presents to her primary care physician because of a growth on her left ear, which
occurred after she had her ear pierced for the first time a week ago. She noticed that her ear seemed to
develop a growth on it quite rapidly. She had never had her ear pierced before. What is the most likely
explanation for these findings?
a. Basal cell carcinoma
b. Blue nevus
c. Juvenile melanoma
d. Keloid
e. Molluscum contagiosum

52. A 52-year-old Asian American female has melanotic pigmentation of the buccal mucosa, lips, and digits.
Colonoscopy reveals hamartomas throughout the gastrointestinal tract. The polyps were removed because of
her increased risk of cancer. What other cancer is associated with this condition?
a. Cervical cancer
b. Kidney cancer
c. Liver cancer
d. Ovarian cáncer → Peutz Jegher Syndrome
e. Pancreatic cancer
53. A 45-year-old female complains of chronic diarrhea and sweating. Colonoscopy is performed, and a biopsy of a
lesion in her ileum is performed. The pathology report shows that the tumor is composed of neuroendocrine
cells. What is a medical treatment for this condition?
a. Corticosteroids, intravenous
b Corticosteroids, topical
c. Furosemide
d. Octreotide→ tx de tumor carcinoide, análogo de somatostatina
e. Tetracycline

54. An 18-year-old male is brought to the emergency department with sudden excruciating abdominal pain
localized to the right lower quadrant, nausea and vomiting, mild fever, and slight tachycardia. He has a prior
medical history of recurrent otitis media. Physical examination reveals marked right lower rebound tenderness
and guarding. Serum white blood cell count is 18,000/ml. KUB x-ray reveals bowel gas in the small and large
bowel. What is the most likely diagnosis?
a. Appendicitis
b. Crohn’s disease
c. Diverticulitis
d. Pancreatitis
e. Ulcerative colitis

55. A 39-year-old woman presents to the emergency department complaining of severe abdominal pain. She has a
history of peptic ulcer disease. Physical examination reveals guarding and rebound tenderness. She is taken to
the operating room for exploratory laparotomy. During the procedure, the surgeon who opens the
gastrosplenic ligament to reach the lesser sac accidentally cuts an artery. Which of the following vessels is the
most likely one injured?
a. Gastroduodenal artery
b. Left gastric artery
c. Left gastroepiploic artery
d. Right gastric artery
e. Splenic artery

56. A 2-year-old female is brought to the emergency department because of several episodes of rectal bleeding. A
technetium-99m perfusion scan reveals a 3-cm ileal outpouching located 50 cm from the ileocecal valve.
Which of the following types of ectopic tissue does this structure most likely contain?
a. Duodenal
b. Esophageal
c. Gastric → diverticulo de Meckel
d. Hepatic
e. Jejunal

57. A 39-year-old woman complains of epigastric pain with eating over the past 3 or 4 months. She admits to a
history of chronic back problems. She notes weight gain of 20 lb in the past 4 months. She denies the use of
nonsteroidal anti-inflammatory agents. She denies nausea and vomiting. Physical examination of the heart,
lungs, and abdomen are within normal limits. What is the most likely pathogen associated with this condition?
a. Enterohemorrhagic Escherichia coli
b. Escherichia coli
c. Helicobacter pylori
d. Shigella sonnei
e. Streptococcus pyogenes

58. A 59-year-old man was injured in a car accident. An abdominal CT scan reveals a ruptured spleen. His blood
pressure is 90/40 mm Hg, and his pulse is 140 beats per minute. The patient is taken for laparotomy.
Splenectomy is performed. Which of the following laboratory abnormalities is likely after this procedure?
a. Anemia
b. Basophilia
c. Eosinophilia
d. Thrombocytopenia
e. Thrombocytosis
59. A 19-year-old man was kicked in the abdomen during a fight in a bar. He went to his primary care physician,
who ordered a CT scan, which revealed a subcapsular splenic hematoma. The man was told to restrict physical
activity. Two weeks later, he presents to the emergency department because of severe abdominal pain. He
undergoes a splenectomy. Postoperatively, a peripheral smear is ordered. Which type of cell can be found in
this patient?
a. Basophilic stippling
b. Blister cells
c. Howell-Jolly bodies
d. Nucleated red blood cells
e. Spherocytes

60. A 15-year-old African American male underwent a splenectomy after sustaining a knife injury during a fight. He
presents to his primary care physician for a sports physical. His mother read on the Web that he is at increased
risk for infection. He should receive which of the following vaccines to prevent serious infections?
a. Rubella
b. Measles
c. Tetanus
d. Vaccines against common encapsulated organisms
e. Varicella

61. A 53-year-old woman presents to her primary care physician with a 12-month history of neck pain. She
complains of a 15-lb weight gain and generalized malaise. She has a past medical history of hypertension and
diabetes mellitus. Her current medications include an oral hypoglycemic. Physical examination reveals
tenderness along the course of the thyroid gland without evidence of a discrete mass. What is the most likely
diagnosis?
a. Acute thyroiditis
b. Hashimoto thyroiditis
c. Papillary thyroid carcinoma
d. Riedel thyroiditis
e. Subacute thyroiditis

62. A 47-year-old woman with a history of a left thyroid mass undergoes left thyroid lobectomy. Pathology reveals
a 1.3-cm papillary carcinoma with no evidence of extracapsular extension. What is the most appropriate next
step in the treatment of this patient?
a. External-beam radiotherapy
b. Multiagent chemotherapy
c. Subtotal thyroidectomy
d. Total thyroidectomy
e. Watchful waiting with periodic follow-up

63. A 34-year-old man with a thyroid nodule is undergoing a neck exploration. During the procedure, it is possible
that he will undergo thyroidectomy. Which of the following statements about the superior laryngeal nerve and
the innervation of the thyroid gland is correct?
a. Injury to the nerve causes bowing of the vocal cords during phonation.
b. Nerve injury may be unnoticeable in singers.
c. The nerve is rarely at risk during thyroid surgical procedures.
d. The superior laryngeal nerve is chiefly a motor nerve.
e. The superior laryngeal nerve is chiefly a sensory nerve.

64. A 19-year-old man leaps from the third floor of his dormitory in an apparent suicide attempt. He is brought to
the emergency department unconscious. He has visible head and lower extremity injuries. He has a pulse of
110 beats per minute but is apneic. What is the best airway management for this patient?
a. Nasotracheal intubation
b. Oral intubation
c. Oral intubation with head-chin lift
d. Tracheostomy
e. Intubation is not necessary for this patient.
65. A 21-year-old woman is stabbed in the chest by her boyfriend. She is brought to the emergency department
for evaluation. Her blood pressure is 130/80 mmHg, and her pulse is 90 beats per minute. Physical
examination reveals a single stab wound to the left fifth intercostal space in the midclavicular line. Neck
examination is normal. Trachea is midline, and the jugular veins are not distended. She does have decreased
breath sounds in the left lung fields. Which of the following diagnoses can be ruled out on the basis of the
above information?
a. Large left hemothorax
b. Open pneumothorax¿?
c. Pericardial tamponade
d. Rupture of the left main stem bronchus
e. Tension pneumothorax

66. A 41-year-old man suffers a traumatic amputation of three of his fingers in a meat slicer. He has no prior
medical or surgical history. Which of the following modalities should be used to transport the amputated
fingers with the patient?
a. Place in clean plastic bag and pack with dry ice.
b. Place in clean plastic bag filled with room temperature water.
c. Place in clean plastic bag in a chest filled with crushed ice and water → se preserva hasta 18h.
d. Place in clean plastic bag filled with hot water.
e. Wrap the amputated fingers in sterile dry gauze.

67. A 19-year-old woman presents to the emergency department after sustaining an injury to her right eye while
placing her contact lens. She has significant right eye pain. She has a prior medical history of seasonal allergies.
Physical examination reveals a simple abrasion. Fluorescein testing is performed and reveals no evidence of a
stained epithelial defect. This rules out the possibility of which of the following?
a. Bacterial infection
b. Iritis
c. Trauma
d. Viral infection
e. Ulcer

68. A 29-year-old man who works in a factory sustained a foreign body injury to his right eye when a piece of
metal shot off a conveyer belt. He is brought to the emergency department for evaluation. Physical
examination of the right eye reveals a metallic foreign body on the eye with an epithelial rust ring. What is the
most useful instrument to remove this foreign body?
a. Cyanoacrylate glue
b. Eye burr
c. Eye spud
d. Fine needle tip
e. Sterile water and alcohol

69. A 37-year-old chemistry teacher sustains a chemical splash of acid to his right eye while attempting to perform
a demonstration to his high school science class. He is in significant pain. While in the classroom and waiting
for an ambulance to transport him to the hospital, which of the following interventions should be performed?
a. Eyedrop instillation with normal saline
b. Eye patch placement
c. Flush eye with 1 to 2 L of normal saline
d. Placement of eye under direct sunlight
e. Watchful waiting until ambulance arrives

70. A 37-year-old construction worker sustained a crush injury to his right thigh after a crane fell on his leg at the
work site. He is brought to the emergency department for evaluation .He has significant right leg pain and pain
with passive stretch. The leg is tense to palpation. What is the most likely intracompartmental pressure
measurement of this patient’s right leg?
a. 5 mm Hg
b. 10 mm Hg
c. 15 mm Hg
d. 25 mm Hg
e. 35 mm Hg
71. A 41-year-old woman who cleans houses for a living presents to her primary care physician complaining of
tenderness in her right knee. The pain is constant and has been present for 3 weeks. She is in a monogamous
relationship. Physical examination reveals that her knee is slightly swollen and tender. Cardiac, pulmonary, and
abdominal examinations are within normal limits. A synovial aspiration is performed. The evaluation reveals
no evidence of crystals or bacteria. What is the most likely diagnosis?
a. Bursitis
b. Infectious arthritis
c. Rheumatoid arthritis
d. Septic thrombophlebitis
e. Trauma-induced infectious arthritis

72. A 12-year-old boy who is the star pitcher of his little league team complains of right shoulder pain. This is his
pitching arm. He has no prior medical or surgical history. Physical examination reveals weakness of the rotator
cuff tendon. What is the most appropriate treatment for this patient?
a. Injection of corticosteroids
b. Intravenous corticosteroids
c. Rest, elevation, and anti-inflammatory agents
d. Sling placement
e. Surgical repair

73. A 65-year-old man with a history of coronary artery disease is undergoing an aortobifemoral bypass. Which of
the following intraoperative management maneuvers will decrease his risk of intraoperative myocardial
infarction?
a. Beta blockade
b. Calcium channel blockade
c. Administration of normal saline instead of Lactated Ringer’s solution
d. Use of propofol
e. Use of morphine

74. A 35-year-old healthy man is diagnosed with an inguinal hernia. He has no history of abnormal bleeding. Which
of the following tests is absolutely required prior to taking him to the operating room?
a. Hematocrit
b. Platelet count
c. Potassium
d. White blood cell count
e. None of the above

75. A 50-year-old man has diarrhea after an uncomplicated bowel resection. The fluid choice that most closely
resembles his output is
a. Normal saline
b. Half normal saline with 20 mEq of potassium
c. D5W with 3 amp bicarbonate
d. Lactated Ringer’s solution
e. D5NS

76. A 27-year-old man is brought to the emergency department after slashing his hand with a knife while
attempting to slice a bagel. He has a prior medical history of recurrent sinus infections. His prior surgical
history is notable for repair of a nasal fracture. Physical examination reveals a 4-cm clean laceration along the
palmar aspect of his hand. The principles most relevant for this case would be:
a. Gentle handling of tissue and closure without tension
b. Skin grafting when necessary to cover area
c. Debridement of devitalized tissue
d. Use of vacuum sponge to promote healing
e. Flap reconstruction with vascularized tissue

77. A 68-year-old man is brought to the emergency department complaining of abdominal and leg pain for
2 weeks. He has a history of hypertension and hypercholesterolemia; he weighs 290 lb. Physical examination reveals a
pulsatile midline abdominal mass. The lower extremities have unequal pulses. Which of the following is the best next
step in the evaluation of this patient?
a. Aortogram
b. CT scan of the abdomen and pelvis
c. Ultrasound, kidneys and bladder
d. Ultrasound, liver and spleen
e. Venacavogram

78. A 28-year-old woman presents to her physician for evaluation of a lump in her right breast found on self
examination. She has a family history of breast cancer in that her mother died in her early 40s from this
condition. The mother had a modified radical mastectomy followed by chemotherapy. Physical examination
reveals a breast lump that is freely mobile and well circumscribed. There is no dimpling, asymmetry, or
retractions. The lesion measures 2 cm. What is the next step in the management of this patient?
a. Biopsy of the lesion with sonographic guidance
b. Mammography followed by stereotactic CT scan
c. Testing for BRCA gene
d. Ultrasound of the breast and consideration for breast biopsy
e. Watchful waiting and follow-up examination by primary care physician in 1 year

79. A 28-year-old man with a history of recurrent abdominal pain and bloody diarrhea presents to his physician
complaining of significant rectal pain with bowel movements. He has lost 15 pounds in the last 3 months.
Physical examination reveals right and left lower quadrant pain to palpation. Laboratory values reveal a
hematocrit of 28% and an elevated erythrocyte sedimentation rate. Colonoscopy performed in this patient
would likely reveal:
a. Colonic mass lesion
b. Diverticulosis
c. Internal hemorrhoids
d. Normal bowel
e. Thickened friable colonic and rectal mucosa → colitis ulcerativa

80. A 55-year-old male presents to his physician complaining of polyuria, polydipsia, polyphagia, and a red, scaly
rash on his face within the past 2 weeks. He voids 18 times/day with a good force of stream. Fasting blood
glucose was 325 mg/dL. He has lost 20 pounds in the past 2 months and has never had elevated blood glucose
levels in the past. Physical examination of the heart, lungs, and abdomen are normal. What is the most likely
explanation for these findings?
a. Diabetes mellitus type I
b. Diabetes mellitus type II
c. Glucagonoma
d. Insulinoma
e. Verner-Morrison Disease

81. A 72-year-old man with an 80 pack-year history of smoking presents to his physician complaining of weakness
and malaise. He has recently developed dysphagia to solid foods. He has lost 15 pounds in the last 3 months.
Physical examination reveals right supraclavicular lymphadenopathy. Cardiac and pulmonary examinations are
unremarkable. He has no guarding or rebound tenderness. Which of the following studies will provide the
most definitive diagnosis?
a. Barium esophagogram
b. CT scan of the abdomen and pelvis
c. Esophagogastroscopy with biopsy
d. MRI of the abdomen
e. Ultrasound of the right upper quadrant

82. A 42-year-old African American female undergoes a laparoscopic cholecystectomy for chronic right upper
quadrant pain. CT scan demonstrated gallstones and pericholecystic fluid. The surgical procedure was
uncomplicated. Pathologic analysis of the gallstones revealed calcium bilirubinate stones. What is the most
likely explanation of these findings?
a. High serum cholesterol levels
b. High serum lipid levels
c. High-fat diet
d. Sickle cell anemia
e. Tumor

83. A 36-year-old female complains of jaundice and peripheral swelling. An echocardiogram is ordered and the
patient is determined to have right-sided heart failure with hepatic congestion and peripheral edema. No
murmur is detected. What is the most appropriate explanation for these findings?
a. Aortic stenosis
b. Atrial septal defect
c. Patent ductus arteriosus
d. Tetralogy of Fallot
e. Ventricular septal defect

84. A newborn male born at term to a 27-year-old intravenous drug-abusing female is found to have a small
umbilical hernia. His vital signs are stable. His cardiac and pulmonary examinations are noncontributory. Which
of the following is the most likely explanation for this finding?
a. Patent foramen ovale
b. Patent omphalomesenteric duct
c. Patent umbilical ring
d. Patent urachus
e. Patent vitelline duct

85. A 28-year-old man presents to the emergency department complaining of left flank pain, nausea, and
vomiting. Physical examination of the heart and lungs are normal. There is left costovertebral (CVA)
tenderness. Urinalysis reveals microhematuria (5 red blood cells/high-power field). CT scan reveals a left-sided
4-mm renal stone, whereas KUB reveals a normal bowel gas pattern and no evidence of calcifications. What is
the most likely explanation of these findings?
a. Calcium oxalate stone
b. Calcium phosphate monohydrate stone
c. Calcium phosphate dehydrate stone
d. Small struvite stone
e. Uric acid Stone

86. A 47-year-old woman with gallstone pancreatitis is hospitalized. She has a history of hypertension and
hypercholesterolemia. Early cholecystectomy is indicated to prevent which of the following complications?
a. Cholangitis
b. Recurrent pancreatitis
c. Gastric ulcer
d. Gallstone ileus
e. Gallbladder perforation

87. A 56-year-old man complains of recurrent cough and hemoptysis. He has a history of recurrent pneumonias.
He is a nonsmoker and has no occupational risk for pulmonary disease. Physical examination reveals decreased
breath sounds in the right upper lobe. Chest x-ray reveals a small right upper quadrant mass. Bronchoscopy
reveals an angioma. What is the most appropriate treatment for this patient?
a. Antibiotics
b. Corticosteroids
c. Left lobectomy
d. Right upper lobe lobectomy
e. Watchful waiting

88. A 55-year-old male presents to his primary care physician after noticing some blood-tinged urine 1 week ago.
He denies any current pain and denies any fevers. His past medical history includes chronic obstructive
pulmonary disease from many years of smoking cigarettes. CT scan reveals bilateral renal simple cysts,
prostate enlargement, and an asymmetric thickening of the left side of the bladder. Left hydronephrosis is also
present. His urinalysis is positive for microscopic hematuria. What is the most likely diagnosis?
a. Nephrolithiasis
b. Prostate cancer
c. Renal cell carcinoma
d. Transitional cell carcinoma of the bladder
e. Urinary tract infection

89. A 30-year-old man undergoes a CT scan of his abdomen after a motor vehicle accident. He was an unrestrained
driver and was thrown from the vehicle. No acute abdominal injuries are found. The CT scan reveals bilateral
enlarged kidneys with multiple cysts present in varying sizes. The right kidney is 15 cm and the left kidney is 16
cm in length. Physical examination of the heart, lungs, and abdomen are within normal limits, other than some
mild tenderness to palpation in the right and left upper quadrants. Which of the following central nervous
system pathologies are most strongly associated with this finding?
a. Circle of Willis aneurysm
b. Cysticercosis
c. Infarction
d. Glioma
e. Subdural hematoma

90. A 46-year-old female with polycystic kidney disease is receiving a kidney transplant. Within minutes of the
anastomoses of the renal artery and vein to the respective external iliac artery and vein, the kidney rapidly
regains a pink coloration and normal tissue turgor and begins excreting urine. The patient is discharged from
the hospital and is seen at 1 month follow up. Serum creatinine is 4.2 mg/dL. Urine output is 20 mL/hour.
Physical examination of the heart, lungs, and abdomen are within normal limits. The transplanted kidney is
palpable in the right iliac fossa. Subsequent biopsy of the transplant shows extensive inflammation and
edema. What is the most likely explanation for these findings?
a. Acute transplant rejection
b. Chronic transplant rejection
c. Graft-versus-host disease
d. Hyperacute transplant rejection
e. Normal posttransplant process

91. A 52-year-old man complains of chronic abdominal pain. He has been hospitalized seven times in the last 2
years for recurrent attacks of pain from chronic pancreatitis. He has been treated with analgesics and a partial
distal pancreatectomy. His pain has still persisted. What is the next step in the treatment of this patient?
a. Continued use of oral analgesics
b. Corticosteroids
c. Splanchnicectomy
d. Total pancreatectomy
e. Watchful waiting

92. A 39-year-old man is evaluated by his physician for recurrent kidney stones. He has been treated in the past
with extracorporeal shock wave lithotripsy, ureteroscopy, and a percutaneous nephrolithotripsy. Which of the
following characteristics would suggest the diagnosis of primary hyperparathyroidism?
a. 1-mm stone on KUB x-ray
b. 2-mm stone on KUB x-ray
c. 1-mm and 2-mm right-sided stone on KUB x-ray
d. 1-mm, 2-mm, and 3-mm right- and left-sided stones on KUB x-ray
e. 4-mm right lower pole stone on CT scan

93. A physician wishes to deliver a local anesthetic subcutaneously to a patient with a suspicious lesion on his
forehead. The lesion measures 1 cm and the surgeon plans an elliptical incision to remove it. Which of the
following epidermal skin layers will the physician penetrate first with the local anesthetic needle?
a. Stratum basale
b. Stratum corneum
c. Stratum granulosum
d. Stratum lucidum
e. Stratum spinosum

94. A 35-year-old man with a history of Crohn’s disease presents to his physician for a follow-up examination. He
has diffuse ileocolonic disease on a recent CT scan. His current medications include sulfasalazine. Physical
examination reveals right lower quadrant pain to deep palpation. Should antibiotic therapy be considered in
this patient, which of the following organisms should be targeted?
a. Mycobacterium species
b. Pseudomonas aeruginosa
c. Staphylococcus aureus
d. Streptococcus pneumoniae
e. Streptococcus pyogenes

95. A 46-year-old woman presents to her physician with a history of progressive dysphagia. She has a history of a
15-lb weight loss in the last 6 months. Physical examination of the neck, heart, lungs, and abdomen are
noncontributory. Laboratory testing reveals a hematocrit of 33% and a mean corpuscular volume of 70. Upper
gastrointestinal endoscopy reveals an esophageal web. What is the most likely diagnosis?
a. Carcinoid tumor
b. Leukemia
c. Lymphoma
d. Plummer-Vinson syndrome→ carcinoma postcricoideo
e. Rheumatoid arthritis

96. A 47-year-old man is involved in a motor vehicle accident. He suffers fractures of ribs 9, 10, and 11 on the left
side. He is hemodynamically unstable and has a blood pressure of 90/50 mm Hg and pulse of 120
beats/minute despite transfusion of 3 U of packed red blood cells. He is taken to the operating room for
exploratory laparotomy. A ruptured spleen is identified and removed. In searching for a potential accessory
spleen, which is the most likely location to encounter it?
a. Greater omentum
b. Right lower quadrant
c. Right upper quadrant
d. Splenic hilum
e. Splenic ligaments

97. A 12-year-old boy is brought to his physician for evaluation of a neck mass. He has a history of recurrent
sinusitis and tonsillar infections. Physical examination reveals a midline neck mass measuring 1.5 cm that
moves with swallowing. There is no evidence of lymphadenopathy.
What is the most likely diagnosis?
a. Leukemia
b. Lymphoma
c. Thyroglossal duct cyst
d. Thyroglossal fistula
e. Thyroid carcinoma

98. During your Emergency Medicine rotation you are called to the emergency department to evaluate a patient
with an ophthalmic injury. Which of the following ophthalmic trauma injuries requires immediate on-site
management and referral to the ophthalmologist on call?
a. Acid burn
b. Corneal clouding
c. Corneal laceration
d. Hyphema
e. Severe conjunctival chemosis

99. A patient presents to the emergency department after being slashed with a knife in the left leg. The wound
appears clean and the edges are well opposed. Which of the following is the simplest method of wound
closure for this injury?
a. Delayed primary closure
b. Graft
c. Local flaps
d. Primary closure
e. Secondary intention

100. A 23-year-old female presents to her physician complaining of tenderness and pain in the right knee. The pain
is constant throughout the day and has been present for approximately 3 weeks. Social history reveals that she
is in a monogamous relationship. Physical examination reveals that her knee is slightly swollen and tender. A
synovial aspiration is performed. Neither crystals nor bacteria are found. What is the most likely diagnosis?
a. Bursitis
b. Gonococcal arthritis
c. Rheumatoid arthritis
d. Septic bursitis
e. Trauma
TRASPLANTE (Respuestas)

1.- Aplicar la terminología correcta a cada tipo de trasplante:


Autotrasplante (A) Isotrasplante (B) Alogénico (C) Ortotópico (D) Xenotrasplante (E)
a.- donante y receptor son el mismo individuo *(A) (B) (C) (D) (E)
b.- donante y receptor pertenecen a especies diferentes (A) (B) (C) (D) *(E)
c.- donante y receptor de la misma especie son genéticamente idénticos (A) *(B) (C)
(D) (E)
d.- el órgano del donante ocupa el mismo sitio anatómico que el del receptor
(A) (B) (C) *(D) (E)
e.- donante y receptor de la misma especie son genéticamente diferentes
(A) (B) *(C) (D) (E)

2.- El primer trasplante de hígado lo realizó:


a.- Carrel
*b.- Starzl
c.- Hardy
d.- Lillehei
e.- Borel

3.- Respecto a los antígenos de trasplante señalar lo correcto:


a.- El MHC esta localizado en el cromosoma 6 *(si) (no)
b.- El MHC codifica el HLA *(si) (no)
c.- El MHC II (HLA D) se encuentra en la superficie de todas las células nucleadas del
organismo (si) *(no)
d.- Los antígenos menores de histocompatibilidad no están codificados por los genes del
MHC *(si) (no)
e.- Los antígenos de clase I y II se pueden detectar con pruebas serológicas en
laboratorio *(si) (no)

4.- Respecto al fenómeno del rechazo:


a.- el rechazo agudo se inicia por las células T *(si) (no)
b.- un fenómeno de rechazo agudo es obligatorio que ocurra inmediatamente después
del trasplante (si) *(no)
c.- en el rechazo crónico se mantiene la estructura histológica del órgano trasplantado
(si) *(no)
d.- las infecciones recurrentes contribuyen al rechazo crónico *(si) (no)
e.- el rechazo hiperagudo depende de anticuerpos preformados *(si) (no)

5.- Cual de los siguientes objetivos del tratamiento del rechazo es INCORRECTO:
a.- suprimir las diferencias antigénicas
b.- bloquear las células efectoras de la reacción inmunitaria
*c.- facilitar los mecanismos de comunicación celular
d.- potenciar los mecanismos supresores naturales
e.- modificar la estructura antigénica del injerto
6.- Entre los efectos crónicos secundarios de los fármacos inmunosupresores
generalmente NO está:
*a.- hipotensión
b.- hiperglucemia
c.- nefrotoxicidad
d.- hepatotoxicidad
e.- osteoporosis

7.- Respecto a los fármacos inmunosupresores:


a.- los fármacos antineoplasicos como la azatioprina sirven para bloquear las células
efectoras de la reacción inmunitaria *(si) (no)
b.- los glucocorticoides bloquean los mecanismos de comunicación celular *(si) (no)
c.- la ciclosporina destruye selectivamente loa macrófagos (si) *(no)
d.- los glucorticoides se utilizan en el rechazo agudo y en la terapia de mantenimiento
*(si) (no)
e.- el OKT3 es un anticuerpo antilinfocitario que bloquea las células efectoras de la
reacción inmunitaria *(si) (no)

8.- Respecto al papel de las citoquinas en la reacción inmunológica:


a.- elevan la capacidad de expresión de los receptores celulares reguladores
b.- aumentan la función de las moléculas de adhesión
c.- elevan la susceptibilidad de las células diana a la lisis por los linfocitos T
*d.- inhiben las funciones de los macrófagos, neutrófilos y eosinófilos
e.- alteran la homeostasis metabólica de los tejidos adyacentes

9.- Establecer el orden correcto de los mecanismos de rechazo:


a.- activación de la respuesta inicial 1 *2 3 4
b.- reconocimiento antigénico *1 2 3 4
c.- destrucción del injerto 1 2 3 *4
d.- amplificación (proliferación/maduración) 1 2 *3 4

10.- Respecto a la conservación de órganos señalar la afirmación ERRONEA


a.- la hipotermia pretende paliar los efectos de la hipoxia del órgano
*b.- la congelación no provoca ningún daños celular
c.- el sistema más utilizado en la conservación es la perfusión hipotérmica.
d.- el hígado debe utilizarse en las primeras 24 horas. Como máximo.
e.- la simple conservación en frío tiene un límite más allá del cual el órgano deja de ser
viable.
Preguntas de Blueprints

1. A 71-year-old man with sudden onset of severe abdominal and back pain is brought to the emergency
department for evaluation. He has a history of hypertension. He weighs 300 lb. He has a 45-pack-a-year history
of smoking. Physical examination reveals a pulsatile abdominal mass. Both lower extremities reveal pallor with
diminished pedal pulses. What is the most likely cause of this patient’s condition?
a. Atherosclerosis (aneurisma de aorta abdominal)
b. Marfan syndrome
c. Meningococcal infection
d. Syphilis
e. Trauma

2. A 78-year-old man is brought to the emergency department with a 12-hour history of abdominal pain,
diarrhea, and vomiting. He has a history of atrial fibrillation and was previously treated for congestive heart
failure with digoxin. Physical examination reveals a distended abdomen with significant guarding. Rectal
examination reveals guaiac positive stool in the vault. White blood cell count is 24,000/mL. Abdominal x-ray
reveals edema of the bowel wall. What is the most appropriate treatment for this patient?
a. Angiographic embolization
b. Antibiotic therapy with ampicillin and gentamicin
c. Antibiotic therapy with gentamicin
d. Heparinization followed by oral warfarin
e. Surgical exploration (isquemia mesentérica aguda) tratamiento incluye resección quirúrgica

3. A 20-year-old male tennis player crashes into a fence trying to chase a ball he thought he could catch up to
during an important match. His right knee sustains the brunt of injury. Physical examination reveals edema
and decreased range of motion of the knee in flexion and extension. Magnetic resonance imaging (MRI) is
performed and reveals dislocation of the joint. No pulse is palpable behind the knee joint. What is the most
likely explanation for this finding?
a. Anterior tibial artery rupture
b. Peroneal artery hematoma
c. Popliteal artery spasm
d. Posterior tibial artery hematoma
e. Superficial femoral artery spasm

4. A 25-year-old woman found a lump in her right breast on self-examination. She has no family history of breast
cancer. The lump is freely mobile and well circumscribed. What is the best option to evaluate a breast mass in
a young female?
a. Biopsy
b. Mammography
c. Testing for breast cancer (BRCA) gene
d. Ultrasound
e. Watchful waiting

5. A 19-year-old woman began breast-feeding for the first time. At first, it was difficult for her infant to feed.
Now, her breasts are red, warm, and sore. She has continued to breast-feed, despite the pain; however, she
has recently begun to use a breast pump instead of breastfeeding. She is begun on a course of oral antibiotics.
What condition is this patient at risk of developing?
a. Breast abscess
b. Fibrocystic disease
c. Inflammatory breast cancer
d. Prolactinoma
e. Tuberculosis

6. A 31-year-old premenopausal woman with a left breast mass undergoes a left modified radical mastectomy.
Pathology reveals infiltrating ductal carcinoma measuring 3 cm in size with negative lymph nodes. Estrogen
receptor status is negative. What is the most appropriate adjuvant therapy for this patient?
a. Chemotherapy (multiagent) terapia de eleccion en premenopausica con estadio 1 o 2 de cancer de mama size <1cm,
linfonodos negativos y estrógeno receptor negativo
b. External-beam radiotherapy
c. High-energy focused ultrasound therapy
d. Tamoxifen estrogeno receptor positive y >1cm
e. Watchful waiting

7. A 31-year-old woman complains of a 6-month history of bloody diarrhea, abdominal pain, and intermittent
fevers. She has a history of irritable bowel syndrome but has had a worsening of her symptoms during the
above time period. Her past medical history is unremarkable. Physical examination reveals abdominal
distension. Bowel sounds are present in all quadrants. Rectal examination reveals multiple anal fissures. What
is the most appropriate diagnostic testing for this patient?

a. Anoscopy
b. Colonoscopy → colitis ulcerativa
c. Flexible sigmoidoscopy
d. Rigid sigmoidoscopy
e. No further diagnostic testing is required for this patient.

8. A 71-year-old woman presents to her primary care physician complaining of rectal bleeding. She had some
mild left-sided abdominal cramps that subsided within a few minutes. She has never had a prior episode of
rectal bleeding. Physical examination reveals mild left lower quadrant abdominal pain without evidence of
guarding or rebound tenderness. Rectal examination reveals no fresh blood in the rectal vault. Colonoscopy
reveals several outpouchings of the sigmoid colon wall without evidence of bleeding or perforation. The
remainder of the colonoscopy is within normal limits. White blood cell count is normal. What is the most
appropriate treatment for this patient?
a. Antibiotic therapy with ampicillin and gentamicin
b. Left hemicolectomy
c. Right hemicolectomy
d. Subtotal colectomy
e. Watchful waiting → diverticulosis

9. An 85-year-old man is brought to the emergency department because of acute abdominal pain and
progressive abdominal distention. He is a resident of a local nursing home. He has not been eating because of
progressive nausea. Abdominal radiographs reveal a massively sigmoid colon. What is the initial treatment for
this patient?
a. Gastrografin enema→ esta también se puede considerar
b. High-fiber diet
c. Lactulose
d. Rectal tube decompression → vólvulo sigmoides (esto es lo ideal)
e. Surgical resection → si fuera calculo cecal

10. A 41-year-old woman complains of constant headaches for the past 6 months. She has also complained of
female infertility and has been unable to have children, despite having unprotected sexual intercourse with
her husband during the past 15 years. Physical examination reveals deficits in the extraocular movements
bilaterally. Breast examination reveals bilateral female gynecomastia. Which of the following laboratory tests
would be most useful in diagnosing this patient?
a. Ferritin
b. Hemoglobin
c. Hematocrit
d. Iron
e. Prolactin → prolactinoma

11. A 41-year-old woman with Crohn’s disease has undergone multiple surgical procedures. She has recently
undergone an ileostomy but still has evidence of some distal jejunal disease. Her current medications include
prednisone and aminosalicylic acid. Which of the following effects of prolonged therapy with glucocorticoids
are possible for this patient?
a. Antibody production
b. Collagen formation
c. Fibroblast dysfunction
d. Inflammatory cell migration
e. Impaired Wound healing
12. A 49-year-old obese man presents to his primary care physician for a follow-up examination. He has a history
of uncontrolled diabetes mellitus and bipolar disorder. His current medications include lithium and milk of
magnesium. Physical examination of the heart, lungs, and abdomen are within normal limits. Laboratory
studies reveal serum calcium of 14 mg/dL. What is the most likely explanation for these findings?
a. Dietary indiscretion
b. Medication overdose
c. Milk-alkali syndrome → esto es en pacientes que toman muchos antiacidos o beben excesiva cantidad de leche en
pacientes con ulcera gastrica que usan estas cosas para aliviar sus sintomas
d. Parathyroid adenoma
e. Parathyroid hiperplasia

13. A 41-year-old man has chronic gastroesophageal reflux. He is currently managed with an H2-blocker. Physical
examination of the heart, lungs, and abdomen are within normal limits. Which of the following factors would
be least protective of the esophagus in terms of the continued exposure induced by this condition?
a. Arcuate ligament
b. Gastric emptying ability
c. Gravitational effect
d. Salivary gland secretory products
e. Secondary peristaltic waves

14. A 40-year-old woman complains of chest pain and dysphagia to solids. She presents to a specialist for
evaluation. Esophageal manometric studies are performed and reveal high-amplitude contractions and
eventual normal relaxation of the lower esophageal sphincter. Barium swallow is normal. What is the most
likely diagnosis?
a. Cricopharyngeal muscle spasm
b. Diffuse esophageal spasm
c. Scleroderma
d. Tuberculosis
e. Psychogenic swallowing disorder

15. A 5-year-old boy is brought to the emergency department after ingesting liquid drain cleaner. The boy was left
unattended while his baby-sitter was on the telephone. The boy is hoarse and has obvious stridor. What is the
most appropriate initial treatment for this patient?
a. Antibiotics
b. Corticosteroids
c. Induction of vomiting with ipecac
d. Placement of nasogastric tube and lavage
e. Tracheostomy

16. A 76-year-old man with a history of vague right upper quadrant pain, a 25-lb weight loss, and anorexia
presents to his primary care physician for evaluation. Physical examination reveals scleral icterus. Abdominal
examination reveals a right upper quadrant mass. Kidney, ureter, and bladder (KUB) reveals a circular
calcification in the right upper quadrant. Exploratory laparotomy reveals a neoplastic process involving the
gallbladder and liver. What is the most likely pathology causing this condition?
a. Adenocarcinoma 80%
b. Sarcoma
c. Squamous cell carcinoma 5%. 10% anaplastic carcinoma
d. Transitional cell carcinoma
e. Tuberculosis granuloma

17. A 38-year-old woman presents to her primary care physician for evaluation of intermittent vague right
upper quadrant pain. She has a history of hypothyroidism and hypertension. Her current medications include synthetic
thyroid hormone replacement and a calcium channel blocker. Physical examination reveals mild right upper quadrant
pain to deep palpation. Ultrasound reveals a 3-cm gallstone. What is the most likely type of stone to be present in this
patient?
a. Black gallstone pequeñas multiples
b. Brown gallstone pequeñas multiples
c. Calcium oxalate gallstone: in the kidney
d. Type I cholesterol stone pequeñas, usually multiples
e. Type II cholesterol Stone 5-20% grandes
18. A 46-year-old woman presents to the emergency department complaining of right upper quadrant pain and a
fever to 102°F.Physical examination reveals scleral icterus and significant right upper quadrant pain to
palpation. Peritoneal signs are absent. Bowel sounds are present. Which of the following should be included in
the initial treatment of this patient?
a. Antibiotics → + fluid resuscitation is the first step (cholangitis porque tiene la triada)
b. Choledochojejunostomy → cuando el ducto biliar esta dilatado
c. Decompression with T-tube → second step also
d. Endoscopic sphincterotomy
e. Percutaneous transhepatic drainage → second step

19. A 17-year-old boy is brought to the emergency department after suffering from chest pain and dyspnea during
a pickup basketball game. Physical examination reveals a systolic crescendo-decrescendo murmur, heard best
at the second right intercostal space. The murmur radiates to the right carotid artery. Chest x-ray reveals a
normal heart size. Which of the following findings would be expected to be seen on an electrocardiogram in
this patient?
a. Inversion of T waves in leads V1–V4
b. Left ventricular hypertrophy (estenosis aortica)
c. Right bundle branch block
d. Right ventricular hypertrophy
e. Right atrial hypertrophy

20. A 72-year-old man collapses while walking in a shopping mall. He is pulseless and apneic. There is no history of
trauma. Cardiopulmonary resuscitation is started until a rescue squad arrives. Advanced cardiac life support
protocol is initiated. He is pronounced dead 40 minutes later. Autopsy reveals myocardial necrosis with
rupture of the left ventricle. Which of the following is the most likely risk factor that contributed to his death?
a. Family history of diabetes mellitus
b. Hypotension
c. Obesity
d. Sedentary lifestyle
e. Trauma

21. A 57-year-old man is brought to the emergency department complaining of dyspnea and chest pain. He also
admits to a 20-lb weight loss. He complains of fevers, chills, and night sweats. Physical examination reveals
supraclavicular adenopathy. Chest examination reveals distant heart sounds. Laboratory studies reveal a white
blood cell count of 170,000/ml. Chest x-ray and echocardiography reveal a pericardial effusion. What is the
most likely explanation of these findings?
a. Atrial myxoma
b. Atrial fibrillation
c. Lymphoma
d. Metastatic colorectal carcinoma
e. Pericarditis

22. A newborn male has an opening of the abdominal wall at the umbilicus. He has no other prior medical or
surgical history. Birth history was unremarkable. During the remainder of the physical examination and
diagnostic testing, which of the following findings are most likely?
a. Cleft lip
b. Cleft palate
c. Diaphragmatic hernia → onfalocele puede ser parte de la Pentalogia de Cantrell que se asocia con: hernia
diafragmática, esternón hendido, pericardio ausente, defectos intracardiacos, extrofia de la vejiga. Cierre incompleto de
los somatic folds en la pared abdominal anterior.
d. Pericardium
e. Urinary bladder in retroperitoneum

23. A 44-year-old male construction worker undergoes a right inguinal hernia repair. The surgical procedure is
uneventful. He has no prior medical or surgical history. He returns for follow-up on postoperative day 3 for a
wound check. The wound is clean, dry, and intact. What is the optimal convalescent period required before
returning to work for this patient?
a. 1 week
b. 4 weeks
c. 6 to 8 weeks
d. 12 weeks
e. Unknown

24. A 40-year-old woman undergoes repair of a right femoral hernia. During the procedure, the femoral canal is
dissected. The anatomic boundaries of the femoral canal include which of the following?
a. Cooper ligament. Bounded superiorly and medially by the iliopubic tract, inferior by cooper ligament, laterally by the
femoral vein.
b. Inguinal ligament
c. Ischial spine
d. Lacunar ligament
e. Nerve (femoral)

25. A 53-year-old man undergoes a radical prostatectomy for presumed organ-confined prostate cancer. The most
important factor in maintaining continence after radical prostatectomy is preservation of the:
a. Bladder neck
b. External urethral sphincter
c. Levator ani muscle complex
d. Nervi erigentes
e. Puboprostatic ligaments

26. A 27-year-old man has bulky retroperitoneal adenopathy after radical orchiectomy for a mixed germ cell
tumor. His chest x-ray is normal. Serum beta-human chorionic gonadotropin (B-hCG) and alpha-fetoprotein
(AFP) are markedly elevated. Liver enzymes are slightly elevated, and the patient relates a history of ethanol
excess. He receives three cycles of chemotherapy. Restaging reveals a 3-cm retroperitoneal mass, a normal
chest x-ray, and normal serum B-hCG. However, the serum AFP is 20 IU/mL (normal _ 0 to 9 IU/mL). What is
the next step in the management of this patient?
a. Computed tomography (CT)—guided needle biopsy
b. External-beam radiotherapy
c. Retroperitoneal lymph node dissection → hepatitis alcoholica
d. Salvage chemotherapy
e. Serial markers and CT scans

27. A 63-year-old man is disease-free two years after bacillus Calmette-Guerin therapy for carcinoma in situ and a
grade 2, stage T1 bladder cancer. In addition to physical examination, cystoscopy, and urinary cytology,
evaluation at this time should include:
a. Intravenous pyelogram
b. Prostatic urethral biopsy
c. Random biopsies of the bladder
d. Selective upper tract cytology
e. Urinary voided cytology, repeated three times

28. A 78-year-old man presents to the emergency department for evaluation of progressive right upper quadrant
pain, nausea, vomiting, and a 30-lb weight loss in the past 3 months. He has a prior medical history of
cholelithiasis, diabetes mellitus, hypertension, and dementia. Physical examination reveals scleral icterus
bilaterally. Abdominal examination reveals right upper quadrant tenderness and a palpable mass. Peritoneal
signs are absent. CT scan reveals pancreatic, duodenal, and choledochal lymph nodes. There is an asymmetric
thickening of the gallbladder. What is the most likely pathologic finding at exploratory laparotomy and biopsy?
a. Adenocarcinoma
b. Fibroma
c. Lipoma
d. Myxoma
e. Myoma

29. An 8-year-old boy undergoes a right upper quadrant ultrasound for persistent right upper quadrant
discomfort. He has no prior medical or surgical history. He has no known allergies and takes no medications.
His mother has a history of gallstones. Ultrasound findings include a fusiform dilation of the common bile duct.
What is the most likely explanation for these findings?
a. Type I choledochal cyst
b. Type II choledochal cyst
c. Type III choledochal cyst
d. Type IV choledochal cyst
e. Type V choledochal cyst

30. An 18-year-old man is stabbed in his abdomen multiple times by an assailant during an altercation involving
sale of illicit drugs. He is brought to the emergency department for evaluation. He has four stab wounds of the
abdomen—three are in the right upper quadrant, and one is in the left lower quadrant. Physical examination
of the abdomen reveals guarding and rebound tenderness. The patient is brought to surgery for an exploratory
laparotomy. A penetrating injury to the gallbladder is found. Which of the following associated viscera are
likely to be injured?
a. Aorta
b. Colon
c. Kidney
d. Liver
e. Urinary bladder

31. A 62-year-old woman presents to her primary care physician with a cough. She also complains of hemoptysis.
Social history reveals a 55-pack-a-year history of smoking. She is a recovering alcoholic. Physical examination
reveals bilateral wheezes. Cardiac, pulmonary, and abdominal examinations are unremarkable. Laboratory
values reveal serum calcium of 13 mg/dL. Serum protein electrophoresis shows no abnormal spikes. What is
the most likely diagnosis?
a. Goodpasture’s syndrome
b. Myeloma
c. Renal adenoma
d. Small-cell carcinoma of the lung
e. Squamous cell carcinoma of the lung

32. A 10-year-old boy is brought to his primary care physician for evaluation of persistent hoarseness. He has just
begun to participate with his school chorus and notes that his hoarseness worsens with singing. Physical
examination of the heart, lungs, and abdomen are unremarkable. Fiber-optic flexible laryngeal examination
reveals multiple lesions on his true vocal cords. What is the most likely diagnosis?
a. Gastroesophageal reflux
b. Granulomatous inflammation of the pharynx
c. Laryngeal papilloma
d. Singer’s nodule
e. Thyroid carcinoma

33. A 75-year-old man presents to his primary care physician because of hoarseness. He has a 60-pack-a-year
history of smoking. He also complains of a 25-lb weight loss over the past 4 months. Direct laryngoscopy
reveals a sessile mass on the high right vocal cord. He also has a palpable lymph node along the right anterior
cervical lymph node chain. If dysplasia is found on biopsy of the laryngeal lesion, what is the most likely
diagnosis?
a. Adenoma
b. Laryngeal polyp
c. Laryngitis
d. Mucoepidermoid cystic disease
e. Squamous cell carcinoma

34. A 21-year-old male college student presents to the outpatient clinic for a routine examination at the beginning
of the fall semester. He has a history of irritable bowel syndrome. Physical examination of the heart, lungs, and
abdomen are unremarkable. Genitourinary examination reveals that the testes are descended bilaterally. A
left grade 1 varicocele is present. There are no testicular masses. The penis is uncircumcised, and the foreskin
is unable to be retracted behind the glans. What is the most likely diagnosis?
a. Balanitis
b. Hypospadias
c. Epispadias
d. Paraphimosis
e. Phimosis
35. A 71-year-old white male presents to his primary care physician complaining of a 1-month history of nocturia,
polyuria, and difficulty starting and stopping his urinary stream. His American Urological Association Symptom
Score is 17/35. Physical examination of the prostate reveals an enlarged gland without masses. His testes are
descended bilaterally. He has a small right hydrocele that transilluminates. His prostate-specific antigen (PSA)
level is 6 ng/mL, and urinalysis is negative. The patient is begun on dutasteride 0.5 mg daily. What is a likely
result of taking this medication?
a. Ejaculatory dysfunction 5-8%
b. Maximal change in urinary flow rate
c. Prostate size decreases by 25%
d. Serum PSA increases by 50%
e. Symptom score remains unchanged

36. A 34-year-old white male has a painless enlargement of his right testis in the past 4 months. He is brought to
his primary care physician by his girlfriend, who urges him to seek evaluation. He has recently become
depressed because of this problem. He had a cryptorchid right testis as an infant, which was surgically
corrected. A scrotal ultrasound confirms the presence of a 3 cm hypoechoic right testicular mass .What is the
most likely diagnosis?
a. Choriocarcinoma
b. Embryonal (mixed germ cell) carcinoma
c. Endodermal (yolk sac) tumor
d. Seminoma
e. Teratoma

37. A 51-year-old man is found to have an intracranial mass and will undergo resection. The surgical procedure is
performed via a transoccipital approach. In this approach, the patient develops a cerebrospinal fluid (CSF) leak.
Which of the following statements is true regarding CSF?
a. Arachnoid villi act as two-way valves.
b. Arachnoid villi open at a pressure of 5 mm Hg.
c. CSF is absorbed through the spinal roots.
d. CSF enters through the foramen of Magendie.
e. Total CSF volume is 150 L.

38. A 19-year-old college student is driving under the influence of alcohol, despite recommendations from friends
not to drive. She is struck by another driver. The force of impact causes her to strike the temporal area of her
skull against the window. She develops a mild headache but does not lose consciousness. Several hours later,
she develops a severe headache with nausea and vomiting. Which is the most likely diagnosis?
a. Bacterial infection
b. Berry aneurysm
c. Epidural hematoma
d. Subarachnoid hematoma
e. Subdural hemorrhage

39. A 59-year-old man presents to his primary care physician complaining of progressive right-sided hearing loss
and gait unsteadiness. He states that when he uses the phone, he must use his left ear to listen instead of his
right ear. He has a past medical history of hypertension. His current medications include a calcium channel
blocker. Physical examination reveals loss of the right corneal reflex and facial weakness. Cardiac, pulmonary,
and abdominal examinations are within normal limits. What is the most appropriate next best step in the
diagnosis of this patient?
a. Audiometric testing
b. Brainstem-evoked potential testing
c. CT scan of the head without contrast
d. MRI of the head → neuroma acustico
e. Nystagmography

40. A 47-year-old man with a history of end-stage pulmonary disease of his right lung is scheduled for lung
transplantation. Preoperative cardiac function is good. He has no history of congenital defects. Which of the
following is the most appropriate surgical incision for this patient to have?
a. Chevron abdominal→ renal surgery
b. Lateral thoracotomy
c. Midline abdominal
d. Transverse anterior thoracotomy→ esta se usa si es doble transplante
e. Pfannenstiel

41. A 47-year-old man with multiple medical problems and end-stage pulmonary parenchymal disease undergoes
lung transplantation. He has a prior medical history of obstructive lung disease. He has an uncle with cystic
fibrosis. His father has restrictive lung disease, and his brother has pulmonary hypertension. Which of the
following portends the best survival after lung transplantation for this patient?
a. Bronchogenic carcinoma
b. Cystic fibrosis→ le sigue en supervivencia a EPOC
c. Obstructive lung disease
d. Pulmonary hypertension→ es la peor
e. Restrictive lung disease
42. A 4-year-old boy is on the waiting list for a liver transplant. He has end-stage hepatic disease and is currently
hospitalized for esophageal variceal hemorrhage. What is the most likely cause of liver failure in this patient?
a. Biliary atresia 50%
b. Hepatitis A
c. Primary biliary cirrhosis
d. Primary sclerosing cholangitis
e. Tuberculosis

43. A 23-year-old woman who complains of greasy and odorous stools, generalized weakness, and hair loss
presents to her primary care physician for evaluation. Physical examination of the heart, lungs, and abdomen
are unremarkable. She has no guarding or rebound tenderness. Bowel sounds are present in all quadrants.
Female pelvic examination was deferred at the patient’s request. What is the most likely explanation of these
findings?
a. Gastric ulcer with bleeding
b. Glucose malabsorption
c. Menstruation
d. Pancreatic insufficiency→ usualmente en pacientes con fibrosis quística se presenta con malabsorción y esteatorrea
severa
e. Pituitary tumor

44. A 27-year-old woman is 12 hours status after cadaveric pancreas transplantation and currently in the surgical
intensive care unit. She has a medical history of insulin-dependent diabetes since age 5. Her vital signs are
normal. Chest is clear to auscultation, and cardiac examination reveals a regular rate with a regular rhythm.
Wound dressing is clean, dry, and intact. Which of the following is the best method of monitoring the
transplanted pancreas?
a. Serum amylase level
b. Serum glucose level
c. Serum insulin level
d. Ultrasonography of the pancreatic vessels
e. Urinary amylase level

45. A 44-year-old man with recurrent pancreatitis is brought to the emergency department with another bout of
pancreatitis. Which of the following is the most reassuring factors regarding the severity of his condition?
a. Age
b. Blood glucose level of 300 mg/dL
c. Lactate dehydrogenase level of 400 IU/L
d. Serum calcium of 6 mg/dL
e. Serum hematocrit level of 29%

46. A 41-year-old man with a long history of renal stones and hypercalcemia is found to have an adenoma of the
right superior parathyroid gland. He is going to undergo surgical excision of this lesion. What is the best
surgical landmark for this lesion?
a. Bifurcation of the carotid arteries
b. Carotid sinus
c. Junction of the inferior thyroid artery and recurrent laryngeal nerve
d. Junction of the upper and middle third of the thyroid gland
e. Recurrent laryngeal nerve

47. Which of the following techniques is best used to define an enlarged parathyroid gland?
a. CT scan of the neck
b. Dual tracer imaging
c. MRI of the neck
d. Thyrocervical angiography
e. Ultrasonography

48. A 44-year-old man with end-stage renal disease successfully undergoes renal transplantation. He has a prior
medical history of hyperparathyroidism. Six months after renal transplantation, his serum calcium is still
13mg/d. Which of the following laboratory findings are possible in this patient?
a. Elevated serum phosphate
b. Elevated serum lactic acid dehydrogenase
c. Elevated urine calcium→ hiperparatiroidismo terciario que ocurre en pacientes que tienen ERC a pesar de un
transplante exitoso.
d. Elevated urine creatinine
e. Elevated urine protein

49. A 46-year-old man presents to his primary care physician for evaluation of a skin lesion. He complains of
hypopigmentation of the skin of his lower back. He has a prior medical history of eczema and basal cell
carcinoma. He is a farmer who spends a great deal of time outdoors. What cells are responsible for this
condition?
a. Adipocytes
b. Keratin-producing cells
c. Langerhans cells
d. Melanocytes
e. Merkel cells

50. A 69-year-old male presents to his dermatologist with a lesion present on his nose. He is a gardener who
spends a great deal of his time outdoors. He has a prior medical history of allergic rhinitis, hypertension, and
diabetes mellitus. His current medications include a beta-blocker and an oral hypoglycemic. Physical
examination of his nose reveals a raised, shiny, papular lesion with small blood vessels. What is the most likely
diagnosis?
a. Basal cell carcinoma
b. Histiocytosis X
c. Melanoma
d. Seborrheic keratosis
e. Squamous cell carcinoma

51. A 29-year-old Black woman presents to her primary care physician because of a growth on her left ear, which
occurred after she had her ear pierced for the first time a week ago. She noticed that her ear seemed to
develop a growth on it quite rapidly. She had never had her ear pierced before. What is the most likely
explanation for these findings?
a. Basal cell carcinoma
b. Blue nevus
c. Juvenile melanoma
d. Keloid
e. Molluscum contagiosum

52. A 52-year-old Asian American female has melanotic pigmentation of the buccal mucosa, lips, and digits.
Colonoscopy reveals hamartomas throughout the gastrointestinal tract. The polyps were removed because of
her increased risk of cancer. What other cancer is associated with this condition?
a. Cervical cancer
b. Kidney cancer
c. Liver cancer
d. Ovarian cáncer → Peutz Jegher Syndrome
e. Pancreatic cancer
53. A 45-year-old female complains of chronic diarrhea and sweating. Colonoscopy is performed, and a biopsy of a
lesion in her ileum is performed. The pathology report shows that the tumor is composed of neuroendocrine
cells. What is a medical treatment for this condition?
a. Corticosteroids, intravenous
b Corticosteroids, topical
c. Furosemide
d. Octreotide→ tx de tumor carcinoide, análogo de somatostatina
e. Tetracycline

54. An 18-year-old male is brought to the emergency department with sudden excruciating abdominal pain
localized to the right lower quadrant, nausea and vomiting, mild fever, and slight tachycardia. He has a prior
medical history of recurrent otitis media. Physical examination reveals marked right lower rebound tenderness
and guarding. Serum white blood cell count is 18,000/ml. KUB x-ray reveals bowel gas in the small and large
bowel. What is the most likely diagnosis?
a. Appendicitis
b. Crohn’s disease
c. Diverticulitis
d. Pancreatitis
e. Ulcerative colitis

55. A 39-year-old woman presents to the emergency department complaining of severe abdominal pain. She has a
history of peptic ulcer disease. Physical examination reveals guarding and rebound tenderness. She is taken to
the operating room for exploratory laparotomy. During the procedure, the surgeon who opens the
gastrosplenic ligament to reach the lesser sac accidentally cuts an artery. Which of the following vessels is the
most likely one injured?
a. Gastroduodenal artery
b. Left gastric artery
c. Left gastroepiploic artery
d. Right gastric artery
e. Splenic artery

56. A 2-year-old female is brought to the emergency department because of several episodes of rectal bleeding. A
technetium-99m perfusion scan reveals a 3-cm ileal outpouching located 50 cm from the ileocecal valve.
Which of the following types of ectopic tissue does this structure most likely contain?
a. Duodenal
b. Esophageal
c. Gastric → diverticulo de Meckel
d. Hepatic
e. Jejunal

57. A 39-year-old woman complains of epigastric pain with eating over the past 3 or 4 months. She admits to a
history of chronic back problems. She notes weight gain of 20 lb in the past 4 months. She denies the use of
nonsteroidal anti-inflammatory agents. She denies nausea and vomiting. Physical examination of the heart,
lungs, and abdomen are within normal limits. What is the most likely pathogen associated with this condition?
a. Enterohemorrhagic Escherichia coli
b. Escherichia coli
c. Helicobacter pylori
d. Shigella sonnei
e. Streptococcus pyogenes

58. A 59-year-old man was injured in a car accident. An abdominal CT scan reveals a ruptured spleen. His blood
pressure is 90/40 mm Hg, and his pulse is 140 beats per minute. The patient is taken for laparotomy.
Splenectomy is performed. Which of the following laboratory abnormalities is likely after this procedure?
a. Anemia
b. Basophilia
c. Eosinophilia
d. Thrombocytopenia
e. Thrombocytosis
59. A 19-year-old man was kicked in the abdomen during a fight in a bar. He went to his primary care physician,
who ordered a CT scan, which revealed a subcapsular splenic hematoma. The man was told to restrict physical
activity. Two weeks later, he presents to the emergency department because of severe abdominal pain. He
undergoes a splenectomy. Postoperatively, a peripheral smear is ordered. Which type of cell can be found in
this patient?
a. Basophilic stippling
b. Blister cells
c. Howell-Jolly bodies
d. Nucleated red blood cells
e. Spherocytes

60. A 15-year-old African American male underwent a splenectomy after sustaining a knife injury during a fight. He
presents to his primary care physician for a sports physical. His mother read on the Web that he is at increased
risk for infection. He should receive which of the following vaccines to prevent serious infections?
a. Rubella
b. Measles
c. Tetanus
d. Vaccines against common encapsulated organisms
e. Varicella

61. A 53-year-old woman presents to her primary care physician with a 12-month history of neck pain. She
complains of a 15-lb weight gain and generalized malaise. She has a past medical history of hypertension and
diabetes mellitus. Her current medications include an oral hypoglycemic. Physical examination reveals
tenderness along the course of the thyroid gland without evidence of a discrete mass. What is the most likely
diagnosis?
a. Acute thyroiditis
b. Hashimoto thyroiditis
c. Papillary thyroid carcinoma
d. Riedel thyroiditis
e. Subacute thyroiditis

62. A 47-year-old woman with a history of a left thyroid mass undergoes left thyroid lobectomy. Pathology reveals
a 1.3-cm papillary carcinoma with no evidence of extracapsular extension. What is the most appropriate next
step in the treatment of this patient?
a. External-beam radiotherapy
b. Multiagent chemotherapy
c. Subtotal thyroidectomy
d. Total thyroidectomy
e. Watchful waiting with periodic follow-up

63. A 34-year-old man with a thyroid nodule is undergoing a neck exploration. During the procedure, it is possible
that he will undergo thyroidectomy. Which of the following statements about the superior laryngeal nerve and
the innervation of the thyroid gland is correct?
a. Injury to the nerve causes bowing of the vocal cords during phonation.
b. Nerve injury may be unnoticeable in singers.
c. The nerve is rarely at risk during thyroid surgical procedures.
d. The superior laryngeal nerve is chiefly a motor nerve.
e. The superior laryngeal nerve is chiefly a sensory nerve.

64. A 19-year-old man leaps from the third floor of his dormitory in an apparent suicide attempt. He is brought to
the emergency department unconscious. He has visible head and lower extremity injuries. He has a pulse of
110 beats per minute but is apneic. What is the best airway management for this patient?
a. Nasotracheal intubation
b. Oral intubation
c. Oral intubation with head-chin lift
d. Tracheostomy
e. Intubation is not necessary for this patient.
65. A 21-year-old woman is stabbed in the chest by her boyfriend. She is brought to the emergency department
for evaluation. Her blood pressure is 130/80 mmHg, and her pulse is 90 beats per minute. Physical
examination reveals a single stab wound to the left fifth intercostal space in the midclavicular line. Neck
examination is normal. Trachea is midline, and the jugular veins are not distended. She does have decreased
breath sounds in the left lung fields. Which of the following diagnoses can be ruled out on the basis of the
above information?
a. Large left hemothorax
b. Open pneumothorax¿?
c. Pericardial tamponade
d. Rupture of the left main stem bronchus
e. Tension pneumothorax

66. A 41-year-old man suffers a traumatic amputation of three of his fingers in a meat slicer. He has no prior
medical or surgical history. Which of the following modalities should be used to transport the amputated
fingers with the patient?
a. Place in clean plastic bag and pack with dry ice.
b. Place in clean plastic bag filled with room temperature water.
c. Place in clean plastic bag in a chest filled with crushed ice and water → se preserva hasta 18h.
d. Place in clean plastic bag filled with hot water.
e. Wrap the amputated fingers in sterile dry gauze.

67. A 19-year-old woman presents to the emergency department after sustaining an injury to her right eye while
placing her contact lens. She has significant right eye pain. She has a prior medical history of seasonal allergies.
Physical examination reveals a simple abrasion. Fluorescein testing is performed and reveals no evidence of a
stained epithelial defect. This rules out the possibility of which of the following?
a. Bacterial infection
b. Iritis
c. Trauma
d. Viral infection
e. Ulcer

68. A 29-year-old man who works in a factory sustained a foreign body injury to his right eye when a piece of
metal shot off a conveyer belt. He is brought to the emergency department for evaluation. Physical
examination of the right eye reveals a metallic foreign body on the eye with an epithelial rust ring. What is the
most useful instrument to remove this foreign body?
a. Cyanoacrylate glue
b. Eye burr
c. Eye spud
d. Fine needle tip
e. Sterile water and alcohol

69. A 37-year-old chemistry teacher sustains a chemical splash of acid to his right eye while attempting to perform
a demonstration to his high school science class. He is in significant pain. While in the classroom and waiting
for an ambulance to transport him to the hospital, which of the following interventions should be performed?
a. Eyedrop instillation with normal saline
b. Eye patch placement
c. Flush eye with 1 to 2 L of normal saline
d. Placement of eye under direct sunlight
e. Watchful waiting until ambulance arrives

70. A 37-year-old construction worker sustained a crush injury to his right thigh after a crane fell on his leg at the
work site. He is brought to the emergency department for evaluation .He has significant right leg pain and pain
with passive stretch. The leg is tense to palpation. What is the most likely intracompartmental pressure
measurement of this patient’s right leg?
a. 5 mm Hg
b. 10 mm Hg
c. 15 mm Hg
d. 25 mm Hg
e. 35 mm Hg
71. A 41-year-old woman who cleans houses for a living presents to her primary care physician complaining of
tenderness in her right knee. The pain is constant and has been present for 3 weeks. She is in a monogamous
relationship. Physical examination reveals that her knee is slightly swollen and tender. Cardiac, pulmonary, and
abdominal examinations are within normal limits. A synovial aspiration is performed. The evaluation reveals
no evidence of crystals or bacteria. What is the most likely diagnosis?
a. Bursitis
b. Infectious arthritis
c. Rheumatoid arthritis
d. Septic thrombophlebitis
e. Trauma-induced infectious arthritis

72. A 12-year-old boy who is the star pitcher of his little league team complains of right shoulder pain. This is his
pitching arm. He has no prior medical or surgical history. Physical examination reveals weakness of the rotator
cuff tendon. What is the most appropriate treatment for this patient?
a. Injection of corticosteroids
b. Intravenous corticosteroids
c. Rest, elevation, and anti-inflammatory agents
d. Sling placement
e. Surgical repair

73. A 65-year-old man with a history of coronary artery disease is undergoing an aortobifemoral bypass. Which of
the following intraoperative management maneuvers will decrease his risk of intraoperative myocardial
infarction?
a. Beta blockade
b. Calcium channel blockade
c. Administration of normal saline instead of Lactated Ringer’s solution
d. Use of propofol
e. Use of morphine

74. A 35-year-old healthy man is diagnosed with an inguinal hernia. He has no history of abnormal bleeding. Which
of the following tests is absolutely required prior to taking him to the operating room?
a. Hematocrit
b. Platelet count
c. Potassium
d. White blood cell count
e. None of the above

75. A 50-year-old man has diarrhea after an uncomplicated bowel resection. The fluid choice that most closely
resembles his output is
a. Normal saline
b. Half normal saline with 20 mEq of potassium
c. D5W with 3 amp bicarbonate
d. Lactated Ringer’s solution
e. D5NS

76. A 27-year-old man is brought to the emergency department after slashing his hand with a knife while
attempting to slice a bagel. He has a prior medical history of recurrent sinus infections. His prior surgical
history is notable for repair of a nasal fracture. Physical examination reveals a 4-cm clean laceration along the
palmar aspect of his hand. The principles most relevant for this case would be:
a. Gentle handling of tissue and closure without tension
b. Skin grafting when necessary to cover area
c. Debridement of devitalized tissue
d. Use of vacuum sponge to promote healing
e. Flap reconstruction with vascularized tissue

77. A 68-year-old man is brought to the emergency department complaining of abdominal and leg pain for
2 weeks. He has a history of hypertension and hypercholesterolemia; he weighs 290 lb. Physical examination reveals a
pulsatile midline abdominal mass. The lower extremities have unequal pulses. Which of the following is the best next
step in the evaluation of this patient?
a. Aortogram
b. CT scan of the abdomen and pelvis
c. Ultrasound, kidneys and bladder
d. Ultrasound, liver and spleen
e. Venacavogram

78. A 28-year-old woman presents to her physician for evaluation of a lump in her right breast found on self
examination. She has a family history of breast cancer in that her mother died in her early 40s from this
condition. The mother had a modified radical mastectomy followed by chemotherapy. Physical examination
reveals a breast lump that is freely mobile and well circumscribed. There is no dimpling, asymmetry, or
retractions. The lesion measures 2 cm. What is the next step in the management of this patient?
a. Biopsy of the lesion with sonographic guidance
b. Mammography followed by stereotactic CT scan
c. Testing for BRCA gene
d. Ultrasound of the breast and consideration for breast biopsy
e. Watchful waiting and follow-up examination by primary care physician in 1 year

79. A 28-year-old man with a history of recurrent abdominal pain and bloody diarrhea presents to his physician
complaining of significant rectal pain with bowel movements. He has lost 15 pounds in the last 3 months.
Physical examination reveals right and left lower quadrant pain to palpation. Laboratory values reveal a
hematocrit of 28% and an elevated erythrocyte sedimentation rate. Colonoscopy performed in this patient
would likely reveal:
a. Colonic mass lesion
b. Diverticulosis
c. Internal hemorrhoids
d. Normal bowel
e. Thickened friable colonic and rectal mucosa → colitis ulcerativa

80. A 55-year-old male presents to his physician complaining of polyuria, polydipsia, polyphagia, and a red, scaly
rash on his face within the past 2 weeks. He voids 18 times/day with a good force of stream. Fasting blood
glucose was 325 mg/dL. He has lost 20 pounds in the past 2 months and has never had elevated blood glucose
levels in the past. Physical examination of the heart, lungs, and abdomen are normal. What is the most likely
explanation for these findings?
a. Diabetes mellitus type I
b. Diabetes mellitus type II
c. Glucagonoma
d. Insulinoma
e. Verner-Morrison Disease

81. A 72-year-old man with an 80 pack-year history of smoking presents to his physician complaining of weakness
and malaise. He has recently developed dysphagia to solid foods. He has lost 15 pounds in the last 3 months.
Physical examination reveals right supraclavicular lymphadenopathy. Cardiac and pulmonary examinations are
unremarkable. He has no guarding or rebound tenderness. Which of the following studies will provide the
most definitive diagnosis?
a. Barium esophagogram
b. CT scan of the abdomen and pelvis
c. Esophagogastroscopy with biopsy
d. MRI of the abdomen
e. Ultrasound of the right upper quadrant

82. A 42-year-old African American female undergoes a laparoscopic cholecystectomy for chronic right upper
quadrant pain. CT scan demonstrated gallstones and pericholecystic fluid. The surgical procedure was
uncomplicated. Pathologic analysis of the gallstones revealed calcium bilirubinate stones. What is the most
likely explanation of these findings?
a. High serum cholesterol levels
b. High serum lipid levels
c. High-fat diet
d. Sickle cell anemia
e. Tumor

83. A 36-year-old female complains of jaundice and peripheral swelling. An echocardiogram is ordered and the
patient is determined to have right-sided heart failure with hepatic congestion and peripheral edema. No
murmur is detected. What is the most appropriate explanation for these findings?
a. Aortic stenosis
b. Atrial septal defect
c. Patent ductus arteriosus
d. Tetralogy of Fallot
e. Ventricular septal defect

84. A newborn male born at term to a 27-year-old intravenous drug-abusing female is found to have a small
umbilical hernia. His vital signs are stable. His cardiac and pulmonary examinations are noncontributory. Which
of the following is the most likely explanation for this finding?
a. Patent foramen ovale
b. Patent omphalomesenteric duct
c. Patent umbilical ring
d. Patent urachus
e. Patent vitelline duct

85. A 28-year-old man presents to the emergency department complaining of left flank pain, nausea, and
vomiting. Physical examination of the heart and lungs are normal. There is left costovertebral (CVA)
tenderness. Urinalysis reveals microhematuria (5 red blood cells/high-power field). CT scan reveals a left-sided
4-mm renal stone, whereas KUB reveals a normal bowel gas pattern and no evidence of calcifications. What is
the most likely explanation of these findings?
a. Calcium oxalate stone
b. Calcium phosphate monohydrate stone
c. Calcium phosphate dehydrate stone
d. Small struvite stone
e. Uric acid Stone

86. A 47-year-old woman with gallstone pancreatitis is hospitalized. She has a history of hypertension and
hypercholesterolemia. Early cholecystectomy is indicated to prevent which of the following complications?
a. Cholangitis
b. Recurrent pancreatitis
c. Gastric ulcer
d. Gallstone ileus
e. Gallbladder perforation

87. A 56-year-old man complains of recurrent cough and hemoptysis. He has a history of recurrent pneumonias.
He is a nonsmoker and has no occupational risk for pulmonary disease. Physical examination reveals decreased
breath sounds in the right upper lobe. Chest x-ray reveals a small right upper quadrant mass. Bronchoscopy
reveals an angioma. What is the most appropriate treatment for this patient?
a. Antibiotics
b. Corticosteroids
c. Left lobectomy
d. Right upper lobe lobectomy
e. Watchful waiting

88. A 55-year-old male presents to his primary care physician after noticing some blood-tinged urine 1 week ago.
He denies any current pain and denies any fevers. His past medical history includes chronic obstructive
pulmonary disease from many years of smoking cigarettes. CT scan reveals bilateral renal simple cysts,
prostate enlargement, and an asymmetric thickening of the left side of the bladder. Left hydronephrosis is also
present. His urinalysis is positive for microscopic hematuria. What is the most likely diagnosis?
a. Nephrolithiasis
b. Prostate cancer
c. Renal cell carcinoma
d. Transitional cell carcinoma of the bladder
e. Urinary tract infection

89. A 30-year-old man undergoes a CT scan of his abdomen after a motor vehicle accident. He was an unrestrained
driver and was thrown from the vehicle. No acute abdominal injuries are found. The CT scan reveals bilateral
enlarged kidneys with multiple cysts present in varying sizes. The right kidney is 15 cm and the left kidney is 16
cm in length. Physical examination of the heart, lungs, and abdomen are within normal limits, other than some
mild tenderness to palpation in the right and left upper quadrants. Which of the following central nervous
system pathologies are most strongly associated with this finding?
a. Circle of Willis aneurysm
b. Cysticercosis
c. Infarction
d. Glioma
e. Subdural hematoma

90. A 46-year-old female with polycystic kidney disease is receiving a kidney transplant. Within minutes of the
anastomoses of the renal artery and vein to the respective external iliac artery and vein, the kidney rapidly
regains a pink coloration and normal tissue turgor and begins excreting urine. The patient is discharged from
the hospital and is seen at 1 month follow up. Serum creatinine is 4.2 mg/dL. Urine output is 20 mL/hour.
Physical examination of the heart, lungs, and abdomen are within normal limits. The transplanted kidney is
palpable in the right iliac fossa. Subsequent biopsy of the transplant shows extensive inflammation and
edema. What is the most likely explanation for these findings?
a. Acute transplant rejection
b. Chronic transplant rejection
c. Graft-versus-host disease
d. Hyperacute transplant rejection
e. Normal posttransplant process

91. A 52-year-old man complains of chronic abdominal pain. He has been hospitalized seven times in the last 2
years for recurrent attacks of pain from chronic pancreatitis. He has been treated with analgesics and a partial
distal pancreatectomy. His pain has still persisted. What is the next step in the treatment of this patient?
a. Continued use of oral analgesics
b. Corticosteroids
c. Splanchnicectomy
d. Total pancreatectomy
e. Watchful waiting

92. A 39-year-old man is evaluated by his physician for recurrent kidney stones. He has been treated in the past
with extracorporeal shock wave lithotripsy, ureteroscopy, and a percutaneous nephrolithotripsy. Which of the
following characteristics would suggest the diagnosis of primary hyperparathyroidism?
a. 1-mm stone on KUB x-ray
b. 2-mm stone on KUB x-ray
c. 1-mm and 2-mm right-sided stone on KUB x-ray
d. 1-mm, 2-mm, and 3-mm right- and left-sided stones on KUB x-ray
e. 4-mm right lower pole stone on CT scan

93. A physician wishes to deliver a local anesthetic subcutaneously to a patient with a suspicious lesion on his
forehead. The lesion measures 1 cm and the surgeon plans an elliptical incision to remove it. Which of the
following epidermal skin layers will the physician penetrate first with the local anesthetic needle?
a. Stratum basale
b. Stratum corneum
c. Stratum granulosum
d. Stratum lucidum
e. Stratum spinosum

94. A 35-year-old man with a history of Crohn’s disease presents to his physician for a follow-up examination. He
has diffuse ileocolonic disease on a recent CT scan. His current medications include sulfasalazine. Physical
examination reveals right lower quadrant pain to deep palpation. Should antibiotic therapy be considered in
this patient, which of the following organisms should be targeted?
a. Mycobacterium species
b. Pseudomonas aeruginosa
c. Staphylococcus aureus
d. Streptococcus pneumoniae
e. Streptococcus pyogenes

95. A 46-year-old woman presents to her physician with a history of progressive dysphagia. She has a history of a
15-lb weight loss in the last 6 months. Physical examination of the neck, heart, lungs, and abdomen are
noncontributory. Laboratory testing reveals a hematocrit of 33% and a mean corpuscular volume of 70. Upper
gastrointestinal endoscopy reveals an esophageal web. What is the most likely diagnosis?
a. Carcinoid tumor
b. Leukemia
c. Lymphoma
d. Plummer-Vinson syndrome→ carcinoma postcricoideo
e. Rheumatoid arthritis

96. A 47-year-old man is involved in a motor vehicle accident. He suffers fractures of ribs 9, 10, and 11 on the left
side. He is hemodynamically unstable and has a blood pressure of 90/50 mm Hg and pulse of 120
beats/minute despite transfusion of 3 U of packed red blood cells. He is taken to the operating room for
exploratory laparotomy. A ruptured spleen is identified and removed. In searching for a potential accessory
spleen, which is the most likely location to encounter it?
a. Greater omentum
b. Right lower quadrant
c. Right upper quadrant
d. Splenic hilum
e. Splenic ligaments

97. A 12-year-old boy is brought to his physician for evaluation of a neck mass. He has a history of recurrent
sinusitis and tonsillar infections. Physical examination reveals a midline neck mass measuring 1.5 cm that
moves with swallowing. There is no evidence of lymphadenopathy.
What is the most likely diagnosis?
a. Leukemia
b. Lymphoma
c. Thyroglossal duct cyst
d. Thyroglossal fistula
e. Thyroid carcinoma

98. During your Emergency Medicine rotation you are called to the emergency department to evaluate a patient
with an ophthalmic injury. Which of the following ophthalmic trauma injuries requires immediate on-site
management and referral to the ophthalmologist on call?
a. Acid burn
b. Corneal clouding
c. Corneal laceration
d. Hyphema
e. Severe conjunctival chemosis

99. A patient presents to the emergency department after being slashed with a knife in the left leg. The wound
appears clean and the edges are well opposed. Which of the following is the simplest method of wound
closure for this injury?
a. Delayed primary closure
b. Graft
c. Local flaps
d. Primary closure
e. Secondary intention

100. A 23-year-old female presents to her physician complaining of tenderness and pain in the right knee. The pain
is constant throughout the day and has been present for approximately 3 weeks. Social history reveals that she
is in a monogamous relationship. Physical examination reveals that her knee is slightly swollen and tender. A
synovial aspiration is performed. Neither crystals nor bacteria are found. What is the most likely diagnosis?
a. Bursitis
b. Gonococcal arthritis
c. Rheumatoid arthritis
d. Septic bursitis
e. Trauma
GRAN  COMPENDIO  DE  CIRUGÍA  
Manejo  Inicial  del  Paciente  con  Trauma  
 
1.   Paciente  de  24  años   de  edad  quien  al  dejar  una   fiesta  colisiona  contra   otro  vehículo  frontalmente  a  más  o  
menos   120   km/h.   Estaba   ingiriendo   licor.   Durante   el   transporte   se   mantuvo   hipotenso   80   mmHg   a   la  
palpación.   Sus   signos   vitales   al   llegar   al   cuarto   de   urgencias   Pa:110/88,   frecuencia   cardiaca   en   120   X,  
frecuencia  respiratoria  32  X  y  temperatura  35,6;  se  palpa  frío,  diaforético  y  responde  frases  incomprensibles  
al  estimularlo.  Presenta  deformidad  de  la  extremidad  inferior  con  exposición  de  hueso  y  sangrado  activo.  La  
primera  prioridad  de  este  paciente  es:  
a.   Radiografía  lateral  de  columna  cervical  
b.   Intubación  orotraqueal  
c.   Controlar  la  hemorragia  de  la  extremidad  inferior  
d.   Monograma  abdominal  de  urgencias  
 
2.   Después  de  la  resucitación  la  frecuencia  cardiaca  se  encuentra  en  100  X,  su  piel  está  caliente  y  su  presión  es  
ahora  de  118/90  mmHg.  Su  GCS  es  de  8.  En  la  placa  de  tórax  se  observa  un  ensanchamiento  del  mediastino.  El  
USG  FAST  no  se  observa  líquido  en  cavidad  abdominal.  Todo  lo  siguiente  se  debe  realizar  en  este  paciente,  
excepto:  
a.   Consulta  a  neurocirugía  
b.   Celiotomía  
c.   Tomografía  de  tórax  
d.   Consulta  a  ortopedia  
 
3.    Paciente  de  30  años   quien  es  atropellado   por   un  autobús.  Ella  fue  intubada  en  la  escena  y  se  le  indicaron  
líquidos  intavenosos.  Al  arribar  al  cuarto  de  urgencias  su  GSC  es  de  8,  pulso  120  X  y  su  presión  arterial  es  de  
100/70.  Los  paramédicos  le  administraron  oxígeno  por  AMBU  con  reservorio  a  más  o  menos  20  respiraciones  
por   minuto.   Su   oxigenación   es   de   90%.   A   la   auscultación   no   se   escuchan   los   ruidos   respiratorios   del   lado  
izquierdo  y  no  se  observan  las  venas  del  cuello  distendidas  o  desviación  de  la  tráquea.  El  tratamiento  debe  
consistir  primero  en:  
a.   Inserción  de  una  aguja  número  14  F  en  el  segundo  EII  
b.   Colocación  de  un  tubo  32  F  en  el  quinto  EII  
c.   Observación   de   las   marcas   del   tubo   endotraqueal,   succionar   el   tubo   y   lentamente   extraerlo   mientras   se  
ausculta  
d.   Colocarlo  en  decúbito  lateral  derecho  para  mejorar  la  ventilación  
 
4.      Un   varón   de   45   años   es   traído   al   servicio   de   urgencias   después   de   sufrir   un   accidente   automovilístico.   El  
paciente   está   alerta   y   orientado,   con   examen   neurológico   normal.   Su   fx   respiratoria   es   de   20   x,   pulmones  
limpios  bien  ventilados,  fx  cardiaca  en  120  x  y  presión  arterial  de  6..mmHg.  A  la  exploración  física  se  nota  con  
distensión,   establece   una   línea   intravenosa   y   el   paciente   recibe   rapidamente   2   L   de   L/R,   sin   cambios   en   la  
frecuencia  del  pulso  ni  en  la  Pa.  El  siguiente  paso  y  el  más  apropiado  en  su  Tx  es:  
a.   Radiografía  de  columna  cervical  
b.   Inserción  de  catéter  central  
c.   Laparotomía  exploradora  
d.   CAT  de  abdomen  
e.   Lavado  peritoneal  
 
5.    Según  el  comité  de  trauma  del  colegio  americano  de  cirugía  para  decisión  del  triade  de  los  pacientes,  lo  más  
importante  a  tomar  en  cuenta  es:  
a.   Sitio  anatómico  de  la  lesión  
b.   Estados  co-­‐morbidos  del  paciente  
c.   Mecanismo  de  la  lesión  
d.   Signos  fisiológicos  anormales  
 
6.    La  primera  causa  de  mortalidad  en  pacientes  menores  de  45  años  en  occidente  es:  
a.   Las  enfermedades  infecciosas  
b.   Las  enfermedades  cardiovasculares  
c.   Las  enfermedades  cardiovasculares  
d.   Las  enfermedades  traumáticas  
 
7.    La  valoración  inicial  del  paciente  traumatizado,  según  la  guía  de  soporte  vital  avanzado  en  trauma  (ATLS)  del  
colegio  americano  de  cirujanos,  se  divide  en  4  fases:  
a.   Valoración  pre  hospitalaria,  transporte  del  paciente,  cirugía  urgente  y  cuidados  intensivos  
b.   Acceso  inmediato  al  sistema,  cuidados  pre  hospitalarios,  cuidados  hospitalarios  y  rehabilitación  
c.   Reconocimiento  primario,  resucitación,  transporte  y  cuidados  hospitalarios  
d.   Reconocimiento  primario,  resucitación  inmediata,  reconocimiento  secundario  y  tx  definitivo  
 
8.    Un   joven   de   28   años   sufre   accidente   al   caer   de   su   moto.   No   tenía   casco   protector.   Al   llegar   al   cuarto   de  
urgencias  llega  en  severo  distres  respiratorio,  Pa  80/50,  cianosis,  con  sangrado  protuso  por  la  nariz  y  fractura  
expuesta  del  fémur.  Los  ruidos  ventilatorios  se  encuentran  disminuídos  del  lado  derecho.  El  Tx  prioritario  es:  
a.   Control  de  la  hemorragia  con  empaque  nasal  anterior  y  posterior  
b.   Rx  lateral  del  cuello  
c.   Intubación  endotraqueal  con  inmovilización  neutra  del  cuello  
d.   Toracotomía  cerrada  derecha  
e.   Canalizar  2  venas  y  cruce  de  sangre  
 
9.    Al  paciente  anterior  se  le  suministran  3L  de  L/R  en  10  minutos  y  continúa  hipotenso,  usted  cree  que  necesita  
sangre  inmediatamente,  usted  transfundiría:  
a.   Sangre  tipo  específico  cruzada  
b.   Sangre  O  negativo  
c.   Sangre  O  positivo  
d.   Sangre  tipo  específico  sin  cruzar  
e.   Ninguna  de  las  anteriores  
 
10.    Después  de  una  laparotomía  por  herida  de  bala  un  hombre  de  20  años  tiene  Hb  8.5g    Pa  100/60  Fc  76x  usted  
:  
a.   Le  transfunde  1  u  de  GRE  
b.   Le  transfunde  de  1  u  de  sangre  completa  
c.   Repite  la  prueba  de  Hg  antes  de  transfundir  
d.   No  lo  transfunde  
 
11.    Una  complicación  común  en  pacientes  traumatizados  que  hay  que  controlar  precozmente  por  ser  nociva  para  
la  coagulación  y  hemodinámica  del  paciente  es  :  
a.   Hipoglucemia  
b.   Hiponatremia  
c.   Agitación  
d.   Hipotermia  
 
12.    Una  mujer  de  25  años  de  edad  se  ve  involucrada  en  un  choque  de  vehículo  de  motor  y  sufre  una  importante  
lesión  cerrada  de  cráneo,  una  contusión  pulmonar  y  fractura  de  pelvis.  Se  encuentra  sin  respuesta  a  estímulos  
externos  y  con  ventilación  en  la  unidad  de  cuidados  intensivos.  El  mejor  enfoque  para  el  Tx  de  las  necesidades  
nutricionales  de  esta  paciente  es:  
a.   Inserción  de  catéter  subclavio  e  inicio  de  hiperalimentación  central  intravenosa  
b.   Esperar  para  la  extubación  y  mejora  de  su  estado  neurológico,  que  permita  instituír  ingresos  calórico  oral  
c.   Instalar  tempranamente  una  sonda  nasogástrica  o  nasoyeyunal  para  alimentación  con  fórmula  elemental  
d.   Esperar   que   se   resuelva   el   ileo   gastrointestinal,   seguido   por   el   inicio   tardío   de   alimentación   por   sonda  
nasogástrica  con  una  fórmula  hipercalórica  
e.   Hiperalimentación  por  vía  venosa  periférica  
 
13.  Paciente  de  27  años  de  edad  presenta  lesiones  en  ambas  extremidades  superiores  al  caer  de  una  motocicleta  
a  alta  velocidad  en  la  autopista.  El  aspecto  más  importante  en  el  manejo  de  este  paciente  debe  ser:  
a.   Enfriar  las  partes  amputadas  o  isquémicas  
b.   Dx  y  Tx  de  las  injurias  que  amenacen  la  vida  del  paciente  
c.   Asegurar  una  adecuada  perfusión  
d.   Dx  y  reconocer  tempranamente  el  síndrome  compartamental  
 
14.  En  pacientes  con  traumatismos  craneoencefálicos  graves,  la  exploración  neurológica  se  realizará:  
a.   Antes  de  la  valoración  de  la  vía  aérea,  ventilación  y  circulación  
b.   Después  de  la  resucitación  inicial  
c.   Nunca  antes  de  la  llegada  a  un  hospital  y  debe  ser  realizada  por  un  neurocirujano  
d.   Después  de  la  cirugía  urgente  
e.   Después  de  la  valoración  de  la  vía  aérea  y  de  la  ventilación  y  antes  de  la  valoración  de  la  circulación  
 
15.    La  valoración  neurológica  durante  el  reconocimiento  primario  constará  de:  
a.   Valorar  nivel  de  conciencia  mediante  escala  de  glassgow  
b.   Valorar  nivel  de  conciencia  mediante  escala  de  Glasgow,  respuesta  pupilar  y  respuesta  motora    
c.   Realizar  escáner  cerebral  
d.   Valorar  nivel  de  conciencia,  reflejos  de  tronco,  miotomas  y  dermatomas  
 
16.  Paciente  de  50  años  es  arrollado  por  un  bus  y  presenta  múltiples  fracturas  costales  en  el  hemotórax  derecho,  
presenta  movimiento  paradójico  del  tórax  y  una  saturación  de  oxígeno  de  60%  con  máscara  facial  a  flujo  de  
10  L/min  y  FIO2  del  100%.  Todo  lo  siguientes  es  cierto  del  trauma  torácico  cerrado,  excepto:  
a.   Este  paciente  tiene  alta  probabilidad  de  tener  una  contusión  pulmonar  
b.   Es  recomendable  que  se  fijen  las  fracturas  costales  para  mejorar  la  ventilación    
c.   Se  le  debe  colocar  un  catéter  epidural  para  el  manejo  del  dolor  
d.   Debe  ser  enviado  a  UCI  y  considerar  entubación  endotraqueal,  luego  de  realizar  toracotomía  cerrada  
 
17.  El  paciente  anterior  es  evaluado  por  el  residente  de  cirugía  y  por  la  severidad  de  la  lesión  torácica  sospecha  
de  contusión  cardíaca  a  pesar  de  que  tiene  un  EKG  normal:  
a.   Se  le  deben  tomar  enzimas  cardiacas  seriadas  
b.   Se  debe  monitorizar  el  ritmo  cardíaco  al  paciente  y  se  trataran  las  arritmias  
c.   Se  debe  tomar  EKG  cada  hora  
d.   Se  debe  colocar  un  catéter  de  Swan  Ganz  
 
18.  Todas  las  siguientes  consideraciones  sugieren  una  lesión  uretral  excepto:  
a.   Hematoma  escrotal  
b.   Sangre  en  la  luz  del  recto  
c.   Ausencia  de  próstata  palpable  
d.   Sangre  en  el  meato  uretral  
 
19.  Después   de   sufrir   un   impacto   automovilístico   llevando   puesto   es   cinturón   de   seguridad   un   paciente   de   26  
años   de   edad   es   trasladado   al   hospital.   A   la   revisión   primaria   no   existe   evidencia   de   lesión   grave   y   solo  
manifiesta  un  discreto  dolorimiento  difuso  en  todo  el  abdomen.  Los  ruidos  intestinales  son  hipoactivos  y  la  
matidez  hepática  parece  estar  ausente.  Las  Rx  de  abdomen  revelan  aire  libre.  El  paciente  debe  de:  
a.   Ser  sometido  a  lavado  peritoneal  
b.   Ser  sometido  a  laparatomia  de  inmediato  
c.   Realizar  estudios  radiológicos  del  trato  GI  con  medio  de  contraste  
d.   Debe  ser  cuidadosamente  observado  para  descartar  evidencia  futura    
de  lesiones  intraabdominales  
 
20.  La  principal  causa  de  acidosis    en  paciente  con  traumas  múltiples  es:  
a.   Hipovolemia  
b.   Hipotermia  
c.   Descarga  adrenérgica  
d.   Rabdomiolisis  
 
21.    La  causa  menos  probable  de  depresión  del  estado  de  conciencia  en  una  paciente  con  traumas  múltiples  es:  
a.   Choque  hipovolémico  
b.   Trauma  craneal  
c.   Oxigenación  disminuida  
d.   Alcohol  y  drogas  
 
22.    Cuál  de  los  siguientes  hallazgos  debe  ser  inmediatamente  tratado  en  la  evaluación  primaria  del  paciente  de  
trauma:  
a.   Abdomen  distendido  y  doloroso  
b.   GCS  de  8  
c.   Fx  respiratoria  de  32  X  (Ventilación  asistida)  
d.   Pulso  de  130  X  
 
23.    Cuál  de  los  siguientes  elementos  es  de  mayor  ayuda  en  la  evaluación  de  un  paciente  para  prepararlo  para  
cirugía  rutinaria:  
a.   Historia  
b.   Rx  de  tórax  
c.   Examen  Físico  
d.   EKG  
 
24.    La  mortalidad  por  trauma  es  trimodal,  el  segundo  pico  corresponde  a  la  mortalidad  precoz  (4-­‐6  horas)  tras  el  
accidente,  habitualmente  en  el  ingreso  hospitalario  siendo  la  causa  más  frecuente:  
a.   El  traumatismo  craneoncefálico  y/o  exanguinación  
b.   Obstrucción  de  la  vía  aérea  y  neumotórax  a  tensión  
c.   Traumatismo  abdominal  y  /o  pélvico  
d.   Rotura  de  aorta  y  /o  cardíaca  
e.   Ninguna  de  las  anteriores  
 
25.    La  lesión  de  la  columna  cervical:  
a.   Es  excluida  por  un  examen  neurológico  normal  
b.   Puede  ser  detectada  al  flexionar  y  extender  cuidadosamente  el  cuello  en  la  evaluación  inicial  
c.   Puede  ser  excluida  al  tomarle  una  placa  lateral  de  la  columna  cervical  
d.   Puede  manifestarse  primariamente  luego  de  mover  el  cuello  sin  ningún  cuidado  
 
26.  Pareo  

a.   Laparotomía  exploradora   Paciente  con  herida  por  arma  de  fuego  en  abdomen  y  dolor  Abdominal  
A  
b.   US  FAST   D   Paciente  de  65  años  hemodinamicamente  estable  con  dolor  Abdominal  
Paciente  hipotenso  luego  de  accidente  automovilistico  sin  dolor  
c.   Laparoscopía  Dx  
B   Abdominal  
Paciente  de  20  años  con  trauma  abdominal  cerrado,  
d.   CAT   hemodinamicamente  estable,  con  USG  FAST  positivo  porlíquido  y  lesión  
  esplénica  
e.   Angiografía      
f.   Lavado  peritoneal      
 
27.  La  intubación  naso-­‐traqueal:  
a.   Se  debe  realizar  en  pacientes  inconscientes  que  no  presenten  lesión  de  columna  cervical.  
b.   Se  debe  realizar  en  pacientes  en  que  se  sospeche  lesión  cervical  y  que  estén  inconscientes.  
c.   Esta  contraindicado  en  pacientes  que  respiren  espontáneamente.  
d.   Se  debe  realizar  en  pacientes  conscientes  con  sospecha  de  lesión  cervical.  
 
28.  Varios  minutos  después  de  haberse  caído  desde  una  ventana,  un  niño  de  9  años  es  traído  por  sus  padres  al  
departamento  de  emergencias.    El  niño  esta  sangrando  profusamente  a  través  de  una  de  herida  de  10  cm  en  
su  muslo  derecho    el  tratamiento  inmediato  de  la  herida  debería  consistir  de:    
a.   Aplicación  de  torniquete.  
b.   Colocación  de  anestesia    y  punto  hemostáticos  
c.   Presión  directa  sobre  la  herida  
d.   Aplicación  de  un  apósito  de  gas  estéril.  
 
29.  Paciente   de   67   años   resulta   severamente   lesionado   al   colisionar   su   automóvil   contra   objeto   fijo,   no   tenía  
colocado   el   cinturón   de   seguridad.   Presenta   deformidad   de   la   pelvis   y   el   fémur   derecho   se   encuentra  
fracturado.     La   extracción   del   vehículo   fue   difícil   y   dura   aproximadamente   30   minutos.     Una   infusión   de  
lactato   ringer   fue   iniciada   en   la   escena   del   accidente   hasta   su   llegada   al   cuarto   de   urgencias.   Su   presión  
arterial   se   eleva   de   70/40   mm   Hg   a   130/80   mm   Hg.   A   la   hora   de   la   resucitación   el   paciente   se   encuentra  
hemodinámicamente  estable  pero  a  través  de  la  sonda  urinaria  no  presenta  diuresis.    El  paso  inicial  para  el  
tratamiento  de  esta  anuria  debe  incluir:  
a.   Furosemida  100  mg  en  bolo.  
b.   Manitol  12,5  gr.  En  bolo.  
c.   Lactato  Ringer  a  chorro.  
d.   Tomografía  axial  computarizada  de  pelvis  y  abdomen  contrastado.  
e.   Ninguna  de  las  anteriores.  
 
 
30.  Un   hombre   de   39   de   edad   es   admitido   al   cuarto   de   urgencias   luego   de   haber     sufrido   una   colisión   es   su  
automóvil.     El   paciente   se   encuentra   cianótico,   tiene   insuficiencia   respiratoria   y   una   escala   de   Glasgow   de  
5/15.  No  hay  trauma  facial  significante;  su  traque  se  encuentra  en  la  línea  media;  y  tiene  desviación  crónica  
del  tabique  nasal  que  impide  la  intubación  naso  traqueal.    La  abundante  barba  del  paciente  dificulta  la  fijación  
de  la  máscara  de  oxígeno.  El  siguiente  paso  más  apropiado  es:  
a)   Realizar  una  cricotiroidotomía  quirúrgica.  
b)   Forzar  un  naso  traqueal  a  través  del  tabique  nasal.  
c)   Intentar  una  intubación  oro  traqueal  usando  a  2  personas  para  mantener  la  columna  alineada.  
d)   Ventilarlo  con  una  mascarilla  con  válvula  y  reservorio  hasta  que  se  descarte  una  lesión  de  la  columna  
cervical.  

Trauma  Cervical  y  Traqueal  


 
31.    Paciente   de   25   años   quién   recibe   herida   por   arma   blanca   en   zona   II   del   cuello,   se   encuentra  
hemodinamicamente  estable,  no  disfonía  o  disfagia.  Todos  los  siguientes  estudios  se  encuentran   indicados,  
excepto:  
a.   Esofagoscopía  Flexible  
b.   Broncoscopía  
c.   Arteriografía  
d.   CAT  de  cuello  
 
32.  El  Tx  inicial  de  un  trauma  laríngeo  severo  es:  
a.   Traqueotomía  
b.   Cricotiroidotomia  por  punción  
c.   Intubación  orotraqueal  
d.   Oxígeno  por  cánula  nasal  
 
33.  Todos   los   siguientes   son   factores   de   riesgos   asociados   al   desarrollo   de   malignidades   de   cabeza   y   cuello,  
excepto:  
a.   Tabaquismo  
b.   Ingesta  de  agua  sin  flùor  
c.   Ingestión  de  alcohol  
d.   Exposición  a  radiación  
e.   Ninguna  de  las  anteriores  
 
34.    Un   niño   de   10   años   de   edad   se   presenta   al   cuarto   de   urgencias   luego   de   ingerir   una   bateria   de   una  
calculadora.  La  rx  de  tórax  demuestra  que  la  bateria  se  encuentra  en  el  esófago.  Cual  es  el  mejor  manejo  del  
paciente:  
a.   Observación  
b.   Endoscopia  
c.   Darle  un  vaso  de  leche  a  tomar  
d.   Administración  de  IV  de  glucagón  
e.   Administración  sublingual  de  nifedipina  
 
35.    Cual  de  los  siguientes  pacientes  debe  ser  transportado  prioritariamente    
             al  hospital:  
a.    Paciente   de   50   años   de   edad   que   cayó   de   una   altura   de   8   pies   con   fractura   de   cadera   y   signos   vitales  
normales.  
b.   Paciente  de  15  años  con  trauma  craneal  cerrado  con  Glasgow  de  13.  
c.   Paciente  de  23  años,  mujer  quien  fue  victima  de  asalto,  con  herida  cortante  en  la  espalda,  con  signos  vitales  
normales  y  sin  distress  respiratorio.  
d.   Paciente  de  3  años  de  edad.  Quien  era  pasajero  y  el  auto    en  que  viajaba  colisionó  contra  objeto  fijo  en  la  
autopista  con  signos  vitales  normales,  con  signos  de  contusión  a  nivel  de  la  pared  abdominal.  
 
36.  ¿Cuál  de  los  siguientes  enunciados  en  falso  a  cerca  de  los  traumas  craneoencefálicos?  
a.   La  mayoría  de  los  muertos  de  los  accidentes  de  autos  son  debidas  a  lesiones  craneales.  
b.   Una  rápida  y  completa  evaluación  neurológica  forma  parte  de  la  evaluación  inicial  de  trauma.  
c.   Optimizar  la  oxigenación  arterial  es  parte  fundamental  de  la  terapia  inicial  de  trauma.  
d.   En   pacientes   hipovolémicos   la   resucitación   con   líquidos   debe   ser   muy   lenta   para   no   aumentar   el   edema  
cerebral.  
 

37.  Paciente   de   25   años   quien   recibe   herida   por   arma   Blanca   en   zona   II   del   cuello,   se   encuentra  
hemodinamicamente  estable,  no  disfonía  o  disfagia.  Todos  los  siguientes  estudios  se  encuentran   indicados,  
excepto:  
a.   Esofagoscopia  flexible  
b.   Broncoscopía  
c.   Ateriografía  
d.   Tomografía  computarizada  el  cuello  
 
Trauma  Torácico  
 
38.  Paciente   de   19   años   de   edad   se   encuentra   hemodinamicamente   estable,   alerta   y   bien   orientado   luego   de  
haber  recibido  una  herida  por  arma  blanca  en  la  región  toraco  abdominal  izquierda.  La  herida  se  encuentra  
por   debajo   del   pezón,   por   arriba   del   márgen   costal   y   en   la   línea   medio   clavicular.   La   radiografía   de   tórax  
revela  un  neumotórax  de  30%.  Después  de  la  evaluación  inicial  es  completada,  el  más  apropiado  Tx  es:  
a.   Exploración  local  de  la  herida  
b.   Colocación  de  tubo  pleural  y  laparatomía  
c.   Dejar  en  observación  y  tomar  una  radiografía  control  en  6  horas  
d.   Pericardiocentesis  
 
39.  Después  de  que  ocurre  una  lesión  torácica  la  primera  maniobra  para  mejorar  la  oxigenación  es:  
a.   Intubar  al  paciente  
b.   Evaluación  de  los  gases  arteriales  
c.   Administración  de  oxígeno  suplementario  
d.   Obtener  una  Rx  lateral  de  la  columna  cervical  
 
40.  Paciente  de  18  años  de  edad  recibe  herida  por  arma  de  fuego  en  hemitórax  izquierdo  presenta  Pa  80/50,  Fc  
130   x   y   las   venas   yugulares   distendidas.   El   tratamiento   inicial   de   este   paciente   puede   consistir   en   todo   lo  
siguiente,  excepto:  
a.   L/R  2L  a  chorro  
b.   Administrar  oxígeno  suplementario  con  una  mascara  facial  con  reservorio  
c.   Toracotomía  cerrada  
                 d.  Toracotomía  abierta  y  pinzamiento  de  la  aorta  torácica  
 
 
 
41.    Todas  las  siguientes  complicaciones  del  IAM  son  indicaciones  para  cirugía,  excepto:  
a.   Extrasítoles  ventriculares  prematuras  
b.   Aneurisma  ventricular  
c.   Ruptura  ventricular  
               d.  Ruptura  del  septum  interventricular    
 
42.  Paciente  de  18  años  de  edad  recibe  herida  por  arma  de  fuego  en  hemitórax  izquierdo  presenta  Pa  80/50,  Fc  
130   x   y   las   venas   yugulares   distendidas.   El   tratamiento   inicial   de   este   paciente   puede   consistir   en   todo   lo  
siguiente,  excepto:  
a.   L/R  2L  a  chorro  
b.   Pericardiocentesis  
c.   Toracotomía  cerrada  
d.  Lavado  Peritoneal  
 
43.  Un  varón  de  26  años  de  edad  fue  atrapado  bajo  una  grúa  en  una  
           construcción.  Después  de  una   prolongada  liberación,  es  traído  al  servicio  de  urgencias,  inmovilizado  en  una  
tabla   posterior   y   recibiendo   oxígeno   al   100%   por   máscara.   Está   alerta   y   se   queja   de   dolor   en   el   pecho.   Tiene  
saturación   de   oxígeno   al   90%,   fX   respiratoria   de   35   X,   fX   cardíaca   en   120   X   y   Pa   en   85/60.   La   tráquea   está  
desviada   a   la   derecha,   hay   crepitación   a   la   palpación   del   tórax   y   disminución   de   los   ruidos   respiratorios   en   el  
hemitórax  izquierdo.    
Cuál  es  el  siguiente  paso  y  el  más  apropiado  en  la  valoración  inicial  y  el  Tx  en  este  paciente:  
a.   Sentar  al  paciente  bien  erguido  para  mejorar  el  esfuerzo  ventilatorio  y  administrar  analgésicos  para  aliviar  las  
molestias  relacionadas  
b.   Intubar  inmediatamente  y  ventilación  asistida  
c.   Tomar  Rayos  X  de  tórax  portátil  
d.   Descompresión  del  hemitórax  izquierdo  con  aguja,  seguido  de  la  inserción  de  un  tubo  de  toracostomía  
e.   Toracotomía  en  el  servicio  de  urgencias  
 
44.    Todo  es  cierto  acerca  del  trauma  cardiaco  penetrante  excepto:  
a.   El  EKG  muestra  cambios  inespecíficos  
b.   La  triada  de  Beck  se  observa  en  menos  del  40%  de  los  pacientes  con  heridas  cardiacas  
c.   Las  heridas  por  arma  de  fuego  tienen  una  mayor  mortalidad  
d.   La  presión  venosa  central  es  de  ayuda  para  descartar  la  lesión  
e.   Las  enzimas  cardiacas  están  elevadas  
 
45.  Paciente  de  25  años  quien  recibe  HPAF  en  región  de  hipocondrio  izquierdo  a  nivel  del  5  EI  y  otra  en  región  
occipital   de   la   cabeza.   El   paciente   está   intubado,   Glasgow   de   3   Pa   90/60   Fc   100   X   Fr   28   X   Rx   de   tórax   hay  
opacidad  en  todo  el  hemitórax  izquierdo,  no  se  observa  el  proyectil.  El  próximo  paso  a  seguir  en  este  paciente  
será:  
a.   Envíar  inmediatamente  CAT  cerebral  
b.   Realizar  toracotomía  cerrada  
c.   Transfundir  inmediatamente  sangre  
d.   Colocar  tubo  nasogástrico  y  sonda  urinario  
 
46.  Paciente  de  18  de  edad  recibe  herida  por  arma  de  fuego  en  hemitorax    
         derecho.   Presenta   presión   de   80/40   mm   Hg,   frecuencia   cardíaca   130   x’,   frecuencia   respiratoria   28   x´   GCS  
14/15.     Al   examen   físico   se   encuentra   intranquilo   diciendo   que   se   va   a   morir.     Los   ruidos   respiratorios   se  
encuentran   abolidos   del   lado   derecho   y   los   ruidos   cardíacos   están   presentes,   además   presentan   ingurgitación  
yugular  bilateral.    La  prioridad  en  la  atención  de  este  paciente  debe  enfocarse  en:  
a.   La  vía  aérea  
b.   La  ventilación    
c.   La  circulación  
d.   El  estado  neurológico  
 
 
47.  La  primera  medida  a  tomar  sería:  
a.   Intubación  endotraqueal  inmediata  
b.   Pericardiocentesis.  
c.   Canalizar  2  venas  periféricas  y  administrar  2  L  de  Lactato  Ringer  a  chorro.  
d.   Colocar  tubo  pleural  derecho  
 
 
48.  Con  respecto  a  la  contusión  miocárdica  causadas  luego  de  un  trauma  toráxico,  todas  son  ciertas  excepto:  
a.   Son  diagnosticadas  por  estudios  cardíacos  invasivos.  
b.   Ocurren  en  menos  del  5%  de  los  casos.  
c.   La  medición  de  enzimas  cardíacos  no  ayuda  en  el  diagnóstico.  
d.   El  tratamiento  es  conservador.  
 
49.  Después  de  un  impacto  vehicular  contra  una  columna  de  cemento  de  un  puente,  un  hombre  de  25  años  es  
trasladado  al  hospital.  El  hombre  está  intoxicado  tiene  un  nivel  de  coma  de  Glasgow  de  13  y  se  queja  de  dolor  
abdominal.     Al   momento   de   ser   admitido   en   el   hospital,   su   presión   sanguínea   era   de   80   sistólica   a   la  
palpación,   pero   con   la   administración   de   fluidos   intravenosos,   está   rápidamente   aumentó   a     110   /70   .   La  
frecuencia  cardíaca  es  ahora  de  120   palpitaciones  por  minuto.    Las  radiografías  del  tórax  muestran  pérdida  
del   botón   aórtico,   ensanchamiento   del   mediastino,     no   tiene   fracturas   costales   o   hemoneumotorax.   El  
diagnóstico  más  probable  sería:  
a.   Hemonotórax  masivo  
b.   Neumotórax  a  tensión  
c.   Ruptura  de  Aorta  
d.   Taponamiento  Cardíaco.  

50.  Un   hombre   de   55   años   fue   arrollado   por   un   auto.   En   el   cuarto   de   urgencias,   después   de   2   litros   de   LR.   Su  
presión  arterial  es  de  60/40.  la  radiografía  del  tórax  es  normal.  GCS  9/15  el  próximo  paso  a  seguir  sería:  
a.   Craneostomía  de  urgencia  
b.   Laparotomía  
c.   Dos  (2)  unidades  de  GRE  y  reevaluación  
d.   CAT  abdominal  de  urgencias  
e.   CAT  cerebral  de  urgencia.  
 
51.  Una  dama  de  40  años  estuvo  involucrada  en  un   choque   frontal  con  otro  vehículo.  Su  presión  arterial  es  de  
100/90  FC  100  xmin.  Después  de  dos  (2)  litrod  de  LR  la  presión  arterial  es  de  115/80  u  si  FC  96  min.  El  próximo  
paso  a  seguir  sería:  
a.   Laparotomía  
b.   Sangre  
c.   Observación  
d.   CAT  abdominal  
e.   Prueba  de  alcoholemia  
 
52.  Un   hombre   de   28   años   recibió   una   puñalada   en   el   cuadrante   inferior   izquierdo   del   abdomen.   Su   presión  
arterial   es   de   110/60   y   su   FC   es   de   80xmin.   Refiere   dolor   en   hemiabdomen   izquierdo.   Todas   las   siguientes  
serían  medidas  de  manejo  apropiadas.  Excepto:  (buscar  respuestas)  
a.   Dos  litros  de  LR  a  chorro  
b.   Laparotomía  
c.   Observación  
d.   Ultrasonido  Abdominal  
e.   CAT  abdominal  
 
53.  Todas   las   siguientes   son   ciertas   respecto   a   la   TAC   de   abdomen   en   el   diagnóstico   del   trauma   abdominal  
cerrado,  excepto:  
a.   Esta  indicada  en  pacientes  estables  con  dolor  abdominal  
b.   Está  indicada  en  pacientes  estables  comatosos  sin  dolor  abdominal  
c.   Está  indicada  en  pacientes  parapléjicos  
d.   Es  altamente  sensible  para  detectar  lesiones  hepáticas  
e.   Es  altamente  sensible  para  detectar  lesiones  diafragmáticas  
 
54.    El  ultrasonido  en  el  trauma  abdominal:  (buscar  respuestas)  
a.   Está  indicado  en  pacientes  con  Shock  
b.   Es  más  sensible  que  el  TAC  para  diagnosticar  lesión  de  viscera  huéca.  
c.   Está  indicado  en  pacientes  estables  con  dolor  abdominal  
d.   Requiere  la  administración  de  contraste  endovenoso  
e.   Sólo  está  indicado  cuado  no  se  puede  hacer  CAT  
 
55.  Un  hombre  de  22  años  recibió  una  puñalada  en  el  hemotórax  izquierdo.  Su  presión  arterial  es  de  60/40  y  su  
FC  140x  min.  Las  venas  yugulares  están  ingurgitadas  a  30º.  Su  diagnóstico:  
a.   Hemotórax  
b.   Lesión  de  arteria  coronaria  
c.   Fistula  broncopleural  
d.   Neumotórax    
e.   Taponamiento  cardíaco  
 
 
 
 
 
56.  Un  hombre  de  30  años  recibió  una  puñalada  en  el  hemotórax  izquierdo.  Su  presión  arterial  es  de  60/40  y  su  
FC   es   de   120   x   min.   Los   ruidos   pulmonares   están   abolidos   en   hemotórax   izquierdo   con   hiperresonancia.  
Ruidos  cardíacos  normales  taquirrítmicos.  El  próximo  paso  a  seguir  sería:  
a.   Descompresióncon  aguja  fina  
b.   Toracotomía  abierta  
c.   Toracotomía  cerrada  
d.   Ultrasonido  
e.   Ventana  pericárdica.  
 
57.  Un  hombre  de  22  años  de  edad  presenta  una  herida  de  bala  en  el  hemitorax  izquierdo.    Se  coloca  un  tubo  
pleural  y  se  evacuó  700  cc  de  sangre.    Diez  minutos  después  de  llegar  al  Cuarto  de  Urgencia  su  presión  arterial  
disminuye  a  80/60  y  la  frecuencia  cardíaca  aumenta  a  136  x’.    El  siguiente  paso  a  seguir  es:  (buscar  respuesta  
b  o  d)  
a.   Pinzar  el  tubo  torácico.  
b.   Programar  toracotomía  de  urgencia.  
c.   2  L  de  LR  a  chorro.  
d.   Repetir  la  evaluación  primaria  y  proceder  con  el  traslado.  
 
58.  Un   hombre   de   60   años   de   edad   sufre   una   puñalada   en   el   costado   posterior   derecho.     Los   testigos   del  
incidente  indican  que  el  arma  era  un  pequeño  cuchillo.    La  frecuencia  cardíaca  del  paciente  es  de  90  x  ‘,    la  
presión  arterial  es  de  120/72  y  la  frecuencia  respiratoria  es  de  24x’.  la  acción  más  apropiada  es:  
a.   Realizar  ultrasonido  abdominal.  
b.   Realizar  PIV  
c.   Realizar  una  evaluación  física  seriada.  
d.   Suturar  la  herida  y  enviar  al  paciente  a  casa.  
 
59.    Un  atleta  de  22  años  de  edad  recibe  una  puñalada  en  el  tórax  izquierdo  a  nivel  del  tercer  espacio  intercostal  
en  la  línea  axilar  anterior.    Al  ser  admitida  al  área  de  urgencias,  se  encuentra  alerta  y  despierto.    La  frecuencia  
cardíaca  es  de  100  x’  ,  la  presión  arterial  es  80/50  y  la  frecuencia  respiratoria  es  de  20  x’  .    Una  radiografía  de  
tórax  revela  un  velamiento  completo  del  hemitorax  izquierdo.  Se  coloca  un  tubo  pleural  y  se  drena  1600  cc  de  
sangre.    El  siguiente  paso  en  el  manejo  de  esta  paciente  debe  consistir  en:  
a.   Realizar  toracoscopia.  
b.   Insertar  otro  tubo  toráxico.  
c.   Prepararía  para  toracotomía  abierta.  
d.   Angiografía  y  embolización  arterial.  
 

Lesión  por  Quemaduras  


 
 
60.  Durante   una   explosión   una   mujer   de   52   años   de   edad   sufre   quemaduras   de   sol   del   50%   de   la   superficie  
corporal.   La   paciente   presenta   quemaduras   alrededor   del   tórax   y   en   ambas   extremidades   superiores.   La  
resucitación  inicial  es  realizada  en  forma  adecuada,  la  intubación  se  efectúa  por  vía  nasotraqueal  y  se  inicia  
una  ventilación  mecánica.  Los  niveles  de  carboxihemoglobina  son  del  10%,  sus  análisis  de  gases  arteriales  en  
sangre   muestran   una   PaO2   de   40   mmHg;   PaCO2   de   60   mmHg   y   pH   de   7.25.   El   manejo   inmediato   de   esta  
paciente  debe  incluír:  
a.   Aumentar  los  líquidos  
b.   Administrar  narcóticos  endovenosos  en  pequeñas  cantidades  
c.   Reevaluar  la  vía  aérea  
d.   Aplicar  presión  positiva  al  final  de  la  expiración  
 
61.    La  mejor  guía  de  adecuada  resucitación  de  los  líquidos  en  el  paciente  quemado  es:  
a.   Gasto  urinario  
b.   Normalización  de  la  frecuencia  cardiaca  
c.   4cc/kg%  de  superficie  corporal  quemada  en  24  horas  (fórmula  de  Parkland)  
d.   Normalización  de  la  presión  venosa  central  
 
62.  Paciente   de   75   años   de   edad   quien   sufre   quemaduras   secundario   a   explosión   de   un   tanque   de   gas,   la  
descripción   de   las   quemaduras   es   la   siguiente,   quemadura   de   primer   grado   en   la   cara,   toda   la   extremidad  
superior  derecha  y  miembro  inferior  derecho,  quemadura  de  segundo  grado  en  la  parte  anterior  del  dorso,  
miembro   inferior   izquierdo,   quemadura   de   3   grado   en   todo   miembro   superior   izquierdo;   el   %   de   la  
quemadura:  
a.   49.5%  
b.   45%  
c.   76.55%  
d.   58.55  
 
63.    El  tx  inicial  de  este  paciente  es:  
a.   Oxigeno  suplementario  
b.   Traslado  inmediatamente  a  un  centro  de  quemados  
c.   Administrar  analgésicos  
d.   Administrar  antibióticos  
e.   Realizar  faciotomía  
 
64.    Paciente  de  45  años  de  edad  quien  se  durmió  en  su  casa  con  un  cigarrillo  encendido  provocando  un  incendio.  
Llega   al   hospital   de   las   tablas   con   quemaduras   de   2   grado   de   50%   de   SCT,   esputo   carbonaceo   y   dolor   de  
garganta.  El  cirujano  de  turno  le  dice  que  lo  traslade  al  HST.  Lo  más  importante  antes  de  trasladarlo  es:  
a.   Canalizar  2  venas  de  grueso  calibre  y  L/R  a  chorro  
b.   Oxígeno  con  máscara  con  reservorio  a  FIO  40%  y  flujo  de  10  LPM  
c.   Entubar  al  paciente  
d.   Cubrir  las  heridas  con  vendas  húmedas  y  aplicar  sulfadizina  de  plata  
 
65.  Varón   de   22   años   previamente   sano   de   80   kg   fue   rescatado   de   una   casa   en   llamas,   después   de   quedar  
atrapado   en   una   pequeña   recámara   de   la   casa   durante   varias   horas.   Cuando   es   llevado   al   servicio   de  
urgencias  se  le  nota  combativo  y  desorientado;  sus  pulmones  están  claros  en  la  auscultación  bilateral.  La  fx  
respiratoria  es  de  30  X,  Pa  100/70  y  fX  cardiaca  de  115  X.  Sufrió  quemaduras  de  2  y  3  grado  en  15%  de  la  STC  
en  extremidades  inferiores,  con  lesión  circunferencial  de  grosor  completo  por  debajo  de  la  rodilla  derecha.  El  
paso  más  apropiado  en  el  Tx  es:  
a.   Sedación  con  midazolam  IV  
b.   Morfina  IV  para  analgesia  
c.   Administración  de  O2  al  100%  con  mascarilla  facial  
d.   Bolo  de  líquidos  IV  a  razón  de  20  cc/kg  
 
 
 
 
 
66.  El  líquido  apropiado  de  reanimación  sería:  
a.   Dextrosa  IV  en  SSN  0.5  a  razón  de  150  cc/h  
b.   L/R  a  tasa  de  mantenimiento  por  hora,  con  bolos  de  albúmina  según  sea  necesario  
c.   L/R  en  volumen  doble  de  la  tasa  de  mantenimiento  por  hora  
d.   4800  cc  de  L/R  ,  administrado  a  200  cc/h  
e.   Líquidos  de  mantenimiento  IV,  además  de  4800  cc  adicionales  de  L/R,  la  mitad  administrada  en  las  primeras  
8  hrs  y  el  resto  en  las  siguientes  16  hrs  
 
67.    Dentro  de  las  primeras  horas  de  ingreso  el  paciente  se  queja  de  dolor  en  su  pie  derecho.  Se  nota  tumefacción  
del   pie   y   del   tobillo,   disminución   del   llenado   capilar   de   los   dedos   del   pie   derecho   y   disminución   de   la  
sensibilidad.  El  Tx  de  elección  debería  ser:  
a.   Elevación  de  la  extremidad  afectada  
b.   Iniciar  antibióticos  
c.   O2  hiperbárico  
d.   Escarotomia  
 
68.  Paciente  de  24  años  de  edad  quien  es  encontrada  por  los  bomberos  dentro  de  una  casa  en  llamas,  alrededor  
de  10  minutos.  
Le   colocan   oxigeno   por   mascara   y   reservorio   a   10   LPM.   Al   llegar   al   cuarto   de   urgencias   se   encuentra  
conciente,  signos  vitales  dentro  de  límites  normales.  Presenta  desaparición  de  los  vellos  de  la  nariz  y  cejas  y  
tiene  cenizas  en  la  nariz.  Todo  lo  siguiente  es  cierto  acerca  del  tratamiento  de  esta  paciente  excepto:  
 
a.   En  los  pacientes  quemados  se  trata  inicialmente  igual  que  los  otros  tipos  de  trauma.  
b.   Si  los  pacientes  no  tienen  compromiso  de  la  vía  aérea  no  es  necesario  intubarlos.  
c.   En  pacientes  concientes  que  necesitan  tener  una  vía  aérea  definitiva  deben  de  ser  sedados.  
d.   Pacientes  quemados  desarrollan  un  Síndrome  de  Respuesta  Inflamatoria  Sistémica.    
 
69.  Paciente   quien   presenta   quemaduras   de   tercer   grado   circunferenciales   en   ambos   miembros   inferiores.   Se  
queja  de  adormecimiento  y  aumento  de  volumen  importante.  El  tratamiento  de  inmediato  debe  ser:  
a.   Elevación  de  las  piernas.  
b.   Escarotomia.  
c.   Diuréticos.  
d.   Analgésicos  
 
70.  Manejo  inicial  de  quemaduras  de  segundo  grado  y  tercer  grado:  
a.   Curación  y  antibióticos  
b.   Llevar  al  SOP  y  luego  a  cuidados  intensivos  
c.   Reanimación  ABC,  Lactato  Ringer  según  cálculo  por  superficie  corporal  quemada.  
d.   T/A  
 
71.  El  tratamiento  de  los  pacientes  por  quemaduras  eléctricas  difiere  en  las  quemaduras  por  fuego  con  respecto  a  
la  necesidad  de:  (verificar)    
a.    Fasciotomia.  
b.    Hemodiálisis.  
c.   Amputación.  
d.   Utilización  de  Oximetria  de  pulso.  
e.   Resucitación  prehospitalaria.  
 
72.  La   cantidad   de   fluido   intravenoso   en   24   horas   que   debe   recibir   una   mujer   de   60   Kg.   de   30   años   de   edad  
victima  de  quemaduras  por  flama  con  80  %  de  superficie  quemada  es  de:    
a.   19.2  litros  de  L/R  en  5%  de  D/A  .  
b.   14.4  litros  de  L/R.  
c.   9.6  litros  de  salina  hipertónica.  
d.   7.2  litros  de  L/R.  
 
73.  Es  causa  de  síndrome  de  respuesta  inflamatoria  sistémica:  
a.   Pancreatitis  aguda.  
b.   Quemaduras  de  segundo  grado  30%  de  SCT.  
c.   Infección  por  CMV.  
d.   Herida  por  arma  de  fuego  en  abdomen.  
e.   Todas  las  anteriores.  
 
Clasificación  de  Heridas  e  Infecciones  Nosocomiales  
74.    Son  señales  indirectas  de  infección  post  operatoria  todas,  excepto;  
a.   Aumento  en  el  requerimiento  de  líquidos  
b.   Ileo  adinámico  
c.   Plaquetopenia  
d.   Intolerancia  a  la  glucosa  
 
75.  Todo  lo  siguiente  incrementa  la  tasa  de  infecciones,  en  una  cirugía  mayor;  excepto:  
a.   Paciente  mayor  de  70  años  
b.   Infecciones  alejadas  del  sitio  operatorio  
c.   DM  controlada  con  glucosa  menor  de  200mg/dl  
d.   Ingesta  de  corticoides  
 
76.  Todo  lo  siguiente  incrementa  la  tasa  de  infecciones,  en  una  cirugía  mayor;  excepto:  
a.   Las  mordeduras  de  gatos  y  monos  se  debridan  y  se  dejan  abiertas  si  el  evento  es  reciente  
b.   Toda  mordedura  de  animal  que  no  ha  recibido  atención  luego  de  24  horas,  se  debe  dejar  abierta  
c.   Se  deben  lavar  con  abundante  agua  y  jabón  
d.   Las  mordeduras  tempranas  de  perro  son  debridadas  y  se  dejan  abiertas    
 
77.    La  causa  más  común  de  fiebre  en  las  primeras  48  horas  post  operatoria  
a.   Infección  de  la  herida  
b.   Colitis  por  antibióticos  
c.   Infección  asociada  a  catéteres  intra-­‐vasculares  
d.   Atelectasias  pulmonares  
 
78.    La  principal  causa  de  la  disminución  de  las  infecciones  quierugicas  se  debe  a:  
a.   Seguir  los  preceptos  de  lister  
b.   Utilización  de  antibióticos  profilácticos  
c.   Dejar  las  heridas  abiertas  
d.   Utilizar  ropas  estériles  
e.   Irrigar  las  heridas  con  desinfectantes  
 
79.    La  CDC  de  Atlanta  define  infección  del  sitio  quirurgico  como  una  infeccion  que  ocurre  en  el  sitio  de  insición  
dentro  de  los:    
a.   30  días  post  operatorio  o  dentro  de  12  meses  si  un  implsnte  protésico  a  sido  colocado  
 
 
 
 
80.  Un   hombre   de   30   años   fue   operado   por   apendicitis   aguda   perforada   con   cierre   primario   de   la   herida  
quirúrgica.  Tres  días  después  tiene  T  de  39C,  está  oligúrico  e  hipotenso,  hay  flictenas,  crepitación  y  eritema  
extenso  alrededor  de  la  herida  quirúrgica.  El  manejo  inmediato  después  de  la  resucitación  debe  ser:  
a.   Exploración  de  la  herida  con  limpieza  y  debridamiento  en  salón  de  operaciones  
b.   Laparotomía  exploratoria  
c.   Cámara  hiperbárica  
d.   Abrir  y  curar  la  herida  en  sala  
 
81.    Son   las   2   de   la   mañana   y   usted   es   llamado   por   la   enfermera   de   la   sala   de   cirugía   porque   el   paciente   fue  
operado  a  las  8  am  de  ayer  (colecistotomia  abierta),  se  queja  de  mucho  dolor.  Su  enfoque  de  Dx  y  manejo  
inicial  debe  ser:  
a.   Hx  clínica  y  examen  físico  
b.   Rx  de  tórax  
c.   Hb  completo  
d.   Urinálisis  
 
82.    Una  paciente  tiene  un  catéter  venoso  central  desde  hace  una  semana.  Hoy  cursa  con  T  38.8  C  y  se  queja  de  
dolor  en  el  sitio  de  inserción  el  cual  se  aprecia  con  escasa  secreción  mucoide.  El  siguiente  paso  a  seguir  sería:  
a.   Retirar  el  catéter  y  colocar  otro  en  el  mismo  sitio  
b.   Retirar  el  catéter  
c.   Hemocultivo  a  través  del  catéter  
d.   Vancomicina  IV  
 
83.  Un  hombre  de  62  años  se  produce  una  herida  cortante  en  la  mano  con  una  botella  de  cerveza,  cuando  estaba  
de  paseo  en  la  playa.  Acude  a  usted  para  suturarlo  y  refiere  que  tiene  todas  sus  vacunas  de  la  niñez  y  que  
hace  14  años  atrás  le  inyectaron  toxoide  tetánico.  Cuál  seria  su  decisión  acerca  de  la  prevención  de  infección  
y  tétano:  
a.   Inmunoglobulina  tetánica  más  toxoide  tetánico  
b.   Toxoide  tetánico  y  lavado  con  agua  estéril  o  solución  salina  
c.   Inmunoglobulina  tetánica  
d.   Antibióticos  más  toxoides  tetánicos  y  solución  gaudiano  
e.   Antibióticos  más  inmunoglobulina  tetánica  
 
 
84.  Un  hombre  de  62  años  se  produce  una  herida  cortante  en  la  mano  con  una  botella  de  cerveza,  cuando  estaba  
de  paseo  en  la  playa.  Acude  a  usted  para  suturarlo  y  refiere  que  tiene  todas  sus  vacunas  de  la  niñez  y  que  
hace  14  años  atrás  le  inyectaron  toxoide  tetánico.  Cuál  seria  su  decisión  acerca  de  la  prevención  de  infección  
y  tétano:  
a.   Inmunoglobulina  tetánica  más  toxoide  tetánico  
b.   Toxoide  tetánico  
c.   Inmunoglobulina  tetánica  
d.   Antibióticos  más  toxoide  tetánico  
e.   Antibióticos  más  inmunoglobulina  tetánica  
 
 
85.    Cada  una  de  las  siguientes  preguntas  escoja  una  de  las  siguientes  opciones  
 
a)   Infecciones  nosocomiales  en  las  primeras  48  hrs  
b)   Infecciones  nosocomiales  posteriores  a  48  hrs  
c)   Ambas  
d)   Ninguna  
 
       C        Hiperglicemia  
       A        Streptococo  Beta  hemolítico  
       B        Diarrea  
       C      Requerimiento  aumentado  de  líquidos  
       C    Ileo  adinámico  
 
86.  Paciente  de  50  años  de  edad  quien  desde  hace  5  días  presenta  dolor  en  epigastrio  y  en  fosa  iliaca  derecha  
asociada  a  nauseas,  vómitos  y  diarreas  liquidas.  Ha  consultado  en  tres  ocasiones,  al  cuarto  de  urgencias  y  lo  
han  tratado  con  antibióticos  y  analgésicos  sin  mejoría.  Ahora  presenta  dolor  en  fosa  iliaca  derecha  y  sensación  
de  masa  en  esta  área  y  defensa,  además  de  fiebre  y  leucocitosis  con  desviación  hacia  la  izquierda.  Es  llevado  
al  salón  de  operaciones  y  se  encuentra  una  apendicitis  aguda  perforada  con  absceso  en  fosa  iliaca  derecha  y  
pelvis.  
 
1.   Según  la  clasificación  de  las  heridas  este  paciente  tiene  una  herida:  
a-­‐  Limpia.  
b-­‐  Limpia  contaminada.  
c-­‐  Contaminada.  
d-­‐  Sucia.  
87.  Los  cuidados  postoperatorios  de  este  paciente  debe  incluir  excepto:  (verificar)  
a-­‐  Antibióticos  intravenosos  y  curación  de  la  herida  con  clorhexidina  a  7%.    
b-­‐  Ejercicios  respiratorios.  
c-­‐  Analgesia.  
d-­‐  Reposición  de  líquidos  y  electrolíticos.  
 
88.  Al    quinto  día  postoperatorio  el  paciente  cursa  con  hipotensión,  taquicardia  y  fiebre.  La  diuresis  es  menor  de  
10   c.c.   por   hora.   Además   de   distensión   abdominal.   Se   revisan   los   líquidos   administrados   y   el   balance   se  
encuentra  positivo.  Este  paciente  se  le  debe  realizar:  
a-­‐  Serie  de  abdomen  agudo.  
b-­‐  Ultrasonido.  
c-­‐  Tomografía  axial  de  abdomen  superior.  
d-­‐  Colon  por  enema.    
 
89.  El  manejo  de  este  paciente  debe  incluir  excepto:  
a-­‐  Utilización  de  inotropicos  
b-­‐  Colocación  de  catéter  central,  para  medir  presión  venosa  central.  
c-­‐  Traslado  a  la  unidad  de  cuidados  intensivos.  
d-­‐  Aumentar  los  líquidos  y  utilizar  diuréticos.  
 
90.    Todas   las   siguientes   situaciones   aumentan   el   riesgo   de   infecciones   del   sitio   quirúrgico   luego   de   una  
Herniorrafia  electiva  excepto:  
             a.        Rasurar  el  sitio  operatorio  el  día  anterior  a  la  cirugía.  
             b.        Colocar  drenajes  en  la  herida.  
             c.          No  continuar  los  antibióticos  profilácticos  más  de  24  horas.  
             d.          Dejar  hematomas  en  la  herida.  
             e.          Aproximar  fuertemente  las  suturas  sobre  la  piel.  
 
91.  Con  respecto  a  la  preparación  pre-­‐operatoria  del  paciente:  
a.   El  rasurado  debe  ser  preciso  y  meticuloso  
b.   En  casos  electivos  se  debe  lavar  al  paciente  con  Clorhexidina.  
c.   La  preparación  de  Yodo  es  bactericida,  fungicida  y  virucida.  
d.   La  tintura  de  Yodo  es  un  iodoforo  altamente  efectivo  
 
92.  Son  señales  indirectas  de  infección  post-­‐operatoria  todas,  excepto:  
a.   Aumento  en  el  requerimiento  de  líquidos  
b.   Ileo  adinámico  
c.   Hiperkalemia  
d.   Intolerancia  a  la  glucosa  
 
93.  Ud.  es  llamada  (o)  a  las  2  am    de  la  sala  de  cirugía  porque  el  paciente  que  se  le  realizó  una  colecistectomia  
abierta    a  las  8  am  del  día  anterior  se  queja  de  mucho  dolor.  
Su  enfoque  de  Dx  y  manejo  inicial  incluye  todas  las  siguientes  excepto:  
a.   Historia  Clínica  y  Examen  Físico  
b.   R-­‐X  de  Tórax  
c.   Hemograma  completo  
d.   Urinalisis  
 
94.  Todo  lo  siguiente  incrementa  la  tasa  de  infecciones,  en  una  cirugía  mayor  excepto:  (verificar  o  es  esa  o  la  E)  
a-­‐    Paciente  mayor  de  70  años.  
b-­‐    Infecciones  alejadas  del  sitio  operatorio.  
c-­‐    Diabetes  mellitus  controlada.  
d-­‐    Ingesta  de  corticoides.  
e-­‐    No  hay  excepto,  todas  aumentan  la  tasa  de  infecciones.  
 
95.  Con  respecto  a  los  antibióticos  profilácticos  todas  son  ciertas  excepto:  
a-­‐   Deben  iniciarse  el  día  antes  de  la  cirugía.  
b-­‐   Están  indicados  si  las  probabilidades  de  infección  son  altas.  
c-­‐   Están  indicados  si  las  probabilidades  de  infección  son  bajas  pero  una  infección  podría  ser  fatal.  
d-­‐   Se  requieren  niveles  séricos  óptimos  al  hacer  la  incisión.  
 
96.  Una  paciente  tiene  un  cateter  venoso  central  desde  hace  1  semana.  Hoy  cursa  con  Tº  38.8  y  se  queja  de  dolor  
en  el  sitio  de  la  inserción  el  cual  aprecia  con  escasa  secreción  mucoide.  
El  siguiente  paso  a  seguir  sería:  
a-­‐   Retirar  el  catéter  y  colocar  otro  en  el  mismo  sitio  
b-­‐   Retirar  el  catéter  
c-­‐   Hemocultivo  a  través  del  cateter  
d-­‐   Vancomicina  IV  
 
97.  El  cultivo  semi-­‐cuantitativo  de  la  punta  del  catéter  venoso  central:  (verificar)  
a-­‐   No  es  necesario  si  hay  signos  evidentes  de  infección  en  el  sitio  de  entrada  del  catéter.  
b-­‐   Es  positivo  si  se  reportan  más  de  15  colonias.  
c-­‐   Se  realiza  en  medio  de  tioglicolato.  
d-­‐   También  se  realiza  en  casos  de  sospecha  de  infección  de  vías  urinaria.  
 
98.  Todos  los  siguientes  datos  son  ciertos  acerca  del  tratamiento  de  las  mordeduras  de  animales  excepto:  
a-­‐   Se  deben  lavar  con  abundate  agua  y  jabón  
b-­‐   Las  mordeduras  de  gatos  y  monos  se  debridan  y  se  dejan  abiertas  si  el  evento  es  reciente.  
c-­‐   Toda  mordedura  de  animal  que  no  ha  recibido  atención  luego  de  24  horas,  se  debe  dejar  abierta  
d-­‐   Las  mordeduras  tempranas  de  perro  son  debridadas  y  se  dejan  abiertas.  
 
99.  Un   hombre   de   30   años   sufrió   heridas   varias   en   las   piernas   con   las   cuchillas   de   una   cortadora   de   césped  
mientras  trabajaba.  (verificar)  
El  manejo  inicial  debe  consistir  en:  
a-­‐   Toxina  tetánica  y  antiglobulina  
b-­‐   Limpieza  y  sutura  en  el  salón  de  operaciones  
c-­‐   Morfina  15  mg  IV  en  el  Servicio  de  Urgencias  
d-­‐   T/A  
 
100.   Las  tres  causas  más  probables  de  fiebre  >  38.5°  C,  36  horas  después  de  una  colectomía  son:  
a.   Atelectasia  pulmonar  
b.   Escapes  de  viscera  hueca  
c.   Infección  necrotizante  de  tejidos  blandos  
 
101.   Un   niño   es   mordido   en   la   mano   por   un   perro.   Tiene   edema   y   secreción   purulenta.   Hay   ganglios  
epitrocleares  y  un  cordón  rojo  en  el  antebrazo.  Cual  de  los  siguientes  sería  el  antibiótico  indicado:  
a-­‐   Eritromicina  
b-­‐   Ampicilina  +  sulbactan  o  Ac.  Clavulónico.  
c-­‐   Clindamicina  
d-­‐   Vancomicina  
 
102.   Con  respecto  a  los  antibióticos  profilácticos  todas  son  ciertas  excepto:  
e-­‐   Deben  iniciarse  el  día  antes  de  la  cirugía.  
f-­‐   Están  indicados  si  las  probabilidades  de  infección  son  altas.  
g-­‐   Están  indicados  si  las  probabilidades  de  infección  son  bajas  pero  una  infección  podría  ser  fatal.  
h-­‐   Se  requieren  niveles  séricos  óptimos  al  hacer  la  incisión.  
i-­‐   El  antibiótico  a  utilizar  se  escogerá  según  la  operación  a  realizar.  
 
103.   La  causa  más  común  de  fiebre  en  las  primeras  48  hrs.  Post-­‐operatorias.  
a-­‐   Infección  de  la  herida  
b-­‐   Colitis  por  antibióticos  
c-­‐   Infección  asociada  a  catéteres  intra-­‐vasculares  
d-­‐   Atelectasias  pulmonares  
 
104.   Es  causa  posible  de  infecciones  quirúrgicas  después  de  72  horas  postoperatorias:  
a-­‐   Sinusitis.  
b-­‐   Sepsis  asociada  al  catéter  venoso  central.  
c-­‐   Absceso  intra-­‐abdominal.  
d-­‐   Infección  de  la  herida  quirúrgica.  
e-­‐   Todas  las  anteriores.  
 
105.   Un   hombre   de   50   años   tiene   fiebre   38.5   y   está   distendido   5   días   después   de   una   apendicectomía   por  
apendicitis  perforada.  GB  19,000  con  85%  Neutrófilos.  El  próximo  paso  a  seguir  sería:  
a-­‐   Laparotomía  
b-­‐   TAC  de  abdomen  
c-­‐   Cambio  de  antibióticos  
d-­‐   Seguir  igual  
 
106.   Un   paciente   de   40   años   fue   operado   por   apendicitis   aguda   perforada   con   cierre   primario   de   la   herida  
quirúrgica.   Tres   días   después   tiene   temperatura   de   39   grados,   esta   oligurico   e   hipotenso   hay   flictenas,  
crepitación  y  eritema  extenso  alrededor  de  herida  quirúrgica.  
El  manejo  inmediato  después  de  la  resucitación  debe  ser:  
a-­‐   Exploración  de  la  herida  con  limpieza  y  debridamiento  extenso  en  el  SOP  
b-­‐   Laparotomía  exploradora  
c-­‐   Cámara  hiperbárica  
d-­‐   Abrir  y  curar  la  herida  en  sala  
e-­‐   Se  debe  realizar  luego  de  exámen  endoscópico  de  la  boca,  laringe,  esófago  y  tráquea.  
 
107.   La  CDC  (centro  de  control  de  infecciones  de  Atlanta)  define  una  infección   del  sitio  quirúrgico  como  una  
infección  que  ocurre  en  el  sitio  de  infección  dentro  de  los    _30    días  postoperatorios  o  dentro  de    12      meses  si  
un  implante  protésico  ha  sido  colocado.  
 
Sangrado  Digestivo  
108.    Un  hombre  de  57  años  tiene  Hb  de  8.5  g  después  de  una  hemorragia  digestiva  alta  que  ha  cesado.  Tuvo  
un  infarto  hace  3  meses  y  está  en  lista  de  espera  para  cirugía  de  re-­‐vascularización  miocárdica.  Usted:  
a.   Le  transfunde  2  u  de  GRE  
b.   Le  transfunde  2  u  de  sangre  completa  
c.   No  le  transfunde  
d.   Repite  la  prueba  de  Hb  antes  de  transfundir  
 
109.    Con  respecto  al  sangrado  por  varices  esofágicas,  excepto:  
a.   El  propanolol  es  útil  en  la  hemorragia  activa  
b.   El  octeortide  es  el  medicamento  de  elección  en  el  manejo  farmacológico  inicial  
c.   La  ligadura  o  escleropatía  de  las  varices  son  parte  del  manejo  rutinario  inicial  
d.   TIPS  significa  shunt  porto  sistémico  intra-­‐hepático  transyugular  
 
110.    Con  respecto  al  sangrado  digestivo  bajo:  
a.   Los  divertículos  son  la  causa  más  común  
b.   50%  de  los  pacientes  con  diverticulos  presentarán  episodio  de  SD  bajo  secundario  a  los  mismos  
c.   La  hemorragia  generalmente  es  de  origen  venoso  
d.   Si  el  sangrado  es  por  diverticulos  en  más  del  50%  de  las  veces  habrá  que  operar  
 
111.   Con  respecto  al  SD  todas  son  ciertas,  excepto:  
a.   Los  SD  altos  son  mucho  más  frecuentes  que  los  SD  bajos  
b.   La  primera  medida  en  el  Dx  consiste  en  pasar  un  tubo  nasogástrico  
c.   Las  plaquetas  y  el  plasma  fresco  son  utilizados  como  parte  usual  del  Tx  
d.   La  hematoquesia  ocasionalmente  puede  originarse  en  TGI  alto  
e.   La  taquicardia  es  un  signo  más  temprano  que  la  hipotensión  
 
112.   Un  hombre  de  70  años  consulta  por  hematoquezia.  Su  Pa  100/60  Fc  110  x.  Para  establecer  el  dx  todas  son  
ciertas,  excepto:  
a.   Para  poder  localizar  el  sitio  de  sangrado  la  angiografía  necesita  una  tasa  de  sangrado  superior  al  centelleo  con  
glóbulos  rojos  marcados  
b.   La  angiografía  da  una  localización  precisa  del  sitio  de  sangrado  
c.   La  colonoscopia  es  el  primer  estudio  a  realizar  
d.   El  scan  con  glóbulos  rojos  marcados  tiene  una  severidad  del  80%  
 
113.   Un    Paciente  cirrótico    acude  por  hematemesis  masiva.  Su  presión  arterial  es  de  90/70  y  su  FC  110  x  min.    
Tiene  várices  esofágicas  diagnosticadas  con  anterioridad.    El  próximo  paso  a  seguir,  sería:  
a.   Gastroscopia  
b.   Derivación  porto  cava  de  urgencia.  
c.   Observación  
d.   Análogo  de  somatostatina  IV.  
e.   Vasopresina  IV  
114.   Una  Dama  de  85  años,  consulta  por  sangrado  transrectal  color  rojo  vino.    Su  Presión  arterial  es  de  110  /  80  
y  su  FC  90  x  min.    Todos  los  siguientes  serían  conductas  apropiadas,  excepto:  
a.   Angiografía  
b.   Centelleo  con  glóbulos  rojos  marcados.  
c.   Colonoscopia  
d.   Hemostasia  quirúrgica.  
e.   Resucitación  y  estabilización.  

115.   Todos  los  siguientes  nos  hacen  sospechar  de  isquemia  intestinal,  excepto:  
a.   Arritmias  cardíacas  
b.   Fiebre  
c.   Leucocitosis  
d.   Taquicardia  
e.   Vómitos  incontrolables  
 
116.   Un   hombre   de   55   años   consulta   por   debilidad   y   evacuaciones   negras.   El   próximo   paso   a   seguir   sería:  
(buscar)  
a.   Colonoscopía  
b.   Colon  por  enema  
c.   CPRE  
d.   Gastroscopía  
e.   Serie  esófago  gastroduodenal  
 
117.   Un   hombre   de   62   años,   se   le   realizó   una   endoscopía   que   mostró   una   úlcera   duodenal   con   un   coágulo  
fresco  en  el  fondo.  El  próximo  paso  a  seguir:  
a.   Cirugía  
b.   Hemostasia  endoscópica  
c.   Observación  
d.   Somatostatina  IV  
e.   Vasopresina  IV  
 
SANGRADO  DIGESTIVO  BAJO  
 
118.   En  relación  a  la  hemorragia  digestiva  baja  el  uso  de  centelleo  con  sulfuro  coloidal  marcado  con  tecnecio:  

a.   es  depurado  por  el  sistema  reticulo  endotelial  


b.   tiene  una  vida  media  de  3  minutos  
c.   requiere  elocidad  de  sangrado  de  0.5  ml  por  minuto  
d.      todas  las  anteriores  
 
119.   Paciente  de  5  años  de  edad  es  traido  al  cuarto  de  urgancias  del  hospital  del  niño  por  presentar  sangrado  
transrectal  abundante  con  palidez  de  tegumento  y  dolor  en  FID  y  flanco  derecho.  La  causa  mas  frecuente  es:  
a.   diverticulo  de  meckel  
b.   intolerancia  a  la  leche  
c.   parasitosis  intestinal  
d.   angiodisplasia  
 
120.   El  agente  etiológico  de  la  colitis  seudomembranosa  es:  
a-­‐   E.  Coli.  
b-­‐   Enterobacter  fecalis.  
c-­‐   Klebsiella.  
d-­‐   Clostridium  difficile.  
e-­‐   Pseudomona.  
 
121.   Todos  los  postulados  en  relación  con  el  divertículo  de  Meckel  son  verdaderos  excepto:  
a-­‐   El  divertículo  de  Meckel  aparece  en  el  ileon  a  mas  o  menos  90  cm.  de  la  válvula  ileocecal.  
b-­‐   El  divertículo  de  Meckel  resulta  de  la  falla  de  obliterarse  el  ducto  vitelino.  
c-­‐   La  incidencia  del  divertículo  de  Meckel  en  la  población  general  es  del  15%.  
d-­‐   La  mucosa  gástrica  es  el  tejido  ectopico  más  comúnmente  encontrado  en  el  divertículo  de  Meckel.  
 
122.   La  complicación  más  frecuente  del  divertículo  de  Meckel  es:  
a.   Sangrado.  
b.   Diverticulitis  
c.   Obstrucción  
d.   Dolor  abdominal  intermitente  
 
123.   Los  siguientes  genes  participan  en  la  secuencia  pólipo,  adenoma  ,carcinoma  excepto:  

A.   DDC(Delete  in  colorectal  cancer)  


B.   K-­‐ras  
C.   HMSH2  
D.   APC  
 
124.   El  síndrome  de  Lynch  todas  son  ciertas  excepto:  
a.   Se  asocia  a  múltiples  pólipos  en  el  colon  
b.   El  cáncer  de  colon  se  presenta  a  edad  temprana  
c.   El  cáncer  es  más  común  en  lado  derecho  
d.   Suele  encontrarse  múltiples  primarios  
 
e.   Todas  son  ciertas  respecto  a  la  enfermedad  diverticular  del  colon  excepto:  
e.   Esta  asociada  a  dietas  bajas  en  fibras  
f.   Son  pseudo  divertículos  
g.   Se  presentan  principalmente  en  el  colon  izquierdo  
h.   Cuando  hay  sangrado  masivo  usualmente  es  del  colon  izquierdo  
i.   Ninguna  de  las  anteriores  
 
125.   Un  hombre  de  68  años  acude  por  dolor  abdominal  inespecífico  en  bajo  vientre.  Ha  perdido  20  libras  en  los  
últimos   7   meses.     Hemoglobina   en   9,8   g%  y   refiere   que   desde   hace   3   meses   viene  teniendo   problemas   de  
constipación  .  El  estudio  mas  efectivo  para  Diagnosticar  su  problema  es:  
a-­‐   SEGD.  
b-­‐   Radiografìa  simple  de  abdomen  
c-­‐   Colonoscopia.  
d-­‐   CAT.  
e-­‐   Antígeno  carcinoembrionario.  
 
126.   Sus  signos  vitales  son  PA  130/80,  FC  76  x',  con  respecto  a  la  anemia  de  este  pte  todos  son  ciertos  excepto:  
a-­‐   No  requiere  transfusión,  si  no  tiene  cardiopatía  isquemica.  
b-­‐   El  examen  de  sangre  oculta  en  heces  debe  ser  positivo.  
c-­‐   Las  transfusiones  de  sangre  perioperatorias  empeoran  el  pronóstico.  
d-­‐   Debe  ser  transfundido  los  más  pronto  posible  para  evitar  complicaciones.  
 
127.   El  mejor  examen  para  evaluar  la  posibilidad  de  metástasis  hepáticas  y  pélvicas  sería                    CAT  
 
Posterior  a  la  cirugía  este  pte  debe  ser  seguido  con  determinaciones  séricas  de:  
a-­‐   Alfa  feto  proteína.  
b-­‐   Antígeno  carcinoembrionario.  
c-­‐   Determinaciones  de  C  A    19-­‐9.  
d-­‐   Gastrina.  
 
128.   Una  mujer  de  75  años  consulta  por  dolor  en  Cuadrante  inferior  izq.  (CII)  de  48  horas  de  evolución.    T=  38.3  
C,    GB=15.9  con  90%  N,  hay  defensa  y  rebote  en  CII.    El  próximo  paso  a  seguir  sería:  
 
a.   Antibióticos  IV,  hidratación  IV.  
b.   Laparotomía  
c.   Colonoscopía  
d.   Colon  por  enema                                                
e.    Ninguna  de  las  anteriores  
 
129.   Un  hombre  de  70  años  esta  siendo  tratado  por  un  episodio  de  diverticulitis  desde  hace  4  días.    Su  T  es  de  
38.5°  C,  G.B.  19.0  con  85%N.    El  próximo  paso  a  seguir  sería:  
a.   Colonoscopía  
b.   Colon  por  enema  
c.   Laparotomía  
d.   Seguir  igual  
e.   Ninguno  de  los  anteriores  
 
Un  hombre  de  70  años  esta  siendo  tratado  por  un  episodio  de  diverticulitis  desde  hace  4  días.    Su  T  es  de  38.5°  C,  
G.B.  19.0  con  85%N.    El  próximo  paso  a  seguir  sería:  
f.   Colonoscopía  
g.   Colon  por  enema  
h.   TAC  abdomino-­‐pelvico  
i.   Seguir  igual  
Ninguno  de  los  anteriores  
 
Un   hombre   de   55   años   de   edad,   con   antecedentes   de   diverticulosis,   se   presenta   con   cuadro   de   2   semanas   de  
evolución   con   disuria,   urgencia   y   neumaturia.   Además,   refiere   un   dolor   vago,   intermitente,   localizado   en  
cuadrante   inferior   izquierdo   del   abdomen,   de   tres   meses   de   evolución   y   hábitos   intestinales   irregulares.   A   la  
exploración   física,   tiene   sensación   de     plenitud   a   la   palpación   en   el   cuadrante   inferior   izquierdo,   el   cultivo   de  
orina  muestra  una  infección  polimicrobiana.  En  la  tomografía  por  computadora  se  aprecia  una  masa  inflamatoria  
en  el  lado  izquierdo  de  la  pelvis.  
130.   El  estudio  que  más  probablemente  ayuda  para  hacer  el  diagnóstico  es:  
a-­‐  Cistouretrografía  de  vaciamiento.  
b-­‐  Cistoscopia.  
c-­‐    Enema  de  bario  con  contraste  aéreo.  
d-­‐  Colonoscopia.  
e-­‐   Laparoscopia.  
 
131.   Después  de  confirmar  el  diagnóstico,  los  pasos  iniciales  más  apropiados  en  el  tratamiento  de  este  paciente  
incluyen:  
 
a-­‐  Antibióticos  de  amplio  espectro  e  inserción  de  sonda  permanente  de  vejiga.  
b-­‐  Tratar  al  enfermo  como  ambulatorio,  antibióticos  orales  y  suavizadores  del  excremento.  
c-­‐  Líquidos  claros,  citrato  de  magnesio  y  antibióticos  orales.  
d-­‐  Laparotomía  urgente.  
e-­‐  Colostomia  proximal  para  desfuncionalizar.  
 
132.   Un   hombre   de   55   años   consulta   por   fecaluria,   neumaturia   e   IVU   a   repetición.     En   la   evaluación   de   su  
padecimiento  los  siguientes  pasos  a  seguir  serían:  
a.   Colon  por  enema  y  cistoscopía  
b.   TAC  y  Urocultivo  
c.   Informarle  de  la  necesidad  de  ser  operado  
d.   Todas  las  anteriores  
e.   Ninguna  de  las  anteriores  
 
133.   El  tratamiento  del  vólvulo  del  sigmoide  es:  
a.   Rectosigmoidoscopía  inicialmente  
b.   Laparotomía  si  hay  signos  de  gangrena  
c.   Resección  y  colostomía  si  hay  que  operar  
d.   Todas  las  anteriores  
e.   Ninguna  de  las  anteriores  
 
134.   Son  indicaciones  de   cirugía  en  la  enfermedad   de  Crohn  ¿¿¿excepto?(   En   un   profeta  dice  excepto  y  otro  
no):  
                       a-­‐        Perforación.  
                       b-­‐        Sangrado.  
                       c-­‐        Obstrucción.  
                       d-­‐        Debilidad  y  desnutrición.  
 
135.   El  agente  etiológico  de  la  colitis  seudomembranosa  es:  
a-­‐   E.  Coli.  
b-­‐   Enterobacter  fecalis.  
c-­‐   Klebsiella.  
d-­‐   Clostridium  difficile.  
e-­‐Pseudomona  
 
136.   El  antibiótico  de  elección  es  para  tratar  la  colitis  seudomembranosa  es:  
a-­‐  Ampicilina.  
b-­‐  Vancomicina.    (Luego  de  Metronidazol)  
c-­‐  Cefoxitina.  
f-­‐   Gentamicina.  
 
SANGRADO  DIGESTIVO  ALTO    
137.   En  relación  al  tratamiento  de  sangrado  digestivo  alto:  
a.   el  80%  se  detiene  solo  
b.   el  50%  requiere  tratamiento  de  urgencias  
c.   el  45%  requiere  tratamiento  endoscopico  
d.   solamente  el  20%  se  detiene  solo.  
 
138.   La  principal  causa  de  sangrado  digestivo  al  en  nuestro  medio  es:  
a.   ulcera  duodenal  cronica  activa  
b.   sindrome  de  mallory  –weiss  
c.   varices  esofàgicas  
d.   leimioma  gastrico  
 
139.   el  angiograma  selectivo  determina  el  sitio  de  sangramiento  agudo  cuando  este  es  mayor  de    
a.   0,1  ml  por  min  
b.   0,5  ml  por  min  
c.   1  ml  por  min  
d.   5  ml  por  minuto  
 
140.   En  relacion  a  la  hemorragia  digestiva  alta.  Usted  tiene  una  pte  que  acude  cuando  con  historia  de  haber  
vomitado  sangre  la  noche  anterior  esta  hemodinamicamente  estable,  le  realiza  una  endoscopia  digestiva  alta  
de  urgencia  y  el   hallazgo  endoscópico  es   una  ùlcera  duodenal  activa  con  vaso  visible,  sin  sangre  digerida  o  
fresca  en  el  estomago  usted.  
a.   lo  envia  a  casa  porqu  eno  hay  sangre  endoscopica.  
b.   Lo  envia  a  casa  con  recomendaciones  de  peptobismol,  ranitidina  y  motronidazol  
c.   Lo  hopitaliza  pues  esta  es  una  lesion  de  alto  riesgo  de  resangrado  
d.   Lo  envia  a  ca  pues  al  igual  que  la  ulcer  e  fondo  limpio  este  hallazgo  no  se  asocia  con  resangrado.  
 
Apendicitis  
 
141.    Paciente  femenina  de  18  años  acude  al  cuarto  de  urgencias  por  dolor  abdominal  de  aparición  súbita  en  
bajo  vientre  de  3  horas  de  evolución.  Al  Examen  físico  el  dolor  es  difuso  pero  de  predominio  en   Fosa  iliaca  
derecha,  sin  signos  peritoneales  GB  9000  con  75%  de  neutrófilos.  Cuál  de  las  siguientes  afirmaciones  es  cierta:  
a.   La  ausencia  de  signos  peritoneales  excluye  un  cuadro  quirúrgico  
b.   El  hemograma  normal  excluye  un  cuadro  quirúrgico  
c.   El  USG  pélvico  no  está  indicado  
d.   La  paciente  debe  ser  re-­‐evaluada  en  4-­‐5  horas  
 
142.    Con  respecto  al  caso  anterior  todas  son  ciertas,  excepto:  
a.   Se  deben  iniciar  antibióticos  inmediatamente  
b.   Se  debe  solicitar  un  urinálisis  
c.   La  historia  menstrual  de  la  paciente  es  muy  importante  
d.   El  dolor  difuso  es  típico  del  dolor  viceral  
 
143.   Una  mujer  de  18  años  tiene  historia  de  24  horas  de  evolución  de  dolor  de  inicio  en  mesogastrio  y  luego  
irradiado  a  bajo  vientre  T  38.5  C,  GGB  13.5  con  96%  de  neutrófilos.  FUM  hace  3  semanas.  Examen  fisico  hay  
signos  de  irritación  peritoneal  en  bajo  vientre,  principalmente  en  fosa  iliaca  derecha.  El  examen  ginecológico  
es  normal,  orto  negativo.  USG  pélvico  es  normal.  El  próximo  paso  a  seguir  sería:  
a.   Observación  por  12  horas  
b.   Apendicectomía  
c.   Laparascopía  diagnóstica  
d.   Colon  por  enema  
 
Dolor  Abdominal  
 
144.    Con  respecto  al  dolor  abdominal  todas  son  ciertas;  excepto:  
a.   La  apendicitis  aguda  es  el  Dx  quirúrgico  más  común  
b.   Los  mayores  de  50  años  deben  ser  estudiados  aún  cuando  desaparezca  el  dolor  
c.   El  dolor  parietal  es  localizado  
d.   El  primer  síntoma  de  una  obstrucción  intestinal  es  la  distensión  
   
145.    Hombre  de  35  años  con  dolor  en  fosa  iliaca  derecha  de  3  días  de  evolución  T  37.5C  GB  9.0  con  75%  de  
neutrófilos,  dolor  y  defensa  en  fosa  ilíaca  derecha.  Varios  episodios  similares  en  las  últimas  dos  semanas.  Al  
examen  físico  hay  aparente  masa  en  fosa  iliaca  derecha.  El  próximo  paso  a  seguir  sería:  
a.   US  abdominal  
b.   TAC  
c.   Laparotomía  
d.   Laparascopía  
 
146.    Después   de   tres   días   de   antibióticos   IV   su   es   de   T   38.5   C,   GB   22   con   82%   neutrófilos.   El   dolor   a  
aumentado.  El  próximo  paso  a  seguir  será:  
a.   Nuevo  estudio  de  imagenología  
b.   Colonoscopía  
c.   Colon  por  enema  
d.   Laparotomía  
 
147.    El  Tx  inicial  de  la  obstrucción  intestinal  incluye  todas,  excepto:  
a.   Reposición  de  líquidos  y  electrólitos  
b.   Colocación  de  sondas  y  monitorización  de  signos  vitales  
c.   Radiografía  de  tórax  y  abdomen  
d.   Cirugía  inmediata  
 
148.    Todas  son  causas  comunes  de  obstrucción  mecánica  del  colon,  excepto:  
a.   vólvulo  del  sigmoides  
b.   Cáncer  de  colon  
c.   Diverticulitis  sigmoidea  
d.   Síndrome  de  Ogilvie  
 
149.   La  causa  más  común  de  obstrucción  intestinal  es:  
a.   Cáncer  de  colon  
b.   Divertículo  de  Meckel  
c.   Adherencia  post  operatorias  
d.   Diverticulitis  sigmoideas  
 
150.    En  la  obstrucción  intestinal  son  signos  de  sufrimiento  todos,  excepto:  
a.   Leucocitosis  
b.   Fiebre  
c.   Oliguria  
d.   Dolor  localizado  
 
151.    El  síntoma  más  temprano  en  la  obstrucción  intestinal  es:  
a.   Distensión  abdominal  
b.   Obstipación  
c.   Dolor  
d.   Vómitos  
 
152.    Hombre  de  65  años  acude  por  hx  de  48  horas  de  evolución  caracterizada  por  dolor  cólico,  obstinación  y  
distensión  abdominal,  tiene  una  laparatomia  previa,  la  rx  muestra  niveles  hidroaéreos  en  intestino  delgado  y  
gas   en   colon   derecho   y   transverso,   no   se   ve   gas   en   el   colon   izquierdo,   ni   en   la   ampolla   rectal,   cual   de   las  
siguientes  es  cierta:  
a.   La  obstrucción  del  colon  por  adherencia  es  común  
b.   La  paciente  tiene  una  obstrucción  total  del  intestino  delgado  
c.   La  obstrucción  intestinal  de  este  paciente  es  poco  probable  que  se  resuelva  con  medidas  conservadoras  
d.   Si  el  ciego  mide  más  de  5  cm  es  urgente  operarlo  
e.   Ninguna  de  las  anteriores  
 
153.    El  TAC  en  el  manejo  del  trauma  abdominal  puede  no  detectar  lesiones  de:  
a.   Diafragma  e  intestino  delgado  
b.   Hígado  y  riñones  
c.   Bazo  y  páncreas  
d.   No  tiene  utilidad  en  el  manejo  del  trauma  abdominal  
e.   Detecta  todas  las  lesiones  intra  abdominales  
 
154.   E  n  el  grupo  etario  entre  20  –  40  años  la  causa  más  común  de  dolor  abdominal  que  requiere  cirugía  es:  
a.    Obstrucción  intestinal.  
b.   Trombosis  mesentérica.  
c.   Apendicitis  aguda.  
d.   Diverticulitis  colónicas.  
 

155.   Paciente  femenina  de  18  años  acude  al  cuarto  de  urgencias  por  dolor    
                       abdominal   de   aparición   súbita   en   bajo   vientre   de   3   hrs   de   evolución.     Al   EF   el   dolor   es   difuso   pero   de  
predominio  en  FID  sin  signos  peritoneales.    GB  9000  con  75%  neu.    
1.    Cuál  de  las  afirmaciones  es  cierta:  
a.   La  ausencia  de  signos  peritoneales  excluye  un  cuadro  quirúrgicos.  
b.   El  hemograma  normal  excluye  un  cuadro  quirúrgico.  
c.   El  ultrasonido  pélvico  no  esta  indicado.  
d.   La  paciente  debe  ser  re-­‐evaluada  en  unas  4-­‐5  hrs.  
 
2.   Con  respecto  al  caso  anterior  todas  son  ciertas  excepto:  
a.   Se  deben  iniciar  antibióticos  inmediatamente.  
b.   Se  debe  solicitar  un  urinálisis.  
c.   La  historia  menstrual  de  la  paciente  es  muy  importante.  
d.   El  dolor  difuso  es  típico  del  dolor  visceral.  
 
156.   Con  respecto  al  dolor  abdominal  todas  son  ciertas  excepto:  
a.   La  apendicitis  aguda  es  e  DX.  Quirúrgico  más  común.  
b.   Los  mayores  de  50  años  deben  ser  estudiados  aún  cuando  desaparezca  el  dolor.  
c.   El  dolor  parietal  es  localizado.  
d.   El  primer  síntoma  de  una  obstrucción  intestinal      es    la  distensión.  
 
157.   Hombre  de  65  años  acude  por  historia  de  48  hrs  de  evolución  caracterizada  por  dolor  cólico,  constipación  
y   distensión   abdominal.     Tiene   una   laparotomía   previa.     RX   muestras   niveles   hidro-­‐aéreos   en   intestino  
delgado  y  gas  en  colon  derecho  y  transverso  no  se  ve  gas  en  colon  izq.  Ni  en  la  ampolla  rectal.    Cual  de  las  
siguientes  es  cierta:  
a.   La  obstrucción  del  colon  por  adherencias  en  común  
b.   El  paciente  tiene  una  obstrucción  total  del  intestino  delgado  
c.   La   obstrucción   intestinal   de   este   paciente   es   poco   probable   que   se   resuelva   con   medicinas  
conservadores  
d.   Si  el  ciego  mide  más  de  5  cms  es  urgente  operarlo  
e.   Ninguna  de  las  anteriores  
 
158.   Mujer   de   18   años   tiene   historia   de   24   horas   de   evolución   de   dolor   de   inicio   en   mesogastrio   y   luego  
irradiado  a  bajo  vientre.  T  38.5  ºC,  GB  13.5  con  96%  de  neutrófilos.  FUM  hace  3  semanas.  (buscar  respuesta)  
a.   Observación  por  12  horas  
b.   Apendicectomía  
c.   Laparoscopía  diagnóstica  
d.   Colon  por  Enema  
e.   Ninguno  de  las  anteriores  
 
159.   Hombre  de  65  años  con  dolor  en  FID  de  3  días  de  evolución  T  37.5  ºC  GB  9.0  con  75%  neu.  Dolor  y  defensa  
en  FID.    Varios  episodios  similares  en  las  últimas  dos  semanas.    Al  E.F.  hay  aparente  masa  en  FID.    El  próximo  
paso  a  seguir  sería  (buscar  respuesta)  
a.   US  abdominal  
b.   TAC  
c.   Laparotomía  
d.   Laparoscopía  
e.   Ninguna  de  las  anteriores  
 
160.   Después  de  tres  días  de  antibióticos  IV  su  Tº  es  de  38.5  ºC  GB  22  con  82%  neu.    El  dolor  ha  aumentado,    El  
próximo  paso  a  seguir  sería:  
a.   Nuevo  estudio  de  imagenología  
b.   Colonoscopía  
c.   Colon  por  enema  
d.   Laparotomía  
e.   Ninguna  de  las  anteriores.  
 
161.   En  el  diagnóstico  de  dolor  abdominal,  el  factor  más  importante  a        
                         considerar  es:  (buscar  respuesta)  
Edad  del  paciente  
Estado  socioeconómico  

a.   Ocupación  del  paciente  


b.   Sexo  
c.   Raza  
 
162.   Una  dama  de  22  años  consulta  por  dolor  abdominal  de  más  de  12  horas  de  evolución.  GB  8,000  con  66%  
neutrófilos.  Hay  defensa  y  rebote  en  todo  el  hemiabdomen  inferior.  El  próximo  paso  a  seguir  sería:  (buscar  
respuesta)  
a.   Apendicectomía  
b.   Laparoscopía  
c.   Observación  
d.   Ultrasonido  
e.   TAC  abdominal  
 
163.   Un  hombre  de   20  años  consulta  por  dolor  abdominal  de  12  horas  de  evolución.  Al  EF  no  hay  signos  de  
irritación   peritoneal.   Sólo   dolor   leve   al   a   palpación   profunda   en   FID   GB   16,000   con   85%   de   neutrófilos.   El  
diagnóstico  más  probable.  
a.   Adenitis  mesentérica  
b.   Apendicitis  aguda  
c.   Gastroenteritis  
d.   Colecititis  
e.   Pancreatitis  
 
164.   Un  hombre  de  70  años,  consulta  por  dolor  abdominal,  de  más  o  menos  24  horas  de  evolución.  El  dolor  es  
tipo  cólico.  No  hay  signos  de  irritación  peritoneal,  pero  si  hay  dolor  leve  en  fosa  ilíaca  izquierda.  GB  10,5000  N  
78%  Hgb  11.2  g.  Hay  escasos  niveles  hidroaéreos  del  intestino  delgado.  Hay  historia  de  pérdida  de  peso  no  
cuantificada  durante  los  últimos  dos  meses.  El  diagnóstico  más  probable  sería:  
a.   Apendicitis  guda  
b.   Diverticulitis  aguda  
c.   Isquemia  intestinal  
d.   Obstrucción  intestinal  
e.   Tiflitis  
 
165.   Un  hombre  de  25  añosconsulta  por  dolor  abdominal  tipo  cólico  de  más  o  menos  6  horas  de  evolución  y  
vómitos.  Tiene  una  laparotomía  previa  hace  1  año  por  trauma  penetrante.  No  esta  distendido,  no  hay  nivles  
hidroáreos.  GB  8,700  con  78%  de  neutrófilos.  El  diagnóstico  más  probable:  
a.   Apendicitis  aguda  
b.   Gastritis  
c.   Litiasis  urinaria  
d.   Obstrucción  intestinal  
 
 
Cáncer  de  Colon  
 
166.    Cuál  de  las  siguientes  se  consideran  lesiones  premalignas  del  colon:  
a.   Pólipos  inflamatorios  
b.   Pólipos  adenomatosos  
c.   Pólipos  hiperplásicos  
d.   Pólipos  hamartomatosos  
 
167.    La  causa  más  común  de  hipercalcemia  es:  
a.   Cáncer  
b.   Hiperparatiroidismo  primario  
c.   Insuficiencia  renal  
d.   Deshidratación  
 
168.   Con  respecto  al  Cancer    Colorectal  todos  son  ciertas  excepto:  
a.   Solo  los  pólipos  adenomatosos  tienen  potencial  de  malignización  
b.   Las  mutaciones  k-­‐ras  están  implicadas  en  la  transformación  de  un  adenoma  en  adenocarcinoma.  
c.   La  detección  y  remoción  de  pólipos  no  ha  logrado  reducir  la  incidencia  de  cáncer  de  colon.  
d.   El  factor  pronóstico  mas  importante  es  la  presencia  de  adenopatías  metastásicas.  
e.   Las  dietas  bajas  en  fibras  y  altas  en  grasas  aumentan  la  incidencia  de  cancer  de  colon.  
 
169.   Con  respecto  al  cáncer  colorectal  todas  son  ciertas  excepto:  
a.   En  15%  de  los  casos  hay  una  historia  familiar  positiva.  
b.   En  75%  de  los  casos  no  hay  factores  predisponentes  identificables.  
c.   Los  exámenes  de  pesquiza  se  deben  iniciar  a  los  60  años.  
d.   La   determinación   de   sangre   oculta   anualmente   mas   sigmoidoscopía   flexible   c/5   años   es   un  protocolo  
aceptable  de  pesquiza.  
e.   Colonoscopía  c/10  años  es  un  protocolo  aceptable  de  pesquiza.  
 
170.   Con  respecto  al  cancer    colorectal  todas  son  ciertas  excepto:  (a  o  c)  
 
a.   El  cancer  colorectal  es  sintomático  desde  muy  temprano  en  su  evolución.  
b.   El  tenesmo  es  un  síntoma  de  cancer  rectal.  
c.   Los  cambios  en  los  hábitos  de  defecación  son  comunes  en  el  cancer  del  colon  pero  no  del  recto.  
d.   La  anemia  es  mas  frecuente  en  canceres  de  colon  derecho.  
e.   Casi  el  70%  de  los  casos  se  presentan  al  alcance  del  sigmoidoscopio  flexible.  
 
171.   Con  respecto  al  cancer  colorectal  todas  son  ciertas  excepto:    
e.   El  antígeno  carcino-­‐embrionario  tiene  mas  utilidad  en  el  seguimiento  que  en  el  tratamiento  inicial.  
f.   La  quimioterapia  adyuvante  siempre  está  indicado  en  los  Estadios  III.  
g.   La  Quimio-­‐radiación  neo  adyuvante  no  mejora  la  sobrevida    en  el  cancer  rectal  estadío  III.  
h.   En  la  evaluación  pre-­‐operatoria  de  los  pacientes  electivos  son  importantes  la  TAC  y  las  PFH.  
i.   Si  el  paciente  esta  obstruído  los  examenes  pre-­‐operatorios  no  son  necesario  
 
172.   El  cáncer  colorectal  el  gen  p53:  
a.   se  asocia  a  mutacion  del  gen  5q21  
b.   participa  en  la  muerte  celular  programada  
c.   sensibiliza  las  cèlulas  a  los  factores  de  crecimiento  
d.   es  un  oncogen  estimulador  de  cancer    
 
173.   cual  es  la  localizacion  màs  frecunte  del  ca  de  colonico:  
a.   colon  ascendente    
b.   recto  
c.   colon  transverso  
d.   colon  descendente  
e.        sigmoides  
 
174.   mujer  anciana  admitida  por  debilidad,  asma  perdida  de  peso,  y    masa  palpable  abdominal  Ella  tiene   un  
carcinoma  de  recto.  El  sitio  anatomico  es:  
a.   recto  
b.   colon  sigmoides  
c.   colon  izquierdo  
d.   colon  transverso  
e.   colon  drecho  
 
175.   Los  pólipos  de  colon  que  se  consideran  potencialmente  malignos  son:  
a-­‐   Pólipos  inflamatorios.  
b-­‐   Pólipos  hiperplasicos.  
c-­‐   Pólipos  hamartomatosos.  
d-­‐   Pólipos  adenomatosos.  
e-­‐   Todos  los  anteriores  
 
176.   En  la  poliposis  adenomatosa  familiar  el  riesgo  de  cancer  de  colon  es;  
a.   1%  sin  tratamiento  
b.   10%  sin  tratamiento  
c.   100%  sin  tratamiento  
d.   no  hay  riesgo  de  cancer  de  colon  
 
177.   A  un  hombre  de  65  años  se  le  realizó  una  colectomía  por  un  Adeno  Carcinoma  de  sigmoides  hace  6  días.    
Actualmente  su  estado  general  es  bueno  pero  tiene  T  de  38.8°C  desde  hace  un  día.    GB=15.3  con  80%  N.    Con  
respecto  a  la  evaluación  de  la  infección  post-­‐operatoria  de  este  paciente  todas  son  ciertas  excepto:  
a.   Una   TAC   de   abdomen   es   el   mejor   método   para   investigar   la   presencia   de   infecciones   de   órganos   y  
espacios  del  sitio  operatorio.  
b.   La   presencia   de   infecciones   relacionadas   a   dispositivos   IV   solo   requiere   de   un   cultivo   cualitativo   de   la  
punta  del  catéter.  
c.   En  este  periodo  las  infecciones  de  la  herida  por  Streptococos    hemolíticos    no  son  frecuentes.  
d.   La  toxina  del  Clostridium  dificcile  es  la  responsable  de  los  síntomas  de  infecciones  entéricas.  
 
178.   según  la  clasificacion  de  dukes  para  el  cancer  de  colorectal  el  tumor  que  se  extiende  hasta  la  muscular,  se  
refiere  al  estadio:  
a.   A  
b.   B1  
c.   B2  
d.   C  
e.   D  
 
179.   cuando  el  tumor  de  colon  afecta  la  serosa  ,  según  la  clasificación  de  dukes  se  clasifica  como:  
a.   dukes  a  
b.   dukes  b2  
c.   dukes  c  
d.   dukes  d  
e.   ninguna  de  las  anteriores  
 
180.   el  síntoma  más  caracteristico  de  sospecha  de  cáncer  de  colon  izquierdo:  
a.   obstrucción  
b.   dolor  còlico  
c.   anemia  
d.   perdida  de  peso  
e.   ninguna  de  las  anteriores  
 
181.   la  prueba  mas  utilizada  para  detección  temprana  de  cáncer  de  colon  es:  
a.   rectoscopia  
b.   colon  por  enema  
c.   sangre  en  heces  
d.   colonoscopia  
e.   ninguna  de  las  anteriores  
 
Anomalías  Anorectales  
 
182.   El  examen  más  preciso  en  la  evaluación  del  cáncer  de  recto  es:  
a.   El  tacto  rectal  
b.   La  tomografía  axial  computarizada  
c.   El  ultrasonido  endorectal  
d.   La  resonancia  magnética  nuclear  
 
183.   En  la  fisura  anal  la  tríada  de  Brodie  se  caracteriza  por:  
a.   Sangrado,  dolor  y  masa  anal  
b.   Fisura  anal  lateral,  prurito  e  ictericia  
c.   Fisura  anal,  colgajo  cutáneo  y  papila  hipertrófica  
d.   Hemorroides  grado  III,  dolor  y  papila  hipertrófica.  
 
184.   Un  hombre  de  35  años  acude  con  dolor  perianal  que  no  le  permite  sentarse,  fiebre  y  diarrea  de  4  días  
de   evolución.     Al   E.F.   la   región   perianal   es   normal   excepto   por   un   endurecimiento   muy   doloroso   del   lado  
derecho  que  hace  muy  difícil  el  examen.    El  manejo  debe  ser:  
a-­‐  Incisión  y  drenaje  del  absceso  perianal  
b-­‐  Baños  de  asiento  por  hemorroides  internas.  
c-­‐  Antibióticos  por  su  gastroenteritis.  
d-­‐  Laxantes  por  su  fisura  anal.  
a-­‐   Ninguna  de  las  anteriores  
 
185.   En  la  fisura  anal  el  dolor  post-­‐defecatorio  se  debe  a:  
a.   Apertura  de  la  fisura  anal  
b.   Espasmo  del  esfínter  externo  
c.   Espasmo  del  esfínter  interno  
d.   Espasmo  del  elevador  del  ano  
 
 
186.   Las  causas  más  comunes  de  dolor  anal  son:  
a.   Fístula,  hemorroides  sangrante  y  proctalgia  fugax  
b.   Abscesos,  fístulas  y  hemorroides  internas  
c.   Cáncer  de  ano,  hemorroides  y  fisuras  
d.   Abscesos,  fisuras  y  trombosis  hemorroidal  externa  
 
187.   Las  hemorroides  internas  grado  II    son:  
a.   Las  que  sangran  sin  prolapsarse  
b.   Las  que  se  prolapsan  y  se  reducen  manualmente  
c.   Las  que  se  prolapsan  y  se  reducen  espontáneamente  
d.   Las  que  se  prolapsan  y  no  se  pueden  reducir  
 
188.   hay  una  indicación  quirurgica  clara  en  pacientes  con  enfermedad  hemorroidal:  
e.   si  las  hemorroides  son  grado  I  
f.   Si  las  hemorroides  estan  trombosadas  
g.   Si  la  paciente  esta  embarazada  y  tiene  hemorroides  grado  III  
h.   Nunca  
b.   Siempre  
 
189.   Las  hemorroides  internas  de  tercer  grado  se  caracterizan  por:  
a.   prolapso  hemorroidal  que  se  reduce  espontàneamente  
b.   prolapso  hemorroidal  que  se  reduce  manualmente  
c.   prolapso  hemorroidal  encarcelado  
d.   ninguna  de  las  anteriores.  
190.   El  Tratamiento  de  los  abscesos  anales  consiste  en  :    
a.   antibioticos  
b.   baños  de  asciento  
c.incisión  y  drenaje  
d.   anti-­‐inflamatorios  
 
191.   El  tratamiento  del  absceso  anal  siempre  es  drenaje,  excepto  en:  
a.   Ancianos  
b.   Inmunosuprimidos  
c.   Pacientes  con  insuficiencia  renal  
d.   Pacientes  leucémicos  
 
192.   En  los  abscesos  anales  los  antibióticos  están  indicados  en  :  
.   diabéticos  
.   inmunosuprimidos  
.   leucémicos  
.   todos  los  anteriores  
.   ninguno  de  los  anteriores  
 
193.   La  complicacion  mas  severa  del  abseco  anorectal  no  tratado  es:  
a.   gangrena  meleney  
b.   fascitis  necrotizante  
c.   mionecrosis  
d.   gangrena  de  fournier  
 
194.   Las  fístulas  anales  se  caracterizan  por  
a.   Orificio  perianal  con  descarga  de  pus  intermitente  
b.   Sangrado  al  evacuar  
c.   Masa  pediculada  en  el  ano  luego  de  un  absceso  
d.   Eritema  perianal  asociado  a  prurito  
 
195.   Fiebre  y  dolor  perianal  son  caracterìsticos  de:  
a.   hemorroides  grado  IV  
b.   Abseso  perianal  
c.   Fisura  anal  crònica  
d.   Quiste  pilonidal  
e.   oxiuriasis  
 
 
Cirugía  Pediátrica  
 
196.    Paciente  de  6  meses  de  edad  la  madre  refiere  que  se  encuentra  intranquilo,  no  quiere  la  leche,  vómitos  
verdosos.   Al   examen   físico   se   encuentra   intranquilo   e   impresiona   masa   dura   en   flanco   derecho.   El   dx   más  
probable  que  tenga  este  paciente  es:  
a.   Intususcepción  
b.   Plastrón  apendicular  
c.   Estenosis  pilórica  
d.   Malrotación  intestinal  
 
197.    Paciente  de  2  días  de  nacido  quién  refiere  la  madre,  que  al  llorar  presenta  masa  en  la  región  inguinal  y  
luego  se  desaparece.  El  diagnóstico  más  probable  de  este  paciente  es:  
a.   Hernia  Inguinal  
b.   Quiste  del  cordón  
c.   Hidrocele  
d.   Tumor  testicular  
     
 
198.    Una   niña   de   4   años   de   edad   previamente   sana,   es   traída   al  servicio   de   urgencias   con   un   cuadro   de   24  
horas   de   evolución   caracterizado   por   sangrado   rectal   y   mareo,   sin   síntomas   GI.   A   la   exploración   física   se  
encuentra  pálida,  Fc  140,  presenta  hipotensión  postural.  Su  abdomen  no  está  distendido  ni  es  doloroso  y  al  
examen  rectal  hay  sangre  fresca  y  coágulos  en  el  ámpula.  El  diagnóstico  más  probable  es:  
a.   Divertículo  de  Meckel  sangrante  
b.   Pólipo  rectal  juvenil  
c.   Hemorroides  
d.   Fisura  anal  
e.   Intusucepción  
 
 
199.   Paciente   de   24   horas   de   nacido,   se   observa   taquipneico,   al   examen   físico   se   encuentra   un   abdomen  
excavado  y  el  murmullo  vesicular  disminuido  en  lado  derecho  y  matidez.    El  diagnóstico  más  probable  es:  
a.   Atresia  esofágica.  
b.   Ruptura  de  bula  enfisematosa  congénita.  
c.   Hernia  diafragmática  
d.   Neumotórax  por  barotrauma.  
 
 
200.   Paciente  de  2  años  de  edad,  es  traído  por  su  madre  al  cuarto  de  urgencias  por  presentarse  irritable  con  
vómitos  biliosos,  además  presenta  heces  diarreicas  rojas  con  moco  y  espesas.  Al  examen  físico  se  palpa  una  
tumoración  en  fosa  y  flanco  derecho    el  diagnostico  más  probable  de  este  paciente  es:    
a.   Mal  rotación  intestinal  
b.   Plastrón  apendicular  
c.   Invaginación  intestinal  
d.   Megacolon  congénito.  

 
Hernias  e  Hidrocele  
 
201.    Paciente   de   25   años   de   edad,   se   queja   que   presenta   una   masa   no   reducible   del   testículo   derecho   al  
examen   físico   no   se   reduce,   hay   transiluminación   y   no   se   palpa   el   testículo   derecho.   El   diagnóstico   más  
probable  es:  
a.   Hernia  inguinal  
b.   Quiste  del  cordón  
c.   Hidrocele  
d.   Tumor  testicular  
 
202.    Un   hombre   obeso   de   58   años   acude   por   presentar   una   tumoración   inguinal   dolorosa   con   signos  
inflamatorios  leves  que  no  desaparece  desde  hace  24  horas.  Dice  que  tiene  una  hernia  del  mismo  lado  pero  
que  normalmente  se  la  puede  reducir.  Usted:  
a.   Usted  le  ordena  AINES  para  re-­‐evaluarlo  en  24  horas  
b.   Le  punciona  la  masa  
c.   Le  dice  que  necesita  cirugía  urgente  
d.   Trata  de  ver  si  se  puede  reducir  la  masa  
 
203.    Un   hombre   obeso   de   58   años   acude   por   presentar   una   tumoración   inguinal   dolorosa   con   signos  
inflamatorios  leves  que  no  desaparece  desde  hace  24  horas.  Dice  que  tiene  una  hernia  del  mismo  lado  pero  
que  normalmente  se  la  puede  reducir.  Usted:  
a.   Usted  le  ordena  AINES  para  re-­‐evaluarlo  en  24  horas  
b.   Le  punciona  la  masa  
c.   Le  dice  que  necesita  cirugía  urgente  
d.   Trata  de  ver  si  se  puede  reducir  la  masa  
e.   Le  prescribe  antibióticos  
 
204.   Cuando  un  órgano  forma  parte  del  saco  herniario,  esta  hernia  recibe  el  nombre  de:  
a.   Hernia  deslizante  
b.   Hernia  de  Ritcher  
c.   Hernia  Encarcelada  
d.   Hernia  Inguinal  indirecta  
e.   Hernia  en  pantalón  
 
205.    La  persistencia  del  proceso  vaginalis  muchas  veces  resulta  en:  
a.   Hidrocele  del  cordón  espermático  
b.   Hernia  inguinal  indirecta  
c.   Hernia  inguinal  directa  
d.   Hernia  Femoral  
e.   A  y  b  
f.   A,b,c  
 
206.    El  músculo  cremaster  que  recubre  el  cordón  inguinal  es  la  prolongación  de:  
a.   Músculo  oblicuo  externo  o  mayor  
b.   Músculo  oblicuo  interno  o  menor  
c.   Músculo  transverso  del  abdomen  
d.   Fascia  transversales  
e.   Músculo  Recto  Abdominal  
 
207.    La  tasa  más  baja  de  recurrencias  en  operaciones  por  hernias  inguinales  se  observa  con:  
a.   Herniorrafia  tipo  Bassini  
b.   Herniorrafia  tipo  Mc  Vay  
c.   Herniorrafia  abierta  con  malla  de  polipropileno  
d.   Todas  tienen  tasas  de  recurrencias  similares  
 
208.    La  hernia  inguinal  directa:  
a.   Sale  por  el  triángulo  de  Hasselbach  
b.   Es  adquirida  
c.   Está  relacionada  a  esfuerzos  físicos  
d.   Todas  las  anteriores  
 
209.    La  hernia  inguinal  indirecta:  
a.   Es  congénita  
b.   Sale  por  el  anillo  profundo  
c.   Está  rodeada  por  el  músculo  cremaster  
d.   Todas  las  anteriores  
 
Tiroides  y  Paratiroides  
 
210.    El  síntoma  más  temprano  de  hipocalcemia  post  operatoria  es:  
a.   Signo  de  Chvostek  
b.   Signo  de  trousseau  
c.   Espasmo  carpo-­‐pedal  
d.   Parestesias  peri-­‐orales  
 
211.    Son  signos  de  mal  pronóstico  en  un  paciente  con  nódulo  tiroideo  todos  menos:  
a.   Tamaño  mayor  de  3  cm  
b.   Sexo  masculino  
c.   Exposición  a  radiación  
d.   Dieta  baja  en  yodo  
e.   Edad  mayor  de  40  años  
 
212.    Una  mujer  de  60  años  asintomática  tiene  un  nódulo  tiroideo  de  3  cm  de  diámetro.  El  primer  estudio  Dx  a  
realizar  debe  ser:  
a.   Punción  con  aguja  fina  
b.   T4  
c.   Centelleo    de  tiroides  
d.   Ultrasonido  de  tiroides  
e.   CAT  de  cuello  
 
213.    Paciente  de  15  años  que  presenta  masa  en  la  región  cervical  del  cuello,  dolorosa  y  eritematosa,  al  elevar  
la  lengua  la  masa  se  mueve  hacia  arriba.  .  El  Dx  más  probable  es:  
a.   Higroma  quístico  
b.   Adenopatía  
c.   Neoplasia  de  tiroides  
d.   Quiste  tirogloso  
 
214.    Cuando  la  biopsia  de  una  masa  de  cuello  en  pacientes  con  riesgo  de  cáncer  debe  ser  realizada:  
a.   Inmediatamente  luego  de  ser  descubierta  
b.   Inmediatamente  luego  de  antibióticos  
c.   Cuando  no  hay  factores  de  malignidad  
d.   Si  la  masa  no  está  presenta  desde  la  infancia  
e.   Se  debe  realizar  luego  de  examen  endoscopico  de  la  boca,  laringe,  esófago  y  tráquea  
 
215.    Paciente  de  65  años  de  edad,  alcohólico  y  fumador  crónico,  además  de  vómitos  post  pandriales  y  pérdida  
de   peso   no   cuantificada.   Presenta   una   masa   en   región   supraesternal   izquierda   del   cuello,   el   manejo   más  
apropiado  sería:  
a.   Antibióticos  y  anti  inflamatorios  
b.   Observación  
c.   Biopsia  luego  de  endoscopia,  laringoscopía  y  broncoscopia  
d.   Biopsia  inmediata  
 
216.    Son  complicaciones  de  las  tiroidectomías  todas,  excepto:  
a.   hipocalcemia  
b.   Lesión  del  nervio  laríngeo  recurrente  
c.   Hipofosfatemia  
d.   Lesión  del  nervio  laríngeo  superior  
e.   Hemorragia  
   
217.    El  tumor  de  parótida  benigna  más  común  es  el:  
a.   Adenoma  pleomórfico  
b.   Lipoma  
c.   Adenocarcinoma  
d.   Linfoma  
e.   Ninguna  de  las  anteriores  
 
218.    La  causa  más  común  de  sodio  sérico  de  125  mEq/L  en  un  paciente  
                             post  operado  es:  
a.   Déficit  de  agua  
b.   Exceso  de  agua  
c.   Déficit  de  sodio  
d.   Exceso  de  sodio  
e.   Ninguna  de  las  anteriores  
 
219.   La  pseudohiponatremia  ocurre  secundariamente  en  la  siguiente  
                     Patología:  
a.   Enfermedades  del  SNC  
b.   Enfermedad  de  Addison  
c.   Acidosis  tubular  renal  
d.   Hiperglicemia  
e.   Hipopituitarismo  
 
Una  mujer  39  años  de  edad  se  presenta  al  consultorio  del  médico  para  la  evaluación  de  un  nódulo  palpable  en  el  
cuello  de  2  años  de  evolución.  Con  antecedente  de  enfermedad  de  Hashimoto  diagnosticada  hace  5  años,  para  la  
que   toma   hormona   tiroidea.   La   paciente   tiene   antecedente   de   radioterapia   en   tórax   de   dosis   baja   por   un  
crecimiento   del   timo   durante   la   infancia.   En   la   exploración   física   se   palpa   un   nódulo   de   2,5   cm   en   el   lóbulo  
izquierdo  de  la  tiroides,  firme  y  no  doloroso.  
220.   ¿Cuál  de  sus  antecedentes  aumenta  el  riesgo  para  cáncer  tiroideo?  
a-­‐    Grupo  de  edad  de  20-­‐40  años.  
b-­‐    Género  femenino.  
c-­‐      Radiación  de  dosis  bajas  durante  la  infancia.  
d-­‐      Antecedentes  de  la  enfermedad  de  Hashimoto.  
e-­‐      Todas  las  anteriores.  
   
221.   ¿Cuál  de  las  siguientes  es  el  mejor  paso  a  seguir.  
a-­‐  Ultrasonido  del  cuello.  
b-­‐  Centelleo  de  tiroides.  
c-­‐  Tomografía  de  cuello  y  tórax.  
d-­‐  Aspiración  del  nódulo  con  aguja  fina.  
e-­‐  Todas  las  anteriores.  
   
222.   El  hipertiroidismo  puede  ser  causado  por  las  siguientes  enfermedades  excepto:  
a.    Enfermedad  de  Graves.  
b.    Enfermedad  de  Hashimoto.  
c-­‐    Enfermedad  de  Plumier.  
d-­‐    Carcinoma  medular  de  tiroides.  
 
223.   Un  hombre  de  55  años  tiene  un  nódulo  en  el  polo  inferior  del  lóbulo  derecho   de  la  tiroides.      El  nódulo  
mide  4  cms  la    PAAF    fue  reportada  como  sospechosa.  Usted  le  recomienda.  
   
a-­‐                                  Cirugía  
b-­‐                                Iodo  radioactivo  
c-­‐                                  Observación  
d-­‐                                Otra  punción  
e-­‐                                  Tratamiento  supresivo  
   
224.   Un  hombre  de  55  años    tiene  un  nódulo  en  el  polo  inferior   del  lóbulo  derecho  de  la  tiroides.  El   nódulo  
mide  4  cm  y  la  PAAF  fue  reportada  como  sospechosa.  Usted  le  recomienda:  
a.            Cirugía  
b.            Yodo  radioactivo  
c.              Observación  
d.            Otra  punción  
e.              Tratamiento  supresivo  
     
225.   El  procedimiento  diagnostico  de  tamizaje  para  diferenciar  un  nódulo  tiroideo  maligno  de  uno  benigno:  
a.    USG  de  tiroides.  
b.    Centelleo  de  tiroides.  
c.    Biopsia  por  aspiración  con  aguja  fina.  
d.    Prueba  de  supresión  de  hormona  tiroidea.  a.  Antibióticos  de  amplio  espectro  
b.  TAC  en  las  primeras  48  horas  
c.  Atención  en  la  unidad  de  cuidados  intensivos  
d.  Laparotomía  por  necrosis  peri-­‐pancreáticas  
e.  Monitoreo  hemodinámica  invasivo  
 
   
226.   La  ronquera  secundaria  a  carcinoma  broncogénico  es  usualmente  debido  a          invasión  del  tumor  a:  
a.  Cuerdas  vocales  
b.  Nervio  Laringeo  superior  
c.  Nervio  laringeo  recurrente  
d.  Laringe  
 
Ulceras    
227.    Un   hombre   de   65   años   que   toma   AINES   diariamente   por   dolores   articulares   acude   por   historia   de  
hematemesis,  taquicardia  y  diaforesis.  La  endoscopía  alta  revela  una  úlcera  en  el  bulbo  duodenal  sin  sangrado  
activo  pero  con  vaso  visible.  El  paso  a  seguir  sería:  
a.   A  sala  con  infusión  de  omeprazole  
b.   Vaguectomía  más  piloroplastía  
c.   Aplicar  electrocauterio  o  clips  endoscopicamente  al  vaso  visible  
d.   Balón  de  compresión  intra  gástrico  
e.   Lavados  gástricos  contínuos  
 
228.    Un   sujeto   de   55   años   de   edad   asiste   al   consultorio   del   médico,   se   queja   de   dolor   en   la   parte   alta   del  
abdomen   de   2   meses   de   evolución.   El   dolor   se   describe   como   retorcijón,   localizado   en   hipogastrio,  
acompañado  de  náuseas,  el  dolor  se  exacerba  con  los  alimentos  y  pérdida  de  peso   de   10  kg,  los  últimos  2  
meses.   Tiene   antecedentes   de   tabaquismo,   ingesta   de   alcohol   ocasional,   así   como   Dx   de   úlcera   gástrica  
benigna.  Cuál  de  las  pruebas  siguientes  es  el  método  más  fidedigno  para  Dx  una  úlcera  gástrica  benigna:  
a.   Serie  esofagogastroduodenal  con  bario  
b.   Endoscopia  con  fibra  óptica  superior  
c.   CAT  abdominal  
d.   USG  endoscópico  
 
229.    Es  indicación  para  cirugía  en  la  enfermedad  úlcero  péptica:  
a.   Las  úlceras  no  curan  luego  de  12-­‐15  semanas  de  tx  
b.   Cuando  hay  recurrencia  de  la  úlcera,  a  pesar  de  su  tx  médico  
c.   Cuando  ocurre  una  complicación  
d.   Todas  las  anteriores  
e.   Ninguna  de  las  anteriores  
 
230.    Las  úlceras  gástricas  tipo  III  se  caracterizan  por:  
a.   Se  encuentran  en  la  curvatura  menor  
b.   Están  siempre  asociadas  a  una  úlcera  duodenal  
c.   Son  úlceras  gástricas  prepilóricas  
d.   Cerca  de  un  tercio  se  encuentran  asociadas  a  la  ingestión  de  aspirinas  o  AINES  
e.   Ninguna  de  las  anteriores  
 
231.    En  este  paciente  se  encontró  una  úlcera  gástrica  benigna  y  se  instaló  el  Tx  con  un  inhibidor  de  la  bomba  
de   protones   y   3   esquema   de   antibióticos   para   Helycobacter   pylori,   el   paciente   regresó   al   consultorio   del  
médico  3  meses  después  con  la  misma  sintomatología  y  en  la  revaloración  se  encontró  que  la  úlcera  gástrica  
persistía.  Cuál  de  las  siguientes  opciones  es  la  mejor  para  continuar  el  Tx:  
a.   Un  segundo  Tx  de  inhibidores  de  la  bomba  de  protones  con  triple  esquema  de  antibiótico  y  reevaluación  en  2  
meses  
b.   Un  Tx  de  prueba  con  sucralfalto  y  reevaluación  en  2  meses  
c.   Tx  quirúrgico  
d.   Un  Tx  con  prostaglandinas  y  revaloración    en  2  meses  
 
232.    Un   hombre   de   65   años   que   toma   AINES   diariamente   por   dolores   articulares   acude   por   hx   de  
hematemesis,  taquicardia  y  diaforesis.  La  endoscopia  alta  revela  una  úlcera  en  el  bulbo  duodenal  sin  sangrado  
activo  pero  con  vaso  visible,  el  paso  siguiente  a  seguir  será:  
a.   A  sala  con  infusión  de  omeprazol  
b.   Vaguectomía  más  piloroplatía  
c.   Aplicar  electrocauterio  o  clips  endoscopicamente  al  vaso  visible  
d.   Balón  de  compresión  intra  gástrico  
e.   Lavados  gástricos  contínuos  
 
233.   La  primera  línea  de  la  terapia  de  la  enfermedad  ulcero  péptica  incluye:  
a.   Vaguectomia  +  piloroplastia  
b.   Antrectomia  
c.   Determinación  de  gastrina  
d.   Descartar  Helicobacter  pylori  
e.   Anti  H2  
 
Cáncer  de  Mama  
 
234.    Paciente  de  60  años  con  cáncer  intraductal  mamario,  tumor  de  2  cm.  Sin  ganglios  palpables  en  axila,  el  
mejor  Tx  a  seguir  es:  
a.   Mastectomía  radical  modificada  
b.   Lumpectomía  más  vaciamiento  axilar  sin  radio  Tx  post  operatorio  
c.   Lumpectomía  más  biopsia  del  ganglio  centinela  con  radio  Tx  post  operatorio  
d.   Lumpectomía  solamente  
 
235.   Una  paciente  tiene  un  tumor  de  6  cm  en  la  mama.  Una  biopsia  con  aguja    
             Dx  carcinoma  ductal  infiltrante.  El  mejor  Tx  inicial  a  seguir  será:  
a.   Quimioterapia  pre  operatoria  
b.   Mastectomía  radical  modificada  
c.   Lumpectomía  más  vaciamiento  axilar  con  radioterapia  post  operatoria  
d.   Lumpectomía  más  vaciamiento  axilar  sin  radioterapia  post  operat  
e.   Ninguna  de  las  anteriores  
 
236.    Todo  lo  siguiente  es  cierto  acerca  de  cáncer  de  mama  no  invasivo,  excepto:  
a.   Se  define  como  aquel  cáncer  de  mama  que  no  avanza  más  allá  de  la  membrana  basal  
b.   Puede  ser  ductal  o  lobular  
c.   El  lobular  nunca  se  palpa  
d.   Ambos  tienen  buen  pronóstico  y  se  observan  como  microcalcificaciones  en  la  mamografía  
     
237.    Paciente  de  60  años  con  cáncer  intraductal  mamario,  tumor  de  2  cm,  sin  ganglios  palpables  en  axila,  el  
mejor  tx  a  seguir  es:  
a.   Mastectomía  radical  modificada  
b.   Lumpectomía  más  vaciamiento  axilar  sin  radio  tx  post  operatorio  
c.   Lumpectomía  más  biopsia  del  ganglio  centinela  con  radio  tx  post  operatorio  
d.   Lumpectomía  solamente  
e.   Ninguna  de  las  anteriores  
 
238.   Una   mujer   de   62   años   se   le   ha   diagnosticado   un   carcinoma   ductal   infiltrante   de   la   mama   T1   N0   M0  
receptores  estrogénicos  negativos  después  de  lumpectomía  y  biopsia  del  ganglio  centinela.    El  próximo  paso  a  
seguir  sería.  
a.   Mastectomía  radical  modificada  
b.   Quimioterapia  
c.   Radioterapia  
d.   Seguimiento  en  3  meses  
e.   Tamoxifen  
 
239.   Una  mujer  de  62  años  con  un  carcinoma  ductal  infiltrante  T2  N0  M0  receptores  estrogénicos  positivos  le  
pregunta  con  respecto  al  Tx  con  Tamoxifen.    Usted  le  aconseja  que:  (verificar  puede  ser  opción  c)  
a.   El  Tamoxifen  no  está  indicado  en  mujeres  post-­‐menopáusicas.  
b.   Debe  tomar  Tamoxifen  por  cinco  años.  
c.   Debe  tomar  Tamoxifen  por  tres  años  
d.   Si  los  ganglios  son  negativos  no  necesita  tomar  Tamoxifen.  
e.   Antes  de  tomar  una  decisión  se  debe  esperar  la  determinación  del  marcador  Her-­‐2  neu  
 
240.   Mujer  de  60  años  acude  por  presentar  una  masa  en  CSE  de  la  mama  derecha,  tiene  mamografías  que  es  
reportada  como  UNAM  asa  de  6  cms  sin  signos  mamográficos  de  malignidad.  El  próximo  paso  a  seguir  sería:  
a.   Biopsia  
b.   Nueva  mamografía  en  3  meses  
c.   Mastectomía  
d.   AINES  y  re-­‐evaluación  en  15  días  
e.   Biopsia  solo  si  tiene  factores  de  riesgo.  
 
241.   Una  paciente  de  45  años  se  le  ha  diagnosticado  un  carcinoma  ductal  infiltrante  T3  N1  M0  el  próximo  paso  
a  seguir  sería.  
 
a-­‐   Mastectomía  radical  modificada  
b-­‐   Quimioterapia  neoadyuvante  
c-­‐   Radioterapia  
d-­‐   Seguimiento  en  1  mes  
e-­‐   Tamoxifen  
 
242.   Una   Dama   de   45   años,   le   consulta   por   presentar   un   hallazgo   mamográfico   caracterizado   por  
microcalcificaciones   en   acúmulo,   con   sospecha   de   tumoración.     Categorizada   como   BIRADS   IV.   Al   examen  
físico    no  hay  tumoración  palpable.    El  próximo  paso  a  seguir  sería:  
   
a.   Biopsia  
b.   Estudios  genéticos  
c.   Observación  
d.   Resonancia  magnética  de  la  mama  
e.   Ultrasonido  mamario  
 
243.   A  una  mujer  de  58  años  se  le  detecta  un  tumor  de  2  cms  que  es  resecado  y  reportado  como  carcinoma  
ductal  infiltrante  con  márgenes  libres.    El  próximo  paso  a  seguir  sería:  
 
a-­‐   Biopsia  de  ganglio  centinela  
b-­‐   Mastectomía  radical  modificada  
c-­‐   Quimioterapia  neo-­‐adyuvante  
d-­‐   Radioterapia  
e-­‐   Tamoxifen  
 
244.   Las  ventajas  de  la  Mastectomía  radical  modificada  sobre  la  terapia  con  cirugía  conservadora  incluyen.    
 
a-­‐   Mayor  sobre  vida  
b-­‐   Menores  recurrencias  locales  
c-­‐   Menor  incidencia  de  metástasis  a  distancia  
d-­‐   Menor  morbilidad  
e-­‐   Ninguna  de  las  anteriores.  
 
245.   Una  mujer  de  55  años  tiene  una  tumoración  en  CSE  de  la  mama  derecha  de  4  cms  de  diámetro,  es  nodular  
cauchosa,  indolora  y  bien  definida,  tiene  6  meses  de  evolución.  Usted  le  recomienda:  
a-­‐   Mamografía  y  biopsia  
b-­‐   Mamografía  y  biopsia  solo  si  sospecha  malignidad  
c-­‐   No  hacer  nada,  es  un  fibroadenoma  
d-­‐   Mamografía  ahora  y  repetirla  en  tres  meses  para  ver  si  hay  cambios.  
e-­‐   Re-­‐evaluar  en  dos  semanas.  
 
246.   Un   Hombre   alcohólico,   de   65   años,   consulta   por   un   área   indurada   de   reciente   aparición   en   la   mama  
izquierda.    Al  examen  físico  usted  diagnostica    ginecomastia  izquierda  con  una  masa  dominante  en  el  C  S  E  y  
una  adenopatía  endurecida  en  la  axila  ipsilateral.        El  próximo  paso  a  seguir  será:  
a.   Biopsia  
b.   Estudios  genéticos  
c.   Evaluación  psiquiátrica  para  que  de  deje  de  tomar  
d.   Resonancia  magnética  de  la  mama  
e.   Ultrasonido  mamario  
 
247.   Pueden  ser  causas  de  ginecomastia  todas  excepto:  
a-­‐   Marihuana  
b-­‐   Cimetidina  
c-­‐   Cirrosis  hepática  
d-­‐   Ibuprofeno  
 
248.   Todas  las  mujeres  deben  realizarse  mamografías  de  pesquiza  anuales:  
a-­‐   A  partir  de  los  20  años  
b-­‐   A  partir  de  los  30  años  
c-­‐   A  partir  de  los  40  años  
d-­‐   A  partir  de  los  50  años    
 
249.   Una  muchacha  de  16  años,  consulta  por  una  tumoración  cauchosa  móvil,  lobulada  de  más  o  menos  3  cms.  
de  diámetro  en  el  CSE  de  la  mama  izquierda,  de  más  o  menos  seis  (6)  meses  de  evolución.  
Todos  los  siguientes  serán  apropiados  en  el  manejo  de  esta  Paciente  excepto:  
a.   Biopsia  
b.   Punción  con  aguja  fina  
c.   Reevaluación  en  tres  (3)  meses.  
d.   Resonancia  magnética  de  la  mama  
e.   Ultrasonido  mamario.  
 
250.   Una  adolescente  de  16  años  tiene  una  tumoración  bien  definida  de  5  cm.  de  diámetro,  en  el  cuadrante  
infero-­‐interno  de  la  mama  D,  es  móvil,  lobulado,  cauchosa  e  indolora.  El  tratamiento  debe  consistir  en:  
a.   Ultrasonido  y  excisión  del  fibroadenoma.  
b.   Mamografía.  
c.   Incisión  y  drenaje  de  absceso.  
d.   AINES.  
 
251.   Al  evaluar  una  masa  mamaria  son  signos  de  mal  pronostico  todos  excepto:  
a.   Edad  mayor  de  40  años.  
b.   Biopsia  previa  que  evidenció  cambios  proliferativos.  
c.   Cáncer  de  mama  en  una  hermana.  
d.   Secreción  verdosa  a  través  del  pezón.  
 
252.   Según   la   Sociedad   Americana   de   Cáncer   las   guías   a   seguir   para   el   tamizaje   del   cáncer   de   mama   son  
excepto:  
a-­‐  Toda  paciente  mayor  de  40  años  se  debe  realizar  mamografía  anual.  
b-­‐  Pacientes  menores  de  20  años  se  deben  hacer  auto  examen  de  la  mama  mensualmente.  
c-­‐  Pacientes  menores  de  38  años  deben  ser  examinadas  las  mamas  anualmente  por  un  médico.  
d-­‐   Pacientes  de  30  años  con  incremento  del  riesgo  de  cáncer  de  mama  se  debe  realizar  cada  6  meses  
mamografía  y  ultrasonido  de  mama.  
 
253.   Todo  es  cierto  acerca  de  la  mamografía  excepto:  
a-­‐  Es  útil  en  la  mujer  asintomática.  
b-­‐  El  10-­‐15%  de  los  canceres  de  mama  no  se  observan  en  la  mamografía.  
c-­‐  La  mayoría  de  los  canceres  de  mama  no  se  palpan.  
d-­‐  Es  útil  para  biopsias  dirigidas.  
 
 
254.   En  una  mujer  de  23  años  todo  los  siguientes  son  ciertos  excepto:  
a-­‐   Debe  efectuarse  un  auto  examen  manual  mensual  de  las  mamas  
b-­‐   Sus  mamas  deben  ser  examinadas  por  un  médico  por  lo  menos  una  vez  al  año.  
c-­‐   La  mamografía  es  de  poca  utilidad  porque  hay  que  irradiar  mucho  a  las  pacientes.  
d-­‐   El  auto  examen  manual  se  debe  realizar  después  de  la  menstruación  
 
255.   Paciente  de  25  años  de  edad  consulta  por  presentar  fiebre,  aumento  de  volumen  de  la  mama  derecha  y  
dolor.  Refiere  que  esta  amamantando  a  su  niña  que  tiene  1  año.  Al  examen  la  mama  se  encuentra  roja  y  dura.  
Usted  debe:  
a-­‐  Enviar  al  SOP  para  incisión  y  drenaje.  
b-­‐  Mamografía.  
c-­‐  Antibióticos,  calor  local  y  AINES.  
d-­‐Tomar  biopsia.  
   
 
256.   Pte  de  60  años  con  cancer  intraductal  mamario,  tumor  de  2  cm.  Sin  ganglios  palpables  en  axila,  el  mejor  
tratamiento  a  seguir  es:  
a-­‐   Mastectomía  radical  modificada.  
b-­‐   Lumpectomía  más  vaciamiento  axilar  sin  radio  Tx  post-­‐op.  
c-­‐   Lumpectomía  más  biopsia  del  ganglio  centinela  con  radio  Tx  post.op  
d-­‐   Lumpectomía  solamente.  
e-­‐   Ninguna  de  las  anteriores  
 
257.   Son  signos  de  mal  pronóstico  en  el  cáncer  de  mama.  Todos  excepto:  
a-­‐   Tumor  mayor  de  4  cms  
b-­‐   Ganglios  positivos  en  axila  
c-­‐   Carcinoma  lobular  in  situ  
d-­‐   Receptores  estrogénicos  negativos  
e-­‐   No  hay  exceptos,  todos  son  signos  de  mal  pronóstico.  
 
258.   Una  paciente  tiene  un  tumor  de  6  cms  en  la  mama.    Una  biopsia  con  aguja    diagnostica    carcinoma  ductal  
infiltrante.    El  mejor  Tx  inicial  a  seguir  sería:  
 
a-­‐   Quimio  terapia  pre-­‐operatoria.  
b-­‐   Mastectomía  radical  modificada.  
c-­‐   Lumpectomía  más  vaciamiento  axilar  con  radio  terapia  post-­‐operatoria.  
d-­‐   Lumpectomía  más  vaciamiento  axilar  sin  radio  terapia  post.op.  
e-­‐   Ninguna  de  las  anteriores  
 
259.   Para  cada  una  de  las  siguientes  preguntas  escoja  una  de  las  siguientes  opciones  
 
a)   Carcinoma  ductal  in  situ  
b)   Carcinoma  lobular  in  situ  
c)   Ambos  
d)   Ninguno  
 
     B    Más  común  en  mujeres  premenopáusicas  
     B      Nunca  es  palpable  
     A    Hallazgos  mamográficos  significativos  
     C      Riesgo  de  cáncer  invasivo  mas  o  menos  30%  
     B      Igual  riesgo  de  cáncer  invasivos  en  ambas  mamas  
     A    Cáncer  invasivo  se  presenta  en  el  mismo  sitio  del  cáncer                
                                     in  situ  
 
260.   Para  cada  una  de  las  siguientes  preguntas  escoja  una  de  las  siguientes  opciones  
 
a.   Carcinoma  lobular  in  situ  
b.   Carcinoma  ductual  in  situ  
c.   Ambos  
d.   Ninguno  
 
B        rara  vez  es  palpable  
A        Nunca  es  palpable  
A        Es  un  factor  de  riesgo  
B        Puede  avanzar  a  carcinoma  invasor  
B        Lumpectomía  sin  vaciamiento  axilar  
A        Observación  
A        Mastectomía  profiláctica  
D        Quimioterapia  pre-­‐operatoria  
B        Quimioterapia  post-­‐operatoria  
B        Microcalcificaciones  
 
 
Balance  Hidroelectrolítico  
261.    Usted   está   en   su   primer   día   de   internado   y   el   residente   le   pide   que   le   corrija   la   hiponatremia   a   un  
paciente.  Usted  debe  tomar  en  cuenta  lo  siguiente:  
a.   Hacer  los  cálculos  del  déficit  y  suministrar  la  mitad  en  8  horas  
b.   La  reposición  no  debe  ser  mayor  d  20  mEq/h  
c.   Examinar  al  paciente  y  revisar  el  expediente  clínico  
d.   Darle  diurético  al  paciente  
 
262.    Usted   está   en   cuidados   intensivos   de   cirugía   y   es   su   primer   día   de   internado.   La   enfermera   lo   llama  
porque  el  paciente  operado  de  bypass  coronario  no  orina,  luego  de  haber  leído  el  expediente,  que  debe  hacer  
usted:  
a.   Aumentar  los  líquidos  intravenosos  
b.   Darle  diurético  al  paciente  
c.   Decirle  a  la  enfermera  que  lo  evalúe  en  una  hora  más  
d.   Revisar  la  sonda  foley  en  otros  dice  colocar  
 
263.    Usted  está  en  sala  9  y  es  su  primer  día  de  internado.  La  enfermera  lo  llama  porque  el  paciente  operado  de  
colecistectomía  no  orina,  luego  de  haber  leído  el  expediente,  que  debe  hacer  usted:  
a.   Aumentar  los  líquidos  intravenosos  
b.   Darle  diurético  al  paciente  
c.   Decirle  a  la  enfermera  que  lo  evalúe  en  una  hora  más  
d.   Revisar  la  sonda  foley  
 
264.    Causa  de  brecha  aniónica  elevada:  
a.   Hipoperfusión  
b.   Diarrea  
c.   Fístula  pancreática  
d.   Excesiva  administración  de  ácidos  
 
265.    La  cantidad  de  mEq/L  de  sodio  que  contiene  un  L  lactato  ringer  es  de    
a.   130  
b.  154  
b.   200  
c.   145  
d.   Ninguna  de  las  anteriores  
 
266.   Paciente   de   80   años   hospitalizado   en   la   unidad   de   cuidados   intensivos   revela   los   siguientes   gases  
arteriales  ph  7.25  PCO2  92  mmHg  bicarbonato  14  mEq/L  El  trastorno  ácido  base  que  esta  padeciendo  es:  
a.   Acidosis  respiratoria  
b.   Alcalosis  respiratoria  
c.   Acidosis  metabólica  
d.   Alcalosis  metabólica  
 
 243.   Paciente   masculino   de   30   años   de   edad   que   recibe   múltiples   impacto   de   bala   en   la   región   abdominal   es  
llevado   al   salón   de   operaciones   realizándole   laparotomía   exploradora   encontrando   devascularización   del  
intestino   delgado.     Se   le   realizó   resección   de   más   o   menos   2   metros   de   intestino   delgado   y   anastomosis.       La  
cirugía  tuvo  una  duración  de  4  horas  y  se  le  transfundieron  4  unidades  de  GRE.    En  el  postoperatorio  se  encuentra  
muy  distendido  con  un  drenaje  aumentando  a  través  del  tubo  de  naso  gástrico  
 Tiene  un  peso  de  80  Kg.    
 Sus  electrolitos  son:  Na  125  meq,  K  3,0  meq,  Cl  80  meq.    Creatinina  2,5.  Bun  48.  Glc  180.    
Gases  arteriales  7,4.    HCO3  30  meq,  PCO2  56  mm  Hg.  PO2  96  mm  Hg  y    Sat  de  oxígeno  100  %.  Exceso  de  Base  más  
4.  
 
267.    La  diuresis  horaria  de  este  paciente  no  debe  ser  menor  de:  (verificar  a,c)  
a.   30  cc  por  hora  
b.   50  cc  por  hora  
c.   40  cc  por  hora  
d.   10cc  por  hora  
 
268.   La  causa  de  la  hiponatremia  es  de  tipo:  
a.   Hipotónica  –  Hipovolemica  
b.   Hipotónica    -­‐  Isovolémica.  
c.   Hipotónica    -­‐  Hipervplemica.  
d.   Isotónica.  
 
269.    La  osmolaridad  calculada  de  este  paciente  es:  
a.   200  mosm  
b.   280  mosm  
c.   150  mosm  
d.   260  mosm  
 
270.   El  déficit  de  sodio  calculado  es:  
a.   600  meq.  
b.   560meq.  
c.   720  meq  
d.   920  meq  
271.   El  desequilibrio  ácido  –  base  que  presenta  es:  
a.   Alcalosis  metabólica  compensada.  
b.   Alcalosis  metabólica  descompensada.  
 
272.    El  déficit  de  potasio  de  este  paciente  es:  
a.   Menos  de  100  meq  
b.   150  meq  
c.   250  meq  
d.   Mayor  de  200  meq.  
 
273.   En  sala  tiene  un  paciente,  el  cual  diagnostica  una  hiponatremia  y  el  déficit  de  sodio  calculado  es  de  200  
meq  .    los  líquidos  que  usted  debe  ordenar  es:  
a.   SSN  500  cc  +  NaCl  a  23,4%          4    cc.  Pp  en  10  hrs.  
b.   L/R  1000  cc    +  NaCl  a  23,4%    18    cc.  Pp  en  10  hrs.  
c.   SSN  1000  cc  +  NaCl  a  23,4%    60  cc  pp  en  10  hrs.  
d.   SSN  1000  cc  +  NaCl  a  23,4%    10    cc    p  p  en  10  hrs.  

 
Shock  
 
274.    En  cuanto  a  la  relación  a  la  patogénesis  del  shock  séptico  y  sepsis  podemos  decir  que:  
a.   El  SIRS  se  puede  producir  solo  en  pacientes  de  trauma  
b.   La  sepsis  se  define  como  un  paciente  que  presenta  SIRS,  foco  infeccioso  e  hipotensión  irreversible  
c.   La   isquemia   intestinal   producida   en   el   shock   promueve   la   aparición   de   MODS   por   aumento   de   la  
translocación  bacteriana  
d.   La   nutrición   enteral   y   parenteral   no   están   indicados   en   pacientes   con   sepsis   por   el   riesgo   de   resistencia  
periférica  a  la  insulina  
 
   
275.   El  signo  más  temprano  de  hipovolemia  en  un  paciente  es:  
a.   Hipotensión  
b.   Taquicardia  
c.   Mucosa  seca  
d.   Disminución  de  la  diuresis  
e.   Estado  de  conciencia  
 
276.    En  cuanto  a  la  relación  del  intestino  en  la  patogénesis  del  shock    
                               séptico  y  sepsis  podemos  decir  que:  
a.   La  recontaminación  intestinal  selectiva  con  antibióticos  orales  a  disminuido  las  infecciones  y  la  mortalidad  
b.   La  utilización  de  fármacos  anti-­‐inflamatorios  ha  logrado  modular  la  respuesta  inflamatoria  en  el  síndrome  de  
disfunción  multiorgánica  
c.   La   isquemia   intestinal   producida   en   el   shock   promueve   la   aparición   de   MODS   por   aumento   de   la  
translocación  bacteriana  
d.   La  nutrición  parenteral  preserva  las  vellosidades  intestinales    
 
277.    Según  American  C..  y  Care  Medicine  1991,  se  define  SIRS  como:  
a.   Signos  inespecíficos  como  temperatura  >  38C  o  <  36,  leucocitosis  o  leucopenia,  taquipnea  y  taquicardia  
b.   Signos  inespecíficos  como  fiebre,  leucocitosis,  taquicardia,  taquipnea  y  gérmenes  detectados  en  hemocultivo  
c.   Situación  de  Shock  asociada  a  signos  clínicos  inespecíficos  como  temperatura>  38C  o  <36C,  leucocitosis   o  
leucopenia,  taquipnea  y  taquicardia  
d.   Infección  diagnosticada  microbiologicamente  acompañada  de  hipotensión  y  oliguria  
e.   Hipotensión  <90  mmHg  mantenida  a  pesar  de  perfusión  de  drogas  inotrópicas  
 
278.    Según  American  C..  y  Care  Medicine  1991,  se  define  SEPSIS  como:  
a.   Signos  inespecíficos  como  temperatura  >  38C  o  <  36,  leucocitosis  o  leucopenia,  taquipnea  y  taquicardia  
b.   Signos   inespecíficos   como   fiebre,   leucocitosis,   taquicardia,   taquipnea   y   gérmenes   detectados   en  
hemocultivo  
c.   Situación   de   Shock   asociada   a   signos   clínicos   inespecíficos   como   temperatura>   38C   o   <36C,   leucocitosis   o  
leucopenia,  taquipnea  y  taquicardia  
d.   Infección  diagnosticada  microbiologicamente  acompañada  de  hipotensión  y  oliguria  
e.   Hipotensión  <90  mmHg  mantenida  a  pesar  de  perfusión  de  drogas  inotrópicas  
 
279.    Paciente  de  35  años  el  cual  es  admitido  al  SOP  por  haber  sido  atropellado  por  un  autobús  con  múltiples  
lesiones  abdominales.  Transoperatoriamente  se  trato  agresivamente  el  shock  hipovolémico  y  la  cirugía  tuvo  
una   duración   prolongada   aproximadamente   de   6   horas.   Luego   de   48   horas   usted   está   en   la   unidad   de  
cuidados  intensivos  de  turno  y  la  enfermera  le  dice  que  no  está  orinando  a  pesar  que  tiene  buena  presión  y  la  
PVC  es  normal.  Usted  luego  de  revisar  la  sonda  urinaria  decide  colocar  un  diurético  pero  no  presenta  diuresis.  
a.   Este  paciente  probablemente  tiene  una  trombosis  de  la  arteria  renal  
b.   Posiblemente  su  presión  abdominal  es  mayor  de  15  mmHg  
c.   Se  debe  iniciar  diálisis  renal  
d.   Aumenta  el  goteo  de  los  líquidos  intravenosos  
 
280.   El  tx  inmediato  del  paciente  anterior  debe  ser:  
a.   Heparina  y  estreptoquinasa  
b.   Medir  la  presión  intraabdominal  
c.   Realización  de  fístula  vascular  para  hemodiálisis  
d.   Colasión  de  filtro  en  la  vena  cava  
 
281.   Los  líquidos  utilizados  en  la  resucitación  del  paciente  en  un  shock    
                               son  los  siguientes,  excepto:  
a.   Cristaloides  
b.   Coloides  
c.   Hemoderivados  
d.   Dextrosa  al  5%  
 
282.   Están  aumentados  en  el  shock  todos,  excepto:  
a.   Vasopresina  
b.   Epinefrina  
c.   pCo2  
d.   Cortisol  
e.   Acido  Láctico  
 
 
Heridas  y  Suturas  
283.    Para  el  tx  con  antibióticos  de  las  heridas  se  debe  tomar  en  cuenta  lo  siguiente:  
a.   Tipo  de  herida  
b.   Factores  del  paciente  
c.   Mecanismo  de  lesión  
d.   Todas  las  anteriores  
 
284.    Con  respecto  a  la  preparación    pre-­‐operatoria  del  paciente:  
a.   El  rasurado  debe  ser  extenso  y  meticuloso  
b.   El  rasurado  se  debe  realizar  la  noche  anterior  a  la  cirugía    
c.   La  preparación  con  yodo  es  bactericida,  fungicida  y  virucida  
d.   La  tintura  de  yodo  es  un  iodoforo  altamente  efectivo  
 
285.    En  cuanto  a  las  suturas  quirúrgicas  todas  son  ciertas,  excepto:  
a.   Las  suturas  absorvibles  producen  menos  reacción  tisular  que  las  no  reabsorvibles  
b.   Las  suturas  polifilamentosas  tienden  a  favorecer  las  infecciones    
c.   Una  sutura  3-­‐0  es  más  delgada  que  una  2-­‐0  
d.   El  ácido  poliglicólico  es  una  sutura  que  se  absorve  por  hidrólisis  
 
286.      Diga  2  ejemplos  de  herida  según  clasificación  por  el  riego  de  infección:  
a.   Limpia:    
b.   Limpia  Contaminada:  
c.   Contaminada:  
d.   Sucia:  
 
287.    En  cuanto  a  los  antisépticos  y  el  tx  de  las  heridas  todo  es  cierto,  
       excepto:  
a.   Los  antisépticos  retrasan  el  proceso  de  cicatrización  
b.   No  se  deben  utilizar  en  heridas  limpias  
c.   Su  penetración  es  superficial  
d.   El  peróxido  de  hidrógeno  es  un  potente  bactericida  
 
288.    Con  respecto  a  los  antibióticos  profilacticos  todo  es  cierto,  
       excepto:  
a.   Están  indicados  en  colecistotomias  sin  factores  de  riesgo  
b.   Están  indicados  en  apendicectomias  
c.   Están  indicados  en  cirugías  por  obstrucción  pilórica    
d.   Deben  ser  administrados  justo  antes  de  iniciar  la  operación  
 
Neoplasias  Gástricas  
289.    Todos  los  siguientes  son  signos  de  neoplasia  gástrica  avanzada:  
a.   Nódulo  de  Wichow  
b.   Nódulo  de  sister  mary  Joseph  
c.   Signo  de  Blumer  
d.   Todas  las  anteriores  
e.   Solo  a  y  c  
 
290.   Factor  etiológico  de  mayor  importancia  en  el  desarrollo  del  Cáncer  Gástrico  es:  
a-­‐   Bajo  consumo  de  vegetales.  
b-­‐   Alto  consumo  de  almidones.  
c-­‐    Alto  consumo  de  alimentos  salados  y  ahumados    
d-­‐   Infestación  por  el  Helicobacter  pylori.  
e-­‐   Ninguna  de  las  anteriores.  
 
291.   Se  debe  sospechar  de  Gastrinoma  o  Síndrome  de  Zollinger-­‐Ellison  en  el  siguiente  caso:  
a-­‐   Ulceras  resistentes  a  tratamiento  antisecretorio.  
b-­‐   Múltiples  ulceras  duodenales.  
c-­‐   Ulceras  que  recurren  luego  de  procedimiento  quirúrgicos  adecuados.  
d-­‐   Todas  las  anteriores.  
e-­‐   Ninguna  de  las  anteriores.  
 
292.   según  la  clasificaciòn  de  Ca  Gastrico  de  Borman  la  linitis  plàstica  es  el  tipo:  
a.tipo  I  
b.   tipo  II  
c.  tipo  III  
d.   tipo  IV  
e.tipo  V  
 
293.   todos  los  siguientes  son  signos  de  neoplasia  gàstrica  avanzada,  excepto:  
a.nodulo  de  vircow  
b.   nodulo  de  sister  mary  joseph  
c.  signo  de  blummer  
d.   signo  del  Moro  
e.ninguna  de  las  anteriores  
 
294.   Pte  de  75  años  el  cual  es  traído  al  cuarto  de  urgencias  del  HST  luego  de  haber  sido  tratado  en  el  centro  de  
Salud  con  antiácidos  y  metoclopramida  con  el  diagnostico  clínico  de  gastritis.  Los  familiares  informan  que  el  
señor  ha  perdido  peso  y  que  desde  hace  un  año  lo  estaban  llevando  al  centro  de  salud  porque  se  quejaba  de  
dolor  en  epigastrio  además  de  presentar  vómitos  post-­‐prandriales.  También  han  notado  la  aparición  de  una  
masa  sobre  la  clavícula  izquierda.  El  estudio  de  elección  de  este  paciente  inicialmente  debe  ser:  
           a.    Tomografía  axial  computarizada.  
           b.    Endoscopia  digestiva  alta.  
           c.    Colonoscopia.    
d.  Biopsia  de  la  masa  clavicular  izquierda.  
 
295.   Hombre  de  68  años  se  presenta  a  su  consultorio  quejándose  de  disfagia  progresiva  durante  los  últimos  
tres  meses,  relacionada  con  incomodidad  moderada  en  el  pecho  ,  el  paciente  refiere  una  pérdida  de  peso  de  
7,5   Kg.   ,   tiene   antecedente   de   tabaquismo   positivo   durante   30   años,   e   ingesta   de   alcohol   ocasional.   A   la  
exploración   física   sin   datos   de   interés,   incluso   los   signos   vitales,   una   radiografía   de   tórax   es   normal   y   el  
esofagograma   con   bario   muestra   un   defecto   de   llenado   irregular   en   el   tercio   distal   del   esófago   con  
deformación  y  estrechamiento  de  la  luz:  
             ¿Cuál  de  los  siguientes  diagnósticos  es  el  más  probable?  
                         a-­‐    Esofagitis  por  estenosis.  
                         b-­‐  Carcinoma  de  esófago.  
                         c-­‐    Carcinoma  pulmonar  con  invasión  a  esófago.  
                         d-­‐    Divertículo  de  Zenker.  
 
296.   El  carcinoma  típico  que  se  desarrolla  en  asociación  con  el  esófago  de  Barrett  es:  
a-­‐   Epidermiode.  
b-­‐   Mucoepidermoide.  
c-­‐   Células  pequeñas.  
d-­‐   Adenocarcinoma.  
e-­‐   Células  escamosas.  
 
297.   ¿Cual  es  la  afirmación  correcta  acerca  del  cáncer  gástrico?  
a-­‐  El  cáncer  gástrico  distal  es  el  más  común  actualmente.  
b-­‐  Los  cánceres  gástricos  tempranos  están  confinados  a  la  mucosa  y  submucosa.  
c-­‐  Los  tumores  gástricos  tipos  difusos  tiene  mejor  pronósticos  que  el  tipo  intestinal.  
               d-­‐  Todas  las  anteriores  son  ciertas.  
 
298.   su   tia   de   60   años   que   desde   hace   4   semanaas   se   vanìa   sintiendo   con   una   sensaciòn   de   llenura   post-­‐
prandial  y  dolor  ocasional  en  FID.  Usted  le  recomienda  que:  
a.   se  haga  mamografìa  
b.   se  haca  una  colonoscopia  
c.   se  haga  un  Pap  cervicouterino  
d.   todas  las  anteriores.  
 
 
 
 
299.   Son  todas  características  de  la  motilina,  excepto:  
a.   Es  un  péptido  intestinal  
b.   Es  secretado  por  las  células  enterocromafines  del  intestino  delgado  
c.   La  acidficación  del  duodeno  causa  aumento  de  su  liberación  
d.   La  eritromicina  inhibe  su  liberación  
 
Enfermedades  del  Hígado  
 
300.    Lesión  benigna  más  común  encontrada  en  el  hígado:  
a.   Metástasis  neoplásicas  
b.   Hepatocarcinoma  
c.   Hemangioma  
d.   Abcesos  hepáticos  
 
301.    La  ecuación  de  Harris  Benedict  nos  sirve  para:  
a.   Cuantificar  las  proteínas  totales  
b.   Cuantificar  los  carbohidratos  
c.   Nos  proporciona  el  gasto  energético  basal  
d.   Cuantificar  los  lípidos  
e.   Todas  las  anteriores  
 
302.    La  irrigación  del  hígado  esta  dada  por:  
a.   Arteria  hepática  25%,  vena  porta  85%  
b.   Arteria  hepática  75%,  vena  porta  25%  
c.   Arteria  hepática  25%,  vena  porta  75%  
d.   Arteria  hepática  50%,  vena  porta  50%  
 
Enfermedades  del  Bazo  
 
303.    Primer  signo  que  sugiere  rotura  traumática  del  bazo:  
a.   Inconciencia  
b.   Dolor  abdominal  
c.   Hipotensión,  palidez  
d.   Hematemesis,  taquicardia,  hipotensión  
 
ERGE  
304.    El  esófago  inferior  de  un  paciente  es  inadvertidamente  perforado  durante  una  endoscopia.  Un  trago  de  
bario   hecho   inmediatamente   luego   de   la   perforación   demuestra   salida   de   bario   hacia   el   espacio   pleural  
izquierdo.  Cuál  es  el  tx  adecuado  más  aceptado:  
a.   Observación  
b.   Observación  más  antibióticos  
c.   Toracotomía  cerrada  izquierda  
d.   Intubación  esofágica  prolongada  
e.   Drenaje  y  reparación  quirúrgica  de  la  lesión  
 
Anestésicos  
305.    Una  paciente  de  70  kg  que  llega  al  cuarto  de  urgencias  con  una  herida  de  cuello  amplia  y  que  usted  va  a  
suturar,  se  debe  considerar:  
a.   La  dosis  máxima  de  xilocaína  que  debe  infiltrarse  es  350  mg  
b.   La  dosis  máxima  de  xilocaína  que  debe  infiltrarse  es  de  250  mg  
c.   La  xilocaína  viene  en  concentración  de  50  mg/cc  
d.   La  vida  media  de  la  xilocaína  es  aprox  4  horas  
 
306.    Cuantos  mg  de  lidocaína  contienen  30  cc  de  lidocaína  al  2%:  
a.   200  mg  
b.   300  mg  
c.   400  mg  
d.   500  mg  
e.   600  mg  
 
307.    La  clasificación  de  ASA  nos  orienta  sobre:  
a.   El  riesgo  de  mortalidad  operatorio  
b.   El  riesgo  de  complicaciones  peri  operatorias  
c.   El  riesgo  cardiológico  peri  operatorio  del  paciente  
d.   Ninguna  de  las  anteriores  
 
308.      La  ventaja  de  la  analgesia  controlada  por  el  paciente  es:  
a.   El  paciente  tiene  control  sobre  su  medicamento  
b.   Independencia  de  la  enfermera  
c.   Inmediata  medicación  
d.   Rápida  analgesia  
e.   Todas  las  anteriores  
 
     
309.   Durante  la  etapa  pre  anestésica  podemos  decir  que:  
a.   La  clasificación  de  ASA  correlaciona  el  estado  general  del  paciente  y  el  riesgo  anestésico  
b.   La  clasificación  de  Mallmpati  correlaciona  el  estado  general  del  paciente  y  el  riesgo  anestésico  
c.   Los  pacientes  ASA  V,  son  siempre  de  intubación  difícil  
d.   Los  pacientes  Mallampati  gradi  I,  son  siempre  de  intubación  dificil  
e.   Los  pacientes  ASA  I  corrientemente  presentan  problemas  durante  el  acto  anestésico  
 
310.    El  orden  de  aparición  clínica  de  signos  de  intoxicación  por  anetésicos  locales  es:  
a.   Adormecimiento  de  la  lengua,  confusión,  paro  cardíaco,  paro  respiratorio  
b.   Adormecimiento  de  la  lengua,  confusión,  paro  respiratorio,  paro  cardiaco  
c.   Confusión,  adormecimiento  de  la  lengua,  paro  respiratorio,  paro  cardiaco  
d.   Confusión,  adormecimiento,  paro  cardiaco,  para  respiratorio  
 
Insuficiencia…  (circulación)  
311.    La  triada  de  Virchow  consta  de:  
a.   Estasis,  edema  y  hipercoaguabilidad  
b.   Estasis,  enrojecimeinto  cutáneo  y  edema  
c.   Estasis,  hipercoaguabilidad,  lesión  vascular  
d.   Ninguna  de  las  anteriores  
 
312.   Paciente  con  riesgo  aumentado  de  trombosis  venosa  elevado,  excepto:  
a.   Encamados  
b.   Uso  de  anticonceptivos  orales  
c.   DM  
d.   Insuficiencia  cardiaca  
 
313.    Todo  es  cierto  de  los  aneurismas  abdominales,  excepto:  
a.   Los  aneurismas  mayores  de  5cm  tienen  mayor  riesgo  de  ruptura  
b.   Su  etiología  es  multifactorial  
c.   Es  una  enfermedad  de  ancianos  
d.   La  mayoría  de  los  aneurismas  son  asintomáticos  
 
   
314.   El  factor  de  mal  pronóstico  más  importante  en  el  shock  hemorrágico  por  traumatismo  es:  
a.   El  aporte  masivo  de  volumen  en  la  atención  pre  hospitalaria  
b.   El  tiempo  transcurrido  desde  el  impacto  hasta  la  cirugía  
c.   La  atención  realizada  en  el  hospital  de  referencia  
d.   La  realización  de  las  pruebas  dx  apropiadas  
e.   Existencia  de  lesiones  ortopédicas  
 
315.    Usted  tiene  una  paciente  que  tiene  dx  de  TEP,  se  le  inicia  la    
     Heparinización  con  un  bolo  de  80  unidades  y  una  infusión  de  18  
                         unidades  por  hora.  Se  toma  un  control  de  TPT  a  las  4  horas  y  es  el    
                           siguiente  TPT    P35  TPTC  35,  usted  debe:  
a.   Bolo  de  80  unidades  de  heparina  
b.   Bolo  de  40  unidades  de  heparina  
c.   Disminuir  la  infusión  y  esperar  2  horas  
d.   Bolo  de  100  u  de  heparina  
 
316.    Subitamente   el   paciente   anterior,   luego   de   haberle   administrado   5000   u   de   heparina   comienza   a  
presentar  hematemesis  y  por  los  sitios  de  punción  hay  sangrado  espontáneo,  se  realiza  un  control  de  TPT  y  es  
informado  así,  TPTP  es  >1  minuto.  Usted  debe:  
a.   Detener  la  heparina  
b.   Detener  la  heparina  +  50  mg  de  sulfato  de  protamina  
c.   Detener  heparina  +  4  u  de  plasma  fresco  congelado  +  50  mg  de  sulfato  de  protamina  
d.   Detener  la  heparina  y  trnsfundir  3  u  de  plasma  congelado  
 
   
317.   Todo  los  siguientes  son  síntomas  de  la  insuficiencia  arterial,  excepto:  
a.   Atrofia  muscular  
b.   Edema  de  miembros  inferiores  
c.   Vello  escaso  
d.   Rubor  
 
Obesidad  Morbida  
318.    El  índice  de  masa  corporal  de  un  paciente  que  pesa  180  kg  y  mide  
                               1.70  metros  es  de:  
a.   105  
b.   62  
c.   181  
d.   0.009  
e.   Ninguna  de  las  anteriores  
 
Patología  Colónica  
319.    Son  indicaciones  para  colocación  de  drenajes,  excepto:  
a.   Luego  de  una  sutura  hepática  secundaria  a  un  traumatismo  
b.   Luego  de  una  coledocotomia  para  explorar  la  vía  biliar  
c.   Luego  de  una  mastectomía  radical  modificada  
d.   Luego  de  una  laparotomía  exploradora  por  peritonitis  generalizada  sin  encontrar  el  foco  infeccioso  
 
320.   Mujer   de   70   años   cardiopata   acude   por   dolor   abdominal,   distensión   abdominal   y   obstinación,   la   rx  
muestra  distensión  del  colon,  con  poco  gas  en  intestino  delgado,  el  ciego  mide  11  cm  no  se  ve  gas  en  el  recto,  
todas  son  cierta,  excepto:  
a.   Tiene  una  obstrucción  en  asa  ciega  
b.   La  causa  más  probable  de  su  problema  es  una  neoplasia  del  sigmoides  
c.   La  perforación  colónica  es  inminente  
d.   Necesita  una  colonoscopia  urgente  
 
321.    Entre  los  cuidados  de  los  drenajes  se  debe  considerar:  
a.   Se  deben  cambiar  periódicamente  los  apósitos  
b.   Se  debe  calcular  cuanto  es  lo  drenado  periódicamente  
c.   Se  debe  reponer  en  los  líquidos  administrados  al  paciente  lo  drenado  
d.   Todas  las  anteriores  
e.   Solo  b  y  c  
 
322.   Un  hombre  de  35  años  acude  por  dolor  perianal  que  no  le  permite  sentarse,  fiebre  y  diarrea  de  4  días  de  
evolución,   al  examen   físico   la   región   perianal   es  normal,   excepto   por   un   endurecimiento   doloroso   del   lado  
derecho  que  hace  muy  difícil  el  examen,  el  manejo  debe  ser:  
a.   Baños  de  asiento  por  hemorroide  izqueirda  
b.   Antibiótico  por  su  gastroenteritis  
c.   Incisión  y  drenaje  del  abceso  perianal  
d.   Laxantes  por  su  fisura  anal  
e.   Ninguna  de  las  anteriores  
 
323.   Paciente  de  80  años  operado  por  enfermedad  diverticular  
perforada  del  colon,  se  le  realiza  hemicolectomía  izquierda  y  colostomia  terminal.  A  los  5  días  presenta  
febrícula,  aumento  de  necesidad  de  líquidos  para  mantener  diuresis,  además  hiperglicemias  y  distensión  
abdominal.  Sería  recomendable  en  este  paciente  efectuar:  
a.   Hemocultivo  
b.   USG  abdominal  
c.   CAT  de  abdomen  
d.   Centelleo  con  leucocitos  marcado  con  tecnecio  99  
   
Pie  Diabético  
 
324.   La  forma  más  efectiva  de  prevenir  la  aparición    de  ulceras  en  los  pies  de  los  pacientes  diabéticos  es:  (C  o  
D)  
a.   Arteriografía  temprana.  
b.   Medición  de  la  oximetría  del  pie  
c.   Detección  temprana  de  pérdida  de  la  sensibilidad.  
d.   Utilización  de  calzados  especiales.  
 
325.   Paciente   de   50   años   de   edad   diabético   tratado   con   hipoglucemiantes   orales,   quien   desde   hace   una  
semana  presenta  fiebre  y  aumento  de  volumen  del  pie  derecho.    Refiere  que  hace  un  mes  presentaba  ulcera  
plantar  en  pie  derecho.    Todas  las  siguientes  son  causas  intrínsecas  de  ulceración  excepto:  (a  o  d)  
a.   Neuropatía.  
b.   Ceguera  
c.   Pobre  conocimiento  de  la  diabetes.  
d.   Ulcera  previa.  
 
Perforación  Intestinal  
326.   La  causa  más  común  de  perforación  intestinal  en  pacientes  con  SIDA  es:  
a.   Linfoma  
b.   Sarcoma  de  Kaposi.  
c.   Gastroenteritis  x  CMV  
d.   Infecciones  por  Mycobacterias.  
 
Enfermedad  De  Hirchprung  
327.   Todo  lo  mencionado  para  la  Enfermedad  de  Hirchsprung  es  cierto,  excepto:  
a-­‐   Se  caracteriza  por  ausencia  de  células  ganglionares.  
b-­‐   El  segmento  más  afectado  en  el  colon  ascendente  e  íleon  terminal.  
c-­‐   El  enema  baritado  y  la  biopsia  rectal  son  fundamentales  para  el  diagnóstico.  
d-­‐   Sospechamos  diagnóstico  en  el  neonato  que  no  expulsa  meconio  en  las  primeras  24  horas  de  vida.  
 
Respuesta  Inflamatoria  Sistémica  
328.   El  síndrome  de  respuesta  inflamatoria  sistémica  se  define  como  dos  o  mas  de  los  siguientes  enunciados  
excepto:  
a.   Temperatura  mayor  de  38º.  
b.   Frecuencia  cardiaca  mayor  de  90  x  
c.   Frecuencia  respiratoria  mayor  de  20  x  
d.   Leucocitos  mayor  de  12  000.  
e.   %  de  bandas  mayor  de  4%.  
 
329.   Es  causa  de  síndrome  de  respuesta  inflamatoria  sistémica:  
f.   Pancreatitis  aguda.  
g.   Quemaduras  de  segundo  grado  30%  de  SCT.  
h.   Infección  por  CMV.  
i.   Herida  por  arma  de  fuego  en  abdomen.  
j.   Todas  las  anteriores.  
 
330.   la  citoquina  que  se  cree  que  es  la  màs  importante  mediadora  del  SRIS  SDMO  es:  
1.   TNF  
2.   IL-­‐1  
3.   IL-­‐6  
4.   IL-­‐8  
5.   Ninguna  de  las  anteriores  
 
331.   El  tratamiento  de  SDMO  para  mejorar  la  cinètica  del  oxigeno  dee  consistir  en:    
a.   mojorar  oxigenacion  
b.   optimizar  la  relacion  DO2/VO2  
c.   Corregir  la  anemia  
d.   Debridamiento  de  tejido  necròtico  
e.   Control  de  hipertermia  
f.   Todas  las  anteriores  
g.   Ninguna  de  las  anteriores  
 
 
Pareo.    
a.   Lynch                  _____        Fiebre,  dolor  con  ictericia    
b.   Ogilvie              _____        Fiebre,  dolor,  ictericia,    obnulación    
                                           e  hipotensión.        
c.   Klatskin            _____        Tumor  de  la  confluencia  de  los  conductos  hepáticos.    
d.   Reynolds        _____    Pseudo  obstrucción  colónica    
e.   Charcot              _____        Cancer  colorectal  hereditario  no  polipósico    
 
332.   Diga  5  características  del  dolor    que  siempre  deben  ser  constatadas  como  parte  del  interrogatorio  ante  un  
cuadro  de  dolor  abdominal  .  
 
a.   Inicio  
b.   Migracion  
c.   Tipo  
d.   Que  lo  mejora  
e.   Que  lo  empeora  
 
 
CICATRIZACIÓN  
 
333.   Todo  lo  siguiente  es  cierto  acerca  de    la  cicatrización  de  las  heridas  quirúrgicas  excepto:  
a-­‐   Luego  de  24  horas  no  es  necesario  cubrir  la  herida.  
b-­‐   En  la  fase  de  inflamación  hay  migración  de  leucocitos,  monocitos  y  macrófagos  hacia  la  herida.  
c-­‐   El  cierre  de  la  herida  por  segunda  intención  se  sutura  luego  de  5  días  cuando  no  hay  signos  de  
infección.  
d-­‐   La  fase  de  contracción  solo  ocurre  en  heridas  grandes.  
 
334.   La  cicatrización  por  segunda  intención  es:  
a.   cuando  el  tejido  es  icidido  y  suturado  
b.   cuando  el  tejido  es  incidido,  debridado  y  suturado.  
c.   cuando  ocurre  en  heridas  abiertas  y  hay  formación  de  tejido  de  granulacion  con  recubrimiento  final  de  la  
herida.  
d.   Cuando  la  herida  cure  abierta  durante  unos  5  dias  y  despues  se  cierra.    
e.   Ninguna  de  las  anteriores    
 
335.   la  respues  de  cicatrizacion    se  inicia:  
a.   al  momento  de  la  lesion  
b.   cuando  la  sangre  es  expuesta  a  la  colágena  
c.   cuando  se  inician  las  curaciones  
d.   a  las  24  horas  después  de  la  lesión  
 
336.   los  principales  componentes  celulares  de  la  cicatrización  son  :  
a.   granulocitos,  linfocitos  y  plaquetas.  
b.   Linfaticos  elementos  trazas  y  enzimas  
c.   Macrófagos  y  fibroblasto  
d.   Todas  las  anteriores.  
e.   Solo  a  y  c  
 
VESICULA  Y  VIAS  BILIARES  /  ICTERICIA  
 
337.      El  objetivo  inicial  de  la  terapia  de  la  colangitis  aguda  es:  
a.   Aliviar  la  ictericia  y  prevenir  el  daño  hepático  
b.   Prevenir  el  desarrollo  de  la  pancreatitis  
c.   Remover  el  cálculo  si  está  presente  
d.   Descomprimir  la  vía  biliar  
 
338.    Se  le  realiza  a  una  paciente  colecistectomía  y  exploración  de  vías  biliares,  2  cálculos  fueron  removidos  y  
se  colocó  un  tubo  en  T  en  el  colédoco,  a  los  9  días  post  operatorios,  se  le  elevan  los  niveles  de  bilirrubina  y  
tiene  dolor  en  hipocondrioderecho  irradiado  a  su  espalda.  Cuál  debe  ser  el  próximo  paso  en  el  manejo  de  esta  
paciente:  
a.   Iniciar  terapia  con  drogas  que  disuelvan  los  cálculos  
b.   Remover  el  tubo  T  
c.   Pinzar  el  tubo  en  T  
d.   Operar  a  la  paciente  
e.   Realizar  colangiografía  por  tubo  en  T  
 
339.   El  dolor  típico  de  la  colelitiasis  es  
a.   En  epigastrio  o  hipocondrio  derecho,  cólico,  menos  de  una  hora  de  duración.  
b.   En  epigastrio  o  hipocondrio  derecho,  constante,  minutos  de  duración.  
c.   En  epigastrio  o  hipocondrio  derecho,  cólico,  minutos  de  duración.  
d.   En  epigastrio  o  hipocondrio  derecho,  constante  más  de  una  hora  de  duración.  
e.   En  epigastrio  y  espalda,  constante,  menos  de  una  hora  de  duración.  
 
340.   Paciente   de   65   años   de   edad   se   presenta   por   presentar   molestias   abdominales   e   ictericia   durante   las  
últimas  tres  semanas.  Tiene  antecedentes  de  tabaquismo  positivo  por  30  años,  e  ingesta  de  alcohol  ocasional,  
y  un  melanoma   ulcerado  de  5,5mm  que  fue  retirado  de  su  espalda   hace   dos  años  y  medio.  La  exploración  
física   mostró   un   paciente   ligeramente   ictérico,   con   los   signos   vitales   normales   y   el   abdomen   ligeramente  
distendido   con   dolor   en   el   cuadrante   superior   derecho   y   hepatomegalia   importante.   El   diagnostico   más  
probable  es:  
a.   Hepatitis  A.    
b.   Metástasis  hepáticas.    
c.   Pancreatitis.    
d.   Hepatoma.  
 
341.   La  evaluación  del  paciente  anterior  debe  incluir:  
a.   Anti  A  IgM.  
b.   Amilasa  y  lipasa  serica.  
c.   Tomografía  axial  computarizada.  
d.   Alfa  feto  proteína.  
 
342.   Para  cada  una  de  las  siguientes  preguntas  escoja  una  de  las  siguientes  opciones:  
a.   Pancreatitis  aguda  
b.   Pancreatitis  crónica  
c.   Ambas  
d.   Ninguna  
 
D    Shock  
C      Dolor  abdominal  
B    Diabetes  y  esteatorrea  
B    Es  necesaria  cirugía  en  más  del  70%  de  los  casos  
C      Pseudoquistes  
 
 
343.   Una  paciente  de  80  años  acude  al  Servicio  de  Urgencias  con  fiebre  de  39°C,  Ictericia,  G.B  20,000  con  85%  
neutrófilos.  
El  manejo  inicial  de  esta  paciente  debe  incluir  todos  excepto.  
 
a.   Antibióticos  
b.   Laparoscopia  
c.   Perfil  de  coagulación  
d.   Reanimación  cardiovascular                                    
e.   Ultrasonido  
 
344.   Una  mujer  de  78  años  consulta  por  fiebre  con  escalofríos,  dolor  abdominal  e  ictericia.    Usted  diagnostica  
colangitis  aguda.    La  PA  es  de  80/70  FC  IIS  x’.    El  tratamiento  a  seguir  es.  
a.   Antibiótico  terapia  por  48  hrs.  
b.   Drenaje  biliar  urgente      
c.   Catéter  de  Swan-­‐Ganz  para  establecer  la  causa  del  shock.  
d.   Tomografía  computarizada  
e.   Tratamiento  en  Unidad  de  Cuidados  Intensivos.    
 
Para  cada  una  de  las  siguientes  preguntas  escoja  una  de  las  siguientes  opciones  
 
a.   ERCP  
b.   Colangio  Resonancia  
c.   TAC  
d.   Ninguno  
 
345.   _B_  Bilirrubinas  totales  8  mg  %  F.  Alcalina  100  (N  <  125)  
a.   US:  vías  biliares  9  mm    (N  <  6  mm),  sin  coledocolitiasis    
 
346.   _A_  Bilirrubinas  totales  8  mg%    F.  Alcalina  200  (N  <  125)    
a.   US:  vías  biliares  9  mm  muestra  coledocolitiasis  
 
347.   _C_  Bilirrubinas  totales  15  mg.%  F.  alcalina  500  (N  <  125)    
a.   US:  vías  biliares  12  mm    masa  en  cabeza  de  páncreas  y  colelitiasis  
 
348.   _D_  Bilirrubinas  totales  10  mg  %  F.  alcalina  100  (N  <  125)      
a.   US:  vías  biliares  5  mm  (  N  <  6  mm)  sin  colelitaisis  
 
349.   Con  respecto  a  la  colelitiasis  todas  son  ciertas  excepto:  
a.   60%  de  pacientes  con  colelitiasis  nunca  tendrán  síntomas  
b.   La  dispepsia  y  mareos  son  indicaciones  de  cirugía  en  pacientes  con  colelitiasis  
c.   2%  de  los  pacientes  con  colelitiasis  tendrán  como  primer  síntoma  una  complicación  
d.   Ninguna  de  las  anteriores.  
 
350.   Con  respecto  a  la  coledocolitiasis  todas  son  ciertas  excepto:  
a.   Su  incidencia  es  el  10-­‐15%  en  pacientes  con  colelitiasis  
b.   La  mayor  parte  de  los  cálculos  son  primarios  del  propio  colédoco  
c.   La  prueba  funcional  hepática  mas  sensible  es  la  fosfatosa  alcalina  
d.   Es  una  causa  frecuente  de  colangitis  
 
351.   Los  pacientes  con  colangitis  pueden  presentar  todos  los  siguientes  excepto:  
a.   Fiebre  con  escalofríos  
b.   Ictericia  
c.   Dolor  en  cuadrante  superior  derecho  
d.   Obstrucción  intestinal  
 
352.   Una   paciente   de   80   años   tiene   un   cuadro   de   colangitis   aguda   de   36   horas   de   evolución   por  
coledocolitiasis.    Un  intento    de  descomprimir    la  vía  biliar  por  CPRE  ha  fallado.    El  radiólogo  intervencionista  
está  fuera  de  la  ciudad.    Su  PA  es  80/40,    Fc  120x'.  Recibe  antibióticos  apropiados   desde  hace  36  horas.  El  
próximo  paso  a  seguir  será.  
a.   Cirugía  
b.   Cambio  de  antibióticos  
c.   Otro  intento  de  CPRE  dentro  de  24  horas  
d.   Esperar  que  regrese  el  radiólogo  intervencionista  en  24  horas  
 
353.   Describa  un  "  Cólico  biliar  "  típico  
a.   Dolor  de  Ubicación  en:      HCD  o    epigastrico  
b.   Constante  o  Espasmódico(  escriba  cual):  CONSTANTE  
c.   Duración  en  horas  mas  de  UNA  y  menos  de  24HORAS  
d.   Las  náuseas  son  frecuentes  si  ó  no:  SI  
e.   Siempre  está  asociado  a  la  ingesta  de  alimentos  si  ó  no:  PUEDE  O  NO  ESTAR  ASOCIADO  A  LA  INGESTA  DE  
ALIMENTOS  
 
354.   La  vena  porta  tiene  su  origen  de  la  confluencia  de:  
a.   La  vena  esplènica  y  la  vena  mesentérica  inferior  
b.   La  vena  mesentérica  superior  y  la  vena  esplénica  
c.   La  vena  cava  inferior  y  la  vena  esplénica  
d.   Ninguna  de  las  anteriores  
 
 
355.   Un   hombre   de   60   años,   diabético   consulta   por   historia   de   mas   o   menos   24   horas   de   evolución  
caracterizada  por  dolor  en   HCD,  fiebre  de  38.5C,  G.B.  19.0  con  95%   N.    El  US  es  compatible  con   colecistitis  
aguda  calculosa.    Todas  las  siguientes  son  ciertas  excepto:  
a.   Requiere  cirugía  en  las  próximas  24  horas  
b.   Las  probabilidades  de  que  la  vesícula  esté  perforado  son  superiores  a  lo  usual  
c.   La  colecistectomía  laparoscópica  está  contraindicada  
d.   Inicialmente  la  colecistitis  es  estéril  
e.   La  etiología  del  cuadro  es  obstructiva  
 
356.   Una  mujer  de  55  años  está  en  su  cuarto  día  de  hospitalización  por  un  cuadro  de  pancretitis  aguda  biliar.    
En  la  TAC  de  hace  24  hrs.  Se  aprecia  necorosis  pancreática  extensa  con  gas  en  el  retroperitoneo.    El  próximo  
paso  a  seguir  sería:  
e.   Cultivo  de  la  necrosis  pancreática  por  punción  percutánea  
f.   Intubación  endotraqueal  y  ventilación  mecánica  
g.   Cirugía  
h.   Continuar  el  manejo  conservador  
 
357.   Todas  las  siguientes  son  ciertas  respecto  a  la  pancreatitis  aguda  biliar  excepto:  
a.   La  colecistectomía  es  parte  fundamental  del  tratamiento  integral  
b.   La  CPRE    siempre  está  indicada  durante  el  episodio  agudo  
c.   Un  cálculo  obstruyendo  la  papila  es  el  evento  desencadenante  
d.   La  TAC  es  el  estudio  de  imagenología  de  elección  
 
358.   Ud.  es  llamada  (o)  a  las  2  am    de  la  sala  de  cirugía  porque  el  paciente  que  se  le  realizó  una  colecistectomia  
abierta    a  las  8  am  del  día  anterior  se  queja  de  mucho  dolor.  
Su  enfoque  de  Dx  y  manejo  inicial  incluye  todas  las  siguientes  excepto:  
a.   Historia  Clínica  y  Exámen  Físico  
b.   R-­‐X  de  Tórax  
c.   Hemograma  completo  
d.   Urinálisis  
 
359.   Varón  de  75  años  de  edad  es  traído  al  servicio  de  urgencias  por  su  familia  para  evaluación  por  ictericia.  El  
paciente  se  queja  de  prurito  de  2  semanas  de  evolución  y  pérdida  de  peso  reciente  de  5  Kg.  A  la  exploración  
física  se  encuentra  ictericia  intensa  y  tiene  una  masa  globosa  en  el  cuadrante  superior  derecho  del  abdomen  
que  se  mueve  con  la  respiración.  El  diagnostico  más  probable  que  tenga  este  paciente  es:  
a.   Cirrosis  hepática.  
b.   Hepatitis  viral.  
c.   Neoplasia  de  la  cabeza  de  páncreas.  
d.   Neoplasia  de  la  cola  del  páncreas  
 
360.   Todas  las  siguientes  son  ciertas  respecto  a  la  pancreatitis  aguda  biliar  excepto:  
a.   La  colecistectomia  es  parte  fundamental  del  tratamiento  integral  
b.   La  CPRE    siempre  está  indicada  durante  el  episodio  agudo  
c.   Un  cálculo  obstruyendo  la  papila  es  el  evento  desencadenante  
d.   La  TAC  es  el  estudio  de  imagenología  de  elección  
 
361.   Son  funciones  de  la  colecistoquinina  excepto:  
a.   Retrasa  el  vaciamiento  gástrico.  
b.   Estimula  la  contracción  de  la  vesícula  biliar.  
c.   Relaja  el  esfínter  anal  promoviendo  la  defecación..  
d.   Estimula  la  secreción  pancreática.  
e.   Juega  un  papel  importante  en  la  regulación  de  la  saciedad  
 
362.   Un  hombre  de  59  años  consulta  por  ictericia  y  prurito.  El  US  muestra  las  vías  biliares  en  9  mm  (Normal  <  
de  6  mm)  y  colelitiasis.    Todos  los  siguientes  serían  adecuados  en  el  manejo  inicial  de  este  paciente  excepto.  
 
a.   Colangio  resonancia  
b.   ERCP  
c.   Laparotomía  
d.   Perfil  de  coagulación  
e.   TAC  de  Abdomen.  
   
363.   En  el  manejo  inicial  de  la  pancreatitis  severa  están  indicados  todos,    
   excepto:  
a.  Antibióticos  de  ampli  espectro  
b.  TAC  en  las  primeras  48  horas  
c.  Atención  en  la  unidad  de  cuidados  intensivos  
d.  Laparotomía  por  necrosis  peri-­‐pancreáticas  
e.  Monitoreo  hemodinámica  invasivo  
 
Piel  
 
364.   Indicaciones  para  la  excisión  de  un  nevo  pigmentado  incluyen:  
a.  Cambio  en  el  color,  tamaño  y  forma  
b.  Desarrollo  de  dolor  de  la  lesión  
c.  La  aparición  de  lesiones  satélites  en  el  aérea  de  un  nevo    previamente  existente  
d.  Adenopatía  regional  inexplicable  
e.  Todas  las  anteriores  
 
365.   Todos  los  siguientes  enunciados  con  respecto  a  las  quemaduras  electricas  son  ciertas  excepto:    
a.  Ellas  son  usualmente  mas  profundas  y  mas  severas  que  lo  que  aparentan  
b.  Los  vasos  sanguíneos  los  nervios  y  los  huesos  tienen  una  resistencia  baja  a  la  corriente  eléctrica  
c.  La  necrosis  muscular  puede  estar  sobreestimada  
d.  Todos  los  pacientes  con  lesiones  eléctricas  necesitan  monitoreo  continuo  
e.  El  manitol  puede  ser  utilizado  para  mantener  una  diuresis  adecuada  para  prevenir  el  daño  renal  secundaria  a  la  
mioglobinuria      
 
Pareo  
a.   Bupivacaína  
b.   Lidocaína  
c.   Ambas  
d.   Ninguna  
 B      Dosis  máxima  de  infiltración  300  mg  
 A    Dosis  máxima  de  infiltración  175  mg  
   B    Efecto  dura  hasta  dos  horas  
 A    Efecto  dura  hasta  ocho  horas  
 B    La  dosis  máxima  de  infiltración  se  puede  incrementar  si  se  usa  adrenalina  
 
a.   Secretina  
b.   Colecistoquinina  
c.   Ambas  
d.   Ninguna  
___Secreción  de  gránulos  de  Zimógeno  
___Secretada  en  el  duodeno  
___Secreción  de  agua  y  electrolitos  
___Secreción  disminuida  por  la  administración  de  inhibidores  de  la  bomba  de  protones  
___Secreción  disminuida  por  resección  pancreático  duodenal  
 
a.   Carcinoma  lobular  in  situ  
b.   Carcinoma  ductal  in  situ  
c.   Ambos  
d.   Ninguno  
 B          Rara  vez  es  palpable  
 A        Nunca  es  palpable  
 A        Es  un  factor  de  riesgo  
 C        Puede  avanzar  a  carcinoma  invasor  
 B        Lumpectomía    sin  vaciamiento  axilar  
 A        Observación  
 A        Mastectomía  profiláctica  
D            Quimioterapia  pre-­‐operatoria  
 B            quimioterapia  post-­‐operatoria  
 B            Microcalcificaciones  
 

Para  Futuras  Generaciones  


24  de  Marzo  de  2008  
ABDOMEN  AGUDO   años   pero   no   sabe   qué   se   le   practicó.   Enumere   las   tres   posibilidades  
intestinales  que  puedan  obligar  a  una  intervención  quirúrgica:  
1.   La  causa  más  común  del  abdomen  agudo  hemorrágico  es:   a.   APENDICITIS  AGUDA  
a.   Trauma  abdominal   b.   DIVERTICULITIS  
b.   Hemorragia  digestiva  alta   c.   DIVERTICULO  DE  MECKEL  
c.   Embarazo  ectópico  roto   8.   __C__   el   origen   principal   de   los   gases   en   la   obstrucción   intestinal  
d.   Aneurisma  intraabdominal  roto   mecánica  es  el  aire  tragado  
2.   En   la   úlcera   péptica   perforada,   el   abdomen   en   tabla   aparece  
inicialmente  por:   ANESTESIA  
a.   Irritación  peritoneal  química  
b.   Abscesos  peritoneales   1.   Para   anestesia   local   por   infiltración   con   xilocaina,   la   dosis   máxima  
recomendada  es:  
c.   Irritación  peritoneal  bacteriana  
d.   DHE   a.   100  mgs  
3.   La   mayor   parte   del   gas   contenido   en   las   asas   intestinales   obstruidas   b.   200  mgs  
c.   300  mgs  
proviene  de:  
d.   400  mgs  
a.   Aire  que  traga  el  paciente  
b.   Fermentación  digestiva   e.   500  mgs  
2.   Los   factores  que   afectan  la  concentración   espiratoria   de   un   anestésico  
c.   Putrefacción  bacteriana  
d.   Todas   inhalado  son:  
4.   La   causa   más   común   de   abdomen   agudo   hemorrágico   es:   EMBARAZO   a.    volumen   del   circuito   respiratorio,   velocidad   del   flujo   de   gas  
ECTÓPICO.   fresco  
5.   Enumere  cuatro  imágenes  en  la  radiografía  simple  de  abdomen  agudo:   3.   Anestésico  más  comúnmente  usado:  
a.   AIRE  SUBDIAFRAGMÁTICO   a.    Sevoflurano.  
4.   La  potencia  de  los  anestésicos  inhalados  es  medida  por:  
b.   ASAS  INTESTINALES  DISTENDIDAS  
c.   NIVELES  HIDROAEREOS   a.      concentración  alveolar  mínima  
d.   FALTA  DE  AIRE  EN  EL  COLON   5.   Paciente   con   muerte   cerebral     para   transplante   de   órganos   es  
6.   Enumere  cuatro  signos  especiales  a  la  exploración  en  el  abdomen  agudo   clasificado  como:  
a.   SIGNO  DEL  PSOAS   a.   ASA  VI.  
b.   SIGNO  DEL  OBTURADOR   6.   Durante  la  anestesia  la  diseminación  del  anéstesico  depende  de:  
a.   Fuerza  con  que  se  inyectó  
c.   SIGNO  DE  McBURNEY  
d.   SIGNO  DE  ROVSING   b.   Gravedad    
7.   Una  señora  de  35  años  presenta  fiebre,  leucocitosis,  malestar  y  nauseas.   c.   Volumen  inyectado  
Tiene   una   cicatriz   antigua   sobre   fosa   iliaca   derecha   donde   presenta   d.   Lugar  de  la  inyección  
dolor,  defensa  y  rebote.  La  operación  se  llevó  a  cabo  cuando  tenía  16   e.   Concentración  del  anestésico    
7.   La  hipotensión  en  la  anestesia  raquídea  se  debe  a:  
a.   Bloqueo  neuromuscular   c.   CERRADO  
b.   Bloqueo  simpático     15.  Tipos  de  relajantes  musculares  
c.   Depresión  miocárdica   a.   DESPOLARIZANTES  
d.   hipovolemia   b.   NO  DESPOLARIZANTES  
8.   Cuál  de  las  siguientes  afirmaciones  es  cierta  en  relación  a  la  colocación   16.  Señale  los  receptores  opioides  
de  anestesia  local  para  una  cirugía  menor:   a.   Mu  
a.   Se  utiliza  marcaína  al  1-­‐2%   b.   Delta  
b.   Es   útil   el   uso   de   un   vasoconstrictor   (adrenalina   1:100   000)   en   c.   Kappa  
áreas  como  dedos  o  nariz   d.   Sigma  
c.   Se   administra   en   forma   subcutánea   desde   el   interior   hasta   el   17.  Escala  post-­‐operatorio  de  Aldrete,  indique  los  cinco  aspectos  que  toma  
exterior  de  la  herida   en  cuenta  
d.   El   anestésico  local  se  puede  utilizar   a   una   dosis  máxima   de  20   a.   RESPIRACIÓN  
mg/kg  de  peso   b.   CIRCULACIÓN  
e.   Es   muy   importante   interrogar   al   paciente   sobre   alergia   a   los   c.   ESTADO  DE  ALERTA  
anestésicos   d.   MOVILIDAD  
9.   La   hipotensión   en   la   anestesia   raquídea   se   debe   a:   BLOQUEO   e.   COLOR  
SIMPÁTICO  
10.  Señale  los  cuatro  componentes  de  la  anestesia  general   CANCER  DE  PIEL  
a.   INCONSCIENCIA  REVERSIBLE  
1.   Un   tumor   maligno   que   tiene   tendencia   a   presentar   regresión  
b.   AMNESIA  
espontánea  es:  
c.   ANALGESIA  
a.   Melanoma  maligno.  
d.   RELAJACIÓN  MUSCULAR  
b.   Carcinoma  broncogénico.  
11.  Tipos  de  anestésicos  locales  
c.   Hipernefroma.  
a.   AMIDAS    
d.   Tumor  de  Wilms.  
b.   ESTERES  
e.   CA  de  vejiga.  
12.  Señale  la  dosis  máxima  en  miligramos  (mg)  de  un  anestésico  local:  400  
2.   Paciente   de   40   años   que   practica   la   espeleología   como   profesión,  
mg    
presenta  una  lesión  color   café  con  leche  en  la  región  plantar  derecha.  
13.  La   absorción,   transporte   y   distribución   de   un   agente   anestésico  
Informa  que  está  aumentando  de  tamaño,  con  prurito  y  cambiando  su  
depende  de  tres  factores  
color  a  un  tono  más  oscuro.  Su  diagnóstico  sería:  
a.   SOLUBILIDAD  DEL  ANESTESICO  
a.   Tinea  pedis.  
b.   FLUJO  SANGUINEO  PULMONAR  
b.   CA  basocelular.  
c.   PRESIÓN  PARCIAL  DEL  ANESTÉSICO  
c.   CA  escamoso.  
14.  Indique  tres  tipos  de  sistema  de  anestesia  
d.   Nevus  de  superficie.  
a.   ABIERTO  
e.   Melanoma  maligno.  
b.   SEMIABIERTO  
3.   El  carcinoma  escamoso  es  más  frecuente  su  aparición  en:   1.   La   ayuda   diagnóstica   más   importante   para   el   diagnóstico   del   ano  
a.   Cualquier  parte  del  cuerpo   imperforado  es:  
4.   Cuándo  es  más  frecuente  el  carcinoma  basocelular:   a.   Serie  gastrointestinal.  
a.   Tercera  edad   b.   Radiografía  de  tórax.  
5.   En  qué  región  del  cuerpo  es  más  frecuente  el  carcinoma  basocelular:   c.   Serie  de  abdomen  agudo.  
a.   Cabeza  y  cuello   d.   Radiografía  de  abdomen  con  el  niño  en  posición  invertida  
6.   Cuál   de  los   cánceres   está   más   relacionado   con   el   estado   inmunológico   e.   Sonografía.  
del  paciente:   2.   Un  paciente  de  2  años  de  edad  procedente  de  San  miguelito  con  cuadro  
a.   Miosarcoma   de  obstrucción  intestinal,  la  causa  más  frecuente  es  
b.   Linfomas   a.   Bridas  
c.   Melanomas   b.   Hernia  inguinal  
7.   El  melanoma  maligno  en  mujeres  es  más  frecuente  en:   c.   Parásitos  intestinales  
a.   Extremidades  inferiores   d.   Intususepción  intestinal  
b.   Melanoma  visceral   3.   El  lugar  más  común  de  presentar  un  pólipo  en  un  niño  es:  
c.   Cabeza  y  cuello   a.   Ciego.  
8.   Melanoma  subungueal  en  la  clasificación  de  Clark  corresponde   b.   Colon  transverso.  
a.   Clark  II   c.   Ileon.  
b.   Clark  III   d.   Rectosigmoide.  
c.   Clark  V   e.   Yeyuno.  
9.   Una   señora   de   40   años   se   queja   de   ulceración   reciente   de   un   nevus   4.   La  causa  más  común  de  sangramiento  rectal  en  un  niño  es:  
desde   la   niñez   y   está   en   buena   salud.   El   tratamiento   inicial   más   a.   Colitis  ulcerativa.  
apropiado  sería:   b.   Fisura  anal.  
a.   Biopsia   c.   Intususcepción.  
b.   Fluorouracilo  local   d.   Divertículo  de  Meckel.  
c.   Radioterapia   e.   Pólipo  rectal.  
d.   Observación  con  corticoide  tópico   5.   Neonato   de   5   días,   vómitos   biliosos   y   marcada   distención   abdominal,  
10.  __C__el   carcinoma   basecelular   y   escamoso   son   más   frecuentes   en   la   sospecha  de  diagnóstico  excepto:  
piel  expuesta  al  sol   a.   Malrotación  intestinal  y  atresia  intestinal  ¿??  
11.  Entre  los  tumores  cancerosos,  el  más  común  es:  LA  PIEL   6.   Paciente   que   se   sospecha   de   Agangliosidosis   intestinal   congénita,   el  
12.  Ante  lo  signos  de  malignidad  de  un  nevus,  cuál  sería  el  procedimiento  a   mejor  método  diagnóstico  es:  
seguir:  BIOPSIA  EXCISIONAL   a.   Biopsia  rectal  
13.  El  melanoma  maligno  tiene  su  origen  en:  MELANOCITOS   7.   Paciente   escolar   masculino   que   presenta   súbitamente   dolor   a   nivel  
escrotal  con  cambio  de  coloración  violáceo,  lo  más  probable  es  que  se  
CIRUGÍA  PEDIÁTRICA   trate  de:  
a.   Torsión  testicular  
8.   Paciente   lactante   masculino   que   presenta   evacuaciones   con   sangre,   24.  La   edad   ideal   para   hacer   orquideopexia   en   un   testículo   que   no  
vómitos  biliosos,  su  diagnóstico  más  probable  sería:   descendió  es:  1  A  2  AÑOS  DE  EDAD  
a.   Invaginación  intestinal   25.  Cuáles  son  las  causas  de  sangrado  digestivo  bajo  más  comunes  en  niños  
9.   Método  estándar  terapéutico  en  el  manejo  de  invaginación  intestinal:   a.   INTUSUSCEPCIÓN    
a.   Reducción  hidrostática  con  control  tomográfico   b.   DIVERTICULO  DE  MECKEL  
10.  Si   el   paciente   presenta   Dextrocardia   y   dificultad   respiratoria,   el  
diagnóstico  más  probable  es:  HERNIA  DIAFRAGMÁTICA   CIRUGÍA  AMBULATORIA  
11.  Un  paciente  de  2  años  de  edad  procedente  de  San  miguelito  con  cuadro  
1.   La  biopsia  incisional  
de   obstrucción   intestinal,   la   causa   más   frecuente   es:   INTUSUSEPCIÓN  
a.   Se  aplica  a  lesiones  de  “grandes  dimensiones”,  dependiendo  de  
INTESTINAL  
la  ubicación  
12.  Neumonía  química  y  bacteriana  en  el  recién  nacido,  el  diagnóstico  más  
b.   Se  usa  una  incisión  en  rombo  que  debe  comprender  parte  de  la  
probable  es:  ATRESIA  ESOFÁGICA  Y  FÍSTULA  TRAQUEOESOFÁGICA  
lesión  y  parte  del  tejido  sano  vecino  
13.  Si  el  lactante  presenta      dolor  abdominal  intenso  y  masa  abdominal,  el  
c.   Debe  realizarse  adecuada  hemostasia  y  sutura  sin  tensión  
diagnóstico  más  probable  es:    MALROTACIÓN  INTESTINAL    
d.   Enviar  la  pieza  en  formol  10%  o  en  fresco  y  eventuales  cultivos  
14.  Si  el  lactante  presenta    vómitos  intestinales  que  se  inician  en  la  segunda  
e.   Son  cierta  todas  las  anteriores  
semana  de  vida,  el  diagnóstico  más  probable  es:  HIPERTROFIA  PILORICA  
2.   Son  algunas  de  las  ventajas  de  la  cirugía  ambulatoria,  excepto  
15.  Si   el   lactante   presenta   constipación   crónica   y   ampolla   rectal   vacía   el  
a.   Permite  aumentar  la  actividad  asistencial,  sin  estar  limitada  por  
diagnóstico  más  probable  es:  MEGACOLON  CONGÉNITO  
la  disponibilidad  de  camas.  
16.  Recién  nacida  con  un  solo  orificio  perineal,  el  diagnóstico  más  probable  
b.   Ahorro  de  estancia  y  camas  
es:  CLOACA  
c.   Los  recursos  ahorrados  se  pueden  destinar  a  otros  pacientes  
17.  Recién  nacido,  síndrome  de  Down  y  vómitos  verdes,  el  diagnóstico  más  
d.   Contribuye  a  disminuir  las  estancias  en  los  pacientes  ingresados  
probable  es:  OBSTRUCCIÓN  INTESTINAL  
e.   Al   incrementar   en   cantidad   disminuye   la   eficiencia   de   los  
18.  Recién   nacido   que   no   pasa   meconio   en   24   horas,   el   diagnóstico   más  
hospitales  
probable  es:  MEGACOLON  CONGENITO  
19.  Medida  inicial  en  paciente  con  invaginación  intestinal:  COLOCAR  SONDA  
ESTOMAGO,  INTESTINOS,  APENDICE,  COLON,  ANORECTAL  
NASOGASTRICA  
20.  Método   inicial   con   sospecha   de   invaginación   intestinal:   COLON   POR   1.   En  una  paciente  con  apendicitis  retrocecal  no  perforada:  
ENEMA  CON  BARIO   NO   Invariablemente   el   dolor   cambia   de  la   región   periumbilical  
21.  Vómitos   biliares   a   los   días   de   vida   después   de   ingesta   oral   normal:   al  CID.  
MALROTACIÓN  INTESTINAL   NO   Habitualmente  hay  hiperestesia  notable  en  el  CID.  
SÍ   Puede  presentarse  hematuria  microscópica.  
22.  Método  estándar  terapéutico  en  el  manejo  de  la  invaginación  intestinal:  
NO   La   cuenta   de   leucocitos   generalmente   está   por   debajo   de  
REDUCCIÓN  HIDROSTÁTICA  CON  CONTROL  TOMOGRÁFICO   10,000/mm3.  
23.  La  ayuda  diagnóstica  más  importante  en  el  manejo  del  ano  imperforado   NO   La  emesis  es  poco  habitual.  
es:  RADIOGRAFIA  DE  ABDOMEN  CON  EL  NIÑO  EN  POSICIÓN  INVERTIDA   2.   En   la   actualidad,   los   enfoques   propuestos   para   el   tratamiento   de  
pacientes  con  síndrome  de  Zollinger-­‐Ellison,  incluyen:  
SÍ   Administrar  Cimetidina.   NO   La   mortalidad   fetal   global   relacionada   con   apendicitis  
SÍ   Administrar  Omeprazol.   materna   es   de   10%   aproximadamente,   pero   puede  
SÍ   Vagotomía  para  células  parietales.   reducirse  empleando  progesterona.  
NO   Gastrectomía  parcial.   6.   Relacionado  con  el  fibroadenoma,  son  afirmaciones:  
SÍ   Gastrectomía  total.   SÍ   Puede   observarse   lesión   única   de   2   centímetros,   que  
3.   Son  afirmaciones  verdaderas  relacionadas  con  apendicitis  aguda:   clínicamente  corresponde  a  fibroadenoma  en  las  mamas  de  
NO     La   infección   es   el   factor   dominante   en   el   desarrollo   de   una  mujer  de  20  años  de  edad.  
apendicitis  aguda.   NO   Cuando   se   observa   una   lesión   que   clínicamente  
NO   La  pared  del  apéndice  normal  es  estéril.   corresponde  a  fibroadenoma  en  las  mamas  de  una  mujer  de  
SÍ   El   líquido   periapendicular   en   una   apendicitis   aguda   no   23  años  de  edad,  se  debe  practicar  mamografía.  
perforada,  generalmente  es  estéril.   SÍ   A   veces   no   es   posible   distinguir   entre   un   fibroadenoma  
SÍ   Se  pueden  cultivar  múltiples  microorganismos  en  el  líquido   juvenil  voluminoso  de  un  tumor  filodes.  
que  rodea  el  apéndice  perforado.   NO   La  superficie  de  corte  del  fibroadenoma  típico  se  retrae  a  lo  
SÍ   Citomegalovirus   puede   relacionarse   con   apendicitis   en   largo  de  la  línea  de  corte.  
pacientes  con  SIDA.   SÍ   Un   fibroadenoma   quizás   sea   doloroso   e   hiperestésico  
4.   En  la  apendicitis  aguda  en  niños  en  edad  pre-­‐escolar:   durante  el  embarazo.  
SÍ   Es  difícil  establecer  el  diagnóstico.   7.   Entre   los   trastornos   relacionados   con   mayor   incidencia   de   cáncer  
NO   Se  puede  desencadenar  un  ataque  cuando  se  administra  un   colorrectal  se  incluyen:  
catártico  fuerte  a  un  niño  estreñido.   SÍ   Poliposis  familiar.  
NO   Un   período   de   observación   cuidadoso   debe   preceder   a   la   SÍ   Síndrome  de  Gardner.  
decisión  de  operar  si  el  diagnóstico  es  dudoso.   NO   Enterocolitis  pseudomembranosa.  
SÍ   Una   complicación   frecuente   es   la   rotura   temprana   del   NO   Pólipos  juveniles.  
apéndice.   NO   Disentería  amebiana  crónica  
NO   Las  paredes  del  epiplón  mayor  se  encuentran  bastante  lejos   8.   Cuál   de   las   siguientes   características   de   úlcera   gástrica   constituyen  
del  área  y  por  lo  tanto,  la  peritonitis  generalizada  es  rara.   indicación  quirúrgica:  
5.   Cuáles   de   las   siguientes   afirmaciones   relacionadas   con   apendicitis   SÍ   Cuadro  clínico  con  hemorragia.  
durante  el  embarazo  son  correctas:   SÍ   Localizadas  en  la  curvatura  mayor.  
NO   La   incidencia   de   apendicitis   es   más   elevada   en   mujeres   SÍ   Acompañada  de  úlcera  duodenal.  
embarazadas   que   en   la   población   de   mujeres   no   grávidas   NO   Recurrencia  después  de  cicatrización  inicial.  
del  mismo  grupo  de  edad.   NO   Menos   de   50%   de   cicatrización   con   3   semanas   de  
NO   Se   puede   lograr   menor   mortalidad   fetal   tratando   la   tratamiento  médico.  
apendicitis   aguda   sin   operación,  con  líquidos  y  antibióticos   9.   Entre  los  síntomas  típicos  de  hemorroides  internas  se  incluyen:  
por  vía  intravenosa.   SÍ   Sangrado  sin  deposición.  
NO   En   la   mujer   embarazada,   la   cifra   elevada   de   leucocitos   es   NO   Sangre  mezclada  con  las  heces.  
útil  para  establecer  el  diagnóstico  de  apendicitis.   SÍ   Producción  de  moco.  
Sí   En   la   mujer   embarazada   se   puede   usar   laparoscopia   para   NO   Dolor.  
establecer  el   diagnóstico  de  apendicitis  en  casos  atípicos  y   SÍ     Prolapso.  
evitar  la  laparotomía.   10.  Cuáles   de   las   siguientes   afirmaciones   relacionadas   con   divertículo   de  
Meckel  son  correctas:  
SÍ   Es   un   verdadero   divertículo,   remanente   del   conducto   c.   Gastroscopia  
onfalomesentérico.   d.   Trago  de  Bario  
SÍ   Habitualmente   muestra   entrada   amplia   y   se   sitúa   sobre   el   15.  _A_  El  síndrome  de  Zollinger-­‐Ellison  incluye  
borde  antimesentérico    del  intestino  delgado.   a.   Hipersecreción  gástrica  
SÍ   En   general,   el   sangrado   se   observa   en   mucosa   gástrica   b.   Úlcera  péptica  intratable  
heterotópica.   c.   Tumor  de  las  células  no  beta  del  páncreas  
NO   Una  complicación  común  es  la  úlcera  del  divertículo.   16.  _B_  Las  causas  más  comunes  de  sangrado  digestivo  bajo  son:  
NO   Si   durante   una   operación   abdominal   se   observa   a.   Cáncer  colorrectal,  fisura  anal  y  hemorroides  
incidentalmente  un  divertículo  siempre  debe  extirparse.   b.   Hemorroides,  fístulas  y  fisuras  
11.  Entre   las   medidas   terapéuticas   eficaces   para   prevenir   o   controlar   la   c.   Diverticulosis,  cáncer  colorrectal  y  angiodisplasias  del  colon  
hemorragia   en   el   tracto   digestivo   alto   debido   a   gastroenteropatía   por   d.   Ninguna  de  las  anteriores  
estress:   17.  _D_   centelleo   con   glóbulos   rojos   marcados   es   positivo   cuando   el  
SI   Mantener  una  nutrición  adecuada   sangrado  es  de:  
NO   Alcalinización  gástrica   a.   5  MM/Minuto  
NO   Lavartelenol  intragástrico   b.   1  MM/Minuto  
SI   Reanimación  eficaz  durante  el  choque   c.   0.5  MM/Minuto  
SI   Vasopresina  intra-­‐arterial   d.   0.1  MM/Minuto  
  18.  _A_  El  tratamiento  inicial  de  la  diverticulitis  aguda  consiste  en:    
Seleccione  su  respuesta  en  la  siguiente  forma:   a.   Tubo  de  Levin  
  b.   Fluidos  IV  
A    si  1,  2  y  3  son  correctas   c.   Antibióticos  sistémicos  
B    si  1  y  3  son  correctas   d.   Sigmoidectomía    
C    si  2  y  4  son  correctas    
D    si  solo  4  es  correcta   19.  Usted   es   el   interno   de   un   hospital   y   es   llamado   para   evaluar   a   un  
E    si  todas  son  correctas   paciente  varón  de  23  años  con  historia  de  un  dolor  abdominal  que  en  la  
  últimas  12  horas  ha  aumentado  en  forma  progresiva  y  localizado  en  fosa  
12.  _A_  Durante  la  perforación  de  úlcera  péptica  ocurre:   iliaca   derecha.   Originalmente,   el   dolor   fue   periumbilical.   No   tiene  
a.   Inicio  súbito   hambre  y  comió  la  última  vez  hace  12  horas.  Presentó  nauseas  pero  no  
b.   Dolor  abdominal   ha  vomitado.  Anteriormente  ha  tenido  buena  salud.  Su  temperatura  es  
c.   Abdomen  en  tabla   de   37.8°C.   Su   presión   arterial   es   de   120/80,   pulso   regular   de   86.   Al  
d.   Si  sólo  4  es  correcta   examen,   el   corazón   y   los   pulmones   están   bien.   Al   examen   abdominal  
e.   Si  todas  son  correctas   había   dolor   a   la   palpación   en   fosa   iliaca   derecha   con   defensa.   No   se  
13.  _A_   Las   indicaciones   para   el   tratamiento   quirúrgico   de   la   úlcera   palpan  masas.  Los  ruidos  hidroaéreos  estaban  hipoactivos.  Se  quejó  de  
duodenal  son:   dolor  al  tacto  rectal  y  las  heces  no  mostraron  sangre  oculta.  La  cuenta  
a.   Perforación   de  blancos  era  de  13.200  con  70%  de  neutrófilos.  El  hematocrito  era  de  
b.   Hemorragia  masiva   43%.  El  diagnóstico  diferencial  debe  ser  entre:  
c.   Obstrucción   a.   APENDICITIS  AGUDA  
14.  _A_  El  manejo  inmediato  de  la  úlcera  duodenal  sangrante  consiste  en:   b.   DIVERTICULITIS  DEL  CIEGO  
a.   Trasfusión  de  sangre   c.   HERNIA  INGUINAL  
b.   Tubo  de  Levin  
20.  De  estos  exámenes,  cuáles  son  los  tres  que  usted  ordenaría  en  orden  de   A.   Rectosigmoidoscopía  flexible  
importancia.   B.   Colocar  tubo  nasogástrico  
A.   Centelleo  hepático   C.   Hacer  CAT  abdominal  
B.   Ultrasonografía  abdominal   D.   Colocar  Sonda  vesical  
C.   Rayos  X  de  tórax   E.   Solicitar  enema  de  Bario  
D.   Enema  de  Bario   F.   Administrar  lactato  Ringer  IV  
E.   Gases  arteriales   G.   Dar  corticoesteroide  
F.   Centelleo  con  Galio   H.   Antibióticos  por  boca  
G.   Amilasemia   I.   Radiografía  de  abdomen  
H.   Pielograma  endovenoso   J.   Enema  evacuante  
I.   Radiografía  de  abdomen  simple   K.   Radiografía  de  tórax  
J.   Serie  gastrointestinal   __B__  
K.   Urinalisis   __F__  
L.   CAT  Abdominal   __D__  
M.   Lavado  peritoneal   __I__  
__K__   __C__  
__I__   24.  Se   sabe   que   todos   los   siguientes   padecimientos   tienen   un   patrón  
__B__   familiar  relacionado  con  predisposición  a  cáncer,  excepto:  
21.  Qué  indicaría  usted  en  este  momento?   a.   Poliposis  de  colon.  
A.   Llamar  al  endoscopista  para  gastroesofagoscopia   b.   Cáncer  mamario.  
B.   Admitir  al  paciente  al  hospital  y  colocar  tubo  nasogástrico   c.   Síndrome  de  Peutz-­‐Jeghers  =  poliposis  intestinal  generalizada  y  
C.   Llamar  al  cirujano  de  turno  para  celiotomía  inmediata   máculas  pigmentadas.  
D.   Administrar  antibióticos  de  amplio  espectro  vía  oral   d.   Síndrome   de   Gardner   =   poliposis   familiar   autosómica  
E.   Observación  por  24  horas  en  casa   dominante.  
F.   Admitir  al  paciente  en  el  hospital  para  observación   e.   Retinoblastoma.  
__C__   25.  En   cuál   de   las   siguientes   circunstancias   puede   ser   útil   determinar  
  antígeno  carcinoembrionario  (ACE):  
22.  El  paciente  recibió  su  tratamiento  adecuado  y  se  le  dio  de  alta.  Dos  años   a.   Prueba  para  detección  masiva  de  cáncer  colorrectal.  
después,   acude   donde   usted   por   haberlo   tratado   muy   bien   y   haberle   b.   Como  indicador  general  de  la  extensión  de  cáncer  colorrectal.  
dado  muy  buena  atención  con  dolor  abdominal  que  no  puede  localizar   c.   Estudio   seriado   para   indicar   recurrencia   post-­‐operatoria   de  
con   precisión  y  se  siente  distendido.  Pasó  gases  y  heces  líquidas  en  la   cáncer  colorrectal.  
mañana.   Al   examen,   el   corazón   y   pulmones   normales.   Abdomen   d.   Predicción   de   la   respuesta   de   cáncer   colorrectal   a  
distendido.  Al  examen  rectal  se  queja  de  dolor.  No  hay  fiebre.  Los  ruidos   quimioterapia.  
hidroaéreos  están  hiperactivos.  La  cuenta  de  blancos  y  electrolitos  están   e.   Indicación  del  éxito  de  la  resección  de  tumor  colorrectal.  
dentro  de  los  límites  normales.  Cuáles  son  los  diagnósticos  diferenciales   26.  La  causa  más  común  de  hemorragia  masiva  en  tubo  digestivo  bajo  es:  
que  usted  haría:   a.   Carcinoma.  
a.   OBSTRUCCIÓN  INTESTINAL  POSTOPERATORIA   b.   Diverticulosis.  
b.   HERNIA  INTESTINAL   c.   Diverticulitis.  
23.  De   estos   procedimientos   o   estudios,   cuáles   recomendaría   hacer   en   d.   Pólipos.  
orden  de  importancia:   e.   Colitis  ulcerosa.  
27.  Un  hombre  de  25  años  de  edad  acude  a  la  sala  de  urgencias  por  haber   e.   Anemia  hemolítica  adquirida.  
tragado   6   horas   antes   2   alfileres   de   seguridad   abiertos.   La   radiografía   32.  El  carcinoma  del  estómago  ocurre  más  frecuentemente  en:  
muestra  los  alfileres  abiertos  en  intestino  delgado.  En  este  momento  el   a.   Fondo.  
tratamiento  más  apropiado  sería:   b.   Cardias.  
a.   Administración  IV  de  antibióticos  de  amplio  espectro.   c.   Cuerpo.  
b.   Un  ciclo  de  10  días  con  Metronidazol  oral.   d.   Antro.  
c.   Administración  de  250  mL  de   citrato  de  magnesio  para  inducir   e.   Unión  gastroesofágica.  
catarsis  e  incrementar  rapidez  del  tránsito  intestinal.   33.  Pólipos   juveniles   del   colon   y   recto   pueden   producir   todo   lo   siguiente  
d.   Seguimiento  con  radiografías  en  serie  y  examen  abdominal.   EXCEPTO:  
e.   Operación  inmediata.   a.   Raro  por  debajo  de  los  45  años.  
28.  Una   mujer   de   65   años   de   edad   presenta   melena  masiva   e   hipotensión   b.   Las  lesiones  son  múltiples  generalmente.  
acompañante,   que   se   corrigen   con   transfusión   sanguínea.   Luego   de   c.   Producen  anomalías  electrolíticas.  
endoscopia   gastrointestinal   alta   negativa,   el   siguiente   procedimiento   d.   Al  examen  histológico  se  observa  malignidad  con  frecuencia.  
que  debe  ejecutarse  para  definir  el  origen  del  sangrado  intestinal  debe   34.  La  peritonitis  química  ocurre  debido  a  todos  los  siguientes,  EXCEPTO:  
ser:   a.   Úlcera  péptica  perforada  
a.   Colonoscopia.   b.   Perforación  de  la  vesícula  
b.   Sigmoidoscopia.   c.   Hemoperitoneo  
c.   Arteriografía  selectiva.   d.   Lesión  de  cisterna  quilosa  
d.   Imagen  con  radionúclidos.   35.  Todas   las   siguientes   manifestaciones   extra   intestinales   de   la   colitis  
e.   Enema  de  bario  en  tubo  digestivo  bajo.   ulcerativa  remiten  con  la  resección  del  colon,  Excepto:  
29.  La  duplicación  del  tubo  digestivo  generalmente  ocurre  en:   a.   Eritema  nodoso  
a.   Esófago.   b.   Uveítis  
b.   Estómago.   c.   Artritis  
c.   Duodeno.   d.   Colangitis  esclerosante  
d.   Ileon.   36.  En  la  enfermedad  de  Crohn  todas  son  correctas,  Excepto:  
e.   Colon.   a.   La  relación  hombre/mujer  es  de  1:1  
30.  Todo   lo   siguiente   es   cierto   con   respecto   a   los   pólipos   del   estómago,   b.   Afecta  sólo  la  mucosa  del  intestino  
EXCEPTO:   c.   Puede  afectar  desde  la  boca  hasta  el  ano  
a.   Ocurren  en  una  mucosa  atrófica.   d.   Se  asocia  a  fistulas  en  el  ano  
b.   Aparecen  después  de  la  adolescencia.   37.  En  la  colitis  ulcerosa,  son  todas  correctas  excepto:  
c.   Pueden  ser  parte  de  una  enfermedad  familiar.   a.   La  Relación  hombre/mujer  10:1  
d.   Rara  vez  se  malignizan.   38.  Manifestaciones   extraintestinales   de   la   enfermedad   de   Crohn   incluye  
31.  Una   mujer   de   50   años   fue   admitida   con   historia   de   pérdida   de   peso   y  
todo  MENOS:  
anemia.   Una   SGD   reveló   un   defecto   de   llenado   en   el   antro   gástrico  
compatible  con  un  cáncer.  ¿a  qué  otro  tipo  de  anemia  está  relacionada   a.   Riesgo  de  malignidad  
ésta  lesión?   39.  La  enfermedad  de  Crohn  se  caracteriza  por:  
a.   Esferocitosis  hereditaria.   a.   Afecta  solamente  la  mucosa  
b.   Anemia  falciforme.   b.   Puede  afectar  todas  las  capas  del  intestino  
c.   Anemia  perniciosa.   c.   Solo  afecta  el  ileon  distal  (Ileitis  Terminal)  
d.   Anemia  mediterránea.  
d.   Ninguna  de  las  anteriores     a.   Salida  de  sangre  por  el  ano  
40.  Los  síntomas  más  común  de  la  enfermedad  de  Chron  son:   b.   Vómitos  de  sangre  
a.   Evacuaciones  mucosas  sanguinolentas   c.   Acción  del  ácido  clorhídrico  sobre  la  sangre  en  el  estómago  
b.   Dolor  y  masa  en  cuadrante  inferior  derecho   d.   Ninguna  
c.   Fiebre,  taquicardia  y  diarrea  con  sangre   47.    Según   la   estadística,   en   qué   nivel   se   encuentra   el   cáncer   colorrectal  
d.   Todas  las  anteriores   como  causa  de  muerte:  
41.  Las  causas  más  comunes  de  sangrado  digestivo  alto  son:   a.   Segunda  
a.   Gastritis  erosiva,  cáncer  gástrico,  várices  esofágicas   b.   Primera  
b.   Gastritis  erosiva,  enfermedad  ulceropéptica,  várices  esofágicas   c.   Tercera  
c.   Esofagitis,  cáncer  gástrico,  úlcera  duodenal   48.  Cuál  es  la  sintomatología  que  NO  es  característica  de  cáncer  de  recto:  
d.   Ninguna   a.   Cambio  de  hábito  de  defecación  
42.  El  estudio  de  elección  para  el  sangrado  digestivo  alto  es:   b.   Disminución  en  diámetro  de  heces  
a.   Serie  esofagogastroduodenal   c.   Intolerancia  a  las  grasas  
b.   Centelleo  con  glóbulos  rojos  marcados   49.  Cuál  es  la  principal  sintomatología  de  cáncer  de  colon  derecho:  
c.   Endoscopía  digestiva  alta   a.   Tumor  palpable  y  anemia  
d.   Todas   b.   Dolor  epigástrico  
43.  El  centelleo  con  glóbulos  rojos  marcados  es  positivo  cuando  el  sangrado   c.   Vesícula  biliar  palpable  e  ictericia  
es:   50.  Cuál  es  la  dieta  principal  para  evitar  el  cáncer  colorrectal  
a.   5  mm/min   a.   Rica  en  carbohidratos  
b.   1  mm/min   b.   Rica  en  grasa  animal  
c.   0.5  mm/min   c.   Rica  en  fibra  vegetal  
d.   0.1  mm/min   51.  Cuál  es  la  característica  del  cólico  biliar  
44.  Las  causas  más  comunes  de  sangrado  digestivo  bajo  son:   a.   Dolor  en  hipocondrio  derecho,  sin  fiebre  y  sin  leucocitosis  
a.   Cáncer  colorectal,  fisura  anal  y  hemorroides   b.   Ictericia  y  dolor  en  hipocondrio  derecho  
b.   Hemorroides,  fístulas  y  fisuras   c.   Dolor  en  hipocondrio  derecho  con  fiebre  y  leucocitosis  
c.   Divertoculosis,  cáncer  colorrectal  y  angiodisplasia  del  colon   52.  El  cáncer  colorrectal  es  más  frecuente  en  los  segmentos:  
d.   Ninguna   a.   Descendente  
45.  La   teoría   corriente  sobre  la  etiología  de  la   enfermedad   diverticular  del   b.   Transverso  
colon  es   c.   Recto-­‐Sigmoides  
a.   La  ingestión  frecuente  de  carnes   53.  En   la   úlcera   duodenal   complicada   con   estenosis,   el   síntoma   que  
b.   El  exceso  de  harina  refinada  en  la  dieta   prevalece  es:  
c.   Disminución  de  fibras  en  la  dieta   a.   Vómitos  biliosos  
d.   Factores  de  degeneración  de  la  pared  intestinal  con  la  edad   b.   Vómitos  postprandiales  
e.   Factores  hereditarios   c.   Vómitos  tardíos  
46.  El  término  melanomesis  corresponde  a:   d.   Vómitos  en  proyectil  
54.  En  un  paciente  de  70  años  con  cuadro  clínico  de  obstrucción  intestinal,   a.   Molestias  epigástricas  
la  causa  más  frecuente  es:   b.   Molestias  peri-­‐umbilicales  
a.   Bridas   c.   Molestas  en  la  fosa  iliaca  derecha  
b.   Hernias  inguinales   d.   Ninguna  
c.   Intususepción   e.   Todas  
d.   Neoplasia  de  colon   60.  La  apendicitis  aguda  puede  ser  producida  por:  
55.  Paciente   femenina   de   19   años   de   edad,   procedente   de   Penonomé,   a.   Fecalitos  
operada   hace   6   días   de   apendicitis   aguda   perforada,   cursa   con   b.   Inflamación  de  los  linfáticos  del  apéndice  
distensión   abdominal,   diarrea,   disuria,   fiebre   y   leucocitosis,   el   c.   Tumor  carcinoide  
diagnóstico  es:   d.   Ninguno  
a.   Obstrucción  intestinal   e.   Todos  
b.   Absceso  subfrénico   61.  En  El  factor  más  importante  en  la  etiología  de  la  apendicitis  aguda  es:  
c.   Absceso  pélvico   a.   Infección  bacteriana  
d.   Absceso  interasas   b.   Obstrucción  mecánica  
56.    Cuál  de  estos  hallazgos  es  el  que  usted  considera  MÁS  importante  en  la   c.   Insuficiencia  vascular  
apendicitis  aguda:     d.   Factores  químicos  
a.   Vómitos   e.   Tejido  linfoide  del  apéndice    
b.   Fiebre   62.  una   paciente   de   30   años   de   edad,   procedente   de   Darién,   con  
c.   Cuenta  de  blancos  elevada   obstrucción   intestinal,   deshidratación   grave,   fiebre   y   presencia   de  
d.   Defensa  en  la  FID     válvulas   conniventes   visibles   en   radiografía   simple   de   abdomen.   Su  
e.   Diarrea     manejo  es:  
57.  La   manifestación   clínica   más   importante   para   el   diagnóstico   de   la   a.   Tratamiento  médico  por  13  horas  
apendicitis  aguda  es:   b.   Sonda  nasogástrica,  hidratación  y  cirugía  en  4  horas  
a.   Rebote   c.   Sonda  nasogástrica,  hidratación  y  antibióticos  por  24  horas  
b.   Dolor  en  fosa  iliaca  derecha   d.   Tratamiento  médico  por  8  horas  
c.   Defensa  muscular   63.  El  tubo  de  Baker  o  Miller  Abott  está  indicado  en  la  obstrucción  intestinal  
d.   Ninguna   cuando  existe:  
e.   Todas   a.   Cáncer  de  colon  
58.  El  diagnóstico  de  apendicitis  aguda  puede  confundirse  con:   b.   Intususepción  intestinal  
a.   Inflamación  pélvica   c.   Recurrencia  de  obstrucción  
b.   Ileitis  regional   d.   Enfermedad  de  Crohn  
c.   Diverticulitis  del  ciego   64.  Paciente  femenina  de  20  años  de  edad,  procedente   de  aguadulce,  fue  
d.   Ninguna   operada  por  apendicitis  aguda  perforada  hace  una  semana,  ingresa  con  
e.   Todas   fiebre  alta,  disnea,  hipotensión  y  leucocitosis.  Su  diagnóstico  es:  
59.  Generalmente,  la  apendicitis  comienza  con:   a.   Deshiscencia  del  muñón  apendicular  
b.   Infección  de  la  herida  quirúrgica   71.  Una   mujer   de   21   años,   previamente   sana,   se   presenta   con   dolor  
c.   Sepsis  intrabdominal   abdominal   de   48   horas   de   evolución.   El   dolor   fue   inicialmente  
d.   Absceso  subfrénico   periumbilical,   y   posteriormente   se   localizó   en   cuadrante   inferior  
65.  Paciente  masculino  de  alto  riesgo  quirúrgico  que  presenta  perforación   derecho.   Tuvo   náuseas   y   disminución   del   apetito.   Niega   disuria.   Su  
de  úlcera  péptica  de  24  a  36  horas  de  evolución,  se  le  realizó  un  estudio   última   menstruación   fue   dos   semanas   antes.   Examen   físico:   febril  
con   gastrografía   que   no   reveló   extravasación   del   medio   a   la   cavidad   (38.2°C),  dolor  en  cuadrante  inferior  izquierda  y  con  defensa  muscular.  
peritoneal,   no   presenta   datos   de   peritonitis   generalizada,   se   decide   Tacto  rectal  normal.  Leucocitosis  moderada.  Diagnóstico  más  probable?  
realizar   manejo   no   quirúrgico   de   la   úlcera   perforada,   que   incluye,   a.   Gastroenteritis  
excepto:   b.   Pancreatitis  aguda  
a.   Succión  con  sonda  nasogástrica   c.   Apendicitis  aguda  
b.   Bloqueadores  H2  E.V.   d.   Enteritis  regional  
c.   Antibióticos   72.  En   la   obstrucción   intestinal   del   intestino   delgado,   cuál   es   el   síntoma  
d.   Octeotride   predominante:  
66.  En  relación  al  sangrado  por  enfermedad  diverticular  del  colon,  podemos   a.   Distención  abdominal  
afirmar:   b.   Cólico  abdominal  
a.   Es  la  causa  más  común  de  sangre  oculta  en  heces   c.   Vómitos  
b.   La  mayoría  de  los  pacientes  requieren  resección  del  colon  para   d.   falta  de  eliminación  de  heces  y  gases  
prevenir  resangrado   73.  La  causa  más  frecuente  de  la  obstrucción  intestinal  
c.   Cuando  el  sangrado  persiste,  la  gammagrafía  con  glóbulos  rojos   a.   Adherencias  
marcados  es  útil  para  el  diagnóstico   b.   Estrecheses  
d.   Rara  vez  se  resuelve  espontáneamente   c.   Vólvulo  
67.  Todo  lo  siguiente  es  cierto  en  lo  referente  a  hemorroides  trombosadas,   d.   Bezoar  
EXCEPTO:   e.   Intususcepsión  
a.   Se  cura  espontáneamente  en  24  horas   74.  Cuándo  aparece  el  signo  radiológico  de  pilas  de  monedas  o  signo  de  la  
68.  Tratamiento  principal  del  ileo  paralítico   escalera   en   la   radiografía   de   abdomen   en   cuadro   de   obstrucción  
a.   Fluidos  IV,  prostigmina,  Tubo  de  Levin   intestinal:  
69.  En  obstrucción  intestinal,  la  depleción  de  líquidos  es  debido  a:   a.   Cuando  hay  signos  de  perforación  
a.   Líquidos  retenidos  en  el  lumen  intestinal,  líquidos  perdidos  por   b.   Cuando  hay  edema  de  la  válvula  ileocecal  y  está  continente  
vómitos,  Succión  gástrica   c.   Cuando  hay  edema  de  las  válvulas  conniventes  
70.  La   incidencia   de   cáncer   colorectal   está   correlacionada   positivamente   d.   Cuando  hay  signos  de  isquemia  
con,  EXCEPTO   75.  Cuál  es  el  manejo  médico  inmediato  en  el  tratamiento  de  la  obstrucción  
a.   Ingesta  aumentada  de  carne   intestinal  
b.   Edad   a.   Venoclisis  con  lactato  
c.   Fumar   b.   Colocación  de  sonda  vesical  
d.   Ingesta  aumentada  de  grasas   c.   Colocación  de  sonda  nasogástrica  
d.   Realizar  tipaje,  Th,  prueba  cruzada  y  trasfundir  GRE   c.   Dolor  abdominal  en  cuadrante  inferior  derecho  
76.  Después   del   drenaje   de   un   absceso   perianal,   qué   debe   informarle   al   d.   Todas  las  anteriores  
paciente   82.  La  enfermedad  diverticular  es  más  frecuente  en:  
a.   Que  va  a  sangrar   a.   Duodeno  
b.   Que  va  a  tener  dificultad  para  evacuar   b.   Yeyuno  
c.   Que  posteriormente  va  a  tener  una  fistula  perianal   c.   Colon  transverso  
d.   Que  va  a  sentir  mucho  dolor   d.   Ciego  
77.  El  sitio  que  menos  presenta  cáncer  de  la  vía  digestiva  es:   e.   Sigmoides    
a.   Esófago   83.  Las  complicaciones  del  divertículo  de  Meckel  son  todas,  MENOS:  
b.   Estómago   a.   Sangramiento  
c.   Intestino  delgado   b.   Inflamación  
d.   Intestino  grueso   c.   Perforación  
e.   Ano-­‐recto   d.   Obstrucción  
78.  El  cáncer  gástrico  está  asociado  con:   e.   Malignización    
a.   Hipoacidez   84.  Los  factores  que  contribuyen  a  la  enfermedad  diverticular  incluye  todo  
b.   Alto  índice  de  incurabilidad   menos:  
c.   Alta  mortalidad  con  gastrectomía  subtotal   a.   Constipación  crónica  
d.   A  y  B   b.   La  edad  
e.   A  y  C   c.   Heces  con  gran  residuo  
f.   A,  B  y  C   d.   Contracciones  circulares  del  intestino  
79.  El  recurso  más  efectivo  en  el  diagnóstico  de  cáncer   gástrico  temprano   e.   Predisposición  heredada  
es:   85.  La  enfermedad  diverticular  inicia  en:  
a.   Serie  esofagogastroduodenal  con  doble  contraste   a.   Quinta  década  
b.   Tomografía  de  epigastrio   86.  En  relación  al  sangrado  por  enfermedad  diverticular  del  colon:  
c.   Ultrasonografía   a.   Cuando  el  sangrado  persiste,  la  gammagrafía  con  GR  marcados  
d.   Endoscopia   es  útil  para  el  diagnóstico  y  tratamiento  
80.  Un  paciente  con  historia  de  polipomatosis  familiar  se  le  hace  biopsia  de   87.  Momento  en  que  se  escuchan  los  ruidos  intestinales  de  lucha:  
uno  de  los  pólipos.  Qué  tipo  de  pólipos  se  espera  encontrar:   a.   Durante  el  dolor  y  el  cólico  
a.   Adenoma  velloso   88.  __C__  El  adenoma  velloso  se  considera  como  un  tumor  premaligno  
b.   Pseudopólipo   89.  __F__  La  apendicitis  aguda  no  debe  ser  diagnosticada  a  menos  que  se  
c.   Polipo  hiperplásico   halle  rigidez  en  la  fosa  iliaca  derecha  
d.   Polipo  adenomatoso   90.  __F__  la  sobrevida  de  casos  con  cáncer  gástrico  se  ha  mejorado  por  el  
81.  Los  síntomas  más  comunes  de  la  colitis  ulcerativa  son:   uso  más  generalizado  de  cirugía  radical  
a.   Evacuaciones  mucosas  sanguinolentas   91.  __F__  La  estrechez  de  un  segmento  del  ileo  terminal  con  obstrucción  en  
b.   Plastrón  en  cuadrante  inferior  derecho   radiografía  es  indicativo  de  cáncer  del  delgado  
92.  __F__   La   apendicitis   no   se   debe   diagnosticar   a   menos   que   presente   105.   Principal  sintomatología  del  cáncer  de  colon  transverso:  
dolor  y  defensa  en  fosa  iliaca  derecha.   a.   CAMBIOS  EN  EL  HÁBITO  DE  DEFECACIÓN  
93.  __C__  El  antígeno  carcino-­‐embrionario  (CEA)  es  útil  en  el  seguimiento  y   b.   TUMORACIÓN  PALPABLE  
manejo  de  casos  operados  de  cáncer  de  colon   106.   Principal  sintomatología  en  la  cáncer  de  colon  izquierdo:  
94.  __C__  El  megacolon  tóxico  no  es  indicación  para  cirugía  de  urgencia   a.   CAMBIOS  EN  EL  HÁBITO  DE  DEFECACIÓN  
95.  __F__  Los  pseudopólipos  en  caso  de  colitis  ulcerativa  son  premalignos   b.   DISTENSIÓN  ABDOMINAL  
96.  Un   paciente   de   60   años   con   historia   de   sangrado   y   dolor   durante   la   c.   DISMINUCIÓN  DEL  DIÁMETRO  DE  LAS  HECES  
evacuación,  mencione  dos  diagnósticos  diferenciales   107.   Sintomatología  del  cáncer  de  recto:  
a.   FISURA  ANAL   a.   CAMBIOS  EN  EL  HÁBITO  DE  DEFECACIÓN  
b.   CÁNCER  COLORRECTAL   b.   SANGRADO  TRANSANAL  
97.  Historia   de   epigastralgias   y   pirosis   crónica,   ahora   con   hemorragia   c.   DISMINUCIÓN  DEL  DIÁMETRO  DE  LAS  HECES  
digestiva  alta:  ULCERA  GASTRODUODENAL   108.   Cuál  es  la  dieta  principal  para  evitar  el  cáncer  colo-­‐rectal:  ALTA  EN  
98.  El   síntoma   más   común   de   la   enfermedad   de   Crohn   es:   FÍSTULA   FIBRAS  Y  BAJA  EN  GRASAS  
PERIANAL   109.   La   incidencia   de   cáncer   colorrectal   está   correlacionada  
99.  El   síntoma   más   común   de   la   colitis   ulcerativa   es:   HECES   positivamente  con:  
SANGUINOLENTAS   a.   EDAD  
a.   Las  causas  más  comunes  de  sangrado  digestivo  alto  son:     b.   INGESTA  AUMENTADA  DE  CARNE  
b.   GASTRITIS  ATRÓFICA   c.   INGESTA  AUMENTADA  DE  GRASAS  
c.   ENFERMEDAD  ACIDO  PÉPTICA   110.   El   centelleo   con   glóbulos   rojos   marcados   es   positivo   cuando   el  
d.   VÁRICES  ESOFÁGICAS   sangrado  es  de:  0.1  ML/MIN  
100.   El  Síndrome  de  Zollinger  Ellison  incluye   111.   La   angiografía   mesentérica   puede   indicar   el   sitio   de   hemorragia  
a.   HIPERSECRECIÓN  GÁSTRICA   (80%)  cuando  el  sangrado  excede:  0.5  ML/MIN    
b.   ÚLCERA  PÉPTICA  INTRATABLE   112.   Paciente  de  35  años,  femenina  que  informa  dolor  antes  y  después  
c.   TUMOR  DE  LAS  CÉLULAS  NO  BETA  DEL  PANCREAS   de   evacuar,   desde   hace   1   mes.   Presenta   sangrado   rojo   después   de  
101.   El   estudio   de   elección   para   evaluar   el   sangrado   digestivo   alto   es:   evacuar.   Dice   tener   una   tetita   de   carne   en   el   ano,   antecedentes   de  
ENDOSCOPÍA  DEL  TRACTO  DIGESTIVO  ALTO   estreñimiento   desde   hace   2   -­‐   3   días   y   que   tiene   2   familiares   con  
almorranas.  ¿Cuál  es  su  diagnóstico  y  a  donde  lo  refiere?  
102.   La  causa  principal  de  obstrucción  intestinal  en  pacientes  adultos  es:  
a.   FISURA  ANAL  
ADHERENCIAS   b.   COLOPROCTOLOGÍA  
103.   Las  causas  más  comunes  del  sangrado  digestivo  bajo  son:     113.   Enumere   3   causas   más   frecuentes   de   la   isquemia   arterial  
a.   CARCINOMA  COLORRECTAL   obstructiva  
b.   ANGIODISPLASIA  DEL  RECTO   a.   TRAUMA  
c.   DIVERTICULOSIS   b.   TROMBOS  
104.   Cuál  es  la  primera  sintomatología  del  cáncer  de  colon  derecho:     c.   EMBOLISMO  
a.   ANEMIA   114.   Masculino   de   65   años   con   historia   de   diarreas   intercaladas   con  
b.   TUMOR  PAPLABLE   estreñimiento  dese  hace  un  año.  Ha  tenido  varios  episodios  de  dolor  en  
fosa   iliaca   derecha   y   fiebre.   Se   ha   tratado   con   antibióticos   y   ha   122.   Hematuria  y  fecaluria  en  paciente  de  45  años,  le  hace  sospechar  de  
mejorado.  Niega  sangrados  vía  rectal,  niega  pérdida  de  peso.  Indique  3   fístula  enterovesical  por:  DIVERTICULOSIS  
diagnósticos  diferenciales   123.   La  clasificación  de  Dukes  es  útil  para  evaluar  el  pronóstico  de  cáncer  
a.   DIVERTICULOSIS   de:  COLON  
b.   CÁNCER  DE  COLON   124.   Dos  entidades  quirúrgicas  que  suelen  mejorar  con  una  ingesta  alta  
c.   COLON  IRRITABLE   en  fibras  
115.   Paciente   de   19   años   operada   hace   4   días   de   apendicitis   aguda   a.   CONTIPACIÓN  
perforada,   cursa   con   fiebre   y   leucocitosis,   no   vómitos,   no   diarrea,   no   b.   ENFERMEDAD  DIVERTICULAR  
dolor  abdominal.  Diagnóstico  y  manejo:   125.   Un   tumor   doloroso   que   se   presenta   súbitamente   en   la   región   anal  
a.   INFECCIÓN  DE  HERIDA  QUIRURGICA   sería  probablemente:  HEMORROIDES  TROMBOSADA  
b.   DEBRIDAR  HERIDA  QUIRURGICA??=   126.   La  sintomatología  de  la  hemorroides  trombosada  es:  DOLOR  
116.   Paciente  femenina  de  19  años  de  edad,  procedente  de  Penonomé,   127.   Patologías  del  ano  en  las  cuales  puede  aparecer  masa  dolorosa  en  
operada   hace   6   días   de   apendicitis   aguda   perforada,   cursa   con   24  horas  
distensión   abdominal,   diarrea,   disuria,   fiebre   y   leucocitosis,   el   a.   HEMORROIDES  INTERNA  TROMBOSADA  
diagnóstico  es:  ABSCESO  PÉLVICO   b.   ABSCESO  PERIANAL  
117.   Paciente  masculino  de  40  años  con  diagnóstico  de  síndrome  pilórico   128.   Dolor  de  la  fisura  anal  post  evacuación  es  debido  a:  HIPERTONÍA  
por   úlcera   duodenal,   acude   a   urgencias   con   historia   de   vómitos   de   7   129.   Hemorragia   digestiva   baja   en   un   anciano   sin   previa   historia   de  
días  de  evolución  y  que  hoy  ha  orinado  poco.  Informa  pérdida  de  peso.   enfermedad  y  asintomático,  actualmente  su  causa  sería:  DIVERTICULO  
Examen  físico:  signos  evidentes  de  deshidratación.   130.   Enumere  cuatro  síntomas  de  la  fístula  anorectal  
a.   Trastorno  metabólico:  ALCALOSIS  METABÓLICA   a.   SALIDA  DE  SECRECIÓN  PURULENTA  
b.   Qué  solución  parenteral  administraría:  LACTATO  RINGER   b.   DOLOR  ANORRECTAL  
c.   Manejo  posterior:  CIRUGÍA   c.   FIEBRE  Y  ESCALOFRIOS  
118.   una   paciente   de   30   años   de   edad,   procedente   de   Darién,   con   d.   POSICIÓN  ANTALGICA  
obstrucción   intestinal,   deshidratación   grave,   fiebre   y   presencia   de   131.   Enumere  cuatro  factores  desencadenantes  del  prolapso  rectal  
válvulas   conniventes   visibles   en   radiografía   simple   de   abdomen.   Su   a.   PARASITOS  EN  NIÑOS  (TRICOCEFALO)  
manejo  es:  SONDA  NASOGÁSTRICA,  HIDRATACIÓN  Y  CIRUGÍA.   b.   RELAJACIÓN  DEL  PISO  PÉLVICO  
119.   Paciente  femenina  de  20  años  de  edad,  procedente   de  aguadulce,   c.   MUJERES  CON  HISTORIA  DE  HISTERECTOMÍA  
fue  operada  por  apendicitis  aguda  perforada  hace  una  semana,  ingresa   d.   NO  FIJACIÓN  DEL  RECTO  AL  SACRO  
con   fiebre   alta,   disnea,   hipotensión   y   leucocitosis.   Su   diagnóstico   es:   132.   Diagnóstico  de  CUCI:  
SEPSIS  INTRABDOMINAL     a.   ANOSCOPIA  
120.   En  qué  resulta  la  enfermedad  de  Menetrier:  HIPOPROTEINEMIA   b.   COLONOSCOPIA  
121.   La  enfermedad  hemorroidal  interna  de  III  grado  se  caracteriza  por:   c.   RECTOSIGMOIDOSCOPIA  
a.   SANGRADO   133.   El  síntoma  principal  de  CUCI  es:  DIARREA  CON  SANGRE  
b.   PROLAPSO  QUE  SE  REDUCE  MANUALMENTE   134.   Cuando  el  colon  parece  un  tubo  de  plomo  en  el  enema  de  bario,  el  
diagnóstico  más  aceptable  es:  COLITIS  ULCERATIVA  
135.   El  síntoma  más  común  de  la  enfermedad  de  Crohn  es:  DOLOR   a.   La  tomografía  es  el  diagnóstico  de  elección  
136.   El   síntoma   más   común   de   la   colitis   ulcerativa   es:   DIARREA   CON   b.   Actualmente   la   morbi-­‐mortalidad   es   igual   independiente   del  
MOCO  Y  SANGRE   tiempo  de  evolución  
c.   Las  perforaciones  por  trauma  cerrado  ocurre  más  frecuente  
137.   El   síntoma   más   frecuente   en   la   obstrucción   intestinal   es:   DOLOR  
d.   La   morbi-­‐mortalidad   es   menor   en   las   lesiones   del   esófago  
COLICO   cervical    
138.   Qué  encuentras  en  una  punción  abdominal:   3.   Es  causa  de  esofagitis  por  Reflujo  Gastro  Esofágico  
a.   ERITROCITO   a.   Hipotonía  del  EEI  
b.   LEUCOCITOS   b.   Vaciamiento  gástrico  anormal  
c.   BILIS   c.   Motilidad  anormal  de  la  parte  inferior  del  esófago  
d.   AMILASA   d.   Todas    
e.   Ninguna  
e.   FIBRAS  O  RESIUDO  
4.   A   vista   de   los   conocimientos   actuales,   el   esófago   de   Barrett   es  
f.   BACTERIAS   probablemente  debido  a  :  
139.   En   la   úlcera   duodenal   complicada   con   estenosis,   el   síntoma   que   a.   Daño  de  la  mucosa  esofágica  por  reflujo  gastroesofágico  
prevalece:  VÓMITOS  TARDIOS   b.   Daño  de  la  mucosa  esofágica  por  ingesta  de  cáusticos  
c.   Daño   de   la   mucosa   esofágica   por   ingesta   de   alimentos   muy  
A.   Colitis  Ulcerosa   D   Mejora  con  dieta  alta  en  residuos   calientes  
B.   Enfermedad  de  Cronh   A   Puede  (n)  causar  megacolon  tóxico   d.   Animalias  del  desarrollo  (fenómeno  congénito)  
C.   Se  sospecha  cuando  hay  fístulas  peri   e.   Cualquiera  de  las  anteriores  
Colitis  Amebiana   B  
anal  complicada   5.   Los  divertículos  epifrenicos  suelen  ser  debidos  a:  
D.   Ninguna   de   las   antes   a.   Anomalías  del  desarrollo  embrionario  
C   Afecta  solo  la  mucosa  del  colon  
mencionadas   b.   Debilidad  de  la  pared  esofágica  
E.   Todas   las   antes   c.   Factores  dietéticos  
C   No  hay  degeneración  maligna  
mencionadas   d.   Obstrucción  distal  del  esófago  causada  por  espasmo  o  acalasia  
  e.   Tracción  por  procesos  cicatriciales  de  vecindad  
6.   La  acalasia  esofágica  es:  
ESOFAGO   a.   Un  tumor  maligno  del  esófago  
b.   Un  trastorno  de  la  motilidad  esofágica  
1.   Cuál   de   las   siguientes   afirmaciones,   respecto   al   síndrome   de   Mallory-­‐ c.   Resultado  del  reflujo  gastroesofágico  
Weiss  son  correctas:   d.   Proceso  hereditario  
NO   El   principal   dato   patológico   es   la   perforación   espontánea   del   e.   Síntomas  del  divertículo  esofágico  
esófago.   7.   La  patología  del  síndrome  de  Mallory  Weiss  es:  
SÍ   El  principal  síntoma  es  hemorragia  digestiva  después  del  vómito.   a.   Laceración  o  rasgadura  de  la  unión  gastroesofágica  
SÍ   Comúnmente   se   relaciona   con   ingestión   crónica   de   bebidas   8.   La  etiopatogenia   de   la   acalasia   es:   TRASTORNO   DE   LA   MOTILIDAD   DEL  
alcohólicas.   ESÓFAGO  
NO   La  endoscopia  está  contraindicada  debido  al  peligro  de  perforación.  
9.   Vómito   de   contenido   alimentario   y   de   jugo   gástrico   seguido   por  
NO   Tratamiento  quirúrgico  con  prontitud  es  el  tratamiento  preferido.  
  hemorragia  digestiva:  SINDROME  DE  MALLORY  WEISS  
2.   En  las  perforaciones  del  esófago:  
10.  La  ruptura  de  la  mucosa  en  la  unión  gastroesofágica  produce  sangrado   mesogastrio   e   irradiado   a   ambos   cuadrantes   superiores   y   hacia   la  
denominado:  SINDROME  DE  MALLORY  WEISS   espalda  en  ambos  lados.  En  los  dos  últimos  años  había  tenido  cuadros  
11.  Los  divertículos  epifrénicos  suelen  ser  debidos  a:  OBSTRUCCIÓN  DISTAL   similares.  Estos  se  asociaban  a  la  ingesta  de  comidas  grasientas.  La  orina  
DEL  ESÓFAGO  CAUSADA  POR  ESPASMO  O  ACALASIA   se   le   oscurece   en   este   último   episodio.   Sin   cambios   en   los   hábitos  
12.  Enumere  2  causas  de  esofagitis  por  RGE:   evacuacionales.    Al  examen  físico  se  encuentra  una  mujer  bien  nutrida.  
a.   MOTILIDAD  ANORMAL  DE  LA  PARTE  INFERIOR  DEL  ESÓFAGO  
Su  presión  arterial  normal,  temperatura  de  38.9  °C.  Las  escleras  estaban  
b.   VACIAMIENTO  GÁSTRICO  ANORMAL  
c.   HIPOTONÍA  DEL  EEI   ictéricas   y   el   abdomen   marcadamente   distendido   con   dolor   a   la  
13.  A   la   vista   de   los   conocimientos   actuales,   el   esófago   de   Barrett   es   un   palpación   en   ambos   cuadrantes   superiores,   sin   masas.   Ligero   rebote.  
fenómeno  probablemente  debido  a:  DAÑO  DE  LA  MUCOSA  ESOFÁGICA   Examen   rectal   normal   con   heces   chocolates.   Los   estudios   iniciales  
POR  RGE   fueron   11.300   glóbulos   blancos   /cu   mm.   Con   88%   de   neutrófilos;   Hb  
14.  La   mucosa   esofágica   está   compuesta   por   una   capa   de   epitelio:   12.5   gh/100   ml;   electrolitos   normales.   Bilis   positiva   en   orina.   Su  
ESCAMOSO  ESTRATIFICADO  NO  QUERATINIZADO   diagnóstico  diferencial:  
15.  La  pared  del  esófago  carece  de:  SEROSA  
A.   Hepatitis  viral  
16.  Cuál   debe   ser  la   secuencia   diagnóstica   para   el   estudio   de   un   trastorno  
motor  esofágico:  PH-­‐METRIA  Y  TOMA  DE  BARIO   B.   Colecistitis  aguda  
  C.   Pancreatitis  aguda  
HEPATICAS  Y  BILIAR   D.   Gastritis  aguda  
E.   Obstrucción  intestinal  
Seleccione  su  respuesta  en  la  siguiente  forma:   F.   Coledocolititasis  
  G.   Crisis  hemolítica  
A    si  1,  2  y  3  son  correctas   1  __F__  
B    si  1  y  3  son  correctas  
2__B__  
C    si  2  y  4  son  correctas  
D    si  solo  4  es  correcta   3__C__  
E    si  todas  son  correctas   4.   Cuál  de  estos  exámenes  de  laboratorio  pediría  en  orden  de  importancia  
  A.   Examen  de  heces  
1.   _A_  El  ileo  por  cálculo  biliar  se  caracteriza  por   B.   Nitrógeno  de  urea  
a.   Aire  en  el  árbol  biliar   C.   Lipasa  en  suero  
b.   Dilatación  en  el  intestino  delgado   D.   Amilasa  en  suero  
c.   Sombra  de  cálculo  en  la  fosa  iliaca  derecha   E.   Dehidrogenasa  láctica  
2.   _A_  Los  síntomas  de  la  coledocolitiasis  son:   F.   Bilirrubinemia  
a.   Ictericia   G.   Glicemia  
b.   Cólico  biliar   H.   Tiempo  de  protrombina  
c.   Fiebre   __F__  
3.   Usted  es  llamado  para  atender  a  una  mujer  de  39  años  con  historia  de   __C__  
24  horas  de  dolor  agudo  y  severo  en  el  abdomen  superior  acompañado   __D__  
de   náuseas   y   vómitos.   El   dolor   era   tipo   cólico   localizado   en   el  
5.   Qué   estudios   radiográficos   pediría,   de   los   anunciados,   en   orden   de   d.   Rastreo  con  TC.  
importancia   e.   Estudio  con  radionúclidos.  
A.   Colecistografía  endovenosa   8.   Cuál  es  el  tratamiento  quirúrgico  de  la  colecistitis  aguda  
B.   Enema  de  bario   a.   Colecistostomía  
C.   Radiografía  de  tórax   b.   Colecistectomía  
D.   Centelleo  hepático   c.   Exploración  de  la  vía  biliar  
E.   Radiografía  simple  de  abdomen   9.   Cuál  es  la  principal  causa  de  colangitis:  
F.   Colecistografía  oral   a.   Parasitosis  
G.   Ultrasonografía  de  abdomen  superior   b.   Litiasis  
H.   Arteriografía  mesentérica   c.   Cáncer  
__G__   10.  En  una  paciente  diabética  de  65  años  con  cuadro  de  colecistitis  aguda  el  
_____   tratamiento  es:  
__E__   a.   Antibiótico  terapia  
6.   De  cuál  de  estos  consideraría  usted  como  tratamiento  inicial   b.   Manejo  de  diabetes  
A.   Colocar  tubo  nasogástrico   c.   Colocar   sonda   naso   gástrica,   antibiótico   terapia   y   manejo   de  
B.   Monitorear  la  presión  venosa  central   diabetes  
C.   Lactato  Ringer  100cc  por  hora   d.   Manejo  médico  inmediato  y  ante  todo  quirúrgico  
D.   Dar  antibióticos   11.  Los  niveles  de  alfa  feto  proteína  están  más  elevados  en:  
E.   Dar  Vit  K  I.M.   a.   Hepatoma.  
F.   Colocar  catéter  de  Foley   b.   CA  de  colon.  
G.   Dar  un  laxativo  oral   c.   Enfermedad  de  Crohn.  
H.   Balance  hídrico  horario   d.   CA  de  páncreas.  
I.   Preparar  para  una  pancreatografía   e.   CA  de  vesícula.  
J.   Solicitar  una  colangiografía  endoscópica   12.  Un   paciente   con   ictericia   en   la   piel   y   mucosas,   con   urobilinogeno  
__C__   negativo  en  la  orina.  La  ictericia  es  de  origen:  
__F__   a.   Hepatocelular  
__H__   b.   Hemolítica  
__D__   c.   Obstructiva  
__J__   d.   Secundaria  a  Sepsis  
  13.  El   cuadro   clínico   del   ileo   biliar   incluye   los   siguientes   parámetros,  
7.   El  procedimiento  inicial  más  adecuado  para  definir  la  causa  de  ictericia   EXCEPTO:  
obstructiva  en  un  hombre  de  75  años  de  edad  es:   a.   Aire  en  las  vías  biliares  
a.   Colangiopancreatografía  endoscópica  retrógrada  (CPRE).   b.   Obstrucción  del  intestino  delgado  
b.   Colangiografía  transhepática  percutánea  (CTP).   c.   El  cálculo  en  encuentra  en  el  sitio  de  la  obstrucción  
c.   Ultrasonografía.   d.   Ictericia  obstructiva  
14.  La   cirrosis   hepática   de   Laenec   o   Alcohólica,   según   la   clasificación   del   c.   Remover  el  cálculo,  si  está  presente  
síndrome  de  hipertensión  portal  es:   d.   Descomprimir  la  vía  biliar  
a.   Pre-­‐sinusoidal  extrahepática   21.  __F__   La   presencia   de   ictericia   en   un   caso   de   colecistitis   aguda   es  
b.   Pre-­‐sinusoidal  intrahepática   patonogmónico  de  coledocolitiasis.  
c.   Post-­‐sinusoidal  intrahepática   22.  __C__  la  fosfatasa  alcalina  y  la  bilirrubina  directa  tienden  a  elevarse  en  
d.   Post-­‐sinusoidal  extrahepática   ictericia  obstructiva.    
15.  Cuál  es  la  consecuencia  de  la  hipertensión  portal   23.  Un   señor   de   58   años   con   historia   de   tomar   licor   diariamente   por  
a.   Síndrome  de  Mallory  Weiss   muchos   años   llega   con   hemoglobina   de   5   g/dL,   historia   de   melena   y  
b.   Obstrucción  intestinal   vómitos   con   sangre,   ascitis,   ictericia   marcada   y   anasarca.   El   primer  
c.   Várices  esofágicas   estudio   diagnóstico   de   la   causa   hemorrágica   será:   ENDOSCOPIA  
d.   Ulcera  péptica  perforada   ESOFÁGICA.  En  caso  de  que  sea  cirrótica,  la  unicaría  en  la  clasificación  
16.  Recomendación  para  paciente  con  cirrosis  hepática  Child  B:   CHIDS  como:  CHIDS  C  
a.   Ligadura  de  várices  esofágicas   24.  Un   paciente   con   ictericia   en   la   piel   y   mucosas,   con   urobilinogeno  
b.   Esplenectomía   negativo  en  la  orina.  La  ictericia  es  de  origen:  HEMOLÍTICA  
c.   Trasplante  de  hígado   25.  En  un  paciente  diabético  de  65  años  de  edad  con  cuadro  de  colecistitis  
d.   Tratamiento  médico   aguda,  el  tratamiento  es:  QUIRURGICO  
17.  Lo  que  aparece  más  frecuentemente  en  la  hipertensión  portal  es:   26.  Cuáles  son  las  características  que  se  presentan  en  el  cólico  biliar:  
a.   Esplenomegalia   a.   DOLOR  EN  EL  HIPOCONDRIO  DERECHO  
b.   Hepatomegalia   b.   SIN  LEUCOCITOSIS  
c.   Varices  esofágicas   c.   SIN  FIEBRE  
d.   Ascitis   27.  Cuál  es  la  causa  de  colangitis:  LITIASIS  
18.  La  anemia,  leucopenia  y  plaquetopenia  en  hipertensión  portal  se  debe:   28.  Síntomas  de  la  coledocolitiasis  
a.   Sangrado  digestivo   a.   DOLOR  BILIAR  
b.   Hiperesplenismo   b.   ICTERICIA  
c.   Desnutrición   c.   FIEBRE  
d.   Infección   29.  El  ileo  por  cálculo  biliar  se  caracteriza  por:  
19.  En   un   paciente   con   cirrosis   hepática   que   desarrolla   un   hepatoma,   la   a.   PRESENCIA  DE  AIRE  EN  LAS  VIAS  BILIARES  
prueba  más  sensitiva  es:   b.   DILATACIÓN  DEL  INTESTINO  DELGADO  
a.   Alfafetoproteina   c.   CALCULO  A  NIVEL  DEL  PSOAS  (FID)  
b.   Alanina  Aminotrasnferasa   30.  En   un   caso   de   ictericia   obstructiva   severa,   el   procedimiento   más  
c.   Glutamil  Trasnpeptidasa   indicado  para  el  diagnóstico  de  colelitiasis  y/o  coledocolitiasis  sería  
d.   Aspartato  aminotrasnferasa   a.   ULTRASONIDO  
20.  El  objetivo  inicial  en  la  terapia  de  la  colangitis  aguda  es:   b.   COLANGIOPANCREATOGRAFIA  RETROGRADA  ENDOSCOPICA  
a.   Aliviar  la  ictericia  y  prevenir  el  daño  hepático   31.  Objetivo  inicial  de  la  terapia  en  la   colangitis  aguda  es:  DESCOMPRIMIR  
b.   Prevenir  el  desarrollo  de  pancreatitis   LAS  VIAS  BILIARES  
32.  La   Cirrosis   hepática   es   causante   de   una   hipertensión   portal:   POST   c.   Fibroblastos  
SINUSOIDAL  INTRAHEPATICA   d.   Capilares  sanguíneos  
33.  Cuál   es   la   recomendación   que   usted   le   da   a   un   Paciente   con   cirrosis   e.   Ninguno  de  los  de  arriba  
hepática  CHILD  B:  TRASPLANTE  DE  HIGADO   5.   Los   factores  que   contribuyen   a   la   dehiscencia  de   la   herida   se   incluyen  
34.  La  clasificación  de  CHILD  toma  en  cuenta  los  siguientes  criterios:   todos,  MENOS:  
a.   BILIRRUBINA  (mg/dL)   a.   Tos  
b.   ALBUMINA   b.   Ancianidad  
c.   PRESENCIA  DE  ASCITIS   c.   Hipoproteinemia  
d.   PRESENCIA  DE  ENCEFALOPATÍA   d.   Anemia  
e.   PRESENCIA  DE  MALNUTRICIÓN   e.   Malignidad  
6.   Mencione  los  5  factores  de  crecimiento  en  la  reparación  de  la  herida  
HERIDAS   a.   FACTOR  DE  CRECIMIENTO  DERIVADO  DE  PLAQUETAS  
b.   FACTOR  DE  CRECIMIENTO  EPIDERMICO  
1.   Una  herida  abierta  se  contrae  por:  
c.   FACTOR  ALFA  TRANSFORMANTE  DE  CRECIMIENTO  
a.   Acercamiento  de  la  piel  alrededor  
b.   Por  injerto  de  piel   d.   FACTOR  BETA  TRANSFORMANTE  DE  CRECIMIENTO  
c.   Por  proliferación  miofibroblástica   e.   FACTOR  DE  CRECIMIENTO  FIBROBLASTICO  
d.   Por  acción  de  los  linfocitos   7.   Enumere  4  fases  de  la  reparación  de  una  herida  
2.   Cuál   de   las   siguientes   es   absolutamente   esencial   en   la   cicatriz   de   las   a.   FIBROPLASIA  
heridas  quirúrgicas:   b.   EPITELIZACIÓN  
a.   Vitamina  D   c.   CONTRACCIÓN  
b.   Carbohidratos  
d.   FUERZA  DE  LA  HERIDA  
c.   Ingesta  calórica  
d.   Vitamina  C   8.   Enumerar   la   clasificación   de   las   heridas   de   acuerdo   con   la  
e.   Dieta  hipoproteica   estimación   clínica   de   la   contaminación   microbiana   y   al   riesgo   de  
3.   Un   hombre   de   62   años   se   produce   una   herida   cortante   en   las   manos   infección  subsiguiente  
con  una  botella  de  cerveza  cuando  estaba  de  paseo  en  la  playa.  Acude  a   a.   HERIDA  LIMPIA,  1.5  –  5.1%  
usted   para   suturarlo   y   refiere   que   tiene   todas   sus   vacunas   desde   la   b.   HERIDA  LIMPIA  CONTAMINADA,  7.7  –  10.8%  
niñez  y  que  hace  14  años  le  inyectaron  toxoide  tetánico.  ¿Cuál  sería  su   c.   HERIDA  CONTAMINADA,  15.3  –  16.2%  
indicación  acerca  de  la  prevención  de  infección  y  tétano?   d.   HERIDA  SUCIA,  28  –  40%  
a.   Inmunoglobulina  tetánica  más  toxoide  tetánico   9.   En  el  manejo  de  las  heridas  contaminadas,  el  factor  más  importante  
b.   Toxoide  tetánico  y  lavado  con  agua  estéril  o  solución  salina   para  evitar  las  infecciones  es:  DESBRIDAMIENTO  DE  TODO  EL  TEJIDO  
c.   Inmunoglobulina  tetánica   NECRÓTICO  
d.   Antibióticos  más  toxoide  tetánico  en  solución  Gaudiane   10.  Factores  involucrados  en  la  reparación  de  las  heridas  
4.   Durante  el  proceso  de  cicatrización  el  colágeno  es  producido  por:   a.   FIBROBLASTOS  
a.   Células  epiteliales   b.   PLAQUETAS  
b.   Células  endoteliales   c.   FACTOR  DE  CRECIMIENTO  TRANSFORMANTE  ALFA  Y  BETA  
11.  Fibroplasia/Patofisiología  de  la  reparación  de  heridas   para   caminar,   no   refiere   vómitos   o   cólicos   abdominal.   El   Diagnóstico  
a.   INFLAMACIÓN   más  probable  es:  
b.   PROLIFERACIÓN   a.   Testículo  criptorquídico  
c.   MADURACIÓN   b.   Quiste  de  cordón  inguinal  
c.   Hidrocele  
HERNIAS   d.   Hernia  inguinoescrotal  
6.   El  saco  herniario  en  la  hernia  inguinal  indirecta,  realiza  un  recorrido  por:  
1.   En  relación  al  conducto  inguinal,  cuál  de  los  conceptos  es  verdadero:  
EL  CORDON  INGUINAL  
a.   El   límite   superior   lo   constituye   la   aponeurosis   del   músculo  
7.   En   relación   al   conducto   inguinal,   escriba   su   límite   lateral:   LIGAMENTO  
transverso  del  abdomen  
INGUINAL  
b.   El  límite  inferior  es  el  ligamento  de  Cooper  
8.   Escriba   el   límite   medial   del   agujero   femoral:   LIGAMENTO   DE  
c.   En  su  contenido  se  encuentra  el  nervio  genito-­‐femoral  
GIMBERNAT  O  LACUNAR  
d.   El  límite  lateral  lo  constituye  el  ligamento  inguinal  
9.   En  relación  a  los  límites  del  agujero  femoral:  
2.   En   relación   a   los   límites   del   agujero   femoral,   cuál   de   los   siguientes  
a.   Límite  anterior:  LIGAMENTO  INGUINAL  (POUPART)  
conceptos  es  verdadero  
b.   Límite  posterior:  LIGAMENTO  PECTINEO  (COOPER)  
a.   El  límite  medial  lo  constituye  el  ligamento  de  Cooper  
c.   Límite  medial:  LIGAMENTO  LACUNAR  (GIMBERNAT)  
b.   El  límite  lateral  lo  constituye  el  ligamento  inguinal  
d.   Límite  lateral:  VASOS  FEMORALES  
c.   El  límite  anterior  lo  constituye  la  cintilla  iliopectinea  
10.  Cuando  una  de  las   paredes  del  órgano  contenido  en   una  hernia  forma  
d.   El  límite  medial  lo  constituye  el  ligamento  de  Gimbernat  
parte   y   está   adherido   al   saco   herniario,   a   la   hernia   se   le   denomina:  
3.   El  saco  herniario  en  la  hernia  inguinal  indirecta,  realiza  su  recorrido  por:  
HERNIA  DESLIZANTE  
a.   Conducto  inguinal  
11.  Fisiopatología   de   la   hernia   inguinal   indirecta:   DEBILIDAD   DE   LA   PARED  
b.   Cordón  inguinal  
POSTERIOR  O  FASCIA  TRANSVERSALIS  
c.   Fascia  transversa  
12.  Hernia  donde  parte  del  saco  herniario  es  parte  de  la  pared  intestinal  se  
d.   Anillo  inguinal  
llama:  HERNIA  DE  RICHTER  
4.   Cuando  una  de  las   paredes  del  órgano  contenido  en  una  hernia  forma  
13.  Hernia   que   produce   encarcelamiento   sin   causar   obstrucción:   HERNIA  
parte  y  está  adherido  al  saco  herniario,  la  hernia  se  denomina:  
DESLIZANTE  
a.   Hernia  de  Richter  
14.  Paciente  de  35  años  de  edad,  procedente  de  Colón  con  historia  de  masa  
b.   Hernia  en  pantalón  
en  la  región  inguino  escrotal  izquierda.  Hace  5  años  que  dicha  masa  no  
c.   Hernia  de  Litre  
aumenta  ni  disminuye  con  los  esfuerzos,  no  se  reduce.  El  Paciente  tiene  
d.   Hernia  deslizante  
la  sensación  de  que  el  testículo  ha  desaparecido,  presenta  incomodidad  
5.   Paciente  de  35  años  de  edad,  procedente  de  Colón  con  historia  de  masa  
para   caminar,   no   refiere   vómitos   o   cólicos   abdominal.   El   Diagnóstico  
en  la  región  inguinoescrotal  izquierda.  Hace  5  años  que  dicha  masa  no  
más  probable  es:  HIDROCELE  
aumenta  ni  disminuye  con  los  esfuerzos,  no  se  reduce.  El  paciente  tiene  
15.  Paciente  de  sexo  masculino  de   35  años  de  edad,  que  hace  6  semanas  
la  sensación  de  que  el  testículo  ha  desaparecido,  presenta  incomodidad  
observó  una  masa  en  la  región  inguinal  derecha.  Escriba  2  Diagnósticos  
diferenciales:  
a.   HERNIA  INGUINAL   d.   Dejar  hematomas  en  la  herida  
b.   ADENOSIS  INGUINAL   e.   Aproximar  fuertemente  las  suturas  sobre  la  piel  
16.  En  la  obstrucción  intestinal  por  hernia  inguinal,  ¿Como  usted  diferencia   3.   Con  respecto  a  los  antibióticos  profilácticos  todas  son  ciertas  EXCEPTO:  
una  hernia  inguinal  extrangulada  de  una  encarcelada?   a.   Deben  iniciarse  el  día  antes  de  la  cirugía  
a.   LA   HERNIA   INGUINAL   EXTRANGULADA   PRESENTA   NECRÓSIS   Y   b.   Están  indicados  si  las  probabilidades  de  infección  son  altas  
COMPROMETE  EL  SISTEMA  VASCULAR.  
c.   Están   indicadas   si   las   probabilidades   de   la   infección  son   bajas,  
b.   LA   HERNIA   ENCARCELADA   NO   COMPROMETE   EL   SISTEMA  
VASCULAR  Y  NO  REDUCE   pero  una  infección  podría  ser  fatal  
17.  Paciente  masculino  de  35  años  desde  hace  6  semanas  observa  masa  en   d.   Se  requiere  niveles  séricos  óptimos  al  hacer  la  incisión  
región  inguinal  derecha.  Diagnóstico  diferencial:   4.   Todo  lo  siguiente  aumenta  la  tasa  de  infecciones  en   una  cirugía  mayor  
a.   HERNIA  INGUINAL   excepto:  
b.   HIDROCELE   a.   Paciente  mayor  de  70  años  
c.   ADENITIS  INGUINAL   b.   Infecciones  alejadas  del  sitio  operatorio  
18.  La   hernia   inguinoescrotal   que   no   causa   obstrucción   y   no   se   reduce:   c.   Diabetes  mellitus  controlada  
ENCARCERADA   d.   Ingesta  de  corticoides  
19.  Dentro   de   las   hernias   inguinales,   cuál   es   la   más   frecuente:   HERNIA   e.   No  hay  excepto,  todas  aumentan  la  tasa  de  infecciones  
INGUINAL  INDIRECTA   5.   La  celulitis  plantea  diagnóstico  diferencial  con:  
20.  En  la   obstrucción  intestinal  por  hernia  inguinal,  cómo  usted   diferencia   a.   Tromboflebitis  
una  her  nia  inguinal  estrangulada  de  una  encarcelada   b.   Alergia  intensa  por  contacto  
a.   EN   LA   ENCARCELADA   NO   SE   PUEDE   REDUCIR   Y   NO   HAY   c.   Inflamación  química  por  inyección  de  fármacos  
COMPROMISO  VASCULAR   d.   Fascitis  necrosante  
b.   EN  LA  ESTRANGULADA  SI  HAY  COMPROMISO  VASCULAR   e.   Todas  
6.   Acerca  de  la  miositis  clostridial  (gangrena  gaseosa),  señale  la  respuesta  
INFECCIONES  Y  COMPLICACIONES  EN  CIRUGÍA   INCORRECTA  
a.   Se  inicia  habitualmente  en  los  tres  días  que  siguen  a  la  lesión  y  
1.   La  fuente  de  bacterias  que  causan  infecciones  en  las  heridas  limpias  más   progresa  rápidamente  
comúnmente  es:   b.   Lo  más  habitual  es  que  se  deba  a  un  retraso  desvascularizante  
a.   las  manos  del  cirujano   c.   Siempre  hay  crepitación  
b.   el  salón  de  operaciones   d.   Las  manifestaciones  locales  consisten  en  dolor  en  la  herida  con  
c.   instrumental  inadecuadamente  esterilizado   exudado   seropurulento   parduzco   y   edema   de   superficie,  
d.   el  paciente   necrosis   y   cambios   de   color   que   son   generalmente   menos  
2.   Todas   las   siguientes   situaciones   aumentan   el   riesgo   de   infecciones   del   extensos  que  la  necrosis  muscular  subyacente  
sitio  quirúrgico  luego  de  una  herniorrafia  electiva  excepto   e.   La   toxemia   es   profunda   e   incluye   taquicardia   grave,   delirio   e  
a.   Rasurar  el  sitio  operatorio  el  día  anterior  de  la  cirugía   ictericia  hemolítica  
b.   Colocar  drenaje  en  la  herida   7.   La   infección   nosocomial   adquirida   más   frecuentemente   dentro   de   un  
c.   No  continua  con  los  antibióticos  profilácticos  más  de  24  horas   hospital  es:  
a.   Herida  quirúrgica   15.  La  celulitis  plantea  diagnóstico  diferencial  con:  
b.   Tracto  respiratorio   a.   TROMBOFLEBITIS  
c.   Tracto  urinario   b.   ALERGIA  INTENSA  POR  CONTACTO  
d.   Lugar  de  inyección  intravenosa   c.   INFLAMACIÓN  QUÍMICA  POR  INYECCIÓN  DE  FÁRMACOS  
8.   Señale   qué   medidas   de   las   que   se   citan     es   menos   importante   para   d.   FASCITIS  NECROTIZANTE  
disminuir  la  incidencia  de  infecciones  hospitalarias   16.  Embolia  gaseosa  se  manifiesta  por:  
a.   Baño  pre-­‐operatorio  del  paciente  con  jabón  antiséptico   a.   TAQUIPNEA  SÚBITA  
b.   Lavado  de  manos  par  parte  del  personal  sanitario   b.   HIPOTENSIÓN  
c.   Aplicación  de  luz  ultravioleta  a  la  sala  de  operaciones   c.   SOPLO  DE  RUEDA  DENTADA    
d.   Reducción   al   mínimo   imprescindible   de   la   ambulación   y   el   17.  Los  signos  clásicos  de  flogosis  que  pueden  estar  presente  en:  
hablar  en  la  sala  de  operaciones.   a.   SEROMA  
9.   La  embolia  gaseosa  se  manifiesta  por:   b.   DEHISCENCIA  DE  LA  HERIDA  
a.   Taquicardia  súbita   c.   HEMATOMA  
b.   Hipotensión   d.   INFECCIÓN  DEL  SITIO  QUIRÚRGICO    
c.   Soplo  de  ruega  dentada   18.  Paciente   a   quién   se   le   infiltró   anestesia   local,   presenta   cuadro   de  
d.   Todos   convulsión.  El  anestésico  probablemente  se  utilizó  en  dosis  inadecuadas  
e.   Ninguno   fue:    BUPIVACAINA  
10.  Todas  menos  una  puede  ser  causa  de  fiebre  pos-­‐operatoria  en  un  caso  
electivo   MANEJO  DEL  PACIENTE  QUIRURGICO  
a.   Atelectasia  
1.   Usted  es  llamado  a  la  sala  por  un  paciente  de  80  años  que  fue  sometido  
b.   Infección  en  la  herida  
a   cirugía   abdominal.   La   enfermera   le   informa   que   está   en   su   3er   día  
c.   Metabolismo  basal  aumentado  
post-­‐operatorio   y   que   luego   de   haber   cenado   presenta   dificultad  
d.   Antibióticos  
respiratoria,   a   la   auscultación   se   escucha   sibilancias   difusas   en   ambos  
11.  Todos  aumentan  el  riesgo  de  infección  de  herida  quirúrgica  menos:  
campos  pulmonares  y  a  la  oximetría  resultó  saturación  baja  de  oxígeno.  
a.   Confianza   injustificada   en   la   eficacia   terapéutica   antibiótica  
Su  diagnóstico  más  probable  es:  
profiláctica.  
a.   neumotórax  
12.  Cuando   usted   tiene   un   paciente   con   una   infección   establecido   cuyo  
b.   broncoaspiración  
agente   patógeno   es   conocido,   el   paso   más   importante   para   tratar   al  
c.   embolismo  pulmonar  
paciente  es:  DAR  ANTIBIÓTICOS  INMEDIATAMENTE  
2.   Paciente  varón  de  75  años  al  cual  se  le  practicó   cirugía  reparadora  por  
13.  Con   respecto   a   los   antibióticos   profilácticos,   enumere   dos   condiciones  
hernia   inguinal   bajo   anestesia   epidural.   La   cirugía   fue   prolongada   y  
pre-­‐operatorias  en  que  esté  indicado  para  su  uso  
dificultosa.  En  el  post-­‐operatorio  desarrolla  una  masa  en  bajo  vientre.  La  
a.   POSIBILIDAD  DE  INFECCIONES  SON  ALTAS  
conducta  correcta  es:  
b.   INICIARSE  EL  DIA  ANTES  DE  LA  CIRUGÍA  
a.   llevarlo  nuevamente  al  salón  de  operaciones  
14.  Acerca   de   la   miositis   difusa   clostridial   (gangrena   gaseosa),   señale   el  
b.   realizarle  un  ultrasonido  de  urgencia  
signo  constante  en  la  gangrena  gaseosa:  CREPITACIONES  
c.   colocarle  sonda  urinaria  y  luego  retirarla   SÍ   Puede  tratarse  sin  toracostomía  con  sonda  cuando  los  síntomas  son  
d.   realizarle  CAT  abdominal  y  de  pelvis   leves.  
3.   En  un  paciente  politraumatizado  séptico  comienza  a  drenar  sangre  roja   NO   Habitualmente   tiene   una   causa   subyacente   grave   que   debe  
definirse.  
por   el   tubo   nasográstrico   y   evacuar   melena   en   el   sexto   día  
SÍ   La   probabilidad   de   recurrencia   es   de   50%   después   del   primer  
postoperatorio.  La  causa  más  frecuente  sería:   episodio.  
a.   Ulcera  de  Stress   SÍ   Habitualmente  el  único  tratamiento  es  toracotomía  con  soda.  
b.   Ulcera  de  la  mucosa  esofágica  gástrica   SÍ   Debe   considerarse   como   indicación   quirúrgica   si   la   fuga   de   aire  
c.   Presencia  de  cáncer  gástrico  no  reconocido   persiste  más  de    una  semana.  
d.   Presencia  de  úlcera  péptica  no  reconocida    
e.   Reacción  a  los  medicamentos   2.   El  timoma:  
4.   Lo  más  útil  para  evaluar  a  un  paciente  con  problemas  de  coagulación:   a.   Es  la  neoplasia  más  común  en  el  mediastino  posterior  
a.   Tiempo  de  sangría   b.   Puede  desarrollar  miastenia  gravis  
b.   Tiempo  de  coagulación   c.   Dan  metástasis  a  distancia  con  gran  frecuencia  
c.   Cuenta  de  plaquetas   d.   El  tratamiento  es  la  quimioterapia  
d.   Tiempo  parcial  de  tromboplastina   3.   Entre  los  tumores  del  mediastino  superior,  el  más  común  es:  
e.   Leucocitosis     a.   Tumor  de  células  germinales  
5.   Es  verdadero  en  relación  a  los  requerimientos  de  potasio  en  el  paciente   b.   Bocio  endotoráxico  
operado   c.   Timoma  
a.   Durante  las  primeras  24  horas  no  se  requiere  potasio.   4.   Cuáles  son  los  3  tumores    más  comunes  del  mediastino  anterior:  
6.   La  morbilidad  y  mortalidad  peri  operatoria  depende  de  la  interacción  de   a.   TIMOMA  
los  siguientes  factores:   b.   TERATOMA  
a.    Todos.   c.   BOCIO  INTRATORÁCICO  
7.   Temperatura   de   38°C   la   noche   de   haberse   operado   de   hernia   inguinal   5.   __F__   en   los   tumores   de   mediastino   posterior   es   particularmente  
sin   mayores   complicaciones.   La   causa   más   probable   de   la   fiebre:   importante  la  determinación  de  marcadores  tumores  
ATELECTASIA  PULMONAR   6.   __F__   los   tumores   neurogénicos   típicamente   se   encuentran   el  
8.   Causas  de  fiebre  postoperatoria   mediastino  anterior  
a.   ATELECTASIA  PULMONAR   7.   __F__  El  timoma  es  el  tumor  más  frecuente  de  mediastino  posterior  
b.   INFECCIÓN  DE  LA  HERIDA  QUIRURGICA    
c.   METABOLISMO  BASAL  AUMENTADO   8.   La   escogencia   entre   el   tratamiento   quirúrgico,   la   quimio   y   la  
9.   __C__  Durante  las  primeras  horas  post-­‐operatorias  la  causa  principal  de   radioterapia  en  el  tratamiento  del  cáncer  pulmonar  se  basa  en:  
fiebre  es  la  atelectasia   a.   Disponibilidad  de  estos  tratamientos  
b.   La  edad  del  paciente  
MEDIASTINO  Y  PATOLOGÍA  PULMONAR   c.   El  debido  estadiamiento  del  tumor  canceroso  
9.   El  Mesotelioma:  
1.   Afirmaciones  ciertas  respecto  a  neumotórax  espontáneo  incluyen  que:   a.   Puede  ser  maligno  o  benigno  
b.   El  tratamiento  es  cirugía  y  generalmente  es  curativa   c.   Neumonía  neumocócica.  
c.   Generalmente   es   multicéntrico   y   su   tratamiento   es   d.   Neumonía  por  Pneumocystis.  
controversial   e.   Tuberculosis.  
d.   En  el  estadio  IV  el  tratamiento  de  elección  es  la  cirugía  
16.  Mencione  3  síndromes  paraneoplásicos  del  cáncer  broncogénico:  
10.  En  un  paciente  con  empiema  post-­‐neumónico  en  fase  fibrino-­‐purulenta:  
a.   El  tratamiento  de  elección  es  el  uso  de  fibrinolíticos   a.   SÍNDROME  DE  CUSHING  
b.   Los   antibióticos   y   un   drenaje   pleural   son   el   tratamiento   de   b.   SÍNDROME  DE  SECRECIÓN  INAPROPIADA  DE  ADH  
elección   c.   HIPERCALCEMIA  
c.   La  tomografía  computada  de  tórax  no  es  útil  en  esta  etapa  del   17.  __C__   La   broncoscopia   es   la   modalidad   invasiva   más   valiosa   en   el  
empiema     diagnóstico  de  la  enfermedad  pulmonar  
d.   La  video  toracoscopía  es  el  tratamiento  indicado   18.  __C__  el  tabaco  es  el  principal  responsable  del  cáncer  de  pulmón  
11.  Usted  evalúa  un  paciente  de  77  años  de  edad,  emaciado  y  cardiópata,  al  
19.  __F__La   escogencia   entre   le   tratamiento   quirúrgico,   la   quimio   y   la  
que   se   le   diagnostica   por   imágenes   empiema   crónico.   ¿Cuál   sería   el  
mejor  tratamiento  para  este  paciente?   radioterapia  en  el  tratamiento  del  cáncer  pulmonar  se  basa  en  la  edad  
a.   Toracotomía  y  decortización   del  paciente  
b.   Solo  antibióticos  
c.   Drenaje  abierto  y  resección  de  costilla   MORDEDURAS  
d.   Toracosentésis  y/o  toracotomía  cerrada  
12.  Un  señor  previamente  en  buen  estado  de  salud,  súbitamente  presenta   1.   Todo  lo  anterior  es  cierto  con  respecto  o  a  las  lesiones  por  mordeduras  
cianosis,  afasia  y  estridor.  Cuál  es  el  diagnóstico  más  probable   humanas  excepto:  
a.   Espasmo  bronquial   a.   En  las  manos  tienen  secuelas  graves  como  infección,  pérdida  de  
b.   Taquicardia  con  isquemia  cerebral   función  y  amputación  si  no  se  diagnostican  y  se  tratan.  
c.   Cuerpo  extraño  en  las  vías  aéreas  
b.   Sospecharla   siempre   que   hay   lesión   en   el   dorso   de   la   región  
d.   Neumotórax  espontaneo  
13.  En  el  neumotórax  espontáneo:   metacarpofalangica  del  puño  
a.   La  aparición  de  nuevos  episodios  es  poco  frecuente   c.   La  automordedura   en   la   lengua   no  se   considera   de  este   grupo  
b.   Generalmente   se   presenta   en   pacientes   altos,   delgados   y   ya  que  es  la  flora  bacteriana  propia  del  huésped  
jóvenes   d.   En   la   lesión   genuina   el   agresor   clava   sus   dientes   en   la   víctima  
c.   El   neumotórax   catamenial   se   ve   con   más   frecuencia   en   produciendo   heridas   produciendo   heridas   por   punción,  
pacientes  post-­‐menopáusicas   desgarros  o  desprendimiento  de  tejidos  
d.   No  se  produce  neumotórax  a  tensión  
e.   Las   lesiones   por   puñetazos   deben   ser   consideradas   como  
14.  Cuando  se  produce  neumotórax  a  repetición:  
a.   Se  indica  cirugía   mordeduras  
b.   Indica  EPOC   2.   Las  mordeduras  humanas  son  
c.   El  tratamiento  de  elección  es  toracosentésis   a.   Son  más  frecuentes  que  las  mordeduras  por  perros  
d.   El  tratamiento  inicial  no  fue  adecuado     b.   Se  consideran  más  infectadas  que  las  mordeduras  de  gato  
15.  La  causa  más  común  de  absceso  pulmonar  es:   c.   Más  frecuentes  en  personas  mayores  de  50  años  
a.   Aspiración.   d.   Se  consideran  heridas  contuso-­‐punzantes  
b.   Obstrucción  bronquial  por  tumor.   e.   Se  localizan  más  frecuentemente  en  las  extremidades  inferiores  
3.   Todo   lo   anterior   es   cierto   en   relación   al   riesgo   de   infección   de   las   OFIDIOS  
lesiones  por  mordedura  humana,  EXCEPTO  
a.   Tiene   alto   riesgo   de   infección   cuando   compromete   1.   Un  paciente  de  45  años  en  el  cuarto  de  urgencias,  12  horas  después  de  
articulaciones  como  la  metacarpofalangica   accidente  de  ofidio  (Verrugosa)  presenta:  
b.   Edad  mayor  de  50  años,  diabetes,  alcoholismo,  son  factores  de   a.   Petequias,  hematuria,  hemoptisis  
riesgo  asociados  a  mayor  riesgo   2.   En   la   evaluación   de  la   severidad   de   un   cuadro   clínico   de   un   accidente  
c.   Tiempo  de  evolución  mayor  de  12  horas  está  asociado  a  mayor   ofídico  de  III  grado  son:  
riesgo   a.   SE  OBSERVAN  MARCAS  DE  COLMILLOS  
d.   Un   75%   de   las   mismas   ocurren   en   actos   agresivos   y   b.   EDEMA  Y  ERITEAMA  EN  EL  AREA  DE  LA  LESIÓN  (MAYOR  DE  25  
principalmente  en  adolescentes  y  adultos  jóvenes   CM)  
e.   Son  ciertas  todas  las  anteriores   c.   DOLR  MARCADO  EN  EL  ÁREA  DE  LA  LESIÓN  
4.   El  antibiótico  profiláctico  de  elección  para  las  mordeduras  humanas,  de   d.   COMPROMISO  SISTÉMICO:  PETEQUIAS,  EQUIMOSIS,  SANGRADO  
no  existir  alergias  es  el  siguiente   GENERALIZADOS,  NAUSEAS  
a.   Clindamicina  +  Ciprofloxacina   e.   ALTERACIONES   EN   LAS   PRUEBAS   DE   LABORATORIO:   AUMENTA  
b.   Trimetropin  +  Sulfamotoxazol   TP,  TPT,  TIEMPO  DE  SANGRIA  Y  DISMINUYEN  PLAQUETAS.  
c.   Amixicilina  +  ácido  clavulánico   3.   Las   manifestaciones   clínicas   y   la   gravedad   del   envenenamiento   por  
d.   Moxifloxacina   ofidios  (Botrops)  depende  de  cuatro  factores  principales  como:  
e.   Doxiciclina   a.   CANTIDAD  DE  VENENO  INOCULADO  
5.   Todo   lo   anterior   es   cierto   en   relación   a   las   mordeduras   humanas   b.   EDAD  Y  TAMAÑO  DE  LA  VÍCTIMA  
excepto   c.   SENSIBILIDAD  DE  LA  VÍCTIMA  AL  VENENO  
a.   Las   mordeduras   en   las   manos   con   menos   de   6   horas   de   d.   TIEMPO   DE   EVOLUCIÓN   DESDE   EL   ACCIDENTE   HASTA   LA  
evolución   se   suturan   previa   irrigación   con   SSN   y   con   su   ATENCIÓN  
respectiva  terapia  antibiótica   4.   En  Panamá  tenemos  3  familias    de  ofidios.  Enumere  estas  tres  familias  y  
b.   Siempre  se  les  debe  realizar  radiografías   un  ejemplo  “Genero”  de  cada  uno  
c.   Siempre  se  debe  administrar  antibióticos  profilácticos   a.   ELAPIEDAE,  Micrurus  
d.   Las  mordeduras  en  las  manos  no  se  suturan   b.   HIDROPHIIDAE,    
e.   El   método   preferido   para   cerrar   una   herida   de   la   mano   es   el   c.   VIPERIDAE,  Botrops  asper  
cierre  primario  diferido  o  la  cicatrización  por  segunda  intención.   5.   El  ministerio  de  salud  de  Panamá  recomienda  el  siguiente  esquema  de  
6.   Tres   días   después   de   haber   sido   mordido   por   un   perro   que   tiene   sus   tratamiento  en  accidente  ofídico:  
vacunas  completas,  la  mano  del  niño  está  edematosa  y  roja,  con  severa   a.   GRADI  I:  4  –  5  VIALES  (40  –  50  ml)  
celulitis,  tiene  además,  un  drenaje  purulento  por  la  herida.  Al  momento   b.   GRADO  II:  6  –  8  VIALES  (60  –  80  ml)  
del   examen   hay   algo   de   linfangitis   y   linfadenopatías   epitrocleares   y   c.   GRADO  III:  9  –  12  VIALES  (90  –  120  ml)  
axilares   dolorosas.   En   combinación   con   el   drenaje   quirúrgico,   qué   d.   GRADO  IV:  MAS  DE  12  VIALES  (MAS  DE  120  ml)  
antibiótico   probablemente   sería   de   primera   elección:   AMOXICILINA  
CLAVULANATO  
PACIENTE   QUIRURGICO   EN   UCI,   NUTRICIÓN,   LIQUIDOS   Y   d.   Gas  bajo  el  diafragma  
6.   _C_  La  hipokalemia  en  la  obstrucción  duodenal  con  vómitos  se  debe  a:  
ELECTROLITOS   a.   Pérdida  de  saliva  
b.   Pérdida  de  jugo  gástrico  
Seleccione  su  respuesta  en  la  siguiente  forma:   c.   Pérdida  de  jugo  pancreático    
  d.   Sustitución  del  K+  por  H+  en  el  riñón    
A    si  1,  2  y  3  son  correctas    
B    si  1  y  3  son  correctas   7.   Las  causas  de  hipokalemia  clínica  incluyen  
C    si  2  y  4  son  correctas  
a.   Vómitos  
D    si  solo  4  es  correcta  
E    si  todas  son  correctas   b.   Ileo  
  c.   Uso  prolongado  de  diuréticos  a  base  de  Clorotiazida  
1.   _A_   Es   verdadero   en   relación   a   los   requerimientos   de   potasio   en   el   d.   Constipación  
paciente  post-­‐operado:     e.   A,  B  y  C  
a.   Durante  las  primeras  24  horas  no  requiere  potasio   8.   La  composición  electrolítica  del  lactato  ringer,  recuerda  la  composición  
b.   La  función  renal  debe  ser  adecuada  antes  de  administrar  K+   electrolítica  de:  
c.   De  30  a  40  mEq  son  los  requerimientos  diarios  
a.   Orina  en  un  paciente  normalmente  hidratado  
d.   Estimulación   a   la   eliminación   de   K+   es   necesario   antes   de  
administrarlo   b.   Pérdida  insensible  de  líquidos  
2.   _B_  El  calcio  juega  un  papel  importante  en:   c.   Líquido  intracelular  
a.   Excitabilidad  de  la  función  nerviosa     d.   Líquido  extracelular  
b.   En   la   contractilidad   del   músculo   tanto   esquelético   como   9.   El  desarrollo  de  hiponatremia  post-­‐operatoria  es  generalmente  causada  
cardíaco     por:  
c.   Tiene  función  en  los  orgánulos  y  membranas  celulares  
a.   Enfermedad  renal  
d.   Ayuda  en  la  liberación  de  hormonas  
3.   _E_  Se  observa  como  complicación  de  la  alimentación  parenteral:   b.   Administración  inadecuada  de  líquidos  
a.   Hiperglicemia   c.   Hipoadrenalismo  
b.   Glucosuria   d.   Diabetes  insípida  oculta  
c.   Coma  hiperosmolar   e.   Succión  gástrica  
d.   Septicemia   10.  Un  paciente  con  hipernatremia  postoperatoria,  que  ocasionalmente  se  
4.   _E_  Son  características  de  la  obstrucción  intestinal:   presenta   después   de   habérsele   administrado   algunos   agentes  
a.   Dolor  tipo  cólico  
anestésicos,  debe  ser  tratado  con  la  aplicación  de:  
b.   Vómitos  
c.   Distensión  intestinal   a.   Cortisol  
d.   No  pasaje  de  gas  o  heces  por  el  ano   b.   Vasopresina  
5.   _A_   En   el   estudio   radiográfico   en   el   abdomen   en   la   obstrucción   c.   Ácido  etacrínico  
intestinal:   d.   Dextrosa  en  agua  al  5%  
a.   Hay  gas  en  el  intestino  delgado   11.  En  relación  a  la  glutamina,  podemos  decir  que  es  un:  
b.   Niveles  líquidos   a.   Aminoácido  no  esencial  (no,  según  Fuentes)  
c.   Ausencia  de  gas  en  el  colon  
b.   Ácido  graso  no  esencial  
c.   Aminoácido   esencial   (según   Matos   que   dio   la   clase,   es   no   operación   es   muy   probable   que   el   paciente   presente   una   de   las  
esencial  que  se  vuelve  esencial  durante  el  trauma)   siguientes  anomalías  metabólicas:  
d.   Ácido  graso  esencial   a.   Hipopotasemia.  
12.  Entre   las   complicaciones  por  nutrición  parenteral   se  incluyen   todas  las   a.   Hiponatremia.  
siguientes,  EXCEPTO:   b.   Hipernatremia.  
a.   Incremento  en  el  riesgo  de  infección   c.   Hipocloremia.  
b.   Hiperglicemia   d.   Hipoglicemia.  
c.   Infiltración  grasa  del  hígado   17.  El   paciente   tiene   mayor   probabilidad   de   presentar   ésta   anomalía  
d.   Producción  excesiva  de  CO2   metabólica  debido:  
13.  Un   método   práctico   recomendado   para   el   balance   hídrico   de   un   a.   Suministro  inadecuado  de  dextrosa.  
paciente  en  el  post-­‐operatorio  es:   b.   Administración  de  líquidos  hipotónicos.  
a.   estimar  el  agua  del  cuerpo   c.   Secreción  inapropiada  de  hormona  antidiurética.  
b.   presión  arterial   d.   Pérdida  de  colon.  
c.   frecuencia  de  pulso   e.   Secreción  de  aldosterone  
d.   pesarlo  diariamente   18.  Luego  de   inyectar   2000  mL   de   líquidos  IV,   después   de   traumatismo,  la  
14.  La  guía  más  útil  para  determinar  el  reemplazo  adecuado  de  líquidos  es   presión  arterial  del  paciente  es  110/70  mmHg.  ¿Cuál  de  los  siguientes  
a.   La  diuresis   datos   sería   la   mejor   guía   clínica   para   una   reanimación   adecuada   con  
b.   El  retorno  de  la  conciencia   líquidos?  
c.   El  llenado  capilar  de  las  uñas   a.   Excreción  urinaria  >  de  30  mL/h.  
d.   Gases  arteriales  normales   b.   Reducción  de  la  frecuencia  de  pulso  <  lpm.  
e.   El  desarrollo  de  leve  edema  pulmonar   c.   Retorno  de  la  temperatura  normal  de  la  piel.  
15.  En  un  hombre  de  70  Kg.  El  volumen  de  agua  intracelular  es  de:   d.   Retorno  de  la  precepción  sensorial  normal.  
a.   3.5  L   e.   Desaparición  de  la  hipotensión  ortostática.  
b.   5  L   19.  En   la   terapéutica   del   choque   hemorrágico,   el   mejor   signo   clínico   para  
c.   10.5  L   juzgar  si  la  reposición  de  líquido  tiene  éxito  es:  
d.   28  L   a.   Incremento  de  presión  arterial  
e.   42  L   b.   Incremento  en  la  excreción  de  orina  
16.  Un  hombre  de  65  años  de  edad,  sufre  una  resección  abdominoperineal   c.   Incremento  de  oxigenación  arterial  
por  carcinoma  rectosigmoideo  sin  complicaciones  durante  la  operación.   d.   Disminución  de  la  sed  
La  sangre  perdida  se  repuso  volumen  a  volumen  y  la  excreción  de  orina   e.   Reducción  de  la  taquicardia  
se  mantuvo  en  20  a  30  mL/h.  durante  el  post-­‐operatorio  se  administró   20.  Cuál   de   las   siguientes   medidas   se   utilizan   en   el   tratamiento   de   los  
al   paciente   líquido   en   cantidad   apropiada   para   restituir   jugo   gástrico   y   pacientes  con  hiperpotasemia  aguda  
conservarlo  bien  hidratado,  también  se  le  suministró  dextrosa  al  5%  en   a.   Administración  de  glucosa  e  insulina  
SSN  al  0.45%  a  una  tasa  de  150  mL/h.  veinticuatro  horas  después  de  la   b.   Administración  de  Lactato  Ringer  
c.   Administración  de  resinas  de  intercambio  iónico  por   vía  oral  o   c.   Falla  renal  
rectal   d.   Vómitos  
d.   Infusión  de  soluciones  hipotónicas  de  Sodio   e.   Fístula  del  intestino  delgado  
e.   A  y  C   27.  La  hipomagnesemia  se  observa  en  todo  lo  siguiente,  MENOS:  
21.  la  alcalosis  metabólica  está  asociada  con:   a.   Alcoholismo  crónico  
a.   Hiperventilación   b.   Hipoparatiroidismo  
22.  Consecuencia  de  administrar  exceso  de  proteínas   c.   Pancreatitis  
a.   Hiperamonemia   d.   Quemaduras  
b.   Elevación  de  creatinina  y  BUN.   e.   Cetosis  diabética  
c.   Oliguria   28.  No  está  indicada  la  nutrición  parenteral  total  en:  
d.   Falla  pulmonar   a.   Paciente  en  coma  y  ECV  
e.   Esteatosis  hepática   b.   Paciente  con  fístula  enterocutanea  o  pérdida  de  60  cc  
23.  Entre   las   complicaciones   por   alimentación   calórica   excesiva   en   el   c.   Pacientes  con  síndrome  de  intestino  corto  
paciente  nutrición  parenteral  se  incluyen  todas  las  siguientes,  excepto   d.   Paciente  con  cáncer  gástrico  obstructivo  
a.   Incrementa  el  riesgo  de  infección   e.   Paciente  con  colitis  ulcerativa  
b.   Hiperglicemia   29.  Ejemplo  de  oligoelementos:  
c.   Infiltración  grasa  del  hígado   a.   Cobre  
d.   Producción  excesiva  de  CO2   30.  Ventaja  de  nutrición  enteral  sobre  parenteral:  
24.  Consecuencia  de  administrar  exceso  de  carbohidratos   a.   mantiene  la  función  inmunológica.  
a.   Hiperglicemia   31.  Indicación  de  nutrición  enteral  
b.   Hipercapnia   a.   Intestino  viable.  
c.   Hígado  graso   32.  Contraindicación  de  terapia  nutricional  
d.   Hipertrigliceridemia   a.   Paciente  politraumatizado  con  FC:  120x’,  PA:  82/40.  
e.   Todas  las  anteriores   33.  Paciente  masculino  de  70  kg  recibe  dextrosa  al  5%  a  100  cc  para  darle  
25.  Las   medidas   para   tratar   la   Hiperkalemia   incluye   todo   lo   siguiente,   algo  de  calorías.  En  realidad  cuantas  calorías  recibe?:  
MENOS:   a.   408  calorías.  
a.   Evitar  la  administración  de  potasio  exógeno   34.  Paciente   de   60   kg,   con   gastrectomía   hace   6   días,   tiene   peristaltismo  
b.   Administrar  glucosa  e  insulina   intestinal,  decide  dar  nutrición  enteral  con  formula  enteral:  100  cc,  20  g  
c.   Kayexalate   de  proteínas,  10  g  de  lípidos,  30  g  de   carbohidratos   está  a   100  cc  por  
d.   Administración  de  Mg   hora,   recibe   nutrición   5:1.   Que   volumen   de   la   formula     recibe   en   24  
e.   Diálisis     horas?  
26.  Las   causas   de   Acidosis   metabólicas   incluyen   todas   las   siguientes,   a.   2000  cc  a  40.  
MENOS:   35.  Cuantos  gramos  de  N2  representa  esas  proteínas  ,  y  cuantos  gramos  de  
a.   Diabetes   carbohidratos?.  
b.   Inanición   a.   N2  a  6.4  CHO  60.  
36.  Llega  el  resultado  de  nitrógeno  egresado  en  2  4horas,  10  gramos,  como   51.  __C__   proteína   ligadora   de   retinol   con   valor   de   2.1   mg/dL   es  
esta  ese  balance?   desnutrición  severa  
a.   Negativo.   52.  __F__  la  desnutrición  hospitalaria  en  promedio  corresponde  a  30%  
37.  Cuál   es   el   método   que   usted   considera   más   práctico   para   el   balance  
hídrico  de  un  paciente  pos-­‐operado:  PESAR  AL  PACIENTE  DIARIAMENTE   PANCREAS  
38.  Enumere   tres   complicaciones   por   alimentación   calórica   excesiva   en   el  
1.   En   la   pancreatitis   aguda,   cuál   de   los   siguientes   factores   identificado  
paciente  con  soporte  nutricional  
desde   el   momento   de   la   admisión   o   en   las   primeras   48   horas   de  
a.   HIPERGLICEMIA   hospitalización  indica  ataque  grave  y  diagnóstico  sombrío:  
b.   INCREMENTO  EN  EL  RIESGO  DE  INFECCIÓN   SÍ   Déficit  de  base  de  6  mEq/L.  
c.   PRODUCCIÓN  EXCESIVA  DE  CO2   SÍ   Leucocitosis  de  20,000/mm3.  
39.  Mencione  2  funciones  de  líquidos  en  nuestro  organismo:   SÍ   Concentración  de  glucosa  en  plasma  de  250mg/100  mL.  
a.   TRANSPORTE   DE   NUTRIENTES   Y   DESECHOS   DESDE   Y   HACIA   LA   SÍ   Concentración  sérica  de  calcio  de  7  mg/100  mL.  
CÉLULA   SÍ   PO2  arterial  de  55  mmHg  
 
b.   MEDIO  DE  REACCIONES  BIOQUÍMICAS  
Seleccione  su  respuesta  en  la  siguiente  forma:  
40.  Cite   2   funciones   terapéuticas   en   la   prescripción   de   soluciones    
parenterales  para  el  tratamiento  de  alteraciones  hidroelectrolíticas   A    si  1,  2  y  3  son  correctas  
a.   ¿???   B    si  1  y  3  son  correctas  
b.   ¿???   C    si  2  y  4  son  correctas  
41.  Cite  una  función  principal  del  líquido  en  nuestro  organismo   D    si  solo  4  es  correcta  
E    si  todas  son  correctas  
a.   ¿???  
 
42.  __F__  Una  de  las  manifestaciones  clínicas  de  intoxicación  hídrica  son  las   2.   _A_  En  el  diagnóstico  de  la  pancreatitis,  es  importante:  
convulsiones  
a.   Historia  de  alcoholismo  
43.  __C__   La   pérdida   de   peso   pre-­‐operatorio   indica   alto   riesgo   post-­‐ b.   Dolor  en  epigastrio  difuso  en  cinturón  
operatorio   c.   Elevación  de  las  amilasas  
44.  __C__  Pacientes  desnutridos  presentan  más  infecciones  nosocomiales  
3.   _A_  En  la  pancreatitis  aguda  se  encuentra  elevada:  
45.  __C__  La  desnutrición  se  puede  definir  como  pérdida  de  peso  mayor  de   a.   Amilasa  en  suero  
10%  en  6  meses   b.   Amilasa  urinaria  
46.  __C__  La  desnutrición  se  puede  definir  como  pérdida  de  peso  mayor  de  
c.   Lipasa  en  suero  
5%  en  1  mes    
47.  __C__   perder   entre   el   10   al   20%   del   peso   se   considera   desnutrición   4.   Cuál  de  los  siguientes  indica  un  pobre  pronóstico  en  pancreatitis?  
moderada  
a.   Los  niveles  de  amilasa  en  el  suero.  
48.  __C__  valor  normal  de  albúmina  sérica  es  de  3.5-­‐5  mg/dL   b.   Hiperglicemia  y  glucosuria.  
49.  __F__  La  pre  albumina  es  un  precursor  de  la  albumina   c.   Niveles  elevados  de  amilasa  en  orina.  
50.  __C__  la  vida  media  de  la  albumina  es  de  20  días   d.   Tiempo  de  coagulación  elevados.  
e.   Disminución  de  la  calcemia.   e.   SGOT  
5.   En  Panamá,  la  primera  causa  de  pancreatitis   10.  Todo   lo   anterior   es   cierto   acerca   de   la   pancreatitis   necrotizante,  
a.   Parásitos   excepto:  
b.   Biliar   a.   Es  una  enfermedad  grave  caracterizada  por  necrosis  glandular  y  
c.   Alcoholismo   una  respuesta  inflamatoria  sistémica  destructiva  
6.   Ante  un  caso  de  pancreatitis  aguda  (PA),  ¿en  qué  situación  consideraría   b.   El  tratamiento  precoz  es  quirúrgico  en  primera  instancia  y  es  de  
indicada  la  CEPRE?:   menor   importancia   las   medidas   enérgicas   de   reanimación   y  
a.   Pancreatitis  aguda  severa  en  fase  aguda   soporte  
b.   Pancreatitis  aguda  recurrente  de  origen  no  esclarecido   c.   Los   resultados   están   determinados   de   manera   primaria   por   la  
c.   Pancreatitis  aguda  persistente  durante  más  de  una  semana   presencia   de   infección   bacteriana   secundaria   tardía   de   la  
d.   Todas   glándula  necrótica  
7.   Un   varón   de   40   años   de   edad   con   antecedentes   de   abuso   de   d.   Los   antibióticos   empíricos   tempranos   y   la   necrosectomía  
alcoholismo  se  presenta  después  de  una  borrachera,  se  queja  de  dolor   quirúrgica   tardía   en   el   contexto   apropiado   son   claves   para   el  
en  el  epigastrio  con  irradiación  a  la  espalda  acompañado  de  náuseas  y   tratamiento  de  estos  pacientes  graves  
vómitos.  A  la  exploración  física  tiene  marcado  dolor  en  el  epigastrio  con   e.   Con   terapia   apropiada   se   puede   minimizar   la   mortalidad   y   es  
defensa,  disminución  de  los  ruidos   aéreos   y   distención   abdominal.   Las   posible  restaurar  la  calidad  de  vida  a  largo  plazo  
placas   del   abdomen   demuestran   dilatado   intestinal   del   hemiabdomen   11.  Permite  establecer  el  pronóstico  en  la  pancreatitis  aguda,  excepto  
superior.  El  diagnóstico  más  probable  es:   a.   Criterios  de  Ranson  
a.   Úlcera  péptica  perforada   b.   Sistema  ASA  
b.   Colecistitis  aguda   c.   Criterios  de  Glasgow  
c.   Pancreatitis  aguda   d.   APACHE  II  
d.   Divertículo  del  sigmoides  perforado   e.   Criterios  de  Baltasar  
8.   El   fármaco   que   se   ha   mostrado   eficaz   en   pacientes   con   pancreatitis   12.  __C__   El   pseudoquiste   de   páncreas   no   debe   ser   drenado   cuando   es  
aguda:   diagnosticado  
a.   Somatostatina   13.  Un   varón   de   40   años   de   edad   con   antecedentes   de   abuso   de  
b.   Anticolinérgicos   alcoholismo  se  presenta  después  de  una  borrachera,  se  queja  de  dolor  
c.   Glucagon   en  el  epigastrio  con  irradiación  a  la  espalda  acompañado  de  náuseas  y  
d.   Morfina   vómitos.  A  la  exploración  física  tiene  marcado  dolor  en  el  epigastrio  con  
e.   Ninguno   defensa,  disminución  de  los  ruidos  aéreos  y  distención  abdominal.  Las  
9.   De  los  signos  pronósticos  de  Ranson  para  valoración  de  la  gravedad  de   placas   del   abdomen   demuestran   dilatado   intestinal   del   hemiabdomen  
pancreattis  aguda,  cuál  se  documenta  en  el  momento  del  ingreso?   superior.  El  diagnóstico  más  probable  es:  PANCREATITIS  AGUDA    
a.   BUN   14.  En  panamá,  cuál  es  la  primera  causa  de  pancreatitis:  BILIAR  
b.   Calcemia   15.  ¿Qué   es   lo   que   indica   un   pobre   pronóstico   en   una   Pancreatitis?  
c.   PaO2   DISMINUCIÓN  DE  LA  CALCEMIA  A  MENOS  DE  8.  
d.   Déficit  de  Base  
16.  La  presencia  de  ictericia  y  vesícula  biliar  palpable  diferencia  al  cáncer  de   NO   En  mujeres  de  30  años  de  edad  debe  practicarse  un  estudio  basal.  
la:  CABEZA  Y  COLA  DE  PANCREAS   NO   En   ese   estudio   se   emplea   menor   radiación   que   en   una   radiografía  
17.  Pancreatitis  aguda:   de  tórax.  
a.   IDIOPÁTICA   NO   La   mamografía   es   un   sustituto   eficiente   de   la   biopsia   en   lesiones  
b.   BILIOSA   sospechosas.  
c.   ALCOHOLICA   NO   Técnicas  termográficas  mejoradas  representan  un  sustituto  seguro  y  
18.  En  el  diagnóstico  de  la  Pancreatitis  en  importante:   eficiente  para  la  detección  mamográfica.  
a.   HISTORIA  DE  ALCOHOLISMO.   SÍ   El   estudio   debe   ser   parte   de   la   vigilancia   regular   de   la   paciente  
b.   DOLOR  EPIGÁSTRICO  DIFUSO  EN  CINTURÓN.   después  de  terapéutica  por  cáncer  mamario  unilateral.  
c.   ELEVACIÓN  DE  LA  AMILASA.    
19.  Un   paciente   de   50   años   se   presenta   al   hospital   con   fiebre   alta,  
3.   Afirmaciones  correctas  respecto  al  cáncer  de  mama,  incluyen:  
escalofríos,   tos   y   dolor   en   el   hipocondrio   derecho   e   ictericia,   acolia   y  
coluria.  Cuáles  serían  sus  diagnósticos  diferenciales   SÍ   La  incidencia  de  cáncer  mamario  en  mujeres  ha  crecido  lentamente.  
a.   COLEDOCOLITIASIS   SÍ   En  la  actualidad,  el  cáncer  de  mama  es  la  principal  causa  de  muerte  
b.   COLANGITIS   relacionada  con  cáncer  en  las  mujeres  panameñas.  
NO   Cifras   de   sobrevivencia   para   etapas   comparables   de   cáncer  
c.   CÁNCER  DE  CABEZA  DE  PANCREAS  
mamario   son   más   altas     en   nuestras   mujeres   mulatas   si   se  
comparan  con  nuestras  mujeres  blancas.  
PATOLOGÍA  DE  LA  MAMA   NO   Personas   con   dietas   ricas   en   grasas   insaturadas   muestran   riesgos  
significativos  mayores  de  padecer  carcinoma  mamario.  
1.   Relacionado  con  el  fibroadenoma,  son  afirmaciones:  
SÍ   Las   monjas   presentan   incidencia   mayor   de   contraer   cáncer  
SÍ   Puede   observarse   lesión   única   de   2   centímetros,   que   clínicamente   mamario.  
corresponde   a   fibroadenoma   en   las   mamas   de   una   mujer   de   20    
años  de  edad.   4.   Afirmaciones  verdaderas  respecto  a  la  mastitis  aguda,  incluyen:  
NO   Cuando   se   observa   una   lesión   que   clínicamente   corresponde   a  
fibroadenoma  en  las  mamas  de   una  mujer  de  23  años  de  edad,  se   SÍ   Ocurre  con  mayor  frecuenta  en  mujeres  en  lactación.  
debe  practicar  mamografía.   SÍ   Es  más  común  en  clima  caluroso.  
NO   Es  extremadamente  dolorosa.  
SÍ   A   veces   no   es   posible   distinguir   entre   un   fibroadenoma   juvenil  
SÍ   El  agente  causal  más  común  es  Staphylococcus  aureus.  
voluminoso  de  un  tumor  filodes.   SÍ   El   tratamiento   inicial   debe   ser   con   Penicilina   o  
NO   La  superficie  de  corte  del  fibroadenoma  típico  se  retrae  a  lo  largo  de   Cefalosporina.  
la  línea  de  corte.   5.   Usted   examina   a   una   de   45   años,   quien   se   queja   de   una   masa   no  
SÍ   Un   fibroadenoma   quizás   sea   doloroso   e   hiperestésico   durante   el   dolorosa   en   la   mama   izquierda.   Cuál   de   estas   cosas   le   gustaría  
embarazo.   preguntar   en   la   historia   clínica   de   esta   paciente   en   orden   de  
  importancia  
2.   Cuáles   de   las   siguientes   afirmaciones   acerca   de   la   mamografía   son   A.   Previas  masas  en  las  mamas  
correctas:   B.   Descarga  por  el  pezón  
C.   Cambio  de  peso   tricomoniasis   y   procede   a   tratarla   con   Flagyl;   además   le   solicita   una  
D.   Trauma  en  mamas   mamografía   de   rutina.   ¿Considera   usted   que   está   indicada   la  
E.   Preñeces   mamografía  de  rutina?  
F.   Historia  familiar   a.   Sí.  
G.   Anticonceptivos   b.   No.  
H.   Historia  menstrual   9.   ¿Cuáles   serían   los   resultados   más   probables   en   esta   mamografía,   en  
I.   Menaquia   relación  con  las  fases  del  ciclo  mestrual?  
J.   Operaciones  previas   a.   Fase  luteínica:  
__F__   i.   Mayor  número  de  mamografía  falso  negativo.  
__G__   ii.   Menor  número  de  mamografía  falso  negativo.  
__B__   b.   Fase  folicular:  
__C__   i.   Menor  número  de  mamografía  falso  negativo.  
6.   Al   examen   físico   se   encuentra   una   masa   de   2   cm   en   el   cuadrante   ii.   Mayor  número  de  mamografía  falso  negativo.  
superoexterno   de   la   mama  izquierda.   El   movible   y  la   piel   sobre   ella   es   10.  El  riesgo  de  cáncer  de  mama,  en  la  mama  restante  es  mayor  en:  
normal.  No  se  palpan  nódulos  en  la  axila  ni  en  el  cuello.  La  otra  mama  es   a.   Comedocarcinoma.  
normal.  No  se  palpa  el  hígado.  Cuál  de  los  siguientes  usted  le  ordenaría   b.   Carcinoma  inflamatorio.  
en  orden  de  importancia?   c.   Carcinoma  lobular.  
A.   Ordena  una  mamografía   d.   Enfermedad  de  Paget.  
B.   Le  pide  regresar  pasado  el  periodo  menstrual   e.   Carcinoma  ductal.  
C.   Obtiene  una  biopsia  por  aguja   11.  El  tipo  más  frecuente  de  cáncer  de  mama  es:  
D.   Inicia  tratamiento  con  Bromocriptina  (Parloled)   a.   Carcinoma  papilar  infiltrante.  
E.   Ordena  ultrasonografía  de  la  mama  izquierda   b.   Carcinoma  ductal  infiltrante.  
F.   Le  dice  a  la  paciente  que  es  una  mama  benigna   c.   Carcinoma  medular.  
__C__   d.   Carcinoma  coloide.  
7.   En  este  momento,  cuáles  de  las  siguientes  considerar  pedir   e.   Carcinoma  lobular.  
A.   Centelleo  óseo   12.  El  área  donde  se  observa  más  frecuentemente  el  cáncer  mamario  es:  
B.   Hemograma  completo   a.   Cuadrante  superior  interno  
C.   Electrolitos   b.   Cuadrante  superior  externo  
D.   Radiografía  de  Tórax   c.   Cuadrante  inferior  interno  
E.   Centelleo  hepático   d.   Cuadrante  inferior  externo  
F.   Calcemia   13.  Sangramiento  por  el  pezón  
__D__   a.   Está  asociado  a  mailignidad  en  10  al  30%  de  los  casos  
__A__   b.   Puede  ser  debido  a  un  papiloma  intraductal  o  carcinoma  ductal  
8.   Paciente  femenina  de  44     años   de   edad   que   hace   uso   de   estrógenos,   c.   Puede  ser  debido  a  múltiples  lesiones  
consulta  a  su  ginecólogo  por  secreción  vaginal.  Su  médico  le  diagnostica   d.   Se  trata  con  hormonas  
e.   Se  trata  con  mastectomía   21.  Enumere  cuatro  factores  en  el  desarrollo  del  cáncer  de  mama:  
14.  La  mastitis  aguda  ocurre  con  frecuencia  en   a.   HIPERPLASIA  
a.   La  pubertad   b.   BRCA1  y  BRCA2  
b.   La  preñez   c.   ENFERMEDAD  FIBROQUISTICA  
c.   En  la  menopausia   d.   FIBROADENOMA  
d.   Lactancia   22.  Factores  de  riesgo  para  el  desarrollo  de  Cáncer  de  Mama:  
e.   Al  nacer   a.   EDAD  
15.  La   mamografía   de   una   señora   de   53   años   es   sospechosa   de   lesión   b.   FACTORES  HEREDITARIOS  
maligna  cuando  muestra:   c.   ENFERMEDAD  MAMARIA  BENIGNA  
a.   Aumento  de  la  densidad   d.   FACTORES  ENDOCRINOS  ENDOGENOS  
b.   Calcificaciones  de  los  vasos   e.   ANTICONCEPTIVOS  ORALES  
c.   Linfadenopatía  axilar   f.   NULIPARIDAD  
d.   Calcificaciones  finas   g.   TRATAMIENTO   SUSTITUTIVO   POSTMENOPAUSICO   CON  
e.   Calcificaciones  redondas   ESTRÓGENOS  
16.  Cuál   de   los   siguientes   hallazgos   es   más   indicativo   de   cáncer   en   una   23.  Actualmente  se  ha   descubierto  que   el   cromosoma   13  está  asociado   al  
mamografía  tomada  en  una  señora  de  70  años:   cáncer  de  mama.  Mencione  los  dos  genes  responsables  
a.   Calcificaciones  finas   a.   BRCA-­‐1  
17.  Enumere  el  %  de  cáncer  en  la  mama  que  ocurre  entre  los  45  y  59  años,   b.   BRCA-­‐2  
antes  de  los  20  años  y  antes  de  los  30  años.   24.  __F__  Mujeres  de  38  años  debe  practicarse  estudio  basal.  
a.   67  –  75%   25.  __F__   los   factores   de   riesgo   están   presentes   en   menos   de   la   mitad   d  
b.   0.08%   elos  casos  de  los  pacientes  que  son  diagnosticados  por  primera  vez  de  
c.   1.8%   cáncer  de  mama  
18.  Métodos   adecuados   para   el   Dx   del   cáncer   de   la   mama   con   el   fin   de   26.  __F__   Hay   mayor   riesgo   de   contraer   cáncer   mamario   con   múltiples  
detectar  el  trastorno  en  su  fase  pre-­‐clínica   embarazos  que  una  nulípara.  
a.   MAMOGRAFIA   27.  __C__  En  este  estudio  (mamografía)  se  emplea  menor  radiación  que  en  
b.   AUTO   EXAMEN   FÍSICO   DE   LA   MAMA   Y   PROMOVER   EL   AUTO   una  radiografía  de  torax.  
EXAMEN  DE  MAMA.   28.  __F__   La   mamografía     es   un     sustituto   de   biopsia   en   lesiones  
19.  Cuál   es   el   tipo   histológico   de   mayor   riesgo   de   cáncer   de   la   mama   sospechosas  
restante:  CANCER  DE  MAMA  LOBULAR   29.  __F__   Técnicas   tecnograficas   mejoradas   representan   un   sustituto  
20.  El  diagnóstico  de  Cáncer  de  Mama  está  basado  en:   seguro  suficiente  para  diagnóstico.  
a.   AUTOEXAMEN  DE  LA  MAMA   30.  __C__  El   estudio   este   parte   de   la   vigilancia   regular   de   un   paciente   de  
b.   HISTORIA  CLINICA  Y  EXAMEN  FISICO   falla  terapéutica  de  cáncer  mamario  unilateral  
c.   MAMOGRAFIA   31.  __C__   La   incidencia   de   cáncer   de   mama   en   mujeres   desciende  
d.   ECOGRAFIA   lentamente  
e.   BIOPSIA  
32.  __C__   En   la   actualidad   el   cáncer   de   mama   es   la   principal   causas   de   a.   Se  hace  traqueostomía  ante  la  primera  sospecha.  
muerte  relacionado  con  cancer  en  mujeres  panameñas.   b.   La  saturación  de  oxígeno  es  la  mejor  guía  para  el  tratamiento.  
33.  __F__  Cifras  de   supervivencia   para  etapas   comparables  para  el   cáncer   c.   Puede  haber  vibrisas  quemadas.  
mamario  son  más  altas  en  mujeres  mulatas  que  en  mujeres  blancas   d.   Es  frecuente  en  los  diabéticos.  
34.  __C__  Las  monjas  presentan  incidencia  de  mayor  cáncer  mamario   5.   Los  efectos  locales  de  una  quemadura  incluyen  los  siguientes,  MENOS:  
35.  Las  mamografías  falsas  negativas  son  más  frecuentes  en:     a.   Aumento  de  la  evaporación  
b.   Pérdida  de  calor  por  la  evaporación    
a.   __F__Mamas  eutróficas   c.   Invasión  al  área  por  microorganismos  
b.   __F__Fase  folicular  
d.   Trombosis  vascular  y  necrosis  tisular  
c.   __C__Fase  luteinica    
d.   __C__En   mujeres   que   estan   en   terapia   de   remplazo   e.   Septicemia  por  Gram  negativos  
hormonal     6.   De   acuerdo   con   la   regla   de   los   “9”   el   área   de   la   cabeza   y   cuello  
e.   __C__En  mujeres  que  toman  estrógenos    de  forma  cíclica     corresponde  a:  
  a.   9%  
QUEMADURAS   b.   18%  
  c.   27%  
1.   La   característica   más   importante   de   reconocer   una   quemadura   de   2º   d.   1%  
grado  es:   e.   36%  
a.   Venas  trombosadas.   7.   Porcentaje   de   superficie   corporal   que   incluye   miembros   superiores,  
b.   Edema.   tórax  y  hemiabdomen  superior:  
c.   Prurito.   a.   27%  
d.   Vesícula  o  ampolla.   8.   La  mejor  forma  de  hacer  el  diagnóstico  de  ruptura  vesical  después  de  un  
2.   El   periodo   en   que   se   escapan   las   proteínas   del   espacio   intravascular   trauma  pélvico  es:  
durante  una  quemadura  es  de:   a.   Citoscopía  
a.   Primeras  12  horas.   b.   Pielograma  IV  
b.   24  a  48  horas.   c.   Cistograma  
c.   Hasta  las  96  horas  desde  la  quemadura.   d.   Radiografía  apical  lord  ótica  
d.   No  ocurre  escape  de  proteínas.   e.   Sangre  en  la  orina  
3.   La  fórmula  de  Baxter  dice  que  el  reemplazo  de  líquido  durante  el  primer   9.   Qué  porcentaje  de  superficie  corporal  representa  una  quemadura  de  los  
día  es  de:   miembros  superiores  y  tórax  anterior:  27%  
a.   cc  por  cc  con  solución  coloide.   10.  El   periodo   en   que   se   escapan   las   proteínas   del   espacio   intravascular  
b.   L/R  3  cc/Kg  x  porcentaje  de  quemadura.   durante  la  quemadura  es  de:  24  A  48  HORAS  
c.   D/A  %%  1000  cc  +  KCl  mEq.  
d.   Equivalente  al  volumen  de  la  diuresis  con  Lactato  Ringer.  
TIROIDES  
4.   En  el  daño  por  inhalación:    
1.   En   una   mujer   de   20   años   de   edad   con   una   masa   asintomática   de   6   NO   El   tratamiento   definitivo   de   la   causa   subyacente   de   un  
centímetros  en  el  polo  inferior  del  lóbulo  tiroideo  derecho,  la  maniobra   neumotórax  a  tensión    requiere  toracostomía  abierta.  
inicial  más  apropiada  es:   2.   Es   llamado   al   cuarto   de   urgencias   para   atender   a   un   obrero   de   la  
a.   Biopsia  excisional.   construcción  de  28  años,  que  es  admitido  después  de  caer  de  9  metros  
b.   Biopsia  incisional.   de  altura  en  su  trabajo.  El  paramédico  de  la  ambulancia  informó  que  el  
c.   Aspiración  con  aguja  para  citología.   paciente  quedó  inconsciente  después  del  su  accidente,  pero  recobró  al  
d.   Estudio  de  captación  de  yodo  radiactivo.   consciencia  en  el  transcurso  al  hospital,  volviéndose  cada  vez  más  alerta  
e.   Examen  de  tiroides  con  ultrasonido   y  quejándose  de  dolor  en  el  muslo  derecho.  Al  llegar,  su  presión  arterial  
2.   El  cáncer  de  tiroides  más  común  es:   era   de   148/86,   pulso   de   100,   y   la   respiración   era   de   18   por   minuto.  
a.   Carcinoma  folicular.   Tenía   una   férula   colocada   en   su   extremidad   derecha   la   cual   se  
b.   Carcinoma  papilar.   observaba  edematosa.  Al  examen  físico,  mostraba  una  abrasión  de  4  cm  
c.   Carcinoma  anaplásico.   en   la  región  temporal,  leve  dolor  al  palpar  el  abdomen  y  dificultad  en  
d.   Carcinoma  medular.   mover   la   pierna   derecha,   la   cual   le   dolía   al   tocarla   en   el   muslo.   Sus  
e.   Cáncer  secundario.   reflejos  plantares  y  pupilares  están  bien.  
3.   Un   don   de   55   años   presenta   nódulo   tiroideo   de   4cms,   solitario   e   Cuál  de  lo  siguiente  haría  usted  en  los  primeros  5  a  10  minutos  en  orden  
indoloro.   T4   es   normal,   centelleo   no   demuestra   captación.   La   de  importancia:  
ultrasonografía  indica  que  es  sólido.  De  las  siguientes  estaría  indicado:   A.   Transfundir  una  unidad  de  sangre  
a.   Biopsia  con  aguja  fina  y  posible  cirugía   B.   Colocar  un  tubo  endotraqueal  
b.   Tratamiento  de  tiroides  y  solución  de  lugol   C.   Infundir  Lactato  Ringer  a  100  cc  por  hora  
c.   Yodo  radiactivo   D.   Infundir  dextrosa  al  5%  en  agua  
d.   Propanolol   E.   Infundir  SSN  
e.   Irradiación  externa     F.   Transfundir  una  unidad  de  células  rojas  empacados  
G.   Colocar  un  catéter  Foley  
TRAUMA   H.   No  dar  líquidos  IV  por  posible  trauma  cerebral  
  I.   Administrar  O2  al  100%  
1.   Cuáles   son   las   siguientes   afirmaciones   caracterizan   correctamente   el   __I__  
neumotórax  a  tensión:   __C__  
3.   Después   de   lo   anterior,   cuáles   de   estas   medidas   considera   usted,   en  
NO   Un   neumotórax   espontáneo   sin   tratamiento   habitualmente   orden  de  importancia:  
evoluciona  hacia    neumotórax  a  tensión.  
A.   Punción  lumbar  
NO   Los  datos  físicos  incluyen  desviación  de  la  tráquea  hacia  el  lado  
B.   Hemograma  completo  
afectado.  
NO   Cuando  un  paciente  presenta  neumotórax  a  tensión  mientras  se   C.   Electrolitos  del  suero  
le  administra  ventilación  con  presión  positiva  debe  suspenderse   D.   Gases  arteriales  
de  inmediato  la  presión  positiva.   E.   Urinalisis  
F.   EKG  
G.   Tipaje  y  cruce  de  una  unidad  de  sangre   6.   Una   mujer   de   18   años   de   edad   ingresa   a   una   sala   de   urgencias   en  
H.   Lavado  peritoneal   choque   hemorrágico   luego   de   un   accidente   automovilístico.   Luego   de  
__G__   administrar  2  L  de  L/R,  presión  arterial  y  pulso  permanecen  inestables  y  
__B__   la   paciente   se   queja   de   dolor   en   abdomen   alto.   En   el   examen   se  
__D__   observan   hiperestesia   y   notable   contractura   involuntaria   en   el   CSI   del  
4.   Cuales   procedimientos   radiográficos   de   los   siguientes   solicitaría,   en   abdomen.   Radiografía   torácica,   hematocrito   y   análisis   de   orina   son  
orden  de  importancia:   normales.   Cuál   de   los   siguientes   procedimientos   se   considera  
a.   Arteriografía  cerebral   indispensable  para  el  tratamiento  de  esta  paciente?  
b.   Radiografía  lateral  de  la  columna  cervical   a.   Tomografía  computarizada  de  abdomen.  
c.   US  abdominal   b.   Lavado  peritoneal.  
d.   Radiografía  de  fémur  derecho   c.   Sonografía  abdominal.  
e.   Aortograma  abdominal   d.   Arteriografía  celíaca  selectiva.  
f.   Pielograma  IV   e.   Laparotomía  exploratoria.  
g.   Radiografía  de  Tórax   7.   Un  evento  metabólico  en  trauma,  excepto:  
h.   Centelleo  esplénico   a.   Fuente  de  glucosa  para  glucogenolisis  
i.   CAT  de  la  cabeza   8.   Todo  lo  siguiente  es  cierto  acerca  de  la  respuesta  metabólica  al  trauma  
__B__   excepto:  
__G__   a.   La  alanina  es  el  principal  aminoácido  utilizado  en  el  ciclo  de  Cori  
__I__   para  obtención  de  energía  
5.   Un   hombre   de   18   años   de   edad   ingresa   al   departamento   de   urgencias   b.   La  principal  fuente  de  aminoácidos  es  proveniente  del  músculo  
después   de   sufrir   un   accidente   automovilístico.   Se   encuentra   en   un   c.   El  glicerol  puede  ser  transformado  en  glucosa  
estado  de  coma  (7  en  la  escala  de  Glasgow).  El  pulso,  con  frecuencia  de   d.   Las   respuestas   metabólicas   en   el   trauma   es   similar   a   los  
140  latidos  por  minuto  es  casi  impalpable  y  la  presión  arterial  es  60/0.   pacientes  en  ayuno  
La   respiración   es   rápida   y   superficial,   ventilando   ambos   campos   9.   En   un  paciente  politraumatizado,  con  enfisema  subcutáneo   de   toda   la  
pulmonares.  El  abdomen  se  encuentra  moderadamente  distendido,  sin   pared  torácica  y  con  signos  de  falla  respiratoria,  lo  correcto  es:  
peristalsis  audible.  Se  detectan  fracturas  cerradas  en  antebrazo  derecho   a.   Hacer  una  radiografía  de  tórax  
y  parte  inferior  de  pierna  izquierda.  Luego  de  administración  IV  de   2   L   b.   Realizar  una  broncoscopía  
de   L/R   en   extremidades   superiores,   el   pulso   es   de   130   y   la   presión   c.   Colocar  un  tubo  pleural  en  cada  hemitórax    
arterial  70/0.  El  siguiente  paso  inmediato  sería:   10.  Un   joven   de   30   años   fue   traído   al   cuarto   de   urgencias   después   de   un  
a.   Practicar   radiografía   lateral   de   columna   cervical   a   través   de   la   accidente   de   tránsito.   Los   signos   vitales   eran:   PA   110/90,   Pulso  
mesa.   110/minuto.  La  radiografía  de  tórax  reveló  leve  neumotórax  (menor  de  
b.   Practicar  TC  de  cabeza  y  abdomen.   1%).   Tenía   signos   de   abdomen   agudo.   Cuál   es   el   próximo   paso   en   el  
c.   Practicar  radiografía  de  abdomen  en  decúbito  supino  y  lateral.   manejo  de  este  paciente:  
d.   Practicar  aortografía  de  cayado.   a.   Administrar  oxígeno  con  máscara  
e.   Explorar  el  abdomen.   b.   Llevarlo  al  salón  por  su  abdomen  agudo  
c.   Colocarle  un  tubo  de  toracostomía   a.   Tratar  el  Shock  hipovolémico  
d.   Hacerle  una  entubación  endotraqueal   b.   -­‐-­‐-­‐-­‐-­‐  neurogénico  
e.   Hacerle  una  toracotomía   c.   Manejo  quirurgico  del  collar  cervical  ¿=  
11.  En   un   individuo   previamente   sano,   la   pérdida   de   15%   a   30%   del   d.   Intubación  nasotraqueal  
volumen  sanguíneo:   16.  Un  joven  herido  en  accidente  moto  ciclístico  llega  al  cuarto  de  urgencia  
a.   Se  distingue  por  hipotensión  profunda   con  presión  arterial  70/40,  pulso  120,  con  tiraje,  y  cianosis  debido  a  un  
b.   Usualmente   produce   todos   los   signos   y   síntomas   clásicos   del   neumotórax  a  tensión  izquierda,  el  abdomen  doloroso,  y  defendido,  con  
Shock   fractura   expuesta   del   fémur   derecho.   Usted   es   la   única   persona   en   el  
c.   Requiere  reemplazo  por  anuria   cuarto  de  recibo.  Indique  el  orden  de  prioridad  que  usted  instalaría:  
d.   Se   manifiesta   por   taquicardia,   taquipnea   y   reducción   de   la   1.   Reducir  la  fractura  y  férula  de  yeso  luego  de  limpieza  
presión  del  pulso  sin  otros  síntomas  de  shock   2.   Canalizar  una  vena  con  aguja  14  o  16  y  dar  L/R  
12.  La  primera  medida  que  se  debe  tomar  en  un  poli  traumatizado  que  llega   3.   Tubo  intra  pleural  izquierdo  
inconsciente  y  PA  90/60  es:   4.   Laparotomía  exploradora  después  de  dar  sangre  
a.   Canalizar  una  vena  con  una  aguja  gruesa  14  0  16   5.   Hipertet,  toxoide  antitetánico,  según  su  criterio  
b.   Llamar  urgente  a  neurocirugía   a.   3,2,4,1,5  
c.   Control  de  la  hemorragia  con  torniquetes   b.   2,5,3,4,1  
d.   Hipertet  IM  y  refuerzo  con  toxoide     c.   4,3,2,1,5  
e.   Asegurar  la  permeabilidad  de  las  vías  aéreas         d.   2,4,3,1,5  
13.  Signo  para  hacer  el  diagnóstico  diferencial  de  taponamiento  cardiaco  y   e.   3,2,1,4,5  
neumotórax  a  tensión   17.  El  tratamiento  inmediato  de  un  paciente  con  múltiples  fracturas  y  shock  
a.   Ruidos  cardiacos   debe  ser:  
b.   Presión  arterial  baja   a.   Transfusión  de  sangre  
c.   Ruidos  respiratorios   b.   Salina  normal  
d.   Taquicardia   c.   Plasma  
14.  Manejo  en  un  centro  de  salud   d.   Lactato  Ringer  
a.   Llamar  al  personal  para  que  le  tome  radiografía   e.   Albúmina    
b.   Pericardiocentesis   18.  Una   joven   de   20   años   es   traída   al   hospital   después   de   accidente   de  
c.   Enviar  a  cuarto  de  urgencias  de  un  hospital   tránsito.   Está   consciente   y   sangra   profusamente   por   una   laceración  
d.   Referir   grande  del  cuero  cabelludo  y  de  herida  abierta  del  muslo  derecho.  Está  
e.   Nada   aprehensiva   y   sedienta.   El   pulso   es   130,   la   presión   es   de   70/40.   Dice  
15.  Manejo   de   femenina   que   sufre   accidente   automovilístico   y   presenta   pesar  110  libras.  Se  calcula  la  pérdida  de  sangre  en:  
disminución   de   la   presión   arterial,   aumento   de   la   frecuencia   cardiaca,   a.   500  ml  
parálisis  de  las  4  extremidades,  está  consciente,  buena  ventilación,  vías   b.   1000  ml  
aéreas  permeables,  ella  está  pálida  y  fría,  y  refiere  tener  sed  y  no  poder   c.   1500  ml  
respirar:   d.   2000  ml  
e.   2500  ml   d.   Es  más  importante  en  el  paciente  inconsciente  que  el  paciente  
19.  En  el  adulto  traumatizado,  los  volúmenes  ventilatorios  deben:     consciente  que  mueve  activamente  las  extremidades  
a.   Ser  por  lo  menos  25  ml/kg  de  peso  estimado   e.   Debe   enfocar   sobre   el   proceso   odontoides   y   los   cuerpos  
b.   Ser  800  a  1200  ml  por  respiración   vertebrales  
c.   Ser  menos  que  el  volumen  corriente  para  evitar  neumotórax   24.  Un   joven   de   20   años   de   edad   recibe   tratamiento   para   lesiones  
d.   Nunca  exceder  10  ml/kg  de  peso  estimado   incurridas  en  un  accidente.  No  hay  sangre  en  el  banco,  solo  se  dispone  
e.   Ser   considerado   como   de   importancia   secundaria   en   de  líquidos  intravenosos  no  sanguíneos.  Su  hemoglobina  es  de  4.5  gms.  
comparación  a  las  presiones  ventilatorias     La  cianosis  es:    
20.  En   la   primera   hora   después   de   trauma   cerrado   lesiones   del   a.   Inevitable  
retroneumoperitoneo:     b.   Probable  
a.   Se  caracteriza  por  sepsis  anaeróbica   c.   Imposible  
b.   Están  asociadas  a  hemorragias  exanguinantes     d.   Evitable  
c.   Son  difíciles  de  diagnosticar   e.   Deseable    
d.   Se  descartan  por  lavado  peritoneal   25.  El   aspecto   más   importante   del   tratamiento   inicial   de   la   acidosis  
e.   Se  sospechan  por  los  signos  y  síntomas  que  presentan   metabólica  en  los  traumas  severos  es:  
21.  La  razón  por  el  monitoreo  de  la  excreción  urinaria  en  la  víctima  de  Shock   a.   El  balance  químico  del  pH  arterial  
es  porque  refleja  con  acierto:   b.   Establecer  una  perfusión  tisular  adecuada  
a.   La  sobrecarga  de  líquido   c.   Reemplazo  del  volumen  perdido  con  electrolitos  
b.   El  transporte  de  agua  libre   d.   Hiperventilación  
c.   El  sodio  en  el  suero   26.  Cuando   hay   muchas   víctimas   de   trauma   en   el   cuarto   de   urgencias,   la  
d.   La  perfusión  renal   prioridad  más  alta  de  manejo  se  le  deba  asignar  al  que  sufre  de:  
e.   Los  niveles  de  catecolamina     a.   Fractura  expuesta  de  ambos  fémures  
22.  Antes   de   intentar   una   de   las   maniobras   de   ventilación   en   el   paciente   b.   Hemorragia  severa  con  heridas  abiertas  
con   trauma   severo   de   la   cabeza   y   cuello,   debe   ser   explorado   por   c.   Trauma  craneoencefálico  severo  
fractura  cervical.  El  mejor  método  diagnóstico  sería:   d.   Obstrucción  de  las  vías  aéreas  
a.   Exploración  digital  del  cuello   e.   Tórax  batiente  
b.   Historia  de  dolor  en  la  columna  cervical   27.  Las   hemorragias   que   producen   pérdidas   del   15%   del   volumen  
c.   Radiografía  posteroanterior  del  cuello   sanguíneo:  
d.   Radiografía  lateral  del  cuello   a.   Pueden  producir  shock  hasta  en  un  tercio  de  individuos  sanos  
e.   Signos  neurológicos   b.   Es  precedido  por  oliguria  
23.  Una   vista   lateral   de   las   vértebras   cervicales   con   el   paciente   acostado   c.   Causa  desmayos,  otras  cosas  poco  
sobre  la  mesa:   d.   Se  pueden  tratar  sin  transfusión  de  sangre  
a.   Descarta  lesiones  serias  de  la  columna  cervical   e.   Usualmente  causan  hipotensión  
b.   Debe  proceder  la  entubación  endotraqueal  
c.   Es  inaceptable  a  menos  que  se  visualicen  las  7  cervicales  
28.  En   el   paciente   con   trauma   cerrado   de   tórax,   y   que   presenta   imagen   31.  Un   mecánico   de   45   años   de   edad   estaba   trabajando   en   su   taller   y  
hiperluscente   en   el   hemitórax   izquierdo,   además   de   un   neumotórax   solicitó  a  su  ayudante  que  arrancara  el  camión  para  evaluar  el  motor.  El  
debe  descartarse   ayudante   poco   experto   en   manejo   pierde   el   control   y   aprisiona   a   su  
a.   Hemotorax   compañero  contra  la  pared.  Este  es  llevado  al  servicio  de  urgencias  de  
b.   Contusión  pulmonar   su   hospital.   El   paciente   presenta   un   estado   evidente   de   shock   con  
c.   Hernia  diafragmática   excoriaciones,  deformidad  y  aumento  de  volumen  en  ambos  muslos.  El  
29.  Un  hombre  de  29  años  de  edad  llega  al  servicio  de  urgencias  luego  de   estado  de  shock  que  presenta  el  paciente:  
haber   sufrido   un   accidente   automovilístico-­‐colisión.   Se   encuentra   a.   No  puede  ser  explicado  a  menos  que  exista  una  fractura  pélvica  
cianótico,   tiene   insuficiencia   respiratoria,   y   una   escala   de   coma   de   asociada  
Glasgow   de   6.   No   hay   trauma   facial   significativo;   su   tráquea   se   b.   No   puede   ser   revertido   si   se   aplican   férulas   de   tracción   en  
encuentra   en   la   línea   media;   tiene   una   desviación   crónica   del   tabique   ambos  miembros  inferiores  
nasal   que   impide   la   intubación   nasotraqueal.   Tiene   abundante   barba   c.   Es   explicado   porque   significa   que   el   paciente   ha   tenido   una  
que  dificulta  la  fijación  de  la  máscara  de  oxígeno.  El  paso  siguiente  más   pérdida   de   aproximadamente   15%   de   su   volumen   sanguíneo  
apropiado  es:   circulante.  
a.   Realizar  una  cricotiroidotomía  quirúrgica.   d.   Solo  se  explica  si  hay  lesión  de  una  arteria  importante  
b.   Forzar  un  tubo  nasotraqueal  a  través  del  tabique  nasal  desviado   e.   Es   explicado   por   pérdida   sanguínea   asociada   a   fracturas  
c.   Intentar  una  intubación  orotraqueal  usando  dos  personas  para   femorales  bilaterales.  
mantener  la  columna  alineada.   32.  Son  objetivos  de  la  intubación  endotraqueal:  
d.   Ventilar  al  paciente  con  mascarilla  con  válvula  y  reservorio  hasta   a.   Establecer  vía  aérea  
que  se  le  pueda  afeitar  la  barba  para  lograr  colocar  una  máscara   b.   Administrar  oxígeno  
de  oxígeno.   c.   Apoyar  la  ventilación  
30.  Usted   está   de   turno   en   el   servicio   de   urgencias   cuando   le   llega   un   d.   Prevenir  la  broncoaspiración  
paciente   acompañado   de   TUM’s   quienes   le   informan   que   el   paciente   e.   Todas  las  anteriores  
recibió  golpes  con  un  garrote  en  la  cabeza  y  en  la  cara.  El  paciente  usted   33.  Un   hombre   de   40   años   sufre   un   traumatismo   cerrado   de   tórax   al  
lo  encuentra  comatoso  y  a  la  palpación  presenta  una  fractura  deprimida   estrellarse   contra   el   timón   del   vehículo   en   un   accidente   en   el   que  
de   cráneo.   La   cara   está   edematizada   y   equimótica,   su   respiración   es   impactó   a   alta   velocidad   un   objeto   fijo.   En   el   servicio   de   urgencias  
gorgoreante  y  se  encuentran  restos  de  vomito  en  la  cara  y  en  la  ropa.  El   observa   que   el   paciente   presenta   distensión   de   las   venas   del   cuello,  
paso   más   apropiado   a   seguir,   luego   de   la   administración   de   oxígeno   desvío  de  la  tráquea  hacia  la  izquierda,  una  perfusión  periférica  pobre  y  
suplementario  y  de  elevarle  la  mandíbula  es:   ausencia  de  ruidos  respiratorios  en  el  hemotórax  derecho.  La  causa  más  
a.   Succionar  la  orofaringe.   probable  de  estos  hallazgos  es:  
b.   Insertar  un  tubo  gástrico   a.   Lesión  aórtica  
c.   Solicitar  una  tomografía  computarizada   b.   Una  contusión  cardiaca  
d.   Obtener  una  radiografía  lateral  de  la  columna  cervical   c.   Neumotórax  a  tensión  
e.   Ventilar   al   paciente   utilizando   una   mascarilla   con   válvula   y   d.   Síndrome  de  vena  cava  superior  
reservorio-­‐AMBU  
34.  Un  motociclista  de  20  años  sufre  una  lesión  muy  importante  en  la  cara   b.   el  peso  y  la  masa  de  cada  individuo  
al  estrellar  su  motocicleta  de  frente  contra  una   camioneta.  El  paciente   c.   la  velocidad  y  la  aceleración  que  iban  viajando  
es   llevado   por   los   TUM’s   al   servicio   de   urgencias   completamente   d.   el  peso  y  la  masa  del  vehículo  
inmovilizado  en  una  tabla  espinal  larga  y  con  un  collar  cervical.  Usted  es   38.  Un   hombre   de   40   años   sufre   un   traumatismo   cerrado   en   el   tórax   al  
el   médico   que   recibe   este   paciente.   Al   examen   tiene   PA   de   150/60   compactar  el  timón  en  un  accidente  a  alta  velocidad  con  un  objeto  fijo,  
mmHg,  la  FC=  90/min.  Rítmico,  la  FR=  26/min.  El  paciente  presenta  una   el  paciente  tiene:  
respiración  laboriosa  y  es  ruidosa.  Su  escala  de  coma  de  Glasgow  es  de   a.   neumotórax  a  tensión  
7.   Usted   efectúa   varios   intentos   de   intubación   orotraqueal     39.  Al  colocar  un   catéter   subclavio,   el   paciente   presenta   disnea,   cianosis   y  
manteniendo   la   columna   cervical   alineada,   sin   éxito   debido   a   la   dolor  torácico.  Probablemente  ha  ocurrido:  
presencia   de   sangrado   y   una   anatomía   distorsionada.   Súbitamente   a.   Sepsis  
presenta  apnea.  El  mejor  procedimiento  para  el  manejo  temporal  de  la   b.   Punción  de  la  arteria  
vía  aérea  en  esta  situación  es:   c.   Trombosis  de  la  vena  
a.   Intubación  nasotraqueal   d.   Hidro  o  neumotórax  
b.   Traqueostomía  de  urgencia   e.   Punción  del  miocardio  
c.   Colocación  de  una  cánula  orofaríngea   40.  Cuál  de  los  casos  siguientes  tendría  prioridad  estando  solo  en  el  cuarto  
d.   Colocación  de  una  cánula  nasofaríngea   de  urgencias  
e.   Cricotiroidotomía  con  aguja  e  insuflación  a  presión   a.   Trauma  cerrado  de  tórax,  disneico  con  tiraje  
35.  Durante   la   revisión   primaria   ¿Cuál   de   los   siguientes   hallazgos   en   un   b.   Trauma  cerrado  de  abdomen,  vomita  sangre  
adulto  indica  un  manejo  inmediato?   c.   Herida  de  bala  sobre  la  tetilla  izquierda,  PA  100/70,  Pulso  de  
a.   Abdomen  distendido   100  
b.   Escala  de  coma  de  Glasgow  de  11   d.   Fractura  expuesta  de  fémur,  sangrado  de  la  herida    
c.   Temperatura  de  36.5ºC   41.  Cuál   de   los   siguientes   indica   tamponamiento   cardiaco   en   una   herida  
d.   Frecuencia  cardiaca  de  120  latidos  por  minuto   punzante  en  el  pericardio  (hemopericardio)  
e.   Frecuencia  respiratoria  de  40  respiraciones  por  minuto   a.   Ruidos  cardiacos  apagados  o  distantes  
36.  Los  exámenes  radiológicos  más  importantes  que  debemos  obtener  en  el   b.   Presión  venosa  de  2  cm  de  agua  
paciente  traumatizado  severo  son:   c.   Hipotensión  sistólica  
a.   Cráneo,  tórax  y  abdomen   d.   Todas  
b.   Tórax,  abdomen  y  pelvis   e.   Solamente  A  y  C  
c.   Cráneo,  columna  cervical  y  tórax   42.  Un  joven  de  18  años  recibe  una  bala  en  el  abdomen  durante  un  asalto.  
d.   Columna  cervical,  tórax  y  pelvis   La  herida  de  la  bala  está  por  debajo  del  ombligo  y  otra  herida  de  10  cms  
e.   Columna  cervical,  tórax  y  abdomen   por   debajo   del   reborde   costal   en   el   lado   derecho   de   la   línea   axilar  
37.  Un  adulto  y  un  menor  viajan  en  un    vehiculo  pequeño  a  una  velocidad   media.  El  pulso  es  de  90  por  minuto  y  la  PA  de  110/60.  El  paciente  debe:  
de   80   millas/hora.   De   ocurrir   un   accidente,   el   factor   predominante   a.   Ser   sometido   a   exploración   de   las   heridas   bajo   anestesia  
causante  de  las  lesiones  corporales  seria:   local  
a.   la  composición  del  objeto  fijo   b.   Ser  sometido  a  un  lavado  peritoneal  diagnóstico  
c.   Ser  observado  cuidadosamente  por  hemorragia  o  peritonitis   b.   Vitamina  K  
d.   Ser  sometido  a  laparotomía  lo  antes  posible   c.   Plasma  fresco  congelado  
e.   Inyectar  las  heridas  con  medio  de  contraste  en  el  sitio  de  la   d.   Crioprecipitados  
lesión   48.  Un   señor   de   65   años,   gran   fumador,   con   enfermedad   obstructiva  
43.  Hasta   que  se   pruebe   lo   contrario   usted   debe   considerar   a   un   paciente   crónica,   se   cae   y   se   fractura   cuatro   costillas   del   lado   derecho.   Se  
de  trauma  que  este  belicoso,  combativo  y  no  cooperador  como:   descarta   otra   lesión   del   lado   derecho   y   otra   lesión   concomitante.   El  
a.   diabético   tratamiento  apropiado  podría  ser  todos  los  siguientes  menos:  
b.   intoxicado   a.   Colocar  cinta  adhesiva  alrededor  del  tórax  para  inmovilización  
c.   impertinente   b.   Bloqueo  de  nervios  intercostales  
d.   hipóxico   c.   Paracentesis  abdominal  
44.  Después  de  un  traumatismo  quirúrgico  mayor,  a  los  2  o  4  días  podemos   d.   Radiografía  de  tórax  
tener  cada  una  de  las  siguientes  situaciones,  EXCEPTO:   e.   Consulta  a  neumólogo  
a.   Aumento  de  la  excreción  de  los  esteroides  adrenocorticales   49.  Cuál   de   los   siguientes   probablemente   NO   será   detectado   durante   la  
b.   Oliguria  relativa   evaluación  inicial  de  un  accidentado  de  transito:  
c.   Disminución  de  la  excreción  de  nitrógeno   a.   Obstrucción  de  la  laringe  
d.   Retención  de  sodio  y  potasio   b.   Desgarro  total  de  la  aorta  torácica  
45.  La  contusión  pulmonar:   c.   Neumotórax  abierto  
a.   Es  la  lesión  potencialmente  mortal  más  común  en  la  evaluación   d.   Neumotórax  a  tensión  
inicial   e.   Tórax  batiente  importante  con  insuficiencia  respiratoria  
b.   Siempre  requiere  de  manejo  con  ventilación  mecánica   50.  Todos  lo  siguientes  sugieren  lesión  traumática  de  la  uretra,  EXCPETO:  
c.   Una   vez   realizada   la   reanimación,   los   líquidos   deben   ser   a.   Sangre  en  el  meato  urinario  
administrados  juiciosamente   b.   Hematoma  del  escroto  
d.   El  tratamiento  incluye  la  fijación  de  la  caja  torácica   c.   Ausencia  de  la  próstata  palpable    al  tacto  rectal  
46.  El   mejor   tipo   de   estudio   de   rayos   X   para   localizar   aire   libre   d.   Próstata  elevada  al  tacto  rectal  
intraabdominal  dudosa  es  una  radiografía:   e.   Priapismo  
a.   Una  radiografía  AP  de  tórax   51.  Cuando   se   lesiona   un   vaso   la   respuesta   inmediata   para   producir  
b.   Una  radiografía  de  de  pie  y  acostado  del  abdomen   hemostasia  es:  
c.   Una  radiografía  en  decúbito  lateral  con  rayos  horizontales,  con   a.   Adherencia  plaquetaria  en  el  lugar  de  la  lesión  
el  lado  derecho  hacia  arriba   b.   Vasoconstricción        
d.   Tomografía  computarizada  de  abdomen   c.   Liberación  de  un  trombo  
47.  Un  joven  sano  recibió  10  unidades  de  sangre  luego  de  sufrir  un  trauma   d.   Ayuda  de  los  fibroblastos  
en  una  colisión  de  un  vehículo  a  motor.  Su  temperatura  corporal  es  de   52.  Enumere  cuatro  signos  OBJETIVOS  de  compromiso  de  la  vía  aérea:  
37°C,   plaquetas   75.000.   Presenta   sangrado   difuso.   El   tratamiento   más   a.   AGITADO  
efectivo  deberá  ser:   b.   OBNUBILADO  
a.   Infusión  de  plaquetas   c.   CIANOSIS  
d.   TIRAJE   a.   ASEGURAR  VÍA  AÉREA    
e.   ESTRIDOR  LARINGEO   b.   VENTILAR  Y  OXIGENAR  
53.  El  paciente  politraumatizado  crítico  muere  por:   62.  Signos  objetivos  de  paciente  en  Shock:  
a.   HIPOTERMIA   a.   FRIO  
b.   ACIDOSIS  METABOLICA   b.   TAQUICARDIA  
c.   COAGULOPATÍA   c.   ¿?==?  
54.  Un   joven   herido   en   accidente   automovilístico   llega   al   servicio   de   63.  Vía  aérea  definitiva:  
urgencia   con   PA   70/40,   pulso   120,   sin   signos   externos   de   hemorragia   a.   VÍA  AÉREA  INSEGURA  
severa,  disnea  severa,  asimetría  de  tórax,  tráquea  desviada  a  la  derecha,   b.   APNEA  
el   abdomen   doloroso   y   defendido,   con   fractura   expuesta   del   fémur.   c.   LESIÓN  CRANEOENCEFÁLICA  CERRADA  
Usted   es   la   única   persona   en   el   servicio.   Indique   en   qué   orden   de   d.   COMPROMISO  INMINENTE  DE  LA  VÍA  AÉREA  
prioridad  usted  atendería  a  este  paciente   e.   OXIGENACIÓN  POBRE  
a.   TUBO  PLEURAL  DERECHO   f.   RIESGO  DE  ASPIRACIÓN  
b.   CANALIZACION  DE  2  VENAS  PARA  LACTATO  RINGER  A  CHORRO   64.  __F__   La   primera   medida   que   se   toma   en   un   paro   cardiaco   es   la  
c.   LAPAROTOMIA     inyección  de  adrenalina  
d.   LIMPIEZA  Y  REDUCCIÓN  DE  FRACTURA  EXPUESTA   65.  __F__  El  diagnóstico  de  neumotórax  a  tensión  es  radiológico  
e.   TOXOIDE  TETÁNICO     66.  __F__   La   primera   acción   a   tomar   con   paciente   que   se   presenta   con  
55.  Cuando   se   lesiona   un   vaso   la   respuesta   inmediata   para   producir   neumotórax  abierto  es  la  sutura  
hemostasia  es:  VASOCONSTRICCIÓN   67.  __F__  En   el  paciente  de  trauma  al  que  se  va  a  intubar  la  maniobra  de  
56.  Cuando   el   Paciente   es   sometido   a   ventilación   mecánica   con   presión   alinear  la  columna  cervical  con  extensión  de  la  cabeza  es  la  indicada  
positiva,   la   causa   más   frecuente   de   neumotórax   a   tensión   es:   EL   68.  __F__  En  el  paciente  de  trauma,  la  hiperextensión  de  la  cabeza  facilita  la  
BAROTRAUMA   maniobra  de  intubación  orotraqueal.  
57.  Historia   de   golpe   en   1/3   inferior   del   hemitórax   izquierdo   asociado   a  
69.  __F__   En   el   paciente   de   trauma,   la   hiperextensión   del   cuello   de   la  
dolor  en  el  hombro  izquierdo  más  defensa  abdominal  indica:  RUPTURA  
DE  BAZO   cabeza  facilita  la  maniobra  de  intubación  endotraqueal  
58.  El   tratamiento   inicial   del   neumotórax   abierto   es:   CUBRIR   LA   HERIDA   70.  __F__   El   shock   cardiogénico   es   el   más   común   por   lesiones     que   se  
CON  APÓSITOS  ESTÉRILES  Y  FIJARLO  EN  3  LADOS.   producen  al  miocardio  en  las  heridas  penetrantes.  
59.  La  tríada  de  Beck  está  constituida  por:   71.  __F__  Todo  paciente  hipotenso  y  frio  está  en  shock  hasta  que  se  pruebe  
a.   HIPOTENSIÓN   lo  contrario  
b.   DISMINUCIÓN  DE  LOS  RUIDOS  CARDÍACOS  A  LA  AUSCULTACIÓN   72.  __C__  En  la  hemorragia  Clase  I  presenta  signos    clínicos  mínimo  porque  
c.   INGURGITACIÓN  YUGULAR  
el  organismo  generalmente  lo  compensa.  
60.  El   desequilibrio   ácido   base   se   debe   a   perfusión   y   reanimación  
73.  __F__   La   sola   permeabilidad   de   la   vía   aérea   asegura   una   adecuada  
inadecuada.  El  tratamiento  sería:  
ventilación  
a.   OXIGENAR  Y  VENTILAR  
74.  __C__  El  nivel  de  conciencia  alterado  en  trauma  es  la  causa  más  común  
b.   ADMINISTRAR  LÍQUIDOS  IV  Y  SANGRE  
de  compromiso  de  la  vía  aérea.  
61.  En  el  manejo  de  vía  aérea  la  primera  prioridad  es:  
75.  __F__   El   objetivo   de   la   evaluación   inicial   es   detectar   lesiones   que   SÍ   La  abstinencia  de  fumar  ayuda  a  impedir  la  gangrena.  
pueden  ser  potencialmente  letales.   SÍ   Una  operación  profiláctica  ayuda  a  impedir  la  gangrena.  
76.  __C__  Un  evento  metabólico  que  ocurre  en  el  paciente  con  trauma  es  la    
excreción  urinaria  de  nitrógeno.   2.   En   la   siguiente   lista   son   síntomas   y   signos   de   la   isquemia   arterial  
77.  __C__   En   el   shock   hipovolémico,   la   resistencia   periférica   está   obstructiva  aguda  
aumentada   Si    Parálisis  
78.  __C__  una  vista  lateral  de  las  vértebras  cervicales  es  recomendable  en   No    Edema  
todo  caso  de  lesión  severa  por  arriba  de  las  claviculas   Si    Palidez  
79.  __C__   En   los   jóvenes,   el   desplazamiento   del   mediastino   en   casos   de   Si    Ausencia  de  pulso  
neumotórax  masivo  es  mayor  que  en  los  de  mayor  edad.   Si    Dolor  
80.  __C__  Una  indicación  para  el  lavado  peritoneal  es  el  estado  de  coma  en    
trauma  cerrado  en  shock  hipovolémico   3.   La  tromboflebitis  de  venas  profundas  de  las  extremidades:  
81.  __C__   La   extirpación   del   bazo   en   jóvenes   expone   al   peligro   de   a.   Se  maneja  con  anticoagulantes  a  largo  plazo  
infecciones  altamente  peligrosas   4.   __C__  La  tromboflebitis  superficial  se  maneja  sin  cirugía  
5.   Manifestación  clínica  de  la  isquemia  crónica:  DOLOR  EN  REPOSO    
TUMOR  DE  CABEZA  Y  CUELLO   6.   Manifestación   clínica   de   la   isquemia   arterial   aguda   en   los   miembros  
  inferiores  
1.   Entre   los   tumores   intraorales   tenemos   las   lesiones   premalignas,   a.   DOLOR  
mencione  3:   b.   AUSENCIA  DE  PULSO  
a.   HIPERQUERATOSIS  
c.   PARALISIS,  PARESTESIA  Y  PALIDEZ  
b.   HIPERPLASIA  PSEUDOEPITELIOMATOSA  
c.   LEUCOPLASIA   7.   Manifestación  clínica  de  la  isquemia  arteria  crónica  
2.   Dentro  de  la  etiología  de  los  tumores  malignos  intraorales  mencione  3:   a.   CLAUDICACIÓN  INTERMITENTE  
a.   IRRITACIÓN  CRÓNICA   b.   DOLOR  EN  REPOSO  
b.   TABAQUISMO   c.   GANGRENA  
c.   ALCOHOL   8.   En  el  síndrome  post-­‐flebítico,  lo  más  importante  es:  PREVENCIÓN    
3.   Entre   los   tumores   malignos   intraorales   cuál   es   el   tipo   histológico   que   9.   Las   complicaciones   más   importantes   de   la   trombosis   venosa   profunda  
predomina:  ESCAMOSO    
son:  
  a.   EMBOLISMO  PULMONAR  
VASCULAR  PERIFÉRICA   b.   SINDROME  POSTFLEBITICO  
 
1.   Cuáles   de   las   siguientes   afirmaciones   relacionadas   con   pacientes   que    
padecen  claudicación  intermitente  son  ciertas:  

SÍ   2/3  de  ellos  pueden  tratarse  sin  intervención  quirúrgica.  


NO   1/3  presentará  gangrena.  
SÍ   En  ausencia  de  cambios  tróficos,  rara  vez  aparece  gangrena.  
SS – Med10

CIRUGÍA
Preguntas del Blueprints

1. Un hombre de 71 años con aparición repentina de abdominales severos, el dolor de espalda y


de espalda se lleva al departamento de emergencias para evaluación. Tiene antecedentes de
hipertensión. Pesa 300 lb. Tiene un historial de 45 paquetes al año de de fumar. El examen
físico revela un pulsátil masa abdominal Ambas extremidades inferiores revelan palidez con
pulsos de pedal disminuidos. ¿Qué es lo más probable causa de la condición de este paciente?
a) Aterosclerosis
b) Síndrome de Marfan
c) Infección meningocócica
d) Sífilis
e) Trauma

2. Un hombre de 78 años es llevado a la sala de emergencias, con antecedentes de dolor


abdominal de 12 horas, diarrea y vómito. Tiene antecedentes de fibrilación auricular y fue
tratado previamente por insuficiencia cardíaca congestiva con digoxina El examen físico revela
una distensión abdomen con protección significativa. Examen rectal revela heces guayaco
positivas en la bóveda. Glóbulos blancos: el recuento celular es de 24,000 / mL. La radiografía
abdominal revela edema de la pared intestinal. ¿Cuál es el tratamiento más apropiado para este
paciente?
a) Embolización angiográfica
b) Terapia antibiótica con ampicilina y gentamicina.
c) Terapia antibiótica con gentamicina.
d) Heparinización seguida de warfarina oral.
e) Exploración quirúrgica

3. Un jugador de tenis masculino de 20 años se estrella contra una cerca tratando de perseguir
una pelota, pensó que podría alcanzar durante un partido importante Su rodilla derecha
sostiene el la mayor parte de las lesiones. El examen físico revela edema y disminución del
rango de movimiento de la rodilla en flexión y extensión. La resonancia magnética (MRI) es
performado y revela dislocación de la articulación. No hay pulsos palpables detrás de la
articulación de la rodilla. ¿Qué es lo más probable explicación de este hallazgo?
a) Rotura de la arteria tibial anterior
b) Hematoma de la arteria peronea
c) Espasmo de la arteria poplítea
d) Hematoma de la arteria tibial posterior
e) Espasmo superficial de la arteria femoral

4. Una mujer de 25 años encontró un bulto en su seno derecho en autoexamen. Ella no tiene
antecedentes familiares de mama cáncer. El bulto es libremente móvil y está bien
circunferenciado. ¿Cuál es la mejor opción para evaluar una masa en el seno en una mujer
joven?
a) Biopsia
b) Mamografía
c) Pruebas para el gen del cáncer de seno ( BRCA )
d) Ultrasonido
e) Espera vigilante

5. Una mujer de 19 años comenzó a amamantar por primera vez. Al principio, era difícil para su
bebé alimentarse. Ahora sus senos son rojos, cálidos y doloridos. Ella ha continuado
SS – Med10
amamantando, a pesar del dolor; sin embargo, ella recientemente comenzó a usar
un extractor de leche en lugar de alimentación. Ella se inicia en un curso de
antibióticos orales.
¿Qué afección tiene este paciente en riesgo de desarrollar?
a) Absceso mamario
b) Enfermedad fibroquística
c) Cáncer de mama inflamatorio
d) Prolactinoma
e) Tuberculosis

6. Una mujer premenopáusica de 31 años con masa en el seno izquierdo se somete a una
mastectomía radical izquierda modificada. La patología revela una medida de carcinoma ductal
infiltrante de tamaño de 3 cm de tamaño con ganglios linfáticos negativos. El estado del
receptor estrogénico es negativo. ¿Cuál es la terapia adyuvante más apropiada para este
paciente?
a) Quimioterapia (multiagente)
a) Radioterapia de haz externo
b) Terapia de ultrasonido enfocada en alta energía
c) Tamoxifeno
d) Espera vigilante

7. Una mujer de 31 años se queja de una historia de 6 meses de diarrea con sangre, dolor
abdominal e intermitente fiebres. Tiene antecedentes de síndrome del intestino irritable pero
ha empeorado sus síntomas durante el anterior periodo de tiempo. Su historial médico pasado es
irrelevante. El examen físico revela distensión abdominal. Los sonidos intestinales están
presentes en todos los cuadrantes. Examen rectal, la inación revela múltiples fisuras anales.
¿Cuál prueba de diagnóstico es la más apropiadas para este paciente?
a) Anoscopia
b) Colonoscopia
c) Sigmoidoscopia flexible
d) Sigmoidoscopia rígida
e) No se requieren más pruebas de diagnóstico para este paciente.

8. Una mujer de 71 años se presenta a su médico de atención primaria, quejándose de sangrado


rectal. Ella tenía un poco calambres abdominales leves del lado izquierdo que disminuyeron en
minutos. Ella nunca ha tenido un episodio previo de sangrado rectal. El examen físico revela
leve dolor abdominal en el cuadrante inferior izquierdo sin evidencia de rebote ni
sensibilidad. Examen rectal no revela sangre fresca en la bóveda rectal. Colonoscopia revela
varias bolsas de la pared del colon sigmoide sin evidencia de sangrado o perforación. El resto de
la colonoscopia está dentro de los límites normales. El recuento de glóbulos blancos es
normal. ¿Cuál es el tratamiento más apropiado para este paciente?
a) Terapia antibiótica con ampicilina y gentamicina.
b) Hemicolectomía izquierda
c) Hemicolectomía derecha
d) Colectomía subtotal
e) Espera vigilante

9. Un hombre de 85 años es llevado al departamento emergencia, debido a dolor abdominal


agudo y distensión abdominal progresiva. Es residente de un asilo de ancianos. No ha estado
comiendo debido a náuseas progresivas. Las radiografías abdominales revelan una masa en colon
sigmoide ¿Cuál es el tratamiento inicial para este paciente?
a) Enema de gastrografin
b) Dieta rica en fibra
c) Lactulosa
SS – Med10
d) Descompresión del tubo rectal
e) Resección quirúrgica

10. Una mujer de 41 años se queja de dolores de cabeza constantes. Durante los
últimos 6 meses. Ella también se ha quejado de infertilidad femenina y no ha
podido tener hijos, a pesar de tener relaciones sexuales sin protección con su esposo durante los
últimos 15 años. Examen físico revela déficits en los movimientos extraoculares bilateralmente.
El examen de los senos revela ginecomastia bilateral. ¿Cuál de las siguientes pruebas de
laboratorio sería más útil para diagnosticar a este paciente?
a) Ferritina
b) Hemoglobina
c) Hematocrito
d) Hierro
e) Prolactina

11. Una mujer de 41 años con enfermedad de Crohn se han realizado múltiples procedimientos
quirúrgicos. Ella recientemente se sometió a una ileostomía pero aún tiene evidencia de alguna
enfermedad yeyunal distal. Sus medicamentos actuales incluyen prednisona y ácido
aminosalicílico. ¿Cuál de los siguientes Efectos reductores de la terapia prolongada con
glucocorticoides son posibles para este paciente?
a) Producción de anticuerpos
b) Formación de colágeno
c) Disfunción de fibroblastos
d) Migración celular inflamatoria
e) Deterioro de heridas

12. Un hombre obeso de 49 años se presenta a su médico de atención primaria, para un examen
de seguimiento. El tiene una historia de diabetes mellitus no controlada y trastorno bipolar.
Sus medicamentos actuales incluyen litio y leche de magnesio. Examen físico del corazón,
pulmones, y abdomen están dentro de los límites normales. Los estudios de laboratorio revelan
calcio sérico de 14 mg / dL. ¿Cuál es lo más explicación probable de estos hallazgos?
a) Indiscreción dietética
b) Sobredosis de medicación
c) Síndrome de leche y álcali
d) Adenoma paratiroideo
e) Hiperplasia paratiroidea

13. Un hombre de 41 años tiene reflujo gastroesofágico crónico. Actualmente se maneja con
un bloqueador de H2. Al examen físico del corazón, pulmones y abdomen son dentro de los
límites normales. ¿Cuál de los siguientes factores, sería menos protector del esófago en
términos de la exposición continua inducida por esta condición?
a) Ligamento arqueado
b) Capacidad de vaciado gástrico
c) Efecto gravitacional
d) Productos secretores de glándulas salivales
e) Ondas peristálticas secundarias

14. Una mujer de 40 años se queja de dolor en el pecho y disfagia a los sólidos. Ella se presenta
a un especialista para evaluación. Se realizan estudios manométricos esofágicos y revelan
contracciones de alta amplitud y eventual relajación normal del esfínter esofágico inferior.
La ingestión de bario es normal. ¿Cuál es el diagnóstico más probable?
a) Espasmo muscular cricofaríngeo
b) Espasmo esofágico difuso
c) Esclerodermia
SS – Med10
d) Tuberculosis
e) Trastorno de deglución psicógena

15. Un niño de 5 años es llevado a la sala de emergencias después de ingerir


líquido limpiador de drenaje. El chico era dejado sin supervisión mientras su niñera estaba en la
tele teléfono. El niño esta ronco y tiene un evidente estridor. ¿Cuál es el tratamiento inicial más
apropiado para este paciente?
a) Antibióticos
b) Corticosteroides
c) Inducción de vómitos con ipecacuana
d) Colocación de sonda nasogástrica y lavado
e) Traqueotomía

16. Un hombre de 76 años con antecedentes de dolor vago en el cuadrante superior derecho,
pérdida de peso de 25 lb y anorexia previa, visita a su médico de atención primaria para su
evaluación. El examen físico revela ictericia escleral. El examen revela una masa abdominal en
el cuadrante superior derecho. Riñón, uréter y vejiga (KUB) revelan un cálculo circular de
fijación en el cuadrante superior derecho. Laparotomía exploratoria revela un proceso
neoplásico que involucra la vesícula biliar der y hígado. ¿Cuál es la patología más probable que
causa esta condición?
a) Adenocarcinoma
b) Sarcoma
c) Carcinoma de células escamosas
d) Carcinoma de células de transición
e) Granuloma tuberculoso
17. Una mujer de 38 años se presenta a su médico atención primaria, para evaluación de dolor
vago intermitente en el cuadrante superior derecho. Ella tiene antecedentes de hipotiroidismo e
hipertensión. Sus medicamentos actuales incluyen reemplazo sintético de hormona tiroidea y un
bloqueador de los canales de calcio. El examen físico revela dolor leve en el cuadrante superior
derecho hasta palpación profunda. La ecografía revela un cálculo biliar de 3 cm.
¿Qué tipo de piedra es probable que esté presente en este paciente?
a) Cálculo biliar negro
b) Cálculos biliares marrones
c) Cálculos biliares de oxalato de calcio
d) Cálculos de colesterol tipo I
e) Cáculos de colesterol tipo II

18. Una mujer de 46 años se presenta al departamento emergencia, quejándose de dolor en el


cuadrante superior derecho y fiebre a 102F°. El examen físico revela ictericia escleral y dolor
significativo en el cuadrante superior derecho a la palpación. Los signos peritoneales están
ausentes. Los sonidos intestinales están presentes. ¿Cuál de los siguientes debe incluirse en el
tratamiento inicial de este paciente?
a) Antibióticos
b) Coledochojejunostomía
c) Descompresión con tubo en T
d) Esfinterotomía endoscópica
e) Drenaje transhepático percutáneo

19. Un niño de 17 años es llevado a la sala de emergencias, después de sufrir dolor torácico y
disnea durante un juego de baloncesto. Examen físico revela un soplo crescendo-decrescendo
sistólico, se escuchó mejor en el segundo espacio intercostal derecho. El soplo se irradia a la
arteria carótida derecha. Radiografía de pecho revela un tamaño de corazón normal. ¿Cuál de
los siguientes hallazgos se esperaría que se vieran en un electrocardiograma en este paciente?
SS – Med10
a) Inversión de ondas T en derivaciones V1 – V4
b) Hipertrofia del ventrículo izquierdo
c) Bloque de rama derecha
d) Hipertrofia ventricular derecha
e) Hipertrofia auricular derecha

20. Un hombre de 72 años se derrumba/desmaya mientras camina en una tienda de Ping


Mall, esta sin pulso y apneico. No hay historia de trauma. Comienza la reanimación
cardiopulmonar hasta que llegue un escuadrón de rescate, Suplemento cardíaco avanzado, se
inicia el protocolo básico. Es declarado muerto 40 minutos más tarde. La autopsia revela
necrosis miocárdica con ruptura del ventrículo izquierdo. ¿Cuál de los siguientes es el factor de
riesgo más probable que contribuyó a su muerte?
a) Antecedentes familiares de diabetes mellitus.
b) Hipotensión
c) Obesidad
d) Estilo de vida sedentario
e) Trauma

21. Un hombre de 57 años es llevado a la sala de emergencias y se queja de disnea y dolor en el


pecho. Él también admite una pérdida de peso de 20 libras. Se queja de fiebres, escalofríos, y
sudores nocturnos. El examen físico revela adenopatía supraclavicular. El examen de tórax
revela sonidos del corazón a distancia. Los estudios de laboratorio revelan un recuento de
glóbulo blanco de 170,000 / mL. Radiografía de tórax y ecocardiografía revelan un derrame
pericárdico. ¿Cuál es la explicación más probable? nación de estos hallazgos?
a) Mixoma auricular
b) Fibrilación auricular
c) Linfoma
d) Carcinoma colorrectal metastásico
e) Pericarditis

22. Un varón recién nacido tiene una abertura del abdomen en la pared del ombligo. No tiene
otro registro médico previo o historia quirúrgica en el historial de nacimientos, ni pruebas
diagnosticas. El resto del examen físico y diagnóstico no hay datos relevantes. ¿Cuáles de los
siguientes hallazgos son más probables?
a) Labio leporino
b) Paladar hendido
c) Hernia de diafragma
d) Pericardio
e) Vejiga urinaria en retroperitoneo

23. Un trabajador de la construcción de 44 años se somete a una reparación de hernia inguinal


derecha. El procedimiento quirúrgico es sin acontecimientos notables. No tiene antecedentes
médicos o quirúrgicos previos. Regresa para seguimiento en el postoperatorio día 3, para un
control de heridas. La herida está limpia, seca e intacta. ¿Cuál es el período de convalecencia
óptimo requerido antes volver a trabajar para este paciente?
a) 1 semana
b) 4 semanas
c) 6 a 8 semanas
d) 12 semanas
e) Desconocido

24. Una mujer de 40 años se repara una hernia femoral derecha. Durante el procedimiento, el
canal femoral es disecado. Los límites anatómicos del canal femoral incluye cuál de los
siguientes?
SS – Med10
a) Ligamento de Cooper
b) Ligamento inguinal
c) Espina ciática
d) Ligamento lacunar
e) Nervio (femoral)

25. Un hombre de 53 años se somete a una prostatectomía radical, para el presunto cáncer de
próstata limitado a órganos. El factor importante para mantener la continencia después de la
prostatectomía radicales es la preservación de:
a) Cuello de la vejiga
b) Esfínter uretral externo
c) Complejo muscular elevador del ano
d) Nervi erigentes
e) Ligamentos puboprostáticos

26. Un hombre de 27 años tiene una adenopatía retroperitoneal voluminosa, después de una
orquiectomía radical para un tumor de célula germinal mixta. Su radiografía de tórax es
normal. Suero beta-humano gonadotropina coriónica (-hCG) y alfa-fetoproteína
(AFP) están marcadamente elevados. Las enzimas hepáticas están ligeramente elevadas, y el
paciente relata una historia de etanol en exceso. Recibe tres ciclos de quimioterapia. El reporte
revela una masa retroperitoneal de 3 cm, una normal radiografía de tórax y suero normal
hCG. Sin embargo, la AFP sérica es de 20 UI / ml (normal de 0 a 9 UI / ml). ¿Cuál es el siguiente
paso en el manejo de este paciente?
a) Tomografía computarizada (TC): biopsia con aguja guiada
b) Radioterapia de haz externo
c) Disección de ganglios linfáticos retroperitoneales
d) Quimioterapia de rescate
e) Marcadores seriales y tomografías computarizadas

27. Un hombre de 63 años está libre de enfermedad después de dos años con el bacilo. Terapia
con Calmette-Guerin para el carcinoma in situ y un grado 2, cáncer de vejiga en estadio
T1. Además de físico examen, cistoscopia y citología urinaria, evaluación en este momento debe
incluir:
a) Pielografía intravenosa
b) Biopsia uretral prostática
c) Biopsias aleatorias de la vejiga.
d) Citología selectiva del tracto superior
e) Citología miccional anulada, repetida tres veces

28. Un hombre de 78 años se presenta en el departamento de emergencias, para la evaluación


del cuadrante superior derecho por dolor progresivo, náuseas, vómitos y una pérdida de peso de
30 lb en el últimos 3 meses Tiene antecedentes médicos previos de colelitiasis, diabetes
mellitus, hipertensión y demencia. El examen físico revela ictericia escleral bilateralmente. El
examen abdominal revela ternura en la parte superior derecha cuadrante y una masa palpable
peritoneal. Los signos están ausentes. La tomografía computarizada revela ganglios pancreático,
duodenal, y coledocales. Hay un asimétrico engrosamiento de la vesícula biliar. ¿Cuál es lo más
probable hallazgo patológico en laparotomía exploratoria y biopsia?
a) Adenocarcinoma
b) Fibroma
c) Lipoma
d) Mixoma
e) Myoma
SS – Med10
29. Un niño de 8 años se somete a un ultrasonido para molestias persistentes en el
cuadrante superior derecho. No tiene antecedentes médicos o quirúrgicos previos.
Sin alergias conocidas y no toma medicamentos. Su madre tiene antecedentes de
cálculos biliares. Hallazgos de ultrasonido incluyen una dilatación fusiforme del
conducto biliar común. ¿Cuál es la explicación más probable para estos hallazgos?
a) Quiste de colédoco tipo I
b) Quiste de colédoco tipo II
c) Quiste de colédoco tipo III
d) Quiste de colédoco tipo IV
e) Quiste de colédoco tipo V

30. Un hombre de 18 años es apuñalado en su abdomen múltiples veces por un asaltante


durante un altercado que involucra venta de drogas ilícitas. Es llevado al departamento
emergencia para su evaluación. Él tiene cuatro heridas de arma blanca el abdomen: tres están
en el cuadrante superior derecho, y uno está en el cuadrante inferior izquierdo. Examen físico
La sección del abdomen revela protección/sensibilidad y rebote. El paciente es llevado a
cirugía para una laparotomía exploratoria. Una lesión penetrante en la vesícula biliar. Se
encuentra la vejiga. ¿Cuál de las siguientes viseras asociadas es probable que se lesionen?
a) Aorta
b) Colon
c) Riñón
d) Hígado
e) Vejiga urinaria

31. Una mujer de 62 años se presenta a su médico de atención primaria con tos. Ella también se
queja de hemoptisis. La historia social revela una historia de 55 paquetes al año de fumar
cigarrillos. Ella es una alcohólica en recuperación. Examen físico revela sibilancias
bilaterales. Cardíaco, pulmonar y los exámenes abdominales no son notables. Laboratorio los
valores revelan calcio sérico de 13 mg / dL. Proteína sérica y la electroforesis no muestra picos
anormales. ¿Cuál es el diagnóstico más probable?
a) Síndrome de Goodpasture
b) Mieloma
c) Adenoma renal
d) Carcinoma de células pequeñas de pulmón
e) Carcinoma de células escamosas de pulmón

32. Un niño de 10 años es llevado a su centro de atención primaria, para la evaluación de la


ronquera persistente. Comenzó a participar con su coro escolar y nota que su ronquera empeora
con el canto. El examen físico del corazón, pulmones y abdomen sin relevancia. Examen
laríngeo flexible de fibra óptica revela múltiples lesiones en sus verdaderas cuerdas vocales.
¿Cuál es el diagnóstico más probable?
a) Reflujo gastroesofágico
b) Inflamación granulomatosa de la faringe.
c) Papiloma laríngeo
d) Nódulo del cantante
e) Carcinoma de tiroides

33. Un hombre de 75 años se presenta a su médico de atención primaria debido a la


ronquera. Tiene historia de fumar con IPA de 60 al año. También se queja de pérdida de peso de
25 lb en los últimos 4 meses. La laringoscopia directa revela una masa sésil en la cuerda vocal
alta derecha. También tiene un ganglio linfático palpable a lo largo del cuello anterior derecho
de la cadena de ganglios linfáticos. Si se encuentra displasia en la biopsia de la lesión laríngea,
¿cuál es el diagnóstico más probable?
a) Adenoma
SS – Med10
b) Pólipo laríngeo
c) Laringitis
d) Enfermedad quística mucoepidermoide
e) Carcinoma de células escamosas

34. Un estudiante universitario masculino de 21 años se presenta a la clínica para un examen de


rutina al comienzo del semestre de otoño. Tiene antecedentes de síndrome de intestino
irritable. Examen físico del corazón, pulmones y abdomen no son notables. Examen
genitourinario revela que los testículos descienden bilateralmente. Un varicocele izquierdo de
grado 1 está presente. No hay masa testicular. El pene no está circuncidado y el prepucio está
incapaz de retraerse detrás del glande. ¿Cuál es el diagnóstico más probable?
a) Balanitis
b) Hipospadias
c) Epispadias
d) Parafimosis
e) Fimosis

35. Un hombre blanco de 71 años se presenta a su médico de atención primaria quejándose de


un historial de un mes de nicturia, poliuria, y dificultad para comenzar y detener su orina
corriente. Su síntoma según la Asociación Americana de Urología tiene un puntaje de
17/35. Examen físico de la próstata revela una glándula agrandada sin masas. Sus testículos son
descendió bilateralmente. Tiene un pequeño hidrocele derecho que transilumina. Su antígeno
prostático específico (PSA) el nivel es de 6 ng / ml y el análisis de orina es negativo. Comenzó
con dutasterida 0,5 mg al día. ¿Cuál es un resultado probable de tomar este medicamento?
a) Disfunción eyaculatoria
b) Cambio máximo en el flujo urinario
c) El tamaño de la próstata disminuye en un 25%.
d) El PSA sérico aumenta en un 50%
e) La puntuación de los síntomas permanece sin cambios

36. Un hombre blanco de 34 años tiene una ampliación indolora de su testículo derecho en los
últimos 4 meses. Es llevado a su médico de atención primaria por su novia, quien lo urge para
buscar evaluación. Recientemente se ha deprimido por este problema. Tenía una criptorquidia
derecha testículo cuando era bebé, que fue corregido quirúrgicamente. UNA la ecografía
escrotal confirma la presencia de 3 cm masa testicular derecha hipoecoica. ¿Cuál es el
diagnostico más probable?
a) Coriocarcinoma
b) Carcinoma embrionario (células germinales mixtas)
c) Tumor endodérmico (saco vitelino)
d) Seminoma
e) Teratoma

37. Un hombre de 51 años tiene una masa intracraneal y se someterá a resección. El


procedimiento quirúrgico se realiza a través de un enfoque transoccipital. En este enfoque, el
paciente desarrolla un líquido cefalorraquídeo (CSF) de fuga. ¿Cuál de las siguientes
afirmaciones es verdadera con respecto al LCR?
a) Las vellosidades aracnoideas actúan como válvulas de dos vías.
b) Las vellosidades aracnoideas se abren a una presión de 5 mm Hg.
c) El LCR se absorbe a través de las raíces espinales.
d) CSF entra por el agujero de Magendie.
e) El volumen total de LCR es de 150 L.

38. Una estudiante universitaria de 19 años conduce bajo la influencia del alcohol, a pesar de
las recomendaciones de amigos de No conducir. Ella es golpeada por otro conductor. La fuerza
SS – Med10
del impacto hace que golpee el área temporal de ella cráneo contra la ventana. Ella
desarrolla un dolor de cabeza leve pero no pierde el conocimiento. Varias horas
después, ella desarrolla un fuerte dolor de cabeza con náuseas y vómitos.
¿Cuál es el diagnóstico más probable?
a) Infección bacteriana
b) Aneurisma de bayas
c) Hematoma epidural
d) Hematoma subaracnoideo
e) Hemorragia subdural

39. Un hombre de 59 años se presenta a su médico de atención primaria, quejándose de pérdida


audición progresiva del lado derecho y marcha con inestabilidad. Él dice que cuando usa el
teléfono, debe usar su oído izquierdo para escuchar en lugar de su oreja derecha. Tiene
antecedentes médicos de hipertenia. Sus medicamentos actuales incluyen un bloqueador de
canal de calcio. El examen físico revela pérdida del reflejo corneal derecho y debilidad
facial. Cardíaco, pulmonar, y los exámenes abdominales están dentro de los límites normales.
¿Cuál es el siguiente paso más apropiado en el diagnóstico de este paciente?
a) Prueba audiométrica
b) Prueba potencial evocada del tronco encefálico
c) Tomografía computarizada de la cabeza sin contraste
d) Resonancia magnética de la cabeza
e) Nistagmografía

40. Un hombre de 47 años con antecedentes de enfermedad pulmonar en etapa terminal del
pulmón derecho está programado para un trasplante de pulmón. La función cardíaca
preoperatoria es buena. No tiene antecedentes de defectos congénitos. ¿Cuál de las siguientes
es la incisión quirúrgica más adecuada para este paciente?
a) Chevron abdominal
b) Toracotomía lateral
c) Línea media abdominal
d) Toracotomía anterior transversal
e) Pfannenstiel

41. Un hombre de 47 años con múltiples problemas médicos y la enfermedad del parénquima
pulmonar en etapa terminal se somete trasplante pulmonar. Tiene antecedentes médicos
previos de enfermedad pulmonar obstructiva. El tiene un tío con fibrosis quistica. Su padre tiene
enfermedad pulmonar restrictiva, y su hermano tiene hipertensión pulmonar. ¿Cuál de los
siguientes Lowing presagia la mejor supervivencia después del trasplante pulmonar para este
paciente?
a) Carcinoma broncogénico
b) Fibrosis quística
c) Enfermedad pulmonar obstructiva
d) Hipertensión pulmonar
e) Enfermedad pulmonar restrictiva

42. Un niño de 4 años está en la lista de espera para un trasplante de hígado. Tiene enfermedad
hepática en etapa terminal y actualmente está hospitalizado por hemorragia varicosa
esofágica. ¿Cuál es la causa más probable de insuficiencia hepática en este paciente?
a) Atresia biliar
b) Hepatitis A
c) Cirrosis biliar primaria
d) Colangitis esclerosante primaria
e) Tuberculosis
SS – Med10
43. Una mujer de 23 años que se queja de heces grasosas abundantes y dolor,
debilidad generalizada y pérdida de cabello y acude a su médico de atención
primaria para su evaluación. Examen físico del corazón, pulmones y abdomen
normales. Ella no tiene rebote. Los sonidos intestinales están presentes en todos los
cuadrantes. El examen pélvico fue diferido a solicitud del paciente. ¿Cuál es la
explicación más probable de estos hallazgos?
a) Úlcera gástrica con sangrado.
b) Malabsorción de glucosa
c) Menstruación
d) Insuficiencia pancreática
e) Tumor pituitario

44. Una mujer de 27 años, con 12 horas después trasplante de páncreas cadavérico y
actualmente en el quirófano unidad de Cuidados Intensivos. Ella tiene un historial médico de
insulinodependiente desde los 5 años. Sus signos vitales son normales. El pecho está claro para
la auscultación y el examen cardíaco revela un ritmo regular es limpio, seco e intacto. ¿Cuál es
el mejor método para controlar el páncreas trasplantado?
a) Nivel sérico de amilasa
b) Nivel de glucosa en suero
c) Nivel de insulina sérica
d) Ecografía de los vasos pancreáticos.
e) Nivel de amilasa urinaria

45. Traen a un hombre de 44 años con pancreatitis recurrente al departamento de emergencias


con otro episodio de pancreatitis ¿Qué factores se relacionan con la gravedad de su afección?
a) Edad
b) Nivel de glucosa en sangre de 300 mg / dL
c) Nivel de lactato deshidrogenasa de 400 UI / L
d) Calcio sérico de 6 mg / dL
e) Nivel de hematocrito en suero del 29%

46. Un hombre de 41 años con una larga historia de cálculos renales e hipercalcemia se
encuentra que tiene un adenoma de la glándula paratiroidea superior derecha. El va a sufrir
escisión quirúrgica de esta lesión. ¿Cuál es la mejor cirugía punto de referencia para esta lesión?
a) Bifurcación de las arterias carótidas.
b) Seno carotídeo
c) Unión de la arteria tiroidea inferior y recurrente nervio laríngeo
d) Unión del tercio superior y medio de la tiroides glándula
e) Nervio laríngeo recurrente

47. ¿Cuál de las siguientes técnicas se utiliza mejor para definir una glándula paratiroides
agrandada?
a) Tomografía computarizada del cuello
b) Doble trazado de imágenes
c) Resonancia magnética del cuello
d) Angiografía tirocervical
e) Ultrasonido

48. Un hombre de 44 años con enfermedad renal en etapa terminal se somete con trasplante
renal. Tiene un historial médico previo de hiperparatiroidismo. Seis meses después del
trasplante renal, su calcio sérico sigue siendo 13 mg / dL. ¿Cuál de los siguientes hallazgos de
laboratorio son posible en este paciente?
a) Fosfato sérico elevado
b) Elevación sérica de ácido láctico deshidrogenasa
SS – Med10
c) Orina elevada en calcio
d) Creatinina en orina elevada
e) Proteína urinaria elevada

49. Un hombre de 46 años se presenta a su médico de atención primaria, para la evaluación de


una lesión cutánea. Se queja de hipopigmentación de la piel de su espalda baja. Él tiene
antecedentes médicos previos de eccema y carcinoma basocelular. Es un granjero que pasa
mucho tiempo al aire libre. ¿Qué células son responsables de esta condición?
a) Adipocitos
b) Células productoras de queratina
c) Células de Langerhans
d) Melanocitos
e) Células de Merkel

50. Un hombre de 69 años se presenta a su dermatólogo con un lesión presente en su nariz. Es


un jardinero que gasta gran parte de su tiempo al aire libre. Él tiene un antecedente médico
previo de rinitis alérgica, hipertensión y diabetes mellitus. Sus medicamentos actuales incluyen
un betabloqueante y un hipoglucemiante oral. Examen físico de su la nariz revela una lesión
papular brillante y elevada con pequeñas vasos sanguíneos. ¿Cuál es el diagnóstico más
probable?
a) Carcinoma de células basales
b) Histiocitosis X
c) Melanoma
d) Queratosis seborreica
e) Carcinoma de células escamosas

51. Una mujer negra de 29 años se presenta a su médico atención primaria debido a un
crecimiento en su oreja izquierda, que ocurrió después de que le perforaron la oreja por
primera vez hace una semana. Ella notó que su oído parecía desarrollarse un crecimiento en él
con bastante rapidez. Ella nunca había tenido su oído perforado antes. ¿Cuál es la explicación
más probable para ¿estos hallazgos?
a) Carcinoma de células basales
b) Nevo azul
c) Melanoma juvenil
d) Queloide
e) Molusco contagioso

52. Una mujer asiática-estadounidense de 52 años tiene una pigmentación melanótica de la


mucosa bucal, labios y dedos. La colonoscopia revela hamartomas en todo el tracto
gastrointestinal Los pólipos fueron eliminados debido a su mayor riesgo de cáncer. ¿Qué otro
cáncer es asociado a esta condición?
a) Cáncer de cuello uterino
b) Cancer de RIÑON
c) Cáncer de hígado
d) Cáncer de ovarios
e) Cáncer de páncreas

53. Una mujer de 45 años se queja de diarrea crónica y constipación. Se realiza una
colonoscopia y una biopsia de una lesión en su íleon. El informe de patología muestra que el
tumor está compuesto de células neuroendocrinas. ¿Cuál es el tratamiento médico para esta
afección?
a) Corticosteroides intravenosos
b) Corticosteroides tópicos
c) Furosemida
SS – Med10
d) Octreotida
e) Tetraciclina

54. Un hombre de 18 años es llevado a la sala de emergencias con dolor


abdominal insoportable y repentino localizado hacia el cuadrante inferior derecho,
náuseas y vómitos, leves fiebre y taquicardia leve. Tiene antecedentes médicos previos como
historia de la otitis media recurrente. Examen físico revela marcado rebote en el cuadrante
inferior derecho. El recuento de glóbulos blancos en suero es de 18,000 / mL. La radiografía
KUB revela gas intestinal en el intestino delgado y grueso. ¿Cuál es el diagnóstico más probable?
a) Apendicitis
b) enfermedad de Crohn
c) Diverticulitis
d) Pancreatitis
e) Colitis ulcerosa

55. Una mujer de 39 años se presenta a la sala de emergencia quejándose de dolor abdominal
severo. Ella tiene antecedentes de enfermedad ulcerosa péptica. Examen físico revela
protección y rebote. Ella es llevado a la sala de operaciones para exploración por laparotomía
exploratoria. Durante el procedimiento, el cirujano que abre el ligamento gastrosplénico para
alcanzar el saco menor corta accidentalmente una arteria. ¿Cuál de los siguientes vasos es el
muy probablemente uno herido?
a) Arteria gastroduodenal
b) Arteria gástrica izquierda
c) Arteria gastroepiploica izquierda
d) Arteria gástrica derecha
e) Arteria esplénica

56. Una mujer de 2 años es llevada a la sala de emergencia debido a varios episodios de
sangrado rectal. En un escáner de perfusión de tecnecio-99m revela un tamaño de 3 cm y
excreción ileal ubicada a 50 cm de la válvula ileocecal. ¿Cuál de los siguientes tipos de tejido
ectópico es probable que esta estructura contenga?
a) Duodenal
b) Esofágico
c) Gástrico
d) Hepático
e) Yeyunal

57. Una mujer de 39 años se queja de dolor epigástrico con comienzo durante los últimos 3 o 4
meses. Ella admite una historia de problemas crónicos de espalda. Ella nota aumento de peso de
20 lb en los últimos 4 meses. Ella niega el uso de agentes antiinflamatorios esteroideos. Ella
niega las náuseas y vómitos. Examen físico del corazón, pulmones, y el abdomen están dentro
de los límites normales. ¿Cuál es el más probable patógeno asociado con esta condición?
a) Escherichia coli enterohemorrágica
b) Escherichia coli
c) Helicobacter pylori
d) Shigella sonnei
e) Streptococcus pyogenes

58. Un hombre de 59 años resultó herido en un accidente automovilístico. La tomografía


computarizada abdominal revela una ruptura del bazo. Su presión arterial es de 90/40 mm Hg,
y su pulso es de 140 latidos por minuto. El paciente es llevado a una laparotomía -
esplenectomía. ¿Cuál de los siguientes trastornos seria probable después de este procedimiento?
a) Anemia
b) Basofilia
SS – Med10
c) Eosinofilia
d) Trombocitopenia
e) Trombocitosis

59. Un hombre de 19 años fue pateado en el abdomen durante un pelear en un


bar. Fue a su médico de atención primaria, quien ordenó una tomografía computarizada, que
reveló un hematoma esplénico subcapsular. Le dijeron al hombre que restringiera la actividad
física. Dos semanas después, se presenta a la emergencia debido a dolor abdominal severo. Él se
somete a una esplenectomía. Después de la operación, se ordena un frotis periférico, qué tipo
de célula se puede encontrar en este paciente?
a) Punteado basófilo
b) Células de la ampolla
c) Cuerpos de Howell-Jolly
d) Glóbulos rojos nucleados.
e) Esferocitos

60. Un hombre afroamericano de 15 años fue sometido una esplenectomía después de sufrir una
lesión en un cuchillo durante una pelea. Se presenta a su médico de atención primaria para un
examen físico ya que su madre leyó en la web que él está en mayor riesgo de infección. ¿Cuál de
las siguientes vacunas debería recibir para prevenir infecciones graves?
a) Rubéola
b) Sarampión
c) Tétanos
d) Vacunas contra organismos encapsulados comunes.
e) Varicela

61. Una mujer de 53 años se presenta a su médico de atención primaria con un historial de
dolor de cuello de 12 meses. Ella se queja de un aumento de peso de 15 libras y malestar
generalizado. Ella tiene un historial médico pasado de hipertensión y diabetes mellitus. Sus
medicamentos actuales incluyen un hipoglucémico oral. El examen físico revela ternura a lo
largo el curso de la glándula tiroides sin evidencia de un trastorno de masa discreta. ¿Cuál es el
diagnóstico más probable?
a) Tiroiditis aguda
b) Tiroiditis de Hashimoto
c) Carcinoma papilar de tiroides
d) Tiroiditis de Riedel
e) Tiroiditis subaguda

62. Una mujer de 47 años con antecedentes de masa de tiroides izquierda, se somete a
lobectomía tiroidea izquierda. Patología revela un carcinoma papilar de 1.3 cm sin evidencia de
extensión extracapsular. ¿Cuál es el siguiente paso mas apropiado en el tratamiento de este
paciente?
a) Radioterapia de haz externo
b) Quimioterapia multiagente
c) Tiroidectomía subtotal
d) Tiroidectomía total
e) Espera vigilante con seguimiento periódico

63. Se está sometiendo a un hombre de 34 años con un nódulo tiroideo a una exploración del
cuello. Durante el procedimiento, es posible que se someterá a tiroidectomía. ¿Cuál de las
siguientes declaraciones bajas sobre el nervio laríngeo superior y la inervación de la glándula
tiroides es correcta?
a) La lesión del nervio provoca la inclinación de las cuerdas vocales. durante la fonación
b) La lesión nerviosa puede pasar desapercibida en los cantantes.
SS – Med10
c) El nervio rara vez está en riesgo durante la cirugía de tiroides
procedimientos
d) El nervio laríngeo superior es principalmente un nervio motor.
e) El nervio laríngeo superior es principalmente un nervio sensorial.

64. Un hombre de 19 años salta del tercer piso de su dormitorio en un aparente intento de
suicidio. Es llevado a al departamento de emergencias inconsciente. Tiene visible lesiones en la
cabeza y extremidades inferiores. Tiene un pulso de 110 late por minuto pero es apneico. ¿Cuál
es la mejor vía aérea para el manejo de este paciente?
a) Intubación nasotraqueal
b) Intubación oral
c) Intubación oral con elevación del mentón
d) Traqueotomía
e) La intubación no es necesaria para este paciente.

65. Una mujer de 21 años es apuñalada en el pecho por su novio. Ella es llevada al
departamento de emergencias para su evaluación. Su presión arterial es de 130/80 mmHg, y su
pulso es de 90 latidos por minuto. Examen físico revela una herida de arma blanca en el quinto
intercostal izquierdo espacio en la línea medioclavicular. El examen del cuello es normal.La
tráquea en la línea media y las venas yugulares no están distorsionadas. Ella tiene una
disminución de los sonidos respiratorios en el campo pulmonar izquierdo. ¿Cuál de los siguientes
diagnósticos puede ser descartado sobre la base de la información anterior?
a) Hemotórax izquierdo grande
b) Neumotórax abierto
c) Taponamiento pericárdico
d) Ruptura del bronquio principal izquierdo
e) Tensión neumotoraxica

66. Un hombre de 41 años sufre una amputación traumática de tres de sus dedos en una
cortadora de carne. No tiene antecedentes médicos ni historia clínica. ¿Cuál de las siguientes
modalidades debería ser utilizado para transportar los dedos amputados con el paciente?
a) Colocar en una bolsa de plástico limpia y empacar con hielo seco.
b) Colóquelo en una bolsa de plástico limpia llena de agua pura a temperatura ambiente
c) Colocar en una bolsa de plástico limpia en un cofre lleno de hielo picado y agua.
d) Colocar en una bolsa de plástico limpia llena de agua caliente.
e) Envuelva los dedos amputados con una gasa seca estéril.

67. Una mujer de 19 años se presenta a la sala de emergencia después de sufrir una lesión en
su ojo derecho mientras coloca su lente de contacto. Ella tiene un significativo dolor de ojo
derecho. Ella tiene un historial médico previo de alergias. El examen físico revela un abrasión
simple. La prueba de fluoresceína se realiza y revela que no evidencia de un defecto epitelial
manchado. Esto descarta el posibilidad de cuál de los siguientes?
a) Infección bacteriana
b) Iritis
c) Trauma
d) Infección viral
e) Úlcera

68. Un hombre de 29 años que trabaja en una fábrica sufrió una lesión por cuerpo extraño en su
ojo derecho cuando un pedazo de metal salió disparado de una cinta transportadora. Él es
llevado al departamento de emergencias para evaluación. Examen físico del ojo derecho revela
un cuerpo extraño metálico con un anillo de óxido. ¿Cuál es el instrumento más útil para
eliminar este cuerpo extraño?
a) Pegamento de cianoacrilato
SS – Med10
b) Rebabas en los ojos o Eye burr
c) Spud de ojo
d) Punta de aguja fina
e) Agua estéril y alcohol

69. Un profesor de química de 37 años sostiene un vaso químico con ácido el cual salpico en su
ojo derecho mientras intenta formar una demostración para su clase de ciencias de la escuela
secundaria. Él tiene mucho dolor. Mientras está en el aula y esperando para que una ambulancia
lo transporte al hospital, que de las siguientes intervenciones deben realizarse?
a) Instilación de gotas para los ojos con solución salina normal
b) Colocación del parche ocular
c) Lavar los ojos con 1 a 2 L de solución salina normal.
d) Colocación del ojo bajo la luz solar directa.
e) Vigilante espera hasta que llegue la ambulancia

70. Un trabajador de la construcción de 37 años sufrió una lesión en el muslo derecho después
de que una grúa cayó sobre su pierna en el sitio de trabajo. Lo llevan al departamento de
emergencia para evaluar y tiene dolor significativo en la pierna derecha y dolor con
estiramiento pasivo. La pierna está tensa a la palpación. ¿Cuál es la medida de presión
intracompartimental más probable que debe utilizarse la pierna derecha de este paciente?
a) 5 mm Hg
b) 10 mm Hg
c) 15 mm Hg
d) 25 mm Hg
e) 35 mm Hg
PREGUNTAS
71. Una mujer de 41 años que limpia casas para ganarse la vida visita a su médico de atención
primaria quejándose de sensibilidad en su rodilla derecha. El dolor es constante y ha sido
presente por 3 semanas. Ella está en una relación monógama. El examen físico revela que su
rodilla está ligeramente hinchado y sensible. Los exámenes cardíaco, pulmonar y abdominal
están dentro de los límites normales. Se realiza una aspiración sinovial. La evaluación no revela
evidencia de cristales o bacterias. ¿Cuál es el diagnóstico más probable?
a) Bursitis
b) Artritis infecciosa
c) Artritis reumatoide
d) Tromboflebitis séptica
e) Artritis infecciosa inducida por trauma

72. Un niño de 12 años que es el lanzador estrella de equipo de la liga de beisbol, se queja de
dolor en el hombro derecho. Este es su brazo de lanzamiento. No tiene antecedentes médicos o
quirúrgicos previos. El examen físico revela debilidad del manguito rotador. ¿Cuál es el
tratamiento más apropiado para este paciente?
a) Inyección de corticosteroides.
b) Corticosteroides intravenosos
c) Descanso, elevación y agentes antiinflamatorios.
d) Colocación de la honda
e) Reparación quirúrgica

73. Un hombre de 65 años con antecedentes de enfermedad coronaria se somete un bypass


aortobifemoral. ¿Cuál de las siguientes maniobras de manejo intraoperatorio disminuirá su
riesgo de infarto de miocardio intraoperatorio?
a) Beta bloqueadores
b) Bloqueadores de los canales de calcio
SS – Med10
c) Administración de solución salina normal en lugar de lactato Ringer
d) Uso de propofol
e) Uso de morfina

74. A un hombre sano de 35 años se le diagnostica un hernia inguinal. No tiene


antecedentes de sangrado anormal. ¿Cuál de las siguientes pruebas es absolutamente necesaria
antes a llevarlo al quirófano?
a) Hematocrito
b) Recuento de plaquetas
c) Potasio
d) Recuento de glóbulos blancos
e) Ninguna de las anteriores

75. Un hombre de 50 años tiene diarrea después de una cirugía sin complicaciones de resección
intestinal. La elección de fluidos más conveniente es:
a) Solución salina normal
b) Solución salina media normal con 20 mEq de potasio
c) D5W con bicarbonato de 3 amperios
d) La solución de Ringer lactato
e) D5NS

76. Un hombre de 27 años es llevado a la sala de emergencias después de cortarse la mano con
un cuchillo mientras intentaba cortar un panecillo. Tiene antecedentes médicos previos de
recurrencia de infecciones de los senos paranasales. Su historial quirúrgico previo es notable por
reparación de una fractura nasal. El examen físico revela un laceración limpia de 4 cm a lo largo
del aspecto palmar de su mano. Los principios más relevantes para este caso serían:
a) Manejo suave del tejido y cierre sin tensión.
b) Injerto de piel cuando sea necesario para cubrir el área
c) Desbridamiento de tejido desvitalizado.
d) Uso de una esponja de vacío para promover la curación.
e) Reconstrucción de colgajo con tejido vascularizado.

77. Un hombre de 68 años es llevado a la sala de emergencias se queja de dolor abdominal y de


piernas hace 2 semanas. Tiene antecedentes de hipertensión e hipercolesterolemia; él pesa 290
lb. Examen físico revela una masa abdominal pulsátil en la línea media. El las extremidades
inferiores tienen pulsos desiguales. ¿Cuál de los siguientes es el mejor paso en la evaluación de
este paciente?
a) Aortograma
b) Tomografía computarizada del abdomen y la pelvis
c) Ultrasonido, riñones y vejiga.
d) Ultrasonido, hígado y bazo.
e) Venacavogram

78. Una mujer de 28 años se presenta a su médico para evaluación de un bulto en su seno
derecho encontrado en autoexamen. Ella tiene antecedentes familiares de cáncer de mama en
que su madre murió a los 40 años por esta condición. La madre tuvo una mastectomía radical
modificada por quimioterapia. El examen físico revela un protuberancia mamaria que es
libremente móvil y bien circunferenciada. No hay hoyuelos, asimetría o retracciones. La lesión
mide 2 cm. ¿Cuál es el siguiente paso en el manejo de este paciente?
a) Biopsia de la lesión con guía ecográfica.
b) Mamografía seguida de tomografía computarizada estereotáctica
c) Prueba para el gen BRCA
d) Ultrasonido del seno y consideración para el seno biopsia
e) Espera vigilante y examen de seguimiento por pri-Mary Care médico en 1 año
SS – Med10

79. Un hombre de 28 años con antecedentes de dolor abdominal recurrentes y


diarrea con sangre se presentan a su médico quejándose de dolor rectal
significativo con evacuación intestinal. Ha perdido 15 libras en los últimos 3 meses.
El examen físico revela dolor en el cuadrante inferior derecha e izquierda a la palpación. Los
valores de laboratorio revelan un hematocrito del 28% y un eritrocito elevado velocidad de
sedimentación. Colonoscopia realizada en este el paciente probablemente revelaría:
a) Lesión de masa colónica
b) Diverticulosis
c) Hemorroides internas
d) Intestino normal
e) Mucosa colónica y rectal engrosada friable

80. Un hombre de 55 años se presenta a su médico quejándose de poliuria, polidipsia, polifagia


y sarpullido rojo y escamoso en su cara en las últimas 2 semanas. Él orina 18 veces / día con una
buena fuerza de flujo. La glucemia en ayunas fue 325 mg / dL. Ha perdido 20 libras en los
últimos 2 meses y nunca ha tenido niveles elevados de glucosa en sangre en el pasado. Examen
físico del corazón, pulmones y abdomen normal. ¿Cuál es la explicación más probable por estos
hallazgos?
a) Diabetes mellitus tipo I
b) Diabetes mellitus tipo II
c) Glucagonoma
d) Insulinoma
e) Enfermedad de Verner-Morrison

81. Un hombre de 72 años con un historial de 80 años de fumar, visita a su médico quejándose
de debilidad y malestar general. Recientemente ha desarrollado disfagia para alimentos
sólidos. Ha perdido 15 libras en los últimos 3 meses. El examen físico revela linfadenopatía
supraclavicular derecha. Los exámenes cardíacos y pulmonares son sin complicaciones. Sin
rebote a la palpación abdominal ¿Cuál de los siguientes estudios proporcionará más diagnóstico
definitivo?
a) Esofagograma de bario
b) Tomografía computarizada del abdomen y la pelvis
c) Esofagogastroscopia con biopsia.
d) Resonancia magnética del abdomen
e) Ultrasonido del cuadrante superior derecho

82. Una mujer afroamericana de 42 años se somete a una colecistectomía laparoscópica por
dolor crónico en el cuadrante superior. La tomografía computarizada demostró cálculos biliares
y líquido colecístico. El procedimiento quirúrgico fue incompleto. Análisis patológico de los
cálculos biliares revela cálculos de bilirrubinato de calcio. ¿Cuál es la explicación más probable?
nación de estos hallazgos?
a) Altos niveles de colesterol en suero
b) Niveles elevados de lípidos en suero
c) Dieta alta en grasas
d) Anemia falciforme
e) Tumor

83. Una mujer de 36 años se queja de ictericia y hinchazón periférica oral. Se ordena un
ecocardiograma y el se determina que el paciente tiene insuficiencia cardíaca derecha con
congestión hepática y edema periférico. No se le detecta ningún soplo. ¿Cuál es la explicación
más apropiada para estos hallazgos?
a) Estenosis aórtica
b) Comunicación interauricular
SS – Med10
c) Ductus arterioso permeable
d) Tetralogía de Fallot
e) Defecto septal ventricular

84. Un varón recién nacido a término niño de una mujer de 27 años de edad y con abuso de
drogas venosa tiene una pequeña hernia umbilical. Sus signos vitales son estables. Su cardíaco y
los exámenes pulmonares no son contributivos.
¿Cuál de las siguientes es la explicación más probable para ¿este descubrimiento?
a) Foramen oval permeable
b) Conducto onfalomesentérico permeable
c) Anillo umbilical permeable
d) Uraco patente
e) Conducto vitelino permeable

85. Un hombre de 28 años se presenta en el departamento de emergencias. Se queja de dolor


en el costado izquierdo, náuseas y vómitos. El examen físico del corazón y los pulmones es
normal. Se queja de sensibilidad costovertebral. Análisis de orina revela microhematuria (5
glóbulos rojos / alta potencia campo). La tomografía computarizada revela un cálculo renal
izquierdo de 4 mm, mientras que KUB revela un patrón normal de gas intestinal y no evidencia
de calcificaciones. ¿Cuál es la explicación más probable de estos hallazgos?
a) Calculo de oxalato de calcio
b) Calculo de monohidrato de fosfato de calcio
c) Calculo deshidratada de fosfato de calcio
d) Pequeño calculo de estruvita
e) Calculo de ácido úrico

86. Una mujer de 47 años con pancreatitis de cálculos biliares está hospitalizada. Ella tiene
antecedentes de hipertensión e hipercolesterolemia. La colecistectomía temprana está indicada
para prevenir cuál de las siguientes complicaciones?
a) Colangitis
b) Pancreatitis recurrente
c) Úlcera gástrica
d) Íleo biliar
e) Perforación de la vesícula biliar

87. Un hombre de 56 años se queja de tos recurrente y hemoptisis. Tiene antecedentes de


neumonías recurrentes. Él es un no fumador y no tiene riesgo laboral de pulso enfermedad
pulmonar. El examen físico revela disminución la respiración en el lóbulo superior derecho. La
radiografía de tórax revela una pequeña masa del cuadrante superior derecho. La broncoscopia
revela un angioma ¿Cuál es el tratamiento más apropiado para este paciente?
a) Antibióticos
b) Corticosteroides
c) Lobectomía izquierda
d) Lobectomía del lóbulo superior derecho
e) Espera vigilante

88. Un hombre de 55 años se presenta a su médico de atención primaria, después de notar un


poco de orina teñida de sangre hace 1 semana. Niega cualquier dolor actual y fiebre. Su historial
médico pasado incluye pulso obstructivo crónico enfermedad pulmonar de muchos años de
fumar cigarrillos. La tomografía computarizada revela quistes renales bilaterales simples,
próstata agrandamiento y engrosamiento asimétrico del lado izquierdo de la vejiga. La
hidronefrosis izquierda también está presente. Su análisis de orina es positivo para hematuria
microscópica. ¿Cuál es el diagnóstico más probable?
a) Nefrolitiasis
SS – Med10
b) Cancer de prostata
c) Carcinoma de células renales
d) Carcinoma de células de transición de la vejiga.
e) Infección del tracto urinario

89. Un hombre de 30 años se somete a una tomografía computarizada de su abdomen después


de un accidente automovilístico. Él era un desenfrenado conductor y fue arrojado del
vehículo. Se encuentran lesiones abdominales agudas. La tomografía computarizada revela
riñones bilaterales agrandados con quistes múltiples presentes en diferentes tamaños. El riñón
derecho mide 15 cm y el niño izquierdo mide 16 cm de largo. Examen físico de la corazón,
pulmones y abdomen están dentro de los límites normales, aparte de una leve sensibilidad a la
palpación en el cuadrantes superiores derecho e izquierdo. ¿Cuál de los siguientes las patologías
del sistema nervioso central son más fuertes asociado con este hallazgo?
a) Círculo de aneurisma de Willis
b) Cisticercosis
c) Infarto
d) Glioma
e) Hematoma subdural

90. Una mujer de 46 años con enfermedad renal poliquística recibio un trasplante de riñón. A
los pocos minutos de la anastomosis de la arteria renal a la respectiva arteria y vena ilíaca
externa, el riñón recupera rápidamente una coloración rosa y turbulencia normal del tejido
y comienza a excretar orina. Es visto al mes siguiente, la creatinina sérica es de 4.2 mg / dL. La
producción de orina es 20 ml / hora. Examen físico del corazón, pulmones, y abdomen están
dentro de los límites normales. El riñón transplantado es palpable en la fosa ilíaca derecha. La
biopsia posterior del trasplante muestra un extenso inflamación y edema ¿Cuál es la probable
explicación de estos hallazgos?
a) Rechazo agudo de trasplante
b) Rechazo crónico de trasplante
c) Enfermedad de injerto contra huésped
d) Rechazo de trasplante hiperagudo
e) Proceso normal de trasplante

91. Un hombre de 52 años se queja de dolor crónico abdominal. Ha sido hospitalizado siete
veces en los últimos 2 años para ataques recurrentes de dolor de pancreatitis crónica. Ha sido
tratado con analgésicos y con pancreatectomía distal. Su dolor aún persiste. ¿Cuál es el
siguiente paso en el tratamiento de este paciente?
a) Uso continuo de analgésicos orales.
b) Corticosteroides
c) Esplacnicectomía
d) Pancreatectomía total
e) Espera vigilante

92. Un hombre de 39 años es evaluado por su médico para detectar recurrencias de cálculos
renales. Ha sido tratado en el pasado con litotricia extracorpórea por ondas de choque,
ureteroscopia y una nefrolitotripsia percutánea. ¿Cuál de las siguientes características sugerirían
el diagnóstico de hiperparatiroidismo primaria?
a) 1 mm en rayos X KUB
b) 2 mm en rayos X KUB
c) Lado derecho de 1 mm y 2 mm en rayos X KUB
d) Lados derecho e izquierdo de 1 mm, 2 mm y 3 mm en rayos X KUB
e) Polo inferior derecho de 4 mm en tomografía computarizada
SS – Med10
93. Un médico desea administrar un anestésico local subcutáneo a un paciente con
una lesión sospechosa en su antebrazo. La lesión mide 1 cm y el cirujano planea una
incisión elíptica para eliminarlo. ¿Cuál capa epidérmica de la piel penetrarán
primero el médico con la aguja anestésica local?
a) Estrato basal
b) Estrato córneo
c) Estrato granuloso
d) Stratum lucidum
e) Estrato espinoso

94. Un hombre de 35 años con antecedentes de enfermedad de Crohn previa, visita a su médico
para un examen de seguimiento. Él tiene enfermedad ileocolónica difusa en una tomografía
computarizada reciente. Su actual lista de medicamentos incluyen sulfasalazina. Examen físico
revela dolor en el cuadrante inferior derecho a palpación profunda. Si se considera la terapia
con antibióticos en este paciente, ¿Cuál de los siguientes organismos debería ser el objetivo?
a) Especies de Mycobacterium
b) Pseudomonas aeruginosa
c) Staphylococcus aureus
d) Steotococos neumonia
e) Streptococcus pyogenes

95. Una mujer de 46 años se presenta donde su médico con una historia de disfagia
progresiva. Ella tiene una historia de pérdida de peso de 15 libras en los últimos 6
meses. Examen físico la región del cuello, corazón, pulmones y abdomen son normales. Las
pruebas de laboratorio revelan un hematocrito de 33% y un volumen corpuscular medio de 70.
La endoscopia gastrointestinal revela una red esofágica superior. ¿Cuál es el diagnóstico más
probable?
a) Tumor carcinoide
b) Leucemia
c) Linfoma
d) Síndrome de Plummer-Vinson
e) Artritis reumatoide

96. Un hombre de 47 años está involucrado en un accidente automovilístico. Sufre fracturas de


costillas 9, 10 y 11 en el lado izquierdo. Él esta hemodinámicamente inestable y tiene una
presión arterial de 90/50 mm Hg y pulso de 120 latidos / minuto a pesar de transfusión de 3 U
de glóbulos rojos empaquetados. Es llevado al quirófano para laparotomía exploratoria. Una
ruptura del bazo se identifica y se extrae. En busca de un bazo accesorio potencial, ¿ cuál es la
localización más probable para encontrarlo?
a) Epiplón mayor
b) Cuadrante inferior derecho
c) Cuadrante superior derecho
d) Hilio esplénico
e) Ligamentos esplénicos

97. Un niño de 12 años es llevado a su médico para su evaluación de una masa en el


cuello. Tiene antecedentes de sinusitis recurrente e infecciones amigdalares. El examen físico
revela una masa en el cuello de la línea media que mide 1,5 cm que se mueve con tragar. No
hay evidencia de linfadenopatía. ¿Cuál es el diagnóstico más probable?
a) Leucemia
b) Linfoma
c) Quiste del conducto tirogloso
d) Fístula tiroglosa
e) Carcinoma de tiroides
SS – Med10

98. Durante su rotación de medicina de emergencia se le llama al departamento de


emergencias para evaluar a un paciente con una lesión oftálmica. ¿Cuál de los
siguientes las lesiones traumáticas requieren un manejo inmediato en el sitio y
referencia al oftalmólogo de guardia?
a) Quemadura ácida
b) Nubosidad corneal
c) Laceración corneal
d) Hifema
e) Quemosis conjuntival severa

99. Un paciente se presenta al departamento de emergencias después ser cortado con un


cuchillo en la pierna izquierda. La herida parece limpia y los bordes están bien opuestos. El
siguiente es el método más simple de cierre de heridas por esta lesión?
a) Cierre primario retrasado
b) Injerto
c) Aletas locales
d) Cierre primario
e) Intención secundaria

100. Una mujer de 23 años se presenta a su médico quejándose dolor en la rodilla derecha. El
dolor es constante durante todo el día y ha estado presente durante aproximadamente 3
semanas. La historia social revela que ella está en una relación monógama. El examen físico
revela que su rodilla está ligeramente hinchada y sensible. Se realiza la aspiración de líquido
sinovial. Ni los cristales ni las bacterias son encontrados ¿Cuál es el diagnóstico más probable?
a) Bursitis
b) sArtritis gonocócica
c) Artritis reumatoide
d) Bursitis séptica
e) Trauma
SS – Med10

CIRUGÍA
Preguntas del Blueprints

1. Un hombre de 71 años con aparición repentina de abdominales severos, el dolor de espalda y


de espalda se lleva al departamento de emergencias para evaluación. Tiene antecedentes de
hipertensión. Pesa 300 lb. Tiene un historial de 45 paquetes al año de de fumar. El examen
físico revela un pulsátil masa abdominal Ambas extremidades inferiores revelan palidez con
pulsos de pedal disminuidos. ¿Qué es lo más probable causa de la condición de este paciente?
a) Aterosclerosis
b) Síndrome de Marfan
c) Infección meningocócica
d) Sífilis
e) Trauma

2. Un hombre de 78 años es llevado a la sala de emergencias, con antecedentes de dolor


abdominal de 12 horas, diarrea y vómito. Tiene antecedentes de fibrilación auricular y fue
tratado previamente por insuficiencia cardíaca congestiva con digoxina El examen físico revela
una distensión abdomen con protección significativa. Examen rectal revela heces guayaco
positivas en la bóveda. Glóbulos blancos: el recuento celular es de 24,000 / mL. La radiografía
abdominal revela edema de la pared intestinal. ¿Cuál es el tratamiento más apropiado para este
paciente?
a) Embolización angiográfica
b) Terapia antibiótica con ampicilina y gentamicina.
c) Terapia antibiótica con gentamicina.
d) Heparinización seguida de warfarina oral.
e) Exploración quirúrgica

3. Un jugador de tenis masculino de 20 años se estrella contra una cerca tratando de perseguir
una pelota, pensó que podría alcanzar durante un partido importante Su rodilla derecha
sostiene el la mayor parte de las lesiones. El examen físico revela edema y disminución del
rango de movimiento de la rodilla en flexión y extensión. La resonancia magnética (MRI) es
performado y revela dislocación de la articulación. No hay pulsos palpables detrás de la
articulación de la rodilla. ¿Qué es lo más probable explicación de este hallazgo?
a) Rotura de la arteria tibial anterior
b) Hematoma de la arteria peronea
c) Espasmo de la arteria poplítea
d) Hematoma de la arteria tibial posterior
e) Espasmo superficial de la arteria femoral

4. Una mujer de 25 años encontró un bulto en su seno derecho en autoexamen. Ella no tiene
antecedentes familiares de mama cáncer. El bulto es libremente móvil y está bien
circunferenciado. ¿Cuál es la mejor opción para evaluar una masa en el seno en una mujer
joven?
a) Biopsia
b) Mamografía
c) Pruebas para el gen del cáncer de seno ( BRCA )
d) Ultrasonido
e) Espera vigilante

5. Una mujer de 19 años comenzó a amamantar por primera vez. Al principio, era difícil para su
bebé alimentarse. Ahora sus senos son rojos, cálidos y doloridos. Ella ha continuado
SS – Med10
amamantando, a pesar del dolor; sin embargo, ella recientemente comenzó a usar
un extractor de leche en lugar de alimentación. Ella se inicia en un curso de
antibióticos orales.
¿Qué afección tiene este paciente en riesgo de desarrollar?
a) Absceso mamario
b) Enfermedad fibroquística
c) Cáncer de mama inflamatorio
d) Prolactinoma
e) Tuberculosis

6. Una mujer premenopáusica de 31 años con masa en el seno izquierdo se somete a una
mastectomía radical izquierda modificada. La patología revela una medida de carcinoma ductal
infiltrante de tamaño de 3 cm de tamaño con ganglios linfáticos negativos. El estado del
receptor estrogénico es negativo. ¿Cuál es la terapia adyuvante más apropiada para este
paciente?
a) Quimioterapia (multiagente)
a) Radioterapia de haz externo
b) Terapia de ultrasonido enfocada en alta energía
c) Tamoxifeno
d) Espera vigilante

7. Una mujer de 31 años se queja de una historia de 6 meses de diarrea con sangre, dolor
abdominal e intermitente fiebres. Tiene antecedentes de síndrome del intestino irritable pero
ha empeorado sus síntomas durante el anterior periodo de tiempo. Su historial médico pasado es
irrelevante. El examen físico revela distensión abdominal. Los sonidos intestinales están
presentes en todos los cuadrantes. Examen rectal, la inación revela múltiples fisuras anales.
¿Cuál prueba de diagnóstico es la más apropiadas para este paciente?
a) Anoscopia
b) Colonoscopia
c) Sigmoidoscopia flexible
d) Sigmoidoscopia rígida
e) No se requieren más pruebas de diagnóstico para este paciente.

8. Una mujer de 71 años se presenta a su médico de atención primaria, quejándose de sangrado


rectal. Ella tenía un poco calambres abdominales leves del lado izquierdo que disminuyeron en
minutos. Ella nunca ha tenido un episodio previo de sangrado rectal. El examen físico revela
leve dolor abdominal en el cuadrante inferior izquierdo sin evidencia de rebote ni
sensibilidad. Examen rectal no revela sangre fresca en la bóveda rectal. Colonoscopia revela
varias bolsas de la pared del colon sigmoide sin evidencia de sangrado o perforación. El resto de
la colonoscopia está dentro de los límites normales. El recuento de glóbulos blancos es
normal. ¿Cuál es el tratamiento más apropiado para este paciente?
a) Terapia antibiótica con ampicilina y gentamicina.
b) Hemicolectomía izquierda
c) Hemicolectomía derecha
d) Colectomía subtotal
e) Espera vigilante

9. Un hombre de 85 años es llevado al departamento emergencia, debido a dolor abdominal


agudo y distensión abdominal progresiva. Es residente de un asilo de ancianos. No ha estado
comiendo debido a náuseas progresivas. Las radiografías abdominales revelan una masa en colon
sigmoide ¿Cuál es el tratamiento inicial para este paciente?
a) Enema de gastrografin
b) Dieta rica en fibra
c) Lactulosa
SS – Med10
d) Descompresión del tubo rectal
e) Resección quirúrgica

10. Una mujer de 41 años se queja de dolores de cabeza constantes. Durante los
últimos 6 meses. Ella también se ha quejado de infertilidad femenina y no ha
podido tener hijos, a pesar de tener relaciones sexuales sin protección con su esposo durante los
últimos 15 años. Examen físico revela déficits en los movimientos extraoculares bilateralmente.
El examen de los senos revela ginecomastia bilateral. ¿Cuál de las siguientes pruebas de
laboratorio sería más útil para diagnosticar a este paciente?
a) Ferritina
b) Hemoglobina
c) Hematocrito
d) Hierro
e) Prolactina

11. Una mujer de 41 años con enfermedad de Crohn se han realizado múltiples procedimientos
quirúrgicos. Ella recientemente se sometió a una ileostomía pero aún tiene evidencia de alguna
enfermedad yeyunal distal. Sus medicamentos actuales incluyen prednisona y ácido
aminosalicílico. ¿Cuál de los siguientes Efectos reductores de la terapia prolongada con
glucocorticoides son posibles para este paciente?
a) Producción de anticuerpos
b) Formación de colágeno
c) Disfunción de fibroblastos
d) Migración celular inflamatoria
e) Deterioro de heridas

12. Un hombre obeso de 49 años se presenta a su médico de atención primaria, para un examen
de seguimiento. El tiene una historia de diabetes mellitus no controlada y trastorno bipolar.
Sus medicamentos actuales incluyen litio y leche de magnesio. Examen físico del corazón,
pulmones, y abdomen están dentro de los límites normales. Los estudios de laboratorio revelan
calcio sérico de 14 mg / dL. ¿Cuál es lo más explicación probable de estos hallazgos?
a) Indiscreción dietética
b) Sobredosis de medicación
c) Síndrome de leche y álcali
d) Adenoma paratiroideo
e) Hiperplasia paratiroidea

13. Un hombre de 41 años tiene reflujo gastroesofágico crónico. Actualmente se maneja con
un bloqueador de H2. Al examen físico del corazón, pulmones y abdomen son dentro de los
límites normales. ¿Cuál de los siguientes factores, sería menos protector del esófago en
términos de la exposición continua inducida por esta condición?
a) Ligamento arqueado
b) Capacidad de vaciado gástrico
c) Efecto gravitacional
d) Productos secretores de glándulas salivales
e) Ondas peristálticas secundarias

14. Una mujer de 40 años se queja de dolor en el pecho y disfagia a los sólidos. Ella se presenta
a un especialista para evaluación. Se realizan estudios manométricos esofágicos y revelan
contracciones de alta amplitud y eventual relajación normal del esfínter esofágico inferior.
La ingestión de bario es normal. ¿Cuál es el diagnóstico más probable?
a) Espasmo muscular cricofaríngeo
b) Espasmo esofágico difuso
c) Esclerodermia
SS – Med10
d) Tuberculosis
e) Trastorno de deglución psicógena

15. Un niño de 5 años es llevado a la sala de emergencias después de ingerir


líquido limpiador de drenaje. El chico era dejado sin supervisión mientras su niñera estaba en la
tele teléfono. El niño esta ronco y tiene un evidente estridor. ¿Cuál es el tratamiento inicial más
apropiado para este paciente?
a) Antibióticos
b) Corticosteroides
c) Inducción de vómitos con ipecacuana
d) Colocación de sonda nasogástrica y lavado
e) Traqueotomía

16. Un hombre de 76 años con antecedentes de dolor vago en el cuadrante superior derecho,
pérdida de peso de 25 lb y anorexia previa, visita a su médico de atención primaria para su
evaluación. El examen físico revela ictericia escleral. El examen revela una masa abdominal en
el cuadrante superior derecho. Riñón, uréter y vejiga (KUB) revelan un cálculo circular de
fijación en el cuadrante superior derecho. Laparotomía exploratoria revela un proceso
neoplásico que involucra la vesícula biliar der y hígado. ¿Cuál es la patología más probable que
causa esta condición?
a) Adenocarcinoma
b) Sarcoma
c) Carcinoma de células escamosas
d) Carcinoma de células de transición
e) Granuloma tuberculoso
17. Una mujer de 38 años se presenta a su médico atención primaria, para evaluación de dolor
vago intermitente en el cuadrante superior derecho. Ella tiene antecedentes de hipotiroidismo e
hipertensión. Sus medicamentos actuales incluyen reemplazo sintético de hormona tiroidea y un
bloqueador de los canales de calcio. El examen físico revela dolor leve en el cuadrante superior
derecho hasta palpación profunda. La ecografía revela un cálculo biliar de 3 cm.
¿Qué tipo de piedra es probable que esté presente en este paciente?
a) Cálculo biliar negro
b) Cálculos biliares marrones
c) Cálculos biliares de oxalato de calcio
d) Cálculos de colesterol tipo I
e) Cáculos de colesterol tipo II

18. Una mujer de 46 años se presenta al departamento emergencia, quejándose de dolor en el


cuadrante superior derecho y fiebre a 102F°. El examen físico revela ictericia escleral y dolor
significativo en el cuadrante superior derecho a la palpación. Los signos peritoneales están
ausentes. Los sonidos intestinales están presentes. ¿Cuál de los siguientes debe incluirse en el
tratamiento inicial de este paciente?
a) Antibióticos
b) Coledochojejunostomía
c) Descompresión con tubo en T
d) Esfinterotomía endoscópica
e) Drenaje transhepático percutáneo

19. Un niño de 17 años es llevado a la sala de emergencias, después de sufrir dolor torácico y
disnea durante un juego de baloncesto. Examen físico revela un soplo crescendo-decrescendo
sistólico, se escuchó mejor en el segundo espacio intercostal derecho. El soplo se irradia a la
arteria carótida derecha. Radiografía de pecho revela un tamaño de corazón normal. ¿Cuál de
los siguientes hallazgos se esperaría que se vieran en un electrocardiograma en este paciente?
SS – Med10
a) Inversión de ondas T en derivaciones V1 – V4
b) Hipertrofia del ventrículo izquierdo
c) Bloque de rama derecha
d) Hipertrofia ventricular derecha
e) Hipertrofia auricular derecha

20. Un hombre de 72 años se derrumba/desmaya mientras camina en una tienda de Ping


Mall, esta sin pulso y apneico. No hay historia de trauma. Comienza la reanimación
cardiopulmonar hasta que llegue un escuadrón de rescate, Suplemento cardíaco avanzado, se
inicia el protocolo básico. Es declarado muerto 40 minutos más tarde. La autopsia revela
necrosis miocárdica con ruptura del ventrículo izquierdo. ¿Cuál de los siguientes es el factor de
riesgo más probable que contribuyó a su muerte?
a) Antecedentes familiares de diabetes mellitus.
b) Hipotensión
c) Obesidad
d) Estilo de vida sedentario
e) Trauma

21. Un hombre de 57 años es llevado a la sala de emergencias y se queja de disnea y dolor en el


pecho. Él también admite una pérdida de peso de 20 libras. Se queja de fiebres, escalofríos, y
sudores nocturnos. El examen físico revela adenopatía supraclavicular. El examen de tórax
revela sonidos del corazón a distancia. Los estudios de laboratorio revelan un recuento de
glóbulo blanco de 170,000 / mL. Radiografía de tórax y ecocardiografía revelan un derrame
pericárdico. ¿Cuál es la explicación más probable? nación de estos hallazgos?
a) Mixoma auricular
b) Fibrilación auricular
c) Linfoma
d) Carcinoma colorrectal metastásico
e) Pericarditis

22. Un varón recién nacido tiene una abertura del abdomen en la pared del ombligo. No tiene
otro registro médico previo o historia quirúrgica en el historial de nacimientos, ni pruebas
diagnosticas. El resto del examen físico y diagnóstico no hay datos relevantes. ¿Cuáles de los
siguientes hallazgos son más probables?
a) Labio leporino
b) Paladar hendido
c) Hernia de diafragma
d) Pericardio
e) Vejiga urinaria en retroperitoneo

23. Un trabajador de la construcción de 44 años se somete a una reparación de hernia inguinal


derecha. El procedimiento quirúrgico es sin acontecimientos notables. No tiene antecedentes
médicos o quirúrgicos previos. Regresa para seguimiento en el postoperatorio día 3, para un
control de heridas. La herida está limpia, seca e intacta. ¿Cuál es el período de convalecencia
óptimo requerido antes volver a trabajar para este paciente?
a) 1 semana
b) 4 semanas
c) 6 a 8 semanas
d) 12 semanas
e) Desconocido

24. Una mujer de 40 años se repara una hernia femoral derecha. Durante el procedimiento, el
canal femoral es disecado. Los límites anatómicos del canal femoral incluye cuál de los
siguientes?
SS – Med10
a) Ligamento de Cooper
b) Ligamento inguinal
c) Espina ciática
d) Ligamento lacunar
e) Nervio (femoral)

25. Un hombre de 53 años se somete a una prostatectomía radical, para el presunto cáncer de
próstata limitado a órganos. El factor importante para mantener la continencia después de la
prostatectomía radicales es la preservación de:
a) Cuello de la vejiga
b) Esfínter uretral externo
c) Complejo muscular elevador del ano
d) Nervi erigentes
e) Ligamentos puboprostáticos

26. Un hombre de 27 años tiene una adenopatía retroperitoneal voluminosa, después de una
orquiectomía radical para un tumor de célula germinal mixta. Su radiografía de tórax es
normal. Suero beta-humano gonadotropina coriónica (-hCG) y alfa-fetoproteína
(AFP) están marcadamente elevados. Las enzimas hepáticas están ligeramente elevadas, y el
paciente relata una historia de etanol en exceso. Recibe tres ciclos de quimioterapia. El reporte
revela una masa retroperitoneal de 3 cm, una normal radiografía de tórax y suero normal
hCG. Sin embargo, la AFP sérica es de 20 UI / ml (normal de 0 a 9 UI / ml). ¿Cuál es el siguiente
paso en el manejo de este paciente?
a) Tomografía computarizada (TC): biopsia con aguja guiada
b) Radioterapia de haz externo
c) Disección de ganglios linfáticos retroperitoneales
d) Quimioterapia de rescate
e) Marcadores seriales y tomografías computarizadas

27. Un hombre de 63 años está libre de enfermedad después de dos años con el bacilo. Terapia
con Calmette-Guerin para el carcinoma in situ y un grado 2, cáncer de vejiga en estadio
T1. Además de físico examen, cistoscopia y citología urinaria, evaluación en este momento debe
incluir:
a) Pielografía intravenosa
b) Biopsia uretral prostática
c) Biopsias aleatorias de la vejiga.
d) Citología selectiva del tracto superior
e) Citología miccional anulada, repetida tres veces

28. Un hombre de 78 años se presenta en el departamento de emergencias, para la evaluación


del cuadrante superior derecho por dolor progresivo, náuseas, vómitos y una pérdida de peso de
30 lb en el últimos 3 meses Tiene antecedentes médicos previos de colelitiasis, diabetes
mellitus, hipertensión y demencia. El examen físico revela ictericia escleral bilateralmente. El
examen abdominal revela ternura en la parte superior derecha cuadrante y una masa palpable
peritoneal. Los signos están ausentes. La tomografía computarizada revela ganglios pancreático,
duodenal, y coledocales. Hay un asimétrico engrosamiento de la vesícula biliar. ¿Cuál es lo más
probable hallazgo patológico en laparotomía exploratoria y biopsia?
a) Adenocarcinoma
b) Fibroma
c) Lipoma
d) Mixoma
e) Myoma
SS – Med10
29. Un niño de 8 años se somete a un ultrasonido para molestias persistentes en el
cuadrante superior derecho. No tiene antecedentes médicos o quirúrgicos previos.
Sin alergias conocidas y no toma medicamentos. Su madre tiene antecedentes de
cálculos biliares. Hallazgos de ultrasonido incluyen una dilatación fusiforme del
conducto biliar común. ¿Cuál es la explicación más probable para estos hallazgos?
a) Quiste de colédoco tipo I
b) Quiste de colédoco tipo II
c) Quiste de colédoco tipo III
d) Quiste de colédoco tipo IV
e) Quiste de colédoco tipo V

30. Un hombre de 18 años es apuñalado en su abdomen múltiples veces por un asaltante


durante un altercado que involucra venta de drogas ilícitas. Es llevado al departamento
emergencia para su evaluación. Él tiene cuatro heridas de arma blanca el abdomen: tres están
en el cuadrante superior derecho, y uno está en el cuadrante inferior izquierdo. Examen físico
La sección del abdomen revela protección/sensibilidad y rebote. El paciente es llevado a
cirugía para una laparotomía exploratoria. Una lesión penetrante en la vesícula biliar. Se
encuentra la vejiga. ¿Cuál de las siguientes viseras asociadas es probable que se lesionen?
a) Aorta
b) Colon
c) Riñón
d) Hígado
e) Vejiga urinaria

31. Una mujer de 62 años se presenta a su médico de atención primaria con tos. Ella también se
queja de hemoptisis. La historia social revela una historia de 55 paquetes al año de fumar
cigarrillos. Ella es una alcohólica en recuperación. Examen físico revela sibilancias
bilaterales. Cardíaco, pulmonar y los exámenes abdominales no son notables. Laboratorio los
valores revelan calcio sérico de 13 mg / dL. Proteína sérica y la electroforesis no muestra picos
anormales. ¿Cuál es el diagnóstico más probable?
a) Síndrome de Goodpasture
b) Mieloma
c) Adenoma renal
d) Carcinoma de células pequeñas de pulmón
e) Carcinoma de células escamosas de pulmón

32. Un niño de 10 años es llevado a su centro de atención primaria, para la evaluación de la


ronquera persistente. Comenzó a participar con su coro escolar y nota que su ronquera empeora
con el canto. El examen físico del corazón, pulmones y abdomen sin relevancia. Examen
laríngeo flexible de fibra óptica revela múltiples lesiones en sus verdaderas cuerdas vocales.
¿Cuál es el diagnóstico más probable?
a) Reflujo gastroesofágico
b) Inflamación granulomatosa de la faringe.
c) Papiloma laríngeo
d) Nódulo del cantante
e) Carcinoma de tiroides

33. Un hombre de 75 años se presenta a su médico de atención primaria debido a la


ronquera. Tiene historia de fumar con IPA de 60 al año. También se queja de pérdida de peso de
25 lb en los últimos 4 meses. La laringoscopia directa revela una masa sésil en la cuerda vocal
alta derecha. También tiene un ganglio linfático palpable a lo largo del cuello anterior derecho
de la cadena de ganglios linfáticos. Si se encuentra displasia en la biopsia de la lesión laríngea,
¿cuál es el diagnóstico más probable?
a) Adenoma
SS – Med10
b) Pólipo laríngeo
c) Laringitis
d) Enfermedad quística mucoepidermoide
e) Carcinoma de células escamosas

34. Un estudiante universitario masculino de 21 años se presenta a la clínica para un examen de


rutina al comienzo del semestre de otoño. Tiene antecedentes de síndrome de intestino
irritable. Examen físico del corazón, pulmones y abdomen no son notables. Examen
genitourinario revela que los testículos descienden bilateralmente. Un varicocele izquierdo de
grado 1 está presente. No hay masa testicular. El pene no está circuncidado y el prepucio está
incapaz de retraerse detrás del glande. ¿Cuál es el diagnóstico más probable?
a) Balanitis
b) Hipospadias
c) Epispadias
d) Parafimosis
e) Fimosis

35. Un hombre blanco de 71 años se presenta a su médico de atención primaria quejándose de


un historial de un mes de nicturia, poliuria, y dificultad para comenzar y detener su orina
corriente. Su síntoma según la Asociación Americana de Urología tiene un puntaje de
17/35. Examen físico de la próstata revela una glándula agrandada sin masas. Sus testículos son
descendió bilateralmente. Tiene un pequeño hidrocele derecho que transilumina. Su antígeno
prostático específico (PSA) el nivel es de 6 ng / ml y el análisis de orina es negativo. Comenzó
con dutasterida 0,5 mg al día. ¿Cuál es un resultado probable de tomar este medicamento?
a) Disfunción eyaculatoria
b) Cambio máximo en el flujo urinario
c) El tamaño de la próstata disminuye en un 25%.
d) El PSA sérico aumenta en un 50%
e) La puntuación de los síntomas permanece sin cambios

36. Un hombre blanco de 34 años tiene una ampliación indolora de su testículo derecho en los
últimos 4 meses. Es llevado a su médico de atención primaria por su novia, quien lo urge para
buscar evaluación. Recientemente se ha deprimido por este problema. Tenía una criptorquidia
derecha testículo cuando era bebé, que fue corregido quirúrgicamente. UNA la ecografía
escrotal confirma la presencia de 3 cm masa testicular derecha hipoecoica. ¿Cuál es el
diagnostico más probable?
a) Coriocarcinoma
b) Carcinoma embrionario (células germinales mixtas)
c) Tumor endodérmico (saco vitelino)
d) Seminoma
e) Teratoma

37. Un hombre de 51 años tiene una masa intracraneal y se someterá a resección. El


procedimiento quirúrgico se realiza a través de un enfoque transoccipital. En este enfoque, el
paciente desarrolla un líquido cefalorraquídeo (CSF) de fuga. ¿Cuál de las siguientes
afirmaciones es verdadera con respecto al LCR?
a) Las vellosidades aracnoideas actúan como válvulas de dos vías.
b) Las vellosidades aracnoideas se abren a una presión de 5 mm Hg.
c) El LCR se absorbe a través de las raíces espinales.
d) CSF entra por el agujero de Magendie.
e) El volumen total de LCR es de 150 L.

38. Una estudiante universitaria de 19 años conduce bajo la influencia del alcohol, a pesar de
las recomendaciones de amigos de No conducir. Ella es golpeada por otro conductor. La fuerza
SS – Med10
del impacto hace que golpee el área temporal de ella cráneo contra la ventana. Ella
desarrolla un dolor de cabeza leve pero no pierde el conocimiento. Varias horas
después, ella desarrolla un fuerte dolor de cabeza con náuseas y vómitos.
¿Cuál es el diagnóstico más probable?
a) Infección bacteriana
b) Aneurisma de bayas
c) Hematoma epidural
d) Hematoma subaracnoideo
e) Hemorragia subdural

39. Un hombre de 59 años se presenta a su médico de atención primaria, quejándose de pérdida


audición progresiva del lado derecho y marcha con inestabilidad. Él dice que cuando usa el
teléfono, debe usar su oído izquierdo para escuchar en lugar de su oreja derecha. Tiene
antecedentes médicos de hipertenia. Sus medicamentos actuales incluyen un bloqueador de
canal de calcio. El examen físico revela pérdida del reflejo corneal derecho y debilidad
facial. Cardíaco, pulmonar, y los exámenes abdominales están dentro de los límites normales.
¿Cuál es el siguiente paso más apropiado en el diagnóstico de este paciente?
a) Prueba audiométrica
b) Prueba potencial evocada del tronco encefálico
c) Tomografía computarizada de la cabeza sin contraste
d) Resonancia magnética de la cabeza
e) Nistagmografía

40. Un hombre de 47 años con antecedentes de enfermedad pulmonar en etapa terminal del
pulmón derecho está programado para un trasplante de pulmón. La función cardíaca
preoperatoria es buena. No tiene antecedentes de defectos congénitos. ¿Cuál de las siguientes
es la incisión quirúrgica más adecuada para este paciente?
a) Chevron abdominal
b) Toracotomía lateral
c) Línea media abdominal
d) Toracotomía anterior transversal
e) Pfannenstiel

41. Un hombre de 47 años con múltiples problemas médicos y la enfermedad del parénquima
pulmonar en etapa terminal se somete trasplante pulmonar. Tiene antecedentes médicos
previos de enfermedad pulmonar obstructiva. El tiene un tío con fibrosis quistica. Su padre tiene
enfermedad pulmonar restrictiva, y su hermano tiene hipertensión pulmonar. ¿Cuál de los
siguientes Lowing presagia la mejor supervivencia después del trasplante pulmonar para este
paciente?
a) Carcinoma broncogénico
b) Fibrosis quística
c) Enfermedad pulmonar obstructiva
d) Hipertensión pulmonar
e) Enfermedad pulmonar restrictiva

42. Un niño de 4 años está en la lista de espera para un trasplante de hígado. Tiene enfermedad
hepática en etapa terminal y actualmente está hospitalizado por hemorragia varicosa
esofágica. ¿Cuál es la causa más probable de insuficiencia hepática en este paciente?
a) Atresia biliar
b) Hepatitis A
c) Cirrosis biliar primaria
d) Colangitis esclerosante primaria
e) Tuberculosis
SS – Med10
43. Una mujer de 23 años que se queja de heces grasosas abundantes y dolor,
debilidad generalizada y pérdida de cabello y acude a su médico de atención
primaria para su evaluación. Examen físico del corazón, pulmones y abdomen
normales. Ella no tiene rebote. Los sonidos intestinales están presentes en todos los
cuadrantes. El examen pélvico fue diferido a solicitud del paciente. ¿Cuál es la
explicación más probable de estos hallazgos?
a) Úlcera gástrica con sangrado.
b) Malabsorción de glucosa
c) Menstruación
d) Insuficiencia pancreática
e) Tumor pituitario

44. Una mujer de 27 años, con 12 horas después trasplante de páncreas cadavérico y
actualmente en el quirófano unidad de Cuidados Intensivos. Ella tiene un historial médico de
insulinodependiente desde los 5 años. Sus signos vitales son normales. El pecho está claro para
la auscultación y el examen cardíaco revela un ritmo regular es limpio, seco e intacto. ¿Cuál es
el mejor método para controlar el páncreas trasplantado?
a) Nivel sérico de amilasa
b) Nivel de glucosa en suero
c) Nivel de insulina sérica
d) Ecografía de los vasos pancreáticos.
e) Nivel de amilasa urinaria

45. Traen a un hombre de 44 años con pancreatitis recurrente al departamento de emergencias


con otro episodio de pancreatitis ¿Qué factores se relacionan con la gravedad de su afección?
a) Edad
b) Nivel de glucosa en sangre de 300 mg / dL
c) Nivel de lactato deshidrogenasa de 400 UI / L
d) Calcio sérico de 6 mg / dL
e) Nivel de hematocrito en suero del 29%

46. Un hombre de 41 años con una larga historia de cálculos renales e hipercalcemia se
encuentra que tiene un adenoma de la glándula paratiroidea superior derecha. El va a sufrir
escisión quirúrgica de esta lesión. ¿Cuál es la mejor cirugía punto de referencia para esta lesión?
a) Bifurcación de las arterias carótidas.
b) Seno carotídeo
c) Unión de la arteria tiroidea inferior y recurrente nervio laríngeo
d) Unión del tercio superior y medio de la tiroides glándula
e) Nervio laríngeo recurrente

47. ¿Cuál de las siguientes técnicas se utiliza mejor para definir una glándula paratiroides
agrandada?
a) Tomografía computarizada del cuello
b) Doble trazado de imágenes
c) Resonancia magnética del cuello
d) Angiografía tirocervical
e) Ultrasonido

48. Un hombre de 44 años con enfermedad renal en etapa terminal se somete con trasplante
renal. Tiene un historial médico previo de hiperparatiroidismo. Seis meses después del
trasplante renal, su calcio sérico sigue siendo 13 mg / dL. ¿Cuál de los siguientes hallazgos de
laboratorio son posible en este paciente?
a) Fosfato sérico elevado
b) Elevación sérica de ácido láctico deshidrogenasa
SS – Med10
c) Orina elevada en calcio
d) Creatinina en orina elevada
e) Proteína urinaria elevada

49. Un hombre de 46 años se presenta a su médico de atención primaria, para la evaluación de


una lesión cutánea. Se queja de hipopigmentación de la piel de su espalda baja. Él tiene
antecedentes médicos previos de eccema y carcinoma basocelular. Es un granjero que pasa
mucho tiempo al aire libre. ¿Qué células son responsables de esta condición?
a) Adipocitos
b) Células productoras de queratina
c) Células de Langerhans
d) Melanocitos
e) Células de Merkel

50. Un hombre de 69 años se presenta a su dermatólogo con un lesión presente en su nariz. Es


un jardinero que gasta gran parte de su tiempo al aire libre. Él tiene un antecedente médico
previo de rinitis alérgica, hipertensión y diabetes mellitus. Sus medicamentos actuales incluyen
un betabloqueante y un hipoglucemiante oral. Examen físico de su la nariz revela una lesión
papular brillante y elevada con pequeñas vasos sanguíneos. ¿Cuál es el diagnóstico más
probable?
a) Carcinoma de células basales
b) Histiocitosis X
c) Melanoma
d) Queratosis seborreica
e) Carcinoma de células escamosas

51. Una mujer negra de 29 años se presenta a su médico atención primaria debido a un
crecimiento en su oreja izquierda, que ocurrió después de que le perforaron la oreja por
primera vez hace una semana. Ella notó que su oído parecía desarrollarse un crecimiento en él
con bastante rapidez. Ella nunca había tenido su oído perforado antes. ¿Cuál es la explicación
más probable para ¿estos hallazgos?
a) Carcinoma de células basales
b) Nevo azul
c) Melanoma juvenil
d) Queloide
e) Molusco contagioso

52. Una mujer asiática-estadounidense de 52 años tiene una pigmentación melanótica de la


mucosa bucal, labios y dedos. La colonoscopia revela hamartomas en todo el tracto
gastrointestinal Los pólipos fueron eliminados debido a su mayor riesgo de cáncer. ¿Qué otro
cáncer es asociado a esta condición?
a) Cáncer de cuello uterino
b) Cancer de RIÑON
c) Cáncer de hígado
d) Cáncer de ovarios
e) Cáncer de páncreas

53. Una mujer de 45 años se queja de diarrea crónica y constipación. Se realiza una
colonoscopia y una biopsia de una lesión en su íleon. El informe de patología muestra que el
tumor está compuesto de células neuroendocrinas. ¿Cuál es el tratamiento médico para esta
afección?
a) Corticosteroides intravenosos
b) Corticosteroides tópicos
c) Furosemida
SS – Med10
d) Octreotida
e) Tetraciclina

54. Un hombre de 18 años es llevado a la sala de emergencias con dolor


abdominal insoportable y repentino localizado hacia el cuadrante inferior derecho,
náuseas y vómitos, leves fiebre y taquicardia leve. Tiene antecedentes médicos previos como
historia de la otitis media recurrente. Examen físico revela marcado rebote en el cuadrante
inferior derecho. El recuento de glóbulos blancos en suero es de 18,000 / mL. La radiografía
KUB revela gas intestinal en el intestino delgado y grueso. ¿Cuál es el diagnóstico más probable?
a) Apendicitis
b) enfermedad de Crohn
c) Diverticulitis
d) Pancreatitis
e) Colitis ulcerosa

55. Una mujer de 39 años se presenta a la sala de emergencia quejándose de dolor abdominal
severo. Ella tiene antecedentes de enfermedad ulcerosa péptica. Examen físico revela
protección y rebote. Ella es llevado a la sala de operaciones para exploración por laparotomía
exploratoria. Durante el procedimiento, el cirujano que abre el ligamento gastrosplénico para
alcanzar el saco menor corta accidentalmente una arteria. ¿Cuál de los siguientes vasos es el
muy probablemente uno herido?
a) Arteria gastroduodenal
b) Arteria gástrica izquierda
c) Arteria gastroepiploica izquierda
d) Arteria gástrica derecha
e) Arteria esplénica

56. Una mujer de 2 años es llevada a la sala de emergencia debido a varios episodios de
sangrado rectal. En un escáner de perfusión de tecnecio-99m revela un tamaño de 3 cm y
excreción ileal ubicada a 50 cm de la válvula ileocecal. ¿Cuál de los siguientes tipos de tejido
ectópico es probable que esta estructura contenga?
a) Duodenal
b) Esofágico
c) Gástrico
d) Hepático
e) Yeyunal

57. Una mujer de 39 años se queja de dolor epigástrico con comienzo durante los últimos 3 o 4
meses. Ella admite una historia de problemas crónicos de espalda. Ella nota aumento de peso de
20 lb en los últimos 4 meses. Ella niega el uso de agentes antiinflamatorios esteroideos. Ella
niega las náuseas y vómitos. Examen físico del corazón, pulmones, y el abdomen están dentro
de los límites normales. ¿Cuál es el más probable patógeno asociado con esta condición?
a) Escherichia coli enterohemorrágica
b) Escherichia coli
c) Helicobacter pylori
d) Shigella sonnei
e) Streptococcus pyogenes

58. Un hombre de 59 años resultó herido en un accidente automovilístico. La tomografía


computarizada abdominal revela una ruptura del bazo. Su presión arterial es de 90/40 mm Hg,
y su pulso es de 140 latidos por minuto. El paciente es llevado a una laparotomía -
esplenectomía. ¿Cuál de los siguientes trastornos seria probable después de este procedimiento?
a) Anemia
b) Basofilia
SS – Med10
c) Eosinofilia
d) Trombocitopenia
e) Trombocitosis

59. Un hombre de 19 años fue pateado en el abdomen durante un pelear en un


bar. Fue a su médico de atención primaria, quien ordenó una tomografía computarizada, que
reveló un hematoma esplénico subcapsular. Le dijeron al hombre que restringiera la actividad
física. Dos semanas después, se presenta a la emergencia debido a dolor abdominal severo. Él se
somete a una esplenectomía. Después de la operación, se ordena un frotis periférico, qué tipo
de célula se puede encontrar en este paciente?
a) Punteado basófilo
b) Células de la ampolla
c) Cuerpos de Howell-Jolly
d) Glóbulos rojos nucleados.
e) Esferocitos

60. Un hombre afroamericano de 15 años fue sometido una esplenectomía después de sufrir una
lesión en un cuchillo durante una pelea. Se presenta a su médico de atención primaria para un
examen físico ya que su madre leyó en la web que él está en mayor riesgo de infección. ¿Cuál de
las siguientes vacunas debería recibir para prevenir infecciones graves?
a) Rubéola
b) Sarampión
c) Tétanos
d) Vacunas contra organismos encapsulados comunes.
e) Varicela

61. Una mujer de 53 años se presenta a su médico de atención primaria con un historial de
dolor de cuello de 12 meses. Ella se queja de un aumento de peso de 15 libras y malestar
generalizado. Ella tiene un historial médico pasado de hipertensión y diabetes mellitus. Sus
medicamentos actuales incluyen un hipoglucémico oral. El examen físico revela ternura a lo
largo el curso de la glándula tiroides sin evidencia de un trastorno de masa discreta. ¿Cuál es el
diagnóstico más probable?
a) Tiroiditis aguda
b) Tiroiditis de Hashimoto
c) Carcinoma papilar de tiroides
d) Tiroiditis de Riedel
e) Tiroiditis subaguda

62. Una mujer de 47 años con antecedentes de masa de tiroides izquierda, se somete a
lobectomía tiroidea izquierda. Patología revela un carcinoma papilar de 1.3 cm sin evidencia de
extensión extracapsular. ¿Cuál es el siguiente paso mas apropiado en el tratamiento de este
paciente?
a) Radioterapia de haz externo
b) Quimioterapia multiagente
c) Tiroidectomía subtotal
d) Tiroidectomía total
e) Espera vigilante con seguimiento periódico

63. Se está sometiendo a un hombre de 34 años con un nódulo tiroideo a una exploración del
cuello. Durante el procedimiento, es posible que se someterá a tiroidectomía. ¿Cuál de las
siguientes declaraciones bajas sobre el nervio laríngeo superior y la inervación de la glándula
tiroides es correcta?
a) La lesión del nervio provoca la inclinación de las cuerdas vocales. durante la fonación
b) La lesión nerviosa puede pasar desapercibida en los cantantes.
SS – Med10
c) El nervio rara vez está en riesgo durante la cirugía de tiroides
procedimientos
d) El nervio laríngeo superior es principalmente un nervio motor.
e) El nervio laríngeo superior es principalmente un nervio sensorial.

64. Un hombre de 19 años salta del tercer piso de su dormitorio en un aparente intento de
suicidio. Es llevado a al departamento de emergencias inconsciente. Tiene visible lesiones en la
cabeza y extremidades inferiores. Tiene un pulso de 110 late por minuto pero es apneico. ¿Cuál
es la mejor vía aérea para el manejo de este paciente?
a) Intubación nasotraqueal
b) Intubación oral
c) Intubación oral con elevación del mentón
d) Traqueotomía
e) La intubación no es necesaria para este paciente.

65. Una mujer de 21 años es apuñalada en el pecho por su novio. Ella es llevada al
departamento de emergencias para su evaluación. Su presión arterial es de 130/80 mmHg, y su
pulso es de 90 latidos por minuto. Examen físico revela una herida de arma blanca en el quinto
intercostal izquierdo espacio en la línea medioclavicular. El examen del cuello es normal.La
tráquea en la línea media y las venas yugulares no están distorsionadas. Ella tiene una
disminución de los sonidos respiratorios en el campo pulmonar izquierdo. ¿Cuál de los siguientes
diagnósticos puede ser descartado sobre la base de la información anterior?
a) Hemotórax izquierdo grande
b) Neumotórax abierto
c) Taponamiento pericárdico
d) Ruptura del bronquio principal izquierdo
e) Tensión neumotoraxica

66. Un hombre de 41 años sufre una amputación traumática de tres de sus dedos en una
cortadora de carne. No tiene antecedentes médicos ni historia clínica. ¿Cuál de las siguientes
modalidades debería ser utilizado para transportar los dedos amputados con el paciente?
a) Colocar en una bolsa de plástico limpia y empacar con hielo seco.
b) Colóquelo en una bolsa de plástico limpia llena de agua pura a temperatura ambiente
c) Colocar en una bolsa de plástico limpia en un cofre lleno de hielo picado y agua.
d) Colocar en una bolsa de plástico limpia llena de agua caliente.
e) Envuelva los dedos amputados con una gasa seca estéril.

67. Una mujer de 19 años se presenta a la sala de emergencia después de sufrir una lesión en
su ojo derecho mientras coloca su lente de contacto. Ella tiene un significativo dolor de ojo
derecho. Ella tiene un historial médico previo de alergias. El examen físico revela un abrasión
simple. La prueba de fluoresceína se realiza y revela que no evidencia de un defecto epitelial
manchado. Esto descarta el posibilidad de cuál de los siguientes?
a) Infección bacteriana
b) Iritis
c) Trauma
d) Infección viral
e) Úlcera

68. Un hombre de 29 años que trabaja en una fábrica sufrió una lesión por cuerpo extraño en su
ojo derecho cuando un pedazo de metal salió disparado de una cinta transportadora. Él es
llevado al departamento de emergencias para evaluación. Examen físico del ojo derecho revela
un cuerpo extraño metálico con un anillo de óxido. ¿Cuál es el instrumento más útil para
eliminar este cuerpo extraño?
a) Pegamento de cianoacrilato
SS – Med10
b) Rebabas en los ojos o Eye burr
c) Spud de ojo
d) Punta de aguja fina
e) Agua estéril y alcohol

69. Un profesor de química de 37 años sostiene un vaso químico con ácido el cual salpico en su
ojo derecho mientras intenta formar una demostración para su clase de ciencias de la escuela
secundaria. Él tiene mucho dolor. Mientras está en el aula y esperando para que una ambulancia
lo transporte al hospital, que de las siguientes intervenciones deben realizarse?
a) Instilación de gotas para los ojos con solución salina normal
b) Colocación del parche ocular
c) Lavar los ojos con 1 a 2 L de solución salina normal.
d) Colocación del ojo bajo la luz solar directa.
e) Vigilante espera hasta que llegue la ambulancia

70. Un trabajador de la construcción de 37 años sufrió una lesión en el muslo derecho después
de que una grúa cayó sobre su pierna en el sitio de trabajo. Lo llevan al departamento de
emergencia para evaluar y tiene dolor significativo en la pierna derecha y dolor con
estiramiento pasivo. La pierna está tensa a la palpación. ¿Cuál es la medida de presión
intracompartimental más probable que debe utilizarse la pierna derecha de este paciente?
a) 5 mm Hg
b) 10 mm Hg
c) 15 mm Hg
d) 25 mm Hg
e) 35 mm Hg
PREGUNTAS
71. Una mujer de 41 años que limpia casas para ganarse la vida visita a su médico de atención
primaria quejándose de sensibilidad en su rodilla derecha. El dolor es constante y ha sido
presente por 3 semanas. Ella está en una relación monógama. El examen físico revela que su
rodilla está ligeramente hinchado y sensible. Los exámenes cardíaco, pulmonar y abdominal
están dentro de los límites normales. Se realiza una aspiración sinovial. La evaluación no revela
evidencia de cristales o bacterias. ¿Cuál es el diagnóstico más probable?
a) Bursitis
b) Artritis infecciosa
c) Artritis reumatoide
d) Tromboflebitis séptica
e) Artritis infecciosa inducida por trauma

72. Un niño de 12 años que es el lanzador estrella de equipo de la liga de beisbol, se queja de
dolor en el hombro derecho. Este es su brazo de lanzamiento. No tiene antecedentes médicos o
quirúrgicos previos. El examen físico revela debilidad del manguito rotador. ¿Cuál es el
tratamiento más apropiado para este paciente?
a) Inyección de corticosteroides.
b) Corticosteroides intravenosos
c) Descanso, elevación y agentes antiinflamatorios.
d) Colocación de la honda
e) Reparación quirúrgica

73. Un hombre de 65 años con antecedentes de enfermedad coronaria se somete un bypass


aortobifemoral. ¿Cuál de las siguientes maniobras de manejo intraoperatorio disminuirá su
riesgo de infarto de miocardio intraoperatorio?
a) Beta bloqueadores
b) Bloqueadores de los canales de calcio
SS – Med10
c) Administración de solución salina normal en lugar de lactato Ringer
d) Uso de propofol
e) Uso de morfina

74. A un hombre sano de 35 años se le diagnostica un hernia inguinal. No tiene


antecedentes de sangrado anormal. ¿Cuál de las siguientes pruebas es absolutamente necesaria
antes a llevarlo al quirófano?
a) Hematocrito
b) Recuento de plaquetas
c) Potasio
d) Recuento de glóbulos blancos
e) Ninguna de las anteriores

75. Un hombre de 50 años tiene diarrea después de una cirugía sin complicaciones de resección
intestinal. La elección de fluidos más conveniente es:
a) Solución salina normal
b) Solución salina media normal con 20 mEq de potasio
c) D5W con bicarbonato de 3 amperios
d) La solución de Ringer lactato
e) D5NS

76. Un hombre de 27 años es llevado a la sala de emergencias después de cortarse la mano con
un cuchillo mientras intentaba cortar un panecillo. Tiene antecedentes médicos previos de
recurrencia de infecciones de los senos paranasales. Su historial quirúrgico previo es notable por
reparación de una fractura nasal. El examen físico revela un laceración limpia de 4 cm a lo largo
del aspecto palmar de su mano. Los principios más relevantes para este caso serían:
a) Manejo suave del tejido y cierre sin tensión.
b) Injerto de piel cuando sea necesario para cubrir el área
c) Desbridamiento de tejido desvitalizado.
d) Uso de una esponja de vacío para promover la curación.
e) Reconstrucción de colgajo con tejido vascularizado.

77. Un hombre de 68 años es llevado a la sala de emergencias se queja de dolor abdominal y de


piernas hace 2 semanas. Tiene antecedentes de hipertensión e hipercolesterolemia; él pesa 290
lb. Examen físico revela una masa abdominal pulsátil en la línea media. El las extremidades
inferiores tienen pulsos desiguales. ¿Cuál de los siguientes es el mejor paso en la evaluación de
este paciente?
a) Aortograma
b) Tomografía computarizada del abdomen y la pelvis
c) Ultrasonido, riñones y vejiga.
d) Ultrasonido, hígado y bazo.
e) Venacavogram

78. Una mujer de 28 años se presenta a su médico para evaluación de un bulto en su seno
derecho encontrado en autoexamen. Ella tiene antecedentes familiares de cáncer de mama en
que su madre murió a los 40 años por esta condición. La madre tuvo una mastectomía radical
modificada por quimioterapia. El examen físico revela un protuberancia mamaria que es
libremente móvil y bien circunferenciada. No hay hoyuelos, asimetría o retracciones. La lesión
mide 2 cm. ¿Cuál es el siguiente paso en el manejo de este paciente?
a) Biopsia de la lesión con guía ecográfica.
b) Mamografía seguida de tomografía computarizada estereotáctica
c) Prueba para el gen BRCA
d) Ultrasonido del seno y consideración para el seno biopsia
e) Espera vigilante y examen de seguimiento por pri-Mary Care médico en 1 año
SS – Med10

79. Un hombre de 28 años con antecedentes de dolor abdominal recurrentes y


diarrea con sangre se presentan a su médico quejándose de dolor rectal
significativo con evacuación intestinal. Ha perdido 15 libras en los últimos 3 meses.
El examen físico revela dolor en el cuadrante inferior derecha e izquierda a la palpación. Los
valores de laboratorio revelan un hematocrito del 28% y un eritrocito elevado velocidad de
sedimentación. Colonoscopia realizada en este el paciente probablemente revelaría:
a) Lesión de masa colónica
b) Diverticulosis
c) Hemorroides internas
d) Intestino normal
e) Mucosa colónica y rectal engrosada friable

80. Un hombre de 55 años se presenta a su médico quejándose de poliuria, polidipsia, polifagia


y sarpullido rojo y escamoso en su cara en las últimas 2 semanas. Él orina 18 veces / día con una
buena fuerza de flujo. La glucemia en ayunas fue 325 mg / dL. Ha perdido 20 libras en los
últimos 2 meses y nunca ha tenido niveles elevados de glucosa en sangre en el pasado. Examen
físico del corazón, pulmones y abdomen normal. ¿Cuál es la explicación más probable por estos
hallazgos?
a) Diabetes mellitus tipo I
b) Diabetes mellitus tipo II
c) Glucagonoma
d) Insulinoma
e) Enfermedad de Verner-Morrison

81. Un hombre de 72 años con un historial de 80 años de fumar, visita a su médico quejándose
de debilidad y malestar general. Recientemente ha desarrollado disfagia para alimentos
sólidos. Ha perdido 15 libras en los últimos 3 meses. El examen físico revela linfadenopatía
supraclavicular derecha. Los exámenes cardíacos y pulmonares son sin complicaciones. Sin
rebote a la palpación abdominal ¿Cuál de los siguientes estudios proporcionará más diagnóstico
definitivo?
a) Esofagograma de bario
b) Tomografía computarizada del abdomen y la pelvis
c) Esofagogastroscopia con biopsia.
d) Resonancia magnética del abdomen
e) Ultrasonido del cuadrante superior derecho

82. Una mujer afroamericana de 42 años se somete a una colecistectomía laparoscópica por
dolor crónico en el cuadrante superior. La tomografía computarizada demostró cálculos biliares
y líquido colecístico. El procedimiento quirúrgico fue incompleto. Análisis patológico de los
cálculos biliares revela cálculos de bilirrubinato de calcio. ¿Cuál es la explicación más probable?
nación de estos hallazgos?
a) Altos niveles de colesterol en suero
b) Niveles elevados de lípidos en suero
c) Dieta alta en grasas
d) Anemia falciforme
e) Tumor

83. Una mujer de 36 años se queja de ictericia y hinchazón periférica oral. Se ordena un
ecocardiograma y el se determina que el paciente tiene insuficiencia cardíaca derecha con
congestión hepática y edema periférico. No se le detecta ningún soplo. ¿Cuál es la explicación
más apropiada para estos hallazgos?
a) Estenosis aórtica
b) Comunicación interauricular
SS – Med10
c) Ductus arterioso permeable
d) Tetralogía de Fallot
e) Defecto septal ventricular

84. Un varón recién nacido a término niño de una mujer de 27 años de edad y con abuso de
drogas venosa tiene una pequeña hernia umbilical. Sus signos vitales son estables. Su cardíaco y
los exámenes pulmonares no son contributivos.
¿Cuál de las siguientes es la explicación más probable para ¿este descubrimiento?
a) Foramen oval permeable
b) Conducto onfalomesentérico permeable
c) Anillo umbilical permeable
d) Uraco patente
e) Conducto vitelino permeable

85. Un hombre de 28 años se presenta en el departamento de emergencias. Se queja de dolor


en el costado izquierdo, náuseas y vómitos. El examen físico del corazón y los pulmones es
normal. Se queja de sensibilidad costovertebral. Análisis de orina revela microhematuria (5
glóbulos rojos / alta potencia campo). La tomografía computarizada revela un cálculo renal
izquierdo de 4 mm, mientras que KUB revela un patrón normal de gas intestinal y no evidencia
de calcificaciones. ¿Cuál es la explicación más probable de estos hallazgos?
a) Calculo de oxalato de calcio
b) Calculo de monohidrato de fosfato de calcio
c) Calculo deshidratada de fosfato de calcio
d) Pequeño calculo de estruvita
e) Calculo de ácido úrico

86. Una mujer de 47 años con pancreatitis de cálculos biliares está hospitalizada. Ella tiene
antecedentes de hipertensión e hipercolesterolemia. La colecistectomía temprana está indicada
para prevenir cuál de las siguientes complicaciones?
a) Colangitis
b) Pancreatitis recurrente
c) Úlcera gástrica
d) Íleo biliar
e) Perforación de la vesícula biliar

87. Un hombre de 56 años se queja de tos recurrente y hemoptisis. Tiene antecedentes de


neumonías recurrentes. Él es un no fumador y no tiene riesgo laboral de pulso enfermedad
pulmonar. El examen físico revela disminución la respiración en el lóbulo superior derecho. La
radiografía de tórax revela una pequeña masa del cuadrante superior derecho. La broncoscopia
revela un angioma ¿Cuál es el tratamiento más apropiado para este paciente?
a) Antibióticos
b) Corticosteroides
c) Lobectomía izquierda
d) Lobectomía del lóbulo superior derecho
e) Espera vigilante

88. Un hombre de 55 años se presenta a su médico de atención primaria, después de notar un


poco de orina teñida de sangre hace 1 semana. Niega cualquier dolor actual y fiebre. Su historial
médico pasado incluye pulso obstructivo crónico enfermedad pulmonar de muchos años de
fumar cigarrillos. La tomografía computarizada revela quistes renales bilaterales simples,
próstata agrandamiento y engrosamiento asimétrico del lado izquierdo de la vejiga. La
hidronefrosis izquierda también está presente. Su análisis de orina es positivo para hematuria
microscópica. ¿Cuál es el diagnóstico más probable?
a) Nefrolitiasis
SS – Med10
b) Cancer de prostata
c) Carcinoma de células renales
d) Carcinoma de células de transición de la vejiga.
e) Infección del tracto urinario

89. Un hombre de 30 años se somete a una tomografía computarizada de su abdomen después


de un accidente automovilístico. Él era un desenfrenado conductor y fue arrojado del
vehículo. Se encuentran lesiones abdominales agudas. La tomografía computarizada revela
riñones bilaterales agrandados con quistes múltiples presentes en diferentes tamaños. El riñón
derecho mide 15 cm y el niño izquierdo mide 16 cm de largo. Examen físico de la corazón,
pulmones y abdomen están dentro de los límites normales, aparte de una leve sensibilidad a la
palpación en el cuadrantes superiores derecho e izquierdo. ¿Cuál de los siguientes las patologías
del sistema nervioso central son más fuertes asociado con este hallazgo?
a) Círculo de aneurisma de Willis
b) Cisticercosis
c) Infarto
d) Glioma
e) Hematoma subdural

90. Una mujer de 46 años con enfermedad renal poliquística recibio un trasplante de riñón. A
los pocos minutos de la anastomosis de la arteria renal a la respectiva arteria y vena ilíaca
externa, el riñón recupera rápidamente una coloración rosa y turbulencia normal del tejido
y comienza a excretar orina. Es visto al mes siguiente, la creatinina sérica es de 4.2 mg / dL. La
producción de orina es 20 ml / hora. Examen físico del corazón, pulmones, y abdomen están
dentro de los límites normales. El riñón transplantado es palpable en la fosa ilíaca derecha. La
biopsia posterior del trasplante muestra un extenso inflamación y edema ¿Cuál es la probable
explicación de estos hallazgos?
a) Rechazo agudo de trasplante
b) Rechazo crónico de trasplante
c) Enfermedad de injerto contra huésped
d) Rechazo de trasplante hiperagudo
e) Proceso normal de trasplante

91. Un hombre de 52 años se queja de dolor crónico abdominal. Ha sido hospitalizado siete
veces en los últimos 2 años para ataques recurrentes de dolor de pancreatitis crónica. Ha sido
tratado con analgésicos y con pancreatectomía distal. Su dolor aún persiste. ¿Cuál es el
siguiente paso en el tratamiento de este paciente?
a) Uso continuo de analgésicos orales.
b) Corticosteroides
c) Esplacnicectomía
d) Pancreatectomía total
e) Espera vigilante

92. Un hombre de 39 años es evaluado por su médico para detectar recurrencias de cálculos
renales. Ha sido tratado en el pasado con litotricia extracorpórea por ondas de choque,
ureteroscopia y una nefrolitotripsia percutánea. ¿Cuál de las siguientes características sugerirían
el diagnóstico de hiperparatiroidismo primaria?
a) 1 mm en rayos X KUB
b) 2 mm en rayos X KUB
c) Lado derecho de 1 mm y 2 mm en rayos X KUB
d) Lados derecho e izquierdo de 1 mm, 2 mm y 3 mm en rayos X KUB
e) Polo inferior derecho de 4 mm en tomografía computarizada
SS – Med10
93. Un médico desea administrar un anestésico local subcutáneo a un paciente con
una lesión sospechosa en su antebrazo. La lesión mide 1 cm y el cirujano planea una
incisión elíptica para eliminarlo. ¿Cuál capa epidérmica de la piel penetrarán
primero el médico con la aguja anestésica local?
a) Estrato basal
b) Estrato córneo
c) Estrato granuloso
d) Stratum lucidum
e) Estrato espinoso

94. Un hombre de 35 años con antecedentes de enfermedad de Crohn previa, visita a su médico
para un examen de seguimiento. Él tiene enfermedad ileocolónica difusa en una tomografía
computarizada reciente. Su actual lista de medicamentos incluyen sulfasalazina. Examen físico
revela dolor en el cuadrante inferior derecho a palpación profunda. Si se considera la terapia
con antibióticos en este paciente, ¿Cuál de los siguientes organismos debería ser el objetivo?
a) Especies de Mycobacterium
b) Pseudomonas aeruginosa
c) Staphylococcus aureus
d) Steotococos neumonia
e) Streptococcus pyogenes

95. Una mujer de 46 años se presenta donde su médico con una historia de disfagia
progresiva. Ella tiene una historia de pérdida de peso de 15 libras en los últimos 6
meses. Examen físico la región del cuello, corazón, pulmones y abdomen son normales. Las
pruebas de laboratorio revelan un hematocrito de 33% y un volumen corpuscular medio de 70.
La endoscopia gastrointestinal revela una red esofágica superior. ¿Cuál es el diagnóstico más
probable?
a) Tumor carcinoide
b) Leucemia
c) Linfoma
d) Síndrome de Plummer-Vinson
e) Artritis reumatoide

96. Un hombre de 47 años está involucrado en un accidente automovilístico. Sufre fracturas de


costillas 9, 10 y 11 en el lado izquierdo. Él esta hemodinámicamente inestable y tiene una
presión arterial de 90/50 mm Hg y pulso de 120 latidos / minuto a pesar de transfusión de 3 U
de glóbulos rojos empaquetados. Es llevado al quirófano para laparotomía exploratoria. Una
ruptura del bazo se identifica y se extrae. En busca de un bazo accesorio potencial, ¿ cuál es la
localización más probable para encontrarlo?
a) Epiplón mayor
b) Cuadrante inferior derecho
c) Cuadrante superior derecho
d) Hilio esplénico
e) Ligamentos esplénicos

97. Un niño de 12 años es llevado a su médico para su evaluación de una masa en el


cuello. Tiene antecedentes de sinusitis recurrente e infecciones amigdalares. El examen físico
revela una masa en el cuello de la línea media que mide 1,5 cm que se mueve con tragar. No
hay evidencia de linfadenopatía. ¿Cuál es el diagnóstico más probable?
a) Leucemia
b) Linfoma
c) Quiste del conducto tirogloso
d) Fístula tiroglosa
e) Carcinoma de tiroides
SS – Med10

98. Durante su rotación de medicina de emergencia se le llama al departamento de


emergencias para evaluar a un paciente con una lesión oftálmica. ¿Cuál de los
siguientes las lesiones traumáticas requieren un manejo inmediato en el sitio y
referencia al oftalmólogo de guardia?
a) Quemadura ácida
b) Nubosidad corneal
c) Laceración corneal
d) Hifema
e) Quemosis conjuntival severa

99. Un paciente se presenta al departamento de emergencias después ser cortado con un


cuchillo en la pierna izquierda. La herida parece limpia y los bordes están bien opuestos. El
siguiente es el método más simple de cierre de heridas por esta lesión?
a) Cierre primario retrasado
b) Injerto
c) Aletas locales
d) Cierre primario
e) Intención secundaria

100. Una mujer de 23 años se presenta a su médico quejándose dolor en la rodilla derecha. El
dolor es constante durante todo el día y ha estado presente durante aproximadamente 3
semanas. La historia social revela que ella está en una relación monógama. El examen físico
revela que su rodilla está ligeramente hinchada y sensible. Se realiza la aspiración de líquido
sinovial. Ni los cristales ni las bacterias son encontrados ¿Cuál es el diagnóstico más probable?
a) Bursitis
b) sArtritis gonocócica
c) Artritis reumatoide
d) Bursitis séptica
e) Trauma
MEDICINA 2019
MINISTERIO
DE SANIDAD, CONSUMO
Y BIENESTAR SOCIAL

PRUEBAS SELECTIVAS 2019 NÚMERO DE MESA:

CUADERNO DE EXAMEN NÚMERO DE EXPEDIENTE:


Nº DE D.N.I. O EQUIVALENTE PARA EXTRANJEROS:
MEDICINA - VERSIÓN: 0
APELLIDOS Y NOMBRE:

ABRIR SOLAMENTE A LA INDICACIÓN DEL TRIBUNAL


ADVERTENCIA IMPORTANTE
ANTES DE COMENZAR SU EXAMEN, LEA ATENTAMENTE LAS SIGUIENTES
INSTRUCCIONES

1. MUY IMPORTANTE: Compruebe que este Cuaderno de Examen, lleva todas sus páginas y no
tiene defectos de impresión. Si detecta alguna anomalía, pida otro Cuaderno de Examen a la Mesa.
Realice esta operación al principio, pues si tiene que cambiar el cuaderno de examen posterior-
mente, se le facilitará una versión "0", que no coincide con su versión personal en la colocación de
preguntas y no dispondrá de tiempo adicional.

2. El cuestionario se compone de 175 preguntas más 10 de reserva. Tenga en cuenta que hay 27
preguntas que están ligadas a una imagen. Todas las imágenes están en un cuadernillo de
imágenes separado.

3. Compruebe que el número de versión de examen que figura en su “Hoja de Respuestas”, coin-
cide con el número de versión que figura en el cuestionario. Compruebe también el resto de sus
datos identificativos.

4. La “Hoja de Respuestas” está nominalizada. Se compone de dos ejemplares en papel autocopia-


tivo que deben colocarse correctamente para permitir la impresión de las contestaciones en todos
ellos. Recuerde que debe firmar esta Hoja.

5. Compruebe que la respuesta que va a señalar en la “Hoja de Respuestas” corresponde al


número de pregunta del cuestionario. Sólo se valoran las respuestas marcadas en la “Hoja de
Respuestas”, siempre que se tengan en cuenta las instrucciones contenidas en la misma.

6. Si inutiliza su “Hoja de Respuestas” pida un nuevo juego de repuesto a la Mesa de Examen y no


olvide consignar sus datos personales.

7. Recuerde que el tiempo de realización de este ejercicio es de cuatro horas improrrogables y


que están prohibidos el uso de calculadoras y la utilización de teléfonos móviles, o de cualquier
otro dispositivo con capacidad de almacenamiento de información o posibilidad de comunicación
mediante voz o datos.

8. No se entregarán, en ningún caso, los cuestionarios con las preguntas de examen. Las distin-
tas versiones de los cuadernos de examen se publicarán en la Web del Ministerio de Sanidad,
Consumo y Bienestar Social, al cierre de la última mesa de examen.
FSE MEDICINA 2019/20
1. Pregunta vinculada a la imagen nº1 4. Pregunta vinculada a la imagen nº4
Mujer de 40 años con obesidad mórbida, que se Mujer de 55 años intervenida quirúrgicamente
somete a cirugía bariátrica. Durante la por lesión tumoral hepática. Se muestra una
intervención quirúrgica se realiza también una imagen macroscópica del tumor y del
biopsia hepática. ¿Qué tipo de lesión histológica parénquima no tumoral, y una imagen
observamos?: microscópica del tumor, que corresponde a uno
de los siguientes diagnósticos:
1. Esteatosis.
2. Esteatohepatitis. 1. Hepatocarcinoma sobre hígado normal.
3. Cirrosis. 2. Hepatocarcinoma sobre hígado cirrótico.
4. Hepatitis crónica. 3. Colangiocarcinoma sobre hígado normal.
4. Colangiocarcinoma sobre hígado cirrótico.
2. Pregunta vinculada a la imagen nº2
5. Pregunta vinculada a la imagen nº5
Hombre de 20 años con antecedentes de asma,
que consulta por disfagia e impactación Mujer de 50 años, sin antecedentes de interés,
alimentaria. Se le realiza una endoscopia que presenta desde hace 3 meses unas lesiones
digestiva alta con toma de biopsias esofágicas. dolorosas en la boca (lengua, paladar, mucosa
¿Con qué diagnóstico es compatible esta biopsia oral) que no han mejorado a pesar de haber

FSE MEDICINA 2019/20


de esófago proximal?: realizado múltiples tratamientos tópicos. Las
lesiones le dificultan la ingesta y refiere haber
1. Esofagitis por reflujo. perdido 7 kg de peso. En las últimas semanas le
2. Esófago de Barrett. han aparecido unas lesiones en el tronco y en la
3. Esofagitis eosinofílica. exploración se observan varias erosiones y
4. Esofagitis por Candida. alguna costra. Al frotar alrededor de las
erosiones la piel vecina se despega fácilmente.
3. Pregunta vinculada a la imagen nº3 ¿Cuál de los siguientes hallazgos NO
encontraríamos en esta paciente?:
Hombre de 67 años con debilidad muscular
proximal de extremidades inferiores de 9 meses
1. Depósitos de IgG y C3 en la membrana basal.
de evolución. No refiere ingesta de fármacos ni
2. Depósitos de IgG y C3 intercelulares.
antecedentes familiares de enfermedades
3. Anticuerpos frente a la desmogleína 1.
neuromusculares. En la exploración física se
4. Anticuerpos frente a la desmogleína 3.
observa debilidad muscular asimétrica de
ambos cuádriceps (3/5 izquierdo, 4/5 derecho) y
6. Pregunta vinculada a la imagen nº6
de los músculos flexores de los dedos de ambas
manos. En la analítica destaca una CK de 750 Paciente de 25 años con crisis epilépticas desde
UI/L (valor normal <150 UI/L). Se realiza una hace 1 año, en tratamiento con carbamazepina y
biopsia del músculo cuádriceps derecho. fenitoína, que por falta de respuesta se cambió
Marque la opción correcta: hace dos semanas a lamotrigina. Consulta por
lesiones cutáneas que comenzaron hace 24
1. Infiltrado inflamatorio perimisial, ragged-red horas con edema facial, y que posteriormente se
fibers (fibras rojo-rotas) y atrofia generalizaron en forma de rash eritematoso
perifascicular: Dermatomiositis. máculopapular. Presenta mal estado general,
2. Infiltrado inflamatorio endomisial y vacuolas poliadenopatías y fiebre de 38,5°C. En la
ribeteadas: Miositis con cuerpos de inclusión. analítica destaca GOT 230 U/L, GPT 253 U/L,
3. Necrosis coagulativa, inclusiones lipídicas y GGT 145 U/L y eosinofilia de 1200 / µL. El
regeneración de fibras musculares: Miopatía diagnóstico más probable es:
necrosante autoinmune.
4. Atrofia de fibras tipo II, infiltrado 1. Sarampión.
inflamatorio endomisial y regeneración de 2. Síndrome DRESS.
fibras musculares: Distrofia miotónica de 3. Síndrome de Stevens Johnson.
Steinert. 4. Angioedema.

- 1-
7. Pregunta vinculada a la imagen nº7 10. Pregunta vinculada a la imagen nº10
Niño de 9 meses con antecedentes de tres Lactante de 25 días que presenta lesiones en la
episodios de infección del tracto urinario (a los cara desde hace 2 días que se van acentuando.
14 días de vida, 1 mes y 2 meses) y en En la exploración se observan máculas
tratamiento antibiótico profiláctico. Los estudios eritematosas confluyentes, descamativas, con
diagnósticos realizados muestran reflujo secreción serosa amarillenta, localizadas en
vésicoureteral bilateral (grado V derecho y II ambas mejillas, surco nasogeniano, orejas y
izquierdo) y disminución del tamaño del riñón región retroauricular, frente y mentón. Resto de
derecho con leve dilatación pielocalicial (II/V) en la exploración normal. ¿Qué diagnóstico es más
la ecografía. Con estos datos y la imagen probable?:
gammagráfica renal que se muestra, el
diagnóstico más probable es: 1. Eritema toxoalérgico del recién nacido.
2. Dermatitis atópica.
1. Nefropatía cicatricial derecha post- 3. Dermatitis seborreica.
pielonefritis. 4. Impétigo neonatal.
2. Estenosis de la unión pieloureteral derecha.
3. Pielonefritis aguda derecha. 11. Pregunta vinculada a la imagen nº11
4. Displasia multiquística del riñón derecho.
Joven de 15 años sin antecedentes de interés.

FSE MEDICINA 2019/20


Presenta tumoración ósea malar izquierda
8. Pregunta vinculada a la imagen nº8
deformante con afectación orbitaria externa de
Mujer de 28 años sin antecedentes personales ni años de evolución. ¿Cuál es el diagnóstico más
familiares de interés, que consulta por aparición probable que sugiere la imagen mostrada?:
de una masa supraclavicular izquierda, sin otra
sintomatología acompañante. Se le realiza 1. Granuloma eosinófilo.
analítica básica en la que únicamente destaca 2. Metástasis ósea.
una LDH de 373 UI/L. Considerando los datos 3. Displasia fibrosa.
clínicos y la prueba de imagen (PET-TC con 4. Hiperparatiroidismo.
18F-FDG) ¿qué diagnóstico es el más probable?:
12. Pregunta vinculada a la imagen nº12
1. Carcinoma tímico con diseminación a
Paciente de 72 años remitido a urgencias tras
distancia.
padecer un síncope. Presenta hipotensión y
2. Enfermedad asociada a IgG4.
dolor abdominal con irradiación lumbar. Se
3. Sarcoidosis.
realiza TC abdominal con contraste
4. Síndrome linfoproliferativo estadio IV.
intravenoso. ¿Cuál sería su diagnóstico y la
actitud a seguir?:
9. Pregunta vinculada a la imagen nº9
Mujer de 39 años que consulta por hemorragia 1. Absceso de psoas izquierdo. Realizaría
genital desde hace 24 horas. Su última regla fue tratamiento sintomático y lo mantendría en
hace 7 semanas. No tiene dolor, aunque presenta observación a la espera de analíticas y otras
náuseas y vómitos persistentes desde hace 4 días. pruebas complementarias.
La exploración ginecológica muestra sangrado 2. Aneurisma de aorta abdominal roto. Es una
escaso procedente de la cavidad uterina. El test emergencia y la primera opción es
de embarazo es positivo y la determinación de ß- tratamiento quirúrgico o endovascular.
HCG en plasma es de 105.000 UI/L. Se aporta 3. Completaría el estudio con una ecografía
imagen de la ecografía transvaginal. ¿Qué abdominal para realizar el diagnóstico
indicaría en ese momento?: diferencial entre absceso de psoas y
aneurisma de aorta abdominal roto.
1. Repetir seriadamente cada 2 días la ecografía 4. Tumoración hipervascular del psoas.
y la ß-HCG. Realizaría resonancia magnética para una
2. Tranquilizar a la paciente, indicar reposo, mejor caracterización.
antieméticos y repetir la ecografía en una
semana.
3. Legrado uterino.
4. Tratamiento con misoprostol por vía
sistémica.

- 2-
13. Pregunta vinculada a la imagen nº13 16. Pregunta vinculada a la imagen nº16
Mujer de 72 años con antecedentes de Mujer de 94 años que acude a urgencias por
fibrilación auricular en tratamiento con dolor abdominal difuso e intenso de 12 horas de
anticoagulantes orales. Presenta dolor evolución y defensa abdominal. Entre sus
abdominal focalizado en hipogastrio y fosa ilíaca antecedentes destaca cardiopatía isquémica
izquierda. INR 5. Hemoglobina 8,7 g/dl, crónica, fibrilación auricular persistente y
hematocrito 25 %. ¿Cuál es el diagnóstico que le enfermedad renal crónica. A raíz de los
sugiere la imagen?: hallazgos de la radiografía de abdomen indique
la actitud a seguir:
1. Hernia complicada de pared abdominal.
2. Seroma post-quirúrgico. 1. Ecografía urgente como test de cribado
3. Absceso en la vaina del recto anterior inicial, si es normal no son necesarios más
izquierdo del abdomen. estudios radiológicos.
4. Hematoma en la vaina del recto anterior 2. Radiografía de abdomen en bipedestación o
izquierdo del abdomen. decúbito lateral izquierdo para valorar
presencia de gas ectópico.
14. Pregunta vinculada a la imagen nº14 3. TC abdominopélvico con contraste
intravenoso con protocolo de angio-TC por
Joven de 16 años sin antecedentes previos, que

FSE MEDICINA 2019/20


sospecha de isquemia mesentérica.
es traído a urgencias por sus familiares por
4. Arteriografía para descartar oclusión aguda
dificultad respiratoria aguda y dolor torácico. A
de arteria mesentérica.
su llegada muestra intenso trabajo respiratorio,
frecuencia cardiaca 120 lpm, tensión arterial
17. Pregunta vinculada a la imagen nº17
75/40 mmHg y SatO2 86 % con aire ambiente.
Considerando la radiografía de tórax, ¿cuál es Mujer de 42 años, peluquera de profesión, que
la medida a tomar más apropiada?: refiere omalgia al elevar el brazo. Tras una
semana de evolución el dolor se presenta
1. Realizar una TC torácica de alta resolución. también en reposo y por las noches. ¿Cuál es el
2. Repetir la radiografía añadiendo una diagnóstico más probable según la radiografía
proyección lateral. del hombro?:
3. Realizar fibrinolisis urgente con 100 mg de
tenecteplasa. 1. Capsulitis adhesiva.
4. Colocar un drenaje torácico en la cavidad 2. Lesión del labrum
pleural derecha. 3. Bursitis subacromial.
4. Tendinitis cálcica.
15. Pregunta vinculada a la imagen nº15
18. Pregunta vinculada a la imagen nº18
Hombre de 37 años no fumador y sin
antecedentes médicos de interés que presenta un Hombre de 56 años, fumador activo, no
cuadro de más de tres semanas de evolución de hipertenso ni dislipémico, que consulta porque
tos seca persistente, febrícula y dolores hace 7 días presentó malestar general con dolor
articulares erráticos. Entre las pruebas que le centrotorácico y disnea de esfuerzo,
solicita su médico se incluye una radiografía de encontrándose asintomático en el momento de la
tórax. ¿Cuál de los siguientes diagnósticos consulta. ¿Cuál es el diagnóstico más probable,
sospecha en primer lugar?: según el trazado electrocardiográfico?:

1. Tuberculosis. 1. Tromboembolismo pulmonar.


2. Artritis reumatoide. 2. Miocardiopatía hipertrófica.
3. Hipertensión arterial pulmonar secundaria a 3. Necrosis e isquemia subepicárdica anterior.
insuficiencia cardiaca izquierda. 4. Miopericarditis aguda.
4. Sarcoidosis.

- 3-
19. Pregunta vinculada a la imagen nº19 22. Pregunta vinculada a la imagen nº22
Hombre de 41 años que consulta por diplopia, Mujer de 85 años que acude a urgencias por
ptosis palpebral y dolor ocular derechos de dos náuseas, vómitos y distensión abdominal. No
días de evolución. Se muestra el estudio de refiere cirugías abdominales previas pero sí
resonancia nuclear magnética (corte coronal). El varios episodios de colecistitis. En la analítica
diagnóstico más probable es: urgente se evidencia discreta leucocitosis con
desviación izquierda y PCR elevada. Se aporta
1. Compresión tumoral del quiasma óptico. radiografía simple de abdomen y TC abdominal.
2. Hemorragia intraparenquimatosa. ¿Cuál de las siguientes actuaciones considera
3. Infiltración tumoral del seno cavernoso más recomendable?:
derecho.
4. Esclerosis múltiple. 1. Colecistectomía laparoscópica.
2. Sonda nasogástrica descompresiva.
20. Pregunta vinculada a la imagen nº20 3. Enterolitotomía quirúrgica.
4. Resección segmentaria de ileon terminal.
Mujer de 32 años con antecedentes de epilepsia
desde la infancia, retraso mental, angiofibromas
23. Pregunta vinculada a la imagen nº23
faciales y manchas hipomelánicas. Por un
cuadro de dolor abdominal se le realiza una Un paciente presenta una fractura cerrada cuya

FSE MEDICINA 2019/20


tomografía computarizada abdominal donde se radiografía en proyección ántero-posterior
aprecian lesiones renales. ¿De qué tipo de lesión puede revisar en la imagen adjunta. ¿Cuál de las
se trata y que enfermedad padece?: siguientes opciones de tratamiento le parece
correcta?:
1. Quiste renal / poliquistosis renal autosómica
dominante. 1. Clavo intramedular acerrojado proximal y
2. Carcinoma renal de células claras / distal.
enfermedad de Von Hippel Lindau. 2. Ortesis de tipo PTB (patellar tendon bearing)
3. Angiomiolipoma renal / esclerosis tuberosa. y deambulación desde el momento inicial.
4. Neurofibroma / neurofibromatosis. 3. Fijador externo circular y deambulación
precoz.
21. Pregunta vinculada a la imagen nº21 4. Reducción abierta y placa y tornillos
bloqueados.
Hombre de 22 años que consulta por dolor
ocular izquierdo desde hace 48 h. En las últimas
24. Pregunta vinculada a la imagen nº24
horas se añade pérdida visual ipsilateral, con
agudeza visual 0,6/1 en ojo izquierdo y 1/1 en A un paciente de 55 años se le solicita una
ojo derecho. Aporta la resonancia magnética de MAPA (Monitorización Ambulatoria de la
la imagen. Señale la respuesta correcta: Presión Arterial) de 24 horas, por presentar en
consulta valores de presión arterial en la
1. Tiene en torno al 85 % de probabilidades de categoría normal-alta, para descartar una
presentar bandas oligoclonales en el líquido hipertensión arterial enmascarada. El resultado
cefalorraquídeo. del estudio se observa en la imagen. Los
2. No necesita pruebas adicionales para promedios de presión arterial fueron: media de
establecer el diagnóstico. 24 horas 121/73 mmHg, media en el periodo
3. Lo importante es tratar el cuadro y luego ya diurno 124/75 mmHg, media en el periodo
completaremos el estudio. nocturno 111/65 mmHg. La presión arterial
4. Posiblemente se trata de una neuromielitis sistólica nocturna descendió un 10,4 % de la
óptica. media de los valores diurnos y la presión
arterial diastólica nocturna descendió un 13,9 %
de la media de los valores diurnos. De las
siguientes opciones ¿cuál se corresponde mejor
con la MAPA realizada?:

1. Normotensión y patrón dipper.


2. Normotensión y patrón no dipper.
3. Hipertensión y patrón dipper.
4. Hipertensión y patrón no dipper.

- 4-
25. Pregunta vinculada a la imagen nº25 27. Pregunta vinculada a la imagen nº27
Hombre de 47 años que acude por presentar Hombre de 56 años con antecedentes de
desde hace 48 horas fiebre y dolor torácico de hipertensión arterial y cáncer de colon con
características pleuríticas en hemitórax metástasis hepáticas, actualmente en
izquierdo, a pesar de estar recibiendo tratamiento quimioterápico. Acude a urgencias
tratamiento desde hace 15 días con por disnea. Constantes: frecuencia cardiaca 110
amoxicilina/clavulánico por una sinusitis de lpm, tensión arterial 115/55 mmHg, frecuencia
evolución tórpida. En la exploración la respiratoria 29 rpm, SatO2 89 %. Exploración:
temperatura es de 37,8°C, la tensión arterial uso de musculatura respiratoria accesoria,
180/100 mmHg, destaca el aspecto postrado del murmullo vesicular conservado en todos los
paciente. En la exploración cardiorrespiratoria campos. La primera prueba diagnóstica de la
y abdominal únicamente destacan unos que se dispone es el electrocardiograma. ¿Cuál
crepitantes audibles en hemitórax izquierdo. es la sospecha clínica más probable?:
Exploración neurológica normal. En la analítica
destaca hemoglobina 10,5 g/dL y VCM 89 fL, 1. Fibrilación auricular.
12.000 leucocitos/mm3 con discreta neutrofilia y 2. Tromboembolismo pulmonar.
plaquetas de 450.000/mm3. La creatinina es de 3. Síndrome coronario sin elevación del
1,7 mg/dL. Una tira reactiva urinaria muestra segmento ST (SCASEST).

FSE MEDICINA 2019/20


5-10 leucocitos, 5-10 hematíes y proteínas +. Se 4. Infarto agudo de miocardio de cara
muestra la radiografía de tórax póstero- diafragmática.
anterior. Indique cuál de las siguientes
determinaciones analíticas solicitaría para 28. En relación con las proteínas plasmáticas, señale
confirmar el diagnóstico: la afirmación INCORRECTA:

1. Anticuerpos anti-proteinasa 3. 1. La albúmina se sintetiza en el hígado y se


2. Enzima convertidor de la angiotensina. metaboliza en numerosos tejidos,
3. Galactomanano. permitiéndoles a los mismos aprovechar los
4. Anticuerpos anti-membrana basal aminoácidos provenientes de su degradación.
glomerular. 2. La albúmina es la proteína mayoritaria del
plasma, por ello participa en el
26. Pregunta vinculada a la imagen nº26 mantenimiento de la presión oncótica.
3. La albúmina interviene en el transporte de
Mujer de 90 años con antecedentes de
bilirrubina directa o conjugada, como así
enfermedad de Alzheimer en estadio terminal
también de ácidos grasos, calcio, y algunas
(Global Deterioration Scale 7), dependencia para
vitaminas, hormonas y fármacos.
todas las actividades básicas de la vida diaria y
4. Al existir numerosas variantes genéticas de la
disfagia ocasional a líquidos. Acude a urgencias
albúmina, es posible observar una doble
por presentar en las últimas horas delírium
banda de albúmina en un proteinograma, que
hipoactivo, taquipnea, taquicardia rítmica y
se conoce como bisalbuminemia, pero que
temperatura axilar de 37,3°C. La exploración
carece de significación clínica.
física es dificultosa y no aporta más datos
destacables. Ante la radiografía de tórax que se
muestra, ¿cuál es el diagnóstico más probable?:

1. Tromboembolismo pulmonar.
2. Edema agudo de pulmón.
3. Neumonía intersticial aguda.
4. Neumonía broncoalveolar aguda.

- 5-
29. En relación con la vía de las pentosas fosfato 33. Referido a los sesgos que afectan a la evaluación
(conocida también como derivación de la hexosa de las pruebas diagnósticas, señale la respuesta
monofosfato), indique la respuesta correcta: INCORRECTA:

1. La primera reacción de la rama no oxidativa 1. El sesgo de adelanto diagnóstico se


de esta vía es la transformación de la glucosa caracteriza porque la prueba, a pesar de no
6-fosfato por la glucosa 6-fosfato mejorar el pronóstico, parece mejorar la
deshidrogenasa. supervivencia de los pacientes.
2. Cuando la célula sólo requiere NADPH 2. El sesgo del voluntario sano se debe a que
(forma reducida de nicotinamida adenina estos sujetos suelen ser más responsables de
dinucléotido fosfato), la pentosa 5-fosfato de su salud, por lo que tendrían una mayor
la vía se recicla a glucosa 6-fosfato mediante supervivencia independientemente del
una carboxilación dependiente de biotina. cribado.
3. Las formas anormales de glucosa 6-fosfato 3. El sesgo de duración se produce por
deshidrogenasa con menor actividad seleccionar casos más graves y más letales.
catalítica afectan gravemente a los tejidos 4. El mejor método para eliminar estos sesgos
porque carecen de mecanismos alternativos es la aleatorización de los participantes; un
para sintetizar NADPH o para inducir la grupo se someterá al cribado y otro no.
síntesis del enzima.

FSE MEDICINA 2019/20


4. La obtención de NADPH por esta vía protege 34. Con respecto al riesgo relativo, señale la
particularmente a los eritrocitos de la respuesta INCORRECTA:
agresión oxidativa.
1. Es la razón entre la incidencia acumulada en
30. Para conocer si el uso habitual de ácido los expuestos y la incidencia acumulada en
acetilsalicílico se asocia a un mayor riesgo de los no expuestos.
hipertensión se selecciona un grupo de sujetos, 2. Sus valores varían entre 0 y 1.
se averigua cuántos están tomando ácido 3. No tiene unidades.
acetilsalicílico y se les sigue durante 5 años para 4. Para su correcta interpretación es necesario
identificar los casos nuevos de hipertensión. conocer su intervalo de confianza.
Señale la respuesta correcta sobre el diseño de
este estudio: 35. De los cinco momentos recomendados en la
higiene de manos, señale la respuesta
1. Es un ensayo clínico porque se realiza con INCORRECTA:
fármacos.
2. Es un estudio ecológico porque se sigue a 1. Antes del contacto con el paciente.
muchos sujetos. 2. Después del contacto con el paciente.
3. Es un estudio de casos y controles, en el que 3. Antes del contacto con el entorno del
los casos toman aspirina y los controles no. paciente.
4. Es un estudio de cohortes, porque se sigue a 4. Después del contacto con el entorno del
sujetos clasificados según su exposición para paciente.
identificar el riesgo de una enfermedad.
36. Entre las precauciones a tomar en el aislamiento
31. En una población se produjeron 2.000 por gotas se EXCLUYE:
accidentes mortales de tráfico desde el 1 de
enero al 31 de diciembre de 2019. ¿Qué medida 1. Habitación individual.
de frecuencia de accidentes se ha utilizado?: 2. Mascarilla de partículas (N95).
3. La puerta de la habitación puede permanecer
1. Prevalencia puntual. abierta.
2. Prevalencia de periodo. 4. Poner al paciente una mascarilla quirúrgica si
3. Letalidad. es preciso trasladarlo fuera de la habitación.
4. Incidencia.

32. De los siguientes criterios de causalidad en una


asociación estadística entre una exposición y un
resultado, ¿cuál es el único necesario?:

1. Plausibilidad o coherencia biológica.


2. Consistencia.
3. Relación temporal adecuada.
4. Fuerza de la asociación.

- 6-
37. Informamos a la gerencia de nuestro hospital de 42. En relación con el control del flujo sanguíneo
que la densidad de incidencia de infección en la coronario, es FALSO que:
Unidad de Cuidados Intensivos (UCI) es de 15
por mil y nos preguntan qué significa esa cifra. 1. Si aumenta la fuerza de la contracción
Nuestra contestación debería ser: cardiaca, también aumenta la velocidad del
flujo coronario.
1. Si hubiera 1.000 pacientes ingresados en la 2. En presencia de concentraciones muy bajas
UCI, cada día habría 15 infecciones nuevas. de oxígeno en los miocitos, aumenta la
2. De cada 1.000 ingresos en el hospital, 15 se disponibilidad de adenosina.
infectan en la UCI. 3. El efecto directo de la activación simpática
3. El riesgo diario de ingresar en la UCI es de coronaria puede provocar tanto
15 cada 1.000 habitantes en la población de vasocostricción como vasodilatación.
referencia. 4. Los efectos directo e indirecto de la actividad
4. Se infectan 15 pacientes de cada 1.000 que parasimpática coronaria (vagal) provocan
ingresan en la UCI. vasodilatación de las arterias coronarias.

38. Una mujer embarazada acude a la consulta por 43. Indique cuál de los siguientes factores provoca
una hipercolesterolemia. ¿Qué fármaco estará un aumento de la cantidad de oxígeno que libera
contraindicado por el riesgo elevado de la hemoglobina:

FSE MEDICINA 2019/20


anomalías fetales demostrado en humanos, que
excede a su posible beneficio terapéutico?: 1. Disminución de los iones hidrógeno.
2. Disminución del CO2.
1. Ácido nicotínico. 3. Aumento de la temperatura.
2. Gemfibrozilo. 4. Disminución del 2,3-bis-fosfoglicerato.
3. Simvastatina.
4. Colestiramina. 44. El proceso de anticipación genética, por el que
las manifestaciones clínicas de una enfermedad
39. Ante un paciente con enfermedad de Parkinson son más graves a medida que pasa de una
que presenta náuseas al comienzo del generación a otra, es consecuencia
tratamiento con agonistas dopaminérgicos, ¿qué habitualmente de:
fármaco utilizaría para mejorar dicho síntoma?:
1. Acumulación de mutaciones puntuales en un
1. Cleboprida. gen a medida que pasan las generaciones.
2. Domperidona. 2. Acumulación de mutaciones puntuales en
3. Metoclopramida. varios genes de la misma vía a medida que
4. Sulpirida. pasan las generaciones.
3. Mutaciones por expansión de tripletes.
40. Uno de los siguientes fármacos es específico para 4. Cambios transgeneracionales en la
prevenir la aparición de crisis epilépticas acetilación de la cromatina.
generalizadas no convulsivas, por bloquear los
canales de calcio de tipo T. Indique cuál: 45. Mujer de 15 años que presenta un retraso en la
menarquia y una talla baja. No tiene
1. Levetiracetam. discapacidad intelectual. ¿Cuál de las siguientes
2. Zonisamida. pruebas genéticas se utilizaría habitualmente
3. Lamotrigina. para el diagnóstico de esta paciente?:
4. Etosuximida.
1. Secuenciación masiva (NGS).
41. Si utilizamos una pauta de administración 2. FISH.
repetida con un fármaco que tiene una vida 3. Microarrays de ADN y/o de ARN.
media de 8 horas, la concentración plasmática 4. Cariotipo.
en equilibrio estacionario se alcanzará:

1. Tras 16 horas del comienzo de la


administración.
2. Tras 24 horas del comienzo de la
administración.
3. Tras 32 horas del comienzo de la
administración.
4. Tras 40 horas del comienzo de la
administración.

- 7-
46. En el tratamiento de la enfermedad inflamatoria 49. Hombre de 60 años en tratamiento con
intestinal pueden utilizarse anticuerpos carbamazepina por padecer epilepsia y que
monoclonales humanizados dirigidos frente al presenta fiebre (38,8°C), odinofagia,
TNF-alfa (como el adalimumab) o frente a la conjuntivitis, lesiones cutáneas parduzcas
integrina a4ß7 (como el vedolizumab). En extendidas en una gran parte de la superficie
relación con la integrina a4ß7 (también llamada corporal, de aspecto similar a una diana y que se
LPAM-1) es cierto que: acompañan de desprendimiento epidérmico
(superior al 30 %) al menor roce. Es muy
1. Se une a ICAM-1 y es responsable del paso probable que esté presentando un cuadro clínico
de linfocitos desde la sangre periférica a los de:
ganglios linfáticos.
2. Es un receptor para TNF-a en los linfocitos T 1. Urticaria aguda de causa farmacológica.
CD4+ del sistema linfoide asociado a 2. Eritema exudativo minor secundario a
mucosas. fármacos.
3. Se expresa en las células del epitelio 3. Síndrome de Stevens-Johnson farmacológico.
gastrointestinal en pacientes con enfermedad 4. Necrolisis tóxica epidérmica.
inflamatoria intestinal.
4. Se une a la adresina MadCAM-1 y favorece 50. Son causa de calcificación metastásica todas las
el paso de linfocitos desde la sangre siguientes EXCEPTO:

FSE MEDICINA 2019/20


periférica a la placa de Peyer y la lámina
propia intestinal. 1. Necrosis celular.
2. Enfermedad de Paget del hueso.
47. El Ministerio de Sanidad recomienda que 3. Metástasis ósea difusa.
durante el tercer trimestre del embarazo las 4. Elevación de hormona paratiroidea.
mujeres sean revacunadas frente a Bordetella
pertussis, causante de la tosferina. ¿Por qué?: 51. El vasopresor de elección para mantener la
presión de perfusión en un enfermo con shock
1. Para que el sistema inmune de la madre séptico es:
produzca anticuerpos de isotipo IgM,
específicos contra el patógeno, que permitan 1. Dopamina.
que tanto ella como el feto se encuentren 2. Noradrenalina.
inmunizados. 3. Dobutamina.
2. Para que el sistema inmune de la madre 4. Fenilefrina.
produzca anticuerpos de isotipo IgG,
específicos contra el patógeno, que permitan 52. Durante una anestesia general la temperatura
que, tras el nacimiento, el bebé se encuentre del paciente ha ido aumentando
inmunizado de forma pasiva. progresivamente y usted llega a la conclusión de
3. Para que el sistema inmune del feto produzca que sufre un cuadro de hipertermia maligna.
anticuerpos de isotipos IgM e IgG, Referente a esta complicación, una de las
específicos contra el patógeno, que permitan siguientes afirmaciones es cierta:
que, tras el nacimiento, el bebé se encuentre
inmunizado de forma activa. 1. Está producida por el uso de anestésicos
4. Para que el sistema inmune de la madre halogenados y relajantes musculares no
produzca linfocitos T memoria, que despolarizantes. Debe tratarse con
atraviesen la placenta y permitan que, tras el benzodiacepinas.
nacimiento, el bebé se encuentre inmunizado 2. Está producida por el uso de barbituricos y
de forma pasiva. opioides. Debe tratarse con dantroleno.
3. Está producida por el uso de anestésicos
48. En una mujer de 40 años, no embarazada y sin halogenados y relajantes musculares
antecedentes patológicos de interés, despolarizantes. Debe tratarse con
diagnosticada de rinitis alérgica leve-moderada dantroleno.
intermitente por sensibilización a pólenes de 4. Está producida por el uso de ketamina y
gramíneas, ¿cuál de los siguientes tratamientos anestésicos locales. No tiene tratamiento.
sintomáticos es de elección?:

1. Hidroxicina oral y bromuro de ipratropio


(aerosol nasal).
2. Bilastina oral y mometasona (aerosol nasal).
3. Dexclorfeniramina oral y cromoglicato
tópico nasal.
4. Ketotifeno oral y oximetazolina tópica nasal.

- 8-
53. ¿Cuál de las siguientes funciones NO 56. En cuanto a la reconstrucción de la sonrisa en
corresponde a los Comités de Ética Asistencial?: pacientes con parálisis facial, podemos afirmar
que:
1. Realizar el asesoramiento ético preceptivo de
un ensayo clínico con seres humanos. 1. No se recomienda realizar una reconstrucción
2. Asesorar en casos clínicos con dilemas éticos dinámica sin hacer previamente una
complejos donde es recomendable una reconstrucción estática.
deliberación cuidadosa. 2. La reconstrucción dinámica consiste en
3. Explorar las necesidades formativas en el realizar una transferencia de músculo
campo de la bioética de los profesionales de neurotizado.
la institución. 3. La reconstrucción estática está indicada en
4. Asesorar a pacientes o familiares que casos en los que la parálisis facial lleva
consultan sobre un problema ético en establecida menos de un año.
relación con la asistencia sanitaria. 4. Las suturas nerviosas directas desde un
nervio motor con el nervio facial dañado
54. Respecto a los tumores de las glándulas pueden ser realizadas en los primeros meses
salivales: de producirse la lesión del nervio facial.

1. El carcinoma adenoide quístico es el tumor 57. En relación con los analgésicos opioides potentes

FSE MEDICINA 2019/20


más frecuente de las glándulas salivales y que se utilizan habitualmente para tratar el
asienta típicamente en la glándula parótida. dolor intenso en pacientes con enfermedad
2. El carcinoma adenoide quístico es el tumor crónica avanzada irreversible, señale la
más frecuente de las glándulas salivales y respuesta INCORRECTA:
asienta típicamente en la glándula
submaxilar. 1. El fentanilo debe ajustarse en caso de
3. El adenoma pleomorfo es el tumor más insuficiencia renal.
frecuente de las glándulas salivales y asienta 2. La morfina presenta una vida media de 4
típicamente en la glándula parótida. horas.
4. El adenoma pleomorfo es el tumor más 3. La oxicodona presenta menor incidencia de
frecuente de las glándulas salivales y asienta náuseas que la morfina.
típicamente en la glándula submaxilar. 4. La buprenorfina se puede administrar
transdérmicamente.
55. Señale cuál de los siguientes es el colgajo
locorregional indicado para la cobertura de un 58. Ante un paciente que se encuentra en situación
defecto cutáneo de la cara interna de la rodilla: terminal, con un pronóstico vital inferior a un
mes, y que presenta un cuadro de anorexia
1. Colgajo de músculo recto abdominal intensa, señale cuál de los siguientes
transverso basado en la arteria epigástrica tratamientos le parece más apropiado para
inferior profunda. tratar este síntoma:
2. Colgajo de gastrocnemio medial, basado en
la arteria sural media. 1. Acetato de megestrol (320 - 480 mg/día).
3. Colgajo anterolateral de muslo, basado en la 2. Paroxetina (10 - 20 mg/día).
rama descendente de la arteria circunfleja 3. Oxibutinina (2,5 – 10 mg/día).
femoral lateral. 4. Dexametasona (4 - 8 mg/día).
4. Colgajo radial fasciocutáneo, basado en
ramas perforantes de la arteria radial. 59. De los siguientes métodos, ¿cuál es el más fiable
para la estimación del intervalo post-mortal en
el período precoz de la muerte?:

1. Cuantificación de potasio en humor vítreo.


2. Observación de la evolución de las livideces
cadavéricas.
3. Determinación de la temperatura rectal.
4. Valoración de la reactividad ocular al colirio
de acetilcolina.

- 9-
60. En una paciente menor de edad, todas las 64. Mujer de 65 años sin comorbilidades, operada
infecciones siguientes se deben sospechar como de cirugía conservadora de mama izquierda con
causadas por contacto sexual, una vez que se ha biopsia selectiva del ganglio centinela. La
descartado la transmisión perinatal. Solo una de anatomía patológica de la pieza muestra un
ellas tiene un origen probablemente NO sexual carcinoma ductal infiltrante grado III de 11
en niñas pequeñas. Indique cuál: mm, dos ganglios centinelas negativos,
receptores de estrógenos: 0%, receptores de
1. Neisseria gonorrhoeae faringea. progesterona: 0%, Ki 67: 70%, HER2: negativo.
2. Chlamydia trachomatis genital. El estudio de extensión es negativo. El
3. Molluscum contagiosum del área genital. tratamiento adyuvante sería:
4. Trichomona vaginalis.
1. Quimioterapia basada en antraciclinas y
61. ¿Cuál sería el manejo de una paciente de 31 taxanos, radioterapia y trastuzumab.
años, nuligesta y asintomática, a la que 2. Quimioterapia basada en antraciclinas y
diagnosticamos en una ecografía rutinaria una taxanos, radioterapia y tamoxifeno.
lesión compatible con un endometrioma ovárico 3. Quimioterapia basada en antraciclinas y
derecho de 4 cm?: taxanos, y radioterapia.
4. Quimioterapia basada en antraciclinas y
1. Exploración ginecológica y estudio taxanos, radioterapia e inhibidores de la

FSE MEDICINA 2019/20


ecográfico reglado para despistaje de aromatasa, dado que es postmenopáusica.
endometriosis profunda.
2. Laparoscopia diagnóstico- terapéutica para 65. Mujer de 30 años que acude a su consulta
diagnóstico de confirmación y de extensión porque se nota un bulto en la mama derecha de
de enfermedad. reciente aparición. Su abuela tuvo cáncer de
3. Quistectomía derecha laparoscópica. mama. A la exploración se palpa en cuadrante
4. Inicio de tratamiento con análogos de la súpero-externo un nódulo de 2,5 cm de bordes
hormona liberadora de gonadotropina regulares. No tiene ninguna prueba de imagen
(GnRH) para evitar la progresión de la previa. Señale la respuesta correcta:
enfermedad.
1. Solicita una ecografía mamaria dada la edad
62. ¿Cuál es el marcador ecográfico más importante de la paciente.
en el cribado de aneuploidías en el primer 2. Solicita una mamografía porque es la prueba
trimestre del embarazo?: diagnóstica “patrón oro".
3. Solicita una resonancia magnética dada la
1. La ausencia de hueso nasal. edad de la paciente y el antecedente familiar.
2. La alteración de la onda del ductus venoso. 4. Le explica que, al tener 30 años,
3. La presencia de quistes de plexos coroideos. probablemente sea un quiste y optará por un
4. La medida de la translucencia nucal. control clínico en 6-12 meses. Si persiste,
entonces solicitará una prueba de imagen.
63. Los siguientes parámetros, EXCEPTO uno, son
un dato rutinario de control periódico en la 66. Primigesta de 40+2 semanas que acude a
rotura prematura de membranas en una urgencias con amniorrea manifiesta y dinámica
gestación pretérmino. Indique cuál: regular. El seguimiento en el embarazo ha sido
normal y posee una determinación vagino-rectal
1. Temperatura corporal. positiva para el estreptococo del grupo B. A la
2. Longitud cervical. exploración obstétrica presenta un cérvix con 5
3. Hemograma y determinación de proteína C cm de dilatación, borrado un 80 % y centrado,
reactiva. con un feto único en presentación cefálica.
4. Registro cardiotocográfico externo. ¿Cuál sería la actitud a seguir con esta
paciente?:

1. Ingreso hospitalario en planta de obstetricia y


observación.
2. Regreso a domicilio y profilaxis antibiótica
con penicilina en periodo expulsivo.
3. Valoración en consultas externas de
obstetricia para realización de ecografía
obstétrica y Doppler fetal.
4. Ingreso hospitalario en sala de dilatación y
profilaxis antibiótica con penicilina.

- 10-
67. Una mujer de 34 años diagnosticada de 70. Mujer de 67 años que consulta por presentar en
síndrome de ovarios poliquísticos presenta su ojo izquierdo desde hace tres días, por la
esterilidad primaria de 2 años de evolución y ha noche, unos destellos luminosos al mover la
sido tratada sin éxito durante 6 meses con cabeza. Desde esta mañana nota una zona
citrato de clomifeno (100 mg/día durante 5 días). oscura que se desplaza con la mirada. La
No asocia otras causas de esterilidad. ¿Cuál agudeza visual es de 1,0 en ambos ojos. ¿Cuál
sería el tratamiento más adecuado en este será la actitud correcta?:
momento?:
1. Se debe realizar un campo visual para
1. Inseminación artificial y FSH (hormona confirmar la presencia de un desprendimiento
foliculoestimulante). de vítreo posterior, ya que esta es la única
2. Aumentar la dosis de citrato de clomifeno a forma de diferenciar un desprendimiento de
200 mg/día 5 días, durante 6 meses más. vítreo posterior de un desprendimiento de
3. Fecundación in vitro (FIV). retina.
4. Metformina. 2. La presencia de fosfenos es un signo
patognomónico del desprendimiento de
68. ¿Qué consejo genético y reproductivo indicaría vítreo posterior, que debe ser tratado con
a una mujer de 30 años que consulta por ser retinopexia neumática.
portadora de una premutación en el gen FMR1, 3. La aparición de fosfenos seguida de una

FSE MEDICINA 2019/20


responsable del síndrome del cromosoma X mancha oscura que se desplaza con la mirada
frágil, y que desea tener descendencia?. Señale es sugestiva de un desprendimiento de vítreo
la respuesta correcta: posterior, aunque se debe realizar una
exploración de fondo de ojo para descartar
1. Todos sus hijos varones serán portadores y, una rotura retiniana o hemorragia vítrea.
por tanto, manifestarán la enfermedad. 4. Se debe instaurar un tratamiento hipotensor
2. Existe un fenómeno de anticipación genética, ocular, ya que ha sufrido un glaucoma agudo
por lo que su descendencia presentará que le provocaron los destellos y hoy están
síntomas más graves y más precoces. apareciendo las alteraciones en el campo
3. El diagnóstico genético preimplantacional visual secundarias a la lesión del nervio.
para la selección de sexo de los embriones
mediante hibridación in situ fluorescente 71. Paciente de 40 años, con exploración física
(FISH) es la mejor opción para tener normal, en el que se ha puesto en evidencia en la
descendencia sana. tomografía computarizada una masa
4. El 50% de sus hijas serán portadoras, pero no mediastínica y un nódulo hepático, y en la
presentarán manifestaciones clínicas de la analítica una elevación sérica de
enfermedad. lácticodeshidrogenasa (LDH), beta-
gonadotropina coriónica humana (beta-GCH) y
69. Varón miope de 83 años en tratamiento por alfafetoproteína. ¿Cuál es su sospecha
diabetes e HTA. Lo llevan a consulta sus diagnóstica?:
familiares porque desde hace unos días derrama
el vaso de agua en la mesa y se ha dado varios 1. Hepatocarcinoma metastásico.
golpes en la cabeza con los muebles de la cocina, 2. Seminoma extragonadal metastásico.
aunque él no refiere ningún déficit visual. La 3. Linfoma mediastínico.
presión intraocular es de 18 mmHg en ambos 4. Cáncer de células germinales no
ojos y la agudeza visual es de 0,9 en ambos ojos. seminomatoso extragonadal metastásico.
¿Cuál sería la respuesta correcta?:
72. En un paciente con cáncer de pulmón sin
1. Se debe instaurar tratamiento con análogos comorbilidad, la combinación de quimioterapia
de prostaglandinas en colirio, porque y radioterapia es el tratamiento de primera
presenta un glaucoma normotensional con elección en:
afectación del campo visual.
2. Puede tratarse de una ceguera cortical y se 1. Cáncer de célula no pequeña estadio I.
debe de realizar un campo visual para valorar 2. Nódulo pulmonar solitario.
una posible hemianopsia homónima con 3. Cáncer de célula no pequeña estadio II.
respeto macular. 4. Cáncer de célula pequeña.
3. Como no refiere problemas visuales, presenta
una presión intraocular normal y una agudeza
visual normal, se trata de despistes normales
a su edad.
4. Presenta un desprendimiento de retina
periférico secundario a su miopía que no ha
llegado a afectar al polo posterior.

- 11-
73. La radioterapia en el cáncer de mama NO está 77. Lactante de 6 meses que acude a urgencias por
indicada en: dificultad respiratoria. Exploración:
temperatura axilar 37,2°C, frecuencia
1. Carcinoma in situ post-tumorectomía. respiratoria 40 rpm, frecuencia cardíaca 160
2. Estadio I post-mastectomía. lpm, tensión arterial 90/45 mmHg, SatO2 95 %
3. Estadios localmente avanzados. con aire ambiente. Muestra dificultad
4. Enfermedad metastásica sintomática. respiratoria moderada con retracción
intercostal y subcostal. Auscultación pulmonar:
74. Hombre de 27 años, sano, que consulta por roncus espiratorios diseminados, espiración
presentar dolor intenso en oído derecho que alargada y ligera disminución en la entrada de
empeora con la presión sobre el trago, secreción aire en ambos campos pulmonares.
purulenta, sensación de taponamiento, prurito y Auscultación cardiaca: sin soplos. Se decide
cierto grado de hipoacusia de varias horas de mantener al paciente en observación en el
evolución. En la exploración otoscópica se hospital durante unas horas. ¿Qué actitud
comprueba la presencia de otorrea y signos considera más adecuada en este momento
inflamatorios importantes en el conducto respecto a las pruebas complementarias?:
auditivo externo. ¿Cuál es el diagnóstico más
probable?: 1. Solicitar gasometría venosa, recuento
leucocitario y reactantes de fase aguda.

FSE MEDICINA 2019/20


1. Otitis media seromucosa. 2. Solicitar radiografía de tórax.
2. Otitis externa difusa. 3. Solicitar gasometría arterial y reactantes de
3. Otitis externa maligna. fase aguda.
4. Otitis media aguda perforada. 4. No solicitar pruebas complementarias.

75. Señale el método de exploración más sensible 78. La causa más frecuente de hipotiroidismo
para establecer el diagnóstico de sinusitis, congénito permanente es:
estudiar la anatomía del complejo osteomeatal e
identificar posibles complicaciones por 1. La alteración en el desarrollo embrionario de
extensión local de la infección: la glándula tiroidea (disgenesias tiroideas).
2. Las dishormonogénesis (errores congénitos
1. Resonancia magnética con gadolinio. en la síntesis/secreción de las hormonas
2. Rinoscopia. tiroideas).
3. Tomografía computarizada coronal. 3. Los hipotiroidismos hipotálamo-hipofisarios.
4. Radiografía simple de senos paranasales. 4. Los síndromes de resistencia a las hormonas
tiroideas.
76. Niño de 5 años diagnosticado de insuficiencia
renal crónica. Acude a urgencias por vómitos, 79. ¿Cuál de las siguientes afirmaciones respecto a
malestar general y palpitaciones. Se realiza un la tiroiditis autoinmune o de Hashimoto es
electrocardiograma donde destaca elevación de INCORRECTA?:
ondas T y analítica sanguínea con niveles de
potasio de 5,9 mEq/L. Se decide la 1. Es una patología que puede aparecer en la
administración inmediata de gluconato cálcico. infancia-adolescencia.
¿Cuál es el objetivo de este tratamiento?: 2. Los pacientes con síndrome de Down
(trisomía 21) tienen mayor incidencia de
1. Favorecer el desplazamiento de potasio desde tiroiditis autoinmune que la población
el espacio plasmático al espacio intracelular. general.
2. Quelar el potasio circulante para favorecer su 3. La tiroiditis autoinmune evoluciona
eliminación hepática. habitualmente hacia un hipotiroidismo, pero
3. Quelar el potasio circulante para favorecer su al inicio de la enfermedad, los pacientes
eliminación renal. pueden presentar una fase transitoria de
4. Antagonizar la acción del potasio sobre la hipertiroidismo.
membrana de la célula miocárdica. 4. Los pacientes con síndrome de Turner
(cariotipo 45, X0) y síndrome de Klinefelter
(cariotipo 47, XXY) tienen una menor
incidencia de tiroiditis autoinmune que la
población general.

- 12-
80. Niño de 8 años que consulta por la aparición de 85. Un paciente de 79 años es traído a urgencias a
vello púbico en la base del pene desde hace 12 las 7 de la tarde desde una residencia, pero no
meses. El testículo no ha aumentado de volumen disponemos del informe de derivación y por
y el pene no ha aumentado de tamaño. No se tanto desconocemos sus antecedentes. Está
aprecia un aumento de la velocidad de desorientado en tiempo y espacio. Tiene
crecimiento en el último año. La edad ósea es un problemas para recordar información sencilla.
año mayor que la cronológica. ¿Cuál sería el En ocasiones se pone muy nervioso y agitado,
diagnóstico más probable?: porque ve personas amenazantes, incluso
cuando se encuentra a solas. En otras ocasiones
1. Pubertad precoz central. aparece somnoliento y desatento. El principal
2. Pubertad precoz periférica. dato que apoya el diagnóstico de delirium frente
3. Adrenarquia precoz. al de demencia es:
4. Tumor suprarrenal.
1. La presencia de alucinaciones visuales.
81. En relación con la enfermedad celiaca, señale la 2. Los síntomas cognitivos.
respuesta correcta: 3. La agitación psicomotriz.
4. La fluctuación del nivel de alerta.
1. Es necesaria la confirmación con biopsia
intestinal para un diagnóstico definitivo. 86. Indique cuál NO es una indicación de la terapia

FSE MEDICINA 2019/20


2. La genética (DQ2/DQ8) tiene un elevado electroconvulsiva:
valor predictivo negativo.
3. Las personas con síndrome de Prader-Willi 1. Trastorno depresivo mayor grave en el
tienen más riesgo de desarrollar una anciano.
enfermedad celiaca que la población general. 2. Depresión durante el embarazo.
4. Es frecuente adquirir tolerancia al gluten con 3. Agorafobia grave e incapacitante en personas
la edad. jóvenes.
4. Formas agudas de esquizofrenia en personas
82. En el tratamiento de una diarrea aguda en el jóvenes.
niño, sólo una de las medidas propuestas es
correcta, señálela: 87. Ante un trastorno obsesivo-compulsivo (TOC)
de inicio súbito en un niño de 9 años que además
1. Hay que recomendar una dieta astringente presenta tics y corea como manifestaciones
pobre en fibra hasta su resolución. neurológicas se debería descartar una infección
2. Se recomienda reducir de entrada el aporte de por:
lactosa los primeros días de la diarrea.
3. Se debe proponer rehidratación oral durante 1. Estreptococo viridans.
4-6 horas y realimentación posterior con 2. Estreptococo alfa-hemolítico.
dieta normal. 3. Estreptococo beta-hemolítico, grupo A.
4. La presencia de Salmonella en heces debe ser 4. Enterococo.
tratada con antibióticos para evitar su
diseminación. 88. ¿Cuál es el mecanismo de acción del
antidepresivo bupropion?:
83. En el síndrome de realimentación al inicio del
tratamiento de la malnutrición crónica, pueden 1. Inhibe la recaptación de noradrenalina y
aparecer todas las alteraciones analíticas serotonina.
siguientes, EXCEPTO una. Indique cuál: 2. Inhibe la recaptación de noradrenalina y
dopamina.
1. Hiperglucemia. 3. Es antagonista de la dopamina e inhibe la
2. Hiperfosfatemia. recaptación de serotonina.
3. Hipopotasemia. 4. Es antagonista del glutamato e inhibe la
4. Hipomagnesemia. recaptación de dopamina.

84. Un paciente con enfermedad de Alzheimer


moderada que presenta episodios de agitación y
agresividad grave junto a sintomatología
psicótica puede ser tratado con uno de los
siguientes fármacos, que está aprobado para
esta indicación. Indique cuál:

1. Escitalopram.
2. Lorazepam.
3. Risperidona.
4. Trazodona.
- 13-
89. Un hombre de 46 años con trastorno bipolar es 93. Un hombre de 58 años, hipertenso, acude a
llevado a urgencias tras una sobreingesta de urgencias por un síncope de 30 segundos
carbonato de litio. En la exploración destaca mientras corre, con recuperación espontánea y
temblor intenso, ataxia, disartria, mioclonías y sin secuelas. La tensión arterial es 135/65
fasciculaciones. La litemia es de 4,1 mEq/L mmHg. En la auscultación cardiaca presenta un
(toxicidad > 1,6 mEq/L). ¿Cuál de las siguientes soplo sistólico rudo, intenso, que se reduce con
opciones terapéutica estaría más indicada?: maniobras de Valsalva y un segundo tono
atenuado. El ECG muestra un ritmo sinusal a 72
1. Aminofilina asociado a un catártico. lpm, con criterios de hipertrofia ventricular
2. Carbón activado. izquierda y ondas T invertidas en cara anterior.
3. Hemodiálisis. Señale la afirmación correcta:
4. Diuresis forzada.
1. El cuadro clínico sugiere un
90. Entre las interacciones de los antipsicóticos con tromboembolismo pulmonar.
otros grupos farmacológicos señale cuál de las 2. Los datos aportados indican miocardiopatía
siguientes es FALSA: hipertrófica con obstrucción grave del tracto
de salida de ventrículo izquierdo.
1. Los fumadores suelen presentar niveles de 3. Estos datos corresponden a estenosis aórtica
antipsicóticos en plasma superiores a los no grave.

FSE MEDICINA 2019/20


fumadores. 4. Debe excluirse una disección de aorta
2. El litio puede empeorar los síntomas mediante una tomografía computarizada con
extrapiramidales y aumentar la contraste.
neurotoxicidad.
3. La carbamazepina disminuye los niveles 94. Entre las siguientes combinaciones de fármacos
plasmáticos de antipsicóticos. ¿cuál ha demostrado reducción de la mortalidad
4. Los antidepresivos aumentan los niveles en la insuficiencia cardiaca con fracción de
plasmáticos de antipsicóticos. eyección deprimida?:

91. Niña de 11 años y 6 meses, premenárquica, con 1. Betabloqueantes e inhibidores de la enzima


escoliosis idiopática conocida. La radiografía de convertidora de angiotensina.
raquis realizada hace 6 meses mostraba un 2. Digoxina y betabloqueantes.
curva tóraco-lumbar izquierda de 18° (ángulo 3. Diuréticos de asa y antagonistas de los
de Cobb). En la actual se observa un ángulo de canales del calcio.
Cobb de 28°, con una maduración de cresta 4. Inhibidores de la enzima convertidora de
iliaca (signo de Risser) de 2. En este momento angiotensina y antagonistas de los canales del
evolutivo, ¿qué tratamiento es el más adecuado calcio.
para intentar detener la progresión de la
curva?: 95. Una mujer de 83 años alérgica a betalactámicos
ingresa por pielonefritis aguda y vómitos. Toma
1. Ejercicios de natación. amiodarona desde hace 3 años por fibrilación
2. Ejercicios de reeducación postural. auricular paroxística. Al ingreso se pauta
3. Corsé ortopédico corrector. levofloxacino y metoclopramida. Por agitación
4. La intervención quirúrgica mediante nocturna se añade haloperidol. Al 6º día sufre
artrodesis espinal. un síncope del que se recupera a los pocos
segundos. El ECG muestra ritmo sinusal a 50
92. Mujer que presenta dificultad para caminar lpm, QRS estrecho y QTc 510 ms. En este
tras una cirugía ginecológica. Tiene dolor leve contexto clínico, ¿qué etiología explicaría el
en el muslo y le falla la pierna al apoyar. En la síncope?:
exploración presenta debilidad para la flexión
de la cadera y para la extensión de la rodilla, y 1. Una taquicardia ventricular polimorfa.
disestesias en la cara anterior del muslo. ¿Cuál 2. Un infarto agudo de miocardio.
es el diagnóstico de sospecha más probable?: 3. Un ictus cardioembólico.
4. Un bloqueo aurículoventricular completo.
1. Neuropatía del nervio femoral.
2. Meralgia parestésica del nervio fémoro-
cutáneo.
3. Neuropatía del nervio obturador.
4. Neuropatía del nervio ciático

- 14-
96. Ante un paciente con pericarditis aguda y 99. Paciente con cirrosis hepática estadio B de Child
derrame pericárdico grave ¿cuál de las con ascitis y antecedentes de peritonitis
siguientes medidas es INCORRECTA?: bacteriana espontánea (PBE). ¿Cuál de las
siguientes pautas de tratamiento antibiótico se
1. Ingreso para monitorización estrecha de la considera adecuada como profilaxis de la PBE?:
aparición de signos de taponamiento.
2. Administración de diuréticos que faciliten la 1. Vancomicina 500 mg / día.
desaparición del derrame. 2. Norfloxacino 400 mg / día.
3. Ingreso y consideración de 3. Metronidazol 250 mg / día.
pericardiocentesis. 4. Amoxicilina 1000 mg / día.
4. Administración de antiinflamatorios no
esteroideos. 100. Paciente joven con enfermedad inflamatoria
intestinal que es diagnosticado de espondilitis
97. Hombre de 42 años con antecedente de cirrosis anquilosante. ¿Cuál de las siguientes
hepática compensada, traído a urgencias por afirmaciones es FALSA?:
ictericia, fiebre, aumento del perímetro
abdominal y deterioro significativo del estado 1. La espondilitis anquilosante es más frecuente
general. ¿Cuál de los siguientes parámetros en pacientes con enfermedad de Crohn.
analíticos NO le aportaría información acerca 2. La actividad de la espondilitis anquilosante

FSE MEDICINA 2019/20


del pronóstico?: guarda relación con la actividad inflamatoria
de la enfermedad inflamatoria intestinal.
1. Albúmina sérica. 3. La actividad de la espondilitis anquilosante
2. Índice internacional normalizado (INR). no remite con glucocorticoides.
3. Bilirrubina total sérica. 4. La evolución de la espondilitis anquilosante
4. Transaminasas (valor de ALT y AST). evolución es lenta y progresiva.

98. Hombre de 55 años remitido a la consulta por 101. El tratamiento médico de los
detectarse en un examen de salud macroprolactinomas debería reducir la masa
hipertransaminasemia y esteatosis hepática en la tumoral, normalizar los niveles séricos de
ecografía. Es hipotiroideo en tratamiento prolactina y restaurar la función gonadal. Esto
sustitutivo y diabético bien controlado con se puede conseguir con:
metformina. Bebe alcohol muy ocasionalmente.
En la exploración física destaca una obesidad 1. Inhibidores selectivos de la recaptación de
tipo I (índice de masa corporal, IMC 33 kg/m2). serotonina.
La analítica muestra: aspartato 2. Agonistas selectivos de los receptores D2
aminotransferasa (AST) 87 UI/L, alanino dopaminérgicos.
aminotransferasa (ALT) 65 UI/L, 3. Análogos de la somatostatina.
gammaglutamiltransferasa (GGT) 100 UI/L, 4. Antagonistas de los receptores V2 de la
colesterol total 230 mg/dL, glucemia basal 132 desmopresina.
mg/dl. Bilirrubina, fosfatasa alcalina, hormonas
tiroideas y ferritina normales. Coagulación 102. La hipoglucemia postabsortiva por tumores no
normal. La serología de hepatitis vírica (VHA, insulinomas se caracteriza:
VHB y VHC) y de autoinmunidad (ANA, AMA,
AML, antiLKM) son negativas. Señale la 1. Por ser habitual en tumores abdominales de
afirmación más apropiada: pequeño tamaño y crecimiento rápido.
2. Porque los niveles plasmáticos de
1. Tiene una esteatosis hepática sin riesgo de proinsulina, insulina y péptido C están
desarrollar cirrosis ni hepatocarcinoma, por suprimidos durante la hipoglucemia.
lo que es suficiente el control metabólico 3. Porque los niveles de IGFI están
periódico. característicamente elevados y la ratio
2. Está indicado realizar una biopsia hepática IGFII/IGFI es anormalmente baja.
por la sospecha de hepatitis autoinmune. 4. Porque la hipoglucemia tumoral es una de las
3. Es útil realizar una elastografía de transición complicaciones más frecuentes en neoplasias
(FibroScan ®) para determinar la existencia o hematológicas en ausencia de caquexia.
no de fibrosis.
4. Está indicado sustituir la metformina por
pioglitazona que ha demostrado ser más
eficaz para mejorar la esteatosis hepática.

- 15-
103. La administración oral de urea produce 107. Paciente de 18 años que acude a urgencias con
aumento de diuresis y disminución de la epistaxis de varios días de evolución, sin
natriuresis y se constituye en un tratamiento antecedentes personales ni familiares de interés.
seguro y eficaz de la: En la exploración está afebril, se observan
equimosis múltiples, no se palpa esplenomegalia.
1. Hiponatremia hipotónica en situación de Analítica: Leucocitos 7,2 x10³/µL, Hb 12,3 g/dL,
euvolemia. Plaquetas 6,0 x10³/µL. La trombocitopenia se
2. Hiponatremia hipotónica en situación de confirma en el frotis, donde se observan
hipovolemia. plaquetas de tamaño aumentado. Estudio de
3. Hiponatremia isotónica asociada a coagulación y bioquímica normales. ¿Cuál
hiperproteinemia. considera el diagnóstico más probable?:
4. Hiponatremia hipertónica.
1. Púrpura trombótica trombocitopénica.
104. La diabetes insípida nefrogénica inducida por 2. Coagulación intravascular diseminada.
litio: 3. Trombocitopenia inducida por infección.
4. Trombocitopenia inmune primaria.
1. Está causada por mutaciones de acuaporina-
2. 108. En relación con el linfoma folicular, señale la
2. Es debida a una reducción marcada de los INCORRECTA:

FSE MEDICINA 2019/20


niveles de acuaporina-2.
3. Responde a dosis bajas de desmopresina. 1. Es el segundo linfoma más frecuente en
4. Los diuréticos tiazídicos y el amiloride están Occidente.
contraindicados en su tratamiento. 2. Las células neoplásicas derivan de un
linfocito de la médula ósea.
105. Paciente de 67 años que en los últimos 6 meses, 3. Puede transformarse en un linfoma B difuso
en dos analíticas de rutina, presenta linfocitosis de célula grande.
progresiva. En la última, hemoglobina 15,4 4. Puede permanecer años sin necesitar
g/dl; leucocitos 18,5 x10³/µL con 82 % de tratamiento.
linfocitos maduros que por citometría de flujo
expresan los antígenos CD5/CD19/CD23 y 109. Sobre la Listeria monocytogenes, señale la
plaquetas 240 x10³/µL. ¿Qué actitud le parece afirmación INCORRECTA:
correcta?:
1. Es un bacilo grampositivo, catalasa positivo.
1. Estudio de mutaciones de TP53 para 2. Está ampliamente distribuida en el suelo,
establecer el pronóstico. aguas residuales y vegetación en
2. Aspirado/biopsia ósea para confirmar el descomposición.
diagnóstico. 3. El hombre y los animales pueden ser
3. PET/CT para establecer la actitud terapéutica. portadores asintomáticos.
4. Nuevo control clínico y analítico en 6 meses. 4. El tratamiento de elección de la meningitis
por L. monocytogenes es una cefalosporina.
106. Mujer de 65 años derivada a urgencias por
fiebre y alteraciones en la analítica: 110. Hombre de 54 años que consulta por clínica
Hemoglobina 11,4 g/dL, leucocitos 0,86 x10³/µL, miccional compatible con patología prostática.
(neutrófilos 41,9 %, linfocitos 55,8 %), I-PSS=22+4, volumen de orina residual 120 cc,
plaquetas 48,0 x10³/µL, fibrinógeno 118 mg/dL, flujo máximo 7,2 ml/s, PSA=0,91 ng/ml y
dímero D 20,2 µg/mL. Se realiza examen de volumen prostático en ecografía 31 cc. Su
médula ósea con la que se le diagnostica una primera recomendación terapéutica será:
leucemia aguda con t(15;17) en el 60 % de las
células. ¿Cuál de las siguientes respuestas es 1. Alfa-bloqueante.
correcta?: 2. Inhibidor de 5-alfa reductasa.
3. Anti-muscarínico.
1. Si está asintomática se iniciará ácido trans 4. Resección transuretral de próstata.
retinoico (ATRA) y se recomendarán
controles en hospital de día.
2. Se iniciará tratamiento con trióxido de
arsénico, ATRA y terapia de soporte.
3. Es una leucemia mieloblástica tipo M3, por
lo que se iniciara tratamiento quimioterápico
y heparina para controlar la coagulación
intravascular diseminada.
4. Se debe iniciar tratamiento antibiótico.
Cuando desaparezca la fiebre se iniciará el
tratamiento de la leucemia.
- 16-
111. Mujer de 37 años sin otros antecedentes de 115. Mujer de 27 años con tos no productiva, fiebre y
interés que una apendicectomía. Acude a dolor pleurítico de un mes de evolución. En la
urgencias por dolor en fosa renal derecha de 3 radiografía de tórax se objetiva un derrame
días de evolución y fiebre de 39°C. En la pleural izquierdo. El análisis del líquido pleural
analítica destaca leucocitosis (14 x10³/µL con muestra un exudado linfocítico con pH 7,32,
neutrofilia) y proteína C reactiva 15 mg/dL. El glucosa 66 mg/dL y adenosin-deaminasa 59 U/L.
sistemático de orina muestra hematuria ¿Cuál es la etiología más probable?:
microscópica, leucocituria y piuria
significativas. Ingresa con el diagnóstico de 1. Insuficiencia cardiaca.
pielonefritis aguda y se inicia tratamiento 2. Síndrome de Meigs.
antibiótico con ceftriaxona intravenosa. Tres 3. Quilotórax.
días después persiste fiebre de 38,5°C, aumento 4. Tuberculosis pleural.
de la leucocitosis y deterioro de la función renal.
Teniendo en cuenta la sospecha diagnóstica 116. En un sujeto mayor de 65 años una prueba de
¿qué prueba solicitaría a continuación?: tuberculina ha mostrado una induración de 3
mm. La induración en una segunda prueba,
1. Tomografía axial computarizada con realizada 10 días después, es de 13 mm. Señale
contraste intravenoso. la respuesta correcta:
2. Urografía intravenosa.

FSE MEDICINA 2019/20


3. Resonancia magnética nuclear abdominal. 1. La primera reacción es un falso positivo.
4. Ecografía urológica. 2. La segunda reacción es un verdadero
positivo.
112. Hombre de 59 años con nefropatía diabética, 3. La primera reacción es un verdadero
con filtrado glomerular estimado de 36 negativo.
ml/min/1,73 m2 e índice 4. La segunda reacción es un falso positivo.
albuminuria/creatininuria de 350 mg/g. Según la
clasificación de enfermedad renal crónica 117. ¿Cuál de las siguientes alternativas al lavado
KDIGO 2012 corresponde a la categoría: pulmonar total NO es apropiada en el
tratamiento de la proteinosis alveolar de origen
1. Estadio G3b A2. autoinmune?:
2. Estadio G4 A3.
3. Estadio G3b A3. 1. Factor estimulante de colonias de
4. Estadio G4 A2. granulocitos-macrófagos (GM-CSF).
2. Corticosteroides.
113. En relación con la nefropatía membranosa 3. Plasmaféresis.
señale la respuesta FALSA: 4. Rituximab.

1. La mayoría de las nefropatías membranosas 118. Hombre de 72 años con antecedentes de EPOC
son de causa idiopática. con obstrucción grave al flujo aéreo, con dos
2. Lo que mejor predice el pronóstico es la exacerbaciones graves en el último año, que
presencia de afectación intersticial y no el acude a urgencias con taquipnea y uso de
daño glomerular (estadios I a IV). musculatura accesoria. Está alerta. En la
3. Los anticuerpos frente al receptor de la gasometría arterial basal se objetiva: pH 7,29,
fosfolipasa A2 (Anti-PLA2R) suelen ser PaCO2 68 mmHg, PaO2 51 mmHg. Se inicia
positivos en las formas idiopáticas. tratamiento broncodilatador, esteroideo,
4. El 90% de los pacientes alcanzan la remisión antibioterapia empírica y se ajusta la
con corticoides únicamente. oxigenoterapia para conseguir una SatO2 entre
88 y 92 %. Tras una hora repite la gasometría
114. ¿En cuál de las siguientes situaciones puede arterial con gafas nasales a 2 lpm y presenta pH
aparecer una acidosis metabólica sin anion gap 7,28, PaCO2 70 mmHg, PaO2 62 mmHg. Señale
elevado?: la actuación correcta:

1. Insuficiencia renal. 1. Iniciar ventilación mecánica no invasiva.


2. Cetoacidosis diabética. 2. Proceder a la intubación orotraqueal e iniciar
3. Diarrea. ventilación mecánica.
4. Ayuno prolongado. 3. Incrementar la oxigenoterapia hasta obtener
una SatO2 de al menos 99 %.
4. Combinar benzodiacepinas con altos flujos
de oxígeno para asegurar una disminución de
la taquipnea y mejorar el patrón ventilatorio.

- 17-
119. Mujer de 16 años que inicia seguimiento en la 122. Un hombre de 70 años consulta por dolor
consulta de Neurología de adultos, remitida lumbar no irradiado de 1 semana de duración,
desde Neuropediatría con el diagnóstico de de intensidad progresiva, acompañado de
epilepsia mioclónica juvenil (EMJ). Está en marcada limitación de la movilidad del raquis y
tratamiento con ácido valproico y bien febrícula. La radiografía de columna lumbar no
controlada. ¿Se plantearía hacer alguna muestra alteraciones. ¿Qué diagnóstico
modificación en su tratamiento?: deberíamos descartar en primer lugar y con qué
técnica?:
1. No, porque el ácido valproico es el
tratamiento de elección en los pacientes con 1. Aplastamiento vertebral con tomografía
este síndrome epiléptico. computarizada (TC).
2. No, porque la paciente está libre de crisis y 2. Metástasis óseas con gammagrafía ósea con
cambiarlo en este momento sería asumir tecnecio.
riesgos innecesarios. 3. Hiperostosis anquilosante vertebral con
3. Sí, porque debe evitarse su uso en la mujer en radiografía de columna dorsolumbar.
edad fértil, debido a que es el fármaco 4. Espondilodiscitis piógena con resonancia
antiepiléptico con mayor teratogenicidad y magnética (RM).
problemas en el neurodesarrollo de los hijos.
4. Sí, por la relación de este fármaco con el 123. Mujer de 60 años que consulta por

FSE MEDICINA 2019/20


síndrome del ovario poliquístico. adormecimiento de la mano derecha de meses de
evolución, que la despierta por la noche y la
120. Hombre de 79 años con antecedentes de obliga a movilizar la mano hasta que se le pasa.
deterioro cognitivo leve, que presenta un cuadro Parece que ha perdido fuerza y a veces se le han
brusco de hemianopsia homónima derecha, caído cosas de esa mano. Señale la respuesta
afasia y paresia facial inferior derecha. A su cierta:
llegada a urgencias se objetiva tensión arterial:
190/85 mmHg. Se realiza una TC craneal que 1. Si la exploración neurológica muestra
evidencia un hematoma intraparenquimatoso adormecimiento de los dedos primero a
lobar frontal izquierdo y otro hematoma tercero pensaremos en afectación del nervio
occipital izquierdo. ¿Cuál es la etiología más cubital a nivel epitroclear.
probable de estos hematomas?: 2. Si la exploración neurológica muestra
adormecimiento de los dedos primero a
1. Hipertensiva. tercero pensaremos en afectación del nervio
2. Angiopatía amiloide. mediano a nivel del canal del carpo.
3. Lesión tumoral subyacente. 3. Para realizar el diagnóstico no es preciso
4. Malformación arteriovenosa. realizar una electromiografía, dado que la
exploración física presenta muy buenos
121. Mujer de 60 años en estudio por varios episodios coeficientes de probabilidad.
de pérdida de conciencia. Se plantea el 4. El tratamiento de la afectación del nervio
diagnóstico diferencial entre crisis epilépticas y cubital a nivel epitroclear o del nervio
síncopes. Señale cuál de los siguientes datos NO mediano a nivel carpiano es la intervención
apoyaría el diagnóstico de síncope: quirúrgica, dado que el tratamiento
conservador no mejora la sintomatología.
1. Incontinencia de esfínteres durante el
episodio. 124. Respecto a la arteritis de la temporal, señale la
2. Cianosis facial durante el episodio. respuesta INCORRECTA:
3. Traumatismo craneal durante el episodio.
4. Sacudidas de las cuatro extremidades de 1-2 1. Se presenta fundamentalmente en personas
segundos de duración, durante el episodio. de raza blanca y mayores de 50 años.
2. Aproximadamente una cuarta parte de los
pacientes desarrollan aneurisma o dilatación
de la aorta torácica.
3. La pérdida de visión unilateral o bilateral es
una de las complicaciones más graves.
4. La máxima incidencia se da entre la sexta y
la séptima década de la vida.

- 18-
125. ¿Cuál de las siguientes afirmaciones es cierta en 128. Hombre de 35 años, ingresado por traumatismo
relación con el síndrome de Leriche?: torácico grave con múltiples fracturas costales.
Tras responder favorablemente al tratamiento
1. Es un cuadro clínico que comprende la triada con analgésicos y oxígeno, comienza a presentar
claudicación plantar, impotencia coeundi y hipoxemia grave. Señale cuál es la causa más
ausencia de pulsos distales. probable de este deterioro:
2. Es consecuencia de una oclusión fémoro-
poplítea bilateral. 1. La inestabilidad de la pared torácica por las
3. Los pulsos femorales están ausentes. fracturas múltiples.
4. La arteriografía es la primera prueba 2. La infección respiratoria por aspiración.
diagnóstica a realizar. 3. La alteración del intercambio gaseoso por la
contusión pulmonar.
126. Hombre de 73 años con antecedente de infarto 4. La hipovolemia postraumática.
agudo de miocardio tratado con angioplastia
coronaria transluminal percutánea hace 6 129. Hombre de 65 años que consulta por disfagia a
semanas. Acude a urgencias por dolor brusco, sólidos desde hace dos meses. La esofagoscopia
frialdad y pérdida de motilidad de la evidencia tumoración a 30 cm de arcada dental,
extremidad inferior derecha. En la exploración parcialmente estenosante, con anatomía
está presente el pulso femoral derecho, con patológica de carcinoma epidermoide. Se solicita

FSE MEDICINA 2019/20


ausencia del resto de los pulsos de esa ecografía endoscópica y PET-TC donde no se
extremidad. Conserva pulsos a todos los niveles observan adenopatías patológicas. ¿Cuál de las
en la extremidad contralateral. Respecto al siguientes opciones sería la más correcta?:
diagnóstico de sospecha, señale la afirmación
FALSA: 1. Esofaguectomía transhiatal.
2. Esofaguectomía según la técnica de Ivor-
1. La causa más probable es de origen Lewis.
embolígeno. 3. Quimioterapia más radioterapia
2. La parálisis es un signo de aparición tardía preoperatoria.
que indica que el tratamiento 4. Quimioterapia neoadyuvante.
revascularizador debe realizarse lo antes
posible. 130. ¿Cuál es el momento más apropiado para la
3. El tratamiento consiste en embolectomía administración de la profilaxis antibiótica en la
arterial femoral con sonda de Fogarty. cirugía abdominal?:
4. En caso de parálisis rígida con anestesia
profunda y ausencia de señal Doppler está 1. Desde 48 horas antes de la intervención
indicada una arteriografía urgente. quirúrgica.
2. En los 30 minutos previos al inicio de la
127. Hombre de 60 años, fumador de 40 intervención quirúrgica.
paquetes/año, que consulta por dolor en borde 3. En los 30 minutos siguientes al cierre de la
cubital del brazo izquierdo y ptosis palpebral incisión quirúrgica.
izquierda desde hace un mes. La radiografía de 4. Únicamente debe aplicarse si se observa
tórax muestra una masa en el lóbulo superior contaminación del campo quirúrgico durante
izquierdo y la TC confirma la lesión con la intervención.
invasión de la segunda costilla. En relación con
el diagnóstico de sospecha, el tratamiento más 131. Con respecto al síndrome de Lynch o carcinoma
adecuado es: colorrectal no polipósico hereditario, señale la
respuesta correcta:
1. Quimioterapia neoadyuvante con
quimiorradioterapia combinada, seguida de 1. Es un trastorno autosómico recesivo.
cirugía. 2. El fenotipo incluye predominio de tumores
2. Quimioterapia neoadyuvante sin cirugía, malignos del colon en el lado derecho.
seguida de radioterapia. 3. La progresión pólipo-cáncer es más lenta que
3. Cirugía seguida de quimiorradioterapia en el cáncer de colon esporádico.
combinada. 4. Los tumores malignos extracolónicos suelen
4. Radioterapia sin quimioterapia, seguida de asentar en pulmón o hígado.
cirugía.

- 19-
132. Mujer de 67 años, obesa, que acude a urgencias 135. Hombre de 64 años que consulta por presentar
por dolor abdominal en cuadrante superior en los últimos 3 días debilidad creciente de
derecho del abdomen, fiebre de 39,5°C y orinas miembros inferiores y parestesias ascendentes.
muy oscuras. Ante este cuadro se debería La exploración muestra debilidad de miembros
sospechar en primer lugar: inferiores y arreflexia global. La analítica
sanguínea y la radiografía de tórax son
1. Colecistitis aguda alitiásica. normales. La punción lumbar revela un LCR
2. Coledocolitiasis. con 3 células mononucleares, 60 mg de
3. Colecistitis aguda litiásica. glucorraquia y 97 mg de proteínas totales.
4. Colangitis ascendente. Señale la afirmación INCORRECTA:

133. Paciente de 47 años, sin antecedentes patológicos 1. El cuadro es compatible con una
de interés. Presenta un cuadro de fiebre y dolor polirradiculoneuropatía autoinmune.
en punta de costado derecho y ha sido 2. En estos cuadros puede aparecer parálisis
diagnosticado de neumonía adquirida en la facial hasta en el 50 % de los casos.
comunidad en su Centro de Salud. En una 3. En casos graves estos cuadros pueden
radiografía simple de control, realizada 3 días asociarse a neuropatía autonómica.
después del inicio del tratamiento antibiótico, se 4. El tratamiento de elección de estos cuadros es
objetiva un derrame pleural ipsilateral que la administración de corticosteroides

FSE MEDICINA 2019/20


ocupa un tercio del hemitórax derecho. Señale la intravenosos.
respuesta correcta:
136. Paciente de 48 años que ha sufrido una
1. Se puede establecer el diagnóstico de hemorragia subaracnoidea por aneurisma de
empiema pleural si se realiza una comunicante anterior. A los 10 días del
toracocentesis y el pH del líquido pleural es sangrado presenta alteración del nivel de
de 7,4. conciencia y desarrolla una hemiparesia. Señale
2. El diagnóstico de empiema pleural solo la respuesta INCORRECTA:
queda confirmado si se halla un
microorganismo en el cultivo de líquido 1. El vasoespasmo ocurre con mayor frecuencia
pleural. entre los días 4 y 14 tras la hemorragia
3. Debe sospecharse un empiema pleural si el subaracnoidea.
paciente presenta persistencia de fiebre y 2. Cuanto mayor sea la cuantía del sangrado
leucocitosis. inicial mayor el riesgo de resangrado.
4. La presencia de un derrame pleural es 3. Se aconseja la restricción hídrica en estos
suficiente para establecer el diagnóstico de pacientes por el riesgo de hiponatremia.
empiema pleural y colocar un drenaje pleural 4. Es necesario hacer una TC craneal para poder
derecho. descartar otras complicaciones.

134. Varón de 35 años, trasladado a urgencias tras


un accidente de tráfico. A su ingreso se observa
una puntuación en la escala de Glasgow de 15,
tensión arterial de 140/90 mmHg, frecuencia
respiratoria de 35 rpm y frecuencia cardiaca de
110 lpm, con una saturación de oxígeno basal
del 91 %. En la exploración hay hipofonesis
torácica derecha y timpanismo a la percusión.
¿Cuál es el diagnóstico de presunción? :

1. Taponamiento cardíaco traumático.


2. Neumotórax a tensión derecho.
3. Hemotórax masivo derecho.
4. Contusión pulmonar.

- 20-
137. Mujer de 93 años con antecedentes de EPOC, 139. Niño de 7 años traído a urgencias tras caerse de
HTA, dislipemia, fibrilación auricular y un columpio sobre la mano derecha. No tiene
cardiopatía isquémica. Vive sola. Presenta antecedentes de interés. Presenta deformidad en
fracción de eyección ventricular 53 %, dorso de tenedor de la muñeca e impotencia
creatinina 1,5 mg/dL, coagulación normal. funcional, con situación neurovascular distal
Recibe tratamiento con broncodilatadores normal. ¿Qué lesión espera encontrar en la
inhalados y AAS 100 mg al día. Sufre una caída radiografía urgente que solicita?:
en la calle, tras la cual le resulta imposible la
bipedestación y sedestación, con dolor intenso en 1. Fractura-luxación de Monteggia.
la cadera derecha a las movilizaciones, gran 2. Fractura de la cabeza radial.
acortamiento y rotación externa de la pierna. 3. Epifisiolisis distal del radio.
¿Cuándo y cómo debe tratarse?: 4. Fractura en tallo verde de cúbito.

1. Ingreso para ecografía transtorácica, 140. Mujer de 65 años, obesa, que sufre una caída
valoración por Cardiología y después sobre la mano con el codo en extensión. Presenta
reducción cerrada y fijación con tornillo- dolor en el brazo con tumefacción e impotencia
placa. funcional del mismo y con imposibilidad para
2. Ingreso para valoración por Nefrología y realizar la extensión de la muñeca y dedos. Lo
Cardiología, y después decidirá más probable es que presente:

FSE MEDICINA 2019/20


Traumatología si usa una artroplastia o un
clavo trocantérico. 1. Fractura luxación de húmero proximal.
3. Ingreso en Medicina Interna/Geriatría, 2. Luxación de codo.
estabilización progresiva de las 3. Fractura diafisaria de húmero asociada a
comorbilidades, tratamiento ortopédico fractura doble de antebrazo.
conservador. 4. Fractura diafisaria de húmero con lesión del
4. Reducción cerrada y fijación de la fractura nervio radial.
antes de 48 horas, sedestación muy precoz,
manejo ortogeriátrico. 141. El diagnóstico más probable de un paciente de
74 años que desde hace dos meses comienza con
138. Mujer de 73 años con antecedentes de obesidad, dolor lumbar irradiado a miembros inferiores,
diabetes mellitus tipo 2, hipertensión arterial y claudicación neurógena y limitación a la
dislipemia. Consulta por dolor insoportable en extensión del tronco es:
la rodilla derecha de 5 días de evolución, sin
trauma previo. Exploración: rodilla globulosa, 1. Hernia discal L4-L5.
varo moderado, extensión y flexión limitadas 2. Fractura vertebral lumbar.
por dolor, dolor difuso medial. En la radiografía 3. Inestabilidad vertebral L5-S1.
se aprecian osteofitos y pinzamiento leve de la 4. Estenosis de canal lumbar.
interlínea medial. ¿Cuál sería su manejo
inicial?:

1. Explicación del diagnóstico, reposo relativo,


paracetamol 1 g/8 h más metamizol 500 mg
/8 h, naproxeno de rescate.
2. Derivación preferente a consultas externas de
Traumatología para valoración de prótesis
total cementada.
3. Derivación preferente a consultas externas de
Traumatología para valorar desbridamiento
artroscópico.
4. Solicitud de resonancia magnética preferente
para evaluación de meniscopatía, quiste de
Baker y/o tendinitis.

- 21-
142. Mujer de 70 años, diabética e hipertensa que 145. Con respecto a los síndromes geriátricos, una de
sufre una caída en su domicilio, presentando las siguientes respuestas es INCORRECTA:
una herida de 9 cm que comunica con un foco de
fractura de la tibia derecha. Radiográficamente 1. Son situaciones de enfermedad con una alta
se observa una fractura oblicua corta de tercio incidencia y prevalencia en la población de
medio-distal de tibia. Se opera de urgencias edad avanzada.
mediante limpieza (Friedrich) y colocación de 2. Suelen ser el resultado de varias etiologías
un clavo endomedular acerrojado. A los 11 confluyentes, que requieren una valoración e
meses presenta una pseudoartrosis atrófica de intervención multidimensional.
tibia con supuración en la zona de la herida. 3. Suelen producir consecuencias importantes,
¿Cuál será su mejor opción terapéutica tanto médicas como funcionales, que
inmediata?: requieren una intervención multidimensional.
4. Su prevalencia es más elevada en los
1. Triple antibioterapia (grampositivos, pacientes mayores hospitalizados que en los
gramnegativos y anaerobios) y limpieza de la pacientes mayores dependientes e
herida quirúrgica, retirando los cerrojos institucionalizados.
distales para favorecer la consolidación ósea.
2. Actitud expectante y tratamiento antibiótico 146. La fragilidad es un estado que puede originar
con quinolonas. importantes consecuencias negativas. De las

FSE MEDICINA 2019/20


3. Aporte de injerto autólogo y de factores de siguientes respuestas, señale la INCORRECTA:
crecimiento (BMP 2 y 7) para estimular el
proceso de consolidación ósea, que está 1. Se asocia a un buen número de
retardado. consecuencias, como caídas, inmovilidad,
4. Retirada del clavo, desbridamiento, discapacidad, mayor tasa de ingresos
colocación de fijador externo y antibioterapia hospitalarios y de institucionalización, y
ajustada a los resultados de los cultivos. mayor mortalidad.
2. Su detección precoz permitiría una
143. ¿En qué tipo de demencia hay que pensar en un intervención temprana para evitar sus
paciente que presenta deterioro cognitivo, consecuencias negativas
alucinaciones visuales, signos parkinsonianos y 3. Su prevalencia no tiene relación directa con
mala tolerancia a neurolépticos?: la edad cronológica.
4. La velocidad de la marcha y el test Timed
1. Enfermedad de Alzheimer atípica. Up&Go, permiten su detección en la práctica
2. Demencia por cuerpos de Lewy difusos. clínica habitual.
3. Degeneración lobular frontotemporal.
4. Demencia por priones. 147. La incontinencia urinaria constituye uno de los
principales síndromes geriátricos. ¿Cuál de las
144. Varón de 92 años, sin antecedentes patológicos siguientes afirmaciones es INCORRECTA?:
de interés, diagnosticado hace 24 horas de
pielonefritis aguda, que presenta alucinaciones 1. Su prevalencia es más elevada en pacientes
visuales, discurso incoherente e inquietud con patología neurológica crónica y/o
psicomotriz. ¿Cuál de las siguientes síndrome de inmovilidad que en los pacientes
afirmaciones se ajusta más al cuadro clínico que hospitalizados por una enfermedad aguda.
está presentando ahora?: 2. El tipo clínico más frecuente de incontinencia
urinaria, en la población mayor en general, es
1. Se trata del debut típico de una demencia en la incontinencia de urgencia.
un paciente mayor con una infección grave. 3. El tratamiento farmacológico ha demostrado
2. Se trata de un cuadro de delirium en un una mayor efectividad clínica en la
paciente mayor con una infección grave. incontinencia de urgencia / vejiga hiperactiva
3. Se trata del debut de un episodio psicótico que en otros tipos de incontinencia.
agudo (esquizofrenia tardía del anciano). 4. El tratamiento de la incontinencia de
4. Se trata de un episodio de depresión grave urgencia con antimuscarínicos está
por la hospitalización con sintomatología recomendado en pacientes con deterioro
psicótica asociada. cognitivo o demencia en fase grave.

148. ¿Cuál de las siguientes NO es una etapa del


modelo de cambio conductual de Prochaska y
DiClemente?:

1. Precontemplación.
2. Postcontemplación.
3. Acción.
4. Mantenimiento.
- 22-
149. Mujer de 65 años que hace 3 meses presentó un 153. Varón de 65 años que consulta por un episodio
cuadro de erupción cutánea vesicante sobre el de hematuria franca indolora dos días antes, sin
dermatoma D6 derecho, acompañado de un otra sintomatología acompañante. La historia
dolor muy intenso. Tras la formación de costras, clínica no indica sospecha de un determinado
el cuadro se resolvió, pero persiste dolor origen y la tira de orina de ese día revela la
moderado que se intensifica al roce y le impide existencia de hematuria, aunque la orina es
el uso de ropa interior sobre el dermatoma clara. ¿Qué prueba solicitaría en primer lugar
afectado. En esta situación ¿qué tratamiento NO para orientar el diagnóstico?:
sería recomendable?:
1. Urografía intravenosa.
1. Gabapentina oral. 2. Radiografía simple de abdomen.
2. Ibuprofeno oral. 3. Ecografía renal y de vías urinarias.
3. Capsaicina tópica. 4. Tomografía computarizada de abdomen.
4. Lidocaína tópica.
154. ¿Cuál de los siguientes enunciados se
150. La Medicina Basada en la Evidencia corresponde con el término prevención
recomienda la estructura PICO para el cuaternaria?:
planteamiento de preguntas clínicas. ¿Cuál de
los siguientes NO es un elemento incluido en la 1. Actividades dirigidas a evitar la aparición o

FSE MEDICINA 2019/20


formulación de la pregunta PICO?: disminuir la prevalencia de factores de riesgo
en individuos sanos.
1. Tipo de intervención a valorar. 2. Intervenciones encaminadas a evitar o reducir
2. Tipo de paciente. los riesgos del contacto con los servicios
3. Tipo de estudio. sanitarios.
4. Resultado a medir. 3. Actividades de rehabilitación y reinserción
social en pacientes con enfermedades
151. ¿Cuál de las siguientes estrategias constituye clínicamente manifiestas.
una actuación de prevención primaria frente al 4. Actividades de cribado y de vacunación en
maltrato contra las mujeres?: personal sanitario con riesgo de
enfermedades infecciosas.
1. Establecer un plan de protección y huida en
mujeres víctimas de maltrato con hijos 155. ¿Según el fenotipo de fragilidad física de Fried
pequeños a su cargo. en el que se valoran cinco criterios (pérdida de
2. Actividades educativas con mujeres peso, poca energía, poca fuerza, lentitud y baja
embarazadas y sus parejas sobre relaciones actividad física), a partir de cuántos criterios se
de igualdad y corresponsabilidad en la considera que una persona mayor es frágil?:
crianza.
3. Realizar un protocolo de cribado rutinario en 1. Dos.
las mujeres que acuden a los servicios de 2. Tres.
urgencia hospitalarios por crisis de ansiedad. 3. Cuatro.
4. Notificación precoz a la autoridad judicial 4. Cinco.
ante cualquier sospecha de maltrato.
156. Paciente con insuficiencia cardiaca crónica en
152. Señale cuál de los siguientes grupos terapéuticos quien detectamos unas ondas "v" prominentes
ha demostrado disminuir la morbimortalidad en el pulso venoso yugular y en la auscultación
cardiovascular y total en pacientes hipertensos, cardiaca un soplo holosistólico en el área del
mayores de 55 años y con antecedentes de apéndice xifoides que se acentúa con la
episodios cardiovasculares o diabetes: inspiración profunda. ¿Cuál es la valvulopatía
que sugiere esta exploración física?:
1. Bloqueantes de los canales del calcio
(calcioantagonistas). 1. Insuficiencia mitral.
2. Diuréticos. 2. Insuficiencia pulmonar.
3. Inhibidores del enzima convertidor de 3. Insuficiencia tricuspídea.
angiotensina. 4. Estenosis aórtica.
4. Betabloqueantes.

- 23-
157. Mujer de 26 años que consulta por sensación de 159. Un paciente de 45 años con antecedentes de
debilidad generalizada que se ha instaurado infección por VIH conocida hace 6 años consulta
progresivamente en el curso de tres semanas, por deseo de reiniciar el tratamiento
haciéndose especialmente intensa en los últimos antirretroviral. Lo abandonó hace dos años por
dos días. Refiere desde hace un par de años motivos personales y en parte por efectos
episodios de dolor articular en las manos que adversos, cuando presentaba carga viral
han precisado de la toma de antiinflamatorios, indetectable y 230 linfocitos CD4; en los últimos
así como la aparición de alguna lesión meses ha sufrido un herpes zoster. Señale la
eritematosa de causa no aclarada en la zona del respuesta correcta respecto al reinicio del
escote, principalmente en verano. En la tratamiento:
exploración física destaca únicamente una
palidez cutáneo-mucosa evidente y una 1. Si no es posible asegurar una adecuada
frecuencia cardiaca de 100 lpm. En el adherencia al tratamiento, lo mejor es que no
hemograma destaca: Hb 6 gr/dL, Hto 27 %, lo reinicie por el riesgo de desarrollo de
VCM 105 fL, 3.420 leucocitos/mm3 (2300 resistencias.
neutrófilos/mm3, 800 linfocitos/mm3, 250 2. Estaría indicado en este momento un estudio
monocitos/mm3, 50 eosinófilos/mm3, 20 de resistencias para conocer las mutaciones
basófilos/mm3), plaquetas 170.000/mm3. En la presentes y guiar el reinicio del tratamiento.
bioquímica: AST 30 UI/L, ALT 35 UI/L, GGT 3. La biterapia con dolutegravir y lamivudina

FSE MEDICINA 2019/20


59 UI/L, fosfatasa alcalina 105 UI/L, LDH 490 podría ser igual de eficaz en esta situación
UI/L, urea 20 mg/dL, creatinina 0,8 mg/dL. que la triple terapia, ya que podría reducir
Teniendo en cuenta la información disponible, parte de los efectos adversos.
indique cuál de los siguientes parámetros 4. La mejor opción en este paciente podría ser
analíticos adicionales necesitaría conocer para una combinación basada en inhibidores de
poder tomar la decisión inmediata más proteasa potenciados.
apropiada:
160. Un varón de 22 años, practicante de deportes de
1. Vitamina B12. aventura, presenta una parálisis facial bilateral.
2. Anticuerpos antinucleares. Un par de meses atrás refiere haber sufrido,
3. Test de Coombs directo. durante un periodo aproximado de cuatro
4. Anticuerpos anti-DNA. semanas, un episodio de debilidad generalizada,
fiebre, cefalea intensa y artromialgias. A pesar
158. Hombre de 60 años diagnosticado de de haber mejorado sustancialmente persiste de
esclerodermia hace once años, con fenómeno de forma fluctuante cierta cefalea y dolor articular.
Raynaud de larga evolución y sensación de La exploración cardiorespiratoria, abdominal y
reflujo gastroesofágico. En la exploración física neurológica son completamente normales, con
destacan telangiectasias en la cara y reflejos osteotendinosos presentes y simétricos.
esclerodactilia. Anticuerpos antinucleares La radiografía de tórax es también normal.
positivos (títulos de 1/2.560) con patrón de ¿Cuál es, con mayor probabilidad, el causante
inmunofluorescencia centromérico, anticuerpos del cuadro clínico descrito?:
anticentrómero positivos. Acude por presentar
desde hace tres meses disnea que en las últimas 1. Síndrome de Guillain-Barré.
semanas se ha intensificado hasta afectar a su 2. Leptospirosis.
actividad física habitual, sin edemas, ortopnea 3. Sarcoidosis.
ni disnea paroxística nocturna. Auscultación 4. Enfermedad de Lyme.
cardíaca y respiratoria normales. La analítica
muestra hemograma y función hepática y renal
normales. Únicamente destaca una elevación del
péptido natriurético atrial (pro-BNP). Teniendo
en cuenta la información aportada ¿cuál debería
ser su primera sospecha diagnóstica?:

1. Enfermedad pulmonar intersticial incipiente.


2. Hipertensión arterial pulmonar.
3. Insuficiencia cardíaca.
4. Tromboembolismo pulmonar.

- 24-
161. Hombre de 45 años que consulta por deterioro 163. Hombre de 87 años, diagnosticado de diabetes
del estado general de 3 semanas de evolución, al mellitus tipo 2, insuficiencia renal crónica,
que se añade un episodio de expectoración EPOC e insuficiencia cardiaca, con una
hemoptoica en las últimas horas. Exploración puntuación en el índice de Barthel de
física normal. En el análisis de sangre destaca: actividades básicas de la vida diaria de 30
hemograma normal; VSG 70 mm/h; creatinina puntos. ¿Cuál sería el objetivo terapéutico
1,8 mg/dl. En el análisis de orina se observa apropiado en el control glucémico de este
microhematuria (60 hematíes/campo) y paciente?:
proteinuria (1,2 g/24h). En la TC de tórax se
observan nódulos pulmonares cavitados, de 1. HbA1c entre 6% y 6,5%.
localización perihiliar bilateral. ¿Cuál de las 2. HbA1c < 7%.
siguientes afirmaciones es cierta?: 3. HbA1c entre 7% y 7,5%.
4. HbA1c entre 8% y 8,5%.
1. La presencia de eosinofilia periférica intensa
(> 1.000 células/L) es una característica de 164. Hombre de 72 años que presenta desde hace 48
la enfermedad. horas fiebre de 38,7°C, tos, expectoración
2. Sin tratamiento, más del 80 % de los purulenta y disnea. Constantes: TA 85/60
pacientes presentan una evolución favorable mmHg, frecuencia cardiaca 100 lpm, frecuencia
con resolución del cuadro. respiratoria 35 rpm, SatO2 80%. Destaca la

FSE MEDICINA 2019/20


3. El rituximab ha demostrado ser efectivo para presencia de confusión y de crepitantes en
la inducción de la remisión de las formas campo pulmonar inferior izquierdo. La
generalizadas con afectación renal. radiografía de tórax confirma la existencia de
4. En torno al 90 % de los pacientes que tienen una neumonía de la língula y del lóbulo inferior
afectación renal muestran positividad de izquierdo. ¿Cuál sería el lugar de atención
anticuerpos anticitoplasma del neutrófilo con adecuado?:
especificidad frente a mieloperoxidasa.
1. Tratamiento ambulatorio en su domicilio.
162. Varón de 27 años con antecedentes de déficit 2. Ingreso en una unidad de observación de
neurocognitivo leve y alteración de la conducta urgencias.
de inicio en la infancia y disfunción hepática 3. Ingreso hospitalario en planta.
crónica (aspartato aminotransferasa 78U/L (5- 4. Ingreso en la Unidad de Cuidados Intensivos.
40 U/L), alanino aminotransferasa 80U/L (5-40
U/L)). En los últimos seis meses ha desarrollado 165. Hombre de 45 años diagnosticado de
una alteración de la marcha. Es remitido a hipertensión arterial con mal control
urgencias por cefalea, vómitos, rechazo de los ambulatorio, que acude por fiebre de 38,8°C
alimentos y somnolencia en el contexto de un desde 2 días antes, dolor retroesternal intenso
síndrome gripal. En la exploración física destaca que se irradia a cuello y a ambos brazos, que
temperatura 37,7°C, disminución de la disminuye cuando se inclina hacia delante y
conciencia (Glasgow 11), pupilas poco reactivas, aumenta con el decúbito supino. En el ECG se
ataxia cerebelosa y paraparesia espástica. ¿Cuál observa elevación del segmento ST en las
de las siguientes afirmaciones es cierta con derivaciones I, II y AVF y de V1 a V6, sin
respecto a su diagnóstico y tratamiento?: alteración del complejo QRS. En el
ecocardiograma se observa un derrame
1. Enfermedad de Wilson. Solicitar pericárdico moderado. ¿Cuál de los siguientes
hemocultivos, pautar tratamiento antibiótico criterios indica mal pronóstico y aconseja el
empírico y dar de alta a domicilio tras ingreso hospitalario?:
resolución de la fiebre.
2. Paraparesia espástica tropical. Administrar 1. Curso agudo (comienzo agudo del dolor
hidratación, asegurar la ingesta oral, solicitar torácico).
VIH y dar de alta a domicilio tras resolución 2. Antecedente de hipertensión arterial mal
de la fiebre. controlada.
3. Enfermedad metabólica congénita del ciclo 3. Fiebre superior a 38°C.
de la urea. Suspender ingesta de proteínas, 4. La presencia de derrame pericárdico.
administrar glucosa, solicitar analítica con
amonio y preparar benzoato sódico.
4. Déficit de alfa-1 antitripsina. Realizar
punción lumbar, administrar aciclovir y ácido
valproico ante la sospecha de meningitis y
estatus epiléptico.

- 25-
166. Un paciente con infección por VIH en 169. Hombre de 65 años, en seguimiento en consulta
tratamiento antirretroviral presenta en sus por astenia, anorexia y febrícula de curso
análisis rutinarios 560 copias de ARN de VIH-1. insidioso. Se realiza TC abdominal con hallazgos
Se encuentra asintomático, los CD4 se de masa retroperitoneal y periaortitis, y
encuentran estables en 280/mm3 y asegura analítica con VSG elevada, anemia moderada y
tomar correctamente el tratamiento. La actitud anticuerpos antinucleares positivos. Indique la
a tomar más adecuada es: respuesta correcta:

1. Determinar niveles plasmáticos de los 1. Se debe a una fibrosis retroperitoneal


fármacos para asegurar que estos se idiopática y no precisa de más estudios
encuentran dentro de los márgenes diagnósticos.
terapéuticos 2. El diagnóstico es clínico y habría que realizar
2. Repetir la carga viral en unas semanas para un tratamiento empírico con antibióticos de
confirmar si se trata de un blip. amplio espectro.
3. Hacer un estudio de resistencias genotípico, 3. La determinación de niveles elevados de
para verificar la existencia de mutaciones de inmunoglobulinas confirma el diagnóstico.
resistencia a los fármacos usados 4. Requiere de un diagnóstico histológico que
4. Intensificar el tratamiento ante la mostrara un infiltrado difuso de células
eventualidad de un fracaso en un paciente plasmáticas y más de 10 células IgG4-

FSE MEDICINA 2019/20


con una infección VIH avanzada. positivas por campo.

167. Las siguientes características sugieren la 170. Hombre de 77 años que acude a urgencias por
presencia de un fenómeno de Raynaud deterioro del nivel de conciencia. En la
secundario, EXCEPTO: anamnesis refiere astenia, anorexia y pérdida de
peso en las últimas semanas. A su llegada
1. Alteraciones capilaroscópicas. presenta febrícula de 37,7°C sin foco definido.
2. Anticuerpos antinucleares positivos. Se realiza una TC craneal sin hallazgos y en el
3. Edad menor de 20 años. LCR obtenido tras punción lumbar se observa
4. Aparición de lesiones necróticas digitales. un líquido ligeramente xantocrómico, con los
siguientes datos: presión de apertura 22 cm de
168. Hombre de 57 años que acude a urgencias por H2O; células 98, 100% mononucleares; glucosa
un dolor epigástrico de elevada intensidad, 5 mg/dL; proteínas totales 2 g/L. ¿Cuál es la
irradiado a espalda y a hipocondrio derecho, etiología más probable?:
junto a náuseas y vómitos de 12 horas de
evolución. En la exploración se encuentra 1. Meningitis neoplásica.
afebril, estable hemodinámicamente, lúcido y 2. Meningitis bacteriana aguda.
con buena perfusión. En la analítica destacan 3. Síndrome de Guillain-Barré.
18.000 leucocitos (80% neutrófilos), siendo el 4. Hemorragia subaracnoidea.
resto del hemograma normal. La bioquímica es
normal, incluyendo calcio, LDH y triglicéridos, 171. Mujer de 67 años con antecedentes de
excepto una amilasa de 3.000 U/L. Señale la dislipemia, que acude a urgencias por un cuadro
respuesta correcta: de disuria y poliaquiuria seguido de fiebre,
escalofríos y deterioro del estado general. A su
1. El cuadro clínico sugiere una colangitis llegada impresiona de gravedad y está
aguda. taquicárdica, taquipneica, con tensión arterial
2. El paciente tiene probablemente una 60/40 mmHg y temperatura de 39°C. ¿Qué
isquemia mesentérica aguda. medida de entre las siguientes NO estaría
3. No es necesario realizar prueba de imagen en incluida en el manejo INICIAL?:
urgencias o en todo caso una ecografía
abdominal. 1. Perfusión intravenosa de dobutamina.
4. Es preciso realizar una TC abdominal 2. Medición de lactato sérico.
urgente. 3. Extracción de hemocultivos.
4. Administración de fluidos.

- 26-
172. ¿Cuál de las siguientes afirmaciones es correcta 175. Un hombre de 18 años llega al servicio de
con respecto a la reanimación urgencias del hospital trasladado por una
cardiorrespiratoria?: unidad de soporte vital básico. Ha sido
atropellado por un automóvil cuando iba en
1. Tras comprobar la ausencia de pulso, la bicicleta. A su llegada tiene una frecuencia
reanimación cardiopulmonar básica debe cardíaca de 115 lpm, una presión arterial de
realizarse siguiendo la pauta de 30 110/75 mmHg, pulso radial palpable, un relleno
compresiones torácicas y 2 insuflaciones capilar de 2,5 segundos y una frecuencia
pulmonares. respiratoria de 25 rpm. ¿Qué nos indican estos
2. En cuanto se disponga de los medios datos?:
necesarios se debe realizar una cardioversión
si el paciente aún no ha recuperado el pulso. 1. Ha perdido entre un 15 y un 30 % de la
3. En caso de asistolia deben administrarse 3 volemia y precisa reposición de volumen.
mg de atropina. 2. Ha perdido más de un 30 % de la volemia y
4. La adrenalina, a dosis de 1 mg cada 3 min, no precisa reposición de volumen y de sangre.
se administrará si el mecanismo de parada es 3. Ha perdido más de un 40 % de la volemia y
una fibrilación ventricular. precisará tratamiento quirúrgico.
4. Ha perdido más de un 50% de la volemia y
173. Un paciente es traído a urgencias desde un precisa reposición de volumen y de sangre.

FSE MEDICINA 2019/20


incendio producido dentro de una nave cerrada
donde había espumas de poliuretano. Está 176. Mujer de 25 años diagnosticada de lupus
consciente, pero presenta creciente torpeza eritematoso sistémico, que ha permanecido en
mental, cefalea e intensa disnea. La saturación remisión los últimos 2 años. Consulta por
de oxígeno mediante pulsoximetría es del 92% y poliartralgias y febrícula. ¿Cuáles de las
el ácido láctico capilar es de 8 mEq/l. ¿Qué siguientes alteraciones indicarán reactivación de
tratamiento específico considera más la enfermedad?:
adecuado?:
1. Elevación de los anticuerpos anti-DNA
1. Administración de oxígeno mediante nativo y de los factores del complemento.
mascarilla tipo Venturi al 50%. 2. Elevación de los anticuerpos anti-DNA
2. Administración de hidroxicobalamina nativo y descenso de los factores del
intravenosa. complemento.
3. Ventilación con cámara hiperbárica. 3. Descenso de los anticuerpos anti-DNA nativo
4. Fluidoterapia con infusión de solución salina y de los factores del complemento.
fisiológica a 21 ml/h. 4. Descenso de los anticuerpos anti-DNA nativo
y elevación de los factores del complemento.
174. Una mujer de 22 años presenta una parada
cardiaca súbita con un ritmo que se considera 177. Considerando la relevancia clínica de los
susceptible de desfibrilación. Se están realizando cocientes de probabilidad positivos (CP+) para
compresiones torácicas y ventilaciones, se ha el proceso de toma de decisiones según The
dado una descarga con el desfibrilador y se ha Evidence - Based Medicine Working Group.
canalizado una vía venosa periférica. ¿Cuál de ¿Cuál de las siguientes opciones se corresponde
las siguientes actuaciones sería correcta a con unos incrementos moderados de la
continuación?: probabilidad de confirmar la enfermedad?:

1. Administrar 150 mg de amiodarona 1. CP+ < 2.


intravenosa. 2. CP+ 2-5.
2. Administrar 300 mg de amiodarona 3. CP+ 5-10.
intravenosa. 4. CP+ > 10.
3. Administrar 1 mg de atropina intravenosa.
4. Administrar 2 mg de atropina intravenosa. 178. ¿Cuál de los siguientes factores NO se asocia al
desarrollo de cálculos de pigmento en la vesícula
biliar?:

1. Hemolisis crónica.
2. Anemia perniciosa.
3. Infección crónica de vías biliares.
4. Colangitis biliar primaria.

- 27-
179. La asociación de fármacos inhibidores de la 184. ¿En qué individuos se observa una elevación
enzima convertidora de la angiotensina y plasmática de la isoenzima BB de la creatina
antagonistas de la aldosterona puede ser útil en quinasa (CK)?
algunos pacientes con insuficiencia cardiaca. No
obstante, deberemos vigilar especialmente en 1. En pacientes con un accidente
estos casos: cerebrovascular.
2. En pacientes con infarto agudo de miocardio.
1. La aparición o agravamiento de dislipemias. 3. Tras una inyección intramuscular.
2. Que pueden aumentar paradójicamente las 4. En pacientes con lesión hepatocelular aguda.
cifras tensionales.
3. La aparición de hiperpotasemia. 185. En 2018 James Allison y Tasuku Honjo
4. Los niveles de bradicininas circulantes. recibieron el Premio Nobel de Medicina por el
descubrimiento de las moléculas CTLA-4 y PD-
180. La osteonecrosis de la mandíbula: 1, que supone un gran avance en la
inmunoterapia del cáncer. El tratamiento de
1. Puede relacionarse con la administración de pacientes con cáncer con anticuerpos anti-PD1
bifosfonatos. (como pembrolizumab) se basa en que:
2. Se asocia en la mayoría de los casos con
radioterapia previa. 1. El anticuerpo anti-PD1 se une al receptor PD-

FSE MEDICINA 2019/20


3. El tratamiento de elección es la 1 expresado en células tumorales induciendo
mandibulectomía. su muerte por apoptosis.
4. Es una forma de atrofia mandibular del 2. El anticuerpo anti-PD1 actúa como puente
desarrollo. entre la célula tumoral y sus ligandos en los
linfocitos T, favoreciendo la destrucción del
181. La distrofia muscular de Duchenne: tumor por linfocitos citotóxicos.
3. El anticuerpo anti-PD1 favorece la
1. Es una miopatía idiopática. opsonización de la célula tumoral y su
2. Afecta por igual a hombres y mujeres. eliminación por las células fagocíticas.
3. Tiene actualmente un tratamiento curativo 4. El anticuerpo anti-PD1 bloquea el
basado en administración de funcionamiento del receptor PD-1, que actúa
glucocorticoides. como un regulador negativo de la actividad
4. Es una enfermedad hereditaria recesiva de los linfocitos T, permitiendo así su
ligada al cromosoma X. activación.

182. Hombre de 60 años con un cuadro de dolor


abdominal y sangrado digestivo alto, en quien se
encuentra una tumoración abdominal
relacionada con la pared gástrica. La histología
muestra un tumor de células fusiformes con
escasas mitosis, positivas para CD117. El
diagnóstico más probable es:

1. Neurofibroma de la pared gástrica.


2. Tumor del estroma gastrointestinal.
3. Leiomioma.
4. Sarcoma granulocítico.

183. ¿Cuál es la mejor medida de carga global de una


enfermedad?:

1. Los años perdidos por discapacidad atribuible


a esa enfermedad.
2. La mortalidad por esa enfermedad.
3. Los años de vida ajustados por discapacidad
atribuible a esa enfermedad.
4. El coste económico de la enfermedad.

- 28-
FSE MEDICINA 2019/20
MEDICINA 2020
MINISTERIO
DE SANIDAD

PRUEBAS SELECTIVAS 2020 NÚMERO DE MESA:

CUADERNO DE EXAMEN NÚMERO DE EXPEDIENTE:


Nº DE D.N.I. O EQUIVALENTE PARA EXTRANJEROS:
MEDICINA - VERSIÓN: 0
APELLIDOS Y NOMBRE:

ABRIR SOLAMENTE A LA INDICACIÓN DEL TRIBUNAL


ADVERTENCIA IMPORTANTE
ANTES DE COMENZAR SU EXAMEN, LEA ATENTAMENTE LAS SIGUIENTES
INSTRUCCIONES

1. MUY IMPORTANTE: Compruebe que este Cuaderno de Examen, lleva todas sus páginas y no
tiene defectos de impresión. Si detecta alguna anomalía, pida otro Cuaderno de Examen a la Mesa.
Realice esta operación al principio, pues si tiene que cambiar el cuaderno de examen posterior-
mente, se le facilitará una versión "0", que no coincide con su versión personal en la colocación de
preguntas y no dispondrá de tiempo adicional.

2. El cuestionario se compone de 175 preguntas más 10 de reserva. Tenga en cuenta que hay 25
preguntas que están ligadas a una imagen. Todas las imágenes están en un cuadernillo de
imágenes separado.

3. Compruebe que el número de versión de examen que figura en su “Hoja de Respuestas”, coin-
cide con el número de versión que figura en el cuestionario. Compruebe también el resto de sus
datos identificativos.

4. La “Hoja de Respuestas” está nominalizada. Se compone de dos ejemplares en papel autocopia-


tivo que deben colocarse correctamente para permitir la impresión de las contestaciones en todos
ellos. Recuerde que debe firmar esta Hoja.

5. Compruebe que la respuesta que va a señalar en la “Hoja de Respuestas” corresponde al


número de pregunta del cuestionario. Sólo se valoran las respuestas marcadas en la “Hoja de
Respuestas”, siempre que se tengan en cuenta las instrucciones contenidas en la misma.

6. Si inutiliza su “Hoja de Respuestas” pida un nuevo juego de repuesto a la Mesa de Examen y no


olvide consignar sus datos personales.

7. Recuerde que el tiempo de realización de este ejercicio es de cuatro horas improrrogables y


que están prohibidos el uso de calculadoras y la utilización de teléfonos móviles, o de cualquier
otro dispositivo con capacidad de almacenamiento de información o posibilidad de comunicación
mediante voz o datos.

8. No se entregarán, en ningún caso, los cuestionarios con las preguntas de examen. Las distin-
tas versiones de los cuadernos de examen se publicarán en la Web del Ministerio de Sanidad, al
cierre de la última mesa de examen.

PROHIBIDA LA REPRODUCCIÓN TOTAL O PARCIAL


FSE MEDICINA 2020/21
1. Pregunta asociada a la imagen 1. 4. Pregunta asociada a la imagen 4.

Paciente miope que consulta por pérdida de visión en Hombre de 68 años que consulta al dermatólogo por
ojo derecho. La exploración del fondo de ojo se ob- lesiones cutáneas ulceradas perianales. El dermató-
serva en la imagen. ¿Cuál de las siguientes afirma- logo se plantea dentro del diagnóstico diferencial un
ciones es correcta respecto a este paciente?: pioderma gangrenoso, una enfermedad ampollosa y
una etiología infecciosa. Realiza una biopsia para
1. Tiene una mejoría de los síntomas al despertarse, estudio histológico y solicita también estudio de im-
que empeoran según avanza el día. munofluorescencia directa. La inmunofluorescencia
2. Refiere una pérdida de visión, como una cortina, directa es negativa y se muestra la histología de la
que le afecta a la zona superior del campo visual. lesión. ¿Cuál es el diagnóstico anatomopatológico de
3. Sería necesario realizar un tratamiento con láser la lesión cutánea perianal?:
YAG previo a la cirugía.
4. Debe realizarse tratamiento quirúrgico mediante 1. Pioderma gangrenoso.
trabeculectomía ab-externo. 2. Pénfigo vulgar.
3. Infección micótica.
2. Pregunta asociada a la imagen 2. 4. Infección por el virus del herpes simple.

Profesora de educación infantil que consulta por 5. Pregunta asociada a la imagen 5.


disfonía y fonastenia de 6 meses de evolución rela-
cionada con el esfuerzo vocal. Durante la semana la Las células indiferenciadas (blastos) de la imagen
voz empeora y en el fin de semana mejora parcial- son típicas, por sus granulaciones en forma de bas-
mente. De los siguientes, ¿cuál es el diagnóstico de tones alargados, de:
sospecha más probable con la imagen de la laringos-
copia que se aporta?: 1. Mieloma secretor de IgA leucemizado.
2. Linfoma folicular leucemizado.
1. Pólipos vocales. 3. Leucemia aguda mieloide.
2. Carcinoma de cuerdas vocales. 4. Leucemia aguda linfoide.
3. Nódulos vocales.
4. Quistes vocales. 6. Pregunta asociada a la imagen 6.

3. Pregunta asociada a la imagen 3. Hombre de 55 años, fumador y con hábito enólico


hasta hace 3 meses. Ingresa por deterioro del estado
Mujer de 60 años, sin antecedentes patológicos de general e ictericia. En la exploración física se palpa
interés, que acude al hospital por pirosis. Durante el hepatomegalia y en la analítica se constata fracaso
proceso diagnóstico se observa en un TC toraco- renal con una creatinina de 4,6 mg/dl. Durante el
abdominal una lesión nodular de 3 cm en la pared ingreso presenta shock séptico por Staphylococcus
posterior de la región cardial, por lo que se realiza aureus y abdomen agudo que se atribuye a isquemia
una resección gástrica atípica. Se muestran dos imá- entérica secundaria al shock séptico, por lo que se
genes, una macroscópica y otra microscópica. Las indica laparotomía exploradora donde se aprovecha
tinciones inmunohistoquímicas de la lesión son posi- para realizar una biopsia hepática. ¿Cuál es el diag-
tivas para c-KIT y DOG1, y negativas para citoque- nóstico anatomopatológico de la lesión hepática?:
ratinas, actina, desmina y S100. ¿Cuál es el diagnós-
tico anatomopatológico de la lesión gástrica?: 1. Cirrosis.
2. Infiltración por carcinoma.
1. Leiomiosarcoma. 3. Amiloidosis.
2. Tumor del estroma gastrointestinal. 4. Hepatitis alcohólica.
3. Schwannoma.
4. Adenocarcinoma poco diferenciado. 7. Pregunta asociada a la imagen 7.

Varón de 58 años con masa mediastínica. El estudio


microscópico muestra los hallazgos de la figura 1A.
En la figura 1B, tinción para citoqueratinas. El diag-
nóstico más probable sería:

1. Metástasis mediastínica de carcinoma sarcomatoi-


de.
2. Sarcoma de nervio periférico.
3. Schwannoma.
4. Timoma.

-1-

FSE MEDICINA 2020/21


8. Pregunta asociada a la imagen 8. 11. Pregunta asociada a la imagen 11.

Mujer de 47 años con debilidad muscular proximal Varón de 71 años, fumador importante activo (IPA
progresiva de extremidades superiores e inferiores >60), sin otros antecedentes de interés, en estudio por
de 7 meses de evolución. Sin antecedentes familiares disnea. Se realiza TC torácica que muestra un nódu-
de interés. Toma arroz rojo para tratar una dislipe- lo sólido de bordes irregulares en LSD y adenopatías
mia. Exploración física: fuerza en psoas 3/5, cuádri- hiliares ipsilaterales. Con estos datos y la prueba de
ceps 4/5 y deltoides 4/5. CK de 3675 UI/L (normal imagen que se muestra (PET-TC con 18F-FDG),
<150 UI/L). Se solicitó un análisis inmunológico y se ¿qué diagnóstico es el más probable?:
practicó una biopsia en el músculo deltoides derecho
(figura). Marque la opción correcta: 1. Neoplasia pulmonar con metástasis ganglionares
ipsilaterales.
1. Análisis inmunológico: Ac. anti-cN1A positivos. 2. Neoplasia pulmonar avanzada con metástasis ex-
Biopsia: infiltrado inflamatorio perivascular y pre- tratorácicas.
sencia de vacuolas ribeteadas. Diagnóstico: miosi- 3. Hamartomas pulmonares.
tis con cuerpos de inclusión. 4. Tuberculosis pulmonar y ganglionar.
2. Análisis inmunológico: Ac. anti-TIF-1 gamma
positivos. Biopsia: infiltrado inflamatorio perivas- 12. Pregunta asociada a la imagen 12.
cular y células en anillo. Diagnóstico: dermatomio-
sitis. Varón de 83 años en quien se detecta de forma ca-
3. Análisis inmunológico: Ac. anti-mitocondriales sual un aumento del tamaño del tiroides al hacerse
positivos. Biopsia: necrosis coagulativa y regene- una TC para estudio de extensión de un cáncer vesi-
ración de fibras musculares con escaso infiltrado cal. En el estudio diagnóstico destaca: TSH <0,005
inflamatorio. Diagnóstico: distrofia muscular con- μUI/mL (VN 0,4 - 4,5), T4 libre 3,49 ng/dl (0,70 -
génita de Ullrich. 1,90), T3 libre 9,56 pg/mL (2,27 - 5,06), anticuerpos
4. Análisis inmunológico: Ac. anti-HMG-CoA reduc- anti-receptor TSH <0,8 IU/l (0 - 1,75), anticuerpos
tasa positivos. Biopsia: necrosis coagulativa y re- antiperoxidasa 8 UI/mL (5 - 34). Ecografía: glándula
generación de fibras musculares con escaso infil- tiroidea aumentada de tamaño, parénquima ocupado
trado inflamatorio. Diagnóstico: miopatía necro- por múltiples nódulos preferentemente sólidos. La
sante inmunomediada. imagen asociada corresponde a una gammagrafía
tiroidea. Con estos resultados morfológicos y funcio-
9. Pregunta asociada a la imagen 9. nales el diagnóstico más probable es:

Mujer de 85 años, trabajadora del campo jubilada, 1. Enfermedad de Graves-Basedow.


que presenta desde hace un año y medio en la nariz 2. Cáncer folicular de tiroides.
una lesión papulosa, redondeada y brillante que le 3. Bocio multinodular tóxico.
sangra con facilidad y crece lentamente. De entre las 4. Tiroiditis autoinmune.
opciones señaladas, ¿cuál es el diagnóstico más pro-
bable?: 13. Pregunta asociada a la imagen 13.

1. Queratoacantoma. Mujer de 50 años, hipertensa, que acude a urgencias


2. Carcinoma basocelular. porque, estando previamente bien, presentó un dolor
3. Queratosis actínica hipertrófica. súbito muy intenso en región torácica interescapular
4. Herpes simple crónico. y en epigastrio, irradiado en cinturón. No refiere
otra patología asociada. Ante el corte sagital con
10. Pregunta asociada a la imagen 10. contraste de TC que se adjunta, ¿cuál de los siguien-
tes diagnósticos sospecha?:
Varón de 45 años, diabético e hipertenso que acude
porque desde hace 3 meses presenta placas eritema- 1. Disección de aorta.
tosas con descamación blanca en cuero cabelludo, 2. Rotura de aorta.
tronco, rodillas y codos (ver imagen), con afectación 3. Hematoma intramural de aorta.
de 4 uñas de las manos. Refiere prurito y decaimien- 4. Úlcera aórtica arteriosclerosa penetrante.
to. ¿Cuál de las siguientes opciones terapéuticas es la
más adecuada?:

1. Prednisona oral.
2. Leflunomida.
3. Adalimumab.
4. Rituximab.

-2-

FSE MEDICINA 2020/21


14. Pregunta asociada a la imagen 14. 18. Pregunta asociada a la imagen 18.

Niña de 3 años de edad que presenta aparición de Paciente de 38 años que acude a urgencias por fie-
una tumoración frontal dolorosa a la palpación sin bre, tos con expectoración purulenta y dolor pleurí-
antecedente traumático ni otras manifestaciones tico en el costado izquierdo. En la exploración física
clínicas o radiológicas asociadas. ¿Qué diagnóstico es está consciente y orientado, la frecuencia respirato-
más probable en la imagen mostrada?: ria es de 34 rpm y la presión arterial sistólica de 85
mmHg. Se auscultan crepitantes en el campo medio y
1. Histiocitosis de células de Langerhans. superior del hemitórax izquierdo. La urea es de 38
2. Rabdomiosarcoma. mg/dl y en la radiografía de tórax se observa una
3. Linfoma. condensación alveolar con broncograma aéreo en el
4. Quiste dermoide. lóbulo superior izquierdo. Se establece el diagnóstico
de neumonía. ¿Cuál de las siguientes opciones sería
15. Pregunta asociada a la imagen 15. la correcta?:

Mujer de 51 años, sin antecedentes de interés, que 1. La puntuación de la escala de riesgo CURB-65 es
consulta por dolor continuo en hipogastrio y fosa 1. Es una persona joven, los gérmenes más proba-
iliaca derecha desde hace 20 días, no irradiado. En la bles son el neumococo y el mycoplasma. Puede
analítica básica únicamente destaca PCR 6 mg/L tratarse ambulatoriamente con azitromicina.
(normal 0-5). Se realiza una ecografía abdominal, 2. La puntuación de la escala de riesgo CURB-65 es
pero al radiólogo le plantea dudas y decide realizarle 2. Debe ingresar en el hospital en una sala conven-
TC abdominopélvica con contraste intravenoso. cional y podría tratarse con una cefalosporina de
¿Cuál es el diagnóstico que sugiere la imagen mos- tercera generación y azitromicina, puesto que hay
trada?: que cubrir el neumococo resistente y la legionella.
3. La puntuación de la escala de riesgo CURB-65 es
1. Absceso pélvico secundario a apendicitis. 3. Debe ingresar en el hospital en una sala conven-
2. Litiasis ureteral derecha con formación de urino- cional. Se tomarán muestras para cultivo con un
ma. catéter telescopado y un lavado broncoalveolar y se
3. Tumor desmoide mesentérico. esperarán los resultados de los cultivos antes de
4. Teratoma ovárico maduro. iniciar cualquier tratamiento antibiótico.
4. La puntuación de la escala de riesgo CURB-65 de
16. Pregunta asociada a la imagen 16. 4. Debe cubrirse como germen más probable la
Pseudomonas aeruginosa y tratarlo con cefalospo-
Mujer de 43 años con diagnóstico de leucemia aguda rinas antipseudomona en una unidad de cuidados
linfoblástica T en tratamiento de inducción con apla- intensivos.
sia tras inicio de prefase con cifra de neutrófilos
<0,5x10ˆ9/L que presenta síndrome febril sin focali- 19. Pregunta asociada a la imagen 19.
dad infecciosa aparente y sin respuesta a tratamiento
antibiótico, por lo que se realiza TC torácica. ¿Cuál Niño de 8 años y medio que acude a urgencias por
de los siguientes diagnósticos sospecha en primer dolor inguinal izquierdo y claudicación de la marcha
lugar ante los hallazgos de la TC?: tras accidente deportivo jugando al fútbol (impacto
con un contrario con posterior caída sobre cadera
1. Neumonía por Pneumocystis jirovecii. izquierda). Los padres relatan algún episodio previo
2. Aspergilosis invasiva. similar que mejoró con reposo y tratamiento médico,
3. Toxicidad pulmonar por fármacos. sin llegar a haber acudido a urgencias. Es un apasio-
4. Afectación neoplásica del pulmón. nado deportista que juega al fútbol y otros deportes,
aunque como dicen los padres últimamente ha debi-
17. Pregunta asociada a la imagen 17. do parar en más de una ocasión. ¿A la vista de la
imagen radiográfica cuál sería su diagnóstico clíni-
Paciente de 40 años, con hipertensión detectada a los co?:
37 años como único antecedente de interés, que con-
sulta por distensión abdominal sin otra sintomatolo- 1. Fractura de cadera como consecuencia de la caída
gía acompañante. La analítica de sangre y orina sufrida.
resultan estrictamente normales. En el contexto de la 2. Epifisiolisis traumática de cadera izquierda.
paciente, ¿qué enfermedad le sugieren las imáge- 3. Artritis séptica de cadera izquierda.
nes?: 4. Enfermedad de Perthes.

1. Poliquistosis renal autosómica dominante.


2. Poliquistosis renal autosómica recesiva.
3. Angiomiolipomas renales en el contexto de escle-
rosis tuberosa.
4. Cáncer renal metastásico.

-3-

FSE MEDICINA 2020/21


20. Pregunta asociada a la imagen 20. 23. Pregunta asociada a la imagen 23.

¿Cuál es el tratamiento de elección para una pacien- Mujer de 25 años que acude a urgencias por dolor
te de 96 años que presenta la fractura de la imagen?: torácico de 2 días de evolución asociado a febrícula
termometrada de hasta 37,6ºC. Este dolor se localiza
1. Clavo trocantérico en mesa ortopédica. a nivel precordial, no se modifica con el ejercicio
2. Prótesis parcial bipolar de cadera. físico y empeora con el decúbito supino. Analítica-
3. Osteosíntesis con tornillos canulados. mente destaca una proteína C reactiva discretamente
4. No cirugía, vida cama-sillón precoz. elevada y leucocitosis moderada. El electrocardio-
grama se muestra en la imagen. Con respecto al
21. Pregunta asociada a la imagen 21. tratamiento de esta patología señale la respuesta
correcta:
Paciente de 44 años con lesión (16x8x12 cm) de cre-
cimiento rápido que interesa las partes blandas del 1. Lo más probable es que se trate de un infarto agudo
tercio proximal del muslo derecho. Refiere que ha de miocardio. Avisaría a Cardiología para realiza-
empezado a notar la masa, que tiene una consistencia ción de coronariografía urgente.
dura y está adherida a planos profundos, en los últi- 2. Los fármacos más utilizados son los AINEs.
mos 6 meses. Previamente hacía deporte habitual- 3. La colchicina está contraindicada en esta patología.
mente, llegando a haber terminado alguna maratón. 4. Lo más correcto sería iniciar tratamiento antibióti-
Se observa a la exploración circulación colateral, co empírico con una fluoroquinolona.
pero no se palpan adenopatías inguinales derechas.
Tras la biopsia se confirma que se trata de un sar- 24. Pregunta asociada a la imagen 24.
coma pleomórfico indiferenciado de alto grado, que
en los estudios de imagen se encuentra proximal al ¿Cuál sería el manejo obstétrico en una mujer se-
trocánter menor. La TC de extensión es negativa. cundigesta de 34 semanas tras el diagnóstico ecográ-
¿Cuál sería su actitud terapéutica?: fico de feto pequeño para edad gestacional, asinto-
mática y que presenta el registro cardiotocográfico
1. Desarticulación de la cadera. externo que aparece en la imagen?:
2. Tratamiento con ifosfamida y tamoxifeno durante 6
ciclos previos a la cirugía de resección para reducir 1. Extracción fetal mediante cesárea programada.
el tamaño de la lesión. 2. Extracción fetal tras inducción de parto.
3. Cirugía marginal extirpando la pseudocápsula que 3. Controles ecográficos fetales periódicos de forma
suelen formar estos tumores como respuesta a su ambulatoria.
rápido crecimiento. 4. Ingreso y control estricto materno-fetal.
4. Hemipelvectomía derecha modificada.
25. Pregunta asociada a la imagen 25.
22. Pregunta asociada a la imagen 22.
Varón de 59 años que consulta por hipertensión
Paciente de 60 años que acude a urgencias por sensa- arterial de al menos 5 años de evolución, actualmente
ción de mareo y sudación de media hora de evolu- en tratamiento con valsartán/hidroclorotiazida y
ción. ¿Qué alteración encontramos en su electrocar- amlodipino a dosis plenas. Aporta cifras de presión
diograma?: arterial ambulatoria de 143/88 mmHg. Con motivo
de un episodio de diverticulitis le habían realizado
1. Taquicardia sinusal. una TC abdominal hace un mes donde llama la aten-
2. Flutter auricular. ción la imagen adjunta. Con estos datos clínicos y
3. Extrasístoles auriculares frecuentes. radiológicos, señale la respuesta correcta:
4. Fibrilación auricular.
1. No es preciso realizar ningún estudio adicional.
2. Está indicada la determinación de renina plasmáti-
ca y aldosterona.
3. El hallazgo en dos analíticas sucesivas de cifras de
potasio <3,5 mEq confirmaría el diagnóstico de
sospecha.
4. La ausencia de hipopotasemia y alcalosis metabóli-
ca, descarta el diagnóstico de sospecha.

-4-

FSE MEDICINA 2020/21


26. La adaptación metabólica al inicio del ayuno inter- 30. En una persona sana y en posición erguida, en rela-
medio (por encima de 24 horas) va acompañado de ción con el cociente ventilación-perfusión, señale la
un aumento en la proteólisis y la liberación de ami- respuesta INCORRECTA:
noácidos desde el músculo. En esta situación, ¿cuál
de las siguientes afirmaciones es cierta?: 1. La ventilación alveolar es mayor en áreas pulmo-
nares superiores.
1. Los aminoácidos ramificados son degradados en el 2. El flujo capilar es menor en áreas pulmonares su-
hígado. periores.
2. La alanina liberada es captada por el hígado para su 3. La ventilación alveolar es ligeramente menor que
uso como sustrato gluconeogénico. el flujo capilar en áreas pulmonares inferiores.
3. La glutamina es transformada en los enterocitos en 4. La ventilación alveolar es mayor que el flujo capi-
alanina, generándose amonio y arginina. lar en áreas pulmonares superiores.
4. Los aminoácidos son utilizados en gran parte en el
hígado para la síntesis de proteínas plasmáticas. 31. El resultado de la gasometría arterial de un paciente
es: pH: 7,40, PaCO2 60 mmHg; bicarbonato 36
27. El 2,3-BPG (2,3-bisfosfoglicerato) es un compuesto mM/L. ¿Cuál es la alteración que presenta?:
orgánico que modula la unión de oxígeno a la hemo-
globina. Este metabolito es sintetizado en los hema- 1. Ninguna, el pH es normal.
tíes a través de una derivación de la vía glicolítica. 2. Acidosis respiratoria.
Señale la respuesta FALSA: 3. Acidosis respiratoria y alcalosis metabólica.
4. Alcalosis metabólica.
1. La concentración de 2,3-BPG en los eritrocitos
(normalmente cercana a 5 mM) aumenta en situa- 32. ¿Cuál de las siguientes consecuencias puede tener
ciones de hipoxia, anemia grave y de adaptación a una mutación del nucleótido G por el nucleótido A
altitudes elevadas. en el primer nucleótido de un intrón?:
2. La presencia de 2,3-BPG apenas influye en la oxi-
genación de la hemoglobina a nivel de los capilares 1. Al ser una mutación intrónica no tiene ninguna
pulmonares en condiciones fisiológicas. repercusión.
3. La adición de 2,3-BPG a la sangre almacenada 2. Al ser una transición (los dos nucleótidos tienen la
para transfusiones es ineficaz. misma estructura) no tiene ninguna repercusión.
4. La hemoglobina fetal tiene mayor afinidad por el 3. Aunque es una mutación intrónica produce la ex-
2,3-BPG que la hemoglobina A1. clusión de todos los exones que están situados de-
trás de la mutación.
28. En cuanto a las interrelaciones entre la presión, el 4. La mutación en este nucleótido puede afectar a la
flujo y la resistencia de los vasos sanguíneos, señale maduración del ARN (splicing) y cambiar la es-
la afirmación INCORRECTA: tructura del ARN y, por consiguiente de la proteí-
na.
1. La resistencia es directamente proporcional a la
diferencia de presión e inversamente proporcional 33. Cuando un mismo cuadro clínico (o con pequeñas
al flujo. modificaciones) está causado por mutaciones dife-
2. En el flujo laminar, la velocidad del flujo en el rentes de un mismo gen nos referimos a la:
centro del vaso es mayor que en los bordes exterio-
res. 1. Heterogeneidad de locus.
3. Cambios pequeños en el diámetro de un vaso pro- 2. Heterogeneidad alélica.
vocan cambios importantes en su conductancia. 3. Heterogeneidad genética.
4. Si se añaden vasos sanguíneos en paralelo en un 4. Impronta parental.
circuito, aumenta la resistencia vascular total.
34. El SARS-CoV-2 es un virus de ARN de hebra senci-
29. En relación con la autorregulación de la filtración lla. Entre los mecanismos que participan en la res-
glomerular (FG) y del flujo sanguíneo renal (FSR) en puesta inmune innata frente a virus de ARN, como el
condiciones fisiológicas señale la afirmación INCO- SARS-CoV-2, se encuentra:
RRECTA:
1. La producción de interferones tipo I tras el recono-
1. Los valores de FG y FSR se mantienen relativa- cimiento del ARN vírico por receptores de recono-
mente constantes a pesar de cambios acentuados en cimiento de patrón como los Toll-like receptors
la presión arterial sistémica. (TLR) o los RIG-like receptors (RLR).
2. Se autorregulan en paralelo, pero en ciertas condi- 2. La producción de interferón gamma por los linfoci-
ciones es más eficiente la autorregulación del FSR. tos CD4+ Th1 tras el reconocimiento de péptidos
3. La reducción del cloruro de sodio en la mácula víricos en las células presentadoras de antígeno.
densa dilata las arteriolas aferentes y aumenta la li- 3. La producción de anticuerpos neutralizantes tipo
beración de renina. IgG producidos en respuesta a las proteínas víricas.
4. La angiotensina II ejerce una acción vasoconstric- 4. La actividad citotóxica mediada por linfocitos T
tora preferente sobre las arteriolas eferentes rena- CD8+ tras el reconocimiento de péptidos víricos en
les. las células infectadas.
-5-

FSE MEDICINA 2020/21


35. Los receptores para los fragmentos Fc de las diferen- 39. ¿Cuál de los siguientes criterios diagnósticos es con-
tes clases de inmunoglobulinas (Ig) se pueden expre- siderado como criterio mayor para realizar el diag-
sar en distintos tipos de células que llevan a cabo nóstico de mastocitosis sistémica?:
funciones efectoras. Indique cuál de las siguientes
afirmaciones es FALSA: 1. Triptasa sérica total superior a 20 ng/ml.
2. Infiltrados densos multifocales de mastocitos (>15
1. La IgG se une al receptor III para Fc-gamma (Fc- mastocitos por agregado) en médula ósea u otros
gamma-RIII) en las células NK activando la cito- tejidos extracutáneos.
toxicidad celular dependiente de anticuerpos 3. Fenotipo superficial de mastocitos anómalos con
(ADCC). expresión de CD25 o CD2 o ambos.
2. La IgA se une al receptor poli-Ig (pIgR) del epite- 4. Morfología alterada de los mastocitos (fusiformes,
lio intestinal para llegar a la luz intestinal. núcleo excéntrico bilobulado o multilobulado, ci-
3. La IgM se une al receptor Fc neonatal (FcRn) en el toplasma hipogranulado).
sincitiotrofoblasto para atravesar la placenta.
4. La IgE se une al receptor para Fc-epsilon (Fc- 40. Recibo una llamada de un compañero de promoción
epsilon-R) en mastocitos e induce su desgranula- que trabaja en otro hospital. Me solicita información
ción. sobre un paciente ingresado en mi hospital para el
que le han pedido una interconsulta de su especiali-
36. La vacuna neumocócica de polisacáridos PPSV23 dad. Como conozco e identifico al médico que me
genera una respuesta inmunitaria de tipo timo- pregunta, la actuación más correcta es:
independiente frente a 23 subtipos de neumococos.
La inmunización con dosis repetidas de PPSV23: 1. Consultar el historial médico del paciente y darle la
información que me pide.
1. Está indicada en niños menores de dos años en 2. Decirle que yo no tengo nada que ver con el pa-
situación de riesgo. ciente y terminar la conversación.
2. Induce una respuesta secundaria con altos niveles 3. Transmitir al médico responsable del paciente la
de IgG específica. pregunta de mi colega y ponerles en contacto.
3. Inducen maduración de la afinidad de los anticuer- 4. Mandarle un informe escrito del paciente para evi-
pos generados. tar dar información por teléfono.
4. No genera memoria inmunológica.
41. En un paciente terminal la omisión o la interrupción
37. En relación con la inmunoterapia basada en linfoci- de tratamientos médicos vitales, para permitir a la
tos T con receptores antigénicos quiméricos (chime- persona que fallezca (interrupción de los tratamien-
ric antigen receptor, CAR) que reconocen CD19 tos que permiten conservar la vida) se denomina:
(CART19):
1. Eutanasia activa voluntaria.
1. Está aprobada para el tratamiento de leucemia mie- 2. Eutanasia activa involuntaria.
loblástica aguda refractaria o en recaída. 3. Eutanasia pasiva.
2. Un efecto secundario grave de esta terapia es el 4. Suicidio asistido por un médico.
síndrome de liberación de citoquinas.
3. El receptor CAR reconoce CD19 de forma restrin- 42. ¿Cuál de las siguientes respuestas NO es cierta en
gida por el HLA. relación con el tratamiento del dolor en el paciente
4. Se asocia a hipergammaglobulinemia permanente con cáncer?:
por activación crónica de linfocitos B CD19+.
1. El fentanilo transmucoso (oral o nasal) es el fárma-
38. Paciente de 72 años de edad que acude a urgencias co de elección en el tratamiento del dolor irruptivo
por presentar aparatoso e importante angioedema oncológico.
lingual. No presenta urticaria asociada ni refiere 2. Los ensayos clínicos demuestran diferencias en la
antecedentes de alergia a fármacos o alimentos. En- eficacia y tolerabilidad entre los diferentes opioides
tre sus antecedentes personales destacan diabetes mayores.
mellitus tipo 2, dislipemia, hipertensión arterial, 3. La rotación de opioides o la analgesia raquídea son
hipotiroidismo y enfermedad de Parkinson. En tra- medidas de tratamiento del dolor refractario.
tamiento habitual con metformina, simvastatina, 4. La eficacia de la escalera analgésica de la Organi-
enalapril, tiroxina y levodopa. ¿Cuál de los siguien- zación Mundial de la Salud (OMS) es del 80-90 %.
tes fármacos es con mayor probabilidad el fármaco
causal del cuadro clínico descrito?: 43. En una intoxicación por insecticidas organofosfora-
dos, señale la respuesta correcta:
1. Enalapril.
2. Simvastatina. 1. No existe absorción a través de la piel.
3. Metformina o levodopa por igual. 2. Es común el fallecimiento al cabo de unos minutos.
4. Levodopa. 3. Se inicia con síntomas nicotínicos.
4. Su acción selectiva es anticolinesterásica.

-6-

FSE MEDICINA 2020/21


44. ¿En cuál de las siguientes intoxicaciones por plantas 49. ¿Cuál de las siguientes enfermedades alcanza mayor
nos debemos plantear una etiología accidental por letalidad?:
uso intencionado con fines recreativos?:
1. Ictus.
1. Taxus baccata (tejo). 2. COVID-19.
2. Thevetia peruviana (adelfa amarilla). 3. Infarto agudo de miocardio.
3. Atropa belladona (escopolamina). 4. Encefalopatía espongiforme bovina.
4. Aconitum napellus (acónito).
50. Respecto a las características de una prueba diagnós-
45. Para conocer si el consumo habitual de una dieta de tica, indique la respuesta correcta:
tipo mediterráneo se asocia a menor riesgo de infarto
de miocardio no fatal se selecciona un grupo de suje- 1. Los valores predictivos dependen de la probabili-
tos, se averigua el grado de adherencia a dicha dieta dad a priori de tener la enfermedad.
y se les sigue durante 5 años para identificar los ca- 2. La especificidad es la capacidad de la prueba de
sos nuevos de infarto de miocardio. Señale la res- clasificar correctamente, con un resultado negativo,
puesta correcta sobre el diseño de este estudio: a los enfermos.
3. La especificidad es la capacidad de la prueba de
1. Es un ensayo clínico porque puede orientar la pre- clasificar correctamente, con un resultado positivo,
vención en la clínica. a los enfermos.
2. Es un estudio de cohortes, porque se sigue a sujetos 4. El valor predictivo positivo indica la probabilidad
clasificados según su exposición para identificar el de que un paciente con un resultado positivo no
riesgo de una enfermedad. tenga la enfermedad.
3. Es un estudio ecológico porque se sigue a un grupo
de sujetos. 51. Respecto a la clorhexidina señale la respuesta correc-
4. Es un estudio transversal porque la adherencia a la ta:
dieta mediterránea se valora en un momento con-
creto en el tiempo (al inicio del seguimiento). 1. Es una sustancia del mismo tipo químico que las
biguanidas.
46. Un forest plot o diagrama de bosque es: 2. Carece de actividad frente a los hongos.
3. No se debe utilizar como antiséptico.
1. Un tipo de representación gráfica de los resultados 4. En mucosas y heridas se utiliza concentrada al 5 %
(por ejemplo, estimadores de efecto) de un meta- en solución acuosa.
análisis de ensayos clínicos.
2. Un tipo de histograma que se usa en las revisiones 52. Sobre los cocientes de probabilidad o razones de
sistemáticas de la literatura. verosimilitud de una prueba diagnóstica, indique la
3. Una forma de presentar las modas de una distribu- respuesta INCORRECTA:
ción no normal.
4. El diagrama de flujos de los artículos en una revi- 1. El cociente de probabilidades se configura como el
sión de la literatura. índice que engloba la sensibilidad y especificidad y
sus complementarios.
47. Las guías de práctica clínica: 2. Las pruebas diagnósticas con cocientes de probabi-
lidad próximos al valor 1 no tienen utilidad, pues
1. Son de obligado cumplimiento para todos los mé- sus resultados no cambiarán la probabilidad previa.
dicos. 3. El cociente de probabilidades del resultado positivo
2. Todo su contenido ha de estar basado en la eviden- de una prueba nos indica cuántas veces más proba-
cia científica. ble es que la prueba sea positiva en una persona
3. Solo deben tener en cuenta los beneficios de salud que no tenga la enfermedad que en una que sí la
de los pacientes (por tanto, no debe considerar el tenga.
coste de las intervenciones clínicas). 4. Los cocientes de probabilidad indican hasta qué
4. Son recomendaciones que el médico debe adaptar a punto un resultado determinado de un examen
la situación clínica de cada paciente. diagnóstico aumentará o disminuirá la probabilidad
preexamen de un trastorno objetivo.
48. Los ensayos clínicos en fase II tienen como objetivo:

1. La estimación inicial de la seguridad y tolerancia.


2. Demostrar el efecto terapéutico.
3. Obtener información sobre la eficacia.
4. Evaluar la aparición de efectos secundarios.

-7-

FSE MEDICINA 2020/21


53. Mujer de 42 años que acude a consulta de su médico 58. Ante un paciente que presenta una lesión úlcero-
de familia por gonalgia. De forma oportunista se vegetante en la encía mandibular a nivel de la región
aprovecha la consulta para valorar estilos de vida, de los molares de 5 meses de evolución, la aparición
especialmente el tabaquismo. Si se quiere seguir la de hipoestesia en la región mentoniana requiere
estrategia de educación para la salud basada en el descartar:
modelo de las cinco “aes”, ¿cuál de las siguientes NO
se incluye en esta estrategia?: 1. Una neuropatía paraneoplásica.
2. El diagnóstico de sarcoidosis con parálisis facial
1. Averiguar: preguntar sobre los factores y las con- "en báscula".
ductas de riesgo (preguntar a la paciente si fuma). 3. Infección por virus herpes en paciente inmunode-
2. Aumentar: incrementar la percepción de riesgo primido.
para facilitar el cambio (explicar las consecuencias 4. El diagnóstico de carcinoma con infiltración del
del tabaquismo). canal mandibular.
3. Aconsejar: dar consejos claros, específicos y per-
sonalizados (aconsejar el abandono del tabaco). 59. Ante un paciente de 9 años de edad que ha sufrido
4. Acordar: pactar colaborativamente los objetivos de recientemente un traumatismo directo sobre el men-
cambio (valorar la disponibilidad para hacer un in- tón y que presenta dificultad para la apertura de la
tento de dejar de fumar). boca y maloclusión dental, debemos descartar como
diagnóstico más probable:
54. Mujer de 86 años de edad en quien se ha detectado
una fibrilación auricular no valvular. Tiene una 1. Fractura tipo Lefort-1.
puntuación CHADS2 de 3 puntos. En la bibliografía, 2. Fractura conminuta de ambas ramas horizontales
pacientes similares en tratamiento con warfarina mandibulares.
tienen un riesgo de ictus del 2,2 % frente al 5,2 % en 3. Fractura de cóndilo mandibular.
los pacientes sin warfarina. ¿Cuál sería el número 4. Fractura en tallo verde de región sinfisaria mandi-
necesario a tratar (NNT) para prevenir un ictus bular.
embolígeno con el tratamiento anticoagulante?:
60. En cuanto a la anatomía de la pared abdominal en
1. 3. relación con la cirugía reconstructiva es FALSO que:
2. 19,2.
3. 33,3. 1. La inervación de los músculos rectos abdominales
4. 49,5. procede de nervios que discurren entre el músculo
transverso abdominal y el músculo oblicuo interno.
55. A la hora de valorar las distintas alternativas farma- 2. Tras la disección y rotación de un colgajo TRAM
coterapéuticas para un paciente con dolor, ¿qué (transversus rectus abdominis musculus) se produ-
parámetro farmacológico considera que determinará ce una debilidad de la pared abdominal especial-
la eficacia máxima del analgésico?: mente por encima de la línea arcuata.
3. La irrigación del músculo recto abdominal provie-
1. Su potencia. ne principalmente de los vasos epigástricos inferio-
2. Su actividad intrínseca. res y superiores profundos.
3. Su índice terapéutico. 4. El músculo recto abdominal puede utilizarse para
4. Su afinidad por el lugar diana de acción. reconstrucción de defectos mediastínicos.

56. La capacidad de un medicamento de producir el 61. ¿Cuál de los siguientes NO es un factor de riesgo de
efecto deseado en condiciones ideales de uso es lo que sufrir un glaucoma primario de ángulo abierto?:
llamamos:
1. Tomar anticonceptivos orales.
1. Efectividad. 2. Tener hipermetropía.
2. Eficacia. 3. Ser de raza negra.
3. Potencia farmacológica. 4. Que los hermanos estén diagnosticados de glauco-
4. Afinidad intrínseca. ma.

57. ¿Cuál de las siguientes situaciones NO es una indica- 62. Acude a urgencias un niño de 3 años con sus padres
ción de los anticoagulantes orales de acción directa?: porque el niño ha presentado de manera aguda y
brusca fiebre alta, escalofríos, estridor, disnea, voz
1. Portador de prótesis valvular aortica mecánica. apagada y babeo. ¿Cuál es la sospecha diagnóstica
2. Tratamiento del tromboembolismo pulmonar. más probable?:
3. Profilaxis de la trombosis venosa en el postopera-
torio de prótesis de rodilla. 1. Laringotraqueítis aguda.
4. Prevención de ictus en fibrilación auricular no val- 2. Edema angioneurótico de la laringe.
vular. 3. Amigdalititis aguda.
4. Epiglotitis.

-8-

FSE MEDICINA 2020/21


63. Mujer de 56 años que consulta por presentar, desde 67. Una mujer de 26 años, con antecedente de un emba-
hace 6 meses, habla entrecortada, con altibajos, co- razo a los 24 años finalizado en aborto espontáneo en
mo estrangulada, y temblor en la voz. Después de la la semana 12ª que no requirió legrado, acude a la
exploración endoscópica laríngea, del análisis acústi- consulta presentando sangrado vaginal diario de un
co de la voz y del habla, la sospecha diagnóstica es mes de evolución, sin fiebre ni dolor. En la ecografía
una distonía laríngea o disfonía espasmódica adduc- se observa un aumento de la línea endometrial, con
tora. ¿Cuál de las siguientes propuestas de trata- una imagen intracavitaria de aspecto polipoideo. La
miento está indicada?: biopsia endometrial muestra proliferación del trofo-
blasto intermedio. ¿Qué diagnóstico es el correcto?:
1. Sección del nervio laríngeo inferior o recurrente.
2. Extirpación del músculo tiroaritenoideo. 1. Tumor del lecho placentario.
3. Tratamiento logopédico y administración de rela- 2. Hiperplasia endometrial atípica.
jantes musculares. 3. Pólipo endometrial.
4. Inyección de toxina botulínica en el músculo de la 4. Coriocarcinoma.
cuerda vocal.
68. Acerca de la terapia hormonal en la menopausia,
64. De los siguientes elementos, ¿cuál actuaría como señale la respuesta correcta:
factor protector frente al cáncer de endometrio?:
1. Los estrógenos vaginales son un tratamiento far-
1. Menopausia tardía. macológico eficaz para los síntomas urogenitales.
2. Implante de etonorgestrel. 2. Las dosis de estrógenos administrados en la terapia
3. Síndrome de ovario poliquístico. hormonal deben alcanzar niveles plasmáticos simi-
4. Endometriosis ovárica. lares o superiores a los alcanzados en un ciclo
menstrual normal.
65. Acude a la consulta una mujer de 15 años con dolor 3. La tibolona es un fármaco indicado en mujeres con
intenso en el inicio de las menstruaciones. El dolor se antecedentes de cáncer de mama y clínica vasomo-
localiza en hipogastrio y se irradia a la zona lumbar. tora intensa.
Asocia en numerosas ocasiones náuseas y diarrea; a 4. En mujeres con útero, está indicada la terapia hor-
veces no puede ir al instituto. La clínica desaparece monal sistémica con sólo estrógenos.
con el fin de la menstruación. El resultado de la ex-
ploración ginecológica es normal al igual que los 69. ¿Cuál de los siguientes datos NO es útil para la de-
valores analíticos. Señale la respuesta correcta: terminación de la reserva ovárica?:

1. Se trata de una dismenorrea secundaria a un pro- 1. Progesterona sérica basal.


blema digestivo. 2. Hormona antimülleriana sérica basal.
2. Le pauta anticonceptivos hormonales, pero no AI- 3. Recuento ecográfico de folículos antrales.
NES por la clínica digestiva. 4. Hormona folículo estimulante sérica basal.
3. Le pauta AINES y le explica que los anticoncepti-
vos hormonales en su caso no están indicados por 70. ¿Cuál de los siguientes factores NO aumenta el ries-
su edad. go de padecer cáncer de mama?:
4. Le pauta AINES y, si no mejora, le podría pautar
anticonceptivos hormonales. 1. Consumo moderado de alcohol.
2. Obesidad.
66. ¿Cuál sería el método anticonceptivo más aconseja- 3. Mastopatía fibroquística.
ble en una mujer de 37 años sin antecedentes de 4. Menopausia tardía.
interés cuyo parto fue hace 4 meses, ha reiniciado sus
ciclos menstruales y mantiene lactancia materna?: 71. Mujer de 31 años que acude a la consulta de su mé-
dico de familia a comunicarle que está embarazada.
1. Ninguno; la lactancia materna, a través de la libe- No tiene antecedentes de interés. Se le recomienda
ración de oxitocina, impide la gestación. que se haga la prueba de cribado de la diabetes ges-
2. Dispositivo intrauterino con levonorgestrel. tacional. ¿En qué periodo debería realizarse?:
3. Preparados hormonales combinados con 20 micro-
gramos de etinilestradiol. 1. Entre las semanas 12 y 16 de gestación.
4. Preparados hormonales combinados con gestáge- 2. Entre las semanas 16 y 20 de gestación.
nos de tercera generación. 3. Entre las semanas 24 y 28 de gestación.
4. Entre las semanas 32 y 38 de gestación.

-9-

FSE MEDICINA 2020/21


72. En relación con la gestación en el lupus eritematoso 77. Lactante de 4 meses que acude a urgencias por vómi-
sistémico, es cierto que: tos y letargia. En la exploración llama la atención
disminución importante del nivel de conciencia con
1. La fertilidad no se ve afectada. incapacidad para despertar a pesar de estímulos. En
2. Durante la gestación los brotes suelen ser más fre- los exámenes complementarios destaca la presencia
cuentes y graves. de anemia, hemorragias retinianas en el fondo de
3. En el puerperio, la enfermedad no suele reactivar- ojo, hematoma subdural en la TC craneal y fractura
se. en parte posterior de tercera costilla. ¿Cuál de los
4. Una paciente con anticuerpos anti-Ro y bloqueo siguientes diagnósticos considera más probable como
AV fetal tiene menos riesgo de presentar el blo- responsable de estas lesiones?:
queo AV fetal en la siguiente gestación.
1. Maltrato físico.
73. En el megacolon agangliónico congénito (enfermedad 2. Politraumatismo por caída de la cuna.
de Hirschsprung) se dan todas las características 3. Traumatismo torácico en lactante con alteración de
siguientes, EXCEPTO una. Señálela: la coagulación.
4. Leucemia promielocítica aguda tras realizar aspira-
1. En la zona afectada existe una concentración au- do de médula ósea.
mentada de acetilcolinesterasa.
2. Está ausente el reflejo inhibitorio recto-anal. 78. Referida a la parotiditis epidémica en la infancia,
3. Las pruebas diagnósticas de referencia son las señale la respuesta FALSA:
biopsias rectales por succión.
4. Es frecuente la presencia de encopresis o inconti- 1. El virus de la parotiditis solo presenta un inmuno-
nencia fecal. tipo y el ser humano es el único huésped natural.
2. La vacuna de la parotiditis está incluida en la va-
74. Varón de 9 años que consulta por hematuria ma- cuna triple vírica y en la tetravírica y precisa dos
croscópica, cefalea y astenia, sin dolor abdominal ni dosis.
edemas. Antecedente de infección faringoamigdalar 3. La orquitis como complicación es más frecuente en
1-2 semanas antes. Niveles de C3 sérico descendidos. la infancia más temprana.
De las siguientes opciones diagnósticas indique la 4. La pancreatitis puede producirse en la parotiditis
más probable por su frecuencia y presentación clíni- con o sin afectación clínica de las glándulas parotí-
ca: deas.

1. Glomerulopatía membranosa. 79. Señale la respuesta FALSA con respecto a la clínica,


2. Nefropatía por inmunoglobulina A (nefropatía de diagnóstico y tratamiento de la fibrosis quística:
Berger).
3. Glomerulonefritis membranoproliferativa o me- 1. La colonopatía fibrosante es consecuencia de la
sangiocapilar. diarrea crónica por dosis insuficientes de enzimas
4. Glomerulonefritis aguda postinfecciosa. pancreáticos.
2. La hepatopatía es independiente del genotipo, aun-
75. ¿Cuál de los siguientes criterios NO es característico que sí se asocia más a insuficiencia pancreática e
de una convulsión febril simple en un niño?: ileo meconial.
3. La determinación de elastasa 1 fecal es la prueba
1. Crisis primaria generalizada. más útil para el diagnóstico de insuficiencia pan-
2. Duración inferior a 15 minutos. creática.
3. Recurrencia en las primeras 24 horas. 4. En el lactante puede manifestarse con una alcalosis
4. No hay focalidad neurológica. metabólica hipoclorémica.

76. Niña de 7 años que acude a consulta porque los pa- 80. En la hiperplasia suprarrenal congénita debida a la
dres le notan la cara roja desde el día anterior y en forma clásica de déficit de 21-hidroxilasa, señale la
las últimas horas le han notado manchas rojas en el respuesta correcta:
cuerpo. En la exploración está afebril, con amígda-
las y oídos sin alteraciones, auscultación cardiopul- 1. Existe un defecto en la síntesis de androstendiona.
monar normal. Presenta un exantema maculo papu- 2. Suele manifestarse ya en el periodo neonatal con
loso confluyente en ambas mejillas, respetando el síndrome pierde-sal y virilización en niñas.
labio superior, semejando cara de “bofetón”. En el 3. El tratamiento de mantenimiento con hidrocortiso-
tórax presenta un exantema reticular, en encaje, no na en monoterapia es suficiente para controlar la
pruriginoso. ¿Cuál es el diagnóstico clínico más pro- pérdida salina.
bable?: 4. Los niveles de 17-OH-progesterona se encuentran
disminuidos.
1. Varicela.
2. Rubeola.
3. Eritema infeccioso o 5ª enfermedad.
4. Exantema súbito o 6ª enfermedad.

- 10 -

FSE MEDICINA 2020/21


81. En relación con la anemia de Fanconi indique la 86. En relación con la hipomanía señale cuál de las si-
respuesta INCORRECTA: guientes características es INCORRECTA:

1. La anemia aplásica suele estar ya presente en los 1. La hipomanía casi nunca es motivo de consulta y
primeros meses de vida. debe buscarse en la anamnesis de los episodios de-
2. Son típicas las malformaciones esqueléticas. presivos.
3. El trasplante de progenitores hematopoyéticos es el 2. El antecedente de hipomanía en un paciente con
tratamiento curativo, pero no hace desaparecer el episodio depresivo mayor modifica el diagnóstico
riesgo de cáncer. de trastorno depresivo e influye en la decisión tera-
4. Los andrógenos suelen producir una respuesta fa- péutica.
vorable pero transitoria. 3. Los pacientes con hipomanía pueden sentirse más
activos y sociables, aunque generalmente su com-
82. Un lactante de 1 mes se encuentra hospitalizado y a portamiento resulta algo inapropiado.
dieta absoluta. Utilizando el método del peso corpo- 4. Estos pacientes presentar síntomas psicóticos, co-
ral para el cálculo del volumen diario de líquido de mo ideas delirantes de grandiosidad.
mantenimiento a este paciente le corresponderá:
87. Un paciente con una recaída psicótica que ha empe-
1. 30 ml/kg/día. zado tratamiento con un antipsicótico al que mostró
2. 60 ml/kg/día. buena respuesta en el episodio anterior acude a con-
3. 80 ml/kg/día. sulta por presentar sensación de inquietud motora
4. 100 ml/kg/día. que le obliga a moverse constantemente y le impide
permanecer sentado. ¿Cuál de los siguientes es el
83. Respecto al tratamiento profiláctico con litio en el tratamiento más adecuado?:
trastorno bipolar, señale la respuesta correcta:
1. Reducir la dosis del antipsicótico y añadir bromo-
1. El margen entre las dosis terapéuticas y tóxicas es criptina.
estrecho. 2. Añadir β-bloqueantes.
2. Su uso es seguro durante el embarazo. 3. Añadir dantroleno.
3. Los nuevos antipsicóticos han hecho que su uso 4. Añadir un anticolinérgico.
quede prácticamente obsoleto.
4. Los diuréticos tiazídicos pueden disminuir su tasa 88. Según el DSM 5, el trastorno dismórfico corporal
sanguínea y por tanto su eficacia. pertenece al grupo de:

84. ¿En cuál de los siguientes antipsicóticos es necesario 1. Trastorno obsesivo-compulsivo y trastornos rela-
realizar hemogramas de control durante el trata- cionados.
miento por presentar un mayor riesgo de agranuloci- 2. Trastornos de ansiedad.
tosis?: 3. Trastornos psicosomáticos.
4. Trastornos del estado de ánimo.
1. Olanzapina.
2. Quetiapina. 89. Para tratar el dolor neuropático periférico se podría
3. Clozapina. utilizar uno de los siguientes fármacos antiepilépti-
4. Risperidona. cos:

85. En un paciente de 85 años con deterioro cognitivo y 1. Etosuximida.


un episodio depresivo, señale el tratamiento antide- 2. Vigabatrina.
presivo que se debe EVITAR: 3. Gabapentina.
4. Levetiracetam.
1. Amitriptilina.
2. Sertralina.
3. Vortioxetina.
4. Venlafaxina.

- 11 -

FSE MEDICINA 2020/21


90. Hombre de 59 años, casado, sin antecedentes psi- 94. Mujer de 55 años con antecedentes de diabetes melli-
quiátricos, que de forma progresiva a lo largo de los tus tipo 2 y ansiedad que acude a urgencias porque
dos últimos años presenta importante irritabilidad y esa tarde de forma súbita comenzó con dificultad
apatía. Ha faltado varias veces a su trabajo y su para la emisión del lenguaje (“quería decir mesa
rendimiento ha disminuido notablemente. Le han pero no me salía") y movimientos incontrolables
amonestado en el mismo, por insultar a varios clien- “como sacudidas” en el brazo y en la pierna dere-
tes o por ausencias injustificadas. En los fines de chos. Pasados dos minutos, los movimientos cedieron
semana es monitor de tiempo libre con adolescentes, y hablaba con normalidad. Ella en todo momento
donde refieren que bromea excesivamente con los fue consciente de la situación. ¿Cuál es el diagnóstico
chicos y una monitora se ha quejado de que le ha más probable?:
hecho insinuaciones inapropiadas. Hasta el presente,
nunca habían ocurrido comportamientos de este 1. Crisis focal compleja.
tipo. Su esposa está muy preocupada porque dice 2. Accidente isquémico transitorio en territorio caro-
cosas raras y está pensando en separarse. Su médico tideo derecho.
de familia le ha practicado un test cognitivo Mini- 3. Crisis de ansiedad.
mental que está dentro del rango normal y en el que 4. Crisis focal simple.
no se aprecian déficit significativos de memoria.
¿Cuál es el diagnóstico MENOS probable?: 95. Hombre de 64 años que consulta por presentar desde
hace un año alteración progresiva del lenguaje con-
1. Esquizofrenia de inicio tardío. sistente en dificultad para encontrar las palabras.
2. Episodio maniaco de inicio tardío. Usa muchos términos ómnibus y circunloquios. En-
3. Enfermedad de Alzheimer. tiende perfectamente y el lenguaje es relativamente
4. Demencia frontotemporal. fluido salvo por las pausas anómicas. En la explora-
ción llama la atención la dificultad para repetir fra-
91. Indicar cuál de los siguiente estudios mediante TC ses largas. No tiene alteración de la memoria ni pro-
puede diferenciar, en un ictus del despertar, el área blemas de conducta evidentes. ¿Cuál es el diagnósti-
cerebral en riesgo de isquemia potencialmente recu- co más probable?:
perable del área de infarto ya establecido irrecupe-
rable: 1. Variante no fluente de la afasia progresiva primaria
probablemente por demencia frontotemporal.
1. AngioTC. 2. Variante logopénica de la afasia progresiva prima-
2. TC de difusión. ria, probablemente por enfermedad de Alzheimer.
3. TC de perfusión. 3. Demencia por cuerpos de Lewy difusos.
4. TC sin contraste. 4. Disnomia inespecífica del paciente mayor.

92. Señale cuál de las siguientes manifestaciones clínicas 96. Mujer de 75 años que consulta por un cuadro de
le harían dudar de un diagnóstico de enfermedad de dolor lancinante que recorre el labio y el ala nasal
Parkinson: derecha. No es capaz de lavarse la cara o los dientes
por el dolor que le produce, que describe como una
1. Edad de inicio a los 55 años. descarga eléctrica. No refiere dolor al dormir. Escoja
2. Caídas dentro de los dos primeros años de sintoma- la respuesta correcta:
tología.
3. Presencia de un trastorno de conducta durante el 1. El cuadro corresponde a una hemicránea paroxísti-
sueño REM. ca.
4. Ausencia de temblor. 2. El tratamiento de elección es la fenitoína.
3. La causa más probable es una compresión del VII
93. Paciente de 65 años con antecedentes de consumo par en su trayecto cisternal.
habitual de alcohol que es atendido en urgencias por 4. Puede tratarse con radiocirugía, pero con riesgo de
cuadro clínico de 24 horas de evolución de fiebre, recurrencia.
cefalea, rigidez de nuca y progresivo deterioro del
nivel de conciencia. Se realiza una TC craneal que es 97. Tras un accidente de tráfico usted atiende a un heri-
normal y seguidamente una punción lumbar que do. El paciente abre los ojos a la llamada, pero úni-
muestra un LCR con pleocitosis (1500 células/µl), camente emite palabras que son inapropiadas. En la
hiperproteinorraquia (325 mg/dl) e hipoglucorraquia exploración motora localiza el dolor en extremida-
(< 10 mg/dl). El tratamiento antibiótico empírico des derechas pero extiende al estimular las extremi-
inicial más correcto de entre los siguientes es: dades izquierdas. ¿Cómo describiría su situación
según la escala de coma de Glasgow?:
1. Cefotaxima y ampicilina.
2. Cefotaxima y vancomicina. 1. E3V2M4.
3. Cefotaxima, vancomicina y ampicilina. 2. E3V3M5.
4. Cefotaxima y tuberculostáticos. 3. E2V3M4.
4. E2V4M5.

- 12 -

FSE MEDICINA 2020/21


98. ¿Cuál de los siguientes NO es un rasgo principal de 101. Varón de 15 años de edad, ingresado tras haber sido
la demencia frontotemporal?: atropellado en la vía pública, resultando despedido.
Ingresa consciente y orientado, con intenso dolor en
1. Habla estereotipada o ecolalia. hemitórax izquierdo y disnea. En la exploración
2. Desinhibición (sexual, locuacidad). física se halla tensión arterial 90/60 mmHg, frecuen-
3. Rápido inicio y progresión de la enfermedad. cia cardiaca 130 lpm, saturación basal de oxígeno 90
4. Pérdida de la higiene personal. %, frecuencia respiratoria 35 rpm. Hay hipofonesis
completa en el hemitórax izquierdo y matidez a la
99. En relación con el síndrome de piernas inquietas en percusión. En la radiografía de tórax se ven múlti-
el paciente mayor, señale la respuesta INCORREC- ples fracturas costales izquierdas y derrame pleural
TA: ipsilateral masivo. Se indica la colocación de un dre-
naje pleural, con salida de 1700 cc de líquido hemáti-
1. Existe una forma familiar. co. ¿Cuál es la decisión que se debe tomar?:
2. La deficiencia de hierro se considera un factor de
riesgo para su aparición. 1. Instaurar ventilación mecánica no invasiva y solici-
3. Su prevalencia aumenta a partir de los 70 años. tar transfusión sanguínea.
4. Es más frecuente en los varones que en las muje- 2. Analgesia, oxigenoterapia, solicitar transfusión
res. sanguínea e indicar una intervención quirúrgica ur-
gente.
100. Un paciente de 73 años con antecedente de enferme- 3. Intubación orotraqueal, solicitar transfusión san-
dad renal crónica que presenta un cuadro de 5 días guínea y observación para, en caso de empeora-
de fiebre, cefalea y confusión, acompañado de una miento, indicar la intervención quirúrgica.
crisis epiléptica tónico-clónica. El estado clínico en el 4. Intubación orotraqueal y realización de una TC
momento de la evaluación es bueno y la exploración urgente para valoración exacta de las lesiones.
física es normal. En los análisis destaca leucocitosis
de 18.000/µl con neutrofilia del 92 %, serie roja y 102. En un paciente en situación de shock séptico con un
plaquetas normales; creatinina 2,3 mg/dl, eFG 32 foco infeccioso abordable, ¿cuál de los siguientes
ml/min y PCR 15 mg/L (VN<5). La TC craneal no enunciados es cierto?:
muestra alteraciones. El LCR muestra un líquido
claro con 200 leucocitos/µl (60 % linfocitos), glucosa 1. El control del foco solo se intentará una vez el pa-
80 mg/dl, proteínas 85 mg/dl (VN<50), ADA 25 UI/l ciente haya alcanzado la estabilidad hemodinámica
(VN<15); la tinción de Gram del LCR no muestra y no dependa de vasopresores.
microorganismos. ¿Qué tratamiento recomendaría 2. El control del foco se realizará tras al menos la
en este momento?: administración de dos dosis de antibiótico.
3. El control del foco se realizará tan pronto como sea
1. Aciclovir. posible.
2. Aciclovir más isoniacida, rifampicina, etambutol y 4. Si la situación de shock séptico lleva más de 12
pirazinamida. horas de evolución el control del foco ya no será
3. Aciclovir más metilprednisolona IV. beneficioso para el paciente.
4. Vancomicina, ampicilina y ceftriaxona IV.
103. Un paciente de 56 años es encontrado en coma en su
domicilio. Tiene antecedentes de hipertensión arte-
rial y diabetes mellitus. No tiene hábitos tóxicos ni
ninguna otra enfermedad crónica. Está en trata-
miento con irbesartan y empaglifozina. No presenta
signos de desnutrición. Presenta una presión arterial
de 110/60 mmHg, una frecuencia cardiaca de 110
lpm, una SatO2 del 90 %, una glucosa capilar de 120
mg/dl y una frecuencia respiratoria de 7 rpm. ¿Qué
actitud terapéutica inicial le parece la más correcta?:

1. Administrar naloxona, flumazenilo y tiamina.


2. Administrar naloxona, flumazenilo y glucosa hi-
pertónica.
3. Administrar naloxona y flumazenilo.
4. Administrar tiamina y glucosa hipertónica.

- 13 -

FSE MEDICINA 2020/21


104. Un varón de 34 años sufre un accidente de tráfico y 108. En una luxación posterior de la cadera, señale la
es atendido en el lugar del accidente. El personal respuesta INCORRECTA:
sanitario objetiva que está pálido, sudoroso, tiene
pulso filiforme con una presión arterial de 90/50 1. El miembro inferior se encuentra en flexión, rota-
mmHg, una frecuencia cardiaca de 127 lpm, una ción externa y abducción.
frecuencia respiratoria de 28 rpm y una saturación 2. Hay que realizar una reducción de urgencia.
de oxígeno de 92 %. Se le infunden 20 ml/kg de cris- 3. Una de sus complicaciones posibles es la necrosis
taloides. Durante su traslado al hospital los signos de la cabeza femoral.
vitales mejoran transitoriamente y después se dete- 4. Una vez reducida, son infrecuentes las recidivas.
rioran a su llegada al hospital. Señale la respuesta
correcta: 109. Un niño sufre, en un accidente deportivo, una epifi-
siolisis distal de tibia. Al informar a los padres sobre
1. Requiere transfusión de sangre tipo-específica. el pronóstico, ¿cuál de los siguientes factores NO
2. Requiere transfusión de sangre de emergencia (O influye en el riesgo de deformidad futura?:
Rh negativo).
3. Debe prepararse una posible transfusión de sangre 1. Edad y género del niño.
con pruebas cruzadas. 2. Fractura intraarticular.
4. Debe probarse la infusión de nuevo de otros 20 3. Tipo de Salter y Harris.
ml/kg de cristaloides. 4. Actividad deportiva.

105. Una mujer de 24 años con antecedentes de migraña, 110. Paciente de 45 años que presenta un primer episodio
en tratamiento con propranolol y anticonceptivos de lumbalgia de 3 semanas de evolución que le impi-
orales, que acude por disnea intensa, ronquera, de realizar su vida normal. No refiere traumatismo
erupción cutánea, náuseas y vómitos 30 minutos ni otra patología subyacente. En la exploración física
después de la toma de metamizol. Presenta una pre- no hay déficit neurológico. ¿Qué prueba de imagen
sión arterial de 90/40 mmHg y una SatO2 de 90 %. estaría indicada?:
El tratamiento inicial más correcto sería adminis-
trar: 1. Al ser el primer episodio de lumbalgia, únicamente
una radiografía simple de columna lumbar.
1. Adrenalina. 2. Resonancia magnética, que aporta más información
2. Adrenalina y dexclorfeniramina. sobre partes blandas y posibles herniaciones.
3. Adrenalina, dexclorfeniramina y metil- 3. TC, para valorar mejor la estructura ósea y las po-
prednisolona. sibles fracturas.
4. Adrenalina, dexclorfeniramina y glucagón. 4. No hay indicación de prueba de imagen.

106. Una paciente de 65 años sufrió hace tres semanas 111. Un paciente de 40 años comienza a sentir molestias
una caída con fractura-luxación anterior de hombro en la nalga derecha tras un viaje prolongado en co-
que fue reducida en urgencias. Le fue colocado un che. Desde ese momento no tolera la sedestación
cabestrillo y se recomendó su retirada a las 3 sema- prolongada por reaparición del dolor. Ocasional-
nas. Al retirar la inmovilización se observa una zona mente siente entumecimiento en el miembro inferior
de disestesia circunscrita a la región lateral del hom- derecho que desaparece al levantarse. Dispone de
bro. La paciente puede realizar abducción, pero solo estudio de resonancia magnética y radiografía de
alcanza 15º. Dispone de un estudio de resonancia cadera en los que no se informan anomalías. El exa-
magnética en el que el manguito de los rotadores está men de la sensibilidad y los reflejos osteotendinosos
indemne. ¿Cuál es el diagnóstico de sospecha más no muestra alteraciones. Las maniobras de rotación
probable?: externa y la abducción resistidas, con la cadera en
90º de flexión, reproducen el dolor. El diagnóstico
1. Tendinopatía del redondo mayor. más probable es:
2. Capsulitis adhesiva.
3. Neuropatía del nervio supraescapular. 1. Síndrome de choque fémoro-acetabular.
4. Lesión del nervio axilar. 2. Síndrome del tensor de la fascia lata.
3. Radiculopatía S1.
107. En la patología de la mano, ¿cuál de las siguientes 4. Síndrome del piramidal.
afirmaciones sobre la enfermedad de Dupuytren es
INCORRECTA?:

1. Afecta con mayor frecuencia a los dedos anular y


meñique.
2. El tratamiento conservador con fisioterapia es poco
eficaz.
3. Es un engrosamiento y retracción de la aponeurosis
palmar.
4. Es más frecuente en trabajadores manuales.

- 14 -

FSE MEDICINA 2020/21


112. Sobre la fiebre mediterránea familiar es FALSO 117. ¿Cuál de los siguientes es un objetivo prioritario en
que: la estrategia terapéutica de la artritis reumatoide?:

1. La gran mayoría de los pacientes presentan fiebre 1. Evitar cambios frecuentes del tratamiento y evitar
alta de forma brusca. combinaciones de fármacos.
2. Los episodios inflamatorios son breves y recurren- 2. Evitar tratamientos innecesarios en pacientes que a
tes, con intervalos asintomáticos. pesar de tener síntomas, pueden mantener una acti-
3. Las manifestaciones digestivas son poco frecuen- vidad diaria aceptable.
tes. 3. El tratamiento debe ser temprano e intensivo.
4. Pueden presentar dolor torácico por pleuritis y/o 4. Debe aplicarse en todos los pacientes el mismo
pericarditis estéril. protocolo.

113. ¿Cuál de las siguientes manifestaciones extraarticu- 118. En la esclerodermia, con objeto de diagnosticar en
lares NO es característica de la artritis reumatoide?: fases tempranas la afección pulmonar, es recomen-
dable la realización de:
1. Pleuritis.
2. Nódulos subcutáneos. 1. Pruebas funcionales respiratorias y TC de alta reso-
3. Glomerulonefritis proliferativa. lución cada año.
4. Enfermedad pulmonar intersticial. 2. Ecocardiograma Doppler y pruebas funcionales
respiratorias, solo si el paciente presenta signos de
114. Sobre la fibromialgia, señale la respuesta correcta: insuficiencia respiratoria y/o cardíaca.
3. Pruebas funcionales respiratorias y ecocardiograma
1. Tiene una prevalencia en la población general de Doppler, ambas con periodicidad anual.
entre un 5 y un 10 %. 4. Pruebas funcionales respiratorias y prueba de la
2. La edad más frecuente de aparición es por encima marcha de caminar 6 minutos.
de los 50 años.
3. Existen múltiples fármacos aprobados para su tra- 119. Una paciente de 45 años con antecedentes de reflujo
tamiento. gastroesofágico presenta desde hace un año episodios
4. La fatiga es un síntoma muy común, por encima de palidez en algunos dedos de las manos con la ex-
del 70 % de los pacientes. posición al frío. Recientemente se le había prescrito
prednisona a dosis de 20 mg/día por dolor articular e
115. Sobre la gota, es FALSO que: induración cutánea en manos y brazos. Presenta
desde hace 48 horas malestar general y cefalea inten-
1. El tratamiento hipouricemiante debe iniciarse du- sa, por lo que acude a urgencias. En la exploración
rante el ataque de gota, tan pronto como sea posi- destaca únicamente una taquicardia rítmica a 100
ble. lpm, no se aprecia focalidad neurológica. La presión
2. El fármaco de elección es el alopurinol comenzan- arterial es de 200/110 mmHg. En la analítica destaca
do con dosis bajas. únicamente una creatinina de 2,5 mg/dl. Indique
3. Como norma general, la hiperuricemia asintomáti- cuál de los siguientes autoanticuerpos se relaciona
ca no debe ser tratada. mejor con el proceso descrito:
4. Alrededor del 50 % de los casos se manifiestan
como monoartritis inflamatoria aguda dela primera 1. Anticuerpos anti-RNA polimerasa III.
articulación metatarsofalángica. 2. Anticuerpos anti-centrómero.
3. Anticuerpos anti-proteinasa 3.
116. Entre los siguientes fármacos para el tratamiento de 4. Anticuerpos anti-PM-Scl.
la osteoporosis, ¿cuál de ellos NO causa osteonecrosis
mandibular o fractura atípica de fémur como efecto 120. Recién nacido de 2 semanas, cuya madre durante el
adverso?: embarazo presentó poliartralgias no filiadas y que
tras la primera exposición solar desarrolla lesiones
1. Zolendronato. cutáneas eritematosas policíclicas que asemejan las
2. Denosumab. del lupus eritematoso cutáneo subagudo. ¿Cuál de
3. Alendronato. los siguientes autoanticuerpos debe investigarse?:
4. Teriparatida.
1. Anticuerpos anti-ADN en el recién nacido.
2. Anticuerpos anti-citoplasma de neutrófilo (ANCA)
en la madre.
3. Anticuerpos anti-Ro/SS-A en la madre.
4. Anticuerpos anti-Sm en el recién nacido.

- 15 -

FSE MEDICINA 2020/21


121. Mujer de 27 años de raza negra que consulta por la 124. Un paciente de 45 años fue diagnosticado de infec-
aparición de edemas en miembros inferiores, dismi- ción VIH hace tres meses, presentando en ese mo-
nución de diuresis, fiebre y un rash malar en alas de mento CD4 45 linfocitos/µl y una carga viral de VIH-
mariposa de 20 días de evolución. Analíticamente 1 de 500.000 copias/ml. En ese momento presentaba
destaca creatinina 3 mg/dl, leucopenia, hemoglobina un Mantoux negativo. Comenzó tratamiento con un
10,5 g/dl (normocítica normocrómica), PCR 9 mg/dl inhibidor de integrasa y dos inhibidores de trans-
y VSG 60 mm. Respecto a la patología que presenta criptasa inversa, y al mes presentaba 25.000 copias y
esta paciente, señale la respuesta correcta: los CD4 habían subido a 80/µl. Consulta por presen-
tar un cuadro de adenopatías cervicales y fiebre de
1. Los anti-Sm no son específicos de esta patología. dos semanas de evolución. La punción de una de las
2. Los anticuerpos anticentrómero se detectan en el adenopatías muestra bacilos ácido-alcohol resisten-
90 % de los casos. tes aislados y granulomas epitelioides:
3. Los ANA (anticuerpos antinucleares) se detectan
en el 50 % de los casos. 1. Se trata de una tuberculosis ganglionar relacionada
4. Los anti-ADN bicatenario o nativo se correlacio- con una incompleta recuperación inmunológica.
nan con el riesgo de nefritis. 2. Presenta fracaso al tratamiento y se trata de una
infección oportunista.
122. Varón de 75 años, hipertenso en tratamiento con 3. Se trata de una infección oportunista desenmasca-
enalapril (20 mg/día), que consulta por deterioro del rada en el seno de un cuadro de reconstitución in-
estado general, cefalea bitemporal y claudicación mune.
mandibular de 3 semanas de evolución. Además, en 4. El hecho de presentar una prueba de Mantoux ne-
las últimas horas, refiere dos episodios de amaurosis gativa descarta la tuberculosis.
fugaz del ojo izquierdo. En la exploración física des-
taca engrosamiento y ausencia de pulso de la arteria 125. Con respecto a la meningitis bacteriana grave por
temporal izquierda. Análisis: PCR 6 mg/dl (VN <1); Streptococcus pneumoniae, ¿cuál de las siguientes es
hemoglobina 10,5 g/dl; VSG 92 mm. Radiografía de FALSA?:
tórax normal. Teniendo en cuenta el diagnóstico más
probable, marque la opción correcta en cuanto al 1. En nuestro medio más del 50 % de los S. pneumo-
tratamiento: niae causantes de meningitis son resistentes a peni-
cilina.
1. Debe administrarse rituximab, puesto que los glu- 2. La mortalidad de la meningitis por S. pneumoniae
cocorticoides han quedado relegados a un trata- que requiere ingreso en UCI supera el 30 %.
miento de segunda línea debido a sus efectos se- 3. La precocidad en el inicio del tratamiento es básica
cundarios. para el buen pronóstico.
2. El tocilizumab se ha mostrado eficaz en reducir las 4. El uso de dexametasona mejora el pronóstico fun-
recidivas y la dosis acumulada de pred-nisona en cional y vital.
más del 50 % de los pacientes.
3. En presencia de clínica isquémica, y para no em- 126. La monitorización de las concentraciones plasmáti-
peorar el riesgo vascular del paciente, se debe evi- cas de fármaco se realiza con frecuencia en la prácti-
tar el uso de glucocorticoides a dosis superiores a ca clínica para ajustar la pauta posológica de:
30 mg/día.
4. Junto con bolos de glucocorticoides de 1 g/día, el 1. Digoxina.
infliximab debe utilizarse como fármaco de prime- 2. Pioglitazona.
ra línea para la inducción a la remisión. 3. Acenocumarol.
4. Carbimazol.
123. Una mujer consulta por haber sufrido una agresión
sexual hace 3 horas. En cuanto a la profilaxis post- 127. La aparición de torsades de pointes, por alargamiento
exposición al VIH señale la respuesta INCORREC- del intervalo QT en el electrocardiograma, puede
TA: aparecer con más probabilidad en el curso de un
tratamiento antiarrítmico con fármacos del grupo:
1. La duración debe ser 28 días.
2. Debe hacerse una serología frente al VIH al inicio 1. Ia (por ejemplo, procainamida).
y al terminar la profilaxis. 2. Ib (por ejemplo, lidocaína).
3. La evidencia de la utilidad de este tratamiento no 3. II (por ejemplo, bisoprolol).
está basada en ensayos clínicos. 4. IV (por ejemplo, diltiazem).
4. El tratamiento es tenofovir y emtricitabina.

- 16 -

FSE MEDICINA 2020/21


128. Señale qué característica corresponde a la insufi- 132. En relación con los aneurismas de arteria poplítea:
ciencia cardiaca con función sistólica conservada:
1. Cuando se sospechan por palpación de un pulso
1. Es más frecuente en varones jóvenes. poplíteo amplio la ecografía es la prueba diagnósti-
2. Las arritmias auriculares son bien toleradas. ca de elección.
3. El pronóstico a 1 año es bueno, con una mortalidad 2. El riesgo de rotura aneurismática es alto.
menor del 5 %. 3. El edema por compresión de la vena poplítea es la
4. La hipertensión arterial es la patología más asocia- manifestación más frecuente.
da. 4. Son más frecuentes en mujeres que en varones en
una proporción 2:1.
129. Un varón de 40 años consulta por disnea de esfuerzo
lentamente progresiva desde hace un año. No tiene 133. Varón de 63 años, con antecedentes de hipertensión
hábitos tóxicos ni antecedentes de interés. No ha arterial, síndrome de Sjögren y revascularización
tenido dolor torácico. Presenta un soplo sistólico ilio-femoral izquierda (tromboendarterectomía).
rudo en foco aórtico, irradiado a ápex cardiaco y Presenta pérdida de peso de 10 kg y dolor abdomi-
carótidas. El ECG en ritmo sinusal muestra criterios nal grave tras la ingesta. La angioTC abdominopél-
de hipertrofia ventricular izquierda y en la Rx de vica muestra estenosis grave de tronco celíaco y de
tórax es evidente una raíz de aorta dilatada. Con arteria mesentérica superior. Señale la respuesta
estos datos, ¿qué diagnóstico es el más probable?: FALSA:

1. Válvula aórtica bicúspide estenótica. 1. La exploración física revela frecuentemente signos


2. Comunicación interventricular perimembranosa. de malnutrición y un soplo abdominal.
3. Insuficiencia aórtica degenerativa. 2. El tratamiento consiste en tromboembolectomía
4. Miocardiopatía restrictiva. con sonda de Fogarty.
3. La ecografía Doppler (duplex) es una exploración
130. Un varón de 66 años, diabético y fumador sufre un no invasiva útil para el diagnóstico.
infarto de miocardio anterior con elevación de seg- 4. La angioplastia con stent de los troncos viscerales
mento ST. La coronariografía urgente muestra una es una opción terapéutica con una tasa de éxito del
lesión única con oclusión aguda en la arteria descen- 80 %.
dente anterior proximal, implantándose un stent
recubierto. Al alta, su electrocardiograma está en 134. Mujer de 53 años de edad con antecedentes persona-
ritmo sinusal con bloqueo de rama izquierda (QRS les de obesidad y migraña. En repetidas ocasiones se
150 ms). La fracción de eyección es de 32 %. Dos le ha tomado la presión arterial en la consulta médi-
meses más tarde acude a revisión. A pesar de cum- ca y de enfermería presentando cifras inferiores a
plir un tratamiento óptimo, su clase funcional es III 140/90 mmHg. Sin embargo, se ha comprado un
de la NYHA y la fracción de eyección del 30 %. ¿Qué aparato homologado para la toma de la presión arte-
tratamiento debe ofrecerle en ese momento?: rial y se le ha enseñado a utilizarlo correctamente.
Acude mostrando registros de presión arterial toma-
1. Implantarle un desfibrilador automático y resincro- dos en su domicilio a lo largo de varias semanas con
nizador cardiaco. valores superiores a 140/90 mmHg. Señale la res-
2. Repetir la coronariografía por sospecha de trombo- puesta correcta:
sis del stent implantado.
3. Añadir digoxina y diuréticos de asa y esperar en 6 1. Presenta una hipertensión arterial secundaria.
meses la mejoría de la fracción de eyección. 2. Presenta una hipertensión arterial enmascarada.
4. Remitirle a una unidad de trasplante para ser in- 3. Presenta una hipertensión clínica aislada.
cluido en lista de espera. 4. Presenta una hipertensión arterial refractaria.

131. Una mujer de 28 años sin antecedentes cardiológicos 135. Paciente de 64 años, fumador de 1 paquete al día,
ingresa por síncope. No toma fármacos ni consume con antecedentes personales de diabetes mellitus tipo
tóxicos. No presenta alteraciones hidroelectrolíticas. 2 y dislipemia, que acude a consulta refiriendo dolor
Su electrocardiograma muestra un bloqueo aurículo- en la pantorrilla derecha cuando camina 200 metros
ventricular completo con un frecuencia ventricular a y que mejora con el reposo. ¿Qué prueba le realiza-
30 lpm que requiere el implante de un marcapasos ría en primer lugar?:
definitivo. Con estos datos, ¿cuál de las patologías
siguientes debe sospecharse?: 1. Índice tobillo brazo.
2. Eco Doppler de miembros inferiores.
1. Una valvulopatía reumática con estenosis mitral. 3. Radiografía de columna lumbar.
2. Una embolia de pulmón con infarto pulmonar. 4. TC abdominal.
3. Una enfermedad infiltrativa o inflamatoria sistémi-
ca.
4. Una trombosis de vena cava inferior.

- 17 -

FSE MEDICINA 2020/21


136. De entre las diferentes maniobras exploratorias del 140. Varón de 54 años con persistencia de cifras altas de
corazón, ¿cuál es la que presenta menor rentabilidad colesterol a pesar de recibir tratamiento con atorvas-
diagnóstica (sensibilidad y especificidad)?: tatina 40 mg (no toleró la dosis de 80 mg por mial-
gias) y haber asociado ezetimiba. No refiere antece-
1. Inspección del precordio. dentes de hipertensión arterial ni diabetes. Aporta
2. Palpación del área precordial. analítica reciente con los siguientes resultados: coles-
3. Percusión cardiaca. terol total 302 mg/dl, LDL-colesterol 270 mg/dl,
4. Auscultación cardiaca. HDL-colesterol 32 mg/dl y triglicéridos normales.
Destaca en la exploración física un IMC de 29 y un
137. Todas las siguientes terapias han demostrado au- engrosamiento a nivel de los tendones de Aquiles.
mentar la supervivencia en el tratamiento de la insu- Con respecto a la sospecha diagnóstica y tratamien-
ficiencia cardiaca con fracción de eyección del ven- to, señale la afirmación INCORRECTA:
trículo izquierdo deprimida EXCEPTO una. Señale
cual: 1. El diagnóstico puede realizarse mediante un análi-
sis genético.
1. IECA o ARA 2. 2. En caso de confirmarse la entidad en el paciente,
2. Digoxina. debemos realizar estudio genético a todos los fami-
3. DAI (desfibrilador automático implantable) o re- liares de primer grado.
sincronizador. 3. Podría beneficiarse de iniciar tratamiento con un
4. Betabloqueantes. fármaco inhibidor de PCSK9.
4. Es fundamental el control de todos los factores de
138. ¿Cuál de las siguientes afirmaciones con respecto al riesgo cardiovascular debido a la asociación de esta
infarto de ventrículo derecho es FALSA?: patología con eventos cardiovasculares precoces.

1. El infarto del ventrículo derecho puede producir 141. En el síndrome de distrés respiratorio del adulto es
signo de Kussmaul. cierto que:
2. La elevación del segmento ST en V4 es la presen-
tación más frecuente en el electrocardiograma. 1. El edema de pulmón se debe a una elevación de la
3. El tratamiento incluye la expansión de volumen presión capilar pulmonar.
para conservar la precarga del ventrículo derecho. 2. El edema de pulmón se debe a una lesión de la
4. Con la ecocardiografía podemos observar hipoci- membrana alveolocapilar.
nesia y dilatación del ventrículo derecho. 3. La lesión pulmonar se debe a una presión pleural
muy negativa.
139. Respecto a la tromboangitis obliterante (enfermedad 4. La lesión pulmonar se debe fundamentalmente a
de Buerger) señale la respuesta correcta: una broncoaspiración.

1. Predomina en el sexo femenino. 142. En un paciente con carcinoma microcítico de pulmón


2. La presencia de reactantes de fase aguda determina la radioterapia torácica:
el pronóstico.
3. La evolución de las lesiones depende fundamen- 1. Está contraindicada en quimio-resistencia.
talmente de la supresión del hábito tabáquico. 2. Solo se emplea en el síndrome de vena cava supe-
4. Se caracteriza por ser una trombosis inflamatoria rior.
segmentaria, ateromatosa, que afecta a arterias y 3. Es más eficaz después de la quimioterapia.
venas de mediano y pequeño calibre. 4. Se recomienda de forma simultánea con los dos
primeros ciclos de quimioterapia.

143. Al mes de tratamiento estándar por una tuberculosis


pulmonar, un paciente presenta hiperuricemia en
una analítica de control, con un valor de 12 mg/dl
(límite alto de la normalidad 7 mg/dl). Está asinto-
mático. ¿Cuál sería su actitud?:

1. Retirar toda la medicación.


2. Continuar con el mismo tratamiento.
3. Retirar la isoniacida.
4. Retirar la pirazinamida.

- 18 -

FSE MEDICINA 2020/21


144. Mujer de 84 años, diagnosticada de enfermedad 147. El cáncer de páncreas localmente avanzado:
pulmonar obstructiva crónica en tratamiento con
oxigenoterapia domiciliaria. Acude a urgencias por 1. Es una presentación clínica infrecuente (menor del
empeoramiento de su estado general, fiebre de 39ºC, 10 %).
tos y expectoración purulenta de 5 días de evolución. 2. Tiene una supervivencia mediana inferior a 3 me-
En la exploración física se halla una tensión arterial ses.
de 90/70 mmHg, frecuencia cardiaca de 125 lpm, 3. Es candidato a quimio-radioterapia neoadyuvante.
saturación basal de oxígeno 86 % y frecuencia respi- 4. Es mínimamente sintomático.
ratoria 35 rpm. En la auscultación hay hipofonesis
basal izquierda. La analítica muestra 30.000 leucoci- 148. Mujer de 72 años sin antecedentes de interés. Tras
tos (85 % neutrófilos). En la radiografía de tórax se un cólico renal complicado se encuentra casualmente
detecta un gran infiltrado pulmonar apical izquierdo en la TC abdominal una lesión quística en cola de
y un derrame pleural moderado ipsilateral. Se reali- páncreas de 2 cm, junto con múltiples lesiones quísti-
za una toracocentesis que obtiene un líquido pleural cas renales bilaterales. La ultrasonografía endoscó-
purulento, del que se solicita cultivo. ¿Cuál es el pica muestra una lesión poliquística formada por
diagnóstico de sospecha y la actitud terapéutica ini- múltiples vesículas con calcificación central en cola
cial?: de páncreas sin conexión con el conducto de Wir-
sung. El análisis del líquido es compatible con un
1. Neumonía adquirida en la comunidad. Se debe cistoadenoma seroso. De las siguientes, ¿cuál es la
solicitar antigenuria de neumococo y pautar cober- actitud más correcta con respecto al manejo de esta
tura antibiótica de amplio espectro. paciente?:
2. Derrame pleural a estudio. Se debe realizar una
cobertura antibiótica y controles radiológicos se- 1. Resección quirúrgica (pancreatectomía corporo-
riados. caudal).
3. Neumonía adquirida en la comunidad y derrame 2. Punción guiada mediante ultrasonografía endoscó-
paraneumónico izquierdo. Se debe realizar cobertu- pica y etanolización de la misma.
ra antibiótica y esperar a los resultados del cultivo 3. Seguimiento de la lesión mediante RM.
del líquido pleural para valorar la colocación de un 4. Trasplante de riñón-páncreas.
drenaje pleural.
4. Neumonía adquirida en la comunidad y empiema 149. El diagnóstico del síndrome de intestino irritable es
pleural paraneumónico izquierdo. Se debe realizar principalmente clínico y se basa en síntomas compa-
cobertura antibiótica, solicitar antigenuria de neu- tibles y la ausencia de signos o síntomas de alarma.
mococo y colocar un drenaje pleural izquierdo ur- ¿Cuál de los siguientes NO se considera un signo o
gente. síntoma de alarma?:

145. En un paciente asmático con bronquiectasias centra- 1. Edad de aparición de los síntomas por encima de
les y sospecha de aspergilosis broncopulmonar alér- los 50 años.
gica, ¿qué prueba complementaria apoyaría el diag- 2. Pérdida de peso.
nóstico de esta entidad?: 3. Sangre oculta en heces positiva.
4. Urgencia defecatoria.
1. Test del sudor.
2. IgE específica frente a Aspergillus fumigatus. 150. Varón de 83 años que consulta por un cuadro recu-
3. IgE específica frente a Aspergillus niger. rrente de dolor abdominal en fosa iliaca izquierda,
4. Broncoscopia. acompañado de diarrea de hasta 6 deposiciones al
día sin productos patológicos, de 10-15 días de evolu-
146. Paciente de 59 años procedente del África subsaha- ción. No refiere fiebre ni afectación del estado gene-
riana, con episodios frecuentes de disminución de ral. En una analítica reciente no se observa leucoci-
conciencia por enolismo crónico. Consulta por suda- tosis y presenta una sangre oculta en heces negativa.
ción nocturna, tos y fiebre de varias semanas de En la exploración se observa leve dolor a la palpa-
duración. En la radiografía y en la TC de tórax se ción de fosa iliaca izquierda sin irritación peritoneal.
observa una lesión cavitada de 3,5 cm de diámetro Tiene realizada una colonoscopia hace tres años en la
con un nivel hidroaéreo en lóbulo inferior derecho. que se informa de divertículos a lo largo de todo el
¿Cuál sería la respuesta INCORRECTA?: colon, más numerosos en colon izquierdo, sin otras
lesiones asociadas. De las siguientes, ¿cuál sería la
1. Es preciso analizar muestras de esputo inducido mejor actitud a seguir?:
para descartar tuberculosis.
2. Se debe iniciar un tratamiento empírico con metro- 1. Realizar un tratamiento empírico con antibióticos.
nidazol. 2. Solicitar una colonoscopia preferente.
3. Si en el esputo no se aísla un patógeno específico 3. Solicitar una ecografía abdominal urgente.
se presupone la presencia de gérmenes anaerobios. 4. Derivar a cirugía para valorar sigmoidectomía.
4. Si sospecha compromiso inmunitario debe realizar
de forma inmediata broncoscopia con biopsia o as-
piración con aguja guiada por TC.

- 19 -

FSE MEDICINA 2020/21


151. Varón de 43 años sin antecedentes de interés que 154. Hombre de 82 años que acude a urgencias por cua-
consulta por un cuadro de diez días de evolución de dro de estreñimiento junto con gran distensión ab-
ictericia de piel y mucosas, coluria y acolia. No refie- dominal. En la radiología simple de abdomen se
re dolor abdominal, pérdida de peso ni otra clínica, observa el signo de grano de café y en el enema opa-
salvo prurito generalizado. Niega consumo de al- co el signo de pico de pájaro. El tratamiento, si no
cohol. No toma medicación salvo ibuprofeno ocasio- existen signos de gangrena o peritonitis, consistirá
nal por dolores musculares tras actividad deportiva. en primer lugar en:
La analítica muestra una bilirrubina aumentada a
expensas de directa. Una ecografía urgente muestra 1. Detorsión endoscópica.
un hígado normal, colesterolosis vesicular y ausencia 2. Colectomía segmentaria y Hartmann.
de dilatación de vía biliar, sin otras alteraciones de 3. Colectomía segmentaria y anastomosis colorrectal
interés. ¿Cuál es el diagnóstico más probable?: laparoscópica.
4. Detorsión quirúrgica y pexia de sigmoides.
1. Colangitis aguda por barro biliar.
2. Hepatitis tóxica por ibuprofeno. 155. Los tumores benignos más frecuentes del esófago
3. Colangiocarcinoma. son:
4. Síndrome de Gilbert.
1. Los pólipos fibrovasculares.
152. Mujer de 79 años hipertensa en tratamiento con 2. Los leiomiomas.
olmesartán que acude a consulta por diarrea acuosa 3. Los papilomas escamosos.
de 4 a 6 deposiciones al día desde hace dos meses. 4. Los hemangiomas.
Hace tres meses recibió tratamiento con antiinflama-
torios no esteroideos durante 3 semanas por lumbo- 156. Paciente de 46 años intervenido hace un año de gas-
ciática. Se realizó gastroscopia y colonoscopia, am- trectomía Bilroth II por úlcera péptica. Presenta un
bas sin alteraciones macroscópicas. Las biopsias cuadro de dolor epigástrico y hemorragia digestiva
duodenales fueron normales, mientras que en las alta. En la endoscopia se detectan úlceras tanto en
biopsias de colon se observa un infiltrado inflamato- estómago como en intestino delgado. Tras realizar
rio crónico de la lámina propia con una banda irre- estimulación con secretina IV no se aprecia aumento
gular de colágeno inmediatamente por debajo del significativo de gastrina sérica. El cuadro correspon-
epitelio de superficie de la mucosa con un grosor de muy probablemente a:
mayor a 10 mm y un número de linfocitos intraepite-
liales >20 por cada 100 células epiteliales. ¿Cuál es el 1. Gastrinoma.
diagnóstico más probable?: 2. Síndrome de antro retenido.
3. Síndrome de asa aferente.
1. Colitis microscópica. 4. Carcinoma de muñón gástrico.
2. Enteropatía por AINEs.
3. Enteropatía asociada a olmesartán. 157. La escisión mesorrectal total junto con resección
4. Síndrome de intestino irritable. anterior de recto en el tratamiento del cáncer de
recto medio-bajo pretende:
153. Paciente diagnosticado de diverticulitis aguda, trata-
do con amoxicilina/clavulánico. Al cabo de 5 días 1. Realizar una anastomosis con mayor facilidad.
comienza con fiebre y diarrea. Se le diagnostica de 2. Realizar una anastomosis más baja.
colitis por Clostridium difficile, se suspende la amoxi- 3. Disminuir el número de recurrencias locales.
cilina/clavulánico y se pauta metronidazol. A los 4 4. Disminuir el intervalo libre de enfermedad.
días no ha respondido al metronidazol, pero se en-
cuentra clínicamente estable. El siguiente paso del 158. Mujer de 52 años histerectomizada años atrás por
tratamiento es: mioma uterino, que consulta por presentar estreñi-
miento distal y abultamiento en cara posterior vagi-
1. Cambiar el metronidazol por vancomicina oral nal que se incrementa durante las maniobras defeca-
(125 mg, cuatro veces al día durante 10 días). torias, debiendo introducir un dedo en la vagina
2. Cambiar el metronidazol por piperacili- para conseguir la defecación. En la exploración se
na/tazobactam (4 g de piperacilina/0,5 g de tazo- observa rectocele grado III. En la resonancia magné-
bactam cada 8 horas). tica se observa descenso de la cúpula vaginal junto
3. Indicar una colectomía subtotal. con el rectocele. El tratamiento quirúrgico más ade-
4. Realizar una ileostomía en asa con lavado anteró- cuado es:
grado de polietilenglicol.
1. Colporrectosacropexia con malla.
2. Intervención de reforzamiento del tabique rectova-
ginal por vía transanal (técnica de Khubchandani).
3. Intervención transperineal con colocación de malla
en el tabique recto-vaginal.
4. Esfinteroplastia anterior.

- 20 -

FSE MEDICINA 2020/21


159. El tratamiento quirúrgico de elección de la compre- 164. En un paciente con diabetes mellitus mal controlada
sión vascular del duodeno, conocida también como e insuficiencia renal crónica (TFGe 52 ml/min) está
síndrome de la arteria mesentérica superior o sín- recomendado añadir por su beneficio cardiovascu-
drome de Wilkie, es: lar, tras metformina:

1. La división del ligamento de Treitz. 1. Sulfonilurea.


2. La duodenoyeyunostomía. 2. Insulina basal.
3. El cambio de la posición del duodeno. 3. Glitazona.
4. El bypass de la arteria mesentérica superior. 4. Inhibidor de SGLT2.

160. La diabetes insípida central se caracteriza por: 165. Mujer de 66 años diagnosticada de diabetes mellitus
tipo 2 desde hace tres meses. Tiene un IMC de 31
1. Adipsia y orinas hipotónicas. Kg/m2 y presenta mal control glucémico a pesar de
2. Hipotonicidad plásmatica y urinaria, polidipsia y realizar un programa de medidas no farmacológicas
poliuria con nicturia. (alimentación saludable, ejercicio). ¿Cuál de los si-
3. Respuesta inapropiada a la administración de vaso- guientes fármacos hipoglucemiantes se asocia a au-
presina. mento de peso y deberíamos evitar en esta paciente?:
4. Natremia en valores altos de la normalidad con
hipotonicidad urinaria. 1. Metformina (biguanida).
2. Pioglitazona (tiazolidindiona).
161. La insuficiencia suprarrenal central debida a la su- 3. Canagliflozina (inhibidor del cotransportador 2 de
presión del eje hipotálamo-hipófiso-adrenal por la sodio-glucosa- iSGLT2).
administración de corticoides: 4. Liraglutida (agonista del receptor para GLP-1).

1. Es la causa más frecuente de insuficiencia supra- 166. Paciente de 18 años que consulta por edemas. Se
rrenal. realiza analítica completa que muestra proteinuria
2. La dosis y la duración de la corticoterapia no se de 8 g/día sin microhematuria, hipoalbuminemia e
asocian directamente con el riesgo de insuficiencia hipercolesterolemia con función renal normal. Se le
adrenal tras su suspensión. administran de forma empírica corticoides. Al cabo
3. La administración de corticoides suprime la secre- de un mes, el cuadro clínico ha desaparecido comple-
ción de ACTH pero no afecta la secreción de CRH. tamente. ¿Cuál es su hipótesis diagnóstica?:
4. La resolución de la insuficiencia suprarrenal tras
corticoterapia crónica se produce 24-48 horas tras 1. Amiloidosis.
la retirada del fármaco. 2. Nefropatía IgA o enfermedad de Berger.
3. Síndrome de Alport.
162. Una vez confirmado un síndrome de Cushing por 4. Nefropatía con cambios mínimos.
criterios clínicos y bioquímicos, ¿cuál es la prueba
fundamental para averiguar su causa?: 167. Ante un paciente con enfermedad renal crónica
avanzada (ERC G4, FGe 20 ml/min) conocida, que
1. Determinación de ACTH plasmática matutina. acude a urgencias por debilidad general y se le detec-
2. Prueba de supresión de cortisol con dosis altas de ta una hiperpotasemia grave (K 7 mEq/l) con altera-
dexametasona. ciones electrocardiográficas. ¿Cuál sería la primera
3. Muestreo de seno petroso inferior y cateterismo medida a tomar?:
venoso selectivo.
4. Resonancia magnética de la región hipotálamo- 1. Administración de resinas de intercambio catióni-
hipofisaria. co.
2. Colocación de un catéter para iniciar diálisis.
163. La apoplejía hipofisaria es habitualmente una ur- 3. Administración de gluconato cálcico endovenoso.
gencia endocrinológica. Señale la respuesta correcta: 4. Administración de furosemida oral.

1. Cursa con sintomatología compresiva, hipopituita- 168. ¿Cuál de las siguientes afirmaciones NO es propia de
rismo e hiperprolactinemia por sección del tallo hi- una nefritis intersticial aguda?:
pofisario.
2. La oftalmoplejía por afectación de pares craneales 1. Frecuentemente está causada por fármacos.
remite espontáneamente. 2. La proteinuria suele ser de rango nefrótico.
3. En pacientes sin alteración del campo visual es 3. Es frecuente la leucocituria estéril.
preciso realizar de forma urgente cirugía transesfe- 4. El diagnóstico definitivo se establece por biopsia
noidal descompresiva. renal.
4. El déficit hormonal más frecuente es el de hormona
antidiurética.

- 21 -

FSE MEDICINA 2020/21


169. ¿Cuál de los siguientes iones urinarios favorece la 174. Una de las siguientes consecuencias NO se puede
formación de litiasis urinaria?: atribuir directamente al síndrome de inmovilidad en
el paciente mayor:
1. Citrato.
2. Magnesio. 1. Insuficiencia cardíaca.
3. Sodio. 2. Hipotensión ortostática.
4. Sulfato. 3. Impactación fecal.
4. Neumonía aspirativa.
170. En una de las siguientes condiciones, la bacteriuria
asintomática NO requeriría ser tratada: 175. Un varón de 75 años, en buen estado de salud y que
podría ser considerado como robusto, consulta para
1. Niños menores de 5 años con reflujo vesicourete- solicitar información sobre la utilización de algún
ral. tipo de tratamiento para mantenerse con buen esta-
2. Pacientes mayores dependientes institucionaliza- do de salud y físico. ¿Cuál sería la mejor recomenda-
dos. ción para este paciente?:
3. Pacientes neutropénicos.
4. Pacientes a los que se les ha de realizar una explo- 1. Hormona de crecimiento.
ración endoscópica o quirúrgica de la vía urinaria. 2. Testosterona.
3. Vitamina D.
171. La causa más frecuente de neutropenia grave (<500 4. Ejercicio físico regular.
neutrofilos/microlitro) es:
176. Una mujer de 25 años acude a urgencias acompaña-
1. Secundaria a enfermedades autoinmunes. da de una amiga relatando que ha sido víctima de
2. Primaria por leucemia aguda. una agresión sexual hace 4 horas. La paciente se
3. Iatrogénica por administración de medicamentos. encuentra muy afectada, contestando a sus pregun-
4. Debida al hiperesplenismo desarrollado en las in- tas de forma incongruente, con lentitud y sin ser
fecciones. capaz de recordar cómo se ha producido la agresión.
Sus constantes vitales (tensión arterial, frecuencia
172. Una mujer de 19 años, con antecedentes familiares cardiaca y temperatura) son normales. ¿Cuál de las
de anemia, presenta datos de anemia hemolítica siguientes actuaciones sería INCORRECTA?:
crónica, Coombs directo negativo. ¿Cuál sería el
diagnóstico más probable?: 1. Contactar con el juzgado de guardia.
2. Realizar de inmediato una exploración ginecológi-
1. Esferocitosis hereditaria. ca con toma de citología cervical.
2. Deficit de piruvatoquinasa eritrocitaria. 3. Administrar 500 mg de ceftriaxona intramuscular,
3. Betatalasemia major. 1 g de azitromicina vía oral y 2 g de tinidazol vía
4. Hemoglobinuria paroxística nocturna. oral en dosis únicas.
4. Administrar 1500 mg de levonorgestrel vía oral en
173. Mujer de 70 años que acude a urgencias por astenia. dosis única.
En el examen físico destaca palidez cutáneomucosa
con subictericia conjuntival y un soplo pansistólico 177. En la TC de abdomen con contraste intravenoso
audible en todos los focos. Analíticamente destaca realizada para estudio de pancreatitis crónica, ¿cuál
hemoglobina 8 g/dl, VCM 100 fl, LDH 900 U/l, hap- de los siguientes hallazgos es típico de esta enferme-
toglobina indetectable y bilirrubina total 3.5 mg/dl. dad?:
Se solicitó la determinación de una prueba inmuno-
hematológica. Señale la respuesta correcta: 1. Ensanchamiento del marco duodenal.
2. Áreas focales de necrosis pancreática.
1. El diagnóstico más probable es el de saturnismo; 3. Aumento en el realce glandular en fase arterial.
debemos solicitar plomo en orina de 24 horas. 4. Dilatación de conducto pancreático.
2. Se trata de una anemia hemolítica; para determinar
su naturaleza debemos solicitar una electroforesis 178. En relación con la encefalitis por anticuerpos anti-
de hemoglobinas. rreceptor de N-metil-D-Aspartato (NMDAR) seña-
3. Se trata de una anemia hemolítica; debemos solici- le la respuesta INCORRECTA:
tar un test de Coombs directo, que determina la
presencia de IgG y/o la fracción C3 del comple- 1. En adolescentes predominan las manifestaciones
mento sobre la membrana del hematíe. psiquiátricas.
4. Se trata de una anemia hemolítica; debemos solici- 2. En algunos casos puede ocurrir tras una infección
tar un test de Coombs directo, que determina la por virus herpes simple 1.
presencia de IgG libre en plasma. 3. No afecta a niños menores de 1 año de edad.
4. El diagnóstico se establece al demostrar la presen-
cia de anticuerpos anti-NMDAR en el LCR y el
suero.

- 22 -

FSE MEDICINA 2020/21


179. Mujer de 32 años con síndrome de neoplasia endo- 182. Una recomendación de clase I en una guía de prácti-
crina múltiple de tipo 2A (MEN-2A) y portadora de ca clínica significa que:
una mutación en RET. En una ecografía de cuello se
identifica un nódulo hipoecogénico de 6 mm con 1. Está basada en evidencias científicas procedentes
calcificaciones en su interior. Se decide tiroidectomía de ensayos clínicos.
total y vaciamiento ganglionar cervical. En el estudio 2. Hay evidencia o acuerdo de que seguir la recomen-
macroscópico se identifican un total de tres nódulos, dación tiene más beneficios que perjuicios para los
dos en el lóbulo derecho de 5 y 6 mm, y uno en el pacientes.
lóbulo izquierdo de 4 mm. En el estudio microscópi- 3. Todos los pacientes en que se siga la recomenda-
co las tres lesiones están constituidas por una proli- ción van a mejorar su nivel de salud.
feración uniforme de células redondeadas que se 4. Que la intervención recomendada tiene alto coste.
disponen con un patrón sólido y se acompañan de
calcificaciones y de depósitos de amiloide. Los nú- 183. Varón de 37 años presenta astenia, febrícula (37,6ºC)
cleos no son claros, ni muestran hendiduras, ni pseu- y pérdida de peso de unos 5 kg de 2 meses de evolu-
doinclusiones. La tinción inmunohistoquímica para ción. En las 2 últimas semanas se añaden parestesias
sinaptofisina es positiva. En el vaciamiento ganglio- urentes e hipoestesia en las cuatro extremidades, así
nar cervical se identifican metástasis. ¿Cuál es el como lesiones purpúricas palpables no dolorosas en
diagnóstico anatomopatológico de las lesiones identi- ambas piernas y pies. La presión arterial, la función
ficadas en la tiroidectomía total?: renal y el sedimento de orina son normales. ¿Cuál
sería la prueba complementaria para confirmar el
1. Carcinoma medular multifocal. diagnóstico más probable?:
2. Carcinoma papilar multifocal.
3. Carcinoma folicular. 1. Una ecografía-Doppler de las arterias renales para
4. Hiperplasia de células parafoliculares. el diagnóstico de una arteritis de Takayasu.
2. Un biopsia cutánea tipo "punch" (sacabocados)
180. Paciente de 25 años de edad, que sufre un accidente para el diagnóstico de una poliarteritis nudosa.
de moto un viernes por la noche. Es trasladado a 3. La detección de crioglobulinas circulantes para el
urgencias y diagnosticado de traumatismo abdomi- diagnóstico de una vasculitis crioglobulinémica.
nal (eco-fast negativa), un traumatismo craneal leve 4. Una biopsia renal para el diagnóstico de una vascu-
(Glasgow = 14) y una fractura intracapsular despla- litis por IgA.
zada de cadera derecha. Hemodinámicamente está
estable. ¿Cuál sería el tratamiento de elección?: 184. A un paciente ingresado por una micosis sistémica
oportunista se le prescribe un fármaco antifúngico
1. Reducción, abierta si se precisa, y osteosíntesis de de índice terapéutico estrecho. La forma activa del
la fractura en las primeras 24-36 horas. fármaco se excreta sin metabolizar por orina en un
2. Artroplastia total de superficialización de la cadera 60 %. El paciente sufre una insuficiencia renal gra-
el lunes de forma programada. ve, con un aclaramiento de creatinina de 20 ml/min.
3. Esperar a que mejore del traumatismo craneal y Si la pauta de dosificación habitual del antifúngico es
programar la semana siguiente una cirugía reglada de 40 mg/kg/10 h, ¿cuál de las siguientes pautas es
consistente en la reducción y osteosíntesis de la más adecuada?:
fractura.
4. Ante el riego de no consolidación de estos tipos de 1. 40 mg/kg/10 h (no se requiere ajuste de dosifica-
fractura le implantaría de urgencias una hemiartro- ción).
plastia bipolar de cadera. 2. 30 mg/kg/10 h (solo se reduce la dosis a 30
mg/kg).
181. Una paciente de 33 años, no fumadora, con antece- 3. 30 mg/kg/20 h (se reduce la dosis a 30 mg/kg y se
dentes de neumotórax en dos ocasiones, acude a la alarga el intervalo interdosis a 20 h).
consulta por disnea progresiva. La Rx de tórax 4. 40 mg/kg/20 h (solo se alarga el intervalo interdo-
muestra un derrame pleural izquierdo que ocupa 1/3 sis a 20 h).
del hemitórax y múltiples imágenes quísticas distri-
buidas de forma regular por ambos pulmones. En la 185. En un paciente de quién se conoce que tenía hace
espirometría presenta un FEV1/FVC del 64 % y un unos años un Mantoux negativo se produce un viraje
test de difusión con una TLCO del 50 %. Señale el del mismo. La primera reacción:
diagnóstico más probable:
1. Es un verdadero negativo.
1. EPOC. 2. Es un falso negativo.
2. Histiocitosis X. 3. Es un verdadero positivo.
3. Malformación adenoide quística. 4. Es un falso positivo.
4. Linfangioleiomiomatosis.

- 23 -

FSE MEDICINA 2020/21

También podría gustarte